aieee-chapterwise-full-book.pdf



Comments



Description

1INSTANT 2 Copywright © 2014 MTG Learning Media (P) Ltd. No part of this publication may be reproduced, transmitted, in any form or by any means, electronic, mechanical, photocopying, recording or otherwise, without the prior permission of the Publisher. Ownership of an ebook does not give the possessor the ebook copywright. All disputes subject to Delhi jurisdiction only. Disclaimer : The information in this book is to give yoU the path to success but it does not guarantee 100% success as the strategy is completely dependent on its execution. And it is based on last 5 years of patterns in AIEEE exam and the pattern may be changed by the authorities. Published by : MTG Learning Media (P) Ltd. Head Office : Plot 99, Sector 44 Institutional Area, Gurgaon, Haryana. Phone : 0124 - 4951200 Regd. Office : 406, Taj Apt., Ring Road, Near Safdarjung Hospital, New Delhi-110029 Web: mtg.in Email: [email protected] Visit www.mtg.in for buying books online. Other recommended books: JEE Main Mathematics (30 days) JEE Main Chemistry JEE Advanced JEE Main Physics (30 days) JEE Main Mathematics JEE Advanced Physics JEE Main Chemistry (30 days) JEE Main Explorer JEE Main Physics 12 + 12 JEE (Main + Advanced) JEE Advanced Chemistry JEE Advanced Mathematics PHYSICS  1.  Physics  and  Measurement ........................................................................................................................................................ 1  2.  Kinematics .................................................................................................................................................................................... 3  3.  Laws  of  Motion ............................................................................................................................................................................ 9  4.  Work,  Energy  and  Power ........................................................................................................................................................ 14  5.  Rotational  Motion ...................................................................................................................................................................... 20  6.  Gravitation .................................................................................................................................................................................. 26  7.  Properties  of  Solids  and  Liquids ............................................................................................................................................ 30  8.  Thermodynamics ....................................................................................................................................................................... 37  9.  Kinetic  Theory  of  Gases .......................................................................................................................................................... 42  10.  Oscillations  and  Waves ........................................................................................................................................................... 44  11.  Electrostatics .............................................................................................................................................................................. 54  12.  Current  Electricity...................................................................................................................................................................... 64  13.  Magnetic  Effects  of  Current  and  Magnetism ........................................................................................................................ 72  14.  Electromagnetic  Induction  and  Alternating  Currents ........................................................................................................... 79  15.  Electromagnetic  W aves ........................................................................................................................................................... 86  16.  Optics .......................................................................................................................................................................................... 88  17.  Dual  Nature  of  Matter  and  Radiation .................................................................................................................................... 94  18.  Atoms  and  Nuclei ..................................................................................................................................................................... 99  19.  Electronic  Devices .................................................................................................................................................................. 106  20.  Experimental  Skills................................................................................................................................................................... 111  CHEMISTRY  1.  Some  Basic  Concepts  in  Chemistry ........................................................................................................................................ 1  2.  States  of  Matter ........................................................................................................................................................................... 3  3.  Atomic  Structure .......................................................................................................................................................................... 7  4.  Chemical  Bonding  and  Molecular  Structure ..........................................................................................................................11  5.  Chemical  Thermodynamics ..................................................................................................................................................... 16  6.  Solutions ..................................................................................................................................................................................... 22  7.  Equilibrium .................................................................................................................................................................................. 28  8.  Redox  Reactions  and  Electrochemistry ................................................................................................................................ 35  9.  Chemical  Kinetics ..................................................................................................................................................................... 41  10.  Surface  Chemistry .................................................................................................................................................................... 45  11.  Nuclear  Chemistry .................................................................................................................................................................... 47  12.  Classification  of  Elements  and  Periodicity  in  Properties ................................................................................................... 49 iv  13.  General  Principles  and  Processes  of  Isolation  of  Metals .................................................................................................. 53  14.  Hydrogen .................................................................................................................................................................................... 55  15.  s­Block  Elements ...................................................................................................................................................................... 57  16.  p­Block  Elements ...................................................................................................................................................................... 59  17.  d­  and  f­Block  Elements .......................................................................................................................................................... 65  18.  Coordination  Compounds ........................................................................................................................................................ 70  19.  Environmental  Chemistry ......................................................................................................................................................... 74  20.  Purification  and  Characterisation  of  Organic  Compounds ................................................................................................. 76  21.  Some  Basic  Principles  of  Organic  Chemistry ...................................................................................................................... 78  22.  Hydrocarbons ............................................................................................................................................................................. 82  23.  Organic  Compounds  Containing  Halogens ........................................................................................................................... 87  24.  Alcohols,  Phenols  and  Ethers ................................................................................................................................................ 91  25.  Aldehydes,  Ketones  and  Carboxylic  Acids ........................................................................................................................... 95  26.  Organic  Compounds  Containing  Nitrogen ............................................................................................................................ 99  27.  Polymers ................................................................................................................................................................................... 101  28.  Biomolecules ............................................................................................................................................................................ 103  29.  Chemistry  in  Everyday  Life ................................................................................................................................................... 107  30.  Principles  Related  to  Practical  Chemistry .......................................................................................................................... 109  MATHEMATICS  1.  Sets,  Relations  and  Functions .................................................................................................................................................. 1  2.  Complex  Numbers ...................................................................................................................................................................... 7  3.  Matrices  and  Determinants ..................................................................................................................................................... 12  4.  Quadratic  Equations ................................................................................................................................................................. 19  5.  Permutations  and  Combinations ............................................................................................................................................. 24  6.  Mathematical  Induction  and  its  Application .......................................................................................................................... 28  7.  Binomial  Theorem ..................................................................................................................................................................... 30  8.  Sequences  and  Series ............................................................................................................................................................. 34  9.  Differential  Calculus .................................................................................................................................................................. 40  10.  Integral  Calculus ....................................................................................................................................................................... 52  11.  Differential  Equations ............................................................................................................................................................... 63  12.  Two  Dimensional  Geometry .................................................................................................................................................... 67  13.  Three  Dimensional  Geometry ................................................................................................................................................. 82  14.  Vector  Algebra ........................................................................................................................................................................... 90  15.  Statistics ..................................................................................................................................................................................... 96  16.  Probability ................................................................................................................................................................................. 100  17.  Trigonometry ............................................................................................................................................................................ 105  18.  Mathematical  Logic .................................................................................................................................................................. 111 v  « S  Y  L  L  A  B  U  S  PHYSICS  SECTION A  Unit ­ 1: Physics and Measurement  Physics, technology and society, S. I. units, fundamental and derived units, least count, accuracy and precision of measuring  instruments,  errors  in  measurement.  Dimensions  of  physical  quantities,  dimensional  analysis  and  its  applications.  Unit ­ 2: Kinematics  Frame  of reference,  motion in  a straight line: position­time  graph,  speed and velocity, uniform  and non­uniform  motion,  average  speed  and  instantaneous  velocity.  Uniformly  accelerated  motion,  velocity­time,  position­time  graphs,  relations  for  uniformly  accelerated  motion.  Scalars  and vectors,  vector  addition  and subtraction,  zero  vector,  scalar  and  vector  products,  unit  vector, resolution  of  a  vector,  relative  velocity,  motion  in  a  plane, projectile  motion,  uniform  circular  motion.  Unit ­ 3: Laws of Motion  Force and inertia, Newton’s first law of motion, momentum, Newton’s second law of motion, impulse, Newton’s third law  of motion,  law  of  conservation of  linear momentum and  its  applications,  equilibrium  of  concurrent  forces.  Static  and  kinetic  friction,  laws  of  friction,  rolling  friction.  Dynamics  of  uniform  circular  motion:  centripetal  force  and  its  applications.  Unit ­ 4: Work, Energy and Power  Work  done  by  a  constant  force  and  a variable  force,  kinetic and  potential  energies, work­energy  theorem,  power.  Potential  energy of a spring,  conservation  of mechanical  energy,  conservative and non­conservative forces,  elastic and  inelastic  collisions  in  one  and  two  dimensions.  Unit ­ 5:  Rotational Motion  Centre of mass of a two­particle system, centre of mass of a rigid body, basic concepts of rotational motion, moment of  a  force, torque, angular  momentum,  conservation of  angular  momentum  and its  applications,  moment  of inertia,  radius  of gyration,  values of  moments of  inertia  for simple  geometrical  objects,  parallel  and perpendicular axes theorems and  their  applications.  Rigid  body  rotation,  equations  of  rotational  motion.  Unit ­ 6: Gravitation  The  universal  law  of  gravitation.  Acceleration  due  to  gravity  and  its  variation  with  altitude  and  depth.  Kepler’s  laws  of  planetary  motion.  Gravitational  potential  energy,  gravitational  potential.  Escape  velocity,  orbital  velocity  of  a  satellite,  geostationary  satellites.  Unit  ­  7: Properties of Solids and Liquids  Elastic  behaviour,  stress­strain  relationship,  Hooke’s  law,  Young’s  modulus,  bulk  modulus,  modulus  of  rigidity.  Pressure  due  to  a  fluid  column,  Pascal’s  law  and  its  applications.  Viscosity,  Stokes’s  law,  terminal  velocity,  streamline  and  turbulent  flow,  Reynolds  number,  Bernoulli’s  principle  and  its  applications.  «  For  latest  information  refer  prospectus  2014 vi  Surface energy and surface tension, angle of contact, application of surface tension ­ drops, bubbles and capillary rise.  Heat,  temperature,  thermal  expansion,  specific heat  capacity, calorimetry,  change  of  state,  latent  heat.  Heat  transfer­conduction,  convection  and  radiation,  Newton’s  law  of  cooling.  Unit ­ 8: Thermodynamics  Thermal equilibrium, zeroth law  of thermodynamics, concept of temperature, heat, work and internal energy, first law  of  thermodynamics.  Second  law  of  thermodynamics,  reversible  and  irreversible  processes,    Carnot engine  and  its  efficiency.  Unit ­ 9: Kinetic Theory of Gases  Equation  of state  of  a perfect gas, work done  on  compressing a  gas.  Kinetic theory of gases ­ assumptions, concept of pressure, kinetic energy and temperature, rms speed of gas molecules,  degrees  of  freedom,  law  of  equipartition  of  energy,  applications  to  specific  heat  capacities  of  gases,  mean  free  path,  Avogadro’s  number.  Unit ­ 10: Oscillations and Waves  Periodic motion ­ period, frequency, displacement as a function of time, periodic functions, simple harmonic motion (S.H.M.)  and its equation, phase, oscillations of a spring ­ restoring force and force constant, energy in S.H.M. ­ kinetic and potential  energies, simple pendulum ­ derivation of expression for its time period, free, forced and damped oscillations, resonance.  Wave motion, longitudinal and transverse waves, speed of a wave, displacement relation for a progressive wave, principle  of superposition of waves, reflection of waves, standing waves in strings and organ pipes, fundamental mode and harmonics,  beats,  Doppler  effect  in  sound.  Unit ­ 11: Electrostatics  Electric charges, conservation of charge, Coulomb’s law­forces between two point charges, forces between multiple charges,  superposition  principle  and  continuous  charge  distribution.  Electric field, electric field due to a point charge, electric field  lines, electric dipole, electric field due to a dipole, torque  on a  dipole  in  a  uniform  electric  field.  Electric flux, Gauss’s law and its applications to find field due to infinitely long, uniformly charged straight wire, uniformly  charged  infinite  plane  sheet  and  uniformly  charged  thin  spherical  shell.  Electric  potential  and  its  calculation  for  a  point  charge,  electric  dipole  and  system  of  charges,  equipotential  surfaces,  electrical  potential  energy  of  a  system  of  two  point  charges  in  an  electrostatic  field.  Conductors and insulators, dielectrics and electric polarization, capacitor, combination of capacitors in series and in parallel,  capacitance of a parallel plate capacitor with and without dielectric medium between the plates, energy stored in a capacitor.  Unit ­ 12: Current Electricity  Electric  current,  drift  velocity,  Ohm’s  law,  electrical  resistance,  resistances  of  different  materials,  V­I characteristics of ohmic and non­ohmic conductors, electrical energy and power, electrical resistivity, colour code for  resistors,  series  and  parallel  combinations  of  resistors,  temperature  dependence  of  resistance.  Electric cell and its internal resistance, potential difference and emf of a cell, combination of cells in series and in parallel.  Kirchhoff’s  laws  and  their  applications,  Wheatstone  bridge,  metre  bridge.  Potentiometer  ­  principle  and  its  applications.  Unit ­ 13: Magnetic Effects  of Current and Magnetism  Biot  ­  Savart  law  and its  application  to  current  carrying  circular  loop.  Ampere’s  law  and  its  applications  to  infinitely  long  current  carrying  straight  wire  and  solenoid.  Force  on  a  moving  charge  in  uniform magnetic and  electric  fields,  cyclotron.  Force  on  a  current­carrying  conductor  in  a  uniform  magnetic  field,  force  between  two  parallel  current­carrying  conductors­definition of ampere, torque experienced by a current loop in uniform magnetic field, moving coil galvanometer,  its  current  sensitivity  and  conversion  to  ammeter  and  voltmeter. vii  Current loop as a magnetic dipole and its magnetic dipole moment, bar magnet as an equivalent solenoid, magnetic field  lines, earth’s  magnetic  field  and  magnetic  elements,    para­,  dia­  and  ferro­ magnetic  substances  .  Magnetic  susceptibility  and  permeability,  hysteresis,  electromagnets  and  permanent  magnets.  Unit  ­ 14:  Electromagnetic Induction  and Alternating  Currents  Electromagnetic induction, Faraday’s law, induced emf and current, Lenz’s law, Eddy currents, self and mutual inductance.  Alternating  currents, peak  and  rms  value  of  alternating  current/  voltage,  reactance  and impedance,  LCR  series  circuit,  resonance,  quality  factor,  power  in AC  circuits,  wattless  current.  AC generator  and transformer.  Unit ­ 15: Electromagnetic Waves  Electromagnetic waves and their characteristics, transverse nature of electromagnetic waves, electromagnetic spectrum  (radio  waves,  microwaves,  infrared,  visible,  ultraviolet,  X­rays, gamma  rays). Applications  of electromagnetic  waves.  Unit ­ 16: Optics  Reflection  and  refraction  of  light  at  plane  and  spherical  surfaces,  mirror  formula,  total  internal  reflection  and  its  applications,  deviation  and  dispersion  of  light  by  a  prism,  lens  formula,  magnification,  power  of  a  lens,  combination  of  thin  lenses  in  contact,  microscope  and  astronomical  telescope  (reflecting  and  refracting)  and  their  magnifying  powers.  Wave optics ­ wavefront  and Huygens  principle,  laws of  reflection and refraction using Huygens principle, interference,  Young’s  double  slit  experiment  and  expression  for  fringe  width,  coherent  sources  and  sustained  interference  of  light,  diffraction due to  a single slit, width  of central maximum, resolving  power of microscopes and  astronomical telescopes,  polarisation,  plane  polarized  light,  Brewster’s  law,  uses  of  plane  polarized  light  and  polaroids.  Unit ­ 17: Dual Nature of Matter and Radiation  Dual nature of radiation,  photoelectric effect, Hertz and Lenard’s observations, Einstein’s photoelectric equation, particle  nature  of  light.  Matter  waves­wave  nature  of  particle,  de  Broglie  relation,  Davisson­Germer  experiment.  Unit ­ 18:  Atoms and Nuclei  Alpha­particle  scattering  experiment,  Rutherford’s  model  of atom,  Bohr  model,  energy  levels,  hydrogen  spectrum.  Composition  and  size  of  nucleus,  atomic  masses,  isotopes,  isobars,  isotones,  radioactivity­alpha,  beta  and  gamma  particles/rays and their properties, radioactive decay law, mass­energy relation, mass defect, binding energy per nucleon  and  its  variation  with  mass  number,  nuclear  fission  and  fusion.  Unit ­ 19: Electronic Devices  Semiconductors, semiconductor diode ­  I­V characteristics in forward and reverse bias, diode as a rectifier, I­V characteristics  of  LED,  photodiode,  solar  cell.  Zener  diode,  Zener  diode  as  a  voltage  regulator,  junction  transistor,  transistor  action,  characteristics  of  a transistor,  transistor  as  an amplifier (common  emitter  configuration)  and  oscillator, logic  gates  (OR,  AND, NOT, NAND and NOR),  transistor as a switch.  Unit  ­  20:  Communication  Systems  Propagation  of  electromagnetic  waves  in  the  atmosphere,  sky  and  space  wave  propagation,  need  for  modulation,  amplitude  and  frequency  modulation,  bandwidth  of  signals,  bandwidth  of  transmission  medium,  basic  elements  of  a  communication  system  (Block  Diagram  only).  SECTION B  Unit ­ 21 : Experimental Skills  Familiarity  with  the  basic  approach  and  observations  of  the  experiments  and  activities:  l  Vernier  callipers­its  use to  measure  internal  and external  diameter  and  depth of  a  vessel.  l  Screw  gauge­its  use  to  determine  thickness/diameter  of  thin  sheet/wire.  l  Simple  Pendulum­dissipation  of  energy by  plotting  a graph  between  square  of  amplitude  and  time.   Co­efficient  of  Viscosity of  a  given  viscous  liquid  by  measuring  terminal  velocity  of  a  given  spherical  body.  real gases.  Surface tension  of  water  by  capillary  rise  and  effect  of  detergents.  molecule. Gas laws ­ Boyle’s law.  physical  quantities  and  their  measurements  in  chemistry.  Potentiometer :  (i)  Comparison of emf of two primary cells.  UNIT ­ 2: STATES OF MATTER  Classification  of  matter  into  solid. Capacitor  from  mixed  collection  of  such  items.  element  and  compound.  Focal  length  of:  (i)  Convex  mirror  (ii)  Concave  mirror  and  (iii)  Convex  lens   using  parallax  method.  units.  Resistance  of  a  given  wire  using  Ohm’s  law.  chemical  equations  and  stoichiometry.  Characteristic  curves of  a  p­n  junction  diode  in  forward  and  reverse  bias.  van  der  Waals  equation.  Plotting  a  cooling  curve  for  the  relationship between  the  temperature  of  a  hot  body  and  time.  Liquid State  ­ Properties  of liquids ­ vapour  pressure.  Solid  State  ­  Classification  of  solids  ­  molecular.  Using  multimeter  to:  (i)  Identify  base  of  a  transistor  (ii)  Distinguish  between  npn  and  pnp  type  transistor  (iii)  See  the  unidirectional  flow  of current  in  case  of  a  diode  and  an  LED.  Resistance  and  figure of  merit  of  a  galvanometer  by half deflection  method.  magnetic  and  dielectric  properties.  significant  figures. Transistor. viscosity and surface tension and effect of  temperature on them  (qualitative  treatment  only).  atomic  and  molecular  masses.  IC.viii  l  l  l  l  l  l  l  l  l  l  l  l  l  l  l  l  l  l  l  Metre Scale ­ mass of  a given object by principle  of moments.  concept  of  atom.  Speed  of  sound  in  air  at  room temperature  using a  resonance  tube. Ideal gas equation.  S.  deviation from  Ideal  behaviour.  Gaseous State ­ Measurable properties of gases.  Bragg’s  Law and  its  applications.  concept of  average.  liquid  and  gaseous  states. Avogadro’s  law.  unit  cell  and  lattices.  empirical  and  molecular  formulae. Charle’s law.  molar  mass.  imperfection  in  solids. Graham’s law of diffusion.  dimensional  analysis.  Young’s  modulus  of  elasticity of  the  material  of  a  metallic  wire.  mole  concept.  Specific  heat  capacity  of  a  given  (i)  solid  and  (ii)  liquid by  method  of  mixtures.  transistor  or  IC).  calculations  involving  unit  cell  parameters.I.  Dalton’s  atomic  theory.  packing  in  solids  (fcc.  compressibility  factor.  electrical. bcc  and  hcp  lattices).  voids. Resistor. .  amorphous  and  crystalline  solids  (elementary  idea).  (iv)  Check  the  correctness  or  otherwise of  a  given  electronic  component (diode. kinetic theory of gases  (only  postulates).  precision  and  accuracy.  (ii)  Determination  of  internal  resistance  of  a  cell.  Refractive  index  of  a  glass  slab  using  a  travelling  microscope. Dalton’s law of partial pressure.  ionic.  Characteristic  curves  of a  Zener  diode  and  finding  reverse  break  down  voltage.  CHEMISTRY  SECTION  ­  A (Physical  Chemistry)  UNIT  ­  1:  SOME  BASIC  CONCEPTS  IN  CHEMISTRY  Matter  and  its  nature.  LED.  percentage  composition.  Identification  of  Diode. root  mean square  and most  probable velocities.  Resistivity of  the  material  of a  given  wire  using  metre  bridge.  Laws  of  chemical  combination. concept of absolute scale of temperature.  covalent  and  metallic  solids.  Characteristic  curves  of  a  transistor  and  finding  current  gain  and  voltage  gain.  Plot  of angle  of deviation vs  angle  of  incidence  for  a  triangular  prism.   state  functions.  liquid  ­  gas  and  solid  ­  gas  equilibria.  ionization  and  solution. molarity.  general  characteristics  of  equilibrium  involving  physical  processes.  effect  of  catalyst.  nature  of  electromagnetic  radiation. quantum mechanical model of atom. molar heat capacity.  Equilibria  involving  chemical  processes  ­  Law  of  chemical  equilibrium.  molecular  orbital  electronic  configurations  of  homonuclear  diatomic  molecules.  dipole  moment.  UNIT ­ 7: EQUILIBRIUM  Meaning  of  equilibrium.  Ionic equilibrium ­ Weak and strong electrolytes.  UNIT ­ 4: CHEMICAL BONDING AND MOLECULAR STRUCTURE  Kossel  ­  Lewis  approach  to chemical  bond  formation.  spectrum of hydrogen atom.  equilibrium  constants  (K p and K c) and their significance.  hydration.  types  of  processes.  Bronsted  ­  Lowry  and  Lewis)  and  their  ionization.  concept  of  atomic  orbitals  as  one electron  wave  functions.  Resonance. vapour pressure of solutions and Raoult’s law ­ Ideal and non­ideal solutions. formation.  Covalent  Bonding  ­  concept  of  electronegativity.  van’t  Hoff  factor  and  its significance. types of molecular orbitals (bonding. various concepts of acids and bases (Arrhenius.    pressure.  Hess’s law of constant heat summation. elementary ideas of quantum mechanics.  Le  ­  Chatelier’s  principle.  bond  length  and  bond  energy.  concept  of  bond  order.  hydrogen  bonding  and  its  applications. DG°  (standard  Gibbs  energy  change)  and  equilibrium  constant.  phase  transition.  extensive  and  intensive  properties. atomization. shapes of  s.  Pauli’s  exclusion principle and Hund’s rule.  Ionic Bonding ­ Formation of ionic bonds. dual nature of matter. angular momentum and magnetic quantum numbers) and their significance.  calculation of lattice enthalpy.  Fajan’s  rule.  concept  of  dynamic  equilibrium.  Henry’s  law. extra stability of half­filled and completely filled  orbitals. electron  spin and  spin  quantum  number.  elevation  of  boiling  point  and  osmotic  pressure. antibonding).  factors affecting the formation of ionic bonds.  Quantum mechanical approach to covalent bonding ­ valence bond theory ­ its important features. electronic configuration of elements.  depression  of  freezing  point. heat internal energy and enthalpy.  photoelectric  effect.  UNIT ­ 6: SOLUTIONS  Different methods for expressing concentration of solution ­ molality. Heisenberg  uncertainty principle. LCAOs. enthalpies of bond dissociation.  Second  law  of  thermodynamics  ­  Spontaneity  of  processes. de­Broglie’s relationship.  acid  ­  base  equilibria  (including  multistage  ionization)  and  ionization . derivation  of the  relations for energy  of the  electron and radii of the different orbits.  variation  of Y and Y 2  with  r  for  1s  and  2s  orbitals. factors affecting equilibrium concentration. rules  for filling  electrons  in orbitals  – Aufbau principle. concept of hybridization  involving  s.  Molecular Orbital Theory ­ its important features.  First law of thermodynamics ­ Concept of work. ionization of electrolytes.ix  UNIT ­ 3: ATOMIC STRUCTURE  Thomson  and  Rutherford  atomic  models  and  their  limitations.  temperature. vapour pressure ­ composition  plots  for  ideal  and  non­ideal  solutions.  concept  of  ionic  and  covalent  bonds. sigma and  pi­bonds. Bohr  model of  hydrogen atom ­  its postulates.  Valence  Shell  Electron  Pair  Repulsion  (VSEPR) theory  and  shapes  of  simple  molecules. mole fraction. DS  of  the universe  and DG  of  the  system as  criteria for  spontaneity. significance of DG and DG° in chemical equilibria. percentage (by volume and  mass both). combustion.  Equilibria  involving  physical  processes  ­  Solid  ­  liquid.  Elementary  idea  of  metallic  bonding. sublimation.  various  quantum numbers (principal.  determination  of  molecular  mass  using  colligative  properties. heat capacity.  abnormal  value  of molar  mass.  colligative  properties  of  dilute  solutions  ­  relative  lowering  of  vapour  pressure. limitations of Bohr’s model.  p  and  d  ­ orbitals.  UNIT ­ 5: CHEMICAL THERMODYNAMICS  Fundamentals  of  thermodynamics:  system  and  surroundings.  p  and  d  orbitals. its important features.   dialysis.  ores. rules for assigning oxidation number. . periodic trends in properties of  elements­atomic and ionic radii. Zn and Fe.  Preparation and properties of some important compounds ­  sodium carbonate.  steps  involved  in  the  extraction  of  metals  ­  concentration.  activation  energy  and  its calculation. order  and  molecularity of  reactions. oxidation states and chemical reactivity. p.  reactions  and  uses  of  hydrogen  peroxide. electron gain enthalpy. lyophobic.  UNIT ­ 14:  s  ­ BLOCK ELEMENTS (ALKALI AND ALKALINE EARTH METALS)  Group ­ 1 and 2 Elements  General introduction. electrode potentials including standard  electrode  potential. different types of electrodes.  dry  cell  and  lead  accumulator. Cu. preparation and properties of colloids ­ Tyndall effect.  minerals. valence. activity and selectivity of solid catalysts.  relationship  between  cell  potential  and  Gibbs’  energy  change. enzyme catalysis and its mechanism. Biological significance of Na.  Brownian  movement.  structure.  physical  and  chemical  properties  of  water  and  heavy  water.  collision  theory  of  bimolecular  gaseous  reactions (no  derivation).  conductance  in  electrolytic  solutions. Mg and Ca.  adsorption  from  solutions.  buffer  solutions.  differential  and integral forms of zero and first order reactions.  balancing  of  redox  reactions.  UNIT ­ 8 : REDOX REACTIONS AND ELECTROCHEMISTRY  Electronic concepts of oxidation and reduction. anomalous  properties  of  the first  element  of  each  group. their characteristics and half ­ lives. s. ionization enthalpy.  coagulation  and  flocculation.  elementary  and complex  reactions.  fuel  cells. sodium hydroxide and sodium hydrogen  carbonate.  solubility  of  sparingly  soluble  salts  and  solubility  products.  isotopes.  multi molecular. colloids and suspensions.  common  ion  effect. oxidation number.  UNIT ­ 9 : CHEMICAL KINETICS  Rate  of  a  chemical  reaction.  properties  and  uses  of  hydrogen.  pH  scale.  reduction (chemical and electrolytic  methods) and refining with special reference to the extraction of Al. effect of temperature on rate of  reactions  ­ Arrhenius theory.  Catalysis ­ Homogeneous and heterogeneous. Plaster of Paris and cement.  emulsions  and  their  characteristics.  .  Freundlich  and  Langmuir  adsorption  isotherms. Industrial uses of lime.  Nernst  equation  and  its  applications.  hydrogen  as  a  fuel.  temperature.  UNIT ­ 10 : SURFACE CHEMISTRY  Adsorption ­ Physisorption and chemisorption  and their characteristics.x  constants.  factors  affecting  the  rate  of  reactions  ­concentration.  diagonal  relationships.  emf  of  a  galvanic  cell  and  its  measurement.  electrophoresis.  Electrolytic  and  metallic  conduction.  Electrochemical cells ­ electrolytic and galvanic cells.  Colloidal state ­ distinction among true solutions.  preparation. rate law.  thermodynamic  and  electrochemical  principles  involved  in  the  extraction  of  metals.  rate constant and  its units. d and f block elements.  UNIT ­ 12: GENERAL PRINCIPLES AND PROCESSES OF ISOLATION OF METALS  Modes  of  occurrence  of  elements  in  nature.  classification  of  hydrides  ­  ionic.  preparation. pressure  and  catalyst.  UNIT ­ 15:  p ­ BLOCK ELEMENTS  Group ­ 13 to Group 18 Elements  General Introduction ­ Electronic configuration and general trends in physical and chemical properties of elements across  the  periods and  down the  groups. limestone. electronic configuration and general trends in physical and chemical properties of elements. redox reactions. K.  ionization  of  water.  Section ­ B (Inorganic Chemistry)  UNIT ­ 11: CLASSIFICATION OF ELEMENTS AND PERIODICITY IN PROPERTIES  Modern periodic law and present form of the periodic table.  UNIT ­ 13: HYDROGEN  Position  of  hydrogen  in  periodic  table.  half  ­  cell  and  cell  reactions.  unique  behaviour  of  the  first element  in  each  group. macromolecular and associated colloids (micelles).  hydrolysis  of  salts  and  pH  of  their  solutions.  molar  conductivities  and  their  variation  with  concentration:  Kohlrausch’s  law  and  its  applications.  covalent  and  interstitial. factors affecting adsorption  of gases on solids. classification of colloids ­ lyophilic.  denticity.  UNIT ­ 17: COORDINATION COMPOUNDS  Introduction to coordination compounds. their harmful effects and  prevention. ligands.  boron  trifluoride. trends in the acidic nature of hydrogen halides. structures and  uses  of  sulphuric  acid  (including  its  industrial  preparation).  fumes. phosphine and phosphorus halides. Werner’s theory. alloy formation. properties and uses of boron and aluminium. electronic configuration.  Water Pollution ­ Major pollutants such as. preparation.  structures  of  fluorides  and  oxides  of  xenon. boric acid.  ionization  enthalpy. general trends in properties of the first row  transition  elements  ­  physical  properties.  dust. importance of coordination compounds (in qualitative analysis.  hydrocarbons.  magnetic properties.  green  house  effect  and  global  warming. properties.  colour. IUPAC nomenclature  of  mononuclear  coordination  compounds.  Stratospheric pollution  ­  Formation  and breakdown  of  ozone.  properties.  their  sources.  Strategies  to  control  environmental  pollution.  zeolites  and  silicones. complex formation. allotropic forms of sulphur.  oxidation  states  and  lanthanoid  contraction. chelation.xi  Group ­ 13  Preparation.  Particulate  pollutants ­  Smoke.  Structures  of  oxoacids  of  sulphur.  Soil Pollution ­ Major pollutants like pesticides (insecticides. silicon tetrachloride. organic wastes and chemical pollutants.  atomic  radii.  UNIT ­ 16:  d ­ and f ­ BLOCK ELEMENTS  Transition  Elements  General introduction. structure. coordination number.  bonding  ­valence  bond  approach  and  basic  ideas  of  crystal  field  theory. preparation. silicates. properties and uses of allotropes and oxides of carbon.  depletion of  ozone layer  ­  its mechanism  and effects. PCl 5 ).  water  and soil.  harmful effects  and  prevention.  aluminium  chloride  and  alums. structure.  Group ­ 14  Tendency for catenation.  catalytic  behaviour.  nitrogen  and  sulphur. herbicides and fungicides). diborane.  allotropic  forms  of  phosphorus. structures and uses of ozone. their  sources.  Tropospheric  pollutants  ­  Gaseous  pollutants:  oxides  of  carbon. properties and uses of hydrochloric acid.  Group ­ 16  Preparation.  structure and  uses of ammonia. (PCl 3 .  Actinoids  ­  Electronic  configuration  and  oxidation  states.  properties and uses of borax.  acid  rain.  UNIT ­ 18: ENVIRONMENTAL CHEMISTRY  Environmental  Pollution  ­ Atmospheric. properties and uses of K 2 Cr 2 O 7  and  KMnO 4 . their harmful effects and prevention. properties.  preparation.  Inner  Transition  Elements  Lanthanoids  ­  Electronic  configuration.  harmful  effects  and  prevention. .  Group ­ 15  Properties  and  uses  of  nitrogen and  phosphorus.  mist. structures of interhalogen  compounds  and  oxides  and  oxoacids  of  halogens. pathogens. smog.  oxidation  states. nitric acid. structures of oxides and oxoacids of nitrogen  and  phosphorus.  isomerism. interstitial compounds.  Atmospheric  pollution  ­  tropospheric  and stratospheric.  Group ­ 18  Occurrence  and  uses  of  noble  gases. occurrence and characteristics. extraction of metals  and  in  biological  systems).  Group ­ 17  Preparation. colour and magnetic properties.   UNIT ­ 24: ORGANIC COMPOUNDS CONTAINING NITROGEN  General  methods  of  preparation.  ozonolysis.  Qualitative  analysis  ­  Detection  of  nitrogen.  Common  types  of  organic  reactions  ­  Substitution.  classification.    mechanisms  of  substitution  reactions.  Alkynes  ­  Acidic  character. halogens.  homologous  series.  Aromatic hydrocarbons  ­  Nomenclature.  mechanism  of  dehydration. aldol condensation.  electromeric  effect.  reactions  and  uses.  Alkanes  ­  Conformations:  Sawhorse  and  Newman projections  (of  ethane).  freons and  DDT.  Aldehydes and Ketones ­ Nature of carbonyl group.  UNIT ­ 20: SOME BASIC PRINCIPLES OF ORGANIC CHEMISTRY  Tetravalency of carbon. halogens.  numerical  problems  in  organic  quantitative  analysis.  reactions  and  uses. nitration.  polymerization.  haloform  reaction.  addition  of  hydrogen. Cannizzaro  reaction.  NH 3  and  its  derivatives).  chemical  tests  to  distinguish  between  aldehydes  and  ketones.  elimination  and  rearrangement. electrophilic  substitution reactions: halogenation. reduction (Wolff Kishner and Clemmensen). acidity of a­hydrogen.  properties.  secondary  and  tertiary  alcohols.  Carboxylic acids  ­ Acidic  strength and factors affecting it.  Calculations  of  empirical  formulae  and  molecular  formulae.  general  methods  of  preparation. iodoform. nitrogen. relative reactivities of aldehydes  and  ketones.  and  polymerization.  distillation.  properties  and  reactions.  Grignard reagent.  Alkenes ­ Geometrical isomerism.  Alcohols  ­  Identification  of  primary. shapes of simple molecules ­ hybridization (s and p).  oxidation. .  important  reactions  such  as  ­  nucleophilic  addition  reactions  (addition  of  HCN. water.  sublimation.  differential  extraction  and  chromatography  ­  principles  and  their  applications. hydrogen  halides  (Markownikoff’s  and  peroxide  effect). oxidation.  nature  of C­X  bond.  Electronic  displacement  in  a  covalent  bond  ­ Inductive  effect. sulphur. mechanism of  halogenation  of alkanes.xii  SECTION ­ C (Organic Chemistry)  UNIT ­ 19: PURIFICATION AND CHARACTERISATION OF ORGANIC COMPOUNDS  Purification  ­  Crystallization.  Reimer ­ Tiemann  reaction. addition.  Diazonium  Salts  ­  Importance  in  synthetic  organic  chemistry.  Ethers  ­  Structure. stability of carbocations  and  free  radicals.  UNIT ­ 22: ORGANIC COMPOUNDS CONTAINING HALOGENS  General  methods  of  preparation.  phosphorus  and  halogens.  UNIT ­ 23: ORGANIC COMPOUNDS CONTAINING OXYGEN  General  methods  of  preparation.  properties.  UNIT ­ 21: HYDROCARBONS  Classification.  Nomenclature  (trivial  and  IUPAC)  Covalent bond fission ­ Homolytic and heterolytic: free radicals.  secondary  and  tertiary  amines  and  their  basic  character.  sulphur.  nitrogen  and  sulphur.  properties  and  reactions.  isomerism.  resonance  and hyperconjugation.  Uses/environmental  effects  of  chloroform.  benzene  ­  structure  and  aromaticity.  Isomerism  ­  structural  and  stereoisomerism. hydrogen. phosphorus.  Amines  ­  Nomenclature. mechanism of electrophilic addition: addition of hydrogen.  oxygen. carbocations and carbanions.  IUPAC  nomenclature.  halogens.  mechanism  of electrophilic substitution:  halogenation. nitration and sulphonation. directive influence of functional group in mono­substituted  benzene. classification of organic compounds based  on  functional  groups:  and  those  containing  halogens.  Phenols  ­ Acidic nature.  Quantitative analysis (Basic principles only) ­ Estimation of carbon.  water  and  hydrogen  halides. nucleophilic addition to >C  O group.  electrophiles  and  nucleophiles.  structure  basic  character  and  identification  of  primary. Friedel – Craft’s alkylation and acylation.  Mohr’s salt  vs  KMnO 4 .  p­nitroacetanilide.  one­one.  nature  of  roots.  peptide bond. some important polymers with emphasis on their monomers  and  uses  ­  polythene.  potash  alum.  Chemistry involved in the titrimetric exercises ­ Acids.  enzymes. union.  Carbohydrates ­  Classification: aldoses and ketoses. AI 3+ .  CI – . Enthalpy  of  solution  of  CuSO 4  2.  polypeptides. disinfectants. antimicrobials. relations. Zn 2+ . proteins  ­  primary.  antacids.  functions.  carbonyl  (aldehyde  and  ketone). antiseptics. constituent monosaccharides  of  oligosaccharides  (sucrose. halogens) in organic compounds.  Chemistry  involved  in  the  preparation  of  the  following:  Inorganic  compounds  ­  Mohr’s  salt.  square root of  a complex number.  Organic  compounds  ­  Acetanilide. Cu 2+ .  artificial  sweetening  agents  ­  common  examples. natural and synthetic rubber and vulcanization. NH 4 + .  Chemicals  in  food  ­  Preservatives.  into  and  onto  functions.  SO 4 2– .  Preparation  of  lyophilic  and  lyophobic  sols. Ca 2+ . quadratic equations in  real and complex number  system and their  solutions. intersection and complement of sets and their algebraic properties.  Br – .  denaturation  of  proteins.  Vitamins  ­  Classification  and  functions.  MATHEMATICS  UNIT ­ 1 : SETS. Fe 3+ .  Chemical  principles  involved  in  the  qualitative  salt  analysis:  Cations ­ Pb 2+ . Ni 2+ .  lactose.  maltose).  types  of  relations. triangle inequality.  I –  (insoluble  salts  excluded).  UNIT ­ 26: BIOMOLECULES  General  introduction  and  importance  of  biomolecules.  NO 3 – . Ba 2+ .  cleansing  action.  iodoform.  nylon.  composition  of  functions.  antihistamins  ­  their  meaning  and  common  examples. bases and the use of indicators.  carboxyl  and  amino  groups  in  organic  compounds. Argand diagram.  general  methods  of  polymerization  ­  addition  and  condensation. S.  NO 2 – .  Cleansing  agents  ­  Soaps  and  detergents. oxalic acid vs KMnO 4 . . representation of complex numbers in the form a + ib and their representation  in a plane.  Nucleic  acids  ­  Chemical  constitution  of DNA  and  RNA.  UNIT ­ 27: CHEMISTRY IN EVERYDAY LIFE  Chemicals in medicines ­ Analgesics.  UNIT ­ 2 : COMPLEX NUMBERS AND QUADRATIC EQUATIONS  Complex numbers as ordered pairs of reals.  Enthalpy of  neutralization  of  strong  acid  and strong  base.  copolymerization. power set.  relation  between  roots  and  coefficients. monosaccharides (glucose and fructose).  4. antibiotics.  UNIT ­ 28: PRINCIPLES RELATED TO PRACTICAL CHEMISTRY  Detection of extra elements (N.  equivalence  relations.  polyester  and  bakelite.  aniline  yellow. modulus and  argument (or amplitude) of a complex number. RELATIONS AND FUNCTIONS  Sets and their representation.  tertiary and  quaternary  structure  (qualitative  idea  only). secondary.  Anions  –  CO 3 2– . algebra of  complex numbers. Kinetic  study of  reaction  of  iodide  ion  with  hydrogen  peroxide  at room temperature.  biological functions of  nucleic  acids.xiii  UNIT ­ 25: POLYMERS  General  introduction  and  classification  of  polymers. detection of the following functional groups: hydroxyl  (alcoholic  and  phenolic).  3. antifertility drugs.  S 2– . Mg 2+ .  Chemical  principles  involved  in  the  following  experiments:  1. tranquilizers.  formation  of  quadratic  equations  with  given  roots.  Proteins  ­  Elementary  Idea  of a ­  amino acids.  . ò x2 ± a2 . Graphs of simple functions. å n  . ò ax 2  + dx + c ò a 2 ± x 2 dx and  ò x 2 . derivatives of order upto two.r) and  C  (n.  simple  applications.  product  and  quotient  of  two  functions. equation of family of lines passing through the point  of  intersection  of  two  lines. insertion  of arithmetic.  Integration  using  trigonometric  identities. permutation as an arrangement and combination as selection. ò a2 .  area  of  triangles  using  determinants. Evaluation of definite integrals. determinants and matrices of order two and three.x 2 . algebra of matrices. locus and its equation. types of matrices. Rolle’s and Lagrange’s Mean value theorems. Properties of determinants.  general  term  and  middle  term.  Evaluation of simple integrals of the type  ( px + q )dx ( px + q ) dx  dx dx dx dx dx dx  ò x 2 ± a2 .  UNIT ­  5 :  MATHEMATICAL INDUCTION  Principle  of  Mathematical  Induction  and  its  simple  applications. orthocentre and circumcentre of a triangle.r). solution  of  homogeneous and  linear  differential  equations  of  the  type: dy  + p( x )y = q ( x )  dx  UNIT ­ 11: COORDINATE GEOMETRY  Cartesian system of rectangular coordinates in a plane.  UNIT  ­ 8  :  LIMITS.M. . algebra of functions. Fundamental integrals involving algebraic. angles between two lines.  Adjoint  and  evaluation  of  inverse  of  a  square  matrix  using determinants and elementary transformations. Test of consistency and solution of simultaneous linear equations in  two  or  three  variables  using  determinants and  matrices.x2 .  Differentiation  of  trigonometric. ò ax2 + bx + c . intersection of lines.  UNIT ­ 4 : PERMUTATIONS AND COMBINATIONS  Fundamental principle of counting.  properties  of  Binomial  coefficients  and  simple  applications. and G. exponential and logarithmic functions.  tangents  and  normals. rational.  Sum upto  n terms of  special series: 2 3  å n.a 2  dx Integral as limit of a sum.  equations  of internal and  external bisectors  of  angles  between two  lines. distance formula. Fundamental theorem of calculus.  coordinates of centroid. å n . Properties of definite integrals. Limits.  CONTINUITY AND  DIFFERENTIABILITY  Real ­ valued functions.  UNIT ­ 10: DIFFERENTIAL EQUATIONS  Ordinary differential equations. continuity and differentiability.  determining  areas  of  the  regions  bounded  by  simple  curves  in  standard  form.  exponential. logarithmic and exponential functions. ò ax 2 + bx + c . Relation  between A. translation  of  axes.  UNIT ­ 7 : SEQUENCES AND SERIES  Arithmetic and  Geometric progressions.  composite and implicit functions. trigonometric.  intercepts  of  a  line  on  the  coordinate  axes. Meaning of  P (n. ò a2 .  logarithmic. their order and degree.  evaluation  of  determinants. Solution of differential equations  by  the  method  of separation  of  variables.  Straight lines ­ Various forms of equations of a line. section formula.  by  parts  and  by  partial  fractions. Applications  of  derivatives:  Rate  of  change  of  quantities.  monotonic  ­  increasing  and  decreasing  functions. Formation of differential equations.  UNIT ­ 6 : BINOMIAL THEOREM AND ITS SIMPLE APPLICATIONS  Binomial  theorem  for  a  positive  integral  index. Arithmetic  ­ Geometric  progression.M. Differentiation of the sum. ò ax2  + bx + c .  inverse  trigonometric.  inverse functions.xiv  UNIT ­ 3 : MATRICES AND DETERMINANTS  Matrices.  Maxima  and  minima  of  functions  of  one  variable.  Integration  by  substitution. trigonometric. conditions for concurrence  of three lines.  slope  of  a  line.  polynomials.  parallel  and  perpendicular  lines.  UNIT ­ 9 : INTEGRAL CALCULUS  Integral as an anti ­ derivative.  distance  of a  point  from  a line. difference. geometric means  between two given numbers.   UNIT ­ 16: MATHEMATICAL REASONING  Statements. ellipse and hyperbola) in standard forms. Baye’s theorem.  Heights  and  Distances. equation of a circle when the end points of a diameter are given.xv  Circles.  UNIT ­ 15: TRIGONOMETRY  Trigonometrical identities and equations.  intersection  of  a  line  and  a  plane. implied by. points of intersection of a line and a circle with  the  centre  at  the  origin  and  condition  for  a  line  to  be  tangent  to  a  circle.  equation  of  the  tangent. distance between two points.  UNIT ­ 14:  STATISTICS AND PROBABILITY  Measures of Dispersion ­ Calculation of mean. Skew lines.  equations of conic sections (parabola. Inverse trigonometrical functions and their properties.  variance  and  mean  deviation  for  grouped  and  ungrouped  data.  UNIT ­ 12: THREE DIMENSIONAL GEOMETRY  Coordinates of a point in space. logical operations and. converse  and  contrapositive.  scalar  and vector  triple product. Understanding of tautology. components of a vector in two dimensions and three dimensional space. general form of the equation of a circle. probability distribution  of  a  random  variate. direction ratios and direction cosines.  Sections  of  cones. section formula. median. condition for y = mx + c to be a tangent  and  point(s)  of  tangency. addition of vectors.  Probability ­ Probability of an event. its radius and  centre. or. implies. Calculation of standard  deviation. addition and multiplication theorems of probability.  coplanar  lines. angle  between two intersecting lines. Trigonometrical functions.  Bernoulli  trials  and  Binomial  distribution. if and only if.  UNIT ­ 13: VECTOR ALGEBRA  Vectors and scalars. conic sections  ­ Standard form of equation of a circle. scalar  and  vector  products. contradiction. mode of grouped and ungrouped data. Equations of a line and  a  plane in  different  forms.  vvv . the shortest distance between them and its equation. xvi . PHYSICS .   7.  (2004)  8. 0. 4.  (2006)  1 .0003 and 2.  (a)  . which one does not have identical  dimensions?  (a)  moment  of inertia  and  moment of  a  force  (b)  work and torque  (c)  angular momentum and Planck’s constant  (d)  impulse and momentum  (2005)  2. 1.  (2010)  The dimension of magnetic field in M. L.  (d)  (b)  3.  The respective number of significant figures for the numbers  23.  Let [Π0] denote the dimensional formula of the permittivity    of vacuum. 2  (c)  5. are  (a)  [L –1 T]  (b)  [L –2 T 2 ]  (c)  [L 2 T –2 ]  (d)  [LT –1 ].  (a)  torque and work  (b)  stress and  energy  (c)  force and  stress  (d)  force and  work.  2.  (2002)  Answer  Key  1.1 ×  10 –3  are  (a)  4.  The physical quantities not having  same dimensions  are  (a)  torque and work  (b)  momentum and Planck's constant  (c)  stress and Young's modulus  (d)  speed and  (m 0e 0 ) –1/2 .  Dimensions  of  3.  (2003)  10.  8.1  Physics and Measurement  PHYSICS AND  MEASUREMENT  CHAPTER  1  1. T = time and A = electric  current. 5.  Identify the pair whose  dimensions are equal.  (a) 5.  where  symbols  have  their  usual  m 0e 0 meaning.  (c)  6.  Which one of the following represents the correct dimensions  of  the  coefficient  of  viscosity?  (a)  ML –1 T –2  (b)  MLT –1  –1  –1  (c)  ML  T  (d)  ML –2 T –2 .  Resistance  of  a  given  wire  is  obtained  by  measuring  the  current flowing in it and the voltage difference applied across  it.  4.  Out of the following pairs.  (c)  (c)  2. T and C (coulomb)  is given as  (a)  MT –2 C –1  (b)  MLT –1 C –1  2  –2  (c)  MT  C  (d)  MT –1 C –1 . 1. If the percentage errors in the measurement of the current  and  the  voltage  difference  are  3%  each. If M = mass. L = length.  (d)  10.  where Q denotes  the  electric  charge?  (a)  weber (Wb)  (b)  Wb/m 2  (c)  henry (H)  (d)  H/m 2 .  9. then  (a)  [Π0] =  [M –1  L 2  T –1  A]  (b)  [Π0 ] =  [M –1  L –3  T 2  A]    (c)  [Π0] =  [M –1  L –3  T 4  A 2 ]  (d)  [Π0] =  [M –1  L 2  T –1  A –2 ]      (2013)  6.  (2008)  Which of the following units denotes the dimensions ML 2 /Q 2 .  then  error  in  the  value of resistance of  the  wire  is  (a)  zero  (b)  1%  (c)  3%  (d)  6%  (2012)  7. 5  (d)  5.  5.023. 2  (b)  5.  (b)  (c)  4.  (2003)  9.  if at all.  (a)  :  Moment  of  inertia  (I)  =  mr 2 \  [I]  =  [ML 2 ]  Moment  of  force  (C)  =  r  F \  [C]  =  [r][F]  =  [L][MLT –2 ]    or  [C]  =  [ML 2  T –2 ]  Moment of inertia and moment of a force do not have identical  dimensions.  or  [L][LT -1 ] 8.  (c)  :  Viscous  force  F  =  6phrv [ F ]  F  h= \  or  [h] =  [ r ][ v ]  6 p rv [MLT -2 ]  [h] =  or  [h]  =  [ML –1  T –1 ].023 has 5 significant figures.  9.  (b)  :  [Momentum]  =  [MLT –1 ]  [Planck's  constant]  =  [ML 2 T –1 ]  Momentum  and  Planck's  constant  do  not  have  same  dimensions.  7.  2  [AT][AT]  = [M -1L-3T 4 A 2 ]  [MLT -2 ][L]2  2.  (c)  : According to Coulomb’s law  F  = é H  ù = [MT -2 A -2 ]  êë m 2  úû  1  q1q 2  1  q1q 2  \ Î0  = 4 p Π0  r 2  4 p  Fr 2  [Î0 ] = 2 Obviously  henry  (H)  has  dimensions  ML  .0003 has 1 significant figures.  (c)  :  Velocity  of  light  in  vacuum  = D R = D V +  D I  R V I The percentage error in R is  DR ´ 100 = DV ´ 100 + DI  ´ 100  = 3% + 3% = 6%  R V I 3.  (iv)The  power  of  10  is  irrelevant  to  the  determination  of  significant figures.  23.2  JEE MAIN CHAPTERWISE EXPLORER  [henry]  =  [ML 2  T –2  A –2 ]  1.  the  zero(s)  on  the  right  of  decimal point but to the left of the first non­zero digit are  not significant.  2.  (d) :  R =  V  \ I Q  6.  no matter where the decimal point is.  10. .  (d) :  Lorentz force =| F |=| qv ´ B |  \ 5.  (a)  :  Torque  and  work  have  the  same  dimensions.1 × 10 –3  has 2 significant figures.  (b) : (i) All the non­zero digits are significant.  r r  r 4.  According to the above rules.  0.  (ii) All the zeros between two non­zero digits are significant.  [ B ] = -2 -2  [ F ]  = MLT = MLT  = [MT –1 C –1 ]  [ q ][v ]  C ´ LT -1 C LT -1  (c)  :  [ML 2 Q –2 ]  =  [ML 2  A –2  [Wb]  =  [ML 2  T –2  A –1 ]  é Wb ù –2 –1  ê 2  ú = [M T A ]  ë m û  T –2 ]  or  é 1  ù [LT -1 ] = ê ú ú ëê m 0 e0  û  or  é ù [L2 T -2 ] = ê 1  ú m e ë 0 0  û \  Dimensions  of  1  = [L2 T -2 ] m 0 e 0  1 m 0 e 0  .  (iii)If  the  number  is  less  than  1.   2 2  ^ ^  (b)  .2 m s –2  20   m P (x. Which of  the following graphs correctly describes (x 1  – x 2 ) as a function  of  time  t?  (x1  –  x 2 )  (x1  –  x 2 )  x  ^  (d)  .5 v . Its speed after 10 s is  (a)  10 units  (b)  7 2 units  (c)  7 units  (d)  8.  The  position  of  the  first  body  is  given  by x 1 (t)  after  time  t  and  that  of  the  second body by x 2  (t) after the same time interval.  If the speed of water coming out of the fountain is v.3 $ j .  3. where v is the instantaneous  dt  5. j and k  are unit vectors along x.  2. where s is in metres and t  is in seconds. to come to rest.  A point P moves in counter­clockwise direction on a circular  path as shown in the figure. The  general equation  for  its  path is  (a)  y 2  =  x 2  +  constant  (b)  y = x 2  +  constant  2  (c)  y  =  x + constant  (d)  xy =  constant  (2010)  (b)  (a)  O  t  O  t  (x1  –  x 2 )  (x1  –  x 2 )  (c)  (d)  O  t  O  t  (2008)  . it starts moving  in the  positive x­direction with a constant acceleration.25 m s –1 .  dv  = - 2. At the same  instant  another  body  passes  through  x  =  0  moving  in  the  positive  x­direction  with  a  constant  speed. would be  (a)  1 s  (b)  2 s  (c)  4 s  (d)  8 s  (2011)  4. the total  area around the fountain that gets  wet is  v 4  p  v 4  v 2  v 2  (a)  p  g (b)  p  g 2  (c)  2  g 2  (d)  p  g 2  (2011)  A small particle of mass m is projected at an angle q with the  x­axis with an initial velocity v 0  in the x­y plane as shown in  v 0 sin q the figure. y  and z‑axis  respectively.  (c)  12 m s –2  (d)  7.  The maximum horizontal distance that the boy can throw the  same stone up to will  be  (d)  20 2 m  (a)  10 m  (b)  10 2 m  (c)  20 m  (2012)  7. At a time  t <  g . the acceleration  a  at  a  point  P(R. q)  on  the  circle  of  radius  R  is  (Here q  is  measured from the x­axis)  ^ ^  r  A particle is moving with velocity  v = K ( y i + x j ).  A water fountain on the ground sprinkles water all around it.  2 ^ 2  ^  (a)  v i +  v  j  R R 8.  the angular momentum of  y  the particle  is  ^  (a)  1  mgv0 t 2 cos q i 2  (b)  . where  A projectile is given an initial velocity of  ($  $  i  is  along  the  ground  and  $  j  is  along  the  vertical.5 units  (2009)  10.v cos q i + v  sin q  j  R R 2 2  2 2  ^ ^  ^ ^  (c)  . is decelerated  at a rate given by  r  For a particle in uniform circular motion. The time taken by the object.  (2010)  (c)  mgv0 t cos q k^  6. y )  x  O  (2010)  A particle has an initial velocity  3$  i + 4 $ j and an acceleration  of  0.  If  g =  10 m/s 2 . The radius of the path is 20 m.  A body  is at rest at x =  0.  i + 2 $ j )m s.v  sin q  j  R R R R (2010)  An object moving with a speed of 6.1  mg v0 t 2 cos q k 2  ^ ^ ^  where  i . the equation  of  its trajectory  is  (a)  4y = 2x – 25x 2  (b)  y = x –  5x 2  2  (c)  y = 2x – 5x  (d)  4y =  2x –  5x 2  (2013)  A boy can throw a stone up to a  maximum height of 10 m.  where K  is a constant.mgv0 t 2  cos q  ^ j v 0  q  y  B  (b)  13 m s –2  9.v sin q i + v  cos q  j  (d)  . At  t =  0. The movement of P is such that  it sweeps out a length s = t 3  + 5. The acceleration  of P when t = 2 s is nearly  (a)  14 m s –2  speed.4$  i + 0.v cos q i .3  Kinematics  CHAPTER  KINEMATICS 2  1. 4  JEE MAIN CHAPTERWISE EXPLORER  11. i.  A ball is thrown from a point with a speed v 0  at an angle of  projection q.  Which  of  the  following  statements  is  false  for  a  particle  moving in a circle with a constant  angular speed?  (a)  The  velocity vector is  tangent  to the circle.  A  projectile  can  have  the  same  range R  for  two  angles  of  projection. Will the person be able to  catch the ball? If yes.  A  projectile  can  have  the  same  range  R  for  two  angles  of  projection. then its displacement after unit time (t = 1) is  (a)  v 0  +  g/2 + f  (b)  v 0  +  2g +  3f  (c)  v 0  +  g/2 + f/3  (d)  v 0  + g +  f  (2007)  12.66 m.  (2004)  13. the stopping distance will  be  (a)  20 m  (b)  40 m  (c)  60 m  (d)  80 m.  The  relation  between  time t  and  distance x  is  t  =  ax 2  +  bx  where a and  b are constants. He reaches the  ground with a speed of 3 m/s.  (2004)  19.20 m  (c)  2.  The  acceleration  is  (a) –2av 3  (b) 2av 2  (c)  –2av 2  (d) 2bv 3  (2005)  16.  A car.  A car moving with a  speed of 50 km/hr. What is  the position  of  the  ball in  T/3  second?  25.  (2004)  (d)  182 m  (2005)  21. 120 km/h. If the same car is moving at a speed  of 100 km/hr.  (b)  The  acceleration vector  is  tangent to the  circle  (c)  the acceleration vector points to the centre of the circle  (d)  the  velocity and  acceleration  vectors are  perpendicular  to  each  other.  The co­ordinates of a moving particle at any time t are given  by x = at 3  and y = bt 3 .  The displacement of  the particle varies with time as  (a)  t 3  (b)  t 2  (c)  t  (d)  t 1/2 .  (2005)  18.  (2006)  (a)  h/9 metre from  the  ground  (b)  7h/9 metre from  the  ground  (c)  8h/9 metre from  the  ground  (2004)  (d)  17h/18 metre from  the  ground. If t 1  and t 2  be the time of flights in the two cases.  (2003)  26.  (2003) .  (2003)  ms –2  towards north­west  ms –2  towards north­east  1  ms –2  towards north  2  23. 60°  (b)  yes. did he bail out?  (a)  293 m  (b)  111 m  (c)  91 m  15. If its position is  x = 0 at t = 0.  When parachute opens. then  1  2  1  2  (b)  s =  f t (a)  s =  f t 2  4  1  2  (2005)  (c)  s = ft  (d)  s =  f t 6  22. The  average acceleration in this  time is  (a)  zero  (b)  (c)  (d)  1  2  1  2  24.  then the product of the two time of flights is proportional to  (a)  1/R  (b) R  (c)  R 2  (d)  1/R 2 .  A  particle  is  moving  eastwards  with  a  velocity  of  5 m/s.  It takes T second to reach  the  ground.  then  the  product  of  the  two  time  of  flights  is  directly  proportional  to  (a)  1/R 2  (b)  1/R  (c)  R  (d)  R 2 .  (2004)  (2005)  17. If the car  is  going twice  as fast. 45°.  sin30° = 1/2. How far from the throwing point will the ball be  at  the  height  of  10  m  from  the  ground  ?  [g  =  10  m/s 2 .  A  parachutist  after  bailing  out  falls  50  m  without  friction. what  should  be the  angle  of  projection?  (a)  yes. If T 1  and T 2  be the time of flights in the two cases.33 m  (d)  8.  (2004)  14. cos30° =  (a)  5.  r r r  r 20. accelerates at the  rate f through a  distance s. The speed of the  particle at  time t  is given by  (a)  3t a 2 + b 2  (b)  3t 2 a 2 + b 2  (c)  t 2 a 2 + b 2  (d)  a 2 + b 2 . then the  angle between A and  B is  (a) p  (b) p/3  (c) p/2  (d) p/4.  If  the  total  distance traversed is 15 s.  A boy playing on the roof of a 10 m high building throws a  ball  with  a  speed  of  10  m/s  at  an  angle  of  30°  with  the  horizontal.e. the minimum stopping distance is  (a)  12 m  (b)  18 m  (c)  24 m  (d)  6 m.60 m  3 / 2 ]  (b)  4. then  continues  at  constant speed  for  time  t  and  then  decelerates  at  the  rate  f/2  to  come  to  rest. In 10 s the velocity changes to 5 m/s  northwards.  The velocity of a particle is v = v 0  + gt + ft 2 .  A ball is released from the top of a tower of height h metre. At what height. 30°  (c)  no  (d)  yes. From the same point and at the same instant a  person starts running with a constant speed v 0 /2 to catch the  ball. it decelerates at 2 m/s 2 .  A particle located at x = 0 at time t = 0.  starting from rest. can brake  to  stop within  a distance of 20  m. starts moving along  the positive x­direction with a velocity v that varies as  v = a  x . can be stopped by  brakes after at least 6 m.  If  A ´ B = B ´ A .  An automobile travelling with a speed of  60 km/h.   10.  22.  18.  (c)  (a)  (*)  (a)  (b)  3.  If  a  body looses half of its velocity on penetrating  3  cm  in  a wooden block. the ratio  of the respective distances in which the two cars are stopped  from that instant  is  (a)  1 : 1  (b)  1 : 4  (c)  1 : 8  29.  15.  11.  (c)  (d)  (a)  (c)  (d)  2.  From a building two balls A and B are thrown such that A is  thrown upwards and B downwards (both vertically).  9.  20. then  (a)  v B  > v A  (b)  v A  = v B  (c)  v A  > v B  (d)  their velocities depend  on their  masses.  26.  (2002)  4.  23.  25.  29. then how much will it penetrate more before  coming to  rest?  (a)  1 cm  (b)  2 cm  (c)  3 cm  (d)  4 cm.  27. 5 N  (c)  10 N.  Two forces are such that the sum of their magnitudes is 18 N  and  their  resultant  is  12  N  which  is  perpendicular  to  the  smaller force.  8.  14.  21.  (a)  (b)  (d)  (c)  (b) .  Speeds of two identical cars are u and 4u at a specific instant.  7.  28. Then  the  magnitudes of the  forces  are  (a)  12 N.  19.  12.  13.  (d)  (c)  (b)  (a)  (a)  6.  (b)  (c)  (b)  (b)  (d)  5.  16.  (2002)  30.  If the same deceleration is applied on both the cars.  (b)  (b)  (a)  (c)  (b)  (d)  1 : 16.  (2002)  Answer  Key  1.  30.  (2002)  28.  24. If v A and  v B  are their respective velocities on reaching the ground. 6 N  (b)  13 N.5  Kinematics  27.  17. 2 N. 8 N  (d)  16 N.  we get  dx  = Ky;  dy  = Kx  dt dt dy dy  dt  \ = ´  dx dt dx For a  particle in uniform circular  motion.6  JEE MAIN CHAPTERWISE EXPLORER  1.  ^ ^  r  ^ ^  v2 v 2  a = .sin q  j  R R 8.25  = - (2.a cos q i ...  v  = dt r  d  ^ ^  1  v= (v cos qt i + (v0 sin qt .  The maximum height a boy can throw  a stone  is  u 2  H max  = = 10 m  .  (d) : The position vector of the particle  from the  origin at  any time t is  r ^ ^  1  r = v0 cos qt i + (v0 sin qt .mgv0 t cos q k ë 2  û  2  r  ^ ^  6.  2  (12) 2  144  a c  = v  = = =  7.  a = (ac ) 2 + (at )2 = (7. q)  q  a  asinq  q  x 4.a sin q j = .25 m s  ò ydy = ò xdx y 2  =  x 2  + constant  y  t  0  2 ´ [v1/ 2 ] 6.5  v 2 sin 90 ° v 2  =  g g 4  p v  2  Area = p ( R max )  =  2  g 5.  (b) :  or  dt v On integrating.2)2 + (12) 2 » 14 m/s 2  9.gt 2 ) j )  dt 0 2  ^ ^  = v0 cos q i + (v0 sin q -  gt )  j The  angular momentum  of  the particle  about the  origin is  r r r  L = r ´ mv r r r  L = m (r ´ v )  7.v0 gt cos q k ú = ..5  v  dv = - 2.  (d) :  O acosq  P( R .  (a) : Here.  (b) : v =  u +  at  r v = (3$i + 4 $j ) + (0.5) t Þ t = -2 ´ (6..4$i + 0.(i)  2 g The maximum horizontal distance the boy can throw the same  stone is  u 2  R max  = =  20 m  (Using (i))  g 1  dv  = - 2.3 $ j ) ´ 10  . within limit (v 1  = 6.v0 gt 2 cos q) k  ^  1  + (v02 sin q cos qt .v02 sin q cos qt k + v0 gt 2 cos q k ú û  2  ^ ù ^  é 1  1  2  2  = m ê ..v0 gt 2 cos q k  ^ ^ ù 1  .2 m/s 2  R 20 20  Net  acceleration.  a =  v  towards the  centre  R From figure.(iii)  Integrating both sides of the above equation.  v = K ( y i + x j )  r  ^ ^  v = Ky i + Kx j r dx  ^ dy  ^  Q  v = i+ j  dt dt Equating equations (i) and (ii).(i)  .5 ò dt .  (c) : Let  u  be  the  velocity  of  projection  of  the  stone.gt 2  2  \  x = t  1  ´ 10 ´ t 2 = 2t - 5 t 2  2  Equation of  trajectory  is  y = 2x – 5x 2  and  y = 2t - 2.1  0  ^ ^  é = m ëv02 sin q cos qt k .  (a) : s =  t 3  + 3  \ v = ds = d  (t 3 + 3) = 3 t 2  dt dt dv = d  (3t 2 ) =  6 t  Tangential  acceleration.  (b) :  R max  = ^ ^  1  = m éê(v0 cos qt i + (v0 sin qt .  v = 3(2) 2  = 12 m/s..25) 1/2  = 2 s  - 2.5 dt  3..gt ) j ) ùû  ^  é = m ë(v02 cos q sin qt .25 m s –1  to v 2  = 0)  \ v 2  = 0  -1/ 2  òv dy  Kx x  = =  dx Ky y v 1 = 6.  a t  = 6(2) = 12 m/s 2  Centripetal  acceleration.. we  get  dv = -2.(ii)  .gt 2 v0 cos q)( .k ) ùú û  2  .gt 2 )  j 2  r r  dr  \  Velocity vector.  2  Acceleration.cos q i .gt 2 ) j )  ë 2  ^ ^  ´ (v0 cos q i + (v0 sin q .  at  = dt dt At t = 2 s.  (c) :  Given : u = $  i + 2 $ j r As  u = u x $i + u y  $ j \  u x  = 1 and u y  = 2  Also x = u xt    1  and  y = u y t . .  (ii)  r  5 ˆj . For the second it is proportional to t. v 2  = constant for the other particle.. Curve (c) satisfies  this..  .  u 2  =  2as 2 2  .t.. Therefore  it is a parabola after crossing x­axis again..  10..  t1t 2  = 18.  14.  (i)  ..  (iv)  s 1  +  s 2  +  s 3  =  15s  or  s  +  f t 1 t  +  2s  =  15s  or  f  t 1 t  =  12s  From  (i)  and  (v).  (c) : Given : velocity v = v 0  +  gt +  ft 2 \  v  =  x dx  dt 0  t  or  x = ò ( v0  + gt + ft 2 ) dt 0  gt 2 ft 3  + + C 2 3  where C is the  constant of  integration  Given : x = 0.75  =  292..75  m \  Total  height  =  50  +  242.. v 1  = at.  (a)  :  Initially.  2 is velocity of 1 – velocity of 2.  the  parachutist  falls  under  gravity \  u 2  =  2ah  =  2  ×  9.  Initially both are zero.  (c) : As u = 0.. Relative velocity of particle 1 w.  (b)  :  Velocity  in  eastward  direction  = 5iˆ  velocity  in  northward  direction  = 5 ˆ  j g f  \  x = v0  + +  ..75  =  293  m.7  Kinematics  r v = (3 + 4)$  i + (4 + 3) $  j r  Þ | v | = 49 + 49 = 98 = 7 2 units  (This value is about 9...  a  =  –2 ms –2 2  2  \  (3)  =  u  –  2  ×  2  ×  h 1  or  9  =  980  –  4  h 1  971 or  h1  =  4  or  h 1  =  242..q)  2 u sin q t1 = and  t 2  = \  g g or  \  \  4 u 2 sin q cos q 2 æ u 2 sin 2 q ö 2 R  = ´ç ÷ = g  g è g g 2  ø t 1  t 2  is  proportional  to  R.  (*)  :  For  first  part  of  journey.  (b)  :  v = a  x dx  dx  = a dt  or  dt = a  x  or  x dx  = a ò dt  or  2 x1 / 2  = a t or  ò x j  5 ˆ  a  45°  W  2  æ a ö 2  or  x = ç 2 ÷ t è ø  or  displacement  is  proportional  to  t 2 . At first the velocity of first  particle is less than that of 2.  - 5iˆ  + 5 iˆ  ... \  C = 0  x = v0 t + or  x = v0 t + gt 2 ft 3  +  2 3  At t = 1 sec 2  ft 1  s  = 12 s 2 ´  ft1 t t  or  t1  =  6  or  s = 2  ft 2  1 2  1  æ t  ö ft1  = f ç ÷ = 2 2 è 6 ø  72  ft 2  72  None  of  the  given  options  provide  this  answer.  t  or  ò dx = ò vdt 0 1 æ f  ö 4 f t 1 2  (2t1 ) 2  or  s3  = 1 ´  ç ÷ 2 è 2  ø  2 2  or  s 3  =  2s 1  =  2s..8  ×  50  =  980  m 2 s –2  He  reaches  the  ground  with  speed  =  3  m/s. If (a) is given  that is also not wrong)..  s3 = \  s2 æ 120 ö 2  s 2  æ u2 ö ç u ÷ = s Þ ç 60 ÷ = 20  è ø 1  è 1ø or  s 2  =  80  m.  (v)  15.....  11.9 units close to 10 units.  2  17...  s 2  =  vt  or  s 2  =  f  t 1  t  For  the  third  part  of  journey.÷ö 2 2  è 2 ø è 2 ø 2  r  1  | a |= ms -2  towards  north­west........  s  =  s 1 .  (d)  :  Let  a  be  the  retardation  for  both  the  vehicles  For  automobile. Then the distance travelled by  particle 1 increases as  x 1  = (1/2) at 1 2 ....  v 2  =  u 2  –  2  as \  u 1 2  –  2as 1  =  0 Þ  u 1 2  =  2as 1  2  Similarly  for  car.. t =  0... .......  1  s1 = f t1 2  =  s 2  v  =  f  t 1  For  second  part  of  journey.  (b) : Range is same for angles of projection q and (90  – q) 2u sin (90 .  (a)  :  t  =  ax 2  +  bx  Differentiate  the  equation  with  respect  to  t dx dx  \  1 = 2 ax + b  dt dt dx  or  1 = 2 axv + bv as  =  v  dt 1  v  = or  2 ax + b dv  = -2a ( dx / dt )  = -2 av ´ v 2  or  dt  (2ax + b) 2  or  Acceleration  =  –2av 3 ...r....  (iii)  E  \  2 13.5 i ˆ  Acceleration  a = 10  or  r  r  1 1 1  1  a = ˆj .  or  s =  16....  2 3  N 12..iˆ  or  | a | = çæ ÷ö + çæ . .  (a) : The person will catch the ball if his speed and horizontal  speed  of  the  ball  are  same  v  1  =  v0 cos q = 0  Þ cos q = = cos60 ° \ q  =  60°...  25..  The  given  statement  is  false. It completes its  journey  to  ground  under  gravity..  u1  = 50  hour 60 ´ 60 9 sec  2  2  u  \  Acceleration  a = .  (i)  s = 0 + .  (b)  :  Resultant  R  is perpendicular  to  smaller force  Q  and  (P  +  Q)  =  18  N \  P 2  =  Q 2  +  R 2  by  right  angled  triangle  Q  90°  or  or  R  (P 2  –  Q 2 )  =  R 2  (P  +  Q)(P  –  Q)  =  R 2  P  or  (18)(P –  Q)  =  (12) 2  or  (P  –  Q)  =  8  Hence  P  =  13  N  and  Q  =  5  N..  (i)  2  u  0 = æç ö÷ – 2 ax è 2 ø  or  u 2  =  8ax  From  (i)  and  (ii)  8ax  =  8a Þ  x  =  1  cm. \ q  = p...  (d)  :  Height  of  building  =  10  m  The  ball  projected  from  the  roof  of  building  will  be  back  to roof ­ height of 10 m after covering the maximum horizontal  range..1  = ..  ..  (ii)  21...(i)  Ball B starts with downwards velocity u and reaches ground  after  travelling  a  vertical  distance  h \  v B 2  =  u 2  +  2gh  . p..  2p.  s 1  =  2 a (4 u ) 2  16 u 2  = For  second  car.  (b)  :  The  acceleration  vector  acts  along  the  radius  of  the  circle.  9 9  r r r  r 20.  (d) :  Both are given the same deceleration simultaneously  and  both  finally  stop.... \  v A 2  =  u 2  +  2gh  ..  (ii)  30...  or  10  26..866  or  R  =  8..  23.  Formula  relevant  to  motion  :  u 2  =  2  as 2  u  For  first  car.  (c)  :  For  first  case..  2  æ u ö = u 2  – 2 a (3)  ç ÷ è 2 ø  or  3u 2  =  24a Þ  u 2  =  8a  For  latter  part  of  penetration..  \  2  gT 2  1  æ T  ö ´gç ÷ = 2 18  è 3 ø  or  18  s  =  gT 2  From  (i)  and  (ii)...  (b)  :  Ball  A  projected  upwards  with  velocity  u  falls  back  to building top with velocity u downwards..  (a)  :  For  first  part  of  penetration.. 9  h 8 h  \  Height  from  ground  =  h ..  18  s  =  2h  h  or  s =  m  from  top.æç 125 ö÷ ´ 1  = -16 m/sec 2  2 s1  è 9 ø  2 ´ 6  For  second  case.  2  Maximum  horizontal  range  ( R )  =  u  sin 2 q g (10)2  ´ sin 60 ° R= = 10 ´ 0...(ii)  From  (i)  and  (ii)  v A  =  v B .  . [Q P + Q = 18]  ....  (c)  :  Equation  of  motion  :  s = ut + 1  2  gt 2  1  h = 0 +  gT 2  2  or  2h  =  gT 2  After  T/3  sec.  (c) : Range is same for angles of projection q and (90º – q) 2u sin (90º -q)  T1 = 2u sin q and T 2  = \  g g \  \  4 u 2 sin q cos q 2 æ u 2 sin 2 q ö 2 R  = ´ç ÷ = g  g è g g 2  ø T 1  T 2  is  proportional  to  R.  by  equation  of  motion.....u  -1 æ 250 ö æ 1  ö s 2  = 2  = ´ = 24 m  \  2a 2 çè 9 ÷ø çè 16 ÷ø  or  s 2  =  24  m....  s 2  = 2a 2 a s 1  1  =  \  s2  16  .8  JEE MAIN CHAPTERWISE EXPLORER  19...=  m..  28..  (b)  :  Q  x  = at 3 dx  = 3at 2 Þ v x  = 3 at 2  \  dt Again  y  = bt 3 dy  Þ v y  = 3 b t 2  \  v 2  =  v x 2  +  v y 2  \  dt or  v 2  =  (3at 2 ) 2  +  (3bt 2 ) 2  =  (3t 2 ) 2  (a 2  + b 2 )  or  v = 3t 2 a2 + b 2 .  (a)  :  A ´ B = B ´ A AB sin q nˆ = AB sin( -q ) nˆ  or  or  sinq  =  –  sinq  or  2  sinq  =  0  or q  =  0..  \  29.  T1T 2  = 22.  2 2  km 50 ´ 1000 125 m  = = 24.  27.......66  m....  u2  = 100  km = 100 ´1000 = 250 m  hour 60 ´ 60 9 sec  2  2  .  .... 4 kg.5]  (a)  100 m  (b)  400 m  (c)  800 m  (d)  1000 m  (2005)  30°  4. the  force of the blow exerted by the ball on the hand of the player  is equal to  (a)  300 N  (b)  150 N  (c)  3 N  (d)  30  N. What  is the relative  vertical acceleration of A  with respect to B?  A  A player caught a cricket ball of mass 150 g moving at a rate  of 20 m/s.4 N s  (c)  0. Find the force  of the block of mass  m.  B  60°  (d)  1.  Then a  constant  force F  starts acting on the block of mass M to pull it. Two blocks  A and B are placed on  the  two  planes.  (2010)  The  figure  shows  the  position  ­  time  (x­t)  graph  of  one­  dimensional motion of a body of mass 0.  The  ratio  of  their  centripetal acceleration  is  (a)  m 1  : m 2  (b)  r 1  :  r 2  (c)  1 : 1  (d)  m 1 r 1  :  m 2 r 2  (2012)  A particle of mass m is at rest at the origin at time t = 0.2 m which applying the  force  and  the  ball  goes  upto  2  m  height  further.9 ms –2  in vertical direction  (a)  (b)  4. The magnitude of its momentum  is recorded  as  (a)  17. Then a is equal  to  (a)  g  (b)  g tana  (c)  g/tana  (d)  g  coseca  mg  sin a a  (2005)  . The  blocks are  kept on a smooth horizontal plane.  (2006)  8.  MF  mF  (a)  ( m + M )  (b)  M  mF  ( M + m ) F  (c)  (2007)  (d)  ( m + M )  m A ball of mass 0.2 N s  (b)  0. Their speeds are such that they make  complete  circles  in  the  same  time  t.  Two cars of masses m 1  and m 2  are moving in  circles of radii  r 1  and r 2 . Initially the blocks are at  rest and the spring is  unstretched. If the hand moves 0.  2.9  Laws of Motion  CHAPTER  LAWS OF MOTION 3  1.8 ms –2  in vertical direction  (d)  zero  4.  A block is kept on a frictionless  inclined  surface  with  angle  of  ma  inclination a.565 kg ms  (d)  17. It  is  subjected  to  a  force  F(t)  =  F o e –bt  in  the  x  direction.57 kg ms –1  (b)  17.2 kg is thrown vertically upwards by applying  a force by hand.  (2006)  9.  Consider  a  car  moving  on  a  straight  road  with  a  speed  of  100 m/s.9 ms –2  in horizontal  direction  (c)  9.00 ms –1 .8 N s  ma  cosa  10.  A body of mass  m  = 3.  The  distance  at  which  car  can  be  stopped  is  [m k  = 0. respectively.6 kg ms –1  –1  (c)  17.513  kg is  moving along  the  x­axis  with a speed of 5.  6. The magnitude  of  each impulse  is  2  x (m)  0  2  4  8  6  t(s)  10  12  14  16  A block of mass m is connected to another block of mass M  by a spring (massless) of spring constant k.  find  the  magnitude of the force. The incline is given  an acceleration a to keep the block  stationary.  Two fixed frictionless inclined  planes making an angle 30°  and 60° with the vertical are shown in the figure.  v(t )  F o  mb  7.56 kg ms –1 .  Its  speed v(t) is depicted  by  which  of  the  following  curves?  (b)  v( t )  t  t  (c)  (d)  v(t )  F o  mb  v(t )  t  t  (2012)  3.6 N s  (2010)  5. If the catching process is completed in 0.1 s. Consider g  = 10 m/s 2  (a)  22 N  (b)  4 N  (c)  16 N  (d)  20  N.  (2008)  F o  mb  Fo b  m  (a)  (a)  0.   An annular ring with inner and outer radii R 1  and R 2 is rolling  without slipping with a uniform angular speed.  (2003)  22.  (2003)  21.0  (c)  1.  (2003)  23. If a force P is applied at the free  end of the rope.06  (d)  0.0 cm  (d)  2.0 cm  (2005)  13.  A  block of  mass  M  is  pulled along  a  horizontal  frictionless  surface by a rope of mass m.  The  coefficient  of  friction  is  1  1  (b)  m s  = 1 -  2  (a)  m s  = 1 -  2  n n (c)  m k  = 1 -  1  n 2  (d)  m k  = 1 -  1  n 2  (2005)  15. A body starting  from rest at the top will again come to rest at the bottom if  the  coefficient of  friction  for  the  lower  half  is  given  by  (a)  2tanf  (b)  tanf  (c)  2sinf  (d) 2cosf (2005)  14.8 kg tied  to  a  string  are  hanging  over  a  light  frictionless pulley. Then the true statement about the scale reading is  (a)  both the scales read M kg each  (b)  the scale of the lower one reads M kg and  of the upper  one  zero  (c)  the reading of the two scales can be anything but the sum  of the reading will be  M  kg  (d)  both the scales read M/2 kg.  A rocket with a lift­off mass 3.10  JEE MAIN CHAPTERWISE EXPLORER  11.  A light spring balance hangs from the hook of the other light  spring  balance  and  a  block  of  mass  M  kg  hangs  from  the  former one.2. the  mass  of the block (in kg) is  (take g  =  10  m/s 2 )  (a)  2.8 m/s 2  (c)  5 m/s 2  (d)  4.  A bullet fired into a fixed  target loses half its velocity after  penetrating 3 cm.01.  What  will  be  its  initial  acceleration  if  it  is  released from a point 20 cm  away from the origin?  (a)  5 m/s 2  (b)  10 m/s 2  (c)  3 m/s 2  (d)  15 m/s 2  (2005)  12. Then the initial thrust  of  the blast is  (a)  3.5  (2004)  17.  How  many  bullets  can  he  fire  per  second  at the most?  (a)  one  (b)  four  (c)  two  (d)  three.  A  marble  block  of  mass  2  kg  lying  on  ice  when  given  a  velocity  of  6  m/s  is  stopped  by  friction  in  10  s.  A horizontal force of 10 N is  necessary to  just hold a block  stationary against a  wall.0  (b)  4.  (2004)  19.  The  coefficient  of  static  friction  between the block and the plane is 0. The time taken to slide is n times as much  to  slide  on  rough  incline  than  on  a  smooth  incline.75 × 10 5  N.  18. The ratio of  the  forces  experienced  by  the  two  particles  situated  on  the  inner and outer parts of  the ring.  A particle of mass 0.02  (b)  0.  F 1 /F 2  is  2  æ R 1  ö R 1  R 2  (a)  1  (b)  (c)  R  (d)  ç ÷ R 2  1  è R2  ø  (2005)  16.  The  upper  half  of  an  inclined  plane  with  inclination f  is  perfectly smooth while the lower half is rough.  Then  the  coefficient  of  friction  is  (a)  0.  A block rests on a rough inclined plane making an angle of  30°  with  the  horizontal.5 × 10 5  N  (b)  7.  A spring balance is attached to the ceiling of a lift.3 kg is subjected to a force F = – kx with  k  =  15  N/m.  (2003)  m 1  m 2  (2004)  24.0 × 10 5  N  5  (c)  14. How much further it will penetrate before  coming to rest assuming that it faces constant resistance to  motion?  (a)  1.8 m/s 2 .8.5 × 10 4  kg is blasted upwards  with an initial acceleration of 10 m/s 2 .  A  machine gun fires  a  bullet  of mass  40 g  with  a  velocity  1200 ms –1 .6  (d)  2.  (2003)  20.8 m/s 2 )  (a)  0. What is the acceleration  of  the  masses when  lift  free  to move?  (g = 9.  when the lift is stationary. The weight  of  the  block is  (a)  20 N  (b)  50 N  (c)  100 N  (d)  2 N.  A  smooth block is released at rest on a 45 o  incline and then  slides a distance d. If the frictional force  on the block is 10 N. the reading of the spring balance  will be  (a)  24 N  (b)  74 N  (c)  15 N  (d)  49 N.0 × 10  N  (d)  1. If the lift moves downward with  an acceleration of  5 m/s 2 .0 cm  (c)  3.03  (c)  0. the force exerted by the rope on the block is  Pm  Pm  (a)  M + m (b)  M - m (c)  P  PM  (d)  M + m . The man holding it can exert a maximum force  of  144  N  on  the  gun.2 m/s 2  (b)  9. A man  hangs his bag on  the spring and  the spring reads 49 N.  Two masses m 1  = 5 kg and m 2 = 4.  (2003) .  The  coefficient  of  friction  between  10 N  the block and the wall is 0.5 cm  (b)  1.    F 3  are  acting  on  a  particle  of  mass  m  such  that  F 2  and  F 3  are  mutually  perpendicular.  A  light  string  passing  over  a  smooth  light  pulley  connects  two blocks of masses m 1 and m 2 (vertically). then  the ratio of the  masses is  (a)  8 : 1  (b)  9 : 7  (c)  4 : 3  (d)  5 : 3.  17.  11. These forces are represented in magnitude  and direction by the three sides of a triangle ABC (as shown).  29.  7.  (d)  (b)  (d)  (a)  (c)  .  10.  (2003)  26.  The  particle  will  now  move  with  velocity  C  r (a)  less than  v  r  (b)  greater than  v  r  (c)  | v  |  in  the  direction  of  the  largest  A B force  BC  r  (d)  v  . If the acceleration  of the system is g/8.  The  acceleration  of  the  ball  as  observed by the man in the lift and a man standing stationary  on  the  ground are respectively  (a)  g.  g.6  ms –2  on  a  frictionless  surface.  (2002)  29. g – a  (c)  g –  a.  One end of a  massless rope. remaining  unchanged.  13.  21.  16. A man in the lift  drops  a  ball  inside  the  lift.  24.  14.  28.  25.  18.  27.  8.  A lift is moving down with acceleration a.  (c)  (b)  (a)  (c)  (b)  5. g  (d)  a.  30.  23.  (b)  (d)  (a)  (a)  (d)  (b) 2.  The minimum velocity (in ms –1 ) with which a car driver must  traverse a flat curve of radius 150 m and coefficient of friction  0.  22.  15.8  (2002)  27.  which passes  over a  massless  and frictionless pulley P is tied to a hook C while the other  end is free.2  N  with  an  acceleration  of  0.  then  the  particle remains  stationary.  (a)  (b)  (a)  (d)  (a)  6.8  B  A  (c)  4  F  (d)  9.  P  C (a)  16  (b)  6  (c)  4  (d)  8.6 to avoid skidding is  (a)  60  (b)  30  (c)  15  (d)  25.  31.  g  (b)  g – a.2  (b)  7.  (b)  (d)  (c)  (a)  (b)  3.  (2002)  Answer  Key  1.11  Laws of Motion  With what value of maximum safe acceleration (in ms –2 ) can  a man  of 60 kg  climb on the rope?  25.  When forces  F 1.  26.  (2002)  31. Maximum tension that the rope can bear is 960 N.  (2002)      30.  20.   F 2.  (a)  (d)  (b)  (d)  (b)  4.  Three forces start acting simultaneously on a particle moving  r  with velocity  v  .  12.  Three  identical blocks  of masses m = 2 kg are drawn by a  force  F  =  10. If  the  force F 1  is now  removed  then  the acceleration of the  particle  is  (a)  F 1/m  (b)  F 2 F 3 /mF 1    (c)  (F 2  –  F 3)/m  (d)  F 2/m.  19. (2002)  28.  then  what  is  the  tension  (in  N)  in  the  string between the blocks B and  C?  C  (a)  9.  9. 9 m s -2  in vertical direction.  \  8.  mF  .  From graph.513 kg ;  v = 5..  m + M (d)  :  Work  done  by  hand  =  Potential  energy  of  the  ball .2 fx è 2 ø  u 2  =  8fx  From  (i)  and  (ii)  3 ×  (8  fx)  =  24  f  x  =  1  cm.  (b)  :  The  incline  is  given  an  acceleration  a.  (b):  F(t) =  F o e –bt  (Given)  ma =  F o e –bt  F  a =  o  e . 17.4 N s  Final momentum. component of g  down  the  incline  =  gsinf .  .2  (d)  :  Force  ×  time  =  Impulse  =  Change  of  momentum \  9.57.  (a) : Momentum is mv.bt  t  t  F  m + M Force = Impulse  3  = = 30 N ..  Pseudo acceleration  a  acts  on  block  to  the  left...  (b)  :  For first  part  of  penetration.  (b)  :  F  =  –  kx  F = -15 ´ æç 20  ö÷ = -3 N  è 100 ø  Initial  acceleration  is  over  come  by  retarding  force. u = = 1 m s -1  (2 - 0)  (0 .1 (d)  :  Retardation  due  to  friction  = mg  v 2  =  u 2  +  2as Q  \  0  =  (100) 2  –  2(mg)s  or  2 mgs  =  100  ×  100  or  s= 100 ´ 100  = 1000 m ..bt  m F  dv  F o  .  where q is the angle  of  inclination \  The  vertical  component  of  acceleration a  is  a (along vertical)  = (gsinq)sinq = gsin 2q  For block A  a A(along vertical)  =  gsin 2 60°  For block B  a B(along vertical)  =  gsin 2 30°  The  relative vertical acceleration of A with respect to B is  a AB(along vertical)  =  a A(along vertical)  –  a B(along  vertical)  = gsin 2 60° – gsin 2 30° æ 3 2  1  2 ö ÷ = g çè 2 2  ø  g = = 4.565 = 17.1 m s -1  (4 - 2)  Initial momentum.57 m s –1  Because  the values will be accurate up to second  decimel  place only.0)  Initial velocity.  2 ´ 0.. p i  =  mu = 0.8 N s  FS = mgh Þ  F  = ( ) ( )  mgh  0.4) N s = – 0.  (c) : Here. Because of impulse direction of velocity changes as  can be seen from the  slopes  of  the  graph.3  or  12.2 f (3)  ç 2 ÷ è ø  or  3u 2  =  24  f  For  latter  part  of  penetration.5 ´ 10  10.  2  æ u ö = ( u )2  .  or  m 0. Acceleration  of  the block  is to  the right.  . m =  0..  (b):  Centripetal acceleration.4 N s  Impulse = Change in momentum = p f  –  p i  = – 0. v = = .00 m/s \  mv = 17. mass of the  body.  (ii)  13.  s 0..  11..  time 0.4 × 1 = 0.4  × (–1) = – 0..bt ]  m ë - b û 0  mb 0 0  3.  or  m  ×  (acceleration  a)  =  3  3 3  a  = = = 10 ms -2 .  (i)  2  or  or  u  0 = æç ö÷ .  (a) :  For upper half smooth incline.12  JEE MAIN CHAPTERWISE EXPLORER  1.4 – (0.  by  equation  of  motion. we  get  v 5. \  macos a  =  mgsina  or  a  =  gtana.e .  (d) : Acceleration of the system  a  = Force on block of mass  m = ma  = 7.bt  F F  ò dv = ò m e dt  Þ  v = o éê e  ùú = o  [1 .  6.bt dt  dt m m Integrating both sides.2 ´ 10 ´ 2  = = 20 N.  (2 . Equate  resolved  parts of  a  and  g  along  incline.8 N s  |Impulse| = 0.  (a) : The acceleration of the body down the  smooth inclined  plane is a = gsinq  It is along the inclined plane.2)  Final velocity. p f  =  mv = 0.  m = 3.bt  =  e  or  dv =  o  e .. a c  = w 2 r  2 p Q  w = T As  T 1  = T 2 Þ w 1  = w 2  Q  a c 1  r 1  = ac 2  r2  2.4  kg  Since position­time (x­t) graph is a straight line.  F o  . so motion is  uniform.  2  4. 2  ×  10  =  2  N.  9.  Frictional  retardation  up  the  plane  = m k  (gcosq)  R  20.  r  Hence  the  particle  velocity  remains  unchanged  at  v  .2 9.m 2 )  (5 . Consequently the acceleration will be  zero.  v = mRg \  v = 0.8 9.8  = = =  \  W1  mg 9.  or  g g 10  or  d= P  ( M + m )  Force  on  block  a  = 4.  (c)  :  Component  of  g  down  the  plane  =  gsinq \  For  smooth  plane.  (a)  :  g = ( m + m ) = (5 + 4.2  17.  24.  acceleration  =  g.m 2 )  27.m k g cos q)( nt ) 2  2  \  Force applied  Total mass  MP  =  Mass  of  block  ×  a  = ( M + m )  .  (d)  :  Acceleration  of  block  ( a ) =  l  l  +  2g(sinf  – m k cosf)  2  2  or  19.8 ´ 0..5 4.6)  or  T BC  =  7.  frictional  f retardation  = m k gcosf \  Resultant  acceleration  = gsinf  – m kgcosf   l  2  \  0  =  v  +  2  (gsinf  – m k gcosf)  2  f  g g cosf  0  =  2(gsinf)  or  or or 0  =  sinf  +  sinf  – m k cosf  m k cosf  =  2sinf  m k  =  2tanf.m  )  k  2 2  or  22.  (d)  :  Suppose  he  can  fire  n  bullets  per  second \  Force  =  Change  in  momentum  per  second  144 = n ´ æç 40  ö÷ ´ (1200)  è 1000 ø  144 ´ 1000  n= or  40 ´ 1200  or  n  =  3..2  or  a = g´ = = 0..  29.  a  ( m1 .5 ×  10 5  N.06  .  16.  (a)  :  When  lift  is  standing..  21.  W 1  =  mg  When  the  lift  descends  with  acceleration  a.8  N.  1  ( g sin q .  f  =  mgsinq \  10 =  m  ×  10  ×  sin30°  or  m  =  2  kg.  14.2  –  (2  ×  2  ×  0.  W 2  =  m (g  –  a) W 2  m ( g .  (b)  :  g = ( m + m )  1 2  m 1  9  1  ( m1 .  m 30.8  25.8 49 ´ 4.mk  g cos q ) n 2t 2  2 2  or  sinq  =  n 2  (sinq  – m k cosq)  Putting q  =  45°  or  sin45°  =  n 2  (sin45°  – m kcos45°)    or  1  .a )  9.8) = 9.  (i)  2  For  rough  plane.  (c)  :  Frictional  force  provides  the  retarding  force \ m  mg  =  ma  a u / t  6 /10  m= = = = 0.8  18.  23.  (b)  :  Centripetal  force  on  particle  =  mRw 2 m k  = 1 -  \  F1 mR1w2  R 1  = = F2 mR2 w 2  R 2  . Obviously the resultant force on the  particle will be zero.  (a)  :  F 2  and  F 3  have  a  resultant  equivalent  to  F 1 F  \  Acceleration  =  1  .8  = =  24 N.5 ×  10 4 )  ×  (10)  =  3.  31.8  1 2  0.  (a)  :  Both  the  scales  read  M  kg  each.8 .  or  \  g sin q  mk g cos q  q q g g cosq  \  1 1  ( g sin q)t 2 = ( g sin q .6 ´ 150 ´ 10 = 30 m  .13 Laws of Motion  \  v 2  =  2(gsinf)  m k  g cos  f  R  l  2  g  sin f  For  lower  half  rough  incline.  (d) : By triangle of forces..  n 2  15.8)  0.8  W2 = W1  ´ 26.  9.  (b)  :  Q  Force  =  mass  ×  acceleration \  F  –  T AB  =  ma  and  T AB  –  T BC  =  ma \  T BC  =  F  –  2  ma  or  T BC  =  10.8 9. s  .4.  (d) :  Weight  of  the  block  is  balanced  by  force  of  friction \ Weight  of  the  block  = mR  =  0.  (b)  :  For  no  skidding  along  curved  track.  (a)  :  Initial  thrust  =  (Lift  ­  off  mass)  ×  acceleration  =  (3.  (c) :  For observer  in the  lift.8 9.  (b)  :  T  –  60g  =  60a  or  960  –  (60 ×  10)  =  60a  or  60a  =  360  or  a  =  6  ms –2 .  1  = n 2  (1 ..  R  mgsin q  q f  q  sq  mg mg co a  ( m1 .m 2 )  = =  .  (a)  :  For  equilibrium  of  block. acceleration  = (g  – a)  For  observer  standing  outside.  \  8 ( m1 + m2 )  or  m2  7  28.2 ms -2 . the particle will be in equilibrium  under the three forces..  1  d = ( g sin q ) t 2  .   choose  the  one  that  best  describes the two statements.  Statement­1 : A point particle of mass m moving with speed  v  collides  with  stationary  point  particle  of  mass  M. The energy loss during the collision  is  (a)  0.000 N/m. Statement­II is false. The spring compresses by  (a)  8. Then the velocity as a function of time and the height  as  function of time  will be  This  question has  Statement­I  and  Statement­II. will be more  than that on S 2 . Statement­II is a  correct  explanation  of  Statement­I. Statement 2 is true. Statement­II is true.  (a)  Statement­1 is  true.  WORK.  Statement­2 : Principle of conservation of momentum holds  7.  A block of mass 0.  (c)  Statement­I  is  true.  Statement­2 is true. Statement 2 is not  the  correct  explanation  of  Statement  1.  and  compresses  it  till  the  block  is  motionless.000 J  (b)  200 J ­ 500 J  (c)  2 × 10 5  J ­ 3 × 10 5  J  (d)  20. respectively.34 J  (b)  0.  (b)  Statement­1 is true.67 J. It strikes another mass of 1.00 J  (d)  0. His kinetic energy can be estimated to be in the range  (a)  2.  (c)  Statement 1 is true.  Of the  four  choices  given  after  the  Statements.  choose  the  one  that  best  describes the two Statements. ENERGY  AND POWER collision is negligible and the collision with the plate is totally  elastic.  (c)  Statement­1  is  true.  (2008)  Statement­1 : Two particles moving in the same direction do  not lose all their  energy  in  a  completely  inelastic  collision. Statement­II is true.000 J ­ 5.5 cm  (c)  2.  3.  Statement 2 : k 1  < k 2 .  (2012)  y  h  (a)  t  t  O  v  y  +v 1  h  t  (b)  O  t  –v 1  v  y  +v 1  (c)  h  t  O  t 1  2t  3t 1  4t  1  1  –v 1  t  y  h  (d)  t 1  2t 1  3t 1  4t 1  t  (2009)  t  5. Statement 2 is true.00 ms –1  on  a smooth surface.  Statement­II  is  true.    Statement 1 : If stretched by the same amount.  Statement­2  is  true;  Statement­2  is  8.  not  the  correct  explanation  of  Statement­1.50 kg is moving with a speed of 2. Statement 2 is the  correct explanation  of  Statement  1.  (d)  Statement 1 is false.16 J  (c)  1. Statement­2 is true; Statement­2 is the  correct  explanation  of  Statement­1.5 cm  (d)  11.  (2013)  This question has  Statement  1  and  Statement  2. work done on  S 1 .  2.  Statement 2 is true. Assume  that  the  duration  of  v  v 1  A particle is projected at  60° to the horizontal with a kinetic  energy K.  (2008)  6.  Statement 2 is false.  The  kinetic  friction  force  is  15  N  and  spring constant is 10.  4.  Statement­2 is false.  (d)  Statement­I is true.  are stretched by the same force.  An  athlete in the olympic games covers a distance of 100 m  in 10 s.14  JEE MAIN CHAPTERWISE EXPLORER  CHAPTER  4  1.  Of the four  choices  given  after  the  statements.0 cm  (2007)  . The kinetic energy at  the  highest point  is  (a)  K/2  (b)  K  (c)  zero  (d)  K/4  (2007)  A 2 kg block slides on a horizontal floor with a speed of 4 m/s. it is found that more work is  done on spring S 1  than on spring S 2.  (a)  Statement­I is false.  (2010)  Consider a rubber ball freely falling from a height h = 4.  (b)  Statement 1 is true.  Statement­II  is  not a  correct explanation  of  statement­I.  true for all kinds  of  collisions.  (a)  Statement 1 is true.000 J ­ 50.5 cm  (b)  5.00 kg and then they  move together as a single body.  If  the  maximum energy loss possible  is given as  æ 1  ö æ m  ö h è mv 2 ø then h = è 2  M + m ø Statement­2 : Maximum energy loss occurs when the particles  get stuck together as  a  result of the  collision.  (d)  Statement­1 is  false.  It  strikes  a  uncompressed  spring.  (b)  Statement­I is true.  If two springs S 1 and S 2  of force constants k 1 and k 2 .9 m  onto  a  horizontal  elastic  plate.000 J.   A force  F = (5iˆ + 3 ˆ  j + 2kˆ ) N  is applied over a particle which  r  displaces it from its origin to the point  r = (2iˆ .  the  maximum speed (in m/s) is  (a)  2  (b)  3/ 2  (c)  (d)  1/ 2  .  (2004)  21. The velocity of the 12 kg mass is  4 ms –1 . The kinetic energy of  the other  mass  is  (a)  96 J  (b)  144 J  (c)  288 J  (d)  192 J.  (2006)  12. The  (c)  work  done  on  the  particle  in  joule is  (a)  –7  (b)  +7  (c)  +10  (d)  +13.  (2003)  .x  ÷ J .5 J  (c)  –1.  (2006)  11.25 N­m.00 N­m  (d)  6. accelerates uniformly from rest to v 1  in  time t 1 .  (2004)  22.  A particle of mass 100 g is thrown  vertically upwards with  a  speed  of  5  m/s.  The  maximum momentum of  the  block  after collision  is  (a)  zero  (b)  ML 2  K  (c)  MK L  (d)  KL 2  2 M  (2005)  16.  A particle moves in a straight line with retardation proportional  to  its  displacement.15  Work.  It follows  that  (a)  its velocity is constant  (b)  its acceleration is constant  (c)  its kinetic energy  is constant  (d)  it moves in a straight line.75 N­m  (c)  25.  A  mass m moves with a  velocity  v  and  collides  v / 3  inelastically with another  after collision  identical  mass.  The potential energy of a 1 kg particle free to move along the  x­axis is given by  2  ö æ 4 V ( x ) = ç x .  2  ø  è 4 The  total  mechanical  energy  of  the  particle  2  J.  A  bomb  of mass  16  kg  at  rest  explodes  into  two  pieces of  masses of 4 kg and 12 kg. The instantaneous power delivered to the body as a  function of time t  is  (a)  mv1t  t 1  (b)  mv1 2t  t 1 2 mv1 t 2  mv1 2t  . The velocity attained by  the ball is  (a)  10 m/s  (b)  34 m/s  (c)  40 m/s  (d)  20 m/s  (2005)  17.  The  block  of  mass  M  moving on the frictionless  horizontal surface collides  M with  the  spring  of  spring  constant K and compresses  it  by  length  L.  the motion  of the particle takes  place in a plane.50 N­m  (b)  18.  (2004)  20.  A  particle  is  acted  upon  by  a  force  of  constant  magnitude  which is always perpendicular to the velocity of the particle.5 J  (b)  –0. The instantaneous power delivered to the  body as a  function  of  time  is given  by  1  mv 2  1  mv 2  2  t  t  (b)  (a)  2  T 2  2  T 2  mv 2  mv 2  2  × t  × t  (c)  (d)  (2005)  2  T T 2  14.  then climbs up another hill of height 30 m and finally rolls  down  to  a  horizontal  base  at  a  height  of  20  m  above  the  ground.  A spring of spring constant 5 × 10 3  N/m is stretched initially  by 5 cm from the unstretched position. Energy and Power  9.  (2004)  19.  The  work  done  by  the  force  of  gravity  during the time the  particle goes up is  (a)  0.  A body is moved along a straight line by a machine delivering  a  constant  power. The distance moved by the  body  in time  t  is  proportional  to  (a)  t 3/4  (b)  t 3/2  (c)  t 1/4  (d)  t 1/2 .  (2004)  (d)  t 1  t 1  r  18.  A spherical ball of mass 20 kg is stationary at the top of a hill  of height 100 m.  A  mass  of  M  kg  is  suspended  by  a  weightless  string.  A  body of  mass  m  is  accelerated  uniformly  from  rest to  a  speed v in a time T.  A  body of  mass  m.  (2006)  2  10.  2  (2006)  13.25 J  (d)  1.  (2003)  23. Find the speed  of the  2 nd  mass after  collision  2  v  v  (b)  (c)  v  (d)  3v  (a)  3  3  (2005)  15.  A uniform chain of length 2 m is kept on a table such that a  length of 60 cm hangs freely from the edge of the table. Then the work required  to  stretch  it  further  by  another  5  cm  is  (a)  12.25 J. It rolls down a smooth surface to the ground.  What  is  the work  done  in  pulling the  entire chain on the  table?  (a)  7.ˆ  j ) m .  Its  loss  of  kinetic  energy  for  any  displacement  x  is  proportional  to  (a)  x 2  (b)  e x  (c)  x  (d)  log e x.  The  horizontal force that is required to displace it until the string  making an angle of 45º with  the initial vertical direction is  (a)  Mg ( 2 - 1)  (b)  Mg ( 2 + 1)  (c)  Mg  2  (d)  Mg  .2 J  (b)  3.  After  collision  the  first  mass  m  moves with velocity in a  m  direction perpendicular to  before collision  the  initial  direction  of  motion.6 J  (c)  120 J  (d)  1200 J. The  total  mass  of the  chain  is  4  kg.  Then.   Linear  momentum  of  a  system  of  particles  is  zero.  (2002)  26.  26.  25.  14.  A ball whose kinetic energy is E.  Then  (a)  A does not imply B and B does not imply A  (b)  A implies B but B does not imply A  (c)  A does not imply B but  B implies  A  (d)  A implies B and B implies  A.  Kinetic  energy  of  a  system  of  particles  is  zero.  8.  (2003)  25.  (c)  (c)  (b)  (b)  5.  (d)  (c)  (b)  (b)  6. when water  is  cooled  to  form  ice.  (a)  (a)  (b)  (c)  .  9.16  JEE MAIN CHAPTERWISE EXPLORER  24.  If mass­energy equivalence is taken into account.  A.  (a)  (d)  (c)  (b)  (b)  2.  10.  20.  7.  16.  11.  B.  27. is projected at an angle of  45°  to  the  horizontal. The  work  done  in  extending  it  from 5  cm  to 15  cm  is  (a)  16 J  (b)  8 J  (c)  32 J  (d)  24 J.  the  mass  of  water  should  (a)  increase  (b)  remain unchanged  (c)  decrease  (d)  first increase then decrease.  (d)  (b)  (a)  (c)  (c)  3.  Consider  the  following  two  statements.  21.  23.  19.  17.  (2002)  Answer  Key  1.  The  kinetic  energy  of  the  ball  at  the  highest  point  of  its  flight  will  be  (a)  E  (b)  E  / 2 (c)  E/2  (d)  zero.  24.  18.  A  spring  of  force  constant  800  N/m  has  an  extension  of  5  cm.  13.  (b)  (b)  (c)  (a)  (c) 4.  22.  (2002)  27.  15.  12. = 1 ´ 1.00  (1.  2  Final K.17  Work.  2  .  It  is  fixed.  (b)  :  Total  energy E T  =  2  J. The change from  +v to –v is almost instantaneous. decreases.  Assuming an athelete has about 50 to 100 kg. v increases. Energy and Power  1.50 × (2.  kinetic  energy  is  maximum  The  potential  energy  should  therefore  be  minimum.  2  av  v  v  Hence. his  kinetic  Maximum energy loss = 1  mM  u 2  2 ( M + m )  \ f  = mM  ( M + m)  energy would have  been  1  2  mv  .  1. the value of v  becomes  +ve.  .(i)  t  (Using (i))  For 100 kg. F = k 1 x 1  =  k 2 x 2  Work done on spring  S 1  is  ( k x  ) 2  F 2  1  W1 = k1x 1 2  = 1 1  =  2 2 k1 2 k1  Work done on spring  S 2  is  2  (k x  ) 2  W2 = 1 k 2 x 2 2  = 2 2  =  F  2 2k 2 2 k2  W1 k 2  \ =  W2 k1  As W 1  > W 2 \  k 2  >  k 1  or k 1  < k 2  Statement 2 is true..(ii)  8.50)  v.  V ( x ) = .50) 6.00 2  = 1.1  J  4  (Kinetic  energy) max  =  E T  –  V min  or  1 9  (Kinetic  energy) max  = 2 – æç . Statement­1  is false. comes to zero and increases.. v = gt when coming  down.  For the same  extension.  (c) : v = u + gt.E.5 cm. (1/2) × 100 × 100 = 5000 J.33  2 3.  2.50 × 2.  V(x)  =  0  or  At  \  x  =  ±  1.ö÷ = J  è 4 ø  4 or  1 m v 2  = 9  m  2 4  or  v m 2  = 9 ´ 2 = 9 ´ 2 = 9  m ´ 4 1´ 4 2  \  vm  = 3  m/s. Therefore  (c).  Therefore  kinetic  energy  of  a  particle  at  highest  point  is  K  K H  = Kcos 2q = Kcos 2 60° =  4 .E.  9.  For  maximum  speed.  (b) : Let the spring be compressed by x  Initial kinetic energy of the mass = potential energy of   the  spring + work done due  to friction  1 1  ´ 2 ´ 4 2 = ´ 10000 ´ x 2  + 15 x 2 2  or  5000 x 2  + 15x – 16 = 0  or  x = 0.50  = v Initial K.00  0.  or  x  =  0.33 = 0.x = x ( x 2  - 1)  dx 4 2  dV = 0  For  V  to  be  minimum.00 = (1 + 0.  (a) : Loss of energy is maximum when collision is inelastic.  3. makes collision. (c) is correct.  (d) : The kinetic energy of a particle is K  At  highest  point  velocity  has  its  horizontal  component.  (b)  4.00) 2  = 1.00 J.  dx \  x(x 2  –  1)  =  0.  v average  = (v/2) = 10 m/s. = (1/2) × 0.  Statement­2  is  true.  (d)  : For the same force.  (Using (ii))  (Using (ii)  As k 1  < k 2 \  W 1  <  W 2  Statement 1 is false.00  –  0. x 1  =  x 2  =  x  Work done on spring  S 1  is  W1 = 1 k1 x12 =  1 k1 x 2  2 2  v/2  .67  J. Using –ve signs when coming  down.  (Using (i))  7. \  (v/2) =  10 m/s..  1  2  Further  h =  gt  is a  parabola. ± 1  At  x  =  0.  Q V ( x ) = x 4 x 2  4 2  dV 4 x 3  2 x  3 = = x .. As the ball is dropped.50 ´ 1.  (d) :  By the  law  of  conservation of  momentum  mu = (M + m)v  1.00  = 0.  \  Loss of energy =  1.  Work done on spring  S 2  is  W2 = 1 k2 x22 =  1 k 2 x 2  2 2  W1 k 1  \ =  W2 k2  time  (1/2)mv a2   v  = (1/2) × 50 × 100 = 2500 J.055 m = 5. t = 10 s.  2 5.  It could be  in the range  2000 to 5000 J.  (a) :  v = v / 2  is average  velocity  s = 100 m. 3  = 3.  (b)  :  The  centre  of  mass  of  the  hanging  part  is  at  0.  2  16....  (c)  :  Elastic  energy  stored  in  spring  = 1 KL2  2  \  kinetic  energy  of  block  E =  1 KL2  2  mass  of  hanging  part  = 4 ´ 0.18  JEE MAIN CHAPTERWISE EXPLORER  10.6  = 1.1  ÷ 2 ø è Fl Mgl ( 2 .2  ×  10 ×  0.  2 12..  Hence  kinetic  energy  of  the  particle  remains  constant.  (a) : Let v 1 = speed of second mass after collision Momentum  is  conserved  Along  X­axis.  (c) :  No work is done when a force of constant magnitude  always acts at right angles to the velocity of a particle when  the  motion  of  the  particle  takes  place  in  a  plane.  or  Mg  Work  done  = F ´ l sin 45 ° =  Fl  2  Gain  in  potential  energy  =  Mg(l  –  lcos45°)  æ ö = Mgl ç 1 .  20.  19.  Potential  energy  measures  the  workdone  against  the  force  of  gravity  during  rise.3  m  from  table  1.  mv 1sinq  ..4 m  0.  (b)  :  mgh  =  1 æ 100 ö 2  5 ´ 5  ´ ( 5)  = 2 ´ 10 = 1.  (b)  :  Work  done  = F × r or  work  done  = (5iˆ + 3 ˆj + 2kˆ ) × (2iˆ – ˆ  j )  or  work  done  =  10  –  3  =  7  J.  17...25 J  2 çè 1000 ÷ø Work  done  by  force  of  gravity  =  –  1.  (c) : Kinetic energy at projection point is converted into potential  energy of  the particle  during  rise.(i)    mv =  Along  Y­axis..25  J  1 2  7 =  2 mv  ∙ 5  = \  11..  è t1 ø è t 1  ø  t1  r r  18.  21.6 m  13.  (ii)    3  From  (i)  and  (ii) 2  2  æ mv ö 2  2  ( mv )  + \  (mv 1 cosq)  +  (mv 1 sinq)  =  ç 3  ÷ è ø  2 2  m 2 v1 2  =  4 m v  or  3  2  v1  =  v ...6 J.1)  =  \  2 2  or  F =  Mg ( 2 – 1) .  (b)  :  Acceleration  a  = \  l sin 45°  1 mv 2 æ1 + k 2  ö ç 2  ÷ 2  è R ø  \  5  5  =  1600  ×  7  7  v 1  t1  v 1  velocity  (v)  =  0 + at = t t  1  Power  P  =  Force  ×  velocity  =  m a v  2  æ v ö æ v t ö mv t  P = m ç 1 ÷ ´ ç 1 ÷ = 2 1  . 1  2  \  (–  work  done)  =  Kinetic  energy  =  2 mv  or  (–  work  done) Since  p 2  =  2ME \  2  p = 2 ME = 2 M ´ KL  =  MK L .  or  3  15.  (c)  :  Linear  momentum  is  conserved \  0  =  m 1 v 1  +  m 2 v 2  =  (12  ×  4)  +  (4  ×  v 2 )  or  4v 2  =  –  48 Þ  v 2  =  –  12  m/s 1  2  \  Kinetic  energy  of  mass  m2 =  m2 v2  2  1  = ´ 4 ´ (–12) 2  = 288 J.  (a) : Work done in displacement is equal to gain in potential  energy  of  mass  l  cos 45°  45°  l  100 m  30 m  20 m  \ F  1  2  æ 7  ö mv ç ÷ = mg ´ 80  2 è 5  ø  or  v 2  =  2  ×  10  ×  80  ×  or  v  =  34  m/s.  (a)  :  Given  :  Retardation µ  displacement  dv  = kx  or  dt æ dv öæ dx ö = kx  ç dx ÷ç dt ÷ or  è øè ø  or  dv (v)  =  kx  dx  v 2  or  x  ò vdv = k ò x dx v 1  0  .2 kg  2 \  W  =  mgh  =  1.  mv 1cosq  =  mv  .  (c)  :  Power  =  Force  ×  velocity  =  (ma)  (v) =  (ma)  (at)  =  ma 2 t  2  or  2 v mv  Power = m æç ö÷ (t ) = 2  t  è T  ø  T 14.. 05  k = ´ [0.05  0.05) 2 ù ë û  W  =  8  J.  1  2  27. By mass energy equivalent.  but  B  is  not  true. mass should decrease.(0.0075) = 18.  (b)  :  Power  =  Time = Time \  Force  ×  velocity  =  constant  (K)  or  (ma) (at)  =  K  1/ 2  K  a  = çæ ÷ö è mt ø  Q s = 1 at 2  2  or  1/ 2 1/ 2  \  1 K s = æç ö÷ 2 è mt ø 1  K  t 2 = æç ö÷ t 3/ 2  2 è m ø  or  s  is  proportional  to  t 3/2 .  u  1(a )  m  m  u  1(b )  1(a ) explodes  Total  momentum  =  0  ( )  1  2  But  total  kinetic  energy  = 2  2  mu  But  if  total  kinetic  energy  =  0.  (c) :  When water is cooled to form ice.15  W = ò  800 xdx = x 2  ë û0.  or  22.19  Work.  26.  x 2  0.05  25.75 N­ m .  Here  A  is  true.0025]  2  (5 ´ 103 )  Workdone  = ´ (0.  (c)  :  Kinetic  energy  point  of  projection  ( E ) =  2  mu At  highest  point  velocity  =  u  cosq \  Kinetic  energy  at  highest  point  1  = m (u cos q ) 2  2  1  2 E = mu cos2  45 °  = .  A  does  not  imply  B.  (c) :  A  system  of  particles  implies  that  one  is  discussing  total  momentum  and  total  energy. its thermal energy  decreases.  2 Work Force× distance  = Force× velocity  23.  but  B  implies  A.15)2 – (0.01 - 0.  (b)  :  Force  constant  of  spring  (k)  =  F/x  or  F  =  kx \  dW =  kxdx  or  or  0. 2  2  .  24.05)  ùû 0.1  =  2 2 2  mv22 mv 1 2  mkx 2  =  or  2 2 2  mk  2  ( K 2 – K1 ) =  x or  2  or    Loss  of  kinetic  energy  is  proportional  to  x 2 .1  k  2 2  ò dW = ò  kx dx = 2 éë (0.15  0.05  = 400 é (0.15  x 1  0.1) . Energy and Power  v 22 v 2  k x 2  .  velocities  are  zero.  (b)  :  W = ò Fdx = ò kx dx \  or  800 é 2 ù0.   A mass m hangs with the help of a string wrapped around a  pulley  on a frictionless  bearing.  is  g  3  2  (b)  g  (c)  g  (d)  (a)  g  2  3  3  (2011)  8.  the  number  of  rotations  made  by  the  pulley before  its direction  of motion  if  reversed. Its maximum  angular  speed  is w. An insect is at rest at a point near  the rim of the disc. What will be the velocity of  the  centre  of the hoop  when  it ceases  to  slip?  rw 0  rw 0  rw 0  (a)  rw 0  (b)  (c)  (d)  4  3  2  (2013)  x as k(x/L) n  where n can be zero or any positive number. During the journey of the insect. If the moment of inertia of the pulley about its axis  of  rotation  is 10  kg  m 2 . Then its acceleration is  .  2. whose centre is  O.  the angular speed of  the disc  (a)  remains unchanged  (b)  continuously  decreases  (c)  continuously  increases  (d)  first increases and  then decreases  (2011)  9. The insect now moves along a diameter of  the disc to reach its other end. Assuming  pulley  to  be  a  perfect  uniform  circular  disc.  which  of  the  following  graphs  best  approximates  the  dependence  of  x CM  on  n?  A  pulley  of  radius  2  m  is  rotated  about  its  axis  by  a  force  F = (20t – 5t 2 ) newton (where t is measured in seconds) applied  tangentially.  is  (a)  less than 3  (b)  more than 3 but less than 6  (c)  more than 6 but less than 9  (d)  more than 9  (2011)  (a)  7.  4.  A thin uniform rod of length l and mass m is swinging freely  about a horizontal axis passing through its end. The  pulley  has  mass  m  and  radius  R.  A thin horizontal circular disc is rotating about a vertical axis  passing through its centre.  6.20  JEE MAIN CHAPTERWISE EXPLORER  CHAPTER  5  1.  ROTATIONAL MOTION A hoop of radius r and mass m rotating with an angular velocity w 0  is placed on a rough horizontal surface.  The  moment of inertia of this plate about an axis perpendicular to  its plane and passing through one of its corners is  A thin rod of length L is lying along the x­axis with its ends  at x = 0 and x = L. If the  position x CM  of the centre of mass of the rod is plotted against  n. Its linear density (mass/length) varies with  x CM  x CM  L  L  2  O  (c)  (b)  n  L  L  2  O  x CM  x CM  L  2  L  L  2  (d)  O  O  (2008)  (a)  2  2  5  2  ma  (b)  ma  3  6  (c)  1  ma 2  12  (d)  7  ma 2  12  (2008)  For the given uniform square lamina ABCD.  Its  centre  of  mass  rises  to  a  maximum  height  of  1  l w  1 l 2w 2  1 l 2w 2  1 l 2w 2  (a)  3  g (b)  6  g (c)  2  g (d)  6  g (2009)  10. (a)  I AC =  2 I EF  (b)  2 I AC =  I EF  F  O  (c)  I AD = 3 I EF  (d)  I AC  =  I EF  C  D  A  E  B  (2007)  Angular momentum of the particle rotating with a central force  is constant due to  (a)  constant  torque  (b)  constant  force  (c)  constant linear momentum  (d)  zero  torque  (2007)  A  round  uniform  body  of  radius  R.  3.  Consider a  uniform  square  plate  of  side  a  and mass  m. The initial velocity  of the centre of the hoop is zero. if the string does not slip  on  the pulley.  5. the acceleration of the mass m.  mass  M  and  moment  of  inertia I rolls down (without slipping) an inclined plane making  an angle q with the horizontal.  The ring  now  rotates  with an  angular  velocity w¢  =  w m  w(m + 2 M )  (a)  ( m + 2 M )  (b)  m w( m .  A  particle  of  mass  m  moves along line PC with  velocity v as shown.  4  3  3  (a)  l  (b)  l  (c)  l  (d)  l  3  4  2  (2005)  17.  Let  F be  the r force  acting  on  a  particle  having  position  r  vector r and  T be the torque of this force about the origin.  the  origin  of  the  coordinate  system. The torque  about the point (1.  A T shaped object with dimensions  B  shown  in the figure.  If  its  angular  frequency  is  doubled  and  its  kinetic energy halved.  1 2  1  2  (2006)  l  16.  Their  moment  of  inertia  about  their diameters are respectively I A  and I B  such that  (a)  I A  = I B  (b)  I A  > I B  (c)  I A  < I B  (d)  I A /I B  =  d A /d B  where d A  and d B  are their densities.  The moment of inertia of a uniform semicircular disc of mass  M and radius r about a line perpendicular to the plane of the  disc through the center is  1  2  1  2  2  2  Mr  (a)  Mr 2  (b)  (c)  Mr  (d)  Mr 2  4  5  (2005)  19. and another disc Y of radius 4R is made from an  iron  plate  of  thickness  t/4. A force  F  is applied  at the point P parallel to AB. What  is the angular momentum  of the  particle about  P?  (a)  mvL  (b)  mvl  C  L P l r O (c)  mvr  (d)  zero.F (iˆ +  ˆ  j )  (d)  F (iˆ +  ˆ  j ) .  Then  the  relation  between  the  moment of inertia I X  and I Y  is  (a)  I Y  = 32I X  (b)  I Y  = 16I X  (c)  I Y  = I X  (d)  I Y  = 64I X . The centre of mass of the new disc is a/R from the  centre  of the bigger  disc.  A  force  of  - Fkˆ  acts  on  O.  One solid sphere A and another hollow sphere B are of same  mass  and  same  outer  radii.  O  x  y  (2006)  15. The center of  mass of  bodies B  3  and C taken together shifts compared to that of body A towards  (a)  body C  (b)  body B  (c)  depends on height of breaking  (d)  does not  shift  (2005)  18.  Then  r r r r r r (a)  rr × T = 0 and F × T ¹ 0  (b)  rr × T ¹ 0 and F × T = 0  r r r r r r r  (c)  rr × T ¹ 0 and F × T ¹ 0  (d)  r × T = 0  and  F × T = 0  (2003)  22.  A  body  A  of  mass  M  while  falling  vertically  downwards  1  under gravity breaks into two parts; a body B of mass  M  3  2  and body C  of mass  M  . is  lying on  a  A  r  smooth floor. so as to  keep the  centre of mass  at  the  same position?  m 2  m 1  m 1  (a)  d  (b)  m  d  (c)  m + m d  (d)  m  d  .  (2006)  13.  by  what  distance  should the  second particle be  moved.  A circular disc of radius R is removed from a bigger circular  disc of radius 2R such that  the circumferences  of the discs  coincide. The  moment of  inertia  of this  system about an axis through A and parallel to BD  is  (a)  ml 2  (b)  2ml 2  (c)  3 ml 2  (d)  3ml 2 .  Consider a two particle system with particles having masses  m 1  and m 2. If the radius of  the  sphere  is  increased  keeping  mass  same  which  one  of  the  following  will  not  be  affected?  (a)  moment  of  inertia  (b)  angular momentum  (c)  angular velocity  (d)  rotational  kinetic  energy.  A solid sphere is rotating in free space.  14.  A circular disc X of radius R is made from an iron plate  of  thickness t.  (2002)  .  (2003)  23. then the  new  angular momentum is  (a)  L /4  (b)  2L  (c)  4L  (d)  L /2. If the first particle is pushed towards the centre    of  mass through  a  distance d.  Four point masses.  A  particle  performing uniform circular motion  has  angular  momentum  L.  (2003)  24. Two objects each  of  mass  M  are  attached  gently  to  the  opposite  ends  of  a  diameter of the  ring. The  value  of  a  is  (a)  1/4  (b)  1/3  (c)  1/2  (d)  1/6. such  P  r  that  the  object  has  only  the  2 l  F  translational  motion  without  rotation.  A thin circular ring of mass m and radius R is rotating about  its axis with a constant angular velocity w. are placed at the corners  of  a  square ABCD  of  side  l.F (iˆ -  ˆ  j )  (b)  F (iˆ -  ˆ  j )  (c)  .21  Rotational Motion  (a)  g sin q 1 - MR 2  / I (b)  g sin q 1 + I / MR 2  (c)  g sin q 1 + MR 2  / I (d)  g sin q 1 - I / MR 2  (2007)  11. each of value m.2 M )  wm  (c)  (m + 2 M )  (2006)  (d)  (m + M ) .  –1)  is  z  (a)  .  (2007)  12.  (2004)  20. Find the location of P with  C respect to C.  (2004)  r  21.   9. The velocity  of centre  of  mass  is  (a)  v  (b)  v/3  (c)  v/2  (d)  zero.  (2002)  Answer  Key  1.  17.  (d)  (*)  (d)  (d)  6.  26.  28.  21.  7.  18.  A solid sphere.  15.  (b)  (d)  (d)  (b)  (d)  3.  16.  (d)  (a)  (a)  (c)  (a)  2.  23.22  JEE MAIN CHAPTERWISE EXPLORER  25.  (c)  (d)  (d)  (d)  (c)  4.  19.  22.  20. Then maximum acceleration down the plane is for  (no rolling)  (a)  solid sphere  (b)  hollow  sphere  (c)  ring  (d)  all same.  Initial  angular  velocity  of a  circular disc  of mass  M  is w 1 .  24.  Two identical particles move towards each other with velocity  2v  and v  respectively.  25. What  is  the  final angular velocity of the disc?  M + m  w  M + m  w  (b) (a) 1  1  M m M  M  w .  (2002)  27.  10.  12.  (b)  (d)  (b)  (d)  .  (d) (2002)  (c) M + 4 m w1  M + 2 m 1  ( ( )  )  ( ( )  )  28.  27.  Then two small spheres of mass m are attached gently to two  diametrically opposite points on the edge of the disc.  14.  13.  8.  Moment of inertia of a circular wire of mass M and radius R  about its diameter is  (a)  MR 2 /2  (b)  MR 2  2  (c)  2MR  (d)  MR 2 /4.  (2002)  26.  (d)  (b)  (a)  (a)  (c) 5.  11. a hollow sphere and a ring are released from  top of an inclined plane (frictionless) so that they slide down  the plane.   6 g × dx ÷öx  ø n  × dx  0  3 2pn = 36 ò çè L n  × x ò Ln  × x t  d q = 2 t -  dt 3  2  ( )  1  l  m × w 2 .  When the centre of mass is raised through  h.  (b) : Torque  exerted  on  pulley t  =  FR \ a= (For circular  disc)  2  mR a \ T =  2  Þ x C.  R  =  2 m.  w = 2 t 2  -  t  3  At t = 6 s.e. the increase in potential energy is mgh. As the  density increases with length.(n + 1) ïü = L  í ý 2 dn  ( n + 2) ( n + 2) 2  ï îï þ  T  mg  m  If the rod has the same density as at x = 0 i. F = (20t – 5t 2 ).23  Rotational Motion  a =  1. the centre of mass would have been at L/2.M  = )  0  L  L  òx 4  2 t t On integration. therefore  uniform. \  Angular  acceleration of  disc  7. n = 0. pulley  to  be  consider  as a  circular disc.(i)  m As per question.  (d)  5..  a =  m 2. perpendicular to the  plate is  ..T  \ a =  . q = 36 rad  Þ  n  k  l 2 × w 2  \ h =  .  (c) : The  free  body  diagram  of  pulley  and  mass  L( n + 1)  (n + 2)  The variation of the centre of mass with  x  is  given  by  R  dx = L ïì (n + 2)1 .  q = -  3 12  At.5t 2 ) ´ 2  10  a = (4t –  t 2 )  (Using (ii))  mg .  2 3  L  dq t 3  = 2 t 2  dt 3  3  2h 2  On integrating.  (d):  t I . Therefore  it  can only  be  (b). the centre of mass shifts towards  the  right.M  = n +1  0  L ò x dx  n  dx n + 2  ( n + 1)  = L  × n +1  n + 2  L  0  n = 36  < 6  2 p  Þ x C.  (d) : The  uniform rod of length  l  and mass  m  is  swinging  about an  axis passing through  the end...  (b) :  x C.(ii)  Here.  a  mg –  T =  ma  mg  mg . t = 6 s.  I  = 10  kg  m 2  )  or 4.  mr 2w 0 = mvr +  mr 2w  ( )  = mvr + mr 2 v  r  mr 2w 0 = mvr + mvr  mr 2w 0 = 2mvr  rw or  v =  0  2  ( or  a = FR  Q  a = t I I Here..  2  dw = (4t –  t 2 )dt  Þ mgh = 3  æ k  ( 6.  q  This is equal to the  kinetic  energy  h d w = (4t - t 2 ) dt  dt = 1  I w 2 .M  = 3.mR a 2  Therefore. w = 0  w= (Using (i)) (Q  a = Ra )  ma = mg . t  =  T ×  R  and  I =  1 mR 2  According  to  law  of  conservation  at  point  of  contact.ma 2  2 g  \ a =  3  (20t .  (a) : For a rectangular sheet moment of inertia passing through  O.   (b)  :  I =  (Mass of semicircular disc) ´ r 2  2  2  or  I =  Mr  .  m + 2 M r  r 14.F ) = F (iˆ +  ˆ  = iF j ).  or  I z  Ma 2  =  2 6  2  I  By the same theorem  I EF  = z  = Ma  2 6  \  I AC  = I EF .  The centre of mass is at R/3 to the left on the diameter of the  original disc. The  force  should  act  at  the  centre  of  mass  2R  M ¢  O ¢ x  O  m  R  2  3 pR s × x + pR s × R  = 0  M Because for the full disc.  18.  I A  =  2 MR 2  5  2  2  For  hollow  sphere.  MR 11.  The  question  should  be  at  a  distance aR  and  not a/R.  None of the  option  is  correct.  for square plate it is  6  a 2 + a 2 = a . R  x = -  Þ  = aR.  2  r I AC + I BD = I z Þ I AC  = a = O  13. 9.  or  m 1 d  =  m 2 x  or  m2  16.  12 12 3  By  perpendicular axes  theorem.  (*) :(M¢ + m) = M = p (2R) 2×s  where s = mass per unit area  m = sR 2×s.  2  19.  the  angular  momentum of a particle is constant.  (a) : It is a case of translation motion without rotation.  I EF  = M B  45°  l  2  or  I  about  B parallel  to the  axis through  O is  2 axis  A 1  l  \ AO ´ =  2  2  l  AO =  or  2  I  =  I D  +  I B  +  I C  a /2  B A  a/2  r= 12.  If  no  external  torque  is  acting  on  a  particle. r = iˆ –  ˆ  j ˆ  iˆ ˆ  j k  r  r \ r ´ F  = 1 -1 0  0 0  .F ˆ – ˆj ( .  (d)  :  Torque  t = r ´ F r r  F = .  \ r 2  =  a 2  4 4 2  2  \  D  r C  2 2  I o  + Md 2  = Ma + Ma =  4 Ma  6 2 6  2  2  I =  Ma 3  8.  (d) : The centre of mass of bodies B and  C taken together  does not  shift as  no external force is  applied horizontally.  or  2 2  a 2 + b 2 M ( a + a  )  2 a 2  = =  M 12 12 12  M (2 a 2 ) M (2 a 2 )  Ma 2  + =  .Fkˆ .  m + 2 m  3 17.  æ 2 l  ö 2 ml 2  + mç ÷ 2  è 2  ø  2 2  I = 2 ml +  4 ml  2 2  I= (d) : Central forces passes through axis of rotation so torque  is  zero. .  (d) : Let m 2  be moved by x so as to keep the centre of mass  at  the  same  position \  m 1 d  +  m 2  (–x)  =  0  m  x =  1  d . the centre of mass is at the centre O.  I B  = 3 MR I A  2 MR 2  3 3  = ´ =  \  I B  5 2 MR 2  5  or  I A  <  I B .  (b) : Acceleration of a uniform body of radius R and mass M  and  moment  of  inertia  I  rolls  down  (without  slipping)  an  inclined plane making an angle q with the horizontal is given  by  g sin q 1 +  I  2  .  (a)  :  Angular  momentum  is  conserved \  L 1  =  L 2 \  mR 2 w =  (mR 2  + 2 MR 2 ) w¢ = R 2  (m + 2M) w¢  I z  =   2  I= (d) : By perpendicular  axes theorem. M¢ = 3pR 2s  2 C  l / 2  l  w¢ = m w .24  JEE MAIN CHAPTERWISE EXPLORER  2 2  I 0  = M çæ a + b  ÷ö è 12  ø  a  O  b  Ma 2 .  10.  (c) :  For  solid  sphere.  3  \ | a | =  D  6 ml 2  = 3 ml 2 .  Y cm  = ( m ´ 2l ) + (2 m ´ l )  4 l  = .  (d)  :  AO cos 45 ° =  l  2  -1  3 .  15.   M + 4 m m1v1 + m2 v 2  m1 + m2  v c  = m (2v ) + m ( .  IY  = or  I Y  4  ( pR t s ) R  pR st  MR = =  2 2 2  2 2  (Mass)(4 R ) p(4 R )  t  = s ´ 16 R 2  2 2 4  =  32pR 4 ts \  I X  pR 4 st  1 = ´ = 1  I Y  2 32 pR 4 st 64  I Y  =  64  I X .25  Rotational Motion  20.  (d)  :  Q T = r ´ F r r r r  r \  r × T = r × ( r ´ F ) = 0  r r r r  r Also F × T = F × ( r ´ F ) = 0 .I.  (d)  :  The  bodies  slide  along  inclined  plane.  (b) : Free space implies that no external torque is operating  on the sphere. about  its  diameter  =  MR .  (c)  :  Angular  momentum  of  the  system  is  conserved 2  23. Here the law of conservation of angular  momentum  applies  to  the  system.  25.  Hence  angular  momentum  is  zero.  r r r 21.  (a)  : Angular momentum  L  =  Iw  1  2  Rotational  kinetic  energy  ( K ) = I w 2  L Iw´ 2 2 2 K  = = Þ  L = \  K  w w I w2  or  L1 K 1 w2  = ´ = 2 ´ 2 = 4  L2 K 2 w1  \  L  L2  = 1  =  L  .  24.  4 4  \  I X  = 2 2  Similarly. Acceleration  for  each  body  down  the  plane  =  gsinq.v )  v  = .  It  is  the  same  for  each  body.  22.  (c)  :  v c  = or  M w1  . The  line  of  motion  is  through  P. Internal changes are responsible for increase  in radius of sphere.  (a)  :  A  circular  wire  behaves  like  a  ring  2  M.  They  do  not  roll.  27.  (d)  :  Mass  of  disc  X = ( pR t ) s  where s  =  density 2 \  \  1 1  MR 2 w1  = 2 mR 2 w + MR 2 w  2 2  or  Mw 1  =  (4m  +  M)w  or  w= 28.  (d)  :  The  particle  moves  with  linear  velocity v  along  line  PC.  m+m 2  .  2  26.  Of the four  choices  given. When both d and h are much smaller than the radius of  earth.4 × 10 8  Joules  (b)  6.67 ´ 10 –9  J  (b)  6.11 km s –1  (b)  1.  7. statement­2 is false.  R 2  R (a)  Statement­1 is  true.  statement­2  is  true;  earth is  statement­2  is  not  a  correct  explanation  for  1  1  statement­1.  (2008)  (2004)  . statement­2 is true.67 ´ 10 –10  J  –10  (c)  13. Find the work to be  done against the gravitational force between them to take the  particle far away from the  sphere.  (you may take G = 6.  the flux  of gravitational  field  passing  through  its sides  is  12.  independent  of  Statement­2 : If the direction of a field due to a point source  (a)  the mass of the satellite  is radial and its dependence on the distance r from the source  (b)  radius  of its orbit  2  is given as 1/r  .1 km s –1  –1  (c)  11 km s  (d)  110 km s –1  (2008)  Average density  of the earth  (a)  is  directly proportional to  g  (b)  is  inversely proportional  to g  (c)  does not depend on  g  (d)  is  a  complex  function of g  (2005)  A  particle  of  mass  10 g is  kept  on the surface  of  a  uniform  sphere of mass 100 kg and radius 10 cm.  The  time  period  of  an  earth  satellite  in  circular  orbit  is  4pGM.  (a) (d) ( 2  )  .  The mass of a spaceship is 1000 kg.  the radius  of  the earth  is  R  (a)  2R  (b)  (c)  R/2  (d)  2R  2  (2009)  10.  mass m from the surface of a planet of mass M and radius R  in a  circular orbit at  an altitude  of 2R?  (a)  GmM  3 R  (b)  5 GmM  2 GmM  (c)  6 R  3 R  (d)  GmM  2 R  (2013)  2.  3. the escape velocity  from the surface of  the  planet would be  (a)  0.67 ´ 10 –11  Nm 2 /kg 2 )  (a)  6. Then the time period of a planet in circular orbit  of radius R around  the sun  will be proportional  to  Directions : The following question contains statement­1 and  statement­2.  mgR  (c)  mgR  (a)  2 mgR  (b)  (d)  mgR.  then which of  the following  is correct?  (a)  d = 2h  (b)  d = h  (c)  d = h/2  (d)  d = 3h/2  (2005)  Suppose the gravitational force varies inversely as the n th power  of distance.  2  4  Statement­1 : For a mass M kept at the centre of a cube of side  (2004)  a. The  required  energy for  this  work will  be  (a)  6.33 ´ 10 –10  J  (2005)  The change in the value of g at  a height  h above the surface  of the earth is the same as at a  depth d below  the  surface  of  earth.  (2004)  (b)  Statement­1 is  false. Given that the  escape velocity from the earth is 11 km s –1 . the  gain in the potential energy of an object of mass m raised from  correct  explanation  for  statement­1.4 × 10  Joules  (d)  6. The  gravitational potential at a point on the line joining them where  the  gravitational field  is  zero  is  (a)  zero  4.  A planet in a distant solar system is 10 times more  massive  than the earth and its radius is 10 times smaller. statement­2 is  true; statement­2 is a  11.  10  (c)  6. The value  of g and R  (radius of  earth)  are  10  m/s 2  and  6400  km  respectively. its flux through a closed surface depends only  (c)  both the mass and radius of the orbit  on the strength  of the source enclosed by the  surface and  not  (d)  neither the mass of the satellite nor the radius of its orbit.  (c)  Statement­1 is  true.4 × 10 9  Joules  8.  the surface of the earth to a height equal to the radius R of the  (d)  Statement­1  is  true.  What is the minimum energy required to launch a satellite of  6.  on the size or  shape of  the  surface.  4Gm  (b)  -  r 6Gm  (c)  -  r 9Gm  (d)  -  r (2011)  9.34 ´ 10  J  (d)  3. It is to be launched from  the  earth’s surface out  into free  space.  5.  If g is the acceleration due to gravity on the earth’s surface.  choose the  one  that  n - 2  ( n + 1 )  (b) R( n 2 - 1)  (c)  R n  best describes the two statements.  The  height at  which the  acceleration due  to  gravity  becomes  g/9 (where g = the  acceleration due to  gravity  on the surface  of  the  earth) in terms of R.4 × 10 11  Joules  (2012)  Two bodies of masses m and 4m are placed at a distance r.26  JEE MAIN CHAPTERWISE EXPLORER  CHAPTER  GRAVITATION 6  1.   (b)  (a)  (d)  (d)  2.5R.  Energy required to move a body of mass m from an orbit of  radius 2R to 3R is  (a)  GMm/12R 2  (b)  GMm/3R 2  (c)  GMm/8R  (d)  GMm/6R.  19.  then  the  satellite will  (a)  continue to move in  its  orbit with same velocity  (b)  move tangentially to the original orbit in the same velocity  (c)  become stationary in its orbit  (d)  move towards the earth.  the  orbital  speed of  the  satellite is  gR  (a)  gx  (b)  R - x 16.  14.  (a)  10.  (2003)  (2004)  17.  (c)  .  (2002)  (c)  2  ö1/ 2  æ gR  (d)  ç R + x ÷ è ø  gR 2  R +  x . the  new  time  period will become  (a)  10 hour  (b)  80 hour  (c)  40 hour  (d)  20 hour. the escape velocity will be  18.  (c)  (b)  (c)  (b) 3.27  Gravitation  13.  If suddenly the gravitational force of attraction between Earth  and a  satellite    revolving  around it  becomes zero.5R  (d)  1.  A satellite of mass  m revolves around the earth of radius R  at a height x from its surface.  Two spherical bodies of mass M and 5M and radii R and 2R  respectively are released in free space with initial separation  between their centres equal to 12R.  (d)  9.5R  (c)  7.  (d)  12. If the separation  between the earth and the satellite is increased to 4 times the  previous value.  The escape  velocity  for  a  body  projected  vertically  upwards  from the surface of earth is 11 km/s.  The kinetic energy needed to project a body of mass m from  the earth surface (radius  R) to infinityis  (a)  mgR/2  (b)  2mgR  (c)  mgR  (d)  mgR/4. If the body is projected  at an angle of 45° with the vertical.  (a)  11 2  km/s  (b)  22 km/s  (c)  11 km/s  (d)  11 / 2  m/s.  (2002)  Answer  Key  1.  8.  (2002)  14.  (2003)  15.  (c)  5.  (2002)  20.  (b)  17.  (c)  4.  The time period of a satellite of earth is 5 hour.  7. (2003)  19.  13.  (a)  18. then the distance covered by  the  smaller body just before  collision  is  (a)  2.  (a)  6.  (a)  15.  The  escape velocity of a  body depends upon mass  as  (a)  m 0  (b)  m 1  (c)  m 2  (d)  m 3 .  (c)  11.  20.5R  (b)  4. If they attract each other  due to gravitational force only.  (a)  16. If g is the acceleration due to  gravity on  the surface  of  the  earth.   \ v e  = 2GM  ´ 10  = 10 ´ 11 = 110 km s -1  R /10  7.  (d) :  v escape  =  )  If v is the velocity of the satellite at a distance 2R from the  surface of  the  planet.  has  B A  to pass  through  B. area of the Gaussian surface is 4pr 2 .  GM = GM  × 1  r2 R 2  9  r  \  r = 3R  The height above the ground  is 2R.  Assuming  the  mass is a point mass.  6.GMm = -  GMm  R R ( 5.  \  Workdone = .  all the lines passing through A are passing through B.  (d) : Let x  be  the  distance of  the  point  P  from the  mass  m  where  gravitational  field  is  zero..67 ´ 10 –11 ) ´ (100) ´ (10 ´ 10 –3 )  10 ´ 10 –2  –10  =  6.28  JEE MAIN CHAPTERWISE EXPLORER  1.  It  is  only  for  M  Gaussian surface.  r  Flux f g  =| g × 4pr 2  |  =  r 2  Here B is a cube. whether  B  is  a  cube  or  any  surface.  (a) : The  acceleration  due  to  gravity  at  a  height  h  from  M the  ground is given as  g/9.x )  G (4m )  Gm =r r -  r  3 3  3G (4m )  Gm = - 9  = .  (b) : Energy of  the  satellite  on the  surface  of  the  planet is E i  = KE + PE = 0 + .x ) 2  or  x =  r  3  Gravitational potential  at a  point  P is  Gm  G (4m )  =x (r .  Statement­2 is correct because when the shape of the earth  is spherical.- GMm  6 R R GMm GMm 5  GMm  =+ =  6R R 6 R = ( 2.E i  = . This ensures  inverse square  law.  .  r  g . r r  Q  Flux f = E × 4 pr 2  = e0  Every line passing through A.  then  total  energy  of  the  satellite  is 1  GMm  E f  = mv 2  + 2 ( R +  2 R)  ( )  2  = 1  æ GM ö GMm  m  2 çè ( R +  2 R ) ÷ø 3 R 1  GMm GMm GMm  =2 3R 3R 6 R \  Minimum energy required  to launch  the satellite  is DE = E f ..GM  r 2  4 pr 2  × GM  = 4 p GM . whatever be the shape.  2  Gm  G (4 m )  x  = or  = 1  ( r -  x ) 4  x2 (r .  (b)  :  Gravitational  force F  = \  dW = FdR = Gm1m 2  R 2  G m1m 2  dR  R 2  ¥ W  ¥ Gm1m 2  dR  é 1 ù \  ò dW = Gm1m2 ò 2  = Gm1m 2  ê ..  (c)  :  Let  A  be  the  Gaussian  surface  enclosing  a  spherical  charge  Q..(i)  g  =  (Using (i))  4.R ú = ë û R  R 0  R  R (6.  r  B  Q  E × 4 pr 2  = A  e0  Q +  r  Q  r  E  = 2  4 pe0 .gravitational field  = .GMm .. As explained earlier.  (i)  GM  R 2  r 3M R 2  3  3  ´ = r= g  \  g = 3  GM 4  p RG  or  4 pRG 4 p R \  Average  density  is  directly  proportional  to  g.4 × 10 10  J  3.  2 GM  for the earth  R 8. the lines of field  r  should  be  normal.  (ii) .  v e  = 11 kms –1  Mass of the planet = 10  M e .  (c) :  Energy required  = = gR 2  ´ )  GMm  R (Q  g = GM  R )  m R  2  = mgR  = 1000 × 10 × 6400 × 10 3  = 64 × 10 9  J =  6. Radius of the planet = R/10. although  all the lines are not normal.3 Gm  -  r r 2 r ( \ () ( )  )  Mass of earth  Volume of earth  M 3 M  r= = (4 / 3) pR 3 4 p R 3  (a)  :  Density =  ..67  ×  10  J.   the  satellite  has  the  tangential  velocity and  it  will  move  in  a  straight  line.  2 h ö æ (a)  :  At  height.  (a)  :  For  motion  of  a  planet  in  circular  orbit.E.  Compare  Lorentzian  force  on  charges  in  the  cyclotron.  From equation of motion.  (d) :  For  a  satellite  centripetal  force  =  Gravitational  force mv 0 2  GMm  \  ( R + x )  = ( R + x ) 2  gR 2  GM  2  = or  v 0  = ( R + x ) ( R +  x )  G ´ 5 M  (12 R - x ) 2  GM  Acceleration of bigger body.  a1 ( x ) = 2 é GM  ù êëQ  g  = R 2  úû .E.E..  13.  17..  æ GM  ö 1  1 = mR ç 2  ÷ = mgR  2 è R ø  2  14.  (c) : Let the spheres collide after time t.R ÷ where  h  <<  R  è ø  2 hg  2 hg  or  g – g h  =  or  D g h  =  R R d  æ ö At  depth..) 2R  = .  (c) : The escape velocity of a body does not depend on the  angle  of  projection  from  earth.  a 2 ( x ) = (12 R - x ) 2  Acceleration of smaller body.  It  is  11  km/sec.  (i)  .  \  mR w =  or  w = GM e R 3 = x2  x1  R  2R  9R  We know that  x1 + x 2  =  9 R x1  + x 1  = 9 R 5  \  x 1  = 45 R  =  7.  2 GM e  R Escape  velocity  does  not  depend  on  mass  of  body  which  escapes  or  it  depends  on  m 0 .  (b)  :  Force  on  object  =  GMm  x 2  \  Work  done  = GMm  dx  x 2  2 R dx  \  ò Work done  = GMm  ò 2  R  x \  Potential  energy  gained  2  15.. when the  smaller  sphere  covered  distance  x 1  and  bigger  sphere  covered  distance x 2 .  (d)  :  Energy  =  (P.  (c) : Escape velocity  ve  = 2 gR \  Kinetic  energy  = 1 mve 2  = 1 m ´ 2 gR = mgR .  =  R R 10.5R.çæ .  F ( x ) = GM ´ 5 M 2  1  1  a ( x ) t 2  and  x 2 =  a 2 ( x) t 2  2  1  2  x a ( x )  Þ 1 = 1  =  5  Þ x1 =  5 x 2  x2 a 2 ( x)  at  x  from  centre  of  earth.29  Gravitation  9.GmM .R ÷ where  d  <<  R  è ø  dg  or  g –  g d  =  R dg  or  D g d  =  R From  (i)  and  (ii).  ( n 2 +1 )  n +1  T = 2 p = 2 p R  = 2 p R  w GM  GM \  T  is  proportional  to R gR 2  . 11.  g h  = g ç 1 . ..5 R 6  Therefore the  two  spheres  collide  when  the smaller  sphere  covered the distance of 7.  3R 2 R ø  è 6 R 20.  R +  x or  v 0  = é GM  ù êëQ  g  = R 2  úû 18.  (ii)  2hg dg  or  d  =  2h.  (c)  :  According  to  Kepler's  law  T 2 µ r 3  12.  when Dg h  = D  g d  .  3  3  T 1  1  æ T1 ö æ r 1  ö æ 1 ö 1  \  ç T ÷ = ç r ÷ = ç 4 ÷ = 64  or  T =  8  è ø 2  2 2  è ø è ø  or  T 2  =  8T 1  =  8  ×  5  =  40  hour.  where R  =  r e  +  h  R 2  (12 R - x)  16.  x1 =  ( n 2 + 1 )  .  The gravitational force acting between two spheres depends  on the distance which is a variable quantity.  2 2  19.  Centripetal  force  =  Gravitational  force 2  \  mR w =  \ GM  or  w = R n +1  GMm  R n  2 R  1 GMm ´ 1  = GMm é .  (a)  :  For  a  satellite  Centripetal  force  =  Gravitational  force GmM e  The gravitational  force.ù = êë x úû R  2 R \  Gain  in  P.  (a)  :  Escape  velocity  = 2 gR  =  GM e  ( re  + h ) 3  3  ( r + h )  \  T  = 2 p = 2 p e  w GM e  \  T  is  independent  of  mass  (m)  of  satellite.GmM  ÷ö = + GmM .) 3R  –  (P.  g d  = g ç 1 .  (b) :  The  centripetal and centrifugal forces  disappear.   Assume  that  a  drop  of  liquid  evaporates  by  decrease  in  its  surface  energy.  it  presses  the  semicircular parts together. The two parts  are held together by a ring made of a metal  R  strip of cross sectional area S and length L.  A  uniform  cylinder  of  length  L  and  mass  M  having  cross­  sectional area A is suspended. To fit the ring on  the wheel.  The length of the slider is 30 cm and its  weight  negligible.025 Nm –1  (d)  0. and its Young’s modulus is Y.  As  it  cools  down  to  surrounding  temperature. it is heated so that its temperature  rises by DT and it just steps over the wheel.  The  surface  tension  of  the liquid film  is  (a)  0.1 Nm –1  (b)  0.4  m  s –1 .  2T  r L  T  T  (a)  r L (c)  (d)  r L (b)  T r L (2013)  5.03 N  m –1 )  (a)  4p mJ  (b)  0. If the coefficient of linear expansion  of the metal is a. The extension  x 0  of the spring when it is in equilibrium is  Mg  Mg  1 +  LA s (a) (b)  k  k M Mg  Mg  LA  s 1 -  1 -  LA s (c) (d) (2013)  k M k 2 M ( ( 3.  )  )  ( )  If a piece of metal is heated to temperature q and then allowed  to cool in a room which is at temperature q 0 .  What should be the minimum radius of the drop for this to be  possible? The surface tension is T.0 × 10 –3  m  (b)  7. then  according  to Newton’s law  of cooling the correct graph between log e (q – q 0 ) and t is  (a)  (d)  7. such that it is half submerged  in a liquid of density s at equilibrium position.30  JEE MAIN CHAPTERWISE EXPLORER  1.5  ×  10 –2  N  (see  figure).  (c)  log (  e q –  q 0 )  7  log (  e q –  q 0 )  PROPERTIES OF SOLIDS  AND LIQUIDS CHAPTER  .5 × 10 –3  m  –3  (c)  9.4p mJ  (2011)  A  liquid in  a  beaker  has  temperature q(t)  at time t  and q 0  is  temperature of surroundings. density  of liquid is r and  L is its latent heat  of vaporization. The water velocity as it leaves the tap is  0.0125 Nm –1  FILM  w  (2012)  Water is flowing continuously from a tap having  an  internal  diameter 8 × 10 –3  m. the graph between  the temperature T of  the metal  and  time t will be closed to  6.  2.  L is slightly less than 2pR.  The  diameter  of  the  water  stream  at  a  distance  2 × 10 –1  m  below the tap is close to  (a)  5.6 × 10 –3  m  (2011)  8. from a  fixed point by a massless spring.  (2013)  4.  so  that  its  temperature  remains  unchanged.2p mJ  (c)  2p mJ  (d)  0.  (a)  (b)  (c)  (d)  t  log (  e q – q 0 )  log (  e q – q 0 )  (b)  t  t  0  t  (2012)  A wooden wheel of radius R is made of two  semicircular parts (see figure).6 × 10  m  (d)  3.05 Nm –1  (c)  0. the force that  one part of the wheel applies on the other part is  (a)  SYaDT  (b) pSYaDT  (c)  2SYaDT  (d)  2pSYaDT  (2012)  A thin liquid film formed between a  U­shaped wire and a light slider supports  a  weight  of  1. with its length vertical.  Work  done  in  increasing  the  size  of  a  soap  bubble  from  a  radius  of  3  cm  to  5  cm  is  nearly  (Surface  tension  of  soap  solution = 0.   The  temperature  at  the  interface  of  the  two  sections is  T 1  l 1  K 1  l 2  K 2 T 2  ..  A long metallic bar is carrying heat from one of  its ends to  the other end under steady­state.  r 2  and r 3?    (a) r 1  < r 3  < r 2 (b) x  x  r 3  (b) r 3  < r 1  < r 2  (c) r 1  > r 3  > r 2  q  (d) r 1  < r 2  < r 3  q  (c) (d) x  x  (2009)  13.  One end of a thermally insulated rod is kept at a temperature  T 1  and the other at T 2 . If the above  ball is in equilibrium in a mixture of this oil and water. A solid ball. The rod is composed of two sections  of  lengths  l 1  and  l 2  and  thermal  conductivities  K 1  and  K 2  respectively.  Assume that the liquid applies a viscous force on the ball that  is proportional to the square of its speed v. i. Another identical tube  (B) is dipped in a soap­water solution.  Two wires are made of the same material and have the same  volume. It comes  to equilibrium    in the position shown in the figure. However wire 1 has cross­sectional area A and wire  2 has cross­sectional area 3A. The oil and water  are immiscible.r 2 )  k (c)  Vg r 1  k (d)  Vg r 1  k (2008)  15. is dropped in the jar.  how  much  force  is  needed  to  stretch wire 2 by the same  amount?  (a)  F  (b)  4F  (c)  6F  (d)  9F  (2009)  12.  A spherical solid ball of volume V is made of a material  of  density r 1 . D  is  2  (a)  b  6 a  (b)  b 2  2 a  2  (c)  b  12 a  2  (d)  b  4 a  (2010)  10. F viscous  = –kv 2  (k > 0). It is falling through a liquid of density r 2  (r 2 < r 1 ).31  Properties of Solids and Liquids  9.  A jar is filled with two non­mixing liquids 1 and 2 having  densities r 1 and r 2 respectively.  A ball is made of a material of density r where r oil < r < r water  with r oil  and r water  representing the densities of oil and water.  where a and b are constants and x is the  x x distance between the atoms.  respectively.  Which of the following  is true  for r 1. The variation of temperature q along the length x of the bar from its hot end is best described  by  which  of  the  following  figures?  q  q  (a) 14.r 2 )  k (b)  Vg (r1 . If the length of wire 1 increases  by Dx  on  applying  force  F. made of a material  of density r 3.e.  A capillary tube (A) is dipped in water.  The potential energy function for the force between two atoms  in  a  diatomic  molecule  is  approximately  given  by  U ( x ) = a12 -  b 6  . The terminal speed of the  ball is  (a)  Vg (r1 . which  of the following pictures represents its equilibrium position?  Oil  (a)  Water  B  A  B  A  B  A  B  (a)  (b)  (c)  Water  (b)  A  Oil  (d)  Water  Oil  (c)  (d)  Oil  Water  (2008)  (2010)  11. Which of the following  shows the  relative  nature  of  the  liquid  columns  in  the  two  tubes?  (2008)  16. If the dissociation energy of the  molecule is D = [U(x = ¥) – U at equilibrium ].   (2004)  24.T1 ) K ö f  ÷ .  (2006)  23.1 m/s.  (2004)  29.5 kg/m 3 )  is 0. at a  distance r from the sun.  (2006)  19.  A wire fixed at the upper end stretches by length l by applying  a force F.  T 1  (2004)  27.  Assuming  the  sun  to  be  a  spherical  body  of  radius  R  at  a  temperature of T K. respectively. The  radial  rate of  flow  of  heat  in  a  substance  between  the  two  concentric  spheres  is  proportional  to  (a)  r1r 2  ( r2 - r1 )  (b)  (r 2  – r 1 )  (c)  r 1  r 2  T 1  T 2  ( r2 - r 1 )  r1r2  æ r 2  ö (d)  ln ç ÷ è r1  ø  (2005)  T 2  (a)  1  (c)  2/3  K  4 x  2K  (b)  1/2  (d)  1/3.  If S is stress and Y is Young’s modulus of material of a wire.  where r 0  is the radius of the earth and s is Stefan's constant.4 m/s  (c)  0.  The  retarding  viscous  force  acting on the spherical  ball is  (a)  directly proportional to R but inversely proportional to v  (b)  directly proportional  to both radius  R  and velocity v  (c)  inversely proportional  to both radius  R  and  velocity v  (d)  inversely proportional  to R  but  directly proportional  to  velocity v. with  f  equal  to  slab. The rate of heat transfer through the  æ A(T2 .  If the terminal speed of a sphere of gold (density = 19.  The figure shows a system of two concentric spheres of radii  r 1  and r 2  and kept at temperatures T 1  and T 2 . The work done  in stretching is  (a)  F/2l  (b)  Fl  (c)  2Fl  (d)  Fl/2.  then  the  ratio  of  the  radiant  energy received on earth to what it was previously  will  be  (a)  4  (b)  16  (c)  32  (d)  64. The momentum transferred to the  surface  is  (a)  E/c  (b)  2E/c  (c)  Ec  (d)  E/c 2 .  the energy  stored in  the wire  per unit volume is  S 2  (a)  2Y/S  (b)  S/2Y  (c)  2S 2 Y  (d)  2 Y  (2005)  21.5 kg/m 3 ) of the  same size in the same liquid  (a)  0.  (2004)  28.  (2004)  26.  the  elongation  of  the  wire  will  be  (in mm)  (a)  l/2  (b)  l  (c)  2l  (d)  zero.1 J. The water  rises up to 8 cm.32  JEE MAIN CHAPTERWISE EXPLORER  (a)  ( K1l1T1 + K 2 l2T 2 )  ( K1l1 + K 2 l2 )  (b)  ( K 2 l2T1 + K1l1T 2 )  ( K1l1 + K 2 l2 )  (c)  ( K 2 l1T1 + K1l2T 2 )  ( K 2 l1 +  K1l2 )  (d)  ( K1l2T1 + K 2 l1T 2 )  ( K1l2 +  K 2 l1 )  (2007)  17.  If two soap bubbles of different radii are connected by a tube.  A radiation of energy E falls normally on a perfectly reflecting  surface.  The  temperature  of  the  two  outer  surfaces  of  a  composite  slab.  (a)  R 2 s T 4  2  r (c)  pr0 2 R 2s T 4  2  r (b)  (d)  4 pr0 2 R 2 s T 4  r 2  r0 2 R 2sT 4  2  4 p r .5  kg/m 3 )  find the  terminal speed of a sphere of silver (density 10.  (2006)  18.133 m/s  (d)  0.  Spherical balls of radius  R  are  falling in a  viscous  fluid  of  viscosity  h  with  a  velocity  v. If the entire arrangement is put in a freely  falling elevator the  length  of water  column in  the  capillary  tube will be  (a)  4 cm  (b)  20  cm  (c)  8 cm  (d)  10 cm  (2005)  22.  (2004)  25. The weight  stretches the wire by 1 mm. incident  on earth. respectively  are T 2  and T 1  (T 2  > T 1 ).  (2003) .2 m/s  (b)  0.  If  the  wire goes over a  pulley and  two weights W each are  hung  at  the  two  ends.2 J  (b)  10 J  (c)  20 J  (d)  0. in a steady state is  ç x è ø  x  20.  A wire elongates by l mm when a load W is hanged from it. evaluate the total radiant power.  If the temperature of the sun were to increase from T to 2T  and  its  radius  from  R  to  2R. Then the elastic energy stored in  the wire is  (a)  0. consisting of two materials having coefficients of thermal  conductivity K and 2K and thickness x and 4x.  (a)  air flows from  the bigger bubble to the smaller bubble  till the sizes become equal  (b)  air flows from  the bigger bubble to the smaller bubble  till the sizes are interchanged  (c)  air flows from the smaller bubble to the bigger  (d)  there  is no flow  of air.  A wire suspended vertically from one of its ends is stretched  by attaching a weight of 200 N to the lower end.2 m/s in a viscous  liquid (density  = 1.  A  20 cm long capillary  tube  is  dipped  in  water.   26.  (2002)  33.  10.  Which of the following is more close  to a  black  body?  (a)  black board paint  (b)  green  leaves  (c)  black holes  (d)  red roses.  (2003)  32.  23. (2002)  34.  9.  31.  17.  (2002)  Answer  Key  1.  (c)  (d)  (b)  (c)  (d)  6. the rate of cooling of  a body is proportional to (Dq) n .  According to Newton's law of cooling.  22.  A  cylinder  of  height  20  m  is  completely  filled  with  water.  14.  12.  The wavelength of the maximum intensity of the spectrum is  correctly  given  by  (a)  Rayleigh Jeans law  (b)  Planck's law of radiation  (c)  Stefan's law of  radiation  (d)  Wien's law.  15.  (d)  (c)  (d)  (a)  (d)  (a) 5.  33.  7.  Two spheres of the same material have radii 1 m and 4 m and  temperatures 4000 K and  2000  K respectively.  25.  34.  The velocity of efflux of water (in ms –1 ) through a small hole  on the  side wall of the  cylinder near its bottom  is  (a)  10  (b)  20  (c)  25.  8.  29.  (d)  (d)  (a)  (b)  (b)  (a)  4. The  ratio of  the energy radiated per second by the first sphere to that by  the second  is  (a)  1 : 1  (b)  16 : 1  (c)  4 : 1  (d)  1 : 9.  27.  11.  The  earth  radiates  in  the  infra­red  region  of  the  spectrum.  (2003)  31.  28.  (a)  (d)  (d)  (c)  (d)  (d)  2. where Dq is the difference of  the temperature of  the  body  and  the  surroundings.  19.  (c)  (b)  (d)  (b)  (d)  .  16.  30.  21.33  Properties of Solids and Liquids  30.  32.  18. and n is  equal  to  (a)  two  (b)  three  (c)  four  (d)  one.  20.  13.5  (d)  5.  (d)  (d)  (b)  (d)  (d)  (b)  3.  24. 4 = p ´ ( d2  )  ´ 2  2  8.  (a)  2.é 13 + 7  ù = é 13 .  (d) : According to Newton’s law of cooling  d q d q = .34  JEE MAIN CHAPTERWISE EXPLORER  According  to  equation  of  motion.b = b .7  ù ú ëê x ú ëê x x û  x û  At equilibrium F = 0  \ 1213a .2 »  2 m s –1 \  According  to  equation  of  continuity  a 1 v 1  =  a 2 v 2 L  Ag =  Mg 2  sLAg  Mg  2  x 0  = k kx0  + s ( p´ \ )  DL  = aD T  L 2  ´ 0. \  F =  2SYaDT  6.q  = ò .  r 2  = 5 cm = 5 ×  10 –2  m  Since bubble  has two surfaces.4 m s –1  h = 0. DL =  LaDT  F  = ( 8 ´ 10 -3  2 w  ( d a b  -  Force.  In equilibrium.  So. F  = .  kx 0  +  F B  =  Mg  v2 = v1 2  + 2 gh = (0.  (d): Let  k  be  the  spring  constant  of  spring and it gets extended by length  x 0  in equilibrium position.  (d) : Here.  (d) : Here.6 b  = 0  or  x 6  = 2 a  b x x 7  a b  U at equilibrium  = -  2  2 a  2 a  b  b ( ) ( )  2 2 2 2 2  = ab2  .  S = 0.b = -  b  2a 4a 2a 4 a  4 a U(x = ¥) = 0  D = [U(x = ¥) –  U at  equilibrium ] ( )  é b 2 ù b 2  = ê 0 .4)2  + 2 ´ 10 ´ 0.84p  × 10 –4  = 4p × 10 –4  J = 0.kdt 0  log e (q – q 0 ) = – kt + C  where C is a constant of integration.  or  F =  2T  Where.03 N m –1  r 1  = 3 cm = 3 × 10 –2  m.q  = .2 m  ) 2  d 2  = 3.  1.03 × 128 × p × 10 –4  = 3.  (d) : According  to  Newton’s  law  of  cooling  the  option  (d)  represents the correct graph.6 × 10 –3  m  Mg  1 -  sLA  k 2 M 3. the graph between log e (q – q 0 ) and t is a straight line with  a negative slope.  F = w  F = 2 TL  2TL = w  w  T  =  2 L Substituting the given  values. surface tension.q 0 )  or  q .4p  mJ  9.025 Nm -1  2 ´ 30 ´ 10 -2  m  7.  4.  Initial surface area  of the  bubble  = 2 × 4pr 1 2  = 2 × 4p  × (3 × 10 –2 ) 2  = 72p  × 10 –4  m 2  Final surface area  of the  bubble  = 2 × 4pr 2 2  = 2 × 4p(5 × 10 –2 ) 2  = 200p × 10 –4  m 2  Increase  in  surface energy  = 200p  × 10 –4  – 72p × 10 –4  = 128p × 10 –4 \  Work done =  S  × increase  in surface energy  = 0. Option (d) represents the correct graph.k (q .  (c) : Increase in length.  (d) :  U  = a -  b  x12 x 6  The  thermal stress developed  is  T D L  =Y = Y aD T  S L T  T  or  T =  SYaDT  From FBD of one  part  of  the  wheel.= ë 4 a ûú 4 a . we  get  1.  5. we get  ò q .  (c) : The  force  due  to  the  surface  tension  will  balance the  weight.kdt dt 0  d q Integrating both sides.5 ´ 10 -2  N  T= = 0. F is the force that one part of the wheel applies on  the  other  part. d 1  = 8 ×  10 –3  m  v 1  = 0.dU  = dx  x12 x6  dx )  -12a 6 b  12 a 6 b  = .   11.  so  oil  should  be  over  the  water.T ] K 2 A [T – T 2 ]  =  l1 l2  or  or  K 1l  T 2)    2(T    1  –  T) =  K 2l    1(T –      T(K 1 l 2  + K 2 l 1 ) = K 1 l 2 T 1  +  K 2 l 1 T 2  or  T  = K1 l2 T1 + K 2 l1 T 2  .  When  the  ball  is  moving  with  terminal  velocity  2  a = 0.  The  viscous  force  rapidly  increases with velocity.  Þ v =  Vg (r1 .  (d)  :  Terminal  velocity  =  v  viscous  force  upwards  =  weight  of  sphere  downwards  4  6 phrv = æç pr 3 ö÷ (r – s ) g è 3  ø  For  gold  and  silver  spheres  falling  in  viscous  liquid.  l1 / 3  l1  12.  R  r  Sun Earth  4 2  Energy  = sT ´ 4 pR 2  ´ time  Area of envelope  4 p r 4 2  Energy  incident  per  unit  area  on  earth  = sT R  2 ´ time  r . mg – Vr 2g –    kv 2  =  ma where V  is  volume.  in  a  sphere  of  radius  r.  r 3  16. the force acting down or  T µ h without making finer corrections. Soap  reduces the  surface  tension  of  water.  according  to  Stefan's  law.  V =  V1  =  V2  V =  A ×  l1  = 3A ×  l2 Þ  l2  =  l1/3  Y= D l  F /  A  Þ F1  = YA  1  D l / l l1  D l  F2  = Y 3 A  2  l2  Given Dl1  = Dl2  = Dx  (for  the  same  extension) \ F2 = Y × 3 A × ( )  Dx YADx  = 9× = 9 F1  or 9 F .  or  2 2  or  19. If r 3  had been greater  than r 2 .  (d) : For the  same  material.  (d)  :  Let  T  be  the  l 1  temperature  of  the  K 1  interface.  (d)  :  The  force  acting  upwards  2prT  .5 9 1  v g  0.  q min . v  is the terminal  velocity. Vmin  Potential difference is proportional to  resistance  (thermal  as  well  as  x electric).35  Properties of Solids and Liquids  10.  Therefore r 1  < r 2 .  (b) : Heat  flow  can  be  compared  to  charges  flowing  in  a  conductor.  (a)  :  The  liquid  1  is  over  liquid  2.  dQ Current is the same. For both sides. But  it is  also  a wetting agent. v g r g  – s 19. q 1 q 2  dt  The potential difference V 1  – V  V 2  rl  V 1  current  V  at any point = I × Resistance  = I  ´  A Potential difference is µ l but negative. Young’s  modulus  is  the  same  and it is given that the volume is the same and the area of  cross­section for the  wire  l1  is  A  and that  of  l2  is  3A.  hpr 2rg.  k 15.  Earth  (E)  in  space  receives  part  of  this  energy.1 m/s. the length  of wire is L/2 on each side and so the  elongation will be l/2. Therefore  Vr 1g –  Vr 2g      –  kv  =  0.5 18 2  = = = = \  vs r s  – s  10. The  height of liquid supported  decreases. Therefore the meniscus will not  be  convex as  in  mercury. attaining  a maximum when the ball reaches  the  terminal  velocity.  13.  (c)  :  Energy  radiated  by  sun. V max  (temperature  decreases)  but  it  is  a  straight line function. thrust  by Archimedes principle upwards  and  the  force due  to    the    force  of  friction  also acting upwards.  (c)  :  As r oil  < r w ater .  potential decreases  q  q  max.  As  l increases.  Then  the  mg = V r 1 g  acceleration is zero.  14.  L  2  l  =  L  W  W  W 18.  (b) : The forces acting on the solid  Vr 2 g ball when it is falling through a  viscous force  liquid are mg downwards.5 – 1.5 – 1.2  vs  = = = 0. so the ball will sink  in the oil  but r < r water  so it  will float in the  water.  it will  not  be partially  inside  but anywhere inside liquid 2 if r 3  = r 2  or it would have sunk totally if r 3  had  been greater than r 2 .  K1 l2 + K 2 l1  17. \ r 1  < r 3  < r 2 .  (b)  :  Y  = Force ´ L = WL  A´l Al P  WL  \  AY Due to pulley arrangement. Therefore  (d).  E  = sT 4  ×  (area  4pR 2 )  (time)  This  energy is spread around  sun in space.r 2 )  .  Since  two  section  of  rod  T 1  are  in  series.  elongation  =  l.  rate  of flow  of heat in them will be equal \  l 2  K 2  T 2  K1 A [T1 .  Hence  option  (c)  is  correct. As r > r oil . T )t 2 KA(T ..36  JEE MAIN CHAPTERWISE EXPLORER \  \  æ 2 4  ö Power  incident  per  unit  area  on  earth  = ç R s2 T  ÷ è r ø 2 4  2  R sT  p r  ´  Power  incident  on  earth  =  0  r 2  20.  25.  (i) Workdone  =  Fl .  (c)  :  Pressure  inside  the  bubble  = P 0  +  r Smaller the  radius.  22.T 1 ù ê ú f  = x ê 3  ú ´ 1  x ë û ë û  1  f =  .  2  29.  (d) : According to Newton's law of cooling.  dQ = .T1 ) t  Q1 = Q 2 Þ  = x 4 x or  2(T 2  –  T)  =  (T  –  T 1 )  2 T + T  T = 2 1  or  3  2 T + T  ù KA é Q1 = T .  (d)  :  Energy  stored  per  unit  volume  = 1  ×stress× strain  2  =  Stress × stress  S 2  =  ..  (d) :  From  first  surface.  (b)  :  In  a  freely  falling  elevator  g  =  0  Water  will  rise  to  the  full  length  i.T2 ) r1r 2  Q  = ( r2 ..  (ii)  E 1  (4000)4 ´ (1)2  ´ 1 ´ 4 pse  1  = = E 2  (2000) 4 ´ (4)2  ´ 1 ´ 4 ps e 1 .1 J  . greater will be  the pressure..  (a)  :  For  conduction  from  inner  sphere  to  outer  one. rate of cooling  is  proportional  to Dq.2 1  ú t  \  x êë 2  3  û  26.T1 ) K ù KA é T2 .r ÷ .  or  ...T1 ) t  From  second  surface.  2 2  30.  (a)  :  Energy  radiated  E  = sT 4  ×  (area  4pR 2 )  ×  time  ×  e  2  YAl  Workdone  =  2 L From  (i)  and  (ii)  E 2  s (2T )4 ´ 4 p (2 R ) 2  ´ t  = = 16 ´ 4  E 1  sT 4 ´ (4 pR ) 2  ´ t E 2  = 64.  Q 2  =  (4 x )  At  steady  state.  Momentum  transferred  to  surface  =  c  Change  of  momentum  =  28.  E1  31..K ´ (4 pr 2 )  or  dt dr \  Radial  rate  of  flow  Q = -4 pKr 2  dT  dr r2 KA(T2  .  2Y 2 Y 21.  (d) :  Wien's  law  32..  (b)  :  v = 2 gh = 2 ´ 10 ´ 20 =  20 m/s.  (a) : A good absorber is a good emitter but black holes do  not  emit  all  radiations.  Hence  air  flows  from  the  smaller  bubble  to  the  bigger.  34.e..T 1 ]  ë r1r2  û  or  4 pK (T1 .  Q 1  =  or  or  27. Air flows  from  higher  pressure  to  lower  pressure. \  (Dq) n  =  (Dq)  or  n  =  1. .KA dT  ´ (time dt )  dr dQ  dT  = ..  YA l  YAl 2  ò dW = L ò l dl  = 2 L 0  or  E æ E ö 2 E  -ç.  (d)  :  Elastic  energy  per  unit  volume  1  = ´ stress ´ strain  2 \  Elastic  energy  1  = ´ stress ´ strain ´  volume  2 1  F DL  = ´ ´ ´ ( AL )  2  A L 1 1  = F DL = ´ 200 ´ 10 –3  = 0..  33.÷ = c è cø c 2 E  .  (d) :  Young's  modulus  Y  =  FL  Al YAl  F  =  \  L YAl ( dl )  or  dW = F dl  =  L é A(T2 .  KA(T2 .  (d)  :  According  to  Stefan's  law..  Radiant  energy E  =  (sT 4 )  ×  area  ×  time 4T  23.  (b)  :  Initial  momentum  =  E/c  Final  momentum =  –E/c \  T 2  dr  = -4 pK ò  dT  r 2  T 1  \  Qò or é r .  (b)  :  Retarding  viscous  force  =  6phRv  obviously  option  (b)  holds  goods..T ) t  x (2 K ) A(T .r1 )  \  \  æ r1r 2  ö Q  is  proportional  to  ç r .  20  cm  to  tube.  è 2 1  ø  \  r1 \  24.  3  .r  ù Q ê 1 2  ú = 4 pK [T2 .   operating  with  an  ideal  monoatomic  gas. The  other chamber has volume V 2 and contains ideal gas at pressure  P 2  and temperature  T 2 .  Ignoring  the  slight expansion of the water. the change in its internal energy  is (specific heat of water is 4184 J kg –1  K –1 )  (a)  4.  the  intake  temperature for the same exhaust (sink) temperature must be  (a)  1200 K  (b)  750 K  (c)  600 K  (d)  efficiency of Carnot engine cannot be made larger than 50%  (2012)  Directions: Question numbers 8.  If the  partition  is removed  without  doing any work on the gas.99  (2010)  (2013)  A Carnot engine.  A Carnot engine operating between temperatures T 1  and T 2  1  . One of the chambers has volume V 1  and contains ideal gas at pressure P 1  and temperature T 1 . whose efficiency is 40%.m 2  and m 3  and the  number of molecules are n 1 . extracted from the source in a  single cycle is  (a)  4p 0 v 0  (b)  p 0 v 0  1 3  11  pv (c) (d) 2  p0v 0  2  0 0  ( )  2.  as shown in the P ­  T diagram.  The  amount  of  heat.1 kJ  (2011)  7.5  (c)  0.  Two moles of helium gas are taken over the cycle ABCDA.  3.1%  (b)  10.  When T 2  is lowered by 62 K. Then T 1  and T 2  are.  ( )  100 g of  water  is heated from 30°C to  50°C. It is desired  to  have  an  engine  of  efficiency  60%. takes in heat from  a source maintained at a temperature of 500 K. 9 and 10 are  based on  the  following paragraph.  A diatomic ideal gas is used in a Carnot engine as the working  substance.  6.4 kJ  (c)  84 kJ  (d)  2.25  (b)  0. The  masses of  molecules are m 1 .  Assuming the gas to  be ideal the  work done on  the gas  in  taking it from A to B is  (a)  200R  (b)  300R  (c)  400R  (d)  500R  9.  Then.2 kJ  (b)  8. n 2  and n 3  respectively. Assuming  no loss of energy.  2008)  .5%  (c)  12. If during the adiabatic expansion part of the cycle  the volume of the gas increases from V to 32V. the efficiency  of the engine is  (a)  0.  The work done on the gas in taking it from D to A is  (a)  – 414R  (b)  + 414R  (c)  – 690R  (d)  + 690R  10. its efficiency  6  1  increases to  .5%  (d)  15. the final equilibrium temperature  of the gas in the container will be  T1T2 ( PV 1 1 + PV  2 2 )  T T ( PV + PV  )  (a)  PV T + P V T (b)  1 2 1 1 2 2  1 1 1 2 2 2  PV 1 1T2 + PV 2 2T1  (c)  PV 1 1T1 + PV 2 2T  2  PV 1 1 + PV 2 2  (d)  PV 1 1T2 + PV 2 2T  1  PV 1 1 + PV 2 2  (2004.  The above p­v diagram represents  the  thermodynamic  cycle  of  an  engine.  5  2 × 10  B  C  2P 0  Helium gas goes through a cycle  ABCDA (consisting of two isochoric  P 0  and two isobaric lines) as shown in  A  D  figure.T 2  and T 3  are  mixed.  An insulated container of gas has two chambers separated by  an insulating partition.  Efficiency  of  this  cycle  is  nearly (Assume the gas to be close  V 0  2V 0  to ideal gas)  (a)  9.  The net work done on the gas in the cycle ABCDA is  (a)  zero  (b)  276R  (c)  1076R  (d)  1904R  (2009)  11.  4. the final temperature of the mixture is  n1T1 + n2T2 + n3T 3  (T + T + T )  (a)  1 2 3  (b)  n1 + n2 + n3  3  (c)  n1T12 + n2T22 + n3T 3 2  n1T1 + n2T2 + n3T3  (d)  n12T12 + n22T22 + n32T 3 2  n1T1 + n2T2 + n3T3  (2011)  5. respectively  3  has efficiency  (a)  372 K and 310 K  (c)  330 K and 268 K  (b)  372 K and 330 K  (d)  310 K and 248 K  (2011)  A  B  D  C  P(Pa)  5  1 × 10  T  500 K  300 K  8.75  (d)  0.37  Thermodynamics  CHAPTER  THERMODYNAMICS 8  1.4%  (2012)  Three perfect gases at absolute temperatures T 1 .   22.  24.  Even Carnot engine cannot give 100% efficiency because we  cannot  (a)  prevent radiation  (b)  find ideal  sources  (c)  reach absolute zero temperature  (d)  eliminate friction.  (a)  (b)  (b.  The  temperature­entropy  diagram  of  a  reversible  engine cycle is given in the  figure.  When a system is taken from state  a  i to state f along the path iaf.  Heat given to a body which raises its temperature by 1°C is  (a)  water equivalent  (b)  thermal capacity  (c)  specific heat  (d)  temperature gradient.4 × 10 6  J  (c)  16. Its efficiency is  (a)  1/3  (b)  2/3  (c)  1/2  (d)  1/4  T  2T 0  (c)  It is not applicable to any cyclic process  (d)  It  is  a  restatement  of  the  principle  of  conservation  of  energy  (2005)  18.  A system goes from A to B via  two processes I and II as shown  P  II  in  figure. W along the path ibf is  (a)  14 cal  (b)  6 cal  (c)  16  cal  f  b (d)  66 cal  (2007)  14.  (2002)  17.2 × 10 6  J  (b)  8.  10.  13.  11.  14.  25.  17.  (b)  (c)  (b)  (a)  .  23.  12. The work done by the  engine is  (a)  4.8 × 10 6  J  (d)  zero.  (2003)  22. it is  found that Q = 50 cal and W = 20  cal.  (c)  (c)  (b)  (b)  (b) 2.  During an adiabatic process.  9.  Which of the following is incorrect regarding the first law of  thermodynamics?  (a)  It introduces the concept of the internal energy  (b)  It introduces the concept of  entropy  25.  (2004)  19.  15.  (2002)  T 0  S  S 0  2 S 0  Answer Key  1. the pressure of a gas is found to  be proportional to the cube of its absolute temperature.  7.  19.  Which  of  the  following  statements  is  correct  for  any  thermodynamic system?  (a)  The internal energy changes in all processes.  (2002)  (2005)  24.  20.  A  Carnot  engine.  A Carnot engine takes 3 × 10 6  cal of heat from a reservoir at  627°C.  (2003)  21.  (2003)  23.  "Heat cannot by itself flow from a body at lower temperature to  a body at higher temperature" is a statement or consequence of  (a)  second law of thermodynamics  (b)  conservation of momentum  (c)  conservation of mass  (d)  first law of thermodynamics.  (b)  (c)  (b)  (c)  3. c)  (c)  6.  Which statement is incorrect?  (a)  all reversible cycles have same efficiency  (b)  reversible cycle has more efficiency than an irreversible  one  (c)  Carnot cycle is a reversible one  (d)  Carnot cycle has the maximum efficiency in all  cycles.  having  an  efficiency  of h  =  1/10  as  heat  engine. and gives it to a sink at 27°C.38  JEE MAIN CHAPTERWISE EXPLORER  12. The  ratio C P /C V  for the gas is  (a)  4/3  (b)  2  (c)  5/3  (d)  3/2.  If  the  work  done  on  the  system  is  10  J.  18.3 J mol –1  K –1 )  (a)  monoatomic  (b)  diatomic  (c)  triatomic  (d)  a mixture of monoatomic and diatomic.  The  gas  is  (R = 8.  the  amount  of  energy  absorbed  from  the  reservoir at lower temperature is  (a)  100 J  (b)  99 J  (c)  90 J  (d)  1 J  (2007)  13.  (b)  (b)  (a)  (a)  5.  (b)  Internal energy and  entropy are state functions.  8.  is  used  as  a  refrigerator.  (c)  The change in entropy can never be zero.  (d)  The work done in an adiabatic process is always zero.  16.  21. then  V  (a) DU 2  > DU 1  (b) DU 2  < DU 2  (c) DU 1  = DU 2  (d)  relation between DU 1  and DU 2  cannot be  determined  (2005)  16. Along  the  path  ibf  Q  =  36  i  cal.  The  work of 146 kJ  is performed  in order  to compress  one  kilo  mole  of  gas  adiabatically  and  in  this  process  the  temperature  of  the  gas  increases  by  7ºC.  (2003)  20.  (2006)  15.  If DU 1  and DU 2  are  the changes in internal energies  A  B  in  the  processes  I  and  II  I  respectively.  (d)  (b)  (c)  (d)  4.  Which of the following parameters does not characterize the  thermodynamic state of matter?  (a)  temperature  (b)  pressure  (c)  work  (d)  volume.   (c) :  Heat is extracted from the  source in path DA and AB.  T  h = æç1 ..  (b) : The final temperature of the mixture is  ( )  DQ = DQ DA  + DQ AB  3 3 3  R DT = V0 D P =  P0V0  2 2 2  T mixture  = T1n1 + n2T2 + n3T 3  n1 + n2 + n3  5.4%  13 3 PV + 5 PV 0 0 0 0  2  4.  T  h = 1 -  2  T1  where  T 1  is  the  temperature  of  the  source  and  T 2  is  the  temperature of the sink. then its efficiency  becomes  \ 1  3  1  = æ1 .62  2  =  6T 3  2  5  5(T 2  . h = ´ 100  Heat supplied to the gas = PV  200  0 0  ´ 100  = = 15.  C p  = \ CV = 3 R and  C P  =  5 R 2 2  5 R 2  2 p0 v 0  ù 10  5 é 2 p 2v \ DQAB  = n R ê 0 0 = p v  2 ë nR nR ûú 2  0 0  \  The amount of heat extracted from the source in a single  cycle is \ D Q AB = nCV D T = n Along the path BC. \  Work done by the gas  W = Area of the rectangle ABCD =  P 0 V 0  Helium gas is a monoatomic gas.2  Þ  2  = 1 . when T 2 is lowered by 62 K.0 0  ú = p0v 0  2 ë nR nR û  2  Along the path AB.40 =  3  100 500  500 100 5  T2  = 3  ´ 500 = 300 K  5  For 2 nd  case h = 60%.. Hence DQ AB  = nC pDT = nC p (T B  – T A )  ( )  For monoatomic gas.  T1  = 6 ´ 310  = 372 K  5  6. heat supplied to the gas at constant volume. DQ DA  = nC vDT = nC v (T A  – T D )  According to ideal gas equation  pv  pv = nRT or  T  =  nR A  B  Along the path AB.  (a) : The efficiency of Carnot engine. work done is equal to the area  under  the  cycle  and  is  taken  to  be  positive  if  the  cycle  is  clockwise. heat supplied to the gas at constant pressure.  (d) : In case of a cyclic process.  Cv  = 3 R 2  p v  ù 3  3 é 2 p v \ DQDA  = n R ê 0 0 . m = 100 g = 100 × 10 –3  kg  s = 4184 J kg –1  K –1  and DT = (50 – 30) = 20°C \ DQ = 100 × 10 –3  × 4184 × 20 = 8.62)  2  =  6T2  3  5T 2  – 310 = 4T 2 Þ  T 2  = 310 K  From equation (i).  (b) : Efficiency of Carnot engine.  D  C  For a monoatomic gas.(i)  As per question.39  Thermodynamics  1.  Along path DA. heat  is rejected by the gas  Work done by the gas  Efficiency.4 × 10 3  J (Using (i))  .  5 5  R D T = (2 P0 ) DV = 5 PV 0 0  2 2  Along the path CD and DA. pressure is constant. volume is constant.  \ D QBC = nC P DT = n = 3 p0v0 + 10 p0 v0 =  13  p0v0  2 2 2  2.62  1  = 1 -  T1  3  T 2  . h = 1  6 çè T1  ø  6  ( T 2  5  =  T1  6  Þ T1  =  )  6 T 2  5  .  (b) : DQ =  msDT  Here. T 1  = 500 K  T T 40  \ = 1 .62 ö 3  çè T1  ÷ø  T 2  .T 2  .  For 1 st  case h = 40%.2  ö÷ Given. T 2  = 300 K  \ 60 300  300 60 2  = 1 = 1 =  Þ  100  T1  T1  100 5  T1  = 5  ´ 300 = 750 K  2  3.2  ö÷ è T1  ø T  1 æ \ = 1 .  Hence.   The  total  number  of  molecules  is  conserved.  from C to D is equal and opposite.  (c)  : DU 1  = DU 2 . \  Work done along AB and CD cancel each other because  pressure changes but temperature is the same.693) =  –415.  They cancel each other. n  =  P2V 2  R1T1 2  RT2  Final state = (n 1  + n 2 )RT  g . DT is 200 K.693 = 277R.75  4 8. temperature remain the same.  (c) : For Carnot engine efficiency  h =  QH . V  \  Work done by the gas  = W =  nRT  ln  2  V1  P 1  = nRT  ln  P2  Þ  W = –600R (0.  Work done on the gas from A to B = 400R. volume increases.  a  f  Q ibf  = DU ibf  + W ibf  Since DU iaf  = DU ibf  . temperature remains the same. Þ  W BC  + DA  = (2 ln 2) × (200R)  = 400R ×  0.  10.  (b) :  As this  is  a simple  mixing  of  gas.  (c) : For an adiabatic process  TV g–1  = constant \  T1V1g -1 = T2V2 g -1  g .1  7  For diatomic gas. P is the same.  Work done on the  gas  in  taking  it  from B to  C. DW = 0)  DU = DQ = 8. .1  T2 (32) 5  = T2 (32) 2/5  = T 2 (2 5 ) 2/5  =  4T 2  T 2  = 1 -  1  T1  4  ( )  h = 3  =  0.40  JEE MAIN CHAPTERWISE EXPLORER  As DQ = DU + DW \  Change in internal energy ( .8  J  mol  K  For diatomic  gas.  T  300 K  ( n1 + n 2 ) = T  = Efficiency of the engine.  (b) : D to A.8R.  Q iaf  = DU iaf  + W iaf  or DU iaf  = Q iaf  – W iaf  = 50 – 20 = 30 cal  For the path ibf.  This is the work done by  the gas \  Work done on the gas = +415.  14.4 × 10 3  J  = 8.  Net work done on the gas of 2 moles of helium through the  whole network = 277R per cycle or nearest to the answer (b).  9.  even  if  adiabatic  conditions are satisfied.  . h = 1 - PV 1 1 .8R.3 =  20.  pressure  is  decreased.1  10  = 9  b= 1/10 Q  (where W is the work done)  Also  b =  L  W or  Q L  = b × W = 9 × 10 = 90 J.  Q ibf  = DU iaf  + W ibf \  W ibf  = Q ibf  – DU iaf  = 36 – 30 = 6 cal.( -146) ´ 10  or  C V  = -DW  = n DT (1 ´ 103 ) ´ 7  –1  –1  =  20.  (b) :  According  to  first  law  of  thermodynamics DQ  = DU  + DW  For  an  adiabatic  process. .  change  in  internal  energy  are  path  i  b independent.  Nearest to (b).  because  the  change  in  internal  energy  depends  only  upon  the  initial  and  final  states  A  and  B.  (b) : According to first law of thermodynamics for the path  iaf. work  done on  the gas  = +414R.4 kJ  7.  15.  g = 5 \  T1 = 7  .  For taking from D to A.  13.  T  = T1T2 ( PV n1 + n2  T2 PV 1 1 + T1P2V2  12.8 J mol -1 K -1  2 2  Hence  the  gas  is  diatomic.  5 5  CV  = R = ´ 8.  P(in pascal)  A  (c)  : 2 × 10 5  B  1 × 10 5  D  C  PV PV T PV + T P V  1 1 + 2 2 =  2 1 1 1 2 2  RT1 RT2 RT1T2  T1n1 + T2 n 2  1 1 + P2V 2 )  .1  = T 2  11.  500 K  Path AB.1  V  T1 = T 2  æç 2 ö÷ è V1  ø \ n1 = ( )  32V  V = T2 (32) g .  (b) : Total work done on the gas when taking from A to B = 400R.  PV = nRT for all process \  PDV = nRDT = 2R 200 = 400R. DQ  =  0 \  0  = DU  + DW  or DU  =  – DW  or  nC V DT  =  – DW  3  . Þ  W BC  + W DA  = 2 ln 2(500R – 300R).h h  1 . PV = nRT for adiabatic as well as  isothermal  changes.Q L  QL  Coefficient of performance of a refrigerator  b = 1.   25.  (d)  :  In  an  adiabatic  process.  (c)  :  We  cannot reach  absolute  zero  temperature.  g = 5 = 1.  0 0  17.3 = g or  2g  =  3 Þ g  =  3/2  5  N.  22.3 =  3 .  (b)  :  Internal  energy  and  entropy  are  state  functions.67  7  For  diatomic  gas.5.  21.B  For  monoatomic  gas.  g = 3 = 1.1 = 3Þ 3 g .  (a)  :  Second  law  of  thermodynamics.  (b)  :  Thermal  capacity.  (b) :  Efficiency  = 1 - T 2  300 1 2  = 1= 1 .2) ´ 3 J  =  8.  18.2  J \  Workdone  by  engine  =  (Heat  energy)  ×  (efficiency)  2  6  =  (3 ´ 10 ´ 4.3T S = 1 .  19.41  Thermodynamics  Q 2  16.=  T1  900 3 3  Heat  energy  = 3  ×  10 6  cal  =  3  ×  10 6  ×  4.  T g = (constant) P g -1  or  Tg/g–1  =  (constant)  P  Given  T 3  =  (constant)  P g \  g .4  when g = 1.  (c)  :  The  work  does  not  characterize  the  thermodynamic  state  of  matter. c) :  Statements  (b) and  (c) are  incorrect regarding  the  first  law of  thermodynamics.  (a) :  Efficiency  h = 1 -  Q 1  Q 2  =  T 0  (2S 0  –  S 0 )  =  T 0  S 0  Q1 = T0 S0 + T0 S 0  3  = T0 S 0  2 2  T0 S 0  ´ 2  2 1  \  h = 1 .  20. the gas must be a suitable mixture of monoatomic  and  diatomic  gases \ g  =  3/2.  (b. .4  ×  10 6  J.  23.  (a) :  All  reversible  cycles  do  not  have  same  efficiency.  24.  Assuming no heat  is lost  to  the surroundings.  3.  (d)  5.  One kg of a diatomic gas is at  a pressure of 8 ×  10 4  N/m 2 . the final temperature of the gases.42  JEE MAIN CHAPTERWISE EXPLORER  CHAPTER  KINETIC THEORY OF GASES  9  1.  (2004)  7.  2.  (a)  3/2  (b)  23/15  (c)  35/23  (d)  4/3.  At  what  temperature  is  the  r.1)  ( g . What is the energy of the  gas due to  its  thermal  motion?  (a)  3 × 10 4  J  (b)  5 ×  10 4  J  (c)  6 × 10 4  J  (d)  7 ×  10 4  J  (2009)  If  C P  and  C V  denote  the  specific  heats  of  nitrogen  per unit  mass at constant pressure and constant volume respectively.  9.  4. The  ratio C P /C V  of the  mixture  is  (a)  1. It is moving with speed  v and is suddenly brought to rest.  A thermally insulated vessel contains an ideal gas of molecular  mass M and ratio of specific heats g.1)  2  2  (a)  2( g + 1) R Mv  K  (b)  2 g R Mv  K  ( g - 1)  2  g Mv 2  Mv  K  (c)  (d)  (2011)  K  2 R 2 R 7  (c)  T f  =  T0  3  Two rigid boxes containing different ideal gases are placed on  a table.  8.  One mole of ideal monoatomic gas (g = 5/3) is  mixed with  one mole of diatomic gas (g = 7/5).  (a)  .  7.  to  that at constant volume.  (d)  6. T f .s. (2002)  8.m.  (2002)  Answer  Key  1.  velocity  of  a  hydrogen  molecule equal  to that  of an  oxygen molecule at 47°C?  (a)  80 K  (b)  –73 K  (c)  3 K  (d)  20  K.  2  4. The temperature of the gas molecules inside  will  (a)  increase  (b)  decrease  (c)  remain same  (d)  decrease for some. The boxes are then put into thermal contact with each  other  and  heat  flows  between  them  until  the  gases  reach  a  common final temperature.62  (2005)  6.  while increase for others.  while  Box  B  contains  one  mole  of  helium  at  temperature  (7/3) T 0 .  (b)  (c) (2006)  5. its temperature increases  by  ( g . then the value of g for the resulting mixture is  (a)  7/5  (b)  2/5  (c)  24/16  (d)  12/7.54  (c)  1. Box A contains one mole of nitrogen at temperature T 0 .  (b)  (d)  3. (Ignore the heat capacity of boxes). What is g for the mixture?  g denotes the ratio  of specific heat  at constant pressure.  Cooking  gas  containers  are  kept  in  a  lorry  moving  with  uniform speed.  1 mole of a gas with g = 7/5 is mixed with 1 mole of a gas  with g = 5/3.  Then.  The density of the gas is 4 kg/m 3 . in terms of T 0  is  5  3  (b)  T f  =  T0  (a)  T f  =  T0  2  7  A  gaseous mixture consists of 16 g of helium and  16  g  of  oxygen.  (d)  (c)  2.59  (d)  1.  (2002)  9.4  (b)  1.  then  (a)  C P  – C V  = 28R  (b)  C P  – C V  =  R/28  (c)  C P  – C V  = R/14  (d)  C P  – C V  = R  (2007)  3  (d)  T f  =  T0  .   (d)  :  v rms  = or  T f  =  3 T0 .43  Kinetic Theory of Gases  1  2  1.1  1+ 1 1 1  = + g m .  32 2  (c) :  It is the relative  velocities between  molecules that is  important.  As the vessel is stopped suddenly.  28  (d)  : DU  =  0 3 æ 7  ö æ5 ö \  1 ´ ç 2 R ÷ (T f .1 = 0.  C V  = or  8g m  –  8  =  4  or  8g m  =  12  12 24  or  g m = 8 = 16  8.  (d) : Kinetic energy  of  vessel  =  mv 2  Increase in internal energy  DU = nCV D T where n is the number  of moles of  the  gas  in vessel.1 g1 .5  =  3/2.  n 2  =  1/2 1  2  mv = nCV D T 2  3.62.  RT  M = ( vrms ) H 2  273 + 47  T  = Þ T = 20 K  .  2  \  ( vrm s )O 2 16  = 4  (d)  :  For  16  g  of  helium.  n 1  =  4  For  oxygen.T0 ) + 1´ 2 R ç T f  .1 g1 .1 g 2  .  f  =  5.1 = 2 + 2  = 4 Þ g m  . its kientic energy is used to  increase the temperature of  the  gas \  or DT = C P  = n1CP1 + n2 C P 2  n1 + n2  f  where  C P  = çæ + 1 ÷ö R è 2  ø  \ (Q  n = Mm )  Mv 2 ( g .  V  \ U = 5 ´ 8 ´ 10 4 ´ 1  = 5 ´ 104  J  2 4  \  28C P  – 28C V  = R  C P  = C V  n1 + n2 n n = 1 + 2  or  1 + 1 = 1 + 1  g m  .  f  =  3.  n 1  =  4 16 1  =  For  16  g  of  oxygen.5  m \ g m  =  1.1 5 .  (c)  :  For  mixture  of  gases  n1 + n2 n n = 1 + 2  g m  .1)  K  2R 6.  (5  is  the  degrees  of  freedom  as  the  gas  is  diatomic)  But PV = mRT  1 kg  mass 1  V= = =  m 3  density 4 kg/m 3  4  P = 8 × 10 4  N/m 2 .  Root  mean  square  velocities are  different  from  lateral translation.1  7 5  -1 - 1  5 3  2 = 5 + 3  g m - 1 2 2  ( ) ( )  (b) : Molar heat capacity = Molar mass × specific  heat capacity  So.  or  9.1 7  - 1  3 5  (Q  C  = (g R .  (4 ´ 32 R ) + ( 12 ´ 5 2 R)  29  (a)  :  For  mixture  of  gases. the molar heat capacities at constant pressure and  constant  volume will be 28C P  and 28C V  respectively 2 8  or  g - 1 = 2  m or  C P .  f  where  CV  =  R 2  1  2  mv = D U 2  1  2  m mv = C DT 2  M V  Mv 2  DT  = 2 CV  4.  2 3 5  or  g .  5.  7.  n1CV1 + n2 C V 2  n1 + n2  For  helium. .1) )  2.3 T0  ÷ = 0  è ø è ø  or  5T f  –  5T 0  +  3T f  –  7T 0  =  0  or  8T f  =  12T 0  (4 ´ 52 R ) + ( 12 ´ 7 2 R ) = 47 = 1.  (b) : The thermal energy or internal energy is  U = 5 m RT for  2  diatomic  gases.1 g 2  - 1  g m .  n 2  =  32 2 For  mixture  of  gases.CV  =  R  . t) of a  wave on  a string  is  given  by  2 y ( x.2 × 10 11  N/m 2  respectively?  (a)  770 Hz  (b)  188. Their  mean position is separated by distance X 0  (X 0  >  A).729  6.    If  the  maximum  separation  between  them  is  (X 0  +  A).  The amplitude of a damped oscillator decreases to 0.  with b as  the  constant of  proportionality.  3. When  the  mass  M passes  through  its  mean position then a smaller mass  m  is placed  over  it  and  both of them move together with amplitude A 2 .50(m)  û  ëê The tension in the string is  (a)  6.04(s) 0.2  Hz  (d)  200.  y = 0.5 N  (d)  0.04 kg m –1  is given by t x  ù . executes  SHM  with a  amplitude A 1 .  An  ideal  gas  enclosed  in  a  vertical  cylindrical  container  supports a freely moving piston of mass M.0 N  (c)  12.  The  tension in  it produces  an  elastic  strain  of  1%.9 times  its original magnitude in 5 s.  has  a  fundamental  frequency.44  JEE MAIN CHAPTERWISE EXPLORER  CHAPTER  OSCILLATIONS AND WAVES 10  1. The tube is dipped vertically in water so  that half of it is in water.  If  x. the velocity  and  the  acceleration of a particle executing simple harmonic motion  of time period T.  (c)  (d)  0.5 N  (2010)  ( )  10.  the  phase  difference  between  their  motion  is  1/ 2  (2011)  The transverse  displacement  y(x. f in air. then.  (2013)  2. which of the following does not change  with time?  (a)  a 2 T 2  + 4p 2 v 2  (b)  aT/x  (c)  aT + 2pv  (d)  aT/v  (2009)  .02 (m) sin é 2p ú 0.  v  and  a denote  the  displacement.( ax + bt This represents a  2  + 2 ab xt )  (a)  wave moving in +x­direction with  speed  (b)  wave moving in –x­direction with  speed  a  b  b  a  (c)  standing wave of  frequency  b  1  (d)  standing wave of  frequency  b  9.  1  b  (c)  2  b  7. What  is  the  fundamental  frequency  of  steel  if  density  and  elasticity  of  steel are 7. The ratio of  æ A 1  ö çè A ÷ø  is  2  (2013)  (2012)  5. the volume of the gas is V 0  and its  pressure  is  P 0 .  (2011)  The  equation  of  a  wave  on  a  string  of linear  mass  density  0.6  (b)  0.  attached  to  a  horizontal  spring.5  Hz  (c)  178. Assuming that the system  is completely isolated from its surrounding.693  b  A  cylindrical  tube.  When the spherical bob of the pendulum suffers a retardation  (due to viscous drag) proportional  to its velocity. The piston and  the  cylinder  have  equal  cross  sectional  area  A.  p  3 (a)  M  M + m (c) ( MM + m )  (b)  M + m  M (d) ( MM+ m )  1/ 2  2  1  A gP 0 (d)  2 p  MV0  If a simple pendulum has significant amplitude (up to a factor  of  1/e  of  original)  only  in  the  period  between  t  =  0  s  to  t = t s. The fundamental frequency of the  air­column is now  f  3 f  (a)  (b)  (c)  2f  (d)  f  2  4  (2012)  p  4 (d)  p  6 A  mass  M.  open  at  both  ends.5  m  is  made  of  steel.  The  piston  is  slightly  displaced  from  the  equilibrium position and released.7  (c)  0. the piston executes  a  simple harmonic motion with  frequency  1  AgP 0  1  MV 0  (a)  2 p Ag P (b)  2 p  V M 0  0  1  V0 MP 0  (c)  2 p A2 g  4.  Two particles are executing simple harmonic motion  of  the  same amplitude  A  and frequency w along the  x­axis.5  Hz  (2013)  (a)  b  p  2 (a)  8. then t may be called the average life of the pendulum. In another 10 s it will decrease  to a times its original magnitude where a equals  (a)  0.25 N  (b)  4.7 × 10 3  kg/m 3  and 2. t ) = e .  When  the  piston is in equilibrium.  the  average  life  time  of  the  pendulum is (assuming damping is small) in seconds  (b)  (b)  (2011)  A  sonometer  wire  of  length  1.81  (d)  0.  then  (a)  A = x 0w 2 .  12  6  4  3  (2006)  23.  Two  springs.  the  maximum value of v upto which he can  hear the whistle is  (a)  30 ms –1  (b)  15 2 ms - 1  - 1  (c)  15 / 2 ms (d)  15 ms –1 .50 p.1 s.  A  whistle  producing  sound  waves  of  frequencies  9500  Hz  and  above  is  approaching  a  stationary  person  with  speed  v ms –1 . Then.  The  bob  of  a  simple  pendulum  is  a  spherical  hollow  ball  filled  with  water.08 2. t) = 0.0  0. is 4.  b =  (b)  a = 25. the lowest resonant frequency for this string is  (a)  10.45  Oscillations and Waves  11. b = p  2. d = p/4  (d)  A = x 0w 2 .  If a simple harmonic motion is represented by  its time period is  (a)  2pa  (b)  2p a  (c)  2p/a  d 2 x  dt 2  + ax = 0 . The average kinetic energy during  its motion from the  position  of  equilibrium to the  end is  (a)  2p 2  m a 2 u 2  (b) p 2 ma 2u 2  1  2 2  (c)  4 m a u  (d)  4p 2 ma 2u 2  (2007)  m 18.25 s  (b)  0.0  . After what time will its kinetic energy  by  75%  of  the total  energy?  1  s  1 s  1  1  (a)  (b)  (c)  s  (d)  s .05 Hz  (d)  1050 Hz.  till water is coming out.  (2006)  w  21. d =  3p/4  (b)  A = x 0 .0 s. The  amplitude of oscillation is gradually increased.04 1. b= (d)  a = (c)  a = p p  p p  (2008)  14. respectively.005 cos(ax – bt).  A  sound  absorber  attenuates  the  sound  level  by  20  dB.  (d)  2p / a  (2005)  .  There  are  no  other  resonant  frequencies  between  these two.  A coin  is placed on a  horizontal platform which undergoes  vertical simple harmonic motion of angular frequency w. They superpose to give beats.08 m and 2.  executing  simple  harmonic motion with an amplitude 7 mm.4 m/s. At the starting point of the motor cycle there  is  a  stationary  electric  siren.  It  is  observed  to  have  resonant  frequencies  of  420  Hz  and  315  Hz. If the person  can  hear  frequencies  upto  a  maximum  of  10000  Hz.  of  force  k 1  k 2  constants  k 1  and  k 2  are  connected  to  a  mass  m  as  shown.  A point mass oscillates along the x­axis according to the law  x =  x 0  cos  (wt  – p/4).  (2006)  24. u. If the wavelength and the time  period of the wave are  0.  A  plugged  hole  near  the  bottom  of  the  oscillating bob gets suddenly unplugged. The  intensity decreases  by a factor  of  (a)  100  (b)  1000  (c)  10000  (d)  10  (2007)  20. then a and b in appropriate units are  p (a)  a = 12.00 p.  The speed of sound in oxygen (O 2) at a  certain temperature    is 460 ms –1 .  The  maximum  velocity  of  a  particle.  (2008)  15.  A motor cycle starts from rest and accelerates along a straight  path at 2 m/s 2 .5 s  (c)  0. d =  –p/4  (2007)  16. If both k 1  and k 2  are made four  times  their  original  values. The velocity of sound in air is 300 ms –1 .  (2006)  25.  Three  sound  waves  of  equal  amplitudes  have  frequencies  (u – 1).  A wave travelling along the x­axis is described by the equation  y(x. a body oscillates simple harmonically  with a  period of 2 s.0 0.  the  frequency  of  oscillation  becomes  (a)  2f  (b)  f/2  (c)  f/4  (d)  4f  (2007)  19. (u + 1).  If  the  acceleration  of  the  particle  is  written as a =  Acos(wt + d).125 s  (2007)  17. The time at which the maximum  speed first occurs  is  (a)  0. the time period of oscillation would  (a)  remain unchanged  (b)  increase towards a saturation value  (c)  first increase  and  then decrease to the  original value  (d)  first decrease  and  then increase to the  original value  (2005)  26. The number  of  beats produced  per second  will  be  (a)  4  (b)  3  (c)  2  (d)  1  (2009)  12. The speed of sound in helium (He) at the same  temperature will be (assume both gases to be ideal)  (a)  330 ms –1  (b)  460 ms –1  (c)  500 ms –1  (d)  650 ms –1 .75 s  (d)  0.  How  far  has  the  motor  cycle  gone when the driver hears the frequency of the siren at 94%  of its  value when  the motor  cycle was  at rest?  (Speed of sound =  330 ms –1 ). The coin will  leave contact with  the platform for the  first time  (a)  at  the  highest position  of the platform  (b)  at  the  mean position of the platform  g (c)  for an  amplitude of  2  w  2  g (d)  for an  amplitude of  2  .  (a)  49 m  (b)  98 m  (c)  147 m  (d)  196 m  (2009)  13. b= .01 s  (c)  10 s  (d)  0.  Starting from the origin.  The  displacement  of  an  object  attached  to  a  spring  and  executing  simple  harmonic  motion  is  given  by  x = 2 × 10 –2  cos pt metre.  The  frequency  of  oscillation of the mass is f. During observation.5 Hz  (b)  105 Hz  (c)  1.  A particle of mass m executes simple harmonic motion with  amplitude a and frequency u. d =  –p/4  (c)  A = x 0w 2 .  (2006)  22.  A string is stretched between fixed points separated by 75 cm. The  period of  oscillation  is  (a)  100 s  (b)  0. E. (2003)  39. The frequency of the piano string  before increasing the tension was  (a)  (256 + 2) Hz  (b)  (256 – 2) Hz  (c)  (256 – 5) Hz  (d)  (256 + 5) Hz.  with a velocity one­fifth of the velocity of sound. The beat frequency  decreases to 2 beats per second when the tension in the piano  string is slightly increased. An external force    F (t) proportional to coswt (w ¹ w 0 ) is applied to the oscillator. (2003)  40.  then  (a) w 1  = w 2  (b) w 1  > w 2  (c) w 1  < w 2  when  damping  is  small  and w 1  > w 2  when  damping is large  (d) w 1  < w 2  (2004)  32. is maximum when x is maximum  (d)  P. but not simple harmonic motion with a period  2p/w  (d)  a periodic.)  are  measured  as  function  of  displacement  x.  (2003) .  An  observer  moves  towards  a  stationary  source  of  sound.w 2 )  (a)  w 2  .  The  potential  energy (P. where t is in second and x  in  meter. The percentage increase in the  time period of  the  pendulum of increased  length  is  (a)  11%  (b)  21%  (c)  42%  (d)  10%.  some  tape is attached on the prong of the fork 2.  executing  simple  harmonic  motion  is  (a) µ x  (b) µ x 2  (c)  independent of  x  (d) µ x 1/2  where x is the  displacement from  the mean  position.  then  what  was  the  original  frequency  of  fork  2?  (a)  196 Hz  (b)  204  Hz  (c)  200  Hz  (d)  202 Hz  (2005)  31. The speed of  the wave  is  (a)  2000 m/s  (b)  5 m/s  (c)  20 m/s  (d)  5p m/s.E.  (2003)  38.  Two particles A and  B of equal masses are suspended from  two  massless  springs  of  spring  constants  k 1  and  k 2 .  The  bob  of  a  simple  pendulum  executes  simple  harmonic  motion in water with a period t.  (2003)  37. What  relationship between t  and  t 0  is true?  (a)  t =  t 0  (b)  t =  t 0 /2  (c)  t = 2t 0  (d)  t =  4t 0 .  The  total  energy  of  a  particle.  the ratio of amplitudes of A and B is  (a)  k1 / k 2  (b)  k 2 /k 1  (c)  k2 / k 1  (d)  k 1 /k 2 .  (2004)  36. What is the  percentage  increase  in  the  apparent  frequency?  (a)  5%  (b)  20%  (c)  zero  (d)  0.  4  beats  per  second  are  heard.  are equal.  Now. is maximum when x = 0. but not simple harmonic motion with a period p/w  (2005)  29.) and total energy (T.  Two  simple  harmonic  motions  are  represented  pö æ by the equations  y1  = 0. When the tuning  forks are sounded again. 6 beats per second are heard.  (2003)  41.  In forced oscillation of a particle the amplitude is maximum  for a frequency w 1  of the force.  A tuning fork of known frequency 256 Hz makes 5 beats per  second with the vibrating string of a piano.  The  function sin 2 (wt)  represents  (a)  a simple harmonic motion with a period  2p/w  (b)  a simple harmonic motion  with a period p/w  (c)  a periodic. If the period of oscillation with the two    springs in series is T.w 2  0  0  1  m (c)  m(w 2  + w 2 )  (d)  w 2  + w 2  .  The length of a simple pendulum executing simple harmonic  motion is increased by 21%.  The time displacement of the oscillator will be proportional to  m 1  (b)  m(w 2  .  (2004)  0  0  33. while the energy is maximum  for  a  frequency w 2  of  the  force.  (2004)  34.E.  A mass M is suspended from a spring of negligible mass.). during oscillations. then  (a)  T =  t 1  +  t 2  (b)  T 2  =  t 12   +  t 22   –1  –1  –1  –2  (c)  T  =  t 1  +  t 2  (d)  T –2  =  t 1–2  (2004)    + t 2  .  When  two  tuning  forks  (fork  1  and  fork  2)  are  sounded  simultaneously. is zero when x = 0  (c)  K.46  JEE MAIN CHAPTERWISE EXPLORER  27.  The displacement y of a particle in a medium can be expressed  as:  y = 10 –6 sin(100t + 20x + p/4) m.1sin ç 100 pt + ÷ and y 2  = 0.E. the kinetic energy (K. If the mass is increased by  m.1cosp t.  respectively. Which  of  the  following statement is true?  (a)  K. while the corresponding period for  another spring is t 2. Then the  ratio of m/M  is  (a)  3/5  (b)  25/9  (c)  16/9  (d)  5/3. is maximum when x = 0  (b)  T.E. The  spring  is  pulled a  little  and then  released so that  the  mass  executes SHM of time period T. Neglecting frictional force of water and  given that the density of the bob is (4/3) × 1000 kg/m 3 .E.  (2005)  28.  3 ø  è The phase difference of the velocity of particle 1 with respect  to  the  velocity  of  particle  2 is  (a)  –p/3  (b) p/6  (c)  –p/6  (d) p/3. the time period becomes 5T/3.  35. if the  frequency  of  fork  1  is  200  Hz. while the period of oscillation  of the bob is t 0  in air.  A particle  at the end of  a  spring  executes  simple  harmonic  motion with a period t 1 .E. If the maximum velocities.  A particle of mass m is attached to a spring (of spring constant  k) and has a natural angular frequency w 0.5% (2005)  30.  A  body  executes  simple  harmonic  motion.  The frequency  of  the  unknown  fork is  (a)  286 cps  (b)  292 cps  (c)  294 cps  (d)  288 cps.  (c)  (b)  (b)  (b)  (d)  (b)  (c)  (c)  (c) 5.  A tuning fork arrangement (pair) produces 4 beats/sec with  one fork of frequency 288 cps.  44.  The  frequency u of  the  alternating  source  is  (a)  50  Hz  (b)  100  Hz  (c)  200  Hz  (d)  25  Hz.  (c)  (a)  (a)  (b)  (c)  (c)  (c)  (b)  (b)  4.  15.  8. The wire passes at its middle point between the  poles  of  a  permanent  magnet.47  Oscillations and Waves  42.  In a simple harmonic oscillator. and is cut into n equal  parts.  If a spring has time period  T. stands up.  23.  29.  18.  (2002)  48. the frequency of a  tuning fork  (a)  increases  (b)  decreases  (c)  remains same  (d)  increases or decreases depending on the material.  36.  51. potential energy is minimum  (d)  both kinetic and potential  energies are minimum. A little wax is placed on the  unknown fork and it then produces 2 beats/sec.  (d)  (d)  (b)  (a)  (c)  (a)  (a)  (c)  (b)  2.  47.  31. The  amplitude of  the  particle is  (a)  – 4  (b)  4  (c)  4 2  (d)  8.  25.  45.  (2002)  47.  30.  11.  (d)  (b)  (*)  (c)  (d)  (b)  (d)  (a)  (b)  3.  A metal wire of linear mass density of 9.  40.  (2002)  51.  13.  12. in ms –1  is  (a)  300  (b)  600  (c)  1200  (d)  200.  The displacement of a particle varies according to the relation  x = 4(cospt + sinpt).  19.  (2002)  50.  Tube A has both ends open while tube B has one end closed.  43.  and  it  vibrates  in  resonance  when  carrying  an  alternating  current  of  frequency u.  When temperature increases.  A child swinging on a swing in sitting position.  50.  42.  26.  34.  14.  7. Then the equation of the unknown  wave is  (a)  y = asin(wt + kx)  (b)  y = –asin(wt + kx)  (c)  y = asin(wt – kx)  (d)  y = –asin(wt – kx).  33.  32.  Length  of  a  string  tied  to  two  rigid  supports  is  40  cm.  27.  at the mean position  (a)  kinetic energy is minimum.  A wave y = a sin(wt – kx) on a string meets with another wave  producing a node at x = 0.  10.  35.  17.  49.  46.  52.  9.  37.  Maximum length  (wavelength  in cm)  of a  stationary  wave  produced  on it  is  (a)  20  (b)  80  (c)  40  (d)  120.  (2003)  43.  (b)  (b)  (a)  (d)  (a)  (c)  (a)  (b)  .  39. (2002)  Answer  Key  1.  38.  then the time period of each part will  be  (a)  T n  (b)  T /  n  (c)  nT  (d)  T.8 g/m is stretched  with  a  tension  of  10  kg­wt  between  two  rigid  supports  1 metre apart.  (2003)  44. potential energy is maximum  (b)  both kinetic and potential  energies  are  maximum  (c)  kinetic energy is maximum.  21.  48.  41.  22.2 x + p  metre. The speed of  the wave­motion.  then the time period of the  swing will  (a)  increase  (b)  decrease  (c)  reamains same  (d)  increases if the  child is long and decreases  if the child  is short.  (2002)  46.  (2003)  ( )  45.  otherwise  they  are  identical.  The displacement y of a wave travelling in the x ­direction is  given  by y = 10-4 sin 600t .  20.  3  where x is expressed in metre and t in second.  24.  The  ratio  of  fundamental  frequency of tube A and  B  is  (a)  1 : 2  (b)  1 : 4  (c)  2 : 1  (d)  4 : 1.  16.  (2002)  49.  (d)  (a)  (b)  (b)  (b)  (b)  (c)  (b)  6.  28.  (2002)  52.   Y = 2.48  JEE MAIN CHAPTERWISE EXPLORER  1..9 = e –5b/2m  . t = 5 s) its amplitude becomes  0. t = 15 s). u = 2  w 1  gP0 A  = 2p 2 p  MV0  3.2 Hz  7 4. r = 7.  (d) :  \ u = 1  T  2 L r A FBD of piston at equilibrium  Here  tension  is due  to elasticity  of wire  DL ù \ T = YA é ë L û P atm A + Mg = P 0 A  …(i)  FBD of piston when piston is pushed down a distance x  é As Y  = Stress  = TL  ù ë Strain  AD L û 1  Y DL  Hence. . we  get  u= ( P0 + dP) A .w 2 x  dt 2  We get  w 2  = gP0 A2 gP0 A 2  or  w =  MV0 MV0  Frequency.  u = 2 L r L Here.. \  f  =  v  2 l l  .5  7.( Patm A + Mg ) = M  d 2 x  dt 2  …(ii)  As  the  system  is  completely  isolated  from  its  surrounding  therefore the change is adiabatic.   (c)  5.01  1  2 ´ 1.5 m  L Substituting the given values.729  d 2 x  = . dV = Ax)  V0  Using (i) and (iii) in (ii).2 × 10 11  N/m 2 . it behaves closed  l  pipe  length  \ f ¢ = l .  For an adiabatic process  PVg  = constant \  Vg dP + Vg–1 PdV = 0  gPdV  or  dP = -  V gP0 ( Ax )  ( .9) 3  (Using (i))  = 0.7 × 10 3  kg/m 3  DL  = 0.  \ dP = -  2. .01.  2.  (d): When the tube of length l is open at  both ends. 2  v v  = = f  (Using(i))  2 l  l 4  2  ( )  2  l  2 ..  As per question  After 5 s (i.e. we get  2  2  gP A 2  gP A 2  M d 2 x  = -  0  x  or  d 2 x  = -  0  V 0  MV 0  dt dt Comparing it with standard equation of SHM.  T  is  the  tension  in  the  wire and m  is the  mass per length of the  wire  As m  = rA  where r is the density of the material of the wire and  A is  the  area of  cross­section  of the  wire.7 ´ 103  …(iii)  =  103  3 2  Hz = 178.  When the tube is dipped vertically in water  and half of it is in water.9A 0  = A 0 e –b(5)/2m  = A 0 e –5b/2m  0.(i)  where  v  is the  speed of sound in air.e. its amplitude becomes aA 0  = A 0 e –b(15)/2m  = A 0 e –15b/2m a = (e –5b/2m ) 3  = (0.  (d) : The amplitude of a damped oscillator at a given instant  of time t is given by  A = A 0 e –bt/2m  where A 0  is its amplitude in the absence of damping.  (c) : Fundamental frequency  of vibration  of wire  is  u= 1  T  2 L m  where  L  is  the  length  of  the  wire.(i)  After 10 more second (i.2 ´ 1011  ´ 0. b is the  damping constant.. L  = 1.   respectively.5  Wave  velocity.5  and each frequency will show 2 beats.  Coefficient of  t  b  Wave speed =  Coefficient of  x =  a 9.  (b)  :  The  wave  travelling  along  the  x­axis  is  given  by  y(x.  (d) :  T 1  = 2 p  (m + M )  .  The intensity waxes and wanes.vsound = - v obs  f f ¢ ö vsound æ .  (b) :  y ( x.  2  2 2  (19.25 N  2  T  or  T  = mw 2  m k Frequencies  Mean  Beats u + 1 and u  (u  + 0. New time period of  oscillation  7. linear mass density m  =  0.  2a 2 ´ 2  13.  Total number of beats = 4  12. t) = f(ax +  bt)  As there is positive sign between x and t terms.  (b) : The source is at rest. a = –w 2 x where w is a constant  =  2 1 2  1 11.  v = w (2 p / 0.04 0.49  Oscillations and Waves  6.  we get  w= k  ( u -2 u )t cos 2p ( u +2 u ) t. the new system is of  mass = (M +  m) attached  to the spring.005 cos(ax – bt).v obs  è f ø  330(0.04 kg  m –1  The  given  equation  of  a  wave  is ( )  t x  ù y = 0.u  = = 98 m..5 and u – 0.80 ms –1 .04 ´ M  ..(ii)  where T is the tension in the string and m is the linear mass  density  Equating equations  (i) and (ii).4p2  × x  Þ aT  = - 4 p . law of conservation of linear momentum  Mv 1  = (m + M)v 2  M(A 1w 1 ) = (m + M)(A 2w 2 )  ( .(i)  .u 2  is modulated by a wave of frequency  1 (rather the  2 difference  of  frequencies/2).50  û  Compare it with the standard  wave equation  y = Asin(wt –  kx)  we get.v obs )  \ f ¢ = f  sound vsound  f ¢ Þ ´ vsound = vsound - v obs  f f ¢ Þ ´ vsound . u + 0.  The resultant frequency obtained is  A 1  m + M  =  A2  M 8.  x y = 2 A cos 2p 2  2 p . u.. the  number of beats = u 1  ~ u 2 .  y 1  = A  cos 2pu 1 t  y 2  = A  cos 2pu 2 t  Superposing.80)  \ s = v .  (u  – 1).  v 1  = A 1w 1  and v 2  = A 2w 2 )  T 2  = 2 p  or  = A 1 (m + M )  w 2  = A2 M w1  ( m M+ M ) ´ TT  (Q  w = 2Tp and w = 2 Tp )  T= 2  10.1  = . t) = 0.  (a) : The  given  sources  of  sound  produce  frequencies.. .  w = 2p rad s -1; k =  2 p rad m -1  0..  acceleration.04 0.( a x + bt )  Comparing equation (i)  with standard equation  y(x. t ) =  e . hence wave  travel in –x  direction.94 – 1) = –vobs Þ  vobs  = 330 × 0..  1 2 1 2  u1 + u 2  and this wave  2 u .5)  Also v = T  m  . .  (b) : For a simple harmonic motion.02sin éê2 p ú ë 0.  T a = .. .  (v .(i)  k When a  mass m is placed on  mass M.  One should detect three frequencies.  For two sources of  frequencies u 1  and u 2 .05  )  2  0.04  ) 2 p  ( 0.  x T  T The period of oscillation T is a constant.  1  2 = 6.  (b)  2 p ( 0.04)  = m s -1  k (2p / 0. one  gets (Using (i) and (ii))  2 + bt 2  + 2 ab xt )  2  = e . u and (u + 1).  (a) : Here. For a cosine curve (or sine  curve).5  1  (u  + 1) and (u –  1) u  2  Total number of beats = 4..( ax 2  \  aT  is a constant.5) Hz  1 u and u – 1 u –  0. 1 beat and 1 beat per  second.  Using.(ii)  k Consider v 1  is the velocity of mass M passing through mean  position and v 2  velocity of mass  (m + M) passing through mean position.06 = 19. the observer is moving away from  the  source. w  =  angular  frequency.00 p .p ÷ö 4 ø  è Acceleration a =  A cos (wt + d)  Velocity  v  =  dx  dt æ v = .  0..  Option  not given.L 2  = 10 log çè I ÷ø . b = p  14.  (a) : Given :  x = x0 cos çæ wt . we get  3 p A = x0 w2 . force will be  towards  mean  position  and  so  it  will  be  downwards.0 \ a = 25.50  JEE MAIN CHAPTERWISE EXPLORER  1  2 2 2  ma w sin  w t 2  Average kinetic energy < K >  1  =  < ma 2 w2 sin 2  wt >  2  2 p Therefore  a = k = ..  Let us choose f = 0 \  x(t) = acoswt  Velocity of a  particle  v = dx  = ..  (b)  :  For  a  particle  to  execute  simple  harmonic  motion  its  displacement at any time t is given  by  x(t) = a(cos wt + f)  where.x0 wsin ç wt .00 = 25. sin pt = 1  p p sin pt = sin  pt =  or  or  2  2  1  t = s = 0.  maximum  acceleration (aw 2 ) and so the maximum force (maw 2 ) will  be at extreme positions. f = phase constant.  æ 2 p ö 21..  1 k eff  1  k1 + k 2  = 2p m 2 p  m As k 1  and k 2  are increased four times  New  frequency. At  lowest position. force will be towards mean position and  so it will be upwards. (iii)  éQ  < sin 2 q > = êë k 2  . (i)  .  (b) : Given : displacement x =  2 ×  10 –2  cos pt  .  1 2 1  2  Kinetic  energy  = 2 mv = 2 m ( a w cos w t )  Total  energy  E = 1 ma 2 w 2  2  .  I 2  100  20.5 s.  (a)  :  In  the  given  figure  two  springs  are  connected  in  parallel.08 0.  æ I  ö æ I  ö 19.  The  value  of the  velocity given for O 2  is  quite high.  (b)  :  During  simple  harmonic  motion.10 log çè I ÷ø  0 0  æ I  ö or  D L = 10log çæ I 1  ÷ö or  20 dB = 10 log ç 1  ÷ I è I 2  ø  è 2  ø  I  I  or  10 2  =  1  or  I 2  =  1  .ö = x0 w 2 cos é p + ( wt .  4  16.4  –2  =  10  sec  =  0.  = g for O 2  = 1 + 2/5 =  1..  l  2  p p \ a= = Þ a = p ´ 100.. (iv)  Velocity v = dx  = -2 ´ 10-2 p sin p t  dt For the first time when v = v max  .08  m..a w sin w t  dt Kinetic energy of a  particle is  K =  1 mv2  2  1 ù 2 úû [Q  w = 2 pu ] 18.  (a) :  L 1  = 10 log  ç 1  ÷ ;  L 2  = 10 log ç 2  ÷ è I 0  ø  è I 0  ø æ I1 ö æ I 2 ö \  L1 . At highest position.  (*) : v = gP = r = 1  m w 2 a 2 < sin 2  w t >  2  1 1  = m w 2 a 2  æ ö è 2 ø  2 =  gRT  M 1  2 ma (2 pu ) 2  4  = p 2  ma 2 u 2 .  1  4( k1 + k 2 )  = 2 f  2 p  m m k 1  f  = f¢= . Therefore the effective  spring constant is given by  k eff  = k 1  + k 2  Frequency  of  oscillation. (ii)  . d =  .  22.  3 4 7 * The value of the speed of sound in He should have been  965  m/s  and  that  of  O 2.    about  320  m/s. (i)  (using (i).p ÷ö 4 ø  è dv  Acceleration  a =  dt p p 2 æ = . The  coin will leave  contact will the platform  for the first time when m(aw 2 ) ³ mg at the lowest position  of  the  platform.  (c)  :  In  vertical  simple  harmonic  motion.00 p.. This is opposite to weight direction  of the  coin.4; g  for He = 1 + 2/3  = 5/3  v 2  æ g He  32  ö =ç ´ ÷ ´ 460  v1  ç 4  g O 2  ÷ è ø  5 1 32 = 460 ´ ´ ´ ´ 5  = 1420m/s.  a  =  amplitude..  15..  As l  = 0.01  sec.x0 w cos wt .04 4 2  p w=bÞ =b Þ p 2.) ù   êë è 4ø 4  úû = x0 w2 cos éwt + 3 p ù êë 4  úû  Compare (iv) with (ii).  or  2  17.  (b)  :  Maximum  velocity  v m  =  a w = a ç T ÷ è ø -3  2 pa  22  (7 ´ 10 )  T  = = 2 ´ ´  \  vm  7 4. .1 ´ p ) sin p t  dt \  \  = (0.  (c) : For a pendulum. \  T  would  first  increase  and  then  decrease  to  the  original  value.  (b)  :  By  Doppler's  effect  u¢ v s + v O  = (where  v s  is  the  velocity  of  sound)  u  vs  = v + (v / 5)  6  = v 5  \  Fractional  increase  = u ¢ .  10000 300  = 9500 300 - v Þ (300 . centre of gravity is restored  to  centre. w 1  = w 2  .  29.1ö = æ 6 .  5 30.  The  centre  of  gravity  of  system  is  at  centre of sphere  when  hole is plugged.w 2 )  .= -  .1 ´ p ) cos çæ pt + p ÷ö 2 ø è p p p D f = .  a  decrease  in u 2  increased  beats/sec.1 ö = 1  è u ø è5 ø 5 u 100  \  Percentage  increase  = = 20%.  When unplugged.  Centre  of  gravity  goes  on  descending.  0  T= 33.  (a)  :  In  case  of  forced  oscillations  (i)  The  amplitude  is  maximum  at  resonance \  Natural  frequency  =  Frequency  of  force  = w 1  (ii) The  energy  is  maximum  at  resonance \Natural  frequency  =  Frequency  of  force  = w 2 \  From  (i)  and  (ii).  But  this  is  not  given  in  question  If u 2  =  196 Hz.  25.  (b)  :  Let  the  successive  loops  formed  be  p  and  (p  +  1)  for  frequencies  315  Hz  and  420  Hz p T  pv  \  u = 2l m = 2 l pv  ( p + 1) v  = 315  Hz  and  =  420 Hz  \  2 l 2 l ( p + 1) v pv  = 420 - 315  or  2l 2 l v 1 ´ v  = 105 Þ  = 105 Hz  or  2l 2 l p  =  1  for  fundamental  mode  of  vibration  of  string.cos2 wt 1 cos2 wt  = -  2 2 2  It  is  a  periodic  motion  but  it  is  not  SHM \  Angular speed  = 2w \  Period  T =  2 p p 2 p = = angular speed  2 w w  Hence  option  (d)  represents  the  answer.  This  is  given  in  the  question  when  beats  increase  to  6 \  Original  frequency  of  second  fork  =  196 Hz.  (b)  :  In  case  of  forced  oscillations.  3 2 6 2  28.  water  drains  out.  w a  d  pö æ 27.  32.u = æ u ¢ .  total  energy  = 1  2 2  ma w  2  Total  energy  is  independent  of  x.  (c)  :  Under  simple  harmonic  motion.  +4 Hz  204 Hz  (u2)  – 4 Hz  196 Hz  (u 2)  200 Hz  (u 1 ) As  mass  of  second  fork  increases. \  Lowest  resonant  frequency  =  105  Hz.  (d)  :  u = v - v s  where  v s  is  the  velocity  of  sound  in  air.  31. \  Length  of  pendulum  first  increases.  (d)  :  y = sin  wt = 1 . u 2  decreases.  26.  6 w 6(2 p / T ) 6 ´ 2 p  6  (Kinetic energy) =  23.51  Oscillations and Waves  Q  or  or  \  or  75  ( E )  100  1 75 1  2 2  ma 2 w2 cos 2 wt = ´ ma w  2 100 2  3  cos2  wt = 3  Þ cos wt = = cos p 4 2 6  p wt =  6  p p 2p 1  t  = = = = sec.1 ´ 100 p ) cos ç 100 pt + 3 ÷ è ø  d  v2 = ( y2 ) = ( -0.  a  decrease  in u 2  decreases  beats/sec.  then  decreases  to  original  value.  T  = 2 p  l  where l is measured upto  g centre  of  gravity.  (a)  :  Let  the  two  frequencies  be u 1  and u 2 u 2  may  be  either  204 Hz  or  196 Hz.  (d)  :  Standard  differential  equation  of  SHM  is  d 2 x  + w2 x  = 0  dt 2  2  Given  equation  is  d 2 x  + ax  = 0  dt \ w 2  = a  or w= a 2 p 2 p = .w2  1  x  is  proportional  to  m( w2 .  x  =  asin(wt  + f)  where  a = \  F0 / m  w0 2 .  If u 2  =  204 Hz.  (c)  :  v1 = dt ( y1 ) = (0.  When the bob becomes empty.v ) = 285 Þ  v = 15 m /s .  v s  u¢ 24.   T = 2 p  M / k 5 T M + m  Finally.  42.  beats  per  second  decrease..10 1  = = \  Fractional  change  = T 10 10  \  Percentage  change  =  10%.  (c)  :  x  =  4(cospt  +  sinpt)  é1 ù 1  =  4 ´ 2 ê cos pt + sin p t ú 2 2  ë û  p p é ù or  x = 4 2 êsin 4 cos pt + cos 4 sin pt ú ë û  p æ ö = 4 2 sin ç pt + ÷ 4 ø  è Hence  amplitude  = 4 2 .  vm  = a  25  M M + m  =  9  k k 9  m  +  9  M  =  25  M  m  16  =  .  38. 44.  39. This is not given  in  the  question.52  JEE MAIN CHAPTERWISE EXPLORER  34.s )  l r-s = 261 Hz  æ 1  k  ö 2 pa 1 = (2 pa ) æç ö÷ = (2 pa ) ç ÷ T T è ø è 2 p m ø  1 10 ´ 9.8 ´ 10 -3  43.  (b)  :  Given  wave  equation  :  y = 10-6 sin çæ 100t + 20 x + p ÷ö m  4 ø  è Standard  equation  :  y  =  a  sin  (wt  +  kx  + f)  Compare  the  two \ w =  100  and  k  =  20 w = 100 Þ 2 pn  = nl = v = 5  \  k 20 2p / l \  v  =  5  m/s. This  is  given  in  the  question.  u = 2 l m lr g  r t  = ´ = t0  g (r .  1  T  For  vibration  of  wire.  u = 2 l m  When  tension  T  increases.8 100  = = 50 Hz.  (c)  :  Initially.  (b) :  When  springs  are  in  series.c.  (d)  :  Let  the  lengths  of  pendulum  be  (100l)  and  (121l) T ¢ = 121 = 11  \  T 100 10  T ¢ – T  11 .  (c)  :  The  possible  frequencies  of  piano  are  (256  +  5) Hz  and  (256  –  5) Hz.  (i)  Due  to  upthrust  of  water  on  the  top.  (a) :  Kinetic  energy  is  maximum  at  x  =  0.s )  \  or  ( 4 ´ 1000 / 3  = 2  4000 - 1000  3 )  t  =  t 0  ×  2  =  2t 0 ..  37.  \  u = 2 ´ 1 2  9. frequency of vibration of wire  become  equal  to  frequency  of  a.(ii) \  g (r .  (a) : At resonance...2 x + p ÷ö m  3 ø  è Standard wave  equation :  y  =  asin(wt  –  kx  + f)  Compare  them  Angular  speed  = w  =  600  sec –1  Propagation  constant  =  k  =  2  m –1 .  3  = 2 p k 5  M M + m  ´ 2p = 2 p  \  3  k k or  35. beats/sec increase.  or  +5 Hz  256 Hz  – 5 Hz  36..  Hence  frequency  of  piano  =  (256  –  5) Hz.  its  apparent  weight  decreases  upthrust  =  weight  of  liquid  displaced \  Effective  weight  =  mg  –  (Vsg)  =  Vrg  –  Vsg  Vrg¢ =  Vg(r – s).  (ii)  If  251 Hz  increases  due  to  tension.  t 1  = 2 p  m  k1  For  second  spring  t 2  = 2p m  k2  2 2  æ k + k  ö 4 p m 4  p m  2 2 t1 + t2  = + = 4 p 2 m ç 1 2  ÷ \  k1 k2 è k1k2  ø  2  or  or  é m ( k1 + k 2 ) ù t12 + t 2 2  = ê 2 p k1k 2  úû  ë t12 + t2 2 = T 2 ..  (c)  :  Maximum  velocity  under  simple  harmonic  motion  v m  =  aw \  vm  = 251 Hz  1  T  For  piano  string..  or  M 9  41.  (a)  :  Given  wave  equation  :  y = 10 -4 sin çæ 600t ..  where s is  density  of  water  ær-sö g¢ = g ç ÷ or  è r ø l r t = 2 p l / g ¢ = 2 p ...  m m a2 k1  40.  Q  k  m (v m ) A  =  (v m ) B \  a1 or  k1 k a k  = a 2  2 Þ  1 = 2  ... u  increases  (i)  If 261 Hz increases...  k  = k1k 2  k1 + k 2  For  first  spring.  (c)  :  t0  = 2 p  l / g .   Consider  option  (b)  Stationary  wave  :  292 cps  288 cps  T ¢ = 2 p m  nk 48.  (b)  :  The  wax  decreases  the  frequency  of  unknown  fork.  49.  Hence  unknown  fork  has  frequency  292 cps.53  Oscillations and Waves  w 2 pu = = ul = velocity  k 2 p / l w 600 = 300 m/sec  .  T = 2 p  l / g .  kinetic  energy  is  maximum and potential energy is minimum at mean position.  \  velocity = = k 2  45.  T  = 2 p  m  k For  each  piece.  46.  It  is  given  that  the  beats  decrease  to  2  from  4.  The  option  is  not  acceptable.  When the child stands  up.  l  increases  Hence  frequency  decreases.  (c)  :  In  tube  A. Wax reduced  292 cps  and  so  beats  should  decrease.  k  n n l max  = 40 Þ l max  = 80 cm.  Y  is  not  zero.  Y  =  a sinwt  –  asinwt  =  zero  This  option  holds  good  Option  (c)  gives  Y  =  2asin(wt  –  kx)  At  x  =  0.  (b)  :  Time  period  will  decrease.  2 50. This frequency is ruled out.  (b)  :  When  temperature  increases.  The  possible  unknown  frequencies  are  (288  +  4)cps  and  (288  –  4) cps.  B AN  l A  = l 2  N l B  = l  4  .  51. l A  =  2l  AN  AN  In  tube  B.  52.  Y  is  not  zero  Option  (d)  gives  Y  =  0  Hence  only  option  (b)  holds  good.  (b) :  For  a  spring.  47. the  centre of   gravity is shifted  upwards  and  so  length  of  swing  decreases.  Y  =  asin(wt  –  kx)  –  asin(wt  +  kx)  At  x  =  0.  (b)  :  + 4 cps  – 4 cps  284 cps  Wax reduces 284 cps and so beats should increases. l B  =  4l v v  \ u A  = l  = 2 l A  N  l  v v  u B  = = l B  4 l u A  2  \  u = 1 .  spring  constant  =  nk \  \  T ¢ = 2 p m ´ 1  =  T  .  (b)  :  Consider  option  (a)  Stationary  wave  :  Y  =  asin(wt  +  kx)  +  asin(wt  –  kx)  when  x  =  0.  (c)  :  In  a  simple  harmonic  oscillator. It is not  given in the question.  The  disc rotates  about an  axis  perpendicular to its plane and passing through its centre with  an angular velocity w.  (2013)  Potential difference  V in volts  11  E  E  (b)  (a)  R  r  R  r  .  0  Statement 2 : The electric field at a distance r(r <  R) from  rr the  centre of the sphere is  3 e  .  Statement  2  is  the  correct  explanation  of  Statement  1. the net force acting on the particle  is  proportional  to  1  1  (a)  -  y (b)  y  (c)  –y  (d)  y  (2013)  3.  (b)  Statement  1  is  false. the electric field E is plotted as a function of distance from  the centre. choose the one that best  describes the two statements.  Statement 2. at the surface of the sphere and also at a point outside  the  sphere. are kept at x = – a and x = a  q  on  the  x­axis.  (b)  B  R  5.  (d)  Statement  1 is  true.  0  (a)  Statement  1  is  true.  20  15  10  5  Time t  in seconds  The figure shows an experimental plot for discharging of a  capacitor in an R­C circuit.  Two charges. The  electric potential  at infinity is zero. Of the four  choices given after the  statements. A  particle  of  mass  m  and  charge  q0  =  is  2  placed at the origin. The electric potential at the  point O lying at a distance L from the end A is  6. If  we keep both the amount of charge placed on the disc and its  angular velocity to be constant and vary the radius of the disc  then the variation of the magnetic induction at the centre of  the  disc will be represented  by the  figure  (a)  B  R  (c)  B  R  R  This question has Statement 1 and Statement 2.  (c)  Statement  1  is  true.54  JEE MAIN CHAPTERWISE EXPLORER  CHAPTER  ELECTROSTATICS 1. As a result of this uniform charge distribution  there is a finite value of electric potential at the centre of the  sphere. The graph which would correspond to the above  will be  (2012)  (d)  B  25  0  50  100  150  200  250  300  7.  Q ln 2  (a)  4 p Π L 0  3 Q  (c)  4 p ΠL 0  4. The time constant t of this circuit  lies between  (a)  0 and 50 sec  (b)  50 sec and  100  sec  (c)  100 sec and  150 sec  (d)  150 sec and  200  sec  (2012)  In a uniformly charged sphere of total charge Q and  radius  R.  Statement  2  is  true. is  true. its potential  energy changes by  3 e  .  Statement 1 : When a charge q is taken from the centre to the  qr surface of the sphere.  Q ln 2  (b)  8 p Π L 0  Q  (d)  4p ΠL ln 2  0  A charge Q is  uniformly distributed over the surface of non­  conducting disc of radius R.  Two capacitors C 1  and C 2  are charged to 120 V and 200 V  respectively. If charge q 0 is given a small displacement  (y < < a) along the y­axis. Then  (a)  9C 1  = 4C 2  (b)  5C 1  =  3C 2  (c)  3C 1  = 5C 2  (d)  3C 1  +  5C 2  = 0  (2013)  2. Statement  2  is  not  the  correct  explanation  of  Statement  1.  (2012)  A  charge Q is uniformly  distributed  over  a  long  rod AB  of  length L as shown in the figure. each equal to q. It is found that by connecting them together the  potential  on each one can  be  made  zero.  An insulating solid sphere of radius R has a uniformly positive  charge density r.  Statement  2  is  false.  Statement  2  is  true. As a result of this rotation a magnetic  field  of induction  B  is obtained  at  the centre  of  the disc.   9. Which  of  the  following  graphs  most  closely  represents  the  electric  field  E(r)  produced  by  the shell in the range 0 £ r < ¥.25 pF  (b)  1.  Two  identical charged  spheres  are  suspended  by  strings  of  equal  lengths.  A  charge  Q  is  placed  at  each  of  the  opposite  corners  of  a  square.  Let there be a spherically symmetric charge distribution with  5  r  charge density varying as r(r ) = r0  -  upto r = R.  One of the dielectrics has dielectric constant k 1 = 3 and thickness  d/3  while  the  other  one  has  dielectric  constant  k 2  =  6  and  thickness 2d/3.  then the  Q/q  equals  1  (a)  - 2 2 (b)  –1  (c)  1  (d)  -  2 (2009)  r  E(r )  (c)  (2008)  (d)  O  (2010)  R  R  r  O  R  r  18.  Let  P( r ) = 0  (2010)  0  ^  j  12.6 g cm –3 . As a  result the charges approach each other with a velocity v.  (d)  Statement­1 is false.24 × 10 –16  J  (2009)  E  (c)  (d)  R  r  R  r  (2012)  8.5 pF.  Statement­1: For a charged particle moving from point P to  point  Q.  If the  net electrical force on Q  is zero. When suspended in a liquid of density 0.8 g cm –3 .2 2  j  (d)  .  (2008) .R 0  0  ( (c) ( )  r r  5  r  4e  ( 3  R )  0  0  )  16. the  angle  remains  the  same.60 × 10 –17  J  –16  (c)  –2. The charge  begins to leak from both the spheres at a constant rate.  the  magnitude of electric field is  Q  Qr 1 2 Qr 1 2 (a)  0  (b)  4 pe  r 2  (c)  (d)  3 pe 0 R 4  0 1  4 pe 0 R 4  (2009)  15.  Two  identical  charged  spheres  suspended  from  a  common  point by two massless strings of length l are initially a distance  d(d < < l) apart because of their mutual repulsion.  A  thin  spherical  shell  of  radius  R  has  charge  Q  spread  uniformly  over  its  surface. The  electric field at a distance r (r < R) from the origin is given by  r0 r  5  r  4 pr0 r  5  r  (a) 3e  4  . Statement­2 is true. The  strings  make an  angle  of  30° with each  other. Then the charge density inside the ball is  (a)  –24pae 0 r (b)  –6ae 0 r  (c)  –24pae 0  (d)  –6ae 0  (2011)  10.  The  net  r  O  field  E  at  the centre O is  ^  ^  q  q  j  (b)  (a)  2 p 2 e  r 2  j  2 2  4  p e  r 0  0  ^  ^  q  q  (c)  .  (c)  Statement­1 is true. The separation between its plates is d.24 × 10  J  (d)  2. b are constants.R (b) 3e  3  . Capacitance of the capacitor is  now  (a)  20. the  dielectric constant of the  liquid is  (a)  1  (b)  4  (c)  3  (d)  2  (2010)  11.  (2009)  17.  This question contains Statement­1 and Statement­2.2 2  j  4 p e 0 r 2 p e 0 r E(r )  E(r )  (a)  (b)  O  R  r  O  E(r )  ^  i  13. Of the  four choices given after the statements. A charge q is placed at each of the other two corners. The  space  between  the  plates  is  now  filled  with  two  dielectrics. Statement­2 is true; Statement­2 is  the  correct explanation  of  Statement­1. The work done in moving 100 electrons  from P to  Q  is  (a)  –9.  Two points P and Q are maintained at the potentials of 10 V  and –4 V respectively. and 4  R r(r) = 0 for r > R.  A thin semi­circular  ring of  radius  r  has  a  positive  charge  q  distributed  uniformly  over  it. choose the one that  best describes the two statements. For  a point ‘p’  inside the sphere at distance r 1  from the centre of the sphere. where r is the distance from the origin.  A  parallel  plate capacitor  with  air  between  the  plates  has  a  capacitance of 9 pF. Statement­2 is true; Statement­2 is  not  the  correct  explanation  of  Statement­1. Statement­2 is false  (b)  Statement­1 is true.  (a)  Statement­1 is true. the  net work  done  by an  electrostatic  field  on  the  particle  is  independent  of  the  path  connecting  point  P  to  point  Q.  Statement­2: The net work done by a conservative force on  an  object moving along  a  closed loop is zero.55  Electrostatics  E  14.60 × 10 –17  J  (b)  9.8 pF  (c)  45 pF  (d)  40. Then  as  a function of distance x between  them  (a)  v µ x –1/2  (b)  v µ x –1  (c)  v µ x 1/2  (d)  v µ x (2011)  The electrostatic potential inside a charged spherical  ball is  given by f = ar 2  + b where r is the distance from the centre;  a. where r is the distance from  the centre  of  the  shell?  ( (d) )  4 r r  5  r  3e  ( 4  R )  Q  r  be  the  charge  density  distribution  for  a  p R 4  solid sphere of radius  R and total charge Q.  If  density  of  the  material  of  the  sphere is 1.   A electric dipole is placed at an angle of 30º to a non­uniform  electric  field. The potential difference between  the centers of the two rings is  ù Q  é 1 1  ú (a)  zero  (b)  4 pe ê R  ê ú 0  ë R 2 + d 2  û  QR  (c)  4 pe 0 d 2  ù Q  é 1 1  ú (d)  2 pe ê R  2 2  0  ê R + d úû  ë (2005)  30.  Two spherical conductors A and B of radii 1 mm and 2 mm  are separated by a distance of 5 cm and are uniformly charged.  then  (a)  E changes.  The  potential  at  a  point  x  (measured  in  hm)  due  to  some  charges situated on the x­axis is given  by  V(x) = 20/(x 2  –  4) volt  The electric field E at  x =  4 mm is  given  by  (a)  (10/9) volt/mm and  in the  +ve x  direction  (b)  (5/3) volt/mm and  in the  –ve x  direction  (c)  (5/3) volt/mm and  in the  +ve x  direction  (d)  (10/9) volt/mm in the  –ve x direction  (2007)  21. plate 2  is at a higher potential).65 × 10 6  m/s  12  (c)  7. The surface charge  +  q density s of the sheet is proportional to  +  S  (a)  sinq  +  (b)  tanq  +  +  (c)  cosq  B (d)  cotq  (2005)  .  If  the  spheres  are  connected  by  a  conducting  wire  then  in  equilibrium condition.  If  the  charges on A and B are interchanged with  those on  r  D and C respectively.  A parallel plate capacitor is made by stacking n equally spaced  plates  connected  alternatively.  Two thin wire rings each having a radius R are placed at a  distance d apart with their axes coinciding.  The  potential  situated  at difference between  the points  A and B will  be  (a)  4.  (2006)  26.56  JEE MAIN CHAPTERWISE EXPLORER  19.  Charges  are  placed  on  r  the  vertices  of  a  square as shown.02 × 10  m/s  (d)  1.  A  parallel  plate  condenser  with  a  dielectric  of  dielectric  constant K between the plates has a capacity C and is charged  to a potential  V volt. 0) of X – Y co­ordinate system.1  m  and  can  be  treated  as  infinitely large. An electron is released from rest on the inner  surface  of  plate  1.  An  electric  charge  10 –3 mC  is  placed  at  the  origin  (0. The  dielectric  slab  is slowly  removed  from between  the  plates  and  then  reinserted. If the  temperature  of  the  block  is  raised  by DT.0)  respectively. The  dipole  will  experience  (a)  a  torque only  (b)  a  translational force  only in the direction  of  the  field  (c)  a  translational  force  only  in  a  direction  normal  to  the  direction  of  the  field  (d)  a torque as well as a translational force. The ratio of the energy stored  in the capacitor and the  work done  by the battery will be  (a)  1/2  (b)  1  (c)  2  (d)  1/4  (2007)  23. V changes  r  (c)  both E  and V change  r  (d)  E and V remain unchanged  q  q  A  B  D  – q  C  – q  (2007)  22. (i. The location of a  point  on the x  axis  at which  the net electric field due to these two point charges is zero is  (a)  8L  (b)  4L  (c)  2L  (d)  L/4  (2005)  31.11 × 10 –31  kg)  (a)  32 × 10 –19  m/s  (b)  2. the ratio of the magnitude of the electric  fields at the surface  of spheres A and B is  (a)  1 : 4  (b)  4 : 1  (c)  1 : 2  (d)  2 : 1.  A fully charged capacitor has a capacitance C. The  Y  0. What  is  its  speed  when  it  hits  plate  2?  (e = 1.  m e  = 9.  The  net work  done by the system  in this  process  is  1  (a)  zero  (b)  2  (K – 1) CV 2  CV 2 ( K - 1)  (c)  (d)  (K – 1) CV 2  (2007)  K 20. It is discharged  through a small coil of resistance wire embedded in a thermally  insulated block of specific heat capacity s and mass m. The  charges  on  the two rings are +Q and –Q.6 × 10 –19  C.87 × 10 6  m/s.  If  the  capacitance  between  any two adjacent plates is C then the resultant capacitance is  (a)  C  (b)  nC  (c)  (n – 1)C  (d)  (n + 1)C  (2005)  29.  (2006)  27.e. (2006)  25.5 volt  (b)  9  volt  (c)  zero  (d)  2  volt  (2007)  ( )  24. which makes an angle q with  +  +  a large charged conducting sheet P. as  P +  shown in the figure.  A  charged  ball  B  hangs  from  a  silk  +  thread S.1 m  X  1  2  plates  are  separated  by  d  =  0. Let  E  be the electric field  and  V  the  potential  at  the  centre.  Two  insulating  plates  are  both  uniformaly  charged  in  such a way that the potential  difference  between  them  is  V 2  – V 1  = 20 V. 2  and  (2. Two points  A and B are  2.  the  potential  difference  V across  the  capacitance  is  msD T  2msD T  (b)  (a)  C C (c)  2mC D T  s (d)  mC D T  s (2005)  28. V remains unchanged  r  (b)  E  remains unchanged.  A  battery  is used  to  charge  a  parallel  plate  capacitor  till the  potential  difference  between  the  plates  becomes  equal  to the  electromotive force of the battery.  Two point charges +8q and –2q are located at x = 0 and x = L  respectively.   (a)  (d)  (c)  (a)  (b)  (d)  (a)  1  2  (d)  2 n CV  .  13.  (2004)  34.  (2003)  40.  23.  with  a  speed  v.  28.  32. If q were  given a speed  2v.  40.      (c)  (b)  (a)  (a)  (b)  (b)  (c)  (*)  2.  the  closest  distances  of  approach  would  be  (a)  r  (b)  2r  (c)  r/2  (d)  r/4.  then  brought  in  contact  with  C  and  finally  removed  away  from  both.  (b)  (c)  (b)  (c)  (d)  (b)  (d)  6.  then the energy stored  is equal to  y –q3 b2 2 Q  (a)  4 pe  R 0  4.2 cos q a q 2  q 3  + sin q  b 2 a 2  a q 2  q 3  + 2 cos q  a q  2Q 2 q  (b)  4pe R .  41.  (2002)  41.  36.  (d)  (d)  (c)  (a)  (c)  (d)  (a)  5.  (2003)  37.(1 + 2 2)  (b)  (1 + 2 2)  4  4  Q (c)  .  If  the  system  is  in  equilibrium the  value of q is  Q Q (a)  .  (c)  (d)  (d)  (d)  (c)  (d)  (b)  .  19.  A thin spherical conducting shell of radius R has a charge q.  A sheet of aluminium foil of negligible thickness is introduced  between  the  plates  of  a  capacitor.  4 pe 0  R 38.  Two  spherical  conductors  B  and  C  having  equal  radii  and  carrying equal charges in them repel each other with a force  F  when  kept  apart  at  some  distance.  21.  (2002)  Answer  Key  1.  14.  31.2 sin q  b a (2003)  36.  Capacitance (in F) of a spherical conductor with radius 1 m is  (a)  1.  35.  11.  30.  +q 2  and  –q 3  are  placed  as  shown  in  the  figure.  10.  A charged particle q is shot towards another charged particle  Q  which  is  fixed. 2 J of work is  done.  20.  If the electric flux entering and leaving an enclosed  surface  respectively is f 1  and f 2.  22.  (2004)  35.  15. the electric charge inside the surface    will be  (a)  (f 2  – f 1 )e 0  (b)  (f 1  + f 2)/e    0  (c)  (f 2  – f 1 )/e 0  (d)  (f 1  + f 2 )e 0 .  It  approaches  Q  upto  a  closest distance r and then returns.  A charged particle q is placed  at  the  centre  O  of  cube  of D O length  L  (ABCDEFGH).5 V.  (2003)  39.  (b)  (a)  (d)  (a)  (d)  (b)  (b)  (a) 3.  38. The  x ­component of the force on –q 1  is proportional to  (a)  (b)  (c)  q 2  q 3  .  Three  charges  –q 1 .4 pe  R 0 0  2 Q q  (c)  4 pe R + 4 pe  R 0 0  (d)  (q + Q ) 2  .(1 + 2 2)  2  (d)  Q (1 + 2 2) .  (2003)  42.  12.  18. The  electrostatic potential at a point P at a distance R/2 from the  centre  of the shell  is  1 nCV 2  (c)  CV 2  2  E 43.1 V  (b)  8 V  (c)  2 V  (d)  0.  (2002)  (a)  CV  (b)  b +q2 x –q1 b2 q 2  q 3  (d)  2 .  Another  charge  Q  is  placed  at  the  centre  of  the  shell.  2  (2004)  33.  If there are n capacitors in parallel connected to V volt source.  (2002)  F C q G B L (2002)  44.1 × 10 –10  (b)  10 –6  –9  (c)  9 × 10  (d)  10 –3 .  The  new  force  of  repulsion  between  B  and  C  is  (a)  F/4  (b)  3F/4  (c)  F/8  (d)  3F/8.  A  third  spherical  conductor having same radius as that of B but uncharged is  brought  in  contact  with  B.  26.  9.  16.  27.  7.  17.  The  capacitance  of  the  capacitor  (a)  decreases  (b)  remains unchanged  (c)  becomes  infinite  (d)  increases.  If a  charge q is  placed at the centre  of the line  joining  two  equal charges Q such that the system is in equilibrium then  the  value of q  is  (a)  Q/2  (b)  –Q/2  (c)  Q/4  (d)  –Q/4.  On moving a charge of 20 coulomb by 2 cm. then  the  potential difference between the  points  is  (a)  0.  q Another  same  charge  q  is  H placed at  a distance L from  O.  Then  the  electric  flux  A through ABCD is  (a)  q/4pe 0 L  (b)  zero  (c)  q/2pe 0L  (d)  q/3pe 0L.  8.  The work done in placing a charge of 8 × 10 –18  coulomb on  a  condenser of capacity  100  micro­farad  is  (a)  16 × 10 –32  joule  (b)  3.  29.  42.  Four charges equal to –Q are placed at the four corners of a  square  and  a  charge  q  is  at  its  centre.  25.1 ×  10 –26  joule  –10  (c)  4 ×  10  joule  (d)  32 ×  10 –32  joule.  24.  43.  44.  37.  34.57  Electrostatics  32.  39.  33. 37 × 25 V = 9.  At  t  = t. Hence.  (d): Consider a elementary ring of  radius r and thickness dr of a disc as  shown in figure.  dQ =  dV = 1 dQ 1  Q  = dx  4p Î0 x 4 pΠ0  Lx m 0Qw R  m Qw R m Q w dr  = 0 2  = 0  2 ò 2 pR  2pR 0  2 p R Since.58  JEE MAIN CHAPTERWISE EXPLORER  1.  ( )  q  When a particle of mass m and charge q0  =  placed is at  2  the  origin is given a small  displacement  along the  y­axis.  5.  after  connection  120C 1  = 200C 2  6C 1  = 10C 2  3C 1  = 5C 2  2.  R 2 r VS  = 1  3  e 0  When a charge q is taken from the centre to the surface.  Potential at  O  due  to the  rod is  2 L  1  Q  dx  V = ò dV = ò 4  pΠ L  0  Lx Q ln2  2 L  1  Q  = [ln x ] L  = 4 pÎ0 L 4 p Π0  L 4.  R 2 r VC  = 2 e 0  Potential at  the  surface  of the  sphere.  (c) : During discharging of a capacitor  V  =  V 0 e –t/t  where t  is the time  constant of  RC  circuit.37 V 0  e From the  graph. .25 V  This voltage occurs at time lies between 100 sec and  500  sec.VS ) q = ç q = 1  ÷ è 2e0 3 e0 ø 6  e 0  Statement 1 is false.  Statement 2 is true.  (c) : For  potential  to  be  made  zero.  (a) :  Consider a small element of length dx at a distance x from O.  then the  situation is shown  in the  figure.  Q  dx  L Potential at  O  due  to  the  element  is  Charge  on the  element.  Q 2 Qr  dq = (2prdr ) = 2  dr  p R 2 R Current due  to  rotation of charge  on  ring is  O  r  dr  R  dqw Qrw dr  = 2 p p R 2  Magnetic field at the  centre  due  to the  ring element  m I m Qrw dr m Qw dr  dB = 0 = 0 2 = 0  2  2 r  2 pR r 2 p R Magnetic field at  the  centre  due  to  the  whole  disc  I  = B = ò dB = By  symmetry. t  = 0.  V 0  = 25 V \  V = 0.  Q and w are  constants  \ B  µ  1  R Hence variation of B with R should be a rectangular hyperbola  as represented in option (d). the  change  in potential energy is  æ R 2r 1 R 2r ö R 2 rq  DU = (VC .  (b): Potential at  the  centre  of the  sphere.  6.  Charge  on the  ring. time constant t of this circuit lies between 100  sec and 150 sec. \  The  net  force acting  on  the  particle  is qq 0  y  Fnet  = 2 F cos q = 2  1  2  2 4 p Î0  ( y + a 2 )  ( y 2 + a 2  )  ( )  q  q  y 2  æQ q  = q  (Given) ö 2  = ø 4p Î0  ( y 2 + a 2 )2  ( y 2 + a 2 )  è 0  2  q 2 y  1  = 2 4 p Î0  ( y +  a 2 ) 3/2  As y < < a  q 2 y  \ F net  = 1  4 p Π0  a 3  or  F net µ  y  3.  (b): The  situation is  as shown  in  the figure.  V  V = 0  = 0. the  components of  forces  on  the  particle  of  charge q 0  due to charges at A and B along x­axis will cancel  each other where along  y­axis  will add  up.   r r q inside  ò E × dS = Ñ  e 0  q  or .  8..Rx ) 4 px dx  2  )  3  r  = 4 pr0  ò 5 x 2  . q¢  = q  From equations (iv) and (iii). we  get K= K= 1  1 ..  9.  O  l  q q l  T cosq  q T  A  Tsinq B  F  F mg  x  2 T  C  F  d  mg  \  T cosq =  mg  2  1  q  4 pe 0  d 2  q 2  1  \ tan q = 4 pe 0  d 2 mg When charge begins to leak from both the spheres at a constant  rate.  (d) :  E  q  R  r  (For r > R )  The variation of E with distance r from the centre is as shown  adjacent  figure.  i.(ii)  Dividing equation (ii) by (i).  dV  = 4px 2 dx  Let us draw a Gaussian surface of radius r(r < R) as shown  in  the  figure  above.6  = = 2  (r .(iii)  mg When the balls are suspended in a liquid of density s and  dielectric  constant  K. the  forces acting on each ball are  (i) Tension  T  (ii) Weight  mg  (iii)  Electrostatic  force  of  repulsion  F  For its equilibrium along vertical.  (b): For  uniformly  charged  sphere  1  Qr  E= (For r < R )  4 pe 0  R 3  1  Q  E= (For r = R )  4 pe 0  R 2  1  Q  4 pe 0  r 2  q  °  30 E= 10.  then  q 2  1  tan q = 4 pe 0  x 2  mg 1 q 2  x x  = Q  tan q = 2l 4 pe 0 x 2 mg 2 l  2  x  q  or  µ  2  2 l  x or  q 2 µ x 3 Þ  q µ  x 3/2  dq  3  1/2  dx  µ  x  dt 2  dt and  T sin q = F  = ( ( or  v µ x -1/ 2  Q  )  )  dq = constant  dt T  T  + + mg  mg  F  Initially.  (d) : f =  ar 2  +  b  .(s / r)]  .d f = - 2 ar  dr According to Gauss’s  theorem..(i)  O  R  Gaussian surface  r  (Using (i))  Volume  of the  shell.  Tcosq =  mg  .s) (1.ù é As V  =  ù ú ëê ú r û  ëê r û  For equilibrium of balls..s r  ( )  r 1..  the  electrostatic  force  will  become  (1/K) times.(i)  and along horizontal.e..  Tsinq = F  .  (c) : Consider a thin spherical shell of radius x and thickness  dx  as shown in the  figure. we  get  tan q =  F .(iv)  According to given problem...  Total charge  enclosed inside  the  Gaussian surface  is r r  Qin = ò rdV = ò r0  0 0  ( ( 5 4  .  E = dx  x  ..6 - 0.  F  tan q¢ = F ¢ = mg ¢  Kmg [1 ..2ar 4 pr 2  = inside  e 0  3  q inside  = –  8e 0apr    Charge density inside the  ball is  q  rinside  = inside  4 p r 3  3  -8 e 0 a pr 3  \ rinside  = 4 p r 3  3  rinside = -6ae 0  Electric  field. F¢  = (F/K) while  weight  mg¢ =  mg  – Upthrust  =  mg  –  Vsg  [As Upthrust =  Vsg]  s m  mg ¢ = mg é1 .x  dx  R 0  4  .  (a) : Figure shows equilibrium positions of the two sphere.59  Electrostatics  7.8)  11.   ò dE cos q = 0  The  net  electric  field  at  O  is  p r  p ^  lrd q sin q( .  (c) : Work done  = potential  difference  ×  charge  = (VB  –  VA) ×  q..  p R The electric  field at the  point p  distant  r 1  from the  centre.  R  Q  E  = outside shell and zero inside..... C  =  0  d e0 A × 3  C1  = = 9 C  ;  d / 3  e A × 6  C2  = 0  = 9 C  2d / 3  18..  Qq - Þ E  = r 1  r  1  1  Qr  × 4 pr 2 dr  e 0 0 ò p R 4  R  Qr 1 2 ....  C  r  k 1  = 3  k 2  = 6  d/3  2d /3  ..  If the loop is completed..  4 pe 0 R 4  16.j  ^  )  E = ò dE sin q( . • – 4 V  P  Q  Work done  in moving 100e –  from  P  to  Q.  (d) : +10 V • ..  (c) : If the  charge  density.ú ë 3  4 ëê 3 Rû R û  According to Gauss’s law  Q  E 4 pr 2  = in  e 0  pr r 4  ù 0 é 5  3  E 4 pr 2 = r  e 0  ëê 3  R ûú = E  = pr0 r 3  é 5  r  ù 4 pr 2 e 0  ë 3  R û E  = r0 r  é 5  r  ù 4e 0  ë 3  R û 15..  according to Gauss’s theorem is  Q  E∙4pr 1 2  =  charge enclosed/e 0  E × 4 pr1 2 = 12.ú = pr0  ê r 3  .60  JEE MAIN CHAPTERWISE EXPLORER  14.  Force  of  repulsion.cos q] 0 p j  2 2  4 p 2 e 0 r 2  0  4 p e 0 r q  ^  =j  2 p 2 e 0 r 2  ( )  13. r = Q 4  r .  Inside the shell.  17...  = 9C ´ 9C  = 9 ´ C  or 9 ´ 9 ´ 10-12  F  18C 2 2  Þ  C total  = 40..  W = (100e – )(V Q  –  V P)    = (–100 × 1... Electrostatic force is a conservative force.  No net work is done as the initial and  final  potentials are the  same.  r  é 5  é 5  x 4  ù r 4  ù = 4 pr0  ê x 3  = 4 pr0  ê r 3  ú ë 12 ë 12 4 R úû  4 R û 0  4 pr0  é 5 3 r 4 ù é 5  r 4  ù r .  (b)  :  The  electric  field  for  a  E  uniformly  charged  spherical  shell  is  given  in  the  figure.   B  dqr \  Charge on the element....24 × 10 –16  J.  (d) :  Linear charge  density.5 pF..  (d) :  C  = \ C total  = C1C 2  C1 + C2  as they are  in series...  (Work done in moving 100 negative charges from the positive  to  the negative  potential)... the field is zero  and it is maximum at the surface  O  and then decreases A  B  µ 1/r 2 ..  VA  and VB  only depend on the initial and final positions and  not on the path..  l = Þ E × 4 pr1 2 = q  p r Consider a small element AB  ^  j  of length  dl  subtending  an  angle dq at the centre O as  A dl  shown in  the  figure.j )  = ò 2  0  4 pe 0 r 0  p qr sin qd q ^  q  =-ò j  Q  l = 2 3  pr 0  4 p e 0 r p q  q sin qd q ^  ^  =-ò j  = [ .  dEcosq  q  dq  = ldl ^  i  q O  dl  = lrdq Q  d q = dE  dEsinq  r The electric field at the centre O due to the charge element is  ( dE  = )  1  dq  l rd q = 4 pe 0  r 2 4 pe 0 r 2  Resolve dE  into two  rectangular  components  By symmetry...  4 pe 0 × r 2  e 0 A  = 9 ´ 10-12 F  d e kA  With dielectric. VA  – VA  = 0.6 × 10 –19 )(–14 V) = 2.  (a) : The force of repulsion  by Q is  +  q –  Q  cancelled  by  the  resultant  attracting  A  force due to q –  and q –  at A and B.  Both  the  statements  are  true  but  A  B statement­2  is  not  the  reason  for  statement­1.  B  –  2  +  q Q2 1 1  Q  Q  F  = = × 2  2 2 4pe 0 (a + a ) 4 pe 0  2 a Total  force  of  attraction  along  the  diagonal  (taking cosq  components) ì Qq  2 ü Qq 1  Qq  1 1  = × + = í ý 4 pe 0  a 2  2 a 2  2  4 pe 0  î a 2  þ  { Q 2  Qq  2  Þ =  2  2 a 2 a 1  rdV e 0  ò }  Q 2  Þ = - 2 2 (a ). dV  = .  (b)  :  Energy  of  capacitor  =  Heat  energy  of  block 1  2  CV = ms D T \  2  2 ms D T  V  =  .  Energy stored in the capacitor  U = 1 CE 2  2  1 CE 2 U  = 2  \  =  1 .  or  C Q  1 Q .6 ´ 10-19  ´ 20  = m 9.  v = 27.  Therefore after it is taken out.d æç 20 ö÷ = 40 x  dx dx  è x 2 . Total work  done =  zero. \  Field  is affected  but not the  potential.61  Electrostatics  19.  2  144 9  [16 - 4]  21.  = V  R A  or  or  2 e ´ DV  2 ´ 1.  If  they  are exchanged.  (b) : An electron on plate 1 has electrostatic potential energy.  (d)  :  V A  = 23.  E B R A  1  A  r 1  (4 pe0 R A ) V  V  RB  A  1 10 -3 ´ 10 -6  =  4 pe  2 0  V B  = = Y (0.4) 2  At x = 4 mm \  \  + Q  X  (2.  2 )  d  B  Potential at point B is  2  4 pe 0 RA  E A R B  2  = =  . \ Kinetic  energy  =  Electrostatic  potential  energy  1  2  mv = e D V or  2  Positive sign indicate E is +ve x direction.  the energy  U i  =  q 2  =  2 KC Once is taken out.  22.  charge  is  redistributed  and  the  spheres  attain  a common  potential  V.  (c) :  rr1  = 2 iˆ + 2 ˆ j 2  1 ´ C A V  28. the dipole will experience  a  torque  as  well  as  a  translational  force. In the case of potential.V B  = = – Q  1 ´ Q  é 2 2  ù ê 2 4 pe0  ê R  R + d 2  úú ë û  Q  é 1 1  ù 2 pe0  êê R  R 2 + d 2  úú . as it is  D a  scalar.  If a dielectric  slab is  slowly introduced.  When  it  moves.65 ×  10 6  m/s.  ë û  30. potential  energy  is  converted  into  kinetic  energy.  (d) : In a non­uniform electric field.4 ø  ( x 2 .  (a) :  The potential  energy of  a charged  capacitor  24.  q  B  C  – q  r  r2  = 2iˆ + 0 ˆ  j r  or  | r2 | = r2  = 2  ( \  1  q  1 10–3 ´10 -6  = 4 pe 0 r2  4pe0  2 V A  – V B  = 0.  (a) : “Unit positive charge” will be repelled  q  by A  and  B  and  attracted  by  – q  and  – q  A  downwards  in  the  same  direction.  (a) : Given : Potential  V ( x ) = 20  x - 4  E= 40 ´ 4 160 10  = = V/m m.1  4 pe0 R  4 pe0  R 2 + d 2  Q  1 ( -Q )  1  V B  = + 4 pe0 R  4 pe0  R 2 + d 2  B  2  2  R  + d R  2 .11 ´ 10 -31  v  =  2.  W  CE 2  2  2 E A  = 26. 0)  \  V A .  (c)  :  Resultant  intensity  =  0  O  A  B  – 2q  + 8q  L  d  . the potential energy come back  to the  old value.  20.  they  cancel  each  other  whatever  – q  may be  their position. 2  q  2 C where U i  is the initial potential energy. the direction of the field will  be opposite. again the energy increases to the old value.  (a) : Let E be  emf of  the  battery  Work done by the battery W = CE 2  ( 2 ) + ( 2 )  or  1  Q A  2  4 pe 0  R A  25.  (d)  :  When  the  spherical  conductors  are  connected  by  a  conducting  wire.  (c)  :  n  plates  connected  alternately  give  rise  to  (n  –  1)  capacitors  connected  in  parallel \  Resultant  capacitance  =  (n  –  1)C.  r  | r1 | = r1  = Intensity  E A  = \  2  Electric field  E  =  . 0)  r 2  1 q  4 pe 0 r1  Similarly  E B  = = 2  Potential at point A is  V A  = 4 pe0 R A2 29.   .  mv  =  .q1 )( .(i)  when the third equal conductor touches B. the charge of B  is  shared  equally  between  them q  \  Charge  on  B = = charge  on  third  conductor....  1  4 r 4  8q 1 1  2 q  = 0  4 pe0 ( L + d ) 2 4 pe0  d 2  or  or  \  (L  +  d) 2  =  4d 2  d  =  L Distance  from  origin  =  2L.(i)  2  r F 1  –q 1  (90° – q ) q  q 2  x  F 2 Resolved  part  of  F 2  along  q 1 q 2  =  F 2  cos  (90°  –q)  = q1q 3 sin q \  4 pe 0 a 2  along  (q 1 q 2 ) Total  force  on  (–q 1 )  é q q  q q  sin q ù = ê 1 2  2 + 1 3  2  ú along x ­axis  4 pe0 a ûú ëê 4 pe 0 b \  é q 2  q 3  ù x­component  of  force  µ ê 2 + 2  sin q ú .. è ø  \  Charge  on  q  T sinq  .  (d)  :  Energy  of  condenser  = 2 -18 2  1 Q  1  (8 ´ 10 )  = ´ = 32 ´ 10-32  J  2 C 2  (100 ´ 10 -6 )  ..  4 pe0 d 2  è 2 4 ø  8  F 2  a  –Q  F 1  3 q  = Charge of third conductor  4  New  force  between  B  and  C  C= 32.q 3 )  4 pe 0 a 2  0  F 2  makes  an  angle  of  (90°  – q)  with  (q 1 q 2 )  y Distance  CA =  2  a –q 3  2 a  a  =  2  2  For equilibrium.  F 3  45°  .(ii) –Q  B  A  a  D  –Q  E  F 4  q  a  a  C  –Q  ...  a ëb û  36.  (d)  :  Initially.q1q 2  35..2  ´ 1  = 0  2 a / 2  2  a 2 2a2 or  é ù Q ê1 + 1  ú = q 2  2 2  ë û  or  q= Q é 2 2 + 1 ù Q  = (1 + 2 2 )...  2  æqö Now  this  third  conductor  with  charge  ç ÷ touches  C.  2  m (2 v )  =  r 1  \  From  (i)  and  (ii)  1  = r 1  Þ r  = r ...  (d)  :  Energy  is  conserved  in  the  phenomenon  1  2  kqQ  Initially. consider  forces along DA  and  equate  the  resultant  to  zero 1 Q ´Q 1  Q ´ Q  + cos 45 ° \  4 pe0 ( DA)2 4 pe 0  (CA ) 2  q  Distance  EA = 1  Q ´ q  cos 45° = 0  4 pe 0  ( EA) 2  or  Q Q q  + ´ 1 ..  2 êë 2 2  úû  4  33..  their  è 2 ø  q ö æ total  charge  ç q + 2 ÷ is equally  shared  between  them...  tan q = \  = 1 æ q 3 q ö 3  ç ´ ÷ = F  ....62  JEE MAIN CHAPTERWISE EXPLORER  kqQ  1  2  Finally.  (b)  :  Force  on  (–q 1 )  due  to  q 2  = \  F 1  =  q1q 2  4 pe 0 b 2  4 pe0 b 2  along  (q 1 q 2 )  Force  on  (–q 1 )  due  to  ( .  F 2 ....q 3 ) = q1q 3  F 2  =  4 pe  a 2  as  shown  ( ...  2  1  q  34.  (b)  :  Tsinq  = sq/e 0  T cosq T  qs e 0  mg  T  cosq =  mg \  \ sq  e0 mg s  is  proportional  to  tanq.  (b)  :  Consider the  four  forces  F 1 ...  F 3  and  F 4  acting  on  charge  (–Q)  placed  at  A.  F  = 4 pe  2  0  d 31.....   C ¢ = 0 ( d - t )  d 39.  it  is  6 e  .  e A  With  air  as  dielectric.  e0  The  flux enters the  enclosure if one  has a  negative  charge  (–q 2 ) and flux  goes  out if  one has  a  +ve  charge  (+q 1 ).1 ´ 10-10  F .  Q´q 4 pe 0 d 2 + Q ´ Q  4 pe 0 (2 d ) 2  = 0  d  Q  or  d  q  Q  Q  q = -  .63  Electrostatics  37.  ( * )  :  Electric  flux  through  ABCD  =  zero  for  the  charge  placed outside the box as the charged enclosed is zero.  For  one  surface.f1 ) e0  .  (b) : Aluminium is  a good  conductor.  (Option  not  given).  Q =  q 1  ~  q 2  40.  C  =  0  d e A e A  = 0  = C .  (a)  :  According  to  Gauss  theorem. 4  42. But  q for  the  charge  inside  the  cube.  (d) :  When  the  system  of three  charges  is  in  equilibrium. f 2  > f 1 .1 volt.  With  space  partially  filled.  (b)  :  Total  capacity  =  nC 1  2  \  Energy  =  nCV 2  43.  9  9 ´ 10  41.  it  is  e  through  all  the  0  q surfaces.  ( f2 .f1 ) = Q  Þ Q = ( f2 .  (a)  :  W  =  QV \  V  = W  2  = = 0.  Q 20  .  (a)  :  C = 4 pe0 R = 1  = 1.  4 pe 0 R 4pe 0 R 4pe 0 R  38.  0  44.  The  capacity  remains  unchanged.  Its sheet introduced  between the plates of a capacitor is of negligible thickness.  (c) : Potential at any internal point of charged shell  = q  4 pe 0 R 1  2 Q  Potential  at  P  due  to  Q  at  centre  = 4 pe R 0  \  Total  potential  point  q 2 Q  1  = + = ( q + 2Q ). As  one  does not know  whether f 1  > f 2 .   1 2 1 2  This question contains Statement­1 and Statement­2.04 Volt  (b)  zero  Volt  (c)  2.  5.  DV  I  I  A  8.03 A P 1  to P 2  (2008)  Directions : Questions 8 and 9 are based on the following paragraph.  (c)  Statement­1 is true.  The  resistance  of  a  wire  changes from 100 W to 150 W when its temperature is increased  from 27°C to 227°C.  (iii)  From the r dependence of E(r). This implies that a = 2.  (2009)  Shown  in  the  figure  below  is  a  meter­bridge  set  up  with  null deflection in the galvanometer. where j  is the current per unit area  at  r. Statement­2 is  true; Statement­2 is  the  correct explanation  of  Statement­1.  (a)  0. Current  I  enters  at  A and  leaves  from  D.  4.05%  (b)  increase by 0. (ii) and (iii) for current I leaving D and superpose  results for  A and  D.  Statement­1: The  temperature  dependence  of  resistance  is  usually  given  as  R  =  R 0 (1  + aDt). The  calculation is done  in the following steps:  (i)  Take current I entering from A and assume it to spread over  a hemispherical surface in the block.  choose the one that  best describes the two  statements.5 × 10 –3 /°C  Statement­2: R = R 0 (1 + aDt) is valid only when the change  in the temperature DT is small and DR =  (R – R 0 ) << R 0 .  We  apply  superposition principle to find voltage DV developed between B  and  C.27 A P 2  to  P 1  (d)  0.64  JEE MAIN CHAPTERWISE EXPLORER  CHAPTER  12  1. A 60 W bulb is already switched on. a + a  (2010)  (c)  a1 + a 2 .  6. The resistance of the  lead wires is 6 W.3 Volt  (2013)  Two electric bulbs marked 25 W­220 V and 100 W­220 V are  connected in  series  to  a  440  V  supply.75 W  (c)  220 W  (d)  110 W.  2. What  is  the  decrease  of  voltage  across  the  bulb.  Statement­2 is true.  (2008)  A  5  V  battery  with  internal  resistance  2 W  and  2 V battery with internal resistance 1 W are connected to a  10 W resistor as shown in the figure.  The  respective temperature coefficients of their series and parallel  combinations are  nearly  a + a 2 a1 + a 2  a + a 2  (a)  1 .  (b)  1 .2%  (d)  decrease by  0.27 A P 1  to P 2  (c)  0. a  B  b  a  C  D DV  measured between B and C  is  . Statement­2 is false  (b)  Statement­1 is true.  (a)  Statement­1 is  true.  Consider a block of conducting material of resistivity r shown in  the  figure.  (iv)  Repeat (i).9 Volt  (d)  13.1% longer.  If  a  wire is stretched to  make it  0. Of the  four choices given after the statements. obtain the  potential V(r) at r.  3.  CURRENT ELECTRICITY The supply voltage to a room is 120 V.  (d)  Statement­1 is  false.05% (2011)  Two  conductors  have  the  same  resistance  at  0°C  but  their  temperature  coefficients  of  resistance  are a 1  and a 2 . Statement­2 is  true; Statement­2 is  not  the  correct  explanation  of  Statement­1. Which  of  the bulbs  will  fuse?  (a)  100 W  (b)  25 W  (c)  neither  (d)  both  (2012)  55 W  G  20 cm  7.2%  (c)  decrease by 0. The value of the unknown  resistance R is  R  (a)  55 W  (b)  13.  its resistance  will  (a)  increase by 0.  when  a  240 W  heater is switched  on in parallel to  the bulb?  (a)  10. a1 + a 2  2 2 2 a1a 2  a + a 2  (d)  a1 + a 2 .  (ii)  Calculate field  E(r)  at distance  r  from  A by using  Ohm’s  law E = rj.03 A P 2  to  P 1 . The current in the 10 W resistor is  P 2 5 V  2 W  2 V  1 W  10 W  P 1  (b)  0. 65  Current Electricity  rI  (a)  2 p(a - b )  rI rI  (c)  a .  The resistance of hot tungsten filament is about 10 times the  cold  resistance.  an  electric  current  will  (a)  flow  from  antimony  to  bismuth  at  the  cold  junction  (b)  flow  from  antimony  to  bismuth  at  the  hot  junction  (c)  flow  from  bismuth  to  antimony  at  the  cold  junction  (d)  not  flow  through  the  thermocouple. The internal resistances of the two sources are  R 1  and  R 2  (R 2  >  R 1 ).  The resistance of a bulb filament is 100 W at a temperature  of 100ºC.  An  electric  bulb  is  rated  220  volt  ­  100  watt.  antimony  and  bismuth.67 A.  rI rI  (b)  pa . its resistance will become 200 W at a temperature of  (a)  200ºC  (b)  300ºC  (c)  400ºC  (d)  500ºC.  If  the  potential  difference  across  the  source  having  internal  resistance  R 2  is  zero. the ratio  l B /l A  of their respective  lengths must be  (a)  2  (b)  1  (c)  1/2  (d)  1/4.  (2006)  18. If its temperature coefficient of resistance be 0.p( a + b )  rI rI  (d)  2pa . the galvanometer G shows zero  deflection.  (2006)  16. The  condition for bridge to  be  balanced  will  be  P R  (a)  Q = S + S 1 2  P 2 R  (b)  Q = S + S 1 2  P  R ( S1 + S 2 )  (c)  Q =  S S 1 2  P  R ( S1 + S 2 )  (d)  Q =  2 S S .( a + b )  9.17 A  (c)  0.2p(a + b)  (2008)  For current entering at  A.33 A  (d)  0. three resistance P.  (2006)  17.  In  a  potentiometer  experiment  the balancing  with  a  cell  is  at  length 240 cm. A circular  wire made of B has twice the diameter of a wire made of A.  are  respectively  based  on  (a)  conservation  of  charge.  An energy source will supply a constant current into the load  if  its  internal  resistance  is  (a)  zero  (b)  non­zero  but  less  than  the  resistance  of  the  load  (c)  equal  to  the  resistance  of  the  load  (d)  very  large  as  compared  to  the  load  resistance  (2005)  22.5 W  (2005)  19.5 A  (b)  0. the  value  of  the  resistor  R  will  be  10 W  500 W  5 V  G  (a)  0.  conservation  of  charge.  The  Kirchhoff’s  first  law  (å i = 0)  and  second  law  (å iR = å E ).  where the symbols have their usual meanings.  (2006)  13.005  per ºC.  Two  sources  of  equal  emf  are  connected  to  an  external  resistance R.  conservation  of  energy  12 V  B  (a)  500 W  (c)  200 W  R  A  (b)  1000 W  (d)  100 W  2 V  (2005)  21.  The resistance of the wire  at 0°C will  be  (a)  3 ohm  (b)  2 ohm  (c)  1 ohm  (d)  4 ohm  (2007)  11.  conservation  of  momentum  (c)  conservation  of  energy.  In the circuit.  conservation  of  charge  (d)  conservation  of  momentum.  Then for the two wires to have the same resistance. What  will  be  the  resistance  of  100 W  and  200 V  lamp  when  not  in  use? .  If  the batteries A and B have negligible internal resistance.  The  power  consumed  by  it  when  operated  on  110  volt  will  be  (a)  50 watt  (b)  75 watt  (c)  40 watt  (d)  25 watt. On shunting the cell with a resistance of  2 W.  1 2  (2006)  15.  the electric  field  at  a distance r  from  A is  rI  rI  (a)  (b)  4 p r 2  8 p r 2  r I  rI  (c)  2  (d)  (2008)  r 2 p r 2  10.  The  internal  resistance  of  the  cell  is  (a)  4 W  (b)  2 W  (c)  1 W  (d)  0.  the  balancing  length  becomes  120  cm.  The resistance of a wire is 5 ohm at 50°C and 6 ohm at 100°C. Q and R connected  in the three arms and the fourth arm is formed by two resistance  S 1  and S 2  connected  in parallel.  A material B has twice the specific resistance of A.  then  (a)  R = R1R 2  R1 + R2  (c)  R = R 2  5 W  10 W  I  20 W  (b)  R = ( R1 + R 2 )  ( R2 - R1 )  R1R 2  R2 - R1  (d)  R  =  R 2  –  R 1  (2005)  20.  (2006)  12.  A  thermocouple  is  made  from  two  metals.  The  current  I  drawn  from  the  5  volt  source  will  be  10 W  (b)  conservation  of  charge.  If  one  junction  of  the  couple  is  kept  hot  and  the  other  is  kept  cold  then.  (2006)  14.  In a Wheatstone’s bridge.   then  the  neutral  temperature  is  (a)  700°C  (b)  350°C  (c)  1400°C  (d)  no neutral temperature is possible for this thermocouple. When they are joined  in parallel the  total resistance is  P.  30 E  (d)  100 .  (2004)  30.  The electrochemical equivalent  of  a  metal  is  1. of its standard cell is E volt.  The length of a given cylindrical wire is increased by 100%. and  the  e. The mass of the metal liberated  at the cathode when a 3 A current is passed for 2 second will  be (a)  19.5 W. where i is  the  current in the potentiometer  wire.  (2003)  36.  of  the  battery  is  30 E  (a)  100. When a  total charge q flows through the  voltmeters.  A 220 volt.  equal  amount  of  metals  are  deposited.  (2004)  32.  The  total  current  supplied  to  the  circuit  by  the  battery  is  (a)  1 A  2 W  (b)  2 A  6 V  (c)  4 A  6 W  3 W  (d)  6 A.  In  a  metre  bridge  experiment  null  point  is  obtained  at  20 cm from one end of the wire when resistance X is balanced  against another resistance Y.  A 3 volt battery with negligible internal resistance is connected  in a circuit as shown in the figure.3 × 10 –7  kg per coulomb.  one  of  copper  and  another  of  silver.  (2004)  27.  I The current I.  The  power  consumed  will  be  (a)  750 watt  (b)  500 watt  (c)  250 watt  (d)  1000 watt. If the electrochemical equivalent of Zn and Cu are  32.5 and  31.5  30 E (b)  100 - 0.  If  the balance point is obtained at l = 30 cm from the positive  end.5 i .  If S  =  nP.  Two  voltameters. decreases in mass by 0.m. sending  a  constant  current through a circuit. then where  will be  the  new  position  of  the  null  point  from  the  same  end. connected in parallel.66  JEE MAIN CHAPTERWISE EXPLORER  (a)  400 W  (c)  40 W  (b)  200 W  (d)  20 W  (2005)  23.5 A  (c)  2 A  (d)  (1/3)  A  (2003)  35.5 W  3.  If  the  electrochemical equivalents of  copper and  silver are z 1  and  z 2  respectively  the  charge  which  flows  through  the  silver  voltameter  is  z 1  z 2  (a)  q z  (b)  q  z  2  1  q  q  (c)  (d)  z  z  (2005)  1 +  1  1 +  2  z2  z1  24.  (2004)  29.  The  thermistors  are  usually  made  of  (a)  metals  with  low  temperature  coefficient  of  resistivity  (b)  metals  with  high  temperature  coefficient  of  resistivity  (c)  metal  oxides  with  high  temperature  coefficient  of  resistivity  (d)  semiconducting  materials  having  low  temperature  coefficient  of  resistivity.m.  (2004)  33.8 ×  10 –7  kg  (b)  9.5 respectively. of a battery whose internal resistance is 0.  if  one  decides  to  balance  a  resistance  of  4X  against  Y?  (a)  50  cm  (b)  80  cm  (c)  40  cm  (d)  70  cm.5  30 E  (c)  100  - 0.  (2004)  28.  (2004)  26.  then  the  minimum  possible  value  of  n  is  (a)  4  (b)  3  (c)  2  (d)  1.  The length of a wire of a potentiometer is 100  cm.  A  heater coil is cut into two  equal  parts  and  only  one part  is  now  used in the  heater.6 ×  10  kg  (d)  1.  the  e.  (2003)  37.9 ×  10 –7  kg  –7  (c)  6. If the lengths  and  radii of  the wires  are  in  the  ratio  of  4/3  and  2/3.  The  negative  Zn pole of a  Daniell cell.  If  the  cold  junction  is  kept  at  0°C.f. It is employed to measure  the e.13 g in 30  minutes.  Due  to  the  consequent  decrease  in  diameter  the  change  in  the  resistance  of  the  wire  will  be  (a)  200%  (b)  100%  (c)  50%  (d)  300%.  The  resistance of the series  combination  of  two  resistances  is S.  1000 watt bulb is connected across  a  110  volt  mains  supply.1 × 10 –7  kg.  the  increase  in  the  mass  of  the  .  are  joined in parallel.f. If X  <  Y.  An electric current is passed through a circuit containing two  wires of the same material.m.f.  then  the  ratio  of  the  currents  passing  through  the  wire  will  be  (a)  3  (b)  1/3  (c)  8/9  (d)  2. in the circuit will  be 3 W  3 V  3 W  (a)  1 A  (b)  3W 1. The  heat generated  will  now be  (a)  one  fourth  (b)  halved  (c)  doubled  (d)  four  times  (2005)  25.  Time taken by a 836 W heater to heat one litre of water from  10°C  to  40°C  is  (a)  50 s  (b)  100 s  (c)  150 s  (d)  200  s.  The thermo emf of a thermocouple varies with the temperature q of the hot junction as E = aq + bq 2  in volt where the ratio  a/b  is  700°C.  (2004)  31.  (2003)  34.  then  R  is  2 W  (a)  2 W  (b)  6 W  15 V  (c)  5 W  (d)  4 W.  (a)  (d)  (d)  (c)  (a)  (b)  (c)  5.  The mass of a product liberated on anode in an electrochemical  cell  depends  on  (a)  (It) 1/2  (b)  It  (c)  I/t  (d)  I 2 t.  22.  (c)  (c)  (b)  (c)  (a)  (c)  (b) . capable  of detecting current as low as 10 –5  A.  (2003)  38.  35.  36.  10.  If  in  the  circuit.  12.  40.  25.  20.  11.  23.  42.  The thermo e.  19.  (2003)  39.qi = 2qn  .  38.  13.  30.m.  (where t is the time period for which the current is passed).242  g.qc = 2qn  qi + qc = qn  (c) (d) qc .  (2002)  42.  7.  14.  34.180  g  (b)  0.  16.  21. The  smallest temperature  difference  that  can  be  detected  by  this  system  is  (a)  16°C  (b)  12°C  (c)  8°C  (d)  20°C.  15.  37.  9.  39.  (a)  (a)  (a)  (d)  (c)  (c)  (b)  6.f.  31.  (2002)  Answer  Key  1.  8.  28.  then  (a) qi + qc = qn  (b) qi . q c  is  the  temperature  of  the  cold  junction.  18.  29.  24. A galvanometer of 40 ohm resistance. is  connected with the  thermocouple.  A wire  when connected  to 220 V  mains  supply  has  power  dissipation  P 1 .141  g  (c)  0.  27.  41.  (b)  (d)  (a)  (a)  (b)  (d)  (b)  4.  (2002)  40.  26.  (a)  (d)  (c)  (d)  (c)  (d)  (c)  2.67  Current Electricity  positive  Cu  pole  in  this  time  is  (a)  0.   Then  P 2  :  P 1  is  (a)  1  (b)  4  (c)  2  (d)  3.  (2002)  2 41.  17.  Power  dissipation  in  this  case  is  P 2.  Now  the  wire  is  cut  into  two  equal  pieces  which are  connected  in parallel  to the same  supply.126  g  (d)  0.  (b)  (d)  (c)  (d)  (a)  (c)  (a)  3. q n is the neutral temperature.  If q i  is the inversion temperature.  power  R  dissipation is 150 W.  33.  32.  of a thermo­couple  is  25 mV/°C at room  temperature. 07 V  240 W + 6 W  Resistance of  heater.  RH  = As bulb and heater are connected in parallel.  Let R 1  and R 2  be their resistance at t°C. the supply voltage is taken as rated  voltage. I = 440 2  =  A  (220)2  / 20  11  Potential  difference  across  25  W  bulb  .  10..  2.  Then  R R  R p  = 1 2  R1 +  R2  R (1 + a1t ) R0 (1 + a 2 t )  R p 0 (1 + a p t ) = 0 R0 (1 + a1t ) + R0 (1 + a 2 t )  R0 R0 R  = 0  where.41 V  .  DR = 2 D l  R l 120 V × 48 W = 106.1% its resistance  will  increase by 0.  (a) :  As  P =  V  R Here.  R p 0  = R0 + R0  2  . when wire is stretched by 0. Then R 1  = R 0 (1 + a 1t) R 2  = R 0 (1 + a 2t)  Let R s  is  the  resistance  of  the  series  combination  of  two  conductors at t°C. because  it can tolerate  only 220 V while  the  voltage  across it is 352 V.  (b) :  25 W‑220 V  100 W‑ 220 V  440 V  (Rated voltage) 2  Rated power \  Resistance of 25 W­220 V bulb  is  (220) 2  W  25  Resistance of 100 W­220 V bulb  is  R1  = (220) 2  W  100  When  these  two  bulbs  are  connected  in  series.68  JEE MAIN CHAPTERWISE EXPLORER  2  1.  120 V ´ 240 W V1  = = 117. \  Resistance  of  bulb  120 V ´ 120 V  RB  = = 240 W  60 W  120 V ´ 120 V  = 60 W  240 W  Voltage  across bulb  before  heater  is  switched  on.2%. Their equivalent  resistance is  (240 W)(60 W)  Req  = = 48 W 240 W + 60 W \  Voltage across  bulb  after heater  is  switched  on  V2  = Potential difference  across 100 W  bulb  (220) 2  = IR2  = 2  ´ = 88 V  11 100  Thus the bulb 25 W  will be  fused.  (b) : Resistance  of  wire  rl  R =  A On stretching. 66 V  48 W + 6 W  Decrease  in the  voltage  across the bulb  is DV  = V 1  –  V 2  = 10.  the  total  resistance is  R2  = (220) 2  Rs  = R1 + R2  = (220) 2  é 1 + 1  ù = W ë 25 100 û  20  Current.  4.  (a) :  Let R 0  be  the  resistance of both conductors  at  0°C. volume (V) remains constant. R s0  = R 0  + R 0  = 2R 0 \  2R 0 (1 + ast) = 2R 0  + R 0t(a 1  + a 2 )  2R 0  + 2R 0ast = 2R 0  + R 0t(a 1  + a 2 )  a + a 2  \ a s =  1 2  Let R p  is the resistance of the parallel combination of two  conductors at t°C..04 V  V  As  R =  (220) 2  = IR1  = 2  ´ = 352 V  11 25  or (Q  V  and r are constants)  D R % =  2 D l %  R l Hence.(i)  V  So V  = Al  or  A =  l rl 2  (Using (i))  \ R =  V Taking logarithm on both sides and differentiating we get.  3.  Then Rs  = R 1  + R 2 Rs0  (1 + ast) = R 0 (1 + a 1t) + R 0 (1 + a 2t)  where.   C  E = Ir/2pr 2 .  j  = -1  (a + a 2 ) t ù = 1 [1 + (a1 + a 2 )t ] éê1 + 1 ú ë û  2 2  Resistance = 1  é (a + a 2 ) t ù [1 + (a1 + a 2 )t ] ê1 .  If r l  is the resistance per  unit length (in cm)  A  x  P 2  x –  y  Q R A = RB  B  2 V 2 W  10 W  C  11.. a  = 2.(iii) = (ii)  × 2  (i) – (iii) gives 32y  = 1  Þ y = 1  A = 0...(ii) Þ 2x  – 22y = 4  .  R 100  = 6 W  R t  = R 0 (1 + at)  where R t  = resistance of a wire at t°C.  a  a  As  (a 1  + a 2 ) 2  is negligible  (c) :  b  D  rl rr  = area  2 p r 2  VB ..(i)  æ r ö æ 2 D A  ö 4 2  = ç A ÷ç ÷ = 2 = 1  è 2r A ø è DA  ø rl rl ´ 4  = 2 pr p D 2  .I r a  1  ..  The resistance of a wire change from 100 W  to 150 W  when  the  temperature  is  increased from  27°C  to 227°C.  R 0  = resistance  of a  wire at 0°C.1 úû  ë 2 2  [By  binomial  expansion]  Þ DV = DV  = (a1 + a 2 )  t 2  a + a 2  a p =  1 2  5.69  Current Electricity  \ R0 R 2  (1 + a1t )(1 + a 2 t )  (1 + a p t ) = 0 2 2 R0 + R0 (a1 + a 2 ) t R0 R 2 (1 + a1t + a 2t + a1a 2 t 2 )  (1 + a p t ) = 0 2 R0 (2 + (a1 + a 2 )t )  (1 + a1t + a 2t + a1a 2 t 2 )  1  (1 + a p t ) = 2 (2 + (a1 + a 2 )t )  8..  a p t =  ò  .  20rl 80 rl  = or R = 80 ´ 55 = 220 W .R 0  1  = 6 - R0  2  This is a Wheatstone bridge.  As a 1  and a 2  are small quantities \ a 1a 2  is negligible  + 2 – 1(x  – y) + 10 ∙  y  = 0  +x – 11y = 2  .  55 R 20  (d) :  I  2 p r 2  a + b  \ 1 (1 + a p t ) = 1 é1 + 1  (a1 + a 2 ) t ù û  2 2 ë 2  7.  (i)  R 100  –  R 0  =  R 0a (100)  .03 A from P2 to P1 . The  current  I  is  a surface  current.  (d) : j × r =  E.Edr It is true that a  is small.  (d) : Given : R 50  = 5 W.1  ùú ..  (a)  :  Resistance  of  a  wire  R  = y  5 V  x  P 1  x –  y  \  1 W  4r Al A pD A2 = 4 r B l B  2  p DB  2  lB æ r A ö æ D B  ö or  l = ç r ÷ ç D ÷ A è B ø è A  ø  D  Applying  Kirchhoff’s  law  for  the  loops  AP 2 P 1 CA  and  P 2 BDP 1 P 2 .VC  = DV = é ( a + a 2 )t (a + a 2 ) 2 t 2  ù = 1  ê1 .I r é 1 ù a  dr  = ò  2  2p 2 p êë r úû a + b  a + b  r I r é 1 ..e.  A  a  1 + (a1 + a 2 ) t  1 + (a1 + a 2 ) t  \ 1  (1 + a p t ) = = 2 2 + (a1 + a 2 ) t (a + a 2 ) t ù 2 éê1 + 1 ú ë û  2  Current density.  one  gets  –10y – 2x + 5 = 0 Þ  2x + 10y = 5  or  10 – 2R 0  = 6 –  R 0  or  R 0  = 4 W..1 úû  2ë 2 2  6.5  × 10 –3 /°C. a  = temperature  coefficient of resistance.  R –  R0  <<  R0  is not true. 9. we get  20 cm  80 cm  5 .  (a) : From the statement  given.  R  .  2 p ëê a a + b û  I r 2 p r 2  10.  = 55 W  B  2  .1 + (a1 + a 2 ) t . (ii)  Divide (i) by (ii)..  32  (d) :  Current is spread over an area 2pr 2 . But (150 – 100) W  or 50 W is not  very much less than  100 W  i..  \ E  = \  R 50  =  R 0  [1 + a 50]  and R 100  =  R 0  [1 + a 100]  or  R 50  – R 0  =  R 0a(50)  .   (d)  :  The  voltameters  are  joined  in  parallel.  Q S1 S2  15.l ) 4 100 .1 ÷ = R ç ÷ l è l2  ø è 2  ø  é 240 .005  (100)]  or  100  =  R 0[1  +  0.5 A .l 2  ö r = R ç 1  .120 ù = 2 W.  \  484 17..  (d)  :  I  = .005t ]  1  +  0.  Resistance  when  not  in  use  =  10 22..  It  means  that  current  flows  from  A  to  B  at  cold  junction.005t]  200  =  R 0 [1  +  0..  (d)  :  Resistance  of  the  bulb  V 2 (220) 2  ( R ) = = = 484 W  P  100 (110) 2  Power  across  110  volt = 484 110 ´ 110  Power = = 25 W .  (c) :  For  balanced  Wheatstone's  bridge.  18.  Hence  no  current  flows  through  arm  BD..  (a) :  If  internal  resistance  is  zero..  I  R  20.005ºC –1  R t  =  200 W \  R 100  =  R 0 [1  +  0. Þ  12R  =  1000+  2R Þ  R  =  100 W  21...  (c) : The equivalent circuit is a balanced Wheatstone's bridge..  we  get  100  = [1 + 0.  P =  R  Q S S1 S 2  Q  S  = S1 + S 2  ( Q S 1  and  S 2  are  in  parallel) R ( S1 + S 2 )  \  P  =  .  2  V 2  (200)  = = 400 W  P 100  400  = 40 W  .005t  =  2  +  1  or  t  =  400ºC.  (c)  :  Resistance  of  hot  tungsten  = 23.  27.70  JEE MAIN CHAPTERWISE EXPLORER  12.  (b)  :  Potential  difference  is  same  when  the  wires  are  put  in  parallel  rl  V = I1R1 = I 1  ´ 1 2  p r1  Again  V = I 2 R2 = I 2  ´ rl 2  pr2 2  .  (a)  :  For  meter  bridge  experiment.  26..005t]  Divide  (i)  by  (ii).005 ´ 100]  200 [1 + 0.  \  r = 2 ê ë 120  úû  or  E  = (Given) E  R 1  E  R 2  2ER 2  R1 + R2  + R or  R 1  +  R 2  +  R  =  2R 2  or  R  =  R 2  –  R 1 .  z 2  ö æ + 1  ç z1  ÷ø  è 24..  B  AB  and  BC  are  in  series \  R ABC  =  5  +  10  =  15 W  5 W  10 W  AD  and  DC  are  in  series \  R ADC  =  10  +  20  =  30 W  A  C  10 W  ABC  and  ADC  are  in  parallel ( R ABC )( R ADC  )  10 W  20 W  \  R eq  = ( R ABC + R ADC  )  D  I  or  Req  = 15 ´ 30 = 15 ´ 30  = 10 W 15 + 30 45  E  5  = = 0.l In  the  first  case.(i)  .  (c) :  Given  :  R 100  =  100 W a  =  0.  16.  (c)  :  Thermistors  are  made  of  metal  oxides  with  high  temperature  co­efficient  of  resistivity.  the  energy  source  will  supply  a  constant  current.  (c)  :  Resistance  of  full  coil  =  R  Resistance  of  each  half  piece  =  R/2 or  q 2  = \  H 2  V 2 t R  2  = ´ = H1  R / 2 V 2 t 1  \  H 2  =  2H 1  Heat  generated  will  now  be  doubled.  (a)  : Antimony­Bismuth  couple  is  ABC  couple.  I 1  =  I 2  12 2  or  500 + R = R .(ii)  13.  2E  R1 + R2  + R Q  E  –  IR 2  =  0  \  E  =  IR 2  19.  4X l 4  l  = Þ = Þ l  = 50 cm  .  (a) : Kirchhoff's first law [S i = 0] is based on conservation  of  charge  Kirchhoff's second law (S i R = S E) is based on conservation  of  energy..  R1 l1 l 1  = =  R2 l2 (100 – l1 )  X  = 20 = 20 = 1  Y 100 - 20 80 4  In  the  second  case.  (d)  :  For  zero  deflection  in  galvanometer.  \  Current  I  = Req  10  +  5 V  14.  25..  Mass  deposited  =  z 1 q 1  =  z 2 q 2 \  q1 z2 q1 + q2 z1 + z 2  = Þ = q2 z1 q2 z1  Þ z  ö q  æ = 1 + 2  ÷ q2 çè z1  ø q  .005  ×  100]    R t  =  R 0 [1  +  0.  (b)  :  The  internal  resistance  of  a  cell  is  given  by  æ l  ö æ l1 .  Y (100 .   (c)  :  qc + qi = 2 qn Þ  qi + qc  = qn .  è øè ø  2  28.5 W  1.5 A.  R 2  or  2 W  3 W  3 R ´ 100% =  300%  .  2  V 2  41.  (b) : Equivalent  resistance  = Q  S  =  nP 6 W  \ % change =  1000 ´ 30 ´ 4.5 W Þ  6 V  3 W  35.  (a)  :  In  series  combination.71 Current Electricity  I ´ rl I ´ rl I æ l ö æ r  ö \  1 2 1 = 2 2  2 Þ  1 = ç 2 ÷ ç 1  ÷ I 2 è l1 ø è r 2  ø pr1 pr2  2  2 l  \ The new resistance  R¢2  = r l ¢ 2 = r = 4 R  2  p r¢ p´ r 2  \ Change in resistance  = R¢ . its radius is going to be decreased in  such a way that the volume of the cylinder remains constant. +  R 2  R 2  . where current i is drawn from battery  1.5 W  6 V  I = 6  = 4 A.  qn  = - 32.  (b)  :  P 1  =  R when  connected  in  parallel.  R 48.  \  25 ´ 10 -6  39.  (b)  :  According  to  Faraday's  laws.5 W 1.  40.  \  P1  2 2  P2 = V = 4 V  = 4 P 1  R / 4  R 2  42.´ (700) = -350° C  2b 2  Neutral  temperature  is  calculated  to  be  –350°C  Since  temperature  of  cold  junction  is  0°C.3 ×  10 –7 )  ×  (3)  ×  (2)  =  19.5  30.  or  K × 30 = E¢ – ir.  P  = R1 R 2  ( R1 + R2 )  R R  \  (R 1  +  R 2 ) 2  =  nR 1 R 2  \  ( R1 + R2 )  = n  1 2  ( R1 + R2 )  For  minimum  value.  (c)  :  Resistance  of  bulb  = P 1000  Required  power  Þ  6 V  (3 + 3) ´ 3  18  = = 2 W (3 + 3) + 3 9 \  Current I  = V  = 3  = 1.  (a)  :  m  =  Z  i  t  or  m  =  (3.    d q  or  0  =  a  +  2bq n  or  E ´ 30  = E ¢ + 0.  (d)  :  E  =  aq  +  bq 2 dE  = a + 2 b q \  d q  a  1  = .5  =  Þ  mCu  31.5  38.  (c)  :  The  equivalent  circuits  are  shown  below  :  1.8  ×  10 –7  kg.18 J = 1cal]  or  2  V 2  (110)  110 ´ 110  = = = 250 W .  R 34.18  t= = 150 sec.5  mCu  = 0.  no  neutral  temperature  is  possible  for  this  thermocouple.  = mZn Z Zn  =  mCu Z Cu  when  i  and  t  are  same \  dE  = 0  At  neutral  temperature  (q n).  36.5 i or  E ¢ = 30 E  - 0.  R 1  =  R 2  =  R \  (R  +  R) 2  =  n(R  ×  R) Þ  4R 2  =  nR 2  or  n  =  4.18)  [Q  4.4 48.  m µ  It.13 ´ 31.5 i 100  100  2  V 2  (220)  = = 48.  31.  ( R / 2) ´ ( R / 2)  R  R eq  = = R R 4  \  + 2 2  P 2  =  4.  (b)  :  Power  = V  R \  150 = (15)2 (15) 2  225 225  = + Þ R  = 6 W . radius  of  the wire be r \  Resistance  R  = r l 2  r = resistivity of the wire  pr 100  Now l is increased by 100%  \ l ¢ = l + 100 l =  2 l As length is increased.  (d) :  Let the length  of  the  wire be l.5 = 0.  (c)  :  Electrical  energy  is  converted  into  heat  energy \  836  ×  t  =  1000  ×  1  ×  (40  –  10)  ×  (4.13 32.  where  E¢  is  emf  of  battery.  (c)  :  According  to  Faraday's  laws  of  electrolysis.126 g  32.  29.4  W  .4  37.  (c) :  Potential  gradient  along  wire  = E volt  100 cm  E  volt  \  K = 100 cm  For  battery  V  =  E¢  –  ir.R = 3 R I 1  æ 3 öæ 2 ö2  3 ´ 4 1  or  I = ç 4 ÷ç 3 ÷ = 4 ´ 9 = 3 .  1.  836  33.  (a)  :  Let  the  smallest  temperature  be q°C \  Thermo  emf  =  (25  ×  10 –6 ) q  volt  Potential  difference  across  galvanometer  =  IR  =  10 –5  ×  40  =  4  ×  10 –4  volt \  (25 ×  10 –6 )q  =  4  ×  10 –4 4 ´ 10 -4  q= = 16° C.  S  =  (R 1  +  R 2 )  In  parallel  combination.  r2 ´ l r2 ´ l r 2  2  pr 2 ´ l = pr ¢2  ´ l¢  Þ r ¢ = l ¢ = 2l = 2  0.   greater is the resistance of  ammeter. deuteron and alpha particle of the same kinetic energy  are  moving  in  circular  trajectories  in  a  constant  magnetic  field. Statement­II is  the  correct  explanation  of  Statement­I.  MAGNETIC EFFECTS OF  CURRENT AND MAGNETISM A metallic rod of length ‘l’ is  tied to a string of length 2l and  made  to  rotate  with  angular  speed w  on  a  horizontal  table  with one end of the string fixed. When  a current is passed through the coil it starts oscillating; it is  very  difficult  to  stop.6 × 10 –5  Wb/m 2 )  (a)  5. The variation of the magnetic field B along the line  XX¢ is given by  2  (d)  4 Bwl 2  (2013)  This question has Statement­I and Statement­II.  But  if  an  aluminium  plate  is  placed  near to the coil.  (d)  Statement­I is true. choose the one that best  describes the two Statements.  The  radii  of  proton.  (d)  development of air current  when  the plate  is  placed. Statement­II is true.  (c)  Statement­I is true. Statement­II is false.  (2012)  6.20 Am 2 and 1. additional  shunt needs to be used  across it. Of the four  choices given after the Statements.  If  there is a vertical magnetic  field ‘B’ in the region. They carry  steady equal currents flowing out of the plane of the  paper  as shown.  Two long parallel wires are at a distance 2d apart.0 cm.72  JEE MAIN CHAPTERWISE EXPLORER  CHAPTER  13  1.f. They are placed  on a horizontal table parallel to each other with their N poles  pointing towards the South. They have a common magnetic  equator and are separated by a distance of 20. it stops. This is due to  (a)  induction of electrical  charge  on the  plate.  (b)  shielding of magnetic lines of  force as aluminium is  a  paramagnetic material.  (c)  electromagnetic induction in the aluminium plate giving  rise to electromagnetic damping.  4.  A current I flows in an infinitely long wire with cross­section  in the form of a semicircular ring of radius R. The magnitude  of the  magnetic induction along its  axis is  m 0 I  m 0 I  m I  m I  (a)  2  (b)  (c)  0  (d)  0  2 p R 4 p R p  R 2 p 2 R (2011)  7.  A  coil  is  suspended  in  a  uniform  magnetic  field.56 × 10  Wb/m  (d)  3.  2  (c)  3 Bw l 2  (a)  ra  = r p  < r d  (c)  ra  = r d  > r p  Two short bar magnets of length  1 cm each  have magnetic  moments 1.  Statement­II : To increase the range of ammeter. Statement­II is  not the  correct  explanation  of  Statement­I.  with  the  plane of the coil parallel to the magnetic lines of force.  3. Which one of the following relation  is  correct?  (2012)  5.  (a)  Statement­I is false. The value  of  the  resultant  horizontal  magnetic  induction  at  the  mid­  point O of the  line  joining their  centres  is  close  to  (Horizontal  component  of  earth’s  magnetic  induction  is  3.  (2013)  (b)  ra  > r d  > r p  (d)  ra  = r p  = r d  B  (a)  X  X¢  d  d  B  (b)  X  X¢  d  d  B  (c)  X  X¢  d  d  B  (d)  X  X¢  d  d  (2010)  . Statement­II is true.m.  Statement­I : Higher the range.80 × 10 –4  Wb/m 2  (b)  3. induced across the ends of  the rod is  2  2  (a)  5 Bw l (b)  2 Bw l 2  2  2. Statement­II is true. r d  and ra .00 Am 2 respectively.  deuteron  and  alpha  particle  are  respectively r p . the e.  (b)  Statement­I is true.6 × 10­ –5  Wb/m 2  –4  2  (c)  2.50 × 10 –4  Wb/m 2  (2013)  Proton. 5  (d) e r  = 0.5.  (2006)  18. thin walled pipe. m r  =  0. A  magnet  when  brought  close  to  them  will  (a)  attract  all  three  of  them  (b)  attract  N 1  and  N 2  strongly  but  repel  N 3  (c)  attract  N 1  strongly. The wire AOB carries an electric  current I 1  and COD carries a current I 2 .  N 2  weakly  and  repel  N 3  weakly  (d)  attract  N 1  strongly.5  (b) e r  = 0.73  Magnetic Effects of Current and Magnetism  Directions : Question numbers 8 and 9 are based on the following  paragraph. The  current is uniformly distributed across its cross section.  (2009)  8.5 × 10 –7  T southward  (c)  5 × 10 –6  T northward  (d)  5 × 10 –6  T southward.  Two identical conducting wires AOB and COD are placed at  right angles to each other. Then  (a)  the magnetic field at all points inside the pipe is the same.  in  a  direction  perpendicular to the plane of the wires AOB and COD.05 × 10 –3  Wb/m 2 .  r  r  uniform  and  mutually  orthogonal  fields E  r  r  and B with  a  r  velocity  v  perpendicular to both  E  and B .  24 ab 10.5  (2008)  12.  A current loop ABCD is held fixed on  B the  plane  of the paper  as  shown  in  the  A  a  figure.  A  charged particle is released from rest in this region.  A horizontal overhead powerline is at a height of 4 m from  the ground and carries a current of 100 A from east to west.   N 2  and  N 3  are  made  of  a  ferromagnetic.  but not zero  (b)  the magnetic field is zero only on the axis  of  the pipe  (c)  the magnetic field is different at  different points inside  the pipe  (d)  the magnetic field at any point inside the pipe  is zero  (2007)  16.a ) + (a + b) ù (c)  (d)  û 4 p  ë ab û 4p  ë 3  Due to the  presence  of  the current  I 1  at  the  origin  (a)  the  forces on  AB and DC are  zero  (b)  the  forces on  AD and BC are  zero  (c)  the  magnitude  of  the  net  force  on  the  loop  is  given by  I1 I  é p m 2(b . The  ratio of the magnetic field at a/2 and  2a is  (a)  1/2  (b)  1/4  (c)  4  (d)  1  (2007)  17. These  two  fields  are  parallel  to  each  other. will  be given by  1  m 0  ( I 2 + I 2 2 )  (a)  2 p d 1 (c)  1  m 0  2 ( I1 + I 2 2 ) 2  2 p d m æ I + I ö 2  (b)  0  ç 1 2 ÷ 2 p è d ø  (d)  m0  ( I + I  )  2 p d 1 2  (2007)  13.05 ×  10 –5  Wb/m 2  (d)  1.05 ×  10 –4  Wb/m 2  (b)  1. Angle  C  made by AB and CD at the origin O is  30°.  (2006)  19. m r  = 1.  (2006)  20.a ) + ( a + b) ù û 4p  0  ë 3  (d)  the  magnitude  of  the  net  force  on  the  loop  is  given by  m 0 II 1  (b - a ).5 × 10 –7  T northward  (b)  2.  In a region. The arcs BC (radius = b) and DA  I  (radius = a) of the loop are joined by  I 1  O  30°  two straight wires AB and CD. A steady  b  D  current I  is flowing in the loop.  A long solenoid has 200 turns per cm and carries a current  i. momentum and kinetic energy of  the particle  are  constant  (2007)  14. The magnetic field on  a  point  lying  at  a  distance  d  from  O.05 ×  10 –2  Wb/m 2  (c)  1. The magnetic field at  its  centre is 6.a ù m 0 I  é p 2(b . m r  = 0. The  value  of  the  magnetic  field  at  its  centre  is  (a)  1.  Needles  N 1.5.  If  an  electron  is  projected  along  the  direction  of  the  fields  with  a  certain  velocity  then . respectively.  The magnitude of the magnetic field (B) due to loop ABCD  at the origin (O) is  m I (b - a )  (a)  zero  (b)  0  24 ab  m 0 I  é b . Then  r r r r r  r  2  2  (a)  v = B ´ E / E (b)  v = E ´ B / B r r r r (c)  vr  = B ´ E / B 2  (d)  vr  = E ´ B / E 2  (2007)  15.28 × 10 –2  weber/m 2 .  a  paramagnetic  and  a  diamagnetic  substance  respectively.5.  A long straight wire of radius a carries a steady current i. Then  (a)  kinetic energy changes but the momentum is constant  (b)  the momentum changes but the kinetic energy is constant  (c)  both  momentum and  kinetic  energy  of the  particle  are  not constant  (d)  both.5. and comes  out  r  without any change in magnitude  or direction of  v  . m r  =  1.  A  uniform  electric  field  and  a  uniform  magnetic  field  are  acting  along  the  same  direction  in  a  certain  region.  (2008)  11.  A charged particle with charge q enters a region of constant.  straight.  9. The path  of  the  particle  will  be  a  (a)  circle  (b)  helix  (c)  straight line  (d)  ellipse. Which of the following values of these  quantities are allowed for a diamagnetic  mateiral?  (a) e r  = 1.5  (c) e r  = 1.  A  current  I  flows  along  the  length  of  an  infinitely  long.  The  magnetic  field  directly below  it on  the  ground is  (m 0  = 4p × 10 –7  T m A –1 )  (a)  2.  Another long solenoid has  100 turns per cm and it carries  a  current  i/3. steady and uniform electric and magnetic fields  are  present.  A  charged  particle  moves  through  a  magnetic  field  perpendicular to its direction.  Relative permittivity and  permeability of a material are e r  and m r .  but  repel  N 2  and  N 3  weakly. Another straight  thin  wire  with steady  current  I 1  flowing  out  of  the  plane  of  the  paper is  kept  at the origin.   The  length  of a  magnet is large compared  to its  width  and  breadth.  They  will  (a)  attract  each  other  with  a  force  of  (b)  repel  each  other  with  a  force  of  (2p d 2 )  m 0 i 2  28.  The magnetic field due to a current carrying circular loop of  radius 3 cm at a point on the axis at a distance of 4 cm from  the  centre is 54 mT.  A particle of mass M and charge Q moving with velocity  v  describes a circular path of radius R when subjected to a uniform  transverse  magnetic field  of induction  B.  (2004)  m0 i 2  (2p d )  (2005)  25. They exert  a force F on each  other. separated by a distance d carry  a  current  of i  A  in  the  same  direction. Now the current in one of them is increased to two times  and its direction is reversed.  The  time  period  of  its  oscillation  in  a  vibration  magnetometer is 2 s.  The materials suitable for making electromagnets should have  (a)  high  retentivity  and  high  coercivity  (b)  low  retentivity  and  low  coercivity  (c)  high  retentivity  and  low  coercivity  (d)  low  retentivity  and  high  coercivity.  A particle of charge –16 × 10 –18 coulomb moving with velocity  10 ms –1  along the  x­axis enters a  region where  a  magnetic  field of induction B is along the y ­axis.  In  order  that  each  division  reads  1  volt. parallel wires.  Two thin long. The distance is also increased to  3d.  3 ampere and 4 ampere are the currents flowing in each coil  respectively.  An  ammeter reads upto 1 ampere. The work  done  by  the  field  when  the  particle  completes  one  full  circle  is .  A  charged    particle  of  mass  m  and  charge  q  travels  on  a  circular path of radius r that is perpendicular to a magnetic  field  B.74  JEE MAIN CHAPTERWISE EXPLORER  (a)  it  will  turn  towards  right  of  direction  of  motion  (b)  it  will  turn  towards  left  of  direction  of  motion  (c)  its  velocity  will  decrease  (d)  its  velocity  will  increase  (2005)  21.  (2003)  r  34.09 W. then the magnetic induction at any point inside  the  tube  is  (a)  infinite  (b)  zero  m 0  2 i  2 i  tesla  × tesla  (c)  . It is bent into a circle  of  one  turn  and the  magnetic field  at the  centre  of  the  coil  is  B. separated by a distance d carry current  I 1  and I 2  in the same direction.  The  time  period  of  this  combination  will  be  2  æ 2  ö (d)  ç (a)  2 s  (b)  s  (c)  (2 3) s  ÷ s  3  è 3 ø  (2p d 2 )  (2004)  m0 i 2  (2p d )  32.  It  is  then  bent  into  a  circular  loop  of  n  turns. and an electric field  of  magnitude  10 4  V/m  is  along  the  negative  z ­axis.  Two  concentric  coils  each  of  radius  equal  to  2p  cm  are  placed  at  right  angles  to  each  other.  Two long conductors.  the  magnitude  of  B  is  (a)  10 3  Wb/m 2  (b)  10 5  Wb/m 2  16  2  (c)  10  Wb/m  (d)  10 –3  Wb/m 2 .  A magnetic  needle is kept in a non­uniform magnetic field. The magnetic induction in weber/m 2 at the center  of  the  coils  will  be  (m 0  =  4p ´  10 –7  Wb/A­m)  (a)  5 ´  10 –5  (b)  7 ´  10 –5  (c)  12 ´  10 –5  (d)  10 –5  (2005)  23.  The  time  taken  by  the    particle  to  complete  one  revolution  is  2 p m  2 p qB  (b)  (a)  qB m 2 p mq  (2005)  (c)  (d)  2pmq  B qB  22.  To  increase  the  range  to  10 A  the  value  of  the  required  shunt  is  (a)  0. Its internal resistance  is  0.3 W  (c)  0.81  ohm.  (2003)  (c)  attract  each  other  with  a  force  of  (d)  repel  each  other  with  a  force  of  m 0 i 2  27. The magnet is cut along its length into  three equal parts and three parts are then placed on each other  with  their  like  poles  together.  A current i ampere flows along an infinitely long straight thin  walled tube.  If  the  charged  particle  continues  moving  along  the  x­axis.  The  new  value  of  the  force  between  them  is  (a)  –2F  (b)  F/3  (c)  –2F/3  (d)  –F/3.  (2004)  29.  (2004)  31.  (2004)  (d)  r  4 p  r 30.  It  experiences  (a)  a  force  and  a  torque  (b)  a  force  but  not  a  torque  (c)  a  torque  but  not  a  force  (d)  neither  a  force  nor  a  torque  (2005)  26.  Its  current sensitivity is 10 divisions per milliampere and voltage  sensitivity  is  2  divisions  per  millivolt.  (2004)  33.  A  moving  coil  galvanometer  has  150  equal  divisions.9 W  (d)  0.  The  magnetic  field  at  the  centre  of  the  coil  will  be  (a)  nB  (b)  n 2 B  (c)  2nB  (d)  2n 2 B. What will be  its value at the  centre  of  the  loop?  (a)  250 mT  (b)  150 mT  (c)  125 mT  (d)  75 mT.  A long wire carries a steady current.03 W  (b)  0.  the  resistance  in  ohms  needed  to  be  connected  in  series  with  the  coil  will  be  (a)  99995  (b)  9995  (c)  10 3  (d)  10 5  (2005)  24.  c)  (c)  (a)  (b)  (d)  (c) 2.  then  (a)  curved path of electron and proton will be same (ignoring  the  sense  of  revolution)  (b)  they  will  move  undeflected  (c)  curved path of electron is more  curved than  that of  the  proton  (d)  path  of  proton  is  more  curved.  13.  22.  20.  33.  If  in a  circular coil  A of  radius  R.  If  an  electron  and  a  proton  having  same  momenta  enter  perpendicular  to  a  magnetic  field.  (c)  (c)  (b)  (b)  (b)  (b)  (a)  6.  (2002)  43.  2 2  2  4  (2003)  36.  (2002)  42.  21.  If  its  period  of  T ¢  oscillation  is  T ¢ .  29. current  I  is  flowing  and  in another coil B of radius 2R  a  current 2I is flowing.  7. then  the ratio of the magnetic fields.  31.  42.  17.  A magnetic needle lying parallel to a magnetic field requires  W units of  work to turn  it  through  60°.  A thin rectangular magnet suspended freely has  a period of  oscillation equal to T.  36.  34.  (2003)  37.  (2002)  40.  (2003)  35.  12.  41.  (c)  (b)  (d)  (b)  (a)  (a)  (a)  4.  40.  (2003)  38.  11.  16.  27.  28.  35.  The time period of a charged particle undergoing a circular  motion  in  a  uniform  magnetic  field  is  independent  of  its  (a)  speed  (b)  mass  (c)  charge  (d)  magnetic induction.  30.  The  torque  needed  to  maintain  the  needle  in  this  position  will  be  3  W  (a)  3W  (b)  W  (c) (d) 2W.  24. Now it is broken into two equal halves  (each  having  half  of  the  original  length)  and  one  piece  is  made  to  oscillate  freely  in  the  same  field.  10.  39.  18.  (a)  (b)  (b. produced by them  will  be  (a)  1  (b)  2  (c)  1/2  (d)  4.  38.  43.  9.  The  magnetic  lines  of  force  inside  a  bar  magnet  (a)  are  from  north­pole  to  south­pole  of  the  magnet  (b)  do  not  exist  (c)  depend upon the area of cross­section of the bar magnet  (d)  are  from  south­pole  to  north­pole  of  the  magnet.  8.  If a  current is passed through a spring then the  spring will  (a)  expand  (b)  compress  (c)  remains same  (d)  none  of  these.  19.  (a)  (d)  (d)  (a)  (b)  (b)  (b)  5.  15.  (2002)  41.  2  ( )  39.  If  an  ammeter  is to  be used in  place  of  a  voltmeter.  Curie  temperature  is  the  temperature  above  which  (a)  a  ferromagnetic  material  becomes  paramagnetic  (b)  a  paramagnetic  material  becomes  diamagnetic  (c)  a  ferromagnetic  material  becomes  diamagnetic  (d)  a paramagnetic  material  becomes  ferromagnetic.  26.  32. B A  and B B .  37.  25.  (a)  (c)  (c)  (c)  (b)  (a)  (a)  .  14.75  Magnetic Effects of Current and Magnetism  æ Mv 2  ö (a)  ç ÷ 2 pR  è R ø  (c)  BQ 2pR  (2003)  (b)  zero  (d)  BQv 2pR.  (2002)  Answer  Key  1.  the  we  must  connect  with  the  ammeter  a  (a)  low  resistance  in  parallel  (b)  high  resistance  in  parallel  (c)  high  resistance  in  series  (d)  low  resistance  in  series.  (a)  (b)  (b)  (c)  (c)  (d)  (a)  3.  23.  the  ratio  is  T 1  1  1  (a)  (b)  (c)  2  (d)  . m.  (b.  11.  their fields  are  equal  and  opposite  and  their  effects also  cancel  each  other.76  JEE MAIN CHAPTERWISE EXPLORER  m : 2m : 4 m  = 1: 2 :1  e e 2 e Þ  ra  =  r p  < r d  1. If it is a.  According to the  table  given.c.f.  (a)  A  They  do  not  contribute  to  the  a  magnetic induction at O. induced  across  the  ends  of the  rod is  3 l  3 l  é x 2  ù B w e = ò Bw xdx = Bw ê ú = [(3l )2 .  K =  1 mv 2  or  v  =  2 K  2  m m 2K 2 Km  =  qB m qB As  K and B are constants  \ r = \ r µ  m  q rp : rd  : r a =  m p  md  m a : :  qp qd  qa r  By Ampere’s theorem.  The  total magnetic  field due to loop  ABCD  at  O  is  B =  BAB  +  BBC  +  BCD  +  BDA  3.  Therefore.* (d) :  i  -7  Bnet  = 4p ´ 10  [1.6 ´ 10 -5  4p ´ (10 ´ 10 -2 ) 3  -7  = 10 -3  ´ 2. F = qv × B.5 and m r  = 0.  When  a  charged  particle  enters  a  field  with  its  velocity  perpendicular to the magnetic field.  Bnet  = 10.  12.2 + 1] + 3.6 ´ 10 -5  10 = 2.  m 0  2 ( I + I 2 2 ) 1/ 2  . the  e. one takes e r  = 1.  (c) : The field at the same point at the same distance from the  mutually perpendicular wires carrying current will be having  the same magnitude but in perpendicular directions.  (a)  As  the  point  O  lies  on  broad­side position with  respect  to  both  the  magnets.  (a) :  = Consider  a  element  of  length  dx  at  a  distance  x  from  the  fixed  end of  the  string. There will be no force.36 × 10 –4  = 2.  e.  Þ B = 0 - m 0 M 1 m 0  M 2  m + + B H  = 0 3  ( M 1 + M 2 ) + B H  4p r 3 4 p  r 3  4 p r Substituting the given  values.  (a) : The radius of the circular path of a charged particle in  the  magnetic  field  is given  by  r =  mv  Bq Kinetic  energy of  a  charged  particle.  4.2 + 3.  The  net magnetic field  at  point  O  is  B net  =  B 1  +  B 2  +  B H  7.  *  It is assumed that this is a direct current.(2l )2 ]  ë 2 û 2 l  2  2 l  =  5 Bw l 2  2  5.  2.  (c)  6.f.56  × 10 –4  Wb/m 2 . the  current at the given instant is in the given direction. Due to parts  AB  and  CD. c) : Due to Lorentzian force.  (b) : O is along the line CD and AB.  (b)  Þ B= m0 I p m I  p ´ + 0 + 0  ´ 4pb 6 4 pa 6  m 0 I  (b - a ) . The field  I  I 1  30°  O  due to DA is positive or out of the  b  D  paper  and  that  due  to  BC  is  into  C  the  paper or negative.  out of  the  paper  or positive.  24 ab 9.  B × 2 pd = m 0 i r  m i  4p ´ 10 -7 ´ 100A  = 50 ´ 10 -7 T  B = 0  = 2p ´ 4 m 2 pd Þ  B  = 5 × 10 –6  T southwards. induced in  the  element  is  de =  B(wx)dx  Hence.  (b) : The straight wire is perpendicular to the segments and  the fields are parallel. Then the  choice (c) is correct. the motion is circular .m.2 × 10 –4  + 0.5. we  get  B 8.  \ B = B12 + B2 2  \ B = 2 p d 1 13.  (c) : The situation is as  shown in  the  figure.  (c)  :  The  values  of  relative  permeability  of  diamagnetic  materials  are  slightly  less  than  1  and e r  is  quite  high.   (c) : Magnet  will attract N 1  strongly.  è 2  ø  Therefore  the  answer  is  (b). if v  is given  only  as  E/B.(i)  .  perhaps  by  slip. (c) is the answer.  a  high  resistance  should  be  connected  in  series  with  the  galvanometer  V = I g ( G + R ) = or  5+ R = (5 + R )  15  (5 + R ) Þ 150 = 15  1000 1000  150 ´ 1000  = 10000  \  15  R  = 9995 W. v  =  E  ..  (a)  :  Field  along  axis  of  coil  B  = At  the  centre  of  coil.  (b)  :  T  = w  = v centripetal  force  =  magnetic  force Q  mv 2  = qvB Þ v  = qBr  \  r m From  (i)  and  (ii) 2 pr ´ m 2 pm  T  = =  .  The  path  of  charged  particle  will  be  a  straight  line..(ii)  22..  16. (Compare to electric field inside  a hollow sphere).  Therefore  its  tangential  momentum  changes  its  direction  but  its  energy  remains  the  same  with  qvB  =  æ 1 I w 2  = constant ö .  B ¢ = m 0 i  2 R m0 iR 2  2( R 2 +  x 2 ) 3/ 2  ..  (a)  :  A force and a  torque  act on a magnetic needle  kept  in  a  non­uniform  magnetic  field. The  two  forces oppose  B r r r  E ´ B  r r  v  = each other if v is along  E ´ B i.  24.  F ¢ = \  \  m0  I1 I 2  l  2 p  d m 0  ( -2 I1 )( I 2 )  l  2p 3 d F ¢ -m 0  2 I1 I 2 l  2 pd  2  = ´ =F 2 p 3d m 0 I1 I 2 l 3  F¢  =  –  2F/3..05  ×  10 –2  Wb/m 2 ..B..  m 0 i 2 L  ..  (b)  :  Vmax  = 2  = 75 m V  150  I max  = = 15 mA =  I g  10  Resistance  of  galvanometer  G  =  75/15  =  5 W  For  conversion  into  a  voltmeter.  B 2  = m 0n    2I   2 B2 n2 I 2  100 i / 3 1  = ´ = ´ = \  B1 n1 I1  200 i 6  \  B2  = B 1  6. N 2  weakly  and repel  N 3  weakly.  B 2  r  r  As  E  and  B  are perpendicular to each other  r r  E ´ B = EB sin 90 ° = E  B  B2 B 2  For  historic  and  standard  experiments  like  Thomson’s  e/m  value..  (c)  :  Force  of  attraction  between  wires  = 25. Iw is a constant.  B 1  = m 0 n 1I    1  In  second  case.  (b) : When  E  and  B  are perpendicular and velocity has no  changes then qE  =  qvB  i.  2  *  The questions could have been more specific..  As  1  2  Iw  is also constant.  v  constantly  changes  its  direction  (but  r not  the  magnitude).  \  qBr qB .  -7  m 0  2 2  = 4 p ´ 10  (3)2 + (4) 2  i1 + i 2  2 ´ (2 p ´ 10 -2 ) 2 r 150  23.  (d) : Magnetic field is shielded and no current is inside the  pipe to apply Ampère’s law.  2 p 2 pr  21..  r  r  14.e.  26.  19.  (a)  :  Magnetic  induction  at  centre  of  one  coil  B 1  =  Similarly  B 2  = m 0 i 1  2 r m 0i 2  2 r \  2 2  m 0 2  2 2  æ m i ö æ m i  ö B 2 = B12 + B 2 2  = ç 0 1 ÷ + ç 0 2  ÷ = 2  ( i1 + i 2  )  è 2 r ø è 2 r ø  4 r \  B= or  B  =  5 ×  10 –5  Wb/m 2 .28 ´ 10 -2  =  =  1.  20.  The  options  do  not  mention  L..  (c) :  Magnetic  field  applied  parallel  to  motion of  electron  exerts no  force  on it as q  = 0 and  force  =  Bevsinq  =  zero  Electric  field  opposes  motion  of  electron  which  carries  a  negative  charge \  velocity  of  electron  decreases.  2 p d N.  B 2  R a 2  17..  15..e.  If angular momentum is taken..  (c) :  Initially.  (b)  :  In  first  case.  27.  6 6  18. although the answer is numerically  correct.  Current  enclosed  in  the  1 st  ampèrean path is  \  I × pr12 = Ir 1 2  pR 2 R 2  m ´ current  m 0 × Ir12  m 0 Ir 1  B  = 0 = = path  2 pr1 R 2 2 p R 2  Magnetic induction  at a distance  r 2  = \  m 0  × I  2 p r2  a  B1 r1r 2  2 × 2 a  = 2 = = 1 .77  Magnetic Effects of Current and Magnetism  mv 2  .  .. whether by  “momentum” it is meant tangential momentum or angular  momentum....  it  would  have  been  better  from the pedagogic view.  (d)  :  Uniform  current  is  flowing.  (c)  :  Magnetic  field  exerts  a  force  = Bevsinq  =  Bevsin0  =  0  Electric  field  exerts  force  along  a  straight  line..  F = Finally. (i)  Torque  =  MB  sin  60°  =  (2W)  sin  60°  = Ig I g G  S  32...  (b) : The spring will compress.  35.  B x  =  B z  =  0 q  \  a x  =  a y  =  0...  (a) : Particle travels  along x­axis.  (b)  :  Workdone  by  the  field  =  zero..  T  = 2 p  I  MB where  I  =  ml 2 /12.  r 1  =  radius  of  coil  =  l/2p m i 2 m i p B = 0 =  0  \  2 r1  2 l l  Finally..  (b) :  For  an oscillating  magnet..  (d)  :  The  magnetic  lines  of  force  inside  a  bar  magnet  are  from  south  pole  to  north  pole  of  magnet..  \ Time  period  T¢  = 2 p 30...  (b) : Materials of low retentivity and low coercivity are suitable  for  making  electromagnets.  (a)  :  Bqv = mv Þ r  = mv  = r Bq Bq r  will  be  same  for  electron  and  proton  as  p..  (c) : High resistance in series with a galvanometer converts  it  into  a  voltmeter.  38.  B  and  q  are  of  same  magnitude.  vx  10  m  34....  B x  =  B z  =  0  Electric  field  is  along  the  negative  z­axis.Ez + vx B y )  m .  Hence  v y  =  v z = 0  Field  of  induction  B  is  along  y­axis...81 0.  m 0 2 pI m 0  I  = 4p R 2  R B A I A R B  æ 1 ö æ 2 ö = ´ = = 1  BB I B RA  çè 2 ÷ø çè 1 ÷ø  42.  v y =  v z  =  0.  a z = ( .  \  T ¢ = I ¢ ´ M = I ¢ ´ M  T M¢ I I M ¢  \  T ¢ = 1 ´ 2 =  1 .  (b) :  For  a  vibrating  magnet.  Again  a z  =  0  as  the  particle  traverse  through  the  region  undeflected \  E z  =  v x  B y  or  B y  = E z  10 4  Wb  = = 10 3  2  ..  (b)  :  Magnetic  field  will  be  zero  inside  the  straight  thin  walled  tube  according  to  ampere's  theorem. .  r 2  =  radius  of  coil  = 2 pn m 0i ´ n nm 0i ´ 2 pn 2 m0 in 2 p = = 2 r2  2l 2 l \  B ¢ = \  2  B ¢ 2 m 0 in  p 2 l  = ´ = n 2  B 2l 2 m 0 ip \  B¢=  n 2  B.  2  39. It will be  on account of force of attraction between  two  adjacent  turns  carrying  currents  in  the  same  direction..  (a) : A ferromagnetic material becomes paramagnetic above  Curie  temperature.  (a)  :  mRw2  = BqRw Þ w = Bq  Þ T  = 2 pm  m Bq T  is  independent  of  speed.(iii) 36...81  = = 0.(i)  2 2  2  Mass× (length)  I ¢ =  12  ( m / 2)( l / 2) 2  ml 2  I = = = ..  B ¢ =  28.  (a)  :  B  = az = \  43..  (b)  :  Initially.09 W in  parallel.  T  = 2 p  I  MB where  I  =  ml 2 /12.  (d)  :  S + G = I Þ S  = I .78  JEE MAIN CHAPTERWISE EXPLORER \  \  or  2 2 3/2 ( R 2 + x 2 ) 3 / 2  B ¢ m 0 i  2( R + x ) = ´ = B 2 R  m 0 iR 2 R 3  2 2 3/ 2  B ´ ( R2 + x 2 ) 3/ 2  = 54 ´ [(3) + (4) ]  =  54 ´ 125  3  3  27  (3) R B¢  =  250 mT.  40...  T 8 1 2  31..  37.  M  =  xl.  x  =  pole  strength  When the magnet is divided into 2 equal parts.  M  =  xl.  2  p  41.  (a)  :  W  =  –  MB  (cos q 2  –  cos q 1 )  MB = –  MB  (cos  60°  –  cos  0)  = 2  \  MB  =  2W  .  10 - 1 9  33. the magnetic  dipole  moment  x ´ l M  M ¢ = Pole strength × length  = =  .I g  \  I¢ M ¢B 2W  ´ 3  =  3 W .  x  =  pole  strength  of  magnet  2  m lö 3  ml 2  I  I ¢ = æç öæ ´ = = (For  three pieces  together)  ÷ç ÷ è 3 øè 3 ø  12 9 ´ 12 9  M ¢ = ( x ) æç l  ö÷ ´ 3 = xl = M (For  three  pieces  together) è 3 ø  \  T ¢ = 2 p \  T¢ = I ¢ = 2 p I / 9  = 1 ´ 2 p I = T  M ¢ B MB 3 MB 3  T  2  =  sec.  E x  =  E y  =  0 r r r  r \  Net  force  on  particle  F = q ( E + v ´ B )  Resolve  the  motion  along  the  three  coordinate  axis F  q  a x = x  = ( E x + v y Bz - v z B y )  \  m m S= ay = F y  q  = ( E + v z Bx - v x B z )  m m y F z  q  = ( E + v x B y - v y B x )  m m z Since  E x  =  E y  =  0.(ii) 12 12 ´ 8 8  29..  3 3  1 ´ 0..  On taking out the inductor from  the  circuit  the  current  leads the  voltage by  30°. I  =  2 Blv  Blv  (c)  I1 = I 2  = (d)  I1 = I 2  = I  =  A fully charged capacitor C with initial charge q 0 is connected  to a coil of self inductance L at t = 0.  On taking out the capacitance from the circuit the current lags  behind the voltage by 30°. The distance between their  centres  is  15  cm. Which of the  following  statement  is  correct?  (a)  At t =  6. The centre of the  small loop  is on the axis of the bigger loop. Calculate the value of R  to make the bulb light up 5 s after the switch has been closed.79  Electromagnetic Induction and Alternating Current  CHAPTER  14  1.  3.1 × 10 –11  weber  –11  (c)  6 × 10  weber  (d)  3. S 2  kept open.15 mV  (2011)  5. Suppose t 1  is the time taken for the energy stored  in the capacitor to reduce to half its initial value and t 2  is the  time  taken  for  the  charge to  reduce  to  one­fourth  its  initial  value.  The  current through the battery is  (a)  V ( R1 + R 2 )  at t = 0 and V  at t  = ¥  R1R2 R2  (b)  VR1R 2  2 1 2  2  at t = 0 and R +  R K  V  L  R 1  R 2  V  at t  = ¥ R 2  (c)  V  at t = 0 and V ( R1 + R 2 )  at t  = ¥  R2 R1R2  LC (2011)  A resistor R and 2 mF capacitor in series is connected through  a  switch  to  200 V  direct  supply.4)  (a)  1.7 × 10 5 W  6 (c)  2. then  the  flux linked  with bigger  loop is  (a)  6.  Let C be the capacitance of a capacitor discharging through  a resistor R.  q =  CV/2  (d)  At t = 2t.3 cm lies parallel to a much bigger  circular loop of radius 20  cm.7 × 10  W  (d)  3. the magnitude of the induced emf in the  wire of aerial is  (a)  1 mV  (b)  0.  t .50 mV  (d)  0.  The  I 1  Q  set­up is placed in a uniform  magnetic field  going into  the plane  of the  paper. Now switch  S 1  is closed.I 2  = 3 R Blv . The  three  currents I 1. q =  CV(1  – e –1 )  2 (b)  Work done by the battery is half of the energy dissipated  in the resistor  (c)  At t = t.  (log 10 2. If the speed of the  boat is 1.6 × 10 –9  weber  (b)  9.  The boat carries a vertical aerial 2 m long.50 m s –1 .75 mV  (c)  0.  VR1R 2  R12 +  R2 2  at t  = ¥ (2010)  In  a series  LCR circuit  R  = 200 W and the voltage  and  the  frequency of the main supply is 220 V and 50 Hz respectively.5 = 0.  A circular loop of radius 0.3 × 10 4 W  (b)  1. The  power  dissipated in the LCR circuit is  (a)  242 W  (b)  305 W  (c)  210 W  (d)  zero  W  (2010)  .  4. Across  the  capacitor  is  a  neon bulb that  lights up at 120 V.  the  key  K  is  closed  at  t  =  0. I    2  and I are  (d)  V  at t = 0 and R 2  9. (q is charge  on  the  capacitor  and t  =  RC  is  capacitive time constant). The time at which the  energy is stored equally between the electric and the magnetic  fields  is  (a)  p  LC P  l  A rectangular loop has a  sliding  connector  PQ  of  length l and resistance R W  R W  R W  v  R W  and  it  is  moving  with  a  I  I 2  speed  v  as  shown.  ELECTROMAGNETIC INDUCTION  AND ALTERNATING CURRENTS In an LCR circuit as shown below both  switches are open initially.  In  the circuit shown below.  Blv . I  =  2 Blv  (b)  I1 = .  If  a  current  of  2. I  =  Blv  (a)  I1 = I 2  = 6R (b)  p  LC 4  (c)  2 p  LC (d)  3R R R 3 R R (2010)  7.3 × 10 7 W  (2011)  Blv .0 × 10 –5 N A –1 m –1 due north and horizontal.0  A  flows  through  the  smaller loop.3 × 10 –11  weber  (2013)  A  boat  is  moving  due  east  in  a  region  where  the  earth’s  magnetic field is 5. q =  CV(1  – e –2 )  (2013)  2. Then  the ratio t 1 /t 2  will  be  (a)  2  (b)  1  (c)  1  2  (d)  1  4  (2010)  8.  After a long time  A  B  the battery is disconnected after short  E  circuiting  the  points  A  and  B.  developed  between  the  two  ends of the  conductor is  (a)  5 mV  (b)  50 mV  (c)  5 mV  (d)  50 mV.t /0. If each tube  D C  moves towards the other  ´ ´ ´ ´ ´ ´  at a constant speed v.  In a series resonant LCR circuit.  will  be  (a)  zero  (b)  2Blv  (c)  Blv  (d)  –Blv  (2005)  23.c.  The  axis  of  rotation  is  perpendicular to the field.8  (d)  0.3 s  (c)  0.  A circuit has a resistance of 12 ohm and an impedance of 15  ohm.  The  ´ ´ ´ ´ ´ magnetic  field  B  is  ´ perpendicular to the plane  ´ ´ ´ ´ ´ ´ of the figure. 2 second after the connection is  made. The current reaches half of its  steady state  value in  (a)  0.  The  switch  S  is  closed  at  t  =  0. the current flowing in ampere in the circuit is  (a)  (1 – e –1 )  (b)  (1  – e)  (c)  e  (d)  e –1  (2007)  13.5 × 10 –2  V  (c)  40 V  (d)  250 V.4  (2005)  20.  In a uniform magnetic field of induction B a wire in the form  of a semicircle of radius r  rotates about the diameter of the  circle  with  angular  frequency w. their mutual inductance is (m 0  = 4p × 10 –7  T mA –1 )  (a)  2. the inductance  should be  changed from L  to  (a)  4L  (b)  2L  (c)  L/2  (d)  L/4.  In order to impart maximum power at 50 Hz. the voltage across R is 100  volts and R = 1 kW with C = 2 mF.  L  (2005)  19.25  (b)  0. circuit  the  voltage  applied  is  E  =  E 0  sin wt.  The  potential drop across L as a function  of  time  is  (a)  6e –5t  V  (b)  E  R 1  S  L  R 2 12  - 3 t  e  V  t (c) 6 (1 - e .  l and v  where  l  is  the  width  of  each  tube.  (2004)  8 R 2 R .4p ×  10 –5  H  –4  (c)  4. If one of the solenoids has 300 turns and the other 400  turns.  One conducting  U  tube  can slide  inside  another  as  shown  in  figure. For  the resonant frequency to remain unchanged.  In an AC generator.  (2008)  12. The power factor of the circuit  will be  (a)  1.  An inductor of inductance L = 400  mH  and  resistors  of  resistances  R 1  = 2 W and R 2  = 2 W are connected  to a battery of emf 12 V as shown  in the figure. If  the  horizontal  component  of  earth’s  magnetic  field  is  0.8p × 10  H  (d)  4.  The  pö æ resulting current in the circuit is I = I 0 sin ç wt . At resonance  the voltage  across L is  (a)  4 × 10 –3  V  (b)  2. Which of the following cannot be the constituent  of  the  circuit?  (a)  LC  (b)  L alone  (c)  C  alone  (d)  R. it should be  connected  to  a  capacitance  of  (a)  1 mF  (b)  2 mF  (c)  4 mF  (d)  8 mF  (2005)  21.80  JEE MAIN CHAPTERWISE EXPLORER  10.125  (c)  0.f.15 s  (b)  0.05 s  (d)  0. (2006)  16.  maintaining  electrical  contacts  ´ ´ ´ ´ ´ ´ A  B  between  the  tubes.  (2006)  18.8p ×  10 –5  H.  The  phase difference between the  alternating current  and  emf is p/2.1 s (2005)  22.m. If the total resistance of the circuit  is  R the mean  power generated per period of rotation  is  2  ( B pr 2w ) 2  (a)  Bpr  w  (b)  8 R 2 R 2  ( B p r w 2 ) 2  ( Bpr w )  (d)  (c)  .  The  current in the circuit 1 ms after the short circuit is  (a)  1 A  (b)  (1/e) A  (c)  e A  (d)  0.1 A.  The self inductance of the motor of an electric fan is 10 H.  then  the  e. a resistor  L  (R = 100 W) and a battery (E = 100  V) are initially connected in series as  R  shown in the figure. The resonant frequency w is 200 rad/s.2 ) V  (d)  12e –5t  V  (2009)  11.  The power  è ø  consumption in the  circuit  is given by  E0 I 0  (a)  P =  2 E0 I 0  (b)  P =  2  E (c)  P = zero  (d)  P =  0 I 0  (2007)  2  14. then  the emf induced  in the circuit in terms of B.  Two coaxial solenoids are made by winding thin insulated wire  over a pipe of cross­sectional area  A = 10 cm 2  and length =  20 cm. The induced emf at t  = 3 s is  (a)  190 V  (b)  –190 V  (c)  –10 V  (d)  10 V.2  ×  10 –4  T.4p × 10 –4  H  (b)  2.  (2004)  24.  An ideal coil of 10 H is connected in series with a resistance  of 5 W and a battery of 5 V.  An inductor (L = 100 mH). all of the same area  A and total resistance R.  (2006)  15.  A coil of inductance 300 mH and resistance 2 W is connected  to a source of voltage 2 V. The maximum value of emf generated in the coil is  (a)  NABw  (b)  NABRw  (c)  NAB  (d)  NABR  (2006)  17.  The  flux  linked  with  a  coil  at  any  instant  t  is  given  by f = 10t 2  – 50t + 250. rotates with frequency w in a magnetic  field B.  (2004)  25. a coil with N turns.2 ÷ .  A  metal  conductor  of  length  1  m  rotates vertically  about  one  of its ends at angular velocity 5 radian per second.  In  an  a.  In a LCR circuit capacitance is changed from C to 2C. The internal resistance  of  the  battery  is  negligible. C.  The  mutual  inductance  of  the  pair  of  coils  depends  upon  (a)  the rates at which currents are changing in the two coils  (b)  relative  position  and  orientation  of  the  two  coils  (c)  the  materials  of  the  wires  of  the  coils  (d)  the  currents  in  the  two  coils.66  H  (b)  9  H  D  (c)  0.66  H  A  3 H  3 H  3 H  (d)  1  H.  20. This  combination  is  moved  in  time  t  seconds  from  a  magnetic  field  W 1  weber  to  W 2  weber.  (2004)  29.8 H  32.  ammeter  because  (a)  A.  35.  11.m.  15.  33. of 8 V is induced in a coil.  8.  Alternating  current  cannot  be  measured  by  D.05 second.C.C.  In a transformer. L.  16.  (2002)  36.  (2003)  31.  changes  direction  (c)  average  value  of  current  for  complete  cycle  is  zero  (d)  D.  (2003)  30.  then  that  in  the  secondary  coil  is  (a)  4 A  (b)  2 A  (c)  6 A  (d)  10 A.2 H  (b)  0.C. The voltage across the LC  combination  will  be  (a)  50 V  (b)  50 2 V  (c)  100 V  (d)  0 V  (zero).  21.f.  A conducting square loop of side  + + + + ++ L and resistance R moves in its  + + + + ++ + + + + ++ plane with a uniform velocity v  + + + + ++ v perpendicular to one of its sides.  (c)  (a)  (d)  (c)  (c)  (d) .  (c)  (a)  (d)  (b)  (a)  (b)  6.  The  induced  current  in  the  circuit  is  W .  22.1  H.  (b)  (c)  (b)  (a)  (b)  (c)  3. cannot  pass  through  D.  Two  coils  are  placed  close  to  each  other.W 1 )  (c)  -  2 Rnt n(W . the voltage across  each of the  components.  12.  25.  24.  13.  23.  ammeter  (b)  A.  27.  36.c.  19.4 H  (c)  0.  9.  (2004)  28.  31.  The  core of  any  transformer  is  laminated  so  as  to  (a)  reduce  the  energy  loss  due  to  eddy  currents  (b)  make it  light  weight  (c)  make  it robust  &  strong  (d)  increase  the  secondary  voltage.  30.  an e.  32.  (2002)  Answer  Key  1.  In  an  oscillating  LC  circuit  the  maximum  charge  on  the  capacitor is Q.  When the current changes from +2 A to –2 A in 0.  + + + + ++ A magnetic induction B constant  + + + + ++ + + + + ++ in  time  and  space.  28.  A coil having n turns and resistance R W is connected with  a  galvanometer  of  resistance  4R W.  (2003)  4.  14.  26.  The  inductance  between  A  and  D  is  (a)  3. The charge on the capacitor when the energy  is stored equally between  the electric  and magnetic field is  (a)  Q/2  (b)  Q / 3  (c)  Q / 2  (d)  Q.  (2002)  35.  (2002)  34. C and R is 50 V. If current in primary  coil  is  4 A. number of turns in the primary coil are 140  and that in the secondary coil are 280.  (2003)  33. ammeter will get damaged.  7.  Rt (2004)  27.  10.  (d)  (a)  (c)  (d)  (c)  (d)  (d)  0. The  induced  emf  is  (a)  zero  (b)  RvB  (c)  vBL/R  (d)  vBL.  (b)  (d)  (a)  (a)  (d)  (b)  5.  34.  The power factor of an AC circuit having resistance (R) and  inductance (L) connected in series and an angular velocity w  is  (a)  R/wL  (b)  R/(R 2  + w 2 L 2 ) 1/2  (c) wL/R  (d)  R/(R 2  – w 2 L 2 ) 1/2 .  29.C.W 1  (a)  -  2 5 Rnt n(W .  18.  17. as shown in figure.  (d)  (d)  (c)  (c)  (b)  (d)  2.W 1 )  (d)  -  2 . circuit. The coefficient of self­  induction  of  the  coil  is  (a)  0.  In an LCR series a.81  Electromagnetic Induction and Alternating Current  26.W 1 )  (b)  -  2 5 Rt (W .  pointing  + + + + ++ perpendicular and into the plane  L  at  the  loop  exists  everywhere  with half the loop outside the field. e . BH  = 5.  (c) :  Emf induced across PQ  is e  = Blv. V 0  = 200 V.  (b):  S  200 V  In case charging of capacitor through the  resistance  is  V = V0 (1 - e .15  mV  Here..5  m s –1  Induced emf.  Charge on a capacitor at  any time  t  is  q =  q 0(1 –  e –t/t )    q =  CV(1 –  e –t/t )  [As  q 0  =  CV]  t At  t =  2  q = CV (1 .7 × 10 6 W  6.3 ´ 10 -2 )2  ´ 2  f1  = 2[(15 ´ 10 -2 ) 2 + (20 ´ 10 -2 )2 ]3/ 2  f 1  = 9. we  get  4 p ´ 10 -7 ´ (20 ´ 10 -2 )2 ´ p ´ (0.50 ×  2  = 15  × 10 –5  V  = 0.t / RC )  As  field due  to current loop  1  at  an  axial  point  m 0 I1 R 2  \ B 1  = 2(d 2 +  R 2 ) 3/2  Flux linked with smaller loop  2  due  to B 1  is  m 0 I1 R 2  f 2 = B1 A2  = pr 2  2(d 2 +  R 2 ) 3/2  The coefficient of mutual inductance between the loops is  f m 0 R 2 pr 2  M  = 2  = I 1  2( d 2 +  R 2 ) 3/ 2  Flux linked with bigger loop 1  is  m 0 R 2 pr 2 I 2  f1 = MI 2  = 2(d 2 +  R 2 ) 3/2  Substituting the given  values.  L  P  O  e = Blv  R  M R  I 1  I  Q  R  I 2  N  Applying Kirchhoff’s  first law at  junction Q. V = 120 V.916  R ´ 2 ´ 10 -6  Þ R = 2.  we get .  Now. R = ? C = 2 mF and t  = 5 s.  q = CV (1 .  capacitor is charging through a resistor R.t /2 t ) = CV (1 - e -1/ 2 )  4.6  or  e -5/ R ´ 2 ´ 10  =  80  200  Taking the  natural logarithm on both sides..e ( p p = LC  4w 4  Q  w = 1  LC )  2 m F R  -2 t / t .(i)  -2  ) = CV (1 - e )  5.0.0 ×  10 –5  N A –1  m –1 l = 2 m and v  = 1..t / t ) = CV (1 - e -1 )  t= At  t = 2t.  (d) : Here.  we  get I = I 1  + I 2  .. e = BHvl  = 5  × 10 –5  ×  1.  (b) : Charge  on the  capacitor at any instant t  is q = q 0 coswt  Equal sharing of energy means  1  Energy  of  a  capacitor =  Total  energy  2 2  2  1 q  1 æ 1  q 0  ö Þ q =  q 0  = 2 C 2 è 2  C ø  2  From equation (i)  q 0  = q0  cos w t 2  1  cos wt = 2  p -1  1  w t = cos  = 2  4  ( )  At  t = t  q = CV (1 .82  JEE MAIN CHAPTERWISE EXPLORER  1.  (c) :  2.  (d):  As  switch  S 1  is  closed  and  switch  S 2  is  kept  open. we  get  -5  = ln(0.1 × 10 –11  weber  3.(i)  Applying Kirchhoff’s second law for the closed loop PLMQP.  The  equivalent circuit diagram is as shown in  the  figure.  -6  \ 120 = 200(1 - e -5/ R ´ 2 ´10  )  .e .4) = .   (c) :  I 1R  + IR  = Blv  V  . the maximum energy stored in the capacitor...  we  get  ln1 . therefore the given series LCR is in resonance.(iv)  2  1  -2 t1 / RC  =  e 2  Taking natural logarithms  of both  sides.  the  inductor  acts  as  a  short  circuit. ..(iii)  and  q 0  =  q0 e .  ln4 = 2ln2)  = 2  V rms  cos f Z (Q I  rms  = V rms  Z )  .  (d) :  During discharging  of capacitor  through  a  resistor.  we get  As XL  = XC . t  ln1 .2  RC t 2  = RC ln4 = 2RC ln2  \  Impedance of the  circuit is Z  = R  = 200 W  The  power dissipated in the circuit is P = V rmsI rms cosf  \ t 1  RC ln 2 1 1  = ´ =  t2  2 2 RC ln 2 4  ( .  I  .(iv)  Blv  3 R Blv  Substituting the value of I in equation (iii). R  = 200 W.  we get  t = 0  R 1  R 2  At  time t  =  0.t / RC  2  q  1  ( q0 e  U  = 1 =  2 C 2  C )  (Using(i))  2  q  .(i)  The energy stored in the capacitor at any instant of time t is  2  K  X C  R tan 30° = X C  or  X C  =  1  R  R 3  Taking natural logarithms of both sides of the above equation.  (a) :  Here. we  get  2 t 1  or  t1  =  RC ln 2  2  RC From equation (iv).. R 1  and R 2  are in  parallel)  V ( R1 + R 2 )  R1R2  9. the phase difference  between the  current and voltage  is  X  tan f =  L  R X  tan 30 ° =  L  or  X L  =  1  R R 3  When only the inductance is removed. .(iii)  Adding equations (ii) and (iii).. . the phase difference  between current and voltage  is  tan f¢ = 1  = e ... u = 50 Hz  When only the capacitance is removed.(ii)  L  Again.t2  / RC  4  V  R 1 7. V rms  = 220 V.(ii)  = 1  0  e -2t / RC = U 0 e -2t / RC  2  C q 2  where  U 0  =  1  0  . we get  I 2  =  3 R Blv 2  Blv  .83  Electromagnetic Induction and Alternating Current  –I 1R – IR + e =  0 8... applying Kirchhoff’s second law for the closed loop PONQP.ln 2 = -  (Q  ln1 = 0) R 2  (ii)  \  The  current through  the  battery is  I  = V = Req  =  V  R1 R 2  R1 + R2  ( ..  I1 = I 2  = 3R 3 R Substituting this value of I in equation (ii).  I  =  Hence.  The  corresponding equivalent circuit diagram is as shown in the  figure  (i).  .  2  C According  to given  problem  U 0  = U 0 e -2 t1 / RC  2  (Using (ii))  .t / RC  (Using (i))  4  From equation (iii)..  the  inductor  acts  as  an  open  circuit.  V  2IR + I 1R + I 2R =  2Blv  2IR + R(I 1  + I 2 ) =  2Blv  2IR + IR  = 2Blv  I  (Using (i))  R 2  3IR = 2Blv  (i)  I  =  2 Blv  3 R . we get  I 1  =  The  current through battery is  I  =  V  R2  At  time t  = ¥.. q = q 0e –t/RC  .  The  corresponding equivalent circuit diagram is as shown in the  figure  (ii).ln 4 = .  we get  –I 2R – IR + e =  0 I 2R + IR  = Blv  .  we  get  .. ..   (d) :  R and  L  cause phase difference to  lie between 0  and p/2  but  never  0  and p/2  at  extremities. p \  Power factor cos f =  cos 2 = 0  Power consumption =  E rms  I rms  cos f =  0. l = 20 cm.  maximum  emf  =  NABw.  (c)  : f  =  10t  –  50t  +  250 d f = 20t - 50  \  dt .  13.  But I 0  = I 2  12 V  S  R 2  -3  D  t = t 0  = L  = 400 ´ 10 H  = 0.  VL  = I L w = 18. N 2  = 400.  (b)  :  Induced  emf  = \  1 1  Bwl 2 = ´ (0.Rt / L  )  or  2  Rt L  1  = ln 2Þ t = ln 2  e .  19.  (c) : Given : E = E 0  sin wt  I = I 0 sin çæ wt .  23.d f Induced  emf  e =  dt or  E  17.c.  I2  =  I0(1–  e –t/t ) where t =  mean life  or  L/R.6  = 50 m V.2 s  R 2  W \  I2  = 6(1 –  e –t/0.  22.e 10  ÷ ÷ 5 ç è ø  I = (1 – e –1 ).  21.  generator.  I  =  I 0  (1  –  e –Rt/L )  I 0  = I 0 (1 - e .  I  =  I 0  e –tR/L  ( -1´10-3 )´100  I = (1) e 100´10 -3  1  I = e -1  =  A. 1 1 1 1 1  = Þ = = \  L1C1 L2 C2 LC L2 (2C ) 2 L2 C \  L 2  =  L/2.p ÷ö 2 ø  è Since the phase  difference  (f) between  voltage and current  p  is  2 .e L  ÷ ç ÷ è ø  æ .R t ö or  I = E  ç 1 .  From f 2  = pr 12   (m 0ni)n    2 l.  14.2 ´ 10-4 )(5)(1) 2  2 2  10 Induced  emf  = -4 2 = 100 ´ 10 .  (d)  :  Current  I  =  Z 2 2  where  E = VR + (VL - VC )  Z = R 2 + ( X L -  X C ) 2  At  resonance.  (d)  :  During  growth  of  charge  in  an  inductance.  10.  2 24.  (a) : The emf induced in the circuit is zero because the two  emf induced are equal and opposite when one U tube slides  inside  another  tube.20 = 2.  X L  =  X C \  Z  =  R  Again  at  resonance. power  factor  cosf  =  1  0  \ P = 2  rms  2  V  (220 V)  = = 242 W.84  JEE MAIN CHAPTERWISE EXPLORER  or  e  =  –(20t  –  50)  =  –[(20  ×  3)–  50]  =  –10  volt  or  e  =  –  10  volt.693)  t  = 2 or  t  =  0.  A = pr 1 2  = 10 cm 2 .  C  11.  -7 -4  m N N A  M  =  0 1 2  =  4p ´ 10 ´ 300 ´ 400 ´10 ´ 10  l 0.8.  N 1  = 300.4p× 10 –4 H  12.  B  A  E (across  BC) = L dI 2  +  R2 I 2  dt I 1  E  12  = = 6 A.  (a) :  For  maximum  power. .2  = 12e –5t  V.  Z (200 W ) 16.Rt / L  = =  2 .1  = C w (2 ´ 10-6 ) ´ (200)  V L  =  250  volt.2 ) =  12e –t/0.  V L  =  V C \  E  =  V R V  100  I  = R  = = 0.  (a)  :  In  an  a.  (c) :  At  resonance.1  sec.5 ´ 2 ö 5 æ ç1 .  (c)  :  Power  factor  cos f = = Z 15  20.  R2  2  L  R 1  I2  =  I0(1–  e –t/t  ).  R  12  = 0.  I1  =  E/R1.R t ö I = I 0 ç 1 .1  or  or  2  L R 300 ´ 10 -3  ´ (0.  (a) : During the growth of current in LR circuit is given by  æ .  e 2  15. L circuit the potential difference across  AD =  VBC  as they are  parallel.2 )  Potential drop across L  =  E  –  R2I2  = 12 – 2 × 6(1 –  e –t/0.e  L  ÷ = ÷ Rç è ø  .  (d) : For the given R.  L w = 1  Cw 1 1  C  = = \  Lw2 10 ´ (2 p ´ 50) 2  1  = = 10 -6  F  10 ´ 104 ´ ( p )2  or  C  =  1 mF.  w =  1  LC when w  is  constant.  (a) : M = m 0 n 1 n 2pr 1 2 l.1 A  \  R 1 ´ 10 3  \  \  I  0.  (b) :  Maximum  current  I 0  = E  = 100  = 1 A  R 100  The current  decays for  1 millisecond =  1  ×  10 –3  sec  During  decay. t =  t0  (given).  At  resonance.   (b)  :  Magnetic  flux  linked  2  = BA cos wt = B pr cos wt  2  .  33.e  .W 1 )  1  I  = =\  .  for  complete  cycle  is  zero.1 H  .  (d)  :  L  = .  34.  can  not  be  measured  by  D.  (b)  :  I 2  N 2  =  I 1  N 1  for  a  transformer I N  4 ´ 140  I 2  = 1 1  = = 2 A .  The  voltage  across  LC  combination  will  be  zero.  29.  (c)  :  Average  value  of  A.d f . .  (a)  :  The  energy  loss  due  to  eddy  currents  is  reduced  by  using  laminated  core  in  a  transformer.  (d)  :  Induced  emf  =  vBL.n d f .  (d)  :  In  an  LCR  series  a.  4R 2 8 R .  (c) : Mutual inductance between two coils depends on the  materials  of  the  wires  of  the  coils.  (b)  :  Power  factor = .C.05  = = 0.85  Electromagnetic Induction and Alternating Current  25.c.  1æ 1 Q ç 2 çè 2 C 31.  R + L2 w 2  36.  circuit.C.  \  N 2  280  35.W1 ) n (W2 .1  = B p r 2 w sin w t  \  Induced  emf  e = dt 2  e 2 B 2 p 2 r 4 w 2 sin 2  wt  =  \ Power  = R 4 R ( B pr 2 w) 2  2  = sin  wt  4 R 2 <sin  wt>  =  1/2 Q  \  Mean  power  generated  ( B pr 2 w) 2 1  ( Bpr 2 w) 2  = ´ =  .  (d)  :  Three  inductors  are  in  parallel  3 H  27.  the  voltages  across  components  L  and  C  are  in  opposite  phase.n dW  = 26.  ( R + 4 R ) t 5 Rt 30.  3 H  A  28.  Let  q  denote  charge  when  energy  is  equally  shared 2 \  \  ö 1  q 2  Þ Q 2 = 2 q 2  ÷÷ = 2  C ø q = Q / 2.  (b)  :  Induced  current  I  = ¢ where R dt R ¢  dt f  =  W  =  flux  ×  per  unit  turn  of  the  coil n (W2 .  Hence A.  di / dt - 4  32.C.  (c)  :  Let  Q  denote  maximum  charge  on  capacitor.8 ´ 0.  R  2 D  3 H \  \  1 = 1 + 1 + 1 = 3 = 1  L( eq )  3 3 3 3 1  L (eq)  =  1 H.  ammeter.   (a)  Statement­1 is  true. choose the one that best  describes the two statements. The peak value of electric field  strength is  (a)  12 V/m  (b)  3 V/m  (c)  6 V/m  (d)  9 V/m  (2013)  2.  (c)  Statement­1 is true.  Consider  telecommunication  through  optical  fibres.  (b)  Statement­1 is true.  An electromagnetic wave of frequency u = 3.  Then  r r r r r r r r r r (a)  X P E and k P E ´ B  (b)  X P B and k P E ´ B  r r r r r r r r r r (c)  X P E and k P B ´ E  (d)  X P B and k P B ´ E  (2012)  This question has Statement­1 and Statement­2. day to day and season  to  season.  (2002)  9.  (2004)  7.  (c)  Optical  fibres  have  extremely  low  transmission  loss. which are always perpendicular to  r  X  and  each other.  (b)  Optical fibres are subject to electromagnetic interference  from  outside. Statement­2 is true.  (c)  (2002)  .3 × 10 –3  J/m 3  (b)  4. It is located on a mountain top of height 500 m.  (d)  (c)  3.  Electromagnetic waves are transverse in nature  is evident by  (a)  polarization  (b)  interference  (c)  reflection  (d)  diffraction. Statement­2 is  not  the  correct  explanation  of  Statement­1.  Statement­2 : The  state of  ionosphere  varies  from hour to  hour.  Statement­2 is true.  The  magnetic  field  in  a  travelling  electromagnetic  wave  has a peak value of 20 nT.  8.  9.  (d)  Statement­1 is  false. The average total energy density of  the  electromagnetic  wave  is  (a)  3.  (c)  (b)  2.58 × 10 –6  J/m 3  –9  3  (c)  6.  The rms value of the electric field of the light coming from  the sun is 720 N/C.  Answer  Key  1.  (2011)  5.35 × 10 –12  J/m 3 .  Statement­1 : Sky wave signals are used for long distance  radio communication. Statement­2 is  the  correct explanation  of  Statement­1.  Which  of  the  following  statements  is  not  true?  (a)  Optical  fibres  can  be  of  graded  refractive  index.  4. These signals are in general.  (2006)  6.  The maximum distance upto which it can detect object located  on the surface of the earth (Radius of earth = 6. Then  (a)  wavelength  is  doubled  and  the  frequency  remains  unchanged  (b)  wavelength  is  doubled  and  frequency  becomes  half  (c)  wavelength is halved and frequency remains unchanged  (d)  wavelength  and  frequency  both  remain  unchanged.  (2002)  10.4 × 10 6  m) is  (a)  16 km  (b)  40 km  (c)  64 km  (d)  80 km  (2012)  3.  An  electromagnetic  wave  in  vacuum  has  the  electric  and  r r  magnetic fields  E and B .  (b)  6.  7.  (a)  (a)  4.0 MHz passes  from  vacuum  into  a  dielectric  medium  with  permitivity e  =  4. Of the four  choices given after the statements.  (b) 5.  (b)  10. statement­2 is false. The direction of polarization is given by  r  that of wave  propagation  by k .  Infrared radiation is detected by  (a)  spectrometer  (b)  pyrometer  (c)  nanometer  (d)  photometer.37 × 10  J/m  (d)  81.  (2003)  8.0.  (d)  Optical fibres may have homogeneous core with a suitable  cladding.  Which  of  the  following  are  not  electromagnetic  waves?  (a)  cosmic rays  (b)  gamma rays  (c) b­rays  (d)  X­rays. Statement­2 is true.  A radar has a power of 1 kW and is operating at a frequency  of 10 GHz. less stable  than ground wave signals.86  JEE MAIN CHAPTERWISE EXPLORER  CHAPTER  15  ELECTROMAGNETIC WAVES  1.   5. frequency remains constant and wavelength changes  7.87  Electromagnetic  Waves  1.  (c)  : In electromagnetic wave. the peak value of electric  field  (E 0 )  and  peak  value  of  magnetic  field  (B 0 )  are  related  by  E 0  =  B 0 c  E 0  = (20 × 10 –9  T) (3 × 10 8  m s –1 ) = 6 V/m  2. .  8.  9.  then  (h  + R) 2  =  d 2  +  R 2  d 2  =  h 2  + 2Rh  Q  h < < R æ E 2  ö 2  = 1 e0 E rms  + 1  ç rms  ÷ 2 2 m 0  ç c 2  ÷ è ø  1 1  2  2 = e0 Erms + E e m 2 2 m 0  rms 0 0  1 1  2 2 2  = e0 E rms + e0 Erms = e 0 Erms  2 2  =  (8.  10.  (c)  : b­rays  are  not  electromagnetic  waves.  (a)  :  Polarization  proves  the  transverse  nature  of  electromagnetic  waves.  (d)  : Maximum distance on earth  where object can be  detected is  d.  d = 2 ´ 6.  m= \ d =  2 Rh 4.58  ×  10 –6  Jm –3 .  r r r \ k P E ´ B  (b)  1 1  2  2 (b)  :  u = 2 e0 Erms + 2 m  Brms  0  e = 4=2 e 0  1 Since  m µ l \  Wavelength  is  halved  Hence  option  (c)  holds  good.85  ×  10 –12 )  ×  (720) 2  =  4.  (b) : Optical fibres are subject to electromagnetic intereference  from  outside.  (a)  : The direction of polarization is parallel to electric field.4 ´ 106 ´ 500 = 8 ´ 10 4  m = 80 km  3.  (b) : Infrared radiation produces thermal effect and is detected  by  pyrometer.  h  d  90°  R  R  6.  (c)  :  During  propogation  of  a  wave  from  one  medium  to  another. r r \ X P E  The  direction of wave propagation  is  parallel  to  r r E ´ B.   The  graph  between  angle  of  deviation  (d)  and  angle  of  incidence (i) for a triangular prism is  represented by  (c) Direction : The question has a paragraph followed by two  statements. choose the one that describes  the statements. Statement­2 is  not  the  correct  explanation  of  Statement­1.  (b)  (c)  5.4 m in front of a lens forms a sharp image on a  film  12  cm  behind  the  lens.50  is  interposed  between  lens  and  film  with its plane faces parallel to film.  of  refractive  index  1. Medium 1 in z ³ 0 has a refractive index of  2  and  medium 2 with z < 0 has a  refractive index of  3 . this film gives an interference pattern  due to light reflected from the top (convex) surface and  the  bottom  (glass plate) surface of the  film. Statement­2 is true.  Statement­1 :  When light  reflects  from  the air­glass  plate  interface. At what distance (from  lens) should object be shifted to be in sharp focus on film?  (a)  2. so that the amplitude of the light from one slit is  double of that from other slit. If I m  be the maximum intensity.  ( )  I f 1 + 8cos  )  9 ( 2  I m  f 1 +  2cos 2 3 2  m  2 ( )  (b) I m  f 1 +  4cos 2 5 2  (d) I m  ( 4 + 5cos f )  9  (a)  (2012)  1  m s - 1  10 (c)  10 m s –1  (b)  1  m s - 1  15 (d)  15 m s –1  (2011)  .4 m  (b)  3.  A ray  of  light  in  medium  1  given  by  the  vector  r A = 6 3 i$ + 8 3 $j .  With  monochromatic light.6 m  (d)  7.10 k$  is  incident  on  the  plane  of  separation. The  intensity  of the  emergent  light is  (a)  I 0 /8  (b)  I 0  (c)  I 0 /2  (d)  I 0 /4  (2013)  4. Statement­2 is  the  correct explanation  of  Statement­1.  An object 2.  (b)  Statement­1 is true.8 m behind the first car is overtaking  the first car at a relative speed of 15 m s –1 .  (c)  Statement­1 is true.  Statement­2 : The centre of the interference pattern is dark. Statement­2 is true.  Statement­2 is false.  the reflected wave  suffers  a  phase  change  of p.  The  fringes  obtained  on  the  screen will be  (a)  concentric  circles  (b)  points  (c)  straight lines  (d)  semi­circles  A beam of unpolarised light of intensity I 0  is passed through  a polaroid A and then through another polaroid B which is  oriented  so that  its  principal  plane  makes  an  angle  of  45°  relative to that of A. Of the given four  alternatives after the statements.  Two    coherent    point sources  S 1  and  S 2  are separated by a  small  distance  ‘d’  as  shown. Statement­2 is true. The  angle of refraction in  medium 2  is  (a)  30°  (b)  45°  (c)  60°  (d)  75°  (2011)  9. A second car 2.  Let the x–z plane be the boundary between two transparent  media.  (2011)  8.88  JEE MAIN CHAPTERWISE EXPLORER  CHAPTER  OPTICS 16  1.  Diameter of a plano­convex lens is 6 cm and thickness at the  centre  is  3  mm. the focal length  of the  lens  is  (a)  10 cm  (b)  15  cm  (c)  20  cm  (d)  30 cm  (2013)  2.  the  resultant  intensity  I  when  they  interfere  at  phase  difference f is  given  by  (a) 7.  A  glass  plate  1  cm  thick.2 m  (c)  5.2 m  (2012)  (2013)  3. Statement­1 and Statement­2. one  of  the  slit is wider  than other.  (a)  Statement­1 is true.  (a)  6.  If  speed  of  light  in  material  of  lens  is  2 × 10 8  m/s.  A thin air film is formed by putting the convex surface of a  plane­convex  lens  over  a  plane  glass  plate.  (d)  Statement­1 is false. The speed of the  image of the second car as seen in the mirror of the first one is  (2013)  (d)  In Young’s  double  slit  experiment.  A car is fitted with a convex side­view mirror of focal length  20 cm. 520 for red light and 1. q  The incident angle q for which the light ray grazes along the  wall of the rod is  1  (a) sin - 1  (b) sin - 1  3  2 2 - 1  2  - 1  1  (c) sin  (d) sin  (2009)  3 3 15.  If  the  refractive  C index of water is 4/3 and the  fish  is  12  cm  below  the  surface. the wavelength of the unknown light is  (a)  393. it will  (a)  travel as a  cylindrical beam  (b)  diverge  (c)  converge  (d)  diverge  near the  axis and  converge  near the  periphery  ( )  v (cm)  u (cm)  equal  to  3  (a)  4  (b)  1  2  (c)  3  2  1  (d)  2  (2007)  18.  illuminates  Young’s  double  slit  and  gives  rise  to  two  overlapping  interference  patterns  on  the  screen. If I 0  denotes the maximum intensity.  A  student  measures  the  focal  length  of  a  convex  lens  by  putting  an  object  pin  at  a  distance  u  from  the  lens  and  measuring the distance v of the image pin.  Then  (a)  D 1  >  D 2  (b)  D 1  <  D 2  (c)  D 1  =  D 2  (d)  D 1  can be less than or greater than depending upon the  angle  of  prism. Its optical power in a liquid medium with refractive  index  1.  Two lenses of power –15 D and +5 D are in contact with each  other.  As the beam enters  the  medium.  An  initially  parallel  cylindrical  beam  travels  in  a  medium  of  refractive  index m(I) = m 0  + m 2I.  A mixture of light.  what is  the  maximum  distance  at  which  these  dots  can  be  resolved  by the  eye?  [Take  wavelength  of  light  =  500  nm]  (a)  6 m  (b)  3 m  (c)  5 m  (d)  1 m  (2005)  22.  Two point white dots are 1 mm apart on a black paper.    where m 0  and m 2  are  positive  constants and I is the intensity of the light beam.  A thin glass (refractive index 1. consisting of wavelength 590 nm and an  unknown  wavelength.  The  initial shape of  the  wavefront  of the  beam  is  (a)  planar  (b)  convex  (c)  concave  (d)  convex  near the  axis  and  concave near  the  periphery  11.4 nm  (b)  885. Let D 1  and D 2  be angles of minimum deviation  for red and blue  light respectively in a  prism of  this  glass.  A transparent solid cylindrical rod has a refractive index of  2  .  it  is  observed  that  the  third  bright  fringe  of  known  light  coincides  with  the  4 th  bright  fringe  of  the  unknown  light. A light ray is incident at the mid­  3  point of one end of  the rod  as  shown  in  the  figure. It is surrounded by air. The intensity  of the  beam is decreasing with  increasing radius.0 nm  (c)  442.  I  is  0  12.  From this data.  the  radius  of  this  circle  in  cm  is  (a)  36 5  (b)  4 5  (c)  36 7  (d)  36 / 7  12 cm (2005)  21.89  Optics  Directions : Questions number 10­12 are based on the following  paragraph.  The refractive index of glass is 1.  The speed of light in the  medium is  (a)  maximum on  the axis  of  the beam  (b)  minimum on  the axis  of  the beam  (c)  the same everywhere  in the beam  (d)  directly proportional to  the  intensity I  (2010)  13. They  are viewed by eye of pupil diameter 3 mm. Approximately. The graph between  u  and  v plotted by  the  student  should  look  like  ( )  ( )  ( )  v (cm)  v (cm)  (a)  (b)  O  u (cm)  O  v (cm)  (c)  (d)  O  O  u (cm)  u (cm)  (2008)  16. The  focal length of  the  combination  is  (a)  + 10 cm  (b)  –20 cm  (c)  –10 cm  (d)  + 20 cm  (2007)  17.  10.  When  an  unpolarized  light  of  intensity  I 0  is  incident  on  a  polarizing sheet.5 nm  (d)  776. The central maximum of both lights coincide.  A fish looking up through the  R  water sees the outside world  contained  in  a  circular  q Cq horizon.  (2006)  19. the intensity of the light which does not get  transmitted is  .8 nm  (2009)  14.525  for blue light. Further.  In a Young’s double slit experiment the intensity at  a point  l  where the  path  difference  is  (l  being  the  wavelength  of  6 I  light used) is I.6  will  be  (a)  25 D  (b)  –25  D  (c)  1  D  (d)  –1 D  (2005)  20.5) lens has optical power of  –5 D in air.   9. Now this  lens has been used to form the image of an object.  (2004)  29. then the number  of  images formed by them is  (a)  5  (b)  6  (c)  7  (d)  8.  If two mirrors are kept at 60° to each other.  An  astronomical  telescope  has  a  large  aperture  to  (a)  reduce  spherical  aberration  (b)  have  high  resolution  (c)  increase  span  of  observation  (d)  have  low  dispersion.  21.  2  27.  (2003)  31.  The image formed by an objective of a compound microscope  is  (a)  virtual and diminished  (b)  real and diminished  (c)  real and enlarged  (d)  virtual and enlarged.  19.  (2002)  33.  11.  26.  (2004)  28.  A  Young’s  double  slit  experiment  uses  a  monochromatic  source.  If I 0  is the intensity of the principal maximum in the single  slit diffraction pattern.  27.  34.  23.  15.  (c)  (c)  (b)  (a)  (c)  .  10.  To  demonstrate  the phenomenon of interference  we  require  two  sources  which  emit  radiation  of  (a)  nearly  the  same  frequency  (b)  the  same  frequency  (c)  different  wavelength  (d)  the  same  frequency  and  having  a  definite  phase  relationship.  Wavelength  of light used  in an  optical  instrument are l 1  =  4000  Å  and l 2  =  5000  Å. one should have two  plane  mirrors  at  an  angle  of  (a)  60°  (b)  90°  (c)  120°  (d)  30°.  12.  (2004)  26.  The maximum number of possible interference  maxima  for  slit­separation  equal  to  twice  the  wavelength  in  Young’s  double­slit  experiment  is  (a)  infinite  (b)  five  (c)  three  (d)  zero.  8.  24.  16.  (d)  (c)  (c)  (*)  (a)  (d)  2.  30.  13.  then  ratio  of  their  respective  resolving  powers  (corresponding  to l 1  and l 2 )  is  (a)  16 : 25  (b)  9 : 1  (c)  4 : 5  (d)  5 : 4.90  JEE MAIN CHAPTERWISE EXPLORER  (b)  I 0  1  (d)  I 0  4  (a)  zero  1  (c)  I 0  2  (2005)  23.  31.  17.  is  (a)  sin –1 (n)  (b)  sin –1 (1/n)  –1  (c)  tan  (1/n)  (d)  tan –1 (n).  (2003)  32.  Which  of  the  following  is  used  in  optical  fibres?  (a)  total  internal  reflection  (b)  scattering  (c)  diffraction  (d)  refraction.  (2003)  30.  (2002)  34.  (a)  (b)  (d)  (d)  (b)  (b)  3.  18. At what  distance from this lens an object be placed in order to have  a  real  image  of  the  size  of  the  object?  (a)  20 cm  (b)  30  cm  (c)  60  cm  (d)  80 cm.  If  the  angle  of  reflection  is  45°.  The  angle  of  incidence  at  which  reflected  light  in  totally  polarized for reflection from air to glass (refractive index n).  22.  (d)  (b)  (d)  (c)  (d)  (a)  4.  7.  (c)  (b)  (a)  (a)  (b)  (a) 6.  33.  (2002)  Answer  Key  1.  To get three images of a single object.  The  shape  of  the  interference  fringes  formed  on  a  screen  (a)  straight line  (b)  parabola  (c)  hyperbola  (d)  circle  (2005)  25.  we  conclude  45°  that  the  refractive  index  n  1  (b)  n >  2  (a)  n <  2  1  (2004)  (c)  n >  (d)  n <  2 .  A  plano  convex  lens  of  refractive  index  1.  29.  20.  35.  A  light  ray  is  incident  perpendicular  to  one  face  of  a  90° prism and is totally internally  reflected  at  the  glass­air  45°  interface.  14.  (2002)  35.  25.  32.  28.  (d)  (a)  (c)  (c)  (b)  (d)  5.5  and  radius  of  curvature 30 cm is  silvered at the curved surface. then  what will be its intensity when  the slit width  is doubled?  (a)  I 0  (b)  I 0/2  (c)  2I 0  (d) 4I 0    (2005)  24.   (3 cm) 2  = 15 cm  2 ´ 0. so  shift produced by it  will be ( ) ( Shift = t 1 - ) ( )  1 1 2 1  =1 1=1 1= cm  m  1.  I = I1 + I 2 + 2 I1I 2  cos f  8  \ m= c  3 ´ 10 m/s  3  = = v 2 ´ 108  m/s  2  = I1 + 4 I1 + 2 I1 (4I1 ) cos f  = 5I 1  + 4I 1 cosf =  I 1  + 4I 1  + 4I 1 cosf  = I 1  + 4I 1 (1 + cosf) ..  A 2  = 2A 1  .  the radius of curvature R of its curved surface in accordance  with figure  will  be  given  by  = I1 + 8 I1 cos 2 R 2  = r 2  + (R –  t) 2  ( 2Rt =  r 2  +  t 2  r 2  (Q  r >> t )  2 t Here.  12 cm  According to thin lens  formula  1 = 1 -  1  f v u Here.91  Optics  5.  2  f 2  Putting the  value  of  I 1  from  eqn. u = – 2. (ii)  and  (iii) in (i).1 = 1 + 1  f 12 ( - 240) 12 240  1 = 21 or f  =  240  cm  f 240 21  When a  glass plate is interposed  between lens and film. (i).5 3 3  To get image at film.4 m = – 240 cm. r = 3 cm.(i)  Resultant intensity. lens should form image  at distance  v ¢  = 12 - 1 35  = cm  3 3  Again using lens formula .  (c) : Here.(i)  R  (m - 1)  or  I1  =  As speed of light in the  medium of lens  is 2 ×  10 8  m/s  I m  9  .4 m  0  (d)  : The graph  between  angle  of  deviation(d) and angle of incidence (i)  for  a  triangular  prism  is  as  shown in  the adjacent  figure.5 - 1) 2. .  (d): According to lens  maker’s  formula  1 1 1  ù = (m .  0 )  1 cm  O  2.  the fringes  will be  concentric  circles..  we  get = I1  1 + 8cos 2  R 2  =  r 2  +  R 2  +  t 2  –  2Rt  \ (Q 1 + cos f = 2cos  f2 )  f 2 1 = 1 . t =  3 mm = 0.  v = 12 cm  \ Now  this  light  will  pass  through  the  second  polaroid  B  I 2 = I1 cos 2  45 ° = ( I m  f 1 + 8cos 2  9 2  Film  (a)  :  When  the  screen  is  placed  perpendicular  to  the  line  joining the  sources..  the  inten‑  sity  of  light  emerging  from  polaroid  B  is ( I2 )( 1 2 )  =  I 4  2  4..(ii)  If r is the radius and t is the thickness of lens (at the centre).3 cm  I= R= R= 6. we  get  15 cm  f  = = 30 cm.  So  in  accordance  with  Malus  law. R 2  = ¥  \ I 2 =  4 I1  2  Maximum intensity. .  I1  =  )  (c)  :  On substituting  the  values of  m  and  R  from  Eqs..1) é ú f ëê R1 R2  û  As the lens  is plano­convex \  R 1  = R.  (1.  Intensity µ  (Amplitude) 2  2 2  I æA ö æ 2 A  ö \ 2 = ç 2 ÷ = ç 1 ÷ = 4  è A1  ø  I1 è A1 ø 1. I m  = ( I1 + I 2  )  1  (m .  (d)  :  Intensity  of  light  after  passing  polaroid  A  is  I 0  2  whose  axis  is  inclined  at  an  angle  of  45°  to  the  axis  of  polaroid A..1)  =  f R or  f = 2 2  = ( I1 + 4 I1 ) = (3 I1 )  = 9 I1  .3 cm  3. .21 ù u ¢  35 240 5 ë 7 48 û 240 35  u ¢  \ 1 1 é144 ...1 = sin -1  3  3 So that q c  is making  total internal reflection.10 k^  10 cos i = (6 3 )2 + (8 3 ) 2  + (-10) 2  = 10  20  ( )  cos i = 1 or i = cos -1  1  = 60 °  2 2  Using Snell’s law.92  JEE MAIN CHAPTERWISE EXPLORER  21 = 3 .  (c)  13.5 nm. A = 6 3 i^ + 8 3 j^  .  The  point  which  meets the curve at  u =  v gives 2f.  (b) :  Given m  = m 0  + m 2 I  Speed of light in vacuum  As  m =  Speed of light in medium c c  m =  c  or  v = m = m + m  I v 0 2  As the intensity is maximum on the axis of the beam.1  = R  D2  for blue m B  - 1  Since m B  > m R.  .  the  path  difference  xd  xd  l = n l  for  bright  fringes  and  = (2 n + 1)  D D 2  for getting dark fringes.  1 1 Focal length of combination  F  = P = - 10 D  = – 0.  m  (b)  10. u  is  negative.  11. u  =  f  and if u = ¥. q i  = q. I 0  = maximum intensity  æfö I  = cos 2 ç ÷ .  12. the shape of wavefront  is planar.  by  Young’s  double  slits.  (c) :  Power of combination =  P 1  +  P 2  = – 15 D + 5 D = – 10 D.  The central fringes when x = 0. m 1 sini = m 2 sinr  2 sin 60° = 3 sin r Þ r = 45 °  9. both are symmetrical and this curve satisfies all  the  conditions  for  a  convex  lens.  I 0  è ø  0  18. (ii)  l  6 3 Substitute eqn.  (a) : In Young’s double slit experiment intensity at a point is  given by  æ fö I = I 0 cos 2 ç ÷ è 2 ø  where f = phase difference.  (a) : As the beam is initially parallel.  sin f cos qc 3  qc m2  - 1 cos qc = m2 .  (d) : According to the  new cartesian  1 1 1  system used in schools.  u has to be negative.6 m  If q c  has to be the critical angle.  (c)  :  For  interference.=  for  v u f a convex  lens. v = f.  m m (b) :  \ q = sin -1 r Here.  17. coincide for all wavelengths.  4 14.  The  third fringe  of l 1  =  590  nm  coincides  with  the  fourth  bright  fringe of unknown wavelength l. .  (d)  : q f qc  m = ref.1  \ sin q = m = m 2  ..  (c)  8.147 ù 1 3  = or  =u ¢ 5 ëê 336 ûú u ¢  1680  u¢ = – 560 cm = – 5.1 m = – 10 cm.  D 1  < 1  \  D2  \  D 1  <  D 2 .  the  rays  are  parallel.  qc = sin -1  1  m  But q c  = 90° – f.1 m 2  . index of the rod ( )  4 1  .  v is +ve  f  f  u  negative  If v = ¥. Therefore v is +ve. (ii) in eqn. therefore  v  is minimum on the  axis  of the  beam.  15.  \  xd  = 3 × 590 nm = 4 × l nm  D \ l= 3 ´ 590  =  442. we get  I  3  I  = cos 2 æ p ö ç 6 ÷ or  I =  4  .  A  parallel  beam  (u  = ¥)  is  focussed  at  f  and  if  the  object  is  at  f.  (b)  : Angle  of  minimum  deviation  D  =  A(m  –  1)  D1 for red m .1.6 m  |u¢| = 5.  1  But  7.21 = 1 é 3 .  (i)  or  I 0  è 2 ø  Phase  difference  f = 2 p ´ path difference l 2p l p f= ´ f =  \  or  .  16.1  or  1 = 3 . (i).  sin qi  = m = 2 Þ sin q = m.   (a)  :  For  diffraction  pattern  30.1 = -1  Pa  1.  \  \  26.  (c)  :  Resolution  limit  = 1.  2 2 23.1 = 60 °  .1  D  22.  (d) : For interference phenomenon. P .  34.22 l =  D d yd  D= 1.  R . P .  (a)  :  Straight  line  fringes  are  formed  on  screen.  32.  (a)  :  Total  internal  reflection  is  used  in  optical  fibres.  sinq  is  never  greater  than  (+1).22 l  n > 27.  (d) : According to Brewster's law of polarization.1) (m g / m l  ) .1  q0  360 ° 3= .m l  = 1. q° 29.22 l d y  1.  ( * )  :  f = ( m g  .5 - 1)  or  P l  = .  (a)  :  n = q° .1) 1.B.5  y  D 360 ° .  Given : i = 45° in the medium and total internal reflection  occurs  at  the  glass  air  interface \  sin qc  Again  resolution  limit  = sin q = q = R 30  = = 10 cm  2m 2 ´ 1.1 = 5.  æ 1 1 1  ö a  19.  R  3 36  = Þ  R = cm. for l 2 l 1  4000 4  360° 360 ° 35.  (c)  :  Intensity  of  polarized  light  =  I 0 /2 \  Intensity  of  light  not  transmitted  I I  = I 0  .  P a  ( -5)  5  ==  8 8 8  or    Optical  power  in  liquid  medium  =  5  Dipotre.1  = a  = fl (m g  .1.1) çè R .  –1.1)  ( m g  - 1)  = m g .1) ç . : This answer is not given in the four options provided  in  the  question.÷ fl  è R1 R2 ø 24.  12  7 7  21.  31.  1.  25.6 (1. the object should be placed  at  (2F)  of  the  concave  mirror.  33. \  Distance  of  object  from  lens  =  2  ×  F  =  2  ×  10  =  20  cm.  sin C sin 45 °  28.  8 N.R ÷ø  a  1 2  æ 1 1 1  ö = (l m g  .22) ´ (5 ´ 10-7 )  6. \  F  of  concave  mirror  so  formed  l  \  f a  ( m g .  20.0 =  0  .  m= \  1 1 3  Þ sin qC  = = sin qC m 4  tan qc  = = 1 .  (b)  :  For  interference  maxima.  (a)  :  A  plano­convex  lens  behaves  like  a  concave  mirror  when  its  curved  surface  is  silvered.0.  dsinq  =  nl \  2lsinq  =  nl  n sin q =  or  2  This  equation  is  satisfied  if  n  =  –2.  Hence  ç f ÷ = 1 è ø  \  R .6  m l (m g .sin 2  q c 3/ 4 = 3 ´ 4 = 3  4  7 7  1 .  .  (1.9 16 or  or  D= 1 > 1  > 2  .  (b)  :  Total  internal  reflection  occurs  in  a  denser  medium  when  light  is  incident  at  surface  of  separation  at  angle  exceeding  critical  angle  of  the  medium.6 ´ 0.  (d)  :  For  total  internal  reflection.93  Optics  Hence  intensity  remains  constant  at  I 0  I  =  I 0  (1)  =  I 0 .5 .  (b) :  Large  aperture  leads to  high  resolution  of  telescope.  less  than  (–1) \  Maximum  number  of  maxima  can  be  five.  2. two sources should emit  radiation of the same frequency and having a definite phase  relationship.5 8  Þ  Pl  = - = To form an image of object size. for  l1 l 2  5000 5  = = = .  (d)  :  Resolving  power  is  proportional  to l –1 2  æ sin f ö I = I 0 ç ÷ where f  denotes  path  difference  è f ø  æ sin f ö For  principal maxima.1 Þ 4 q° = 360° Þ q° = 90°. f  =  0. n = tan i p  where  i p  is  angle  of  incidence \  i p  =  tan –1 (n).  (b)  :  n = \  y q (10 -3 ) ´ (3 ´ 10-3 )  30  = » 5 m.  0.  (c) :  The  objective  of  compound  microscope  forms  a  real  and enlarged  image.  statement­2 is false.94  JEE MAIN CHAPTERWISE EXPLORER  CHAPTER  17  1. Statement­2 is  the  correct explanation  of  Statement­1. Statement 2 is  true. Of the four  choices given after the  statements.  (c)  Statement 1 is true.  DUAL NATURE OF MATTER  AND RADIATION The anode voltage of a photocell is kept fixed.  (b)  O 2.  Statement­2 is false. choose the one that best  describes the two statements.  Statement­2 : The maximum kinetic energy and the stopping  potential of photoelectrons emitted from a surface are linearly  dependent  on  the frequency  of  incident  light. The  plate current I of the  photocell  varies  as  follows  I  I  (a)  O l  If a source of power 4 kW produces 10 20  photons/second.  Statement 2 is true.  The  surface  of  a  metal  is  illuminated  with  the  light  of  400 nm. choose the one that best  describes the two statements :  Statement­1  :  A  metallic  surface  is  irradiated  by  a  monochromatic  light  of  frequency u  > u 0  (the  threshold  frequency). 8 and 9 are based  on the  following  paragraph. they can interfere  and  show  diffraction.  Statement 1 : Davisson ­ Germer experiment established the  wave nature  of  electrons.41 eV  (c)  1.  its stopping potential is V 0  and the maximum kinetic energy  of  the  photoelectrons  is  K max .09 eV  (b)  1.  incoming  electrons  outgoing  electrons  i  d  crystal plane  . Statement 2 is  true.  (d)  Statement­1 is false.  (d)  Statement­1 is false. Statement­2 is true; Statement­2 is  not  the  correct  explanation  of  Statement­1. the  radiation belongs to a  part of the  spectrum called  (a) g­rays  (b)  X­rays  (c)  ultraviolet rays  (d)  microwaves  (2010)  l  5. Statement 2 is  not the  correct  explanation  of  Statement  1.  Wave property of electrons implies that they will show diffraction  effects. Statement­2 is true; Statement­2 is  the  correct explanation  of  Statement­1. The  maximum  kinetic  energy  and  the  stopping  potential are K max and V 0 respectively.  Statement­2 : Photoelectrons are emitted with speeds ranging  from  zero  to  a  maximum  value  because  of  the  range  of  frequencies present in the  incident light. Statement­2 is true.68 eV  (2009)  Directions : Questions  7.  (2012)  This question has Statement­1 and Statement­2.  (2008)  6.  (c)  Statement­1 is true. Statement 2 is  the  correct  explanation  for  Statement  1. When  the  ultraviolet  light  is  replaced by X­rays.  (a)  Statement­1 is  true. If the frequency incident  on  the  surface  is  doubled.  4.  Statement­1 : When ultraviolet light is incident on a photocell. The law governing the diffraction from  a crystal is obtained by requiring that electron waves  reflected  from the planes of atoms in a crystal interfere constructively (see  figure).  (2010)  I  I  (c)  (2013)  (d)  O 3.68  eV.  (a)  Statement­1 is true.  Statement 2 is false. The kinetic energy of the ejected photoelectrons was  found  to  be  1. both V 0  and K max  increase. The wavelength l of the light falling on the cathode is gradually changed.  (d)  Statement 1 is false. Statement­2 is  not  the  correct  explanation  of  Statement­1. Of the four  choices given after the  statements.  (b)  Statement 1 is true.  The  work  function  of  the  metal  is  (hc = 1240 eV  nm)  (a)  3.  (c)  Statement­1 is true. Statement­2 is true.  (b)  Statement­1 is true. Statement­2 is true.  Davisson and  Germer demonstrated  this by  diffracting  electrons from crystals.  (a)  Statement 1 is true.51 eV  (d)  1. Statement­2 is  true.  Statement 2 : If electrons have wave nature.  (b)  Statement­1 is  true.  (2011)  l  O l  This question has Statement 1 and Statement 2.  both  the  K max  and  V 0  are  also  doubled.   If c is the velocity of  light.  If g E and g M  are the accelerations due to gravity on the surfaces  of the earth and the moon respectively and if Millikan’s oil  drop experiment could be performed on the two surfaces.  (2006)  15.  A  charged  oil  drop  is  suspended  in  a  uniform  field  of  3 × 10 4  V/m so that it neither falls nor rises.6 × 10 –19  C)  (a)  1000 V  (b)  2000 V  (c)  50 V  (d)  500 V. The charge on the  drop will be (take the mass of the charge = 9.  9.  D  Which of the following graphs can be expected to represent  the number of electrons N detected as a function of the detector  position y  (y = 0 corresponds to  the  middle  of  the  slit)?  y  (a)  N  d  d  y  (c)  (d)  N  d  10.  V  should  be  about  (h = 6.  8.  If a strong diffraction peak is observed when electrons are  incident at an angle i from the normal to the crystal planes  with  distance  d  between  them  (see  figure).  (2006)  y  =0  d  I  (a) (2007)  11.  the  number  of  electrons  emitted  by  photocathode  would  (a)  decrease  by  a  factor  of  2  (b)  increase  by  a  factor  of  2  (c)  decrease  by  a  factor  of  4  (d)  increase  by  a  factor  of  4  (2005)  y  (b)  N  I  (b) 14. of the incident radiation gives a straight  line  whose  slope  (a)  depends  on  the  nature  of  the  metal  used  (b)  depends  on  the  intensity  of  the  radiation  (c)  depends  both  on  the  intensity  of  the  radiation  and  the  metal  used .8 ×  10 –18  C. When the same source of light is placed (1/2) m away.  and  the  stopping  potential  for  a  radiation incident on this surface 5 V.  The  wavelength l of the light falling on the cathode is gradually  changed. its de Broglie  wavelength  changes  by  the  factor  (c)  1/2  (d)  2. the  momentum is  (a)  hu/c  (b) u/c  (c)  huc  (d)  hu/c 2  (2007)  12.0  eV.95  Dual Nature of Matter and Radiation  7.  (2005)  (a)  1/ 2  (b)  2  16.  (2004)  19. e = 1.  The  threshold  frequency for a  metallic  surface  corresponds  to  an  energy  of  6.  O  (2006)  l  13.  de  Broglie  wavelength l dB  of  electrons  can  be  calculated  by  the  relationship (n is an  integer)  (a)  d cosi = nl dB  (b)  d sini  = nl dB  (c)  2d cosi =  nl dB  (d)  2d sini =  nl dB  In an experiment.  Photon of frequency u has a  momentum associated with it.  The  work  function  of  a  substance  is  4. electrons are made to pass through a narrow  slit of width d comparable to their de Broglie wavelength.  The  time  by  a  photoelectron  to  come  out  after  the  photon  strikes is  approximately  (a)  10 –1  s  (b)  10 –4  s  –10  (c)  10  s  (d)  10 –16  s.  The  longest  wavelength  of  light  that  can  cause  photoelectron  emission  from  this  substance  is  approximately  (a)  540  nm  (b)  400  nm  (c)  310  nm  (d)  220  nm.  (2004)  18.  Electrons  accelerated  by  potential  V are  diffracted  from  a  crystal. The incident radiation  lies  in  (a)  X­ray  region  (b)  ultra­violet  region  (c)  infra­red  region  (d)  visible  region.  If the kinetic energy of a free electron doubles.  According  to  Einstein’s  photoelectric  equation.9 × 10 –15  kg and  g = 10 m/s 2 )  (a)  3.  the  plot  of  the  kinetic  energy  of  the  emitted  photo  electrons  from  a  metal vs the frequency.6 × 10 –34  Js. one  will find the ratio  electronic charge on the moon  electronic charge on the earth  to be  (a)  g M /g E  (c)  0  (b)  1  (d)  g E /g M  O  O  l  l  I  I  (c) (d) O  l  .  A photocell is illuminated by a small bright source placed 1 m  away. The plate current I of the photocell varies as follows  17. m e  = 9.2 ×  10 –18  C  –18  (c)  1.  If  d  =  1  Å  and  i  =  30°.6 ×  10  C  (d)  4.  The  anode  voltage  of  a  photocell  is  kept  fixed.3 ×  10 –18  C  (b)  3.1 × 10 –31  kg.2  eV.  They are detected on a screen at a distance D from the slit.   7.  20.  Sodium and copper have work functions 2.  (a)  11.f 2 ) ùú 2 . Then the ratio of the wavelengths is nearest to  (a)  1 : 2  (b)  4 : 1  (c)  2 : 1  (d)  1 : 4.  (a)  17.  (b)  (c)  (c)  (a)  3.  then  2  2 h  2  (a)  v1 .  9.3 eV and 4.  (c)  .  8.  13.  If  the  velocities  of  the  photoelectrons  (of  mass  m)  coming  out  are  respectively  v 1  and  v 2 .  (a)  6.  14.  21.  (2004)  20.  (b)  10.  (b)  12. (2002)  Answer  Key  1.  (a)  (c)  (b)  (d)  2.  (b)  16.  (c)  18.v2 = éê 2 h ( f1 .  (d)  5.  15.v2 = m ( f1 -  f 2 )  1  (b)  v1 + v2 = éê 2 h ( f1 + f 2 ) ùú 2  ëm û  2  2  2 h  (c)  v1 + v2 = m ( f1 +  f 2 )  1  (d)  v1 .  19.  ëm û  (2003)  21.96  JEE MAIN CHAPTERWISE EXPLORER  (d)  is the same for all metals and independent of the intensity  of  the  radiation.  Two  identical photocathodes  receive  light  of  frequencies  f 1  and  f 2 .5 eV  respectively.  (d)  (a)  (a)  (c) 4. e.27 ´ 10 -10 )  3 ´ 10 -10  V = 50.  The stopping potential  is  eV 0  = K max  = hu –  hu 0  Therefore.1 – 1.  N  =  10 20  i  q  Energy  of  photon. u 0  is threshold  frequency.1 × 10 –31  kg.  1240 eV nm  i.  13.  (a)  :  According  to  Einstein’s  photoelectric  equation  K max  = hu – f 0  where.  d  = 1  Å.  It does not  depend on g.68 eV = 1.  (i)  . . the work function  =  hu –  kinetic  energy  = 3.  400 nm  Max.  3. (ii)  12.  5. d  = 4. hu = = 3. This shows  the wave nature of  electrons as waves  can  exhibit  interference  and  diffraction. P = 4  kW =  4 ×  10 3  W  Number of photons  emitted  per second.  c .  (b)  :  Davisson­Germer  experiment  showed  that  electron  beams can undergo diffraction when passed through atomic  crystals.6 × 10 –19  C.  i  = 30°  i.  (a) :  Energy of a photon E=  hu  Also E =  pc  where p is the momentum of a photon  From (i) and (ii). grazing angle q = 60°. h = 6. which is also used in electron  diffraction gives  nl  = 2d  sinq..  (c)  :  The  graph  (c)  depicts  the  variation  of l  with  I. 4.  E = hu = hc  l  P  .  (b)  :  For  photo­electron  emission.97  Dual Nature of Matter and Radiation  1. if u is doubled  K max  and V 0  is  not doubled.  (a) : The electron diffraction pattern from a single slit will  be as shown below.  m e  = 9.  E  =  N hc = P  l  N 20 -34 8  Nhc  10 ´ 6.  (a)  2.  6. .  9.68  eV.  (Incident  energy  E)  =  (K. V = (12.1 eV.  7.6 × 10 –19 C) universal constant.  Power of a  source.. \  Electronic  charge on the  moon =  electronic  charge on the earth  electronic charge on the moon  = 1.  (c) :  For electron diffraction.63 ´ 10 ´ 3 ´ 10  or  l = = 3  P 4 ´ 10  But as the angle  of incidence is given.972 × 10 –9  m  = 49.  Statement­2 is false.  hu = Wf  +  kinetic  energy \  Wf .  2p  The line of maximum intensity for the zeroth order will exceed  d  very  much.18 Volt.E.  (b) :  Here.  or  electronic charge on the earth  11..  8.6 × 10 –34  J s.  (d)  : The maximum kinetic  energy  of  the  electron  K max  =  hu  – hu 0  Here.) m ax  +  (Work  function f) .e.  (b)  : The wavelength of light illuminating the photoelectric  surface = 400 nm.  nl  = 2d cosi  is the formula for finding a peak... both K max  and V 0  increase when ultraviolet light  is replaced by X­rays.42 eV.  kinetic energy  of the  electrons  = 1.  e = 1.  (b) : Since electronic charge (1.  \ l =  ° 2 ´ 1(A) ´ sin 60 ° (assuming first order)  1 ° l =  3 A.  d sin q = 10..72  Å  It  lies in the  X­ray region. u =  frequency of  incident  light f 0  = work function  of  the  metal  Since K max  =  eV 0  h u f0  V 0  = -  e e As u X ­rays  > u Ultraviolet  Therefore.  q  D  x  l . we get  h u hu = pc  or  p  =  .  Bragg’s equation for X­rays.  (c) : Bragg’s relation nl = 2d sinq for having an intensity  maximum  for  diffraction  pattern. P remains constant  as  the  source  is  the  same.  (a)  :  de  Broglie  wavelength  l = h / p = h / (2mK )  h  l =  \  where  K  =  kinetic  energy  of  2 mK particle \  l2 K1 K 1  1  = = = .  or  or  14. we assume light to spread uniformly in all directions.2 ´ 1.  (a)  :  For  equilibrium  of  charged  oil  drop.  Option  (d) represents  the  answer.  E (3 ´ 104 )  18.2  =  11.  l m  = 19. 16.f 2 )  \  2  1 2 h  v12 .  Hence the slope is same for all metals and independent of  the  intensity  of  radiation.  \  m 1 21.  The  incident  radiation  lies  in  ultra  violet  region.f 2  2 1  m ( v 2 .  Kinetic energy of emitted electron = hf  – (work function f) 1  2  mv = hf1  .63 ´ 10 ) ´ (3 ´ 10 )  = f 4.3 ´ 10-18 C.6 ´ 10-19  or l  =  1110  ×  10 –10  m  =  1110  Å.  qE  =  mg \  mg  (9.63 ´ 10 -34 ) ´ (3 ´ 108 )  l= m  or  11.  n1  çè P ÷ç øè 1/ 2 ø  1  17.6  ×  10 –19 )  J hc = 11.  y  =  mx  +  c  Kinetic energy  q  f f Work function \  slope  of  line  =  h  h  is  Planck's  constant.f  \  2  1 1  2  mv = hf 2  .  (c)  :  Work  function  =  hc/l  W Na  4.v22  = ( f -  f 2 ).2  ×  (1.  l 1 K2 2 K1  2  P  of source  P  4 p (distance) 4 p d 2  Here.5 2  = = . according to Einstein's equation.  (c)  :  Emission  of  photo­electron  starts  from  the  surface  after  incidence  of  photons  in  about  10 –10  sec.  (d)  :  I  = 2 = 2  \  I 2 n2 P æd ö n  = Þ 2 ç 1 ÷ = 2  I1 n1 P1 è d 2 ø n1  \  n 2  æ P öæ 1 ö2  4  = ÷ = .v22 ) = h ( f1 .0 ´ 1.  WCu  2.6 ´ 10 -19  l m  =  310  nm.  Number of photo­electrons  emitted from a  surface  depend  on  intensity  of  light  I  falling  on  it.  15.  (c)  :  Let l m  =  Longest  wavelength  of  light hc = f (work function)  \  l m  \  or -34 8  hc (6.98  JEE MAIN CHAPTERWISE EXPLORER  E =  K m  + f  E  =  5  +  6.  Thus  the  number  of  electrons emitted n depends directly on I.  (d)  :  According  to  Einstein's  equation.6 ´ 10 -19  \  l  (6.2 ´ 1.  compare  it  with  equation.3 1  .  Kinetic energy = hf – f where kinetic energy and f (frequency)  are  variables.2  eV  =  11.  20.9 ´ 10 -15 ) ´ 10  q = = = 3.  (a) : For photoelectric effect.  E.73 MeV  (2012)  A diatomic molecule is made of two masses m 1  and m 2  which  are  separated by  a  distance  r. C.4  Z - 2  (b)  A . The ratio of  number  of  neutrons  to  that  of  protons  in  the  final  nucleus  will be  (a)  A . B.1 eV  (b)  36.  (b)  14  min  (d)  28  min  (2011)  A  radioactive  nucleus  (initial  mass  number  A  and  atomic  number Z) emits 3a­particles  and  2  positrons.  Speed of light is c.  If  we  calculate  its  rotational  energy  by  applying  Bohr’s  rule  of  angular  momentum  quantization.Z  .4  eV  (2011)  The  half life of a  radioactive  substance  is  20  minutes.Z  .  In  a  hydrogen  like  atom  electron  makes  transition  from an  energy level with quantum number n to another with quantum  number (n – 1). its energy will be  given by (n is  an integer)  (a)  n 2 h 2  2(m1 +  m2 ) r 2  (b)  2 n 2 h 2  ( m1 +  m2 ) r 2  (c)  ( m1 +  m2 ) n 2 h 2  2 m1m2 r 2  (d)  ( m1 +  m2 ) 2 n 2 h 2  2 m12 m2 2 r 2  (2012)  Energy required for the  electron excitation  in Li ++  from the  first to the  third Bohr orbit  is  (a)  12.99  Atoms and Nuclei  CHAPTER  ATOMS AND NUCLEI 18  1.  M  each.8  Z -  4  (c)  A .  The energy released during  this process is  (Mass of neutron = 1. against  the nuclear mass M; A.  5. the frequency of radiation emitted  is  proportional  to  1  1  1 1  (a)  3  (b)  (c)  2  (d)  3/ 2  n  n  n  n  (2013)  Hydrogen atom is excited from ground state to another state  with principal quantum number equal to 4.30 MeV  (c)  5.Z  .  6.4  Z - 8  (d)  A .  4.6725 × 10 –27  kg  Mass of electron =  9 ×  10 –31  kg)  (a)  7.  2  The  speed of daughter nuclei is  Dm  (a)  c  M + D m Dm  (b)  c M + D m (c)  c  2 D m  M (d)  c  D m  M (2010)  The binding energy per nucleon for the parent nucleus is E 1  and that for the daughter nuclei is  E 2 .  E b  F  A  M  The above is a plot of binding energy per nucleon E b . If n > > 1. Then  (a)  E 1  = 2E 2  (b)  E 2  = 2E 1  (c)  E 1  >  E 2  (d)  E 2  > E 1  (2010)  10.12  Z -  4  (2010)  Directions  :  Questions number  8­9 are  based  on  the  following  paragraph.Infrared radiation will be  obtained in the  transition from  (a)  2 ®  1  (b)  3 ® 2  (c)  4 ®  2  (d)  5 ® 4  (2009)  B  C  D  E  11.3 eV  (c)  108.  The transition from the state n = 4 to n = 3 in a hydrogen like  atom results in ultraviolet radiation.  The  approximate time interval (t 2  – t 1 ) between the time t 2  when  2  1  of it has decayed and time t 1  when  of it had decayed is  3  3  (a)  7 min  (c)  20 min  7.4 MeV  (d)  0. F correspond to different  nuclei. D.Z  .6725 ×  10 –27  kg  Mass of proton = 1.  3.10 MeV  (b)  6.  2.  A nucleus of mass M + Dm is at rest and decays into two daughter  nuclei of equal mass  8. Then the number  of spectral lines in the emission spectra  will  be  (a)  3  (b)  5  (c)  6  (d)  2  (2012)  Assume that a neutron breaks into a proton and an electron.8 eV  (d)  122.  9.  Consider four  reactions:  (i)  A + B ® C + e  (ii)  C ® A + B + e  (iii) D + E ® F + e  (iv)  F ® D +  E + e  where e is the energy released? In which reactions is e positive?  (a)  (i) and (iv)  (b)  (i) and (iii)  (c)  (ii) and (iv)  (d)  (ii) and (iii)  (2009)  .   (2006)  E  E 0  E  1  mv 2  bombards a heavy nuclear  2  target  of charge  Ze.  If M O  is the mass of an oxygen isotope 8 O 17 .  (c)  Statement­1  is true.  The half­life period of a radio­active element X is same as the  mean life time of another radio­active element Y. The  corresponding  half­life  is  1  2  (a)  5 minutes  (b)  7  minutes  (c)  10  minutes  (d)  14  minutes  (2005)  26.  The ‘rad’ is the correct unit used to report the measurement of  (a)  the  rate  of  decay  of  radioactive  source  (b)  the ability of a beam of gamma ray photons to produce  ions  in  a  target  (c)  the  energy  delivered  by  radiation  to  target  (d)  the  biological  effect  of  radiation.  statement­2  is  true;  statement­2  is  a  correct  explanation  for  statement­1.  12.  rn  µ n  (2008)  n 14.  Suppose an electron is attracted towards the origin by a force  k/r where k is a constant and r is the distance of the electron  from the origin. Which  of  the  following  is  the  nucleus  of  element X?  (a)  9  5 B  (b)  11  4 Be  (c)  12  6 C  (d)  10  5 B  (2005)  24. statement­2 is true; statement­2 is not a  correct  explanation  for  statement­1.  Statement­1 : Energy is released when heavy nuclei undergo  fission  or  light nuclei  undergo  fusion. Which transition shown represents the emission  of  a  photon  with  the  most  energy?  n = 4  n  = 3  n  = 2  I  (a)  I  II  (b)  II  IV  III  (c)  III  n = 1  (d)  IV  (2005)  25. statement­2  is true.  (a)  Statement­1 is true.  When 3 Li 7 nuclei are bombarded by protons.  Starting  with  a  sample  of  pure  66 Cu.100  JEE MAIN CHAPTERWISE EXPLORER  Directions : Question 12 contains statement­1 and statement­2. the  radius of the n th  orbital of the electron is found to be r n  and  the  kinetic  energy of  the  electron  to be T n .  rn  µ n 2  n n (c)  T n  independent  of  n. a) 3  Li . Then  the distance  of  closest  approach  for  the  alpha  nucleus  with  be  proportional  to  (a)  1/Ze  (b)  v 2  (c)  1/m  (d)  1/v 4 .  (b)  Statement­1 is false. By applying Bohr model to this system.  On passing through 36 mm of lead.  The intensity of gamma radiation from a given  source is I. and the resultant  nuclei  are  4 Be 8 .  (2006)  22.28 MeV  (d)  1. Then which  of  the  following is  true?  1  1  (a)  Tn µ . Initially they  have the same number of  atoms.  (2006)  21.  (2006)  20.  (2008)  13.60  MeV  and  7.24  MeV  (c)  17. choose the one that best describes the  two statements.  The energy spectrum of b­particles [number N(E) as a function  of b­energy  E]  emitted  from  a  radioactive  source  is  N (E )  N (E )  (a)  (b)  E  E 0  (c)  N (E )  E 0  (d)  E 0  N (E )  E  .  An alpha nucleus of energy  7  23.  r n µ  n  1  (d)  Tn µ .  If the binding energy per nucleon in  3  nuclei are  Li  and  4  2 He  5. Then  (a)  X and Y decay at same rate always  (b)  X will decay faster  than Y  (c)  Y will decay faster than  X  (d)  X and Y have same  decay rate initially  (2007)  16.  the  emitted  particles  will  be  (a)  neutrons  (b)  alpha  particles  (c)  beta  particles  (d)  gamma  photons. statement­2  is false. M P  and M N  are  the masses of a proton and a neutron respectively. it is reduced to I/8.  energy  of  proton  must  be  p + 73 Li ® 2 4  2 He  (a)  39.  Which of the following  transitions in  hydrogen atoms emit  photons of  highest  frequency  ?  (a)  n = 1 to n = 2  (b)  n =  2 to n = 6  (c)  n = 6 to n = 2  (d)  n =  2 to n = 1  (2007)  15.  In gamma ray emission from a nucleus  (a)  only the proton  number  changes  (b)  both the neutron number and the proton number change  (c)  there is no change in the proton number and the neutron  number  (d)  only the neutron  number  changes  (2007)  17.  then  in  the  reaction  :  .  (d)  Statement­1 is true.  The  diagram shows  the energy  levels  for  an  electron  in  a  certain atom.  Of the four choices given.  A nuclear transformation is denoted by X(n. The  thickness of lead which will reduce the intensity to I/2 will be  (a)  18  mm  (b)  12  mm  (c)  6 mm  (d)  9 mm  (2005) .  (2006)  19. binding  energy per nucleon  increases with increasing Z while for light nuclei it decreases  with increasing  Z.46  MeV.2 MeV  (b)  28. the nuclear  binding energy of the isotope  is  2  (a)  (M O  – 17 M N ) c 2  (b)  (M O  –  8 M P) c    (c)  (M O  – 8 M P  –  9 M N ) c 2  (d)  M O  c 2  (2007)  7  18.06  MeV  respectively.  7/8  of  it  decays  into  Zn  in  15  minutes.  Statement­2 : For heavy nuclei.  rn  µ n 2  (b)  Tn µ 2  .   7.  (c)  (a)  (c)  (d)  (c)  (a)  (c)  6.  If N 0  is the original mass of the substance of half­life period  t 1/2  = 5 years. If two  deuteron nuclei react to form a  single helium nucleus.  42.  24.7  ×  10 –14  J.2  MeV.  (c)  (a)  (c)  (c)  (d)  (b)  (a) .  (2004)  ( )  29. a.  (2002)  42.  40.  then  the energy  required to remove an  electron  from  n  =  2  is  (a)  10.  If radius of the  13  Al  nucleus is estimated to be 3. b – . The  Z  of  the  resulting  nucleus  is  (a)  76  (b)  78  (c)  82  (d)  74.  (c)  (c)  (d)  (d)  (b)  (c)  (b)  3. b – .  35. the rate is  1250  disintegrations  per  minute.  The  compound  can  emit  (i)  electrons  (ii)  protons  (iii) He 2+  (iv)  neutrons  The  emission  at  the  instant  can  be  (a)  i.  (2002)  Answer  Key  1.  25.6 eV.  33.  12.8  eV. then the amount of substance left after 15 years  is  (a)  N 0/8  (b)  N 0/16  (c)  N 0/2  (d)  N 0/4.  35.  (*)  (d)  (b)  (d)  (c)  (b)  (a)  (2003)  4.  38.  If the  binding energy  of the  electron in  a hydrogen  atom is  13.  (c)  (d)  (c)  (c)  (c)  (d)  (a)  5.  28.  Which  of  the  following  atoms  has  the  lowest  ionization  potential?  (a)  14  7 N  (b)  133 55 Cs  (c)  40  18  Ar  (d)  16  8 O.  In  the  nuclear  fusion  reaction.  (2004)  31.  the  decay  constant  (per  minute)  is  (a)  0.  the  temperature at which the gases must be heated to initiate the  reaction  is  nearly  [Boltzmann's  constant  k  =  1. b + .  39.1 MeV and 7 MeV respectively.  13.  15. The  ratio  of  their  nuclear  sizes  will  be  (a)  2 1/3  :  1  (b)  1  :  3 1/2  (c)  3 1/2  :  1  (d)  1 : 2 1/3 .9 MeV  (b)  26.  (2003)  38.  27.  41. b – . a.101  Atoms and Nuclei  27  27.  iii.1  ln 2  (d)  0. a. a.  14.  A  nucleus disintegrates  into  two  nuclear  parts  which  have  their velocities in the  ratio  2  :  1.  ii. the recoil speed of the residual  nucleus  is  4 u  4 u 4 u  4 u (b)  -  (c)  (d)  -  . The distance of the closest approach  is  of  the  order  of  (a)  1  Å  (b)  10 –10  cm  –12  (c)  10  cm  (d)  10 –15  cm.  11. a. a.6  eV  (c)  3.4  ln 2  (b)  0.  A radioactive sample at any instant has its disintegration rate  5000 disintegrations per minute.  (2003)  33.  32.  iv  (c)  iv  (d)  ii. After 5 minutes. b + .  When U 238  nucleus originally at rest.  iii  (b)  i.  The binding energy per nucleon of deuteron 1 2 H  and helium  nucleus 4  2 He  is 1. the energy required to remove the electron from the  first  excited  state  of  Li ++  is  (a)  30.  2 3 4  1 H +1 H ® 2 He + n  given  that  the  repulsive  potential  energy  between  the  two  nuclei  is  ~  7. decays by emitting an  alpha particle having a speed u.  37.  (2002)          41.  (2003)  36.8  ln 2.  22.2  eV  (b)  0  eV  (c)  3.  (a)  (c)  (c)  (d)  (a)  (a)  (a)  2.  36. a.9  MeV  (c)  23. then  the  energy  released  is  (a)  13.  19.2  ln 2  (c)  0.  (2003)  40.4  eV.  ii.  At  a  specific  instant  emission  of  radioactive  compound  is  deflected  in  a  magnetic  field.  29. b – .4  eV  (d)  122. because  (a)  size  of  the  two  nuclei  are  different  (b)  nuclear  forces  are  different  in  the  two  cases  (c)  masses  of  the  two  nuclei  are  different  (d)  attraction between the electron and the nucleus is different  in  the  two  cases.  30.  A  nucleus  with  Z  = 92  emits  the  following in  a  sequence: a.6 eV  energy is required to ionize  the  hydrogen  atom.  18.  Then.  10.  20.  9.  An a­particle of energy 5 MeV is scattered through 180° by  a fixed uranium nucleus.4  eV  (d)  6.  16.  (2003)  37.  26.  23.  Which  of  the  following  cannot  be  emitted  by  radioactive  substances  during  their  decay?  (a)  protons  (b)  neutrinos  (c)  helium  nuclei  (d)  electrons.  21.  34.  iii.  8.6  eV  (b)  13.6 MeV  (d)  19.38  ×  10 –23  J/K]  (a)  10 7  K  (b)  10 5  K  (c)  10 3  K  (d)  10 9  K.  Which of the following radiations has the least wavelength?  (a) g­rays  (b) b­rays  (c) a­rays  (d)  X­rays.  17.  (2003)  34.  (2003)  (a)  238  234  234  238  39.  The wavelengths involved in the spectrum of deuterium  (1 2 D)  are slightly different from that of hydrogen spectrum.  31.  If  13.  (2004)  30.  (2003)  ( )  32.6 fermi  then the radius  of  125 52 Al  nucleus be  nearly  (a)  4  fermi  (b)  5  fermi  (c)  6  fermi  (d)  8  fermi  (2005)  28.   E  =  2 I 3.1)  N =  2  Here.1)  =  6  2  = m1m 2  2  r  m1 +  m2  nh  2 p  n 2 h 2  4 p 2  E  = = 16  = = 0.  \ N= m1 r  m1 +  m2  Therefore.  N 0  =  N 0 e -lt  3  Number  of undecayed atoms after time t 1 .1) û  As n > > 1  2 2 æ m2 r ö æ m1 r  ö I = m1 ç + m 2 ç è m1 + m2 ÷ø è m1 +  m2  ÷ø 2  RcZ 2n 2 RcZ  =  n4 n3  or  u µ  1 3  n = L= n( n .(ii)  2  9 ´ 10 ´ 9 ´ 10  MeV  1.  (c) : A diatomic molecule consists of two atoms of masses  m 1  and m 2  at a distance r apart.  4.102  1.51 MeV  *  None  of the  given  option  is  correct.(i)  .  m2 r  m1 +  m2  (c)  :  Number of  undecayed atoms after time t 2 .  m  or  r1 =  2  r 2  m1  m  \ r1 = 2  ( r -  r 1 )  m1  (Using (ii))  In the question instead  of h.  JEE MAIN CHAPTERWISE EXPLORER  On rearranging.lt 1  0  3  0 Dividing (ii) by (i).6 ´ 10 -13  As  m1r1 =  m2 r2  .  (*) : Mass defect.(ii)  .8 eV  6.6725 × 10 –27  + 9 × 10 –31  –  1.(i)  L  Rotational  energy.6(3) 2  E1  = -  eV  (1) 2  E 1  = – 122..6 eV  (3) 2  DE =  E 3  –  E 1  = – 13. we get .  h should be given.6 ´ (3) 2  = -13. Dm  =  m p  + m e  –  m n  = (1.  Q  r1 + r2  = r .  E n  = -  2  eV  n Here.2  ù 2 ú ëê ( n . the  moment of inertia  can be  written  as  \ u= 2.1) ë (n )( n .1)  = RcZ 2  ê 2 = 2 ú n 2 ( n .  2  2  N =  N e .4 eV  \  COM  r 2  r 1  r  and  E3  = The moment of inertia of this molecule about an axis passing  through its centre of mass and perpendicular to a line joining  the atoms is  I = m1r12 + m2 r 2 2  -13.(n .1) 2  ù RcZ 2 (2n ... Let r 1  and r 2  be the distances  of  the  atoms from  the  centre  of  mass.6725 ×  10 –27 )  kg  = 9 × 10 –31  kg  Energy released = Dmc 2  = 9 × 10 –31  × (3  × 10 8 ) 2  J  -31 2  According  to  Bohr’s  quantisation  condition  (c)  : Number of spectral lines  in the emission spectra. the frequency  of  emitted radiation  is  1 1  u = RcZ 2  é . we get  (a)  : In a hydrogen  like  atom..  r 2  = é n 2 ..6 + 122.  n 2 h 2  8 p 2 I n 2 h 2 (m1 + m 2 )  8p 2 ( m1m2 ) r 2  n 2 h 2 (m1 + m 2 )  2 m1m2 r 2  (Using (i)) (Q h = 2 h p )  5. n = 4 or  L2  = 4(4 .4 = 108. Z = 3 (For Li ++ ) m 1  m 2  13..  13.  when  an electron  makes an  transition from an energy level with n to n – 1..6 Z 2  (c) :  Using..1)  n û  r 1  = Similarly.   –13.t1 ) = l  As per question.e.  n = 2 Balmer  Balmer  n = 1 Lyman  Lyman  n = 1     – 13.  When a radioactive nucleus.6 eV  æ 1 1 ö hu2®1  = -13. E 2  > E 1.  hence binding energy per nucleon of daughter nuclei is more  than that  of their  parent nucleus.2 eV . Q = ( ) ( )  1 M 2 1 M  2 1  v + v .( M +Dm) ´ (0) 2  2 2 1 2 2 2  2  =  M  v1 2  (Q  v1 = v2 )  2  11..  + 3a 2 e  \ ZA X ¾¾ ® AZ. only A and  B  are  light  elements  and  D  and  E  are  heavy  elements.  (a) : Statement­1 states that energy is released when heavy  nuclei  undergo  fission  and  light  nuclei undergo  fusion  is  correct. we  get ( M 2  ) v = D mc 2 1  v 1 2  =  2 Dmc 2  M = + 13. The answer is (a). B/A.  (c) : When a radioactive nucleus emits an alpha particle.  This  is  not  in  the  ultraviolet region  but this is in infrared region.  Hence.  (a)  : When two nucleons combine to form a third one. In the possibility of fission is only  for  F and  not  C. there is no change  in the proton and neutron number. DM = éêë ( M + Dm) .Z  .6 ´ (d)  :  After  decay.(i)  According to law  of  conservation  of  momentum.  we  get  ( M + Dm ) ´ 0 = M ´ v1 .4  \ = . starts at a small value. The possibility of  fusion is  more  for  light  elements  and  fission  takes  place  for  heavy  elements.-12 8 W  In the  final nucleus. Therefore  F ®  D + E  + e is the correct  choice. t 1/2  =  half life time =  20  min éQ  t = ln 2 ù 1/2  êë l úû \  t 2  –  t 1  = 20 min  7.  15.  Out of the  choices  given for fusion.  1  e 2  mv 2  4 pe 0  r would have been  =  r 2  \ mv 2 = e  = k Þ 1 mv 2  = 1 k ..  4 pe 0  2 2  This is independent of  n. r n µ n.  the  daughter  nuclei  will  be  more  stable.  (b) : T 1/2 .2 ÷ eV  è 2 1  ø  2  v1  =  c  2 Dm  M 9.  The binding energy per nucleon..6 ç 12 .  Statement­2 is wrong.  Number of protons.5 MeV for  the  heavy nuclei.(ii)  13.  2 p  as mv  is independent of r..12  ÷ = +13.  ( )  M M  (c) : Mass defect. If a single nucleus  splits  into two.6 ç 2 . .    n = 5  Pfund  10.6 eV  2  2  AX  =  A0 e-l X t  ;  AY  =  A0 e -lY t  16. then decreases to 7. half life of X = t Y .6 ´ 2  9  6 2 è ø  This is <E n =  2 ® E n =  1 .  14.  (c)  :  Supposing  that  the  force of  attraction  in  Bohr  atom  does not follow inverse square law but inversely proportional  to r.  N n  =  A – 12 – (Z –  8)  = A – Z – 4  N n  A .2 + 2  ùúû  = [M + Dm – M] = Dm  Energy released. Q = DMc 2  = Dmc 2  .  12.  æ ö -13.-126Y ¾¾ ¾ ®  A Z .  N p  =  Z –  8  Number of neutrons.103  Atoms and Nuclei  2 = e l (t2 . its  mass  number  decreases  by  4  while  the  atomic  number  decreases by 2. higher n to lower n.  (d) :  n = 2  hu 2 ®  1 Equating equations (i) and (ii).  Therefore  A + B ® C + e is correct.  rises to a maximum at 62 Ni.  4 Emission is n = 2 ® n = 1 i. one has fission.  (c)  : g­ray  emission  takes  place  due  to  deexcitation  of  the  nucleus.  Transition from lower to higher levels are absorption lines.  or   ln 2 = l(t 2  –  t 1)    ln 2  or  (t 2 .  N p  Z  - 8 8..t 1 )  Transition  4 ®  3  is  in  Paschen  series. emits a b +  particle (or positron  (e + )) its mass number remains unchanged while the atomic  number decreases by 1.  From mvrn  = nh . one has fusion reaction.  (d)  :  3  eV = 10. and  energy is released.M  ´ v2 or  v1 = v2  2 2  Also. mean life of Y  ln 2 1  = Þ l X = lY  ln 2  l X lY l X  > l Y n = 4 Brackett  \  n = 3 Paschen  X will decay faster than Y.  .  Transition 5 ® 4 will also be in infrared region  (Brackett). Therefore during g­ray emission. 60 = 39..2  = 17.  (c)  :  Kinetic  energy  is  converted  into  potential  energy  at  closest  approach \  K..  T \  1/ 3  æ m v  ö = ç 2 2  ÷ .. from  (i) è m2 v1  ø  R 1  æ 1 ö 1/ 3  1  = = 1/ 3  .  (c)  :  Binding  energy  of  7  3 Li = 7 ´ 5. kinetic energy is converted into  potential  energy \  1 mv 2  = 1 q1q 2  = 1  ( Ze )(2 e )  2 4 pe0 r0 4 pe 0 r0  or  r 0  = or  æ 1  ö r 0  is  proportional  to  ç m ÷ ..6 ´ 10 -19 )  = 5 ´ 106  –14  r  =  5.2  MeV  Total  binding  energy  for  helium  =  4  ×  7  =  28  MeV \  Energy  released  =  28  –(2  ×  2.3  ×  10  m  =  5.28  MeV.  18. III  having  maximum  energy  from  æ 1 1  ö DE  µ ç 2 .(i)  In  second  case.2 Me V Binding  energy  of  4  2 He = 4 ´ 7.kx  è I 0 ø  In  first  case  \  + 10 n ®  24 He + 3 7 Li  According  to  conservation  of  mass  number  A  +  1 =  4  +  7  or  A  =  10  According  to  conservation  of  charge  number  Z  +  0 ®  2  +  3  or  Z  =  5  \  I  .  22.  (b)  :  Q  I  =  I 0  e –kx Þ  I = e  0  R1 æ A 1  ö = R2 çè A2  ÷ø  1/ 3 3 æm ö =ç 1 ÷ è m2 ø 15 / T 1 æ1ö 1 1  = Þ æç ö÷ = æç ö÷ 8 çè 2 ÷ø è 2 ø è 2 ø 15  = 3 Þ T  = 5 min.  7 1 A  8  20.  =  P.  24.2)  =  28  –  4.  (c)  :  R  is  proportional  to  A 1/3  where  A  is  mass  number  3.06 = 28.k ´ x è 2 ø  or  ln(2 –1 )  =  –  kx  or  ln2  =  kx  From  (i)  and  (ii)  3 ×  (kx)  =  k  ×  36  or  x  =  12  mm..  for  (3 × 6)  R= ´ 5 = 6 fermi  .24MeV \  4 7  Energy  of  proton  =  Energy  of  éë 2(2 He) -3  Li ùû  =  2  ×  28.(i)  1/ 3  \  t / T  15 / T \  æ I  ö ln ç ÷ = ..  ln æ 1 ö = .  for  27  13  Al ..  R2  è 2 ø  2 .  (d)  :  Graph  (d)  represents  the  variation.  30.E.  (c) : Binding energy =  [ZM P  +  (A – Z) M N  –  M]c 2  = [8M P  + (17  – 8) M N  –  M O ]c 2  = (8M P  + 9M N  – M O )c 2  [But the option given is  negative of  this]..  R  =  R 0 A 1/3 .k ´ 36  è 8 ø  ln  (2 –3 )  =  –  k  ×  36  or  3ln2  =  k  ×  36  .  (d)  :  The  'rad'  the  biological  effect  of  radiation.6  MeV..6  =  R 0  (27) 1/3  =  3R 0 .E..  (c)  :  I  is  showing  absorption  photon..1  = 2...104  JEE MAIN CHAPTERWISE EXPLORER  17..  (a)  :  N = ç 2 ÷ è ø 0  1  ln æ ö = .  (d) :  Momentum is  conserved  during  disintegration \  m 1 v 1  =  m 2 v 2  For  an  atom.  (d)  :  3 Li + 1H ® 4 Be  +  Z  X Z  for  the  unknown  X  nucleus  =  (3  +  1)  –  4  =  0  A  for  the  unknown  X  nucleus  =  (7  +  1)  –  8  =  0  Hence  particle  emitted  has  zero  Z  and  zero  A  It  is  a gamma  photon..  è ø  4 Ze2 = Ze 2  æ 1  ö ç ÷ 4 pe0 mv 2 pe 0 v 2  è m ø 23.24  –  39..  (c) : For closest approach..  çn n2  ÷ø  è 1 N  æ 1 ö 25..  19..kx  26..  (d)  :  The  nuclear  transformation  is  given  by  A  X Z  So  the  nucleus  of  the  element  be 10 5 B..  (ii)  27.  From rest of three.4  =  23.2  ÷ ..  21..3  ×  10 –12  cm.  .  \  3  28.  (c)  :  Total  binding  energy  for  (each  deuteron)  =  2 ×  1. q q  5 MeV = 1  1 2  \  4 pe 0  r or  or  \  (9 ´ 109 ) ´ (92 e )(2 e )  r 9 ´ 109  ´ 92 ´ 2 ´ e  r= 5 ´ 10 6  9 ´ 109 ´ 92 ´ 2 ´ (1.  5 ´ 10 6  ´ e  =  29.  125  52  Al Again  R  =  R 0  (125) 1/3 . .  4( –1b 0 ) \  Increase  in  Z  =  4  ×  1  =  4  . Electrons in the outer  most orbit are at large distance from nucleus in a large­size  atom.7 ´ 10-14  J  2  -14  2  = 3...(ii)  2  positrons  are  emitted  i..4ln 2..  (b) :  133 55 Cs  has the lowest ionization potential..  5 38.  33.e..  34.  finally  =  1250 \  Nl  =  1250  N l 1250 1  = = \  N 0 l  5000 4  or  \  \  .  234  39..  Hence  (a)  represents  the  answer.  t / T  36.6  41..6 eV  2 n (2) 2  energy  required  =  30..4 eV  n2 (2) 2  42..  8( 2 He 4 ) \  Decrease  in  Z  =  8  ×  2  =  16  .e..  T = 7. Cs has the largest size...  (i)  ...  35. They are not deflected  by  magnetic  field.38 ´ 10  Disintegration  rate..  (b)  :  Linear  momentum  is  conserved a­particle  =  4 2 He  U 238 ®  X 234  +  He 4 \  (238 ×  0) =  (238  ×  v)  +  4u  4 u  or  v = -  ..  (ii) N e -5 l 1  N  1 = Þ 0  = Þ e -5 l = (4) -1  N0 4 N 0  4  5l  =  ln4  =  2ln2 2  l = ln 2 = 0.  37. molecules of a gas acquire a kinetic  3  energy  =  kT where  k  =  Boltzmann's  constant 2  \  To  initiate  the  fusion  reaction  3  kT = 7...105  Atoms and Nuclei  \  ... initially  =  5000 \  N 0l  =  5000  Disintegration  rate.  (a) : Neutrons are electrically neutral..  (d) : At temperature T..  31.  (a)  :  Let  decay  constant  per  minute  = l  15 / 5 N  æ 1 ö æ1ö 40.6  = -30.  (c)  :  En  = 13.(iii) \  Z of resultant  nucleus  =  92  –  16  +  4  –  2  =  78..7 ´ 109  K.(i)  Four b –  particles  are  emitted  i.  (a) :  Gamma  rays  have  the  least  wavelength.  Hence  the  corresponding  wavelengths  are  different...  (a)  :  Protons  are  not  emitted  during  radioactive  decay.  3  1 1  = æç ö÷ = 8  è2ø 13.  Hence  the  ionization  potential  is  the  least....6 Þ E 2  = = 3.  2( 1b 0 ) \  Decrease  in  Z  =  2  ×  1  =  2  .(3)2  ´ 13. Of the four  atoms given.Z 2 E 0  .6  eV...e.  (c)  :  Masses  of  1 H 1  and  1 D 2  are  different... ...  (a)  :  Energy  E 2  = = 32..  (b)  :  The  nucleus  emits  8a  particles  i.  (a)  :  N = ç 2 ÷ = ç 2 ÷ è ø è ø 0  \  N  =  N 0 /8......7 ´ 10 ´-23  \  3 ´ 1.   3.  The current(I) in the resistor(R) can be shown  by  .31 MHz  (2013)  Input B  Output is  (a)  Truth  table  for  system  of  four  NAND  gates  as  shown  in  figure  is  A  (b)  Y  B  (a)  A  B  Y  A  B  Y  0  0  0  0  0  1  0  1  0  0  1  1  1  0  1  1  0  0  1  1  1  1  1  0  (b)  (c)  6.  The I­V characteristic  of an  LED is  4.  The  combination of gates shown  below yields A X  (a)  (b)  B (a)  NAND gate  (c)  NOT gate  5.106  JEE MAIN CHAPTERWISE EXPLORER  CHAPTER  ELECTRONIC DEVICES 19  1.31 kHz  (b)  10.  D  R  V  (2012)  An alternating current source (V) is connected in the circuit. Pick out the correct  output waveform.  A  Y  B  (2013)  Input A  2.  (a)  5.  A  diode  detector is  used  to  detect  an  amplitude  modulated  wave of 60% modulation  by  using  a condenser  of  capacity  250  pico  farad  in  parallel  with  a  load  resistance  100  kilo  ohm.62 kHz  (d)  5.62 MHz  (c)  10. Find the maximum modulated frequency which could  be  detected by  it.  (c)  (d)  (b)  OR gate  (d)  XOR gate  (2010)  The logic circuit shown below has the input waveforms ‘A’  and ‘B’ as shown.  (c)  A  B  Y  A  B  Y  0  0  1  0  0  0  0  1  0  0  1  1  1  0  1  1  1  0  1  0  0  1  1  1  (d)  (2009)  (d)  A p­n junction (D) shown in the figure can act as a rectifier.   (d)  The number of free electrons for conduction is significant  in all the three.  E v  decrease  all  E c .488  mA for an  emitter current  of 5.00 A  (b)  1. No conduction is found between  P and Q.  but  E g  decreases  E c .  silicon  and  germanium  have  four  valence  electrons  each.  (b)  It is an npn  transistor with  R as base.  In the following.  What  is  the  current  following  in  the  circuit?  (b)  t  4 W  I  (c)  D 1  D 2  t  (2009)  (d)  t  7.  9.  then  which  of  the  following  is  correct?  E c  Conduction band width  Band gap  E g  Valence band width  (a)  (b)  (c)  (d)  all  E c .  In  common base mode  of  a  transistor.71 A  (d)  2.  (b)  The number of free conduction electrons is significant in  C but small in Si and  Ge.  (b)  NOR gate  (d)  NAND gate.60 mA.  A  and  B  represent  two  inputs  and  C  represents the  output.  (d)  It is a pnp  transistor with  R as  emitter.  If in a p ­ n junction diode. The  circuit represents (a)  1.  E v  (2006)  14.  and  E v  decrease.  and  E v  increase. Which of  the  following is true  for the  transistor?  (a)  It is an npn transistor with  R as collector.  (2008)  A working transistor with its three legs marked P.107  Electronic  Devices  I  (a)  10 V  + 5V  (b)  (a)  t  (c)  I  (d)  –10 V  (2007)  – 5V  11.  E v  increase  E c .  E g .  (c)  The  number  of  free  conduction  electrons  is  negligibly  small in all the three.  (2008)  Carbon. a square input signal of 10 V is  applied as shown  5 V  R L  –5 V  Then the output signal across R L  will  be  – 12V  (c)  R  – 5V  R  (d)  . At  room  temperature  which  one  of  the  following  statements is most appropriate ?  (a)  The number of free electrons for conduction is significant  only in Si and  Ge but small in C.  but  E g  increases.  If  the  lattice  constant  of  this  semiconductor  is  decreased.31 A. Q and R  is tested using a multimeter. which one of the diodes is reverse biased?  + 5V  A  + 10V  R  C  (a)  B  R  (b)  + 5V  (a)  OR gate  (c)  AND gate  8. some resistance is seen on the multimeter.  (c)  It is a pnp  transistor with  R as  collector.  In  the  ratio  of  the  concentration  of  electrons  that  of  holes  in a semiconductor is 7/5 and the ratio of currents is 7/4 then  what  is  the  ratio  of  their  drift  velocities?  (a)  4/7  (b)  5/8  (c)  4/5  (d)  5/4.  E g .  (2006)  12.  A  solid  which  is  transparent  to  visible  light  and  whose  conductivity  increases  with  temperature  is  formed  by  (a)  metallic binding  (b)  ionic  binding  (c)  covalent  binding  (d)  van der Waals binding  (2006) .  (2006)  16.  The  value  of  the  base  current  amplification  factor  (b)  will  be  (a)  48  (b)  49  (c)  50  (d)  51.  (2006)  – 10V  13.  (2006)  15.33 A  (c)  2. By connecting the common (negative) terminal of  the  multimeter to R and  the  other (positive)  terminal to  P  or Q. the collector  current  is  5.  The circuit has two oppositely connect ideal diodes in parallel.  (2007)  10.  2 W  3 W  12V  I  In the  circuit below. 78.  13.  25.  The  manifestation  of  band  structure  in  solids  is  due  to  (a)  Heisenberg’s  uncertainty  principle  (b)  Pauli’s  exclusion  principle  (c)  Bohr’s  correspondence  principle  (d)  Boltzmann’s  law  (2004)  22.2  (b)  –15.  Si  acts  as  (a)  non­metal  (b)  metal  (c)  insulator  (d)  none  of  these.1  eV  (d)  2.5  eV  (b)  0.  14.  9.5  eV  (2005)  20.  The  energy  band  gap  is  maximum  in  (a)  metals  (b)  superconductors  (c)  insulators  (d)  semiconductors.  The  resistance  of  (a)  each  of  these  decreases  (b)  copper strip increases and that of germanium decreases  (c)  copper strip decreases and that of germanium increases  (d)  each  of  these  increases.  23.  (a)  (c)  (a)  (d)  (a)  6.8  (d)  – 48.  the  specific  resistance  of  a  conductor  and  a  semiconductor  (a)  increases  for  both  (b)  decreases  for  both  (c)  increases.  26.  20.  7.  31.  22.  12.  (2004)  25.  8.  When  p­n  junction  diode  is  forward  biased.  then  (a)  the  depletion  region  is  reduced  and  barrier  height  is  increased  (b)  the  depletion  region  is  widened  and  barrier  height  is  reduced.  At  absolute  zero.108  JEE MAIN CHAPTERWISE EXPLORER  17.  In  the  middle  of  the  depletion  layer  of  a  reverse­biased  p­  n  junction.  24.  (b)  (a)  (c)  (c)  (a)  5.  (2002)  29.  (2003)  27.  (2004)  24.  (2002)  32.  29.  When npn  transistor  is  used  as  an  amplifier  (a)  electrons  move  from  base  to  collector  (b)  holes  move  from  emitter  to  base  (c)  electrons  move  from  collector  to  base  (d)  holes  move  from  base  to  emitter.  decreases  (d)  decreases.  (2002)  Answer  Key  1.  Formation  of  covalent  bonds  in  compounds  exhibits  (a)  wave  nature  of  electron  (b)  particle  nature  of  electron  (c)  both  wave  and  particle  nature  of  electron  (d)  none  of  these.  11.  A  strip  of  copper  and another  germanium  are  cooled  from  room  temperature  to  80  K.  30.7  eV  (c)  1.  (2003)  28.  21.  In  a full  wave rectifier circuit operating from 50 Hz mains  frequency. the fundamental frequency in the ripple would be  (a)  100  Hz  (b)  70.  10.  The difference in the variation of resistance with temperature  in a metal and a semiconductor arises essentially due to the  difference  in  the  (a)  crystal  structure  (b)  variation  of  the  number  of  charge  carriers  with  temperature  (c)  type  of  bonding  (d)  variation  of  scattering  mechanism  with  temperature.  In a  common base amplifier.  The part of a transistor which is most heavily doped to produce  large  number  of  majority  carriers  is  (a)  emitter  (b)  base  (c)  collector  (d)  can  be  any  of  the  above  three.  (c)  (a)  (b)  (c)  (b)  (c) 3.  31.  16.  The electrical conductivity of a semiconductor increases when  electromagnetic  radiation  of  wavelength  shorter  than  2480  nm  is  incident  on  it.  increases.7  (c)  – 24.  27.  (c)  (a)  (a)  (a)  (c)  .  By  increasing  the  temperature.  the  (a)  electric  field  is  zero  (b)  potential  is  maximum  (c)  electric  field  is  maximum  (d)  potential  is  zero.  19.  The  band  gap  in  (eV)  for  the  semiconductor  is  (a)  0.  For a transistor amplifier in common emitter configuration  for load impedance of 1 kW  (h fe  = 50 and h oe  = 25) the  current  gain  is  (a)  –5.  17.  18.7  Hz  (c)  50  Hz  (d)  25  Hz  (2005)  18.  the  resistance  of  (a)  each  of  them  increases  (b)  each  of  them  decreases  (c)  copper  decreases  and  germanium  increases  (d)  copper  increases  and  germanium  decreases.  A piece of copper and another of germanium are cooled from  room  temperature  to  77  K.  (d)  (a)  (d)  (b)  (c)  (2002)  4. the phase difference between  the  input  signal  voltage  and  output  voltage  is  (a)  0  (b) p/2  (c) p/4  (d) p  (2005)  19.  (b)  (a)  (d)  (a)  (a)  (c)  2.  32.  (2002)  30.  (c)  both the depletion region and barrier height are reduced  (d)  both the depletion region and barrier height are increased  (2004)  21.  15.  28.  (2004)  23.  (2003)  26. 6 \  u= A  B  A  B  A × B 0  0  1  1  1  0  1  1  0  0  1  0  0  1  0  1  1  0  0  0  This is same as that of OR gate.  (c)  : The maximum frequency which  can  be  detected is  1  u= 2 pma t  where. If it is connected  to base. the gate  conducts.  = 10.  11.  (c)  :  Since diode D 1  is reverse  biased.  Y  0  0  1  Y  1  1  1  1  0  Y  1  1  X B  B (c)  : (a) is original wave (b) is a full­wave rectified (c) is the  correct  choice.61 kHz.  12.  A  0  0  0  3.  (a) : It is OR gate.  9. these electrons are in the second  orbit.  the  option  (b) represents  the  correct  graph.109  Electronic  Devices  1.  1  1  1  1  0  1  1  B  1  1  1  1  1  0  0  0  1  1  1  1  7.  (a) : It is npn transistor with R as  collector. ( A + B ) = A × B.  0  0  A (b) : A 6.  (d) : E c and E v  decrease but E g  increases if the lattice constant  of  the  semiconductor  is  decreased. Both are  semiconductors.  Alternative method  The truth table of the given circuit is as shown in the table  (b)  : The I­V characteristics of a LED is similar to that of a  Si junction diode.  (a)  :  Y  B  1  2p ´ 0.61 ×  10 3  Hz =  10. Si and  Ge have the same lattice structure and their  valence electrons are 4.  13.  8.  In  solid  state.  A  5.6 ´ 250 ´ 10 -12 ´ 100 ´ 103  By de Morgan’s theorem. Ionization energies are also less  therefore Ge has more conductivity compared to Si. When either of them conducts.Carbon is an insulator.  greater  the  possibility  of  overlapping of energy bands. it will be  in forward bias.  C = 250 pico farad = 250 × 10 –12  farad  R = 100 kilo ohm = 100 ×  10 3  ohm  m a  = 0.  (d)  :  B  0  1  1  1  1  1  0  0  0  X = A × B 0  1  1  1  Y  1  1  A B A B A+ B A + B Verify  A × B  1 1 0 0 0 1  1  0 0 1 1 1 0  0  0 1 1 0 1 0  0  1 0 0 1 1 0  0  This is the same as AND Gate of A and B. therefore  it will  act  like  an  open  circuit. for Si it is third and germanium it is the fourth orbit.  Effective resistance of the circuit is R = 4 + 2 = 6 W.  higher  the  orbit. For C.  (a) : The current will flow through R L  when diode is forward  biased.  This is same as the Boolean expression  of OR gate.  The  Boolean expression  of  the  given  circuit  is  X = A× B = A +  B (Using De  Morgan theorem)  = A + B (Using Boolean identity)  10.  The  negative  waves  are  cut  off  when  the  diode is connected in reverse bias (d) is not the diagram for  alternating current.  (a) : C.  (a)  :  Figure  (a)  represent  a  reverse  biased  diode.  A  0  0  0  B  A  1  1  1  1  1  B  A  0  0  1  4.  2. .  Current in the circuit is I = E/R = 12/6 = 2 A.  Hence. t = CR  Here. But the threshold voltages are much higher  and  slightly  different  for  each  colour.   (b) : Pauli's exclusion principle explains band structure of  solids.  31.60 - 5. like Si.  I b I e .  (d)  :  Drift  velocity  v d  =  nAe ( vd ) electron  æ I e ö æ n h  ö = ç ÷ ç ÷ = 7 ´ 5 =  5  .  (a)  :  Electric  field  is  zero  in  the  middle  of  the  depletion  layer  of  a  reverse  baised  p­n  junction.  27.  (c)  :  Copper  is  a  conductor. r  decreases  as  temperature  rises.  ( vd ) hole  4 7 4 è I h ø è ne  ø  14. r  increases  as  temperature  rises.488  = 49. the  resistance of copper strip decreases  and  that  of  germanium  increases.025 24.  (b) : Variation of number of charge carriers with temperature  is  responsible  for  variation of resistance  in a  metal  and  a  semiconductor. Ge.  (d)  :  In  common  emitter  configuration.488 = = = 5.  20. act  as insulators  at  low  temperature.110  Ic I c  5.112  I  15.  22.  (a) : Electrons of n­type emitter move from emitter to base  and then  base  to  collector  when  npn  transistor  is  used  as  an  amplifier.  28.  1 + 0.I c  5.  (c) :  Semiconductors.  30.63 ´ 10 -34 ) ´ (3 ´ 108 )  eV  \  Band  gap  = (2480 ´ 10 -9 ) ´ (1.  21. the resistance of copper decreases and that of  germanium increases.  (c)  :  Covalent  binding.  (c)  :  For  conductor.  (c) :  The energy  band  gap  is  maximum  in  insulators.  25.  (a) : In a common base amplifier.  19.  18.  (a)  :  Frequency  of  full  wave  rectifier  =  2  ×  input  frequency  =  2  ×  50    =  100  Hz.  Germanium  is  a  semiconductor.  17.  (c)  :  When  p­n  junction  diode  is  forward  biased.  When  cooled.  (a)  :  Band  gap = Energy  of  photon  of l  =  2480  nm hc hc  \  Energy = l J = le (eV)  (6.488 0.  23.  (a) : Wave nature of electron and covalent bonds are correlated.  For  semiconductor.  (c) : Copper is conductor and germanium is semiconductor.  current  gain  is  .  =- 26.  32.  29.( h fe )  -50  = A i  = 1 + (25 ´ 10 -6 ) ´ (1 ´ 103 ) 1 + ( hoe )( RL  )  50 = -50  = –  48. .  (a)  :  The  emitter  is  most  heavily  doped. the phase difference between  the  input  signal  and  output  voltage  is  zero.  both  the  depletion  region  and  barrier  height  are  reduced.025 1.78.  (b)  :  b = 16.5  eV.6 ´ 10-19 ) =  0.  JEE MAIN CHAPTERWISE EXPLORER  When cooled.   Then  (a)  36 >  x  > 18  (b)  18  >  x  (c)  x  >  54  (d)  54 >  x > 36.  (c)  . f)  (d)  (4f. she  measures  the  column  length  to  be  x  cm    for  the  second  resonance. a student notes the  main scale reading of 3 mm and the number of circular scale  divisions in line with the main scale as 35. The total number of  divisions on the circular scale is 50. The diameter of  the wire is  (a)  3.73 mm  (d)  3.  An experiment is performed to find the refractive index of  glass  using  a  travelling  microscope. 29 divisions of the main scale exactly  coincide  with  the  30  divisions  of  the  vernier  scale.026 cm  (d)  0.  While  measuring the diameter of a thin wire.67 mm.  Total divisions on the vernier scale is 30 and match with 29  divisions of the main scale.  7. it is found that  the  screw  gauge  has  a  zero  error  of  –  0.  then the least count of the  instrument is  (a)  one  minute  (b)  half  minute  (c)  one  degree  (d)  half  degree  (2009)  distance v. 4f)  (2009)  5. The angle of the prism from the  above data  (a)  58. The coordinates of P will  be  (a)  (2f. is plotted using the same scale for  the two axes.  2.  Two full turns of the circular scale of a screw gauge cover  a distance of 1 mm on its main scale.  Main scale reading : 58.  (2008)  7. A graph between the object distance u and the image  Answer  Key  1.  f/2)  (c)  (f.  a  student  gets  the  first  resonance condition  at a  column length  of   18  cm  during  winter.  the screen  is  adjusted  to  get  a  clear  image of  the  object. Repeating the same experiment during summer.65  degree  (c)  59  degree  (d)  58.  4.052  cm  (c)  0.  (b)  6. 2f)  (b)  (f/2.77  degree  (b)  58.59  degree  (2012)  A  screw  gauge  gives  the  following  reading  when  used  to  measure the diameter  of  a wire.  If  the  smallest division of the main scale is half­a­degree (= 0.5 degree.  (b)  3.5°).111  Experimental  Skills  CHAPTER  EXPERIMENTAL SKILLS  20  1.  (2008)  6.  3.  (a)  5. with the position of the object fixed.  (b)  (a) 2.  In  this  experiment  distances are measured by  (a)  a screw gauge provided  on the microscope  (b)  a vernier  scale provided  on the  microscope  (c)  a standard laboratory scale  (d)  a meter  scale provided  on the microscope. Further.5 degree  Vernier scale reading : 09 divisions  Given that 1 division on main scale corresponds to 0.  (2008)  In an optics experiment.52 cm  (b)  0. A straight line passing through the origin and  making an angle of 45° with the x­axis meets the experimental  curve at P.  The  diameter of wire from the above  data is  :  (a)  0.  Main scale reading  :   0  mm  Circular scale reading  :     52 divisions  Given that  1 mm on main  scale  corresponds to  100 divisions of the  circular  scale.  While  measuring  the  speed  of  sound  by  performing  a  resonance  column  experiment.32 mm  (c)  3.005  cm  (2011)  In  an  experiment  the  angles  are  required  to  be  measured  using an instrument.38 mm  (b)  3.  a  student varies the  position  of a  convex  lens  and  for each  position.  (a)  4.03  mm. from the lens.  A  spectrometer  gives  the  following  reading  when  used  to  measure the angle of a  prism.   30 30 2 60 (a)  : According  to  New  Cartesian  coordinate  system  used  in  our  12 th  classes.  v u f 2 f 2 f f The answer is the same (a).5 ° + 0.5 mm  = = 0.S.  v and u are have the same value when the object is   at  the  centre  of curvature. = L 2  = v  P  u  v 2  3 l = > 3 ´ 18.D  L 1  = (n .  n 29  L .D  n L.1  M. v = 2f  1 1 1  i.  (The figure given is according to New Cartesian system). The solution is (a).S.C.052 cm  = 6.  provided  with  the  microscope.C.  g RT  assuming M is the average molar mass of  M the air (i.  as  u  is  negative.S.  v1 = \ 1 V.C.  it  1 1 1  effectively comes  to  v + u =  f .  (a)  :  Least count =  L 2  value of 1 main scale division  The number of divisions  on the vernier scale as shown below.  v u f One  has to  take that  u  is  negative  again  for  calculation.  0. =  n .5 °  30 = 1 M.e.D.  n 4  \  L 2  > 54 cm.D. u is +ve and v is also  +ve as both are real.1 =  1 .35 = 3. = 0.5 ° = 58.  v 2  >  v 1 .  \ 1 + 1 = 1 Þ 1 + 1 =  1 .03)  = 3.  2 f + 2 f =  f .S.1)  M.5 °  30 30 Reading = Main scale reading + Vernier scale  reading × least count  = 1- = 58.D -  Þ Þ 4.01 mm  100 Diameter of wire =  Main scale reading  + circular scale reading ×  Least count  = 0 + 52 ×  0.  45° 1 .52 mm = 0.03 \  actual diameter of the wire = 3.  (b)  : Least count of  screw  gauge  Pitch  =  Number of divisions on circular scale 5.5° ´ 0.65° .. summer.1 V.  Here n vernier scale  divisions =  (n –  1)  M. nitrogen) and g  is also for nitrogen.  30 2.112  1.  (a) :  Least count of the screw gauge  0. u = 2f  = radius of curvature.5 1 1 1  = ´ = ° = 1 min .e.  According the real and virtual system.01  = 0.5° + 9 ´ 0.  If u = radius of curvature.01 mm  50 Main scale reading = 3 mm.  v 1  l = = 18 cm.  (c) :  v 1  =  n  n  L 1  3.S.  Vernier scale reading = 35 \ Observed reading = 3 + 0.  1  mm = 0.  JEE MAIN CHAPTERWISE EXPLORER  (b)  : 30 VSD = 29 MSD  1 VSD =  29  MSD  30 Least count = 1 MSD – 1 VSD ( )  29 1  MSD = ´ 0.  L .  At temperature T 1  n 4  At T 2 .  .  (b) : A travelling microscope moves horizontally on a main  scale  provided  with  a  vernier  scale.  2f.35 – (–0.S.D. .  for  a  convex  lens.35  zero error = –0. = 1 M.15° = 58.38 mm.D.S.. If u =  v.  1 st  resonance  g RT1 g RT 2  ;  v 2  =  where T 1  and T 2  stand for winter  M M and summer temperatures.  the  lens  equation  is  2 nd  resonance  7. CHEMISTRY . 8)  from  the  reduction  of  boron  trichloride  by  hydrogen?  (a)  89. which of these changes?  (a)  Molality  (b)  Weight fraction  of solute  (c)  Fraction of solute present in water  (d)  Mole  fraction  (2002)  Number  of  atoms  in  558.  (a)  0.023 × 10 23  (2002)  Answer  Key  1.25 × 10  (d)  2.2 L  (c)  44.  2Al (s)  + 6HCl (aq) ® 2Al 3+  (aq)  + 6Cl –  (aq)  + 3H 2(g)  (a)  11.8 L  (d)  22.  (b)  (a)  2.6 g of elemental boron (atomic  mass  =  10. the  mass of one  mole of a substance will  (a)  decrease twice  (b)  increase two  fold  (c)  remain unchanged  (d)  be a  function of the molecular mass of the substance.6 L  (b)  67.2  L  H 2(g)  at  STP  is  produced  for  every  mole  HCl (aq)  consumed  (b)  6 L HCl (aq)  is consumed for every 3 L H 2(g)  produced  (c)  33.85 g mol –1 ) is  (a)  twice that in 60 g carbon  (b)  6.  (a)  3.75 M  (2013)  5.2  L  H 2(g)  at  STP  is  produced  for  every  mole Al  that  reacts.  of  Fe = 55.  (b)  4.  7.  4.1  Some Basic Concepts in Chemistry  SOME BASIC CONCEPTS  IN CHEMISTRY CHAPTER  1  1.  (a)  5.25 mole  of  oxygen  atoms?  7. mass of carbon atom  is taken to be the relative  atomic  mass  unit.5 M HCl with 250 mL of 2 M HCl will be  (a)  0.023 × 10 22  (c)  half that in 8 g He  (d)  558.875 M  (c)  1.  3. in place of 1/12.00 M  (d)  1.5 × 6. at 273 K and  1 atm.  Mg 3 (PO 4 ) 2  will  contain 0.975 M  (b)  0.  The  molarity  of  a  solution  obtained  by  mixing  750  mL  of  0.  wt.5 × 10 –2  (2006)  If we consider that 1/6.  (2005)  What volume of hydrogen gas.  (2007)  How  many  moles  of  magnesium  phosphate.02  (b)  3.4 L  (2003)  With increase of temperature.125 × 10 –2  –2  (c)  1.  (b)  6.6 L  H 2(g)  is  produced regardless  of  temperature  and  6. pressure  will be consumed in obtaining  21.  (c)  .  pressure for every  mole Al that reacts  (d)  67.  2.5  gram  Fe  (at.  In the reaction. 875 M  Mmix  = 2.  (a)  :  1  atomic  mass  unit  on  the  scale  of  1/6  of  C­12 = 2 amu on the  scale  of 1/12  of  C­12.2  JEE MAIN CHAPTERWISE EXPLORER  1.  –  2Al 3+  (aq )  + 6Cl (aq ) + 3H 2(g )  6 moles of HCl produced H 2  at STP = 3 ×  22.  558.25 mole  of oxygen  atoms  are  present  in  =  6.2 L  6 Mass of one atom of the element  1  1 amu (Here on the scale of   of C - 12)  6 5.125 ×  10 –2  moles of  Mg 3 (PO 4 ) 2  7.  Mass of one atom of the element  1  2 amu (Here on the scale of   of C -12)  12 \  Numerically the mass of a substance will become half  of the  normal scale.8 (c) : Volume increases with rise in temperature.  (b) : 1 mole  of Mg 3 (PO 4 ) 2 Þ  3 moles of Mg  atom +  2  moles  of P  atom  +  8  moles of O atom  8 moles of oxygen atoms are present in = 1 mole of Mg 3 (PO 4 ) 2  1 ´ 0.4 L \  1 mole of HCl will produce H 2  at  STP  3 ´ 22. of moles) = 60/12 = 5 moles.6 = 67.4 L  2 21. atomic  mass of an element  M1V1 + M2 V 2  Vmix  = 0.2 L  =  ´ 2 10.  (b) : M mixV mix  = M 1V 1  + M 2V 2  M mix  =  Now.6 g boron will require hydrogen  3 22.  4.  (a) :  2Al (s)  + 6HCl (aq )  = 3.  (atomic weight of carbon = 12)  . (b) : 2BCl 3  + 3H 2 ®  6HCl + 2B  or  3  BCl 3  +  H 2  ®  3HCl + B  2 3  10.5 ´ 750 + 2 ´ 250  1000  Mmix = 0.25  0.4 ´ 21.85 C (no.  8 = 3.5  = 10 moles  (a) : Fe (no. of moles) =  55.8 g boron requires hydrogen =  ´ 22.4  = = 11. 3  States of Matter  CHAPTER  2  STATES OF MATTER 1.  Experimentally it was found that a metal oxide has formula  M 0.98 O. Metal M, is present as M 2+ and M 3+ in its oxide. Fraction  of the metal which exists as  M 3+  would  be  (a)  5.08%  (b)  7.01%  (c)  4.08%  (d)  6.05%  (2013)  2.  For gaseous state, if most probable speed is denoted by C*,  average speed by  C  and mean square speed by C, then for  a large  number  of  molecules the ratios  of  these  speed are  (a)  C* :  C  :  C  =  1 : 1.225  : 1.128  (b)  C* :  C  :  C  =  1.225 : 1.128  : 1  (c)  C* :  C  :  C  =  1.128 : 1  : 1.225  (d)  C* :  C  :  C  =  1 : 1.128  : 1.225  (2013)  3.  4.  5.  6.  7.  8.  If  10 –4  dm 3  of  water  is  introduced  into  a  1.0  dm 3  flask  at  300  K, how  many  moles  of  water are  in  the  vapour  phase  when equilibrium is established?  (Given  :  Vapour  pressure  of  H 2 O  at  300  K  is  3170  Pa;  R = 8.314 J K –1  mol –1 )  (a)  1.27 × 10 –3  mol  (b)  5.56 × 10 –3  mol  –2  (c)  1.53 × 10  mol  (d)  4.46 × 10 –2  mol  (2010)  9.  Percentages of free space in cubic close packed structure and  in body  centred packed  structure  are  respectively  (a)  48% and 26%  (b)  30% and 26%  (c)  26% and 32%  (d)  32% and 48%  (2010)  10.  Copper crystallizes in fcc with a unit cell length of 361 pm.  What is the radius of copper atom?  (a)  108 pm  (b)  127 pm  Lithium  forms  body  centred  cubic  structure.  The  length  of  (c)  157 pm  (d)  181 pm  (2009)  the side of its unit cell is 351 pm. Atomic radius of the lithium  will be  11.  In a compound, atoms of element Y form ccp lattice and those  (a)  300 pm  (b)  240 pm  of element X  occupy 2/3 rd  of tetrahedral voids. The formula  of  the compound  will be  (c)  152 pm  (d)  75 pm  (2012)  (a)  X 3 Y 4  (b)  X 4 Y 3  The compressibility factor for a real gas at high pressure is  (c)  X 2 Y 3  (d)  X 2 Y  (2008)  (a)  1  (b)  1 +  Pb/RT  (c)  1 –  Pb/RT  (d)  1 +  RT/Pb  (2012)  12.  Equal masses of methane and oxygen are mixed in an empty  container at 25°C. The fraction of the total pressure exerted by  In a face centred cubic  lattice, atom A  occupies  the  corner  oxygen is  positions and atom B  occupies  the  face centre  positions. If  (a)  1/2  (b)  2/3  one atom of B is missing from one of the face centred points,  1 273  ´  (c)  (d)  1/3  (2007)  the  formula  of the compound  is  3 298 (a)  A 2 B  (b)  AB 2  13.  Total volume of atoms present in a face­centred cubic unit cell  (c)  A 2 B 3  (d)  A 2 B 5  (2011)  of a metal is (r is atomic radius)  20  3  24  3  ‘a’ and ‘b’ are van der Waals’ constants for gases. Chlorine  p r p r (a)  (b)  3  3  is more  easily liquefied  than ethane  because  12  3  16  3  (a) a and b for Cl 2  > a  and b for C 2 H 6  p r p r (c)  (d)  (2006)  3  3  (b) a and b for Cl 2  < a  and b for C 2 H 6  (c) a for Cl 2  < a for C 2 H 6  but b for  Cl 2  > b  for  C 2 H 6  14.  Which  one  of  the  following  statements  is  not  true  about  the  (d) a for Cl 2  > a for C 2 H 6  but b for  Cl 2  < b  for  C 2 H 6  effect  of  an  increase  in  temperature  on  the  distribution  of  molecular speeds in a gas?  (2011)  (a)  The most probable speed increases.  The edge length of a face centred cubic cell of an ionic substance  (b)  The fraction of the molecules with the most probable speed  is 508 pm. If the radius of the cation is 110 pm, the radius  increases.  of  the anion is  (c)  The distribution becomes broader.  (a)  144 pm  (b)  288 pm  (d)  The area under the distribution curve remains the same as  under the lower temperature.  (c)  398 pm  (d)  618 pm  (2010)  (2005)  4  JEE MAIN CHAPTERWISE EXPLORER  15.  An ionic compound has a unit cell consisting of A ions at the  20.  According to the kinetic theory of gases, in an ideal gas, between  corners of a cube and B ions on the centres of the faces of the  two successive collisions a gas molecule travels  cube. The empirical  formula for  this  compound would be  (a)  in a circular path  (a)  AB  (b)  A 2 B  (b)  in a wavy path  (c)  AB 3  (d)  A 3 B  (2005)  (c)  in a straight line path  (d)  with an  accelerated velocity.  16.  What type of crystal defect is indicated in the diagram below?  (2003)  Na +  Cl –  Na +  Cl –  Na +  Cl –  21.  How many unit cells are present in a cube­shaped ideal crystal  Cl –  Cl –  Na +  Na +  of NaCl of mass 1.00 g?  Na +  Cl –  Cl –  Na +  Cl –  [Atomic masses : Na = 23, Cl = 35.5]  +  –  +  –  +  Na  Cl  Na  Cl  Na  21  (a)  2.57 × 10  (b)  5.14 × 10 21  (a)  Frenkel  defect  21  (c)  1.28 × 10  (d)  1.71 × 10 21  (b)  Schottky  defect  (2003)  (c)  Interstitial  defect  (2004)  22.  Na and Mg crystallize in bcc and fcc type crystals respectively,  then the number of atoms of Na and Mg present in the unit cell  17.  In van der Waals equation of state of the gas law, the constant  of their respective crystal is  b is a measure of  (a)  4 and 2  (b)  9 and 14  (a)  intermolecular repulsions  (c)  14 and 9  (d)  2 and 4  (b)  intermolecular  attraction  (2002)  (c)  volume occupied  by  the  molecules  (d)  Frenkel and Schottky  defects  (2004)  23.  For an ideal gas, number of moles per litre in terms of its pressure  P, gas constant R and temperature T is  18.  As the temperature is raised from 20°C to 40°C, the average  (a)  PT/R  (b)  PRT  kinetic energy of neon atoms changes by a factor of which of  (c)  P/RT  (d)  RT/P  the  following?  (2002)  (a)  1/2  (b)  313/ 293  (c)  313/293  (d)  2  (2004)  24.  Kinetic theory of gases proves  (a)  only Boyle's  law  19.  A  pressure cooker reduces cooking time  for food  because  (b)  only Charles' law  (a)  heat is more evenly distributed  in the cooking space  (c)  only Avogadro's law  (b)  boiling point of water  involved in cooking is  increased  (d)  all of these.  (2002)  (c)  the  higher  pressure  inside  the  cooker  crushes  the  food  (d)  intermolecular collisions per  unit volume.  material  25.  Value of gas constant R is  (d)  cooking  involves  chemical  changes  helped  by  a  rise  in  (a)  0.082 L atm  (b)  0.987 cal mol –1  K –1  –1  –1  temperature.  (c)  8.3 J mol  K  (d)  83 erg mol –1  K –1  (2003)  Answer  Key  1.  7.  (c)  (a)  2.  8.  (d)  (a)  3.  9.  (c)  (c)  4.  (b)  10.  (b)  5.  (d)  11.  (b)  6.  (d)  12.  (d)  13.  (d)  14.  (b)  15.  (c)  16.  (b)  17.  (c)  18.  (c)  19.  (b)  25.  (c)  20.  (c)  21.  (a)  22.  (d)  23.  (c)  24.  (d)  (2002) 5  States of Matter  1.  (c)  :  Let  the fraction  of metal  which exists  as  M 3+  be  x.  Then the fraction of  metal as  M 2+  = (0.98 –  x) \  3x + 2(0.98 –  x) = 2  x + 1.96 = 2  x = 0.04 \  % of M 3+  =  2.  0.04  ´ 100  =  4.08%  0.98 2 RT = M (d) :  C * : C : C = 8 RT = p M 3 RT  M 8  = 3  3.14 = 2: (c) :  a = 351 pm  For bcc unit cell,  a 3 =  4r r= 4.  a 3 351 ´ 3  = = 152 pm  4 4 æ 5.  a ö (b) :  For real gases,  çè P + 2  ÷ø (V - b) = RT V At high pressure, P >> a/V 2  Thus neglecting  a/V 2  gives  P(V – b) =  RT  or  PV = RT  + Pb  PV RT + Pb =Z= RT RT or Þ  Z = 1  +  Pb/RT  (d) :  A            B  8 ´  1  8 5 ´  1  2 Formula of the  compound  is A 2 B 5 .  6.  (d) :  a (dm 3  atm mol –2 )  b(dm 3  mol –1 )  Cl 2  6.49  0.0562  C 2 H 6  5.49  0.0638  From the above  values,  a for  Cl 2  > a  for  ethane (C 2 H 6 )  b  for ethane  (C 2 H 6 ) >  b for  Cl 2 .  7.  (a) : In fcc lattice,  Given, a  = 508 pm  r c  = 110 pm \  110 + r a  = 8.  508 Þ  r a  =  144 pm  2 (a) : The  volume occupied  by  water  molecules  in vapour  phase is (1 – 10 –4 ) dm 3 , i.e., approximately  1 dm 3 .  PV = nRT  3170 × 1 × 10 –3  =  n H 2 O  × 8.314  × 300  nH = 2 O  (c) :  The packing efficiency in a  ccp  structure  =  74% \  Percentage free space  = 100 – 74 = 26%  Packing efficiency  in a body  centred structure  =  68%  Percentage free space  = 100 – 68 = 32%  10.  (b)  : Since Cu crystallizes in fcc  lattice, \  radius of Cu atom,  r= a  (a = edge length)  2 2  Given, a  = 361 pm  \  C * : C : C  = 1 : 1.128  : 1.225  3.  9.  3170 ´ 10 -3  = 1.27 ´ 10 -3  mol  8.314 ´ 300 \ r= 361  » 127 pm  2 2  11.  (b) : Number of Y atoms per unit cell in ccp lattice (N) = 4  Number of tetrahedral voids = 2N  = 2 × 4 = 8  Number  of tetrahedral voids  occupied  by X  =  2/3  rd  of  the  tetrahedral void = 2/3 × 8 = 16/3  Hence the formula of  the  compound will  be  X 16/3 Y 4  =  X 4 Y 3  12.  (d) :  Let the mass of methane  and oxygen be m  g.  Mole fraction  of oxygen,  x O 2  m  m  32 1  32  = ´ = =  m m  32 3m 3  +  32 16  Let the total  pressure be P. \  Partial pressure of O 2 , p O 2  =  P ×  x O 2  1 1  =  P  3 3  13.  (d)  :  In  case  of  a  face­centred  cubic  structure,  since  four  atoms are  present in a unit cell, hence volume  = P × æ4 ö 16  V = 4 ç pr 3 ÷ = pr 3  è3 ø  3  14.  (b) : Most probable velocity is defined as the speed possessed  by  maximum  number  of  molecules  of  a  gas  at  a  given  temperature. According to Maxwell's distribution curves,  as  temperature increases, most probable velocity increases and  fraction  of  molecule  possessing  most  probable  velocity  decreases.  1  15.  (c) : Number of A ions per unit cell =  ´ 8 = 1  8 1  Number of B ions per unit cell =  ´ 6 = 3  2 Empirical formula = AB 3 6  JEE MAIN CHAPTERWISE EXPLORER  16.  (b) : When an atom or ion is missing from its normal lattice site,  24.  (d) : Explanation of the Gas Laws on the basis  of Kinetic  Molecular Model  a lattice vacancy is created. This defect is known as Schottky  One of  the postulates of kinetic theory of gases is  defect.  +  –  Average K.E. µ T  Here equal number of Na  and Cl  ions are missing from their  2  regular lattice position in the crystal. So it is Schottky defect.  1  mnC 2  µ  T  1  or,  or,  mnCrms  =  kT  rms  2 2 17.  (c) : van der Waals  constant for  volume correction b  is the  2 2  Now,  PV = 1 mnCrms = 2 ´ 1 mnC rms  =  2 kT measure of the effective volume occupied by the gas molecules.  3 3 2 3  (i)  Boyle’s Law :  18.  (c) : K b  = 3/2 RT  l  Constant temperature means that the average kinetic energy  K 40 T 40  273 + 40 313  of the gas molecules remains constant.  = = = K 20 T20  273 + 20 293  l  This means that the rms velocity of the molecules,  C rms  remains unchanged.  19.  (b) : According to Gay Lussac's law, at constant pressure of a  l  If  the  rms  velocity  remains  unchanged,  but  the  volume  given  mass of a gas is directly  proportional  to the absolute  increases, this  means that there  will be  fewer  collisions  temperature  of the  gas.  Hence,  on  increasing  pressure, the  with the container walls over a given time.  temperature is also increased. Thus in pressure cooker due to  l  Therefore, the pressure will decrease  increase  in pressure  the  boiling  point  of  water  involved in  cooking is also increased.  i.e.  P µ  1  V 20.  (c) : According to the kinetic theory of gases, gas molecules are  or PV = constant.  always in rapid random motion colliding with each other and  (ii)  Charles’ Law :  with the wall of the  container  and  between  two  successive  l  An increase in temperature means an increase in the average  collisions a gas molecule travels in a straight line path.  kinetic energy of the gas molecules, thus an increase in C rms .  l  There will be more collisions per unit time, furthermore,  21.  (a) : Mass (m) = density × volume = 1.00 g  the momentum of each collision increases (molecules strike  Mol. wt. (M) of NaCl = 23 + 35.5 = 58.5  the wall harder).  l  Number of unit cell present in a cube shaped crystal of NaCl of  Therefore, there will be an increase in pressure.  l  If we allow  the volume to change to maintain constant  r ´ a 3 ´ N A m ´ N A  mass 1.00 g =  = pressure,  the  volume  will  increase  with  increasing  M ´Z M ´ Z temperature (Charles law).  23  (iii)  Avogadro’s Law  =  1 ´ 6.023 ´ 10  58.5 ´ 4 It states that under similar conditions of pressure and  (In NaCl each unit cell has 4 NaCl units. Hence Z = 4). temperature, equal volume of all gases contain equal number  \  Number of unit cells = 0.02573 × 10 23  of molecules. Considering two gases, we have  = 2.57 × 10 21  unit cells  2 2  PV 1 1 = kT1 and  PV 2 2 =  kT2  3 3  22.  (d) : bcc ­ Points are at corners and one in the centre of the unit  Since P 1  = P 2  and  T 1  =  T 2 , therefore  cell.  1  Number of atoms per unit cell =  8 ´ + 1 =  2  8 fcc ­ Points are at the corners and also centre of the six faces  of  each cell.  1 1  Number of atoms per unit cell =  8 ´ + 6 ´ =  4  8 23.  (c) : From ideal gas equation, PV = nRT \  n/V = P/RT (number of moles = n/V)  2 PV (2 / 3) kT1 V n  1 1 = Þ 1 =  1  P2V2 (2 / 3) kT2 V2 n2  If volumes are identical, obviously n 1  =  n 2 .  25.  (c) : Units of R  (i)  in L atm Þ 0.082 L atm mol –1  K –1  (ii)  in C.G.S. system Þ 8.314 × 10 7  erg mol –1  K –1  (iii)  in M.K.S. system Þ 8.314 J mol –1  K –1  (iv)  in calories Þ 1.987 cal mol –1  K –1 7  Atomic Structure  CHAPTER  3  1.  ATOMIC STRUCTURE Energy of an electron is given by  E = -2.178 ´ 10 -18  æ Z 2 ö J ç 2  ÷ .  è n ø  Wavelength  of  light  required  to  excite  an  electron  in  an  8.  hydrogen atom from level  n  = 1  to n  = 2  will be  (h  = 6.62 × 10 –34  J s and  c  = 3.0 × 10 8  m s –1 )  (a)  8.500 × 10 –7  m  (b)  1.214 × 10 –7  m  (c)  2.816 × 10 –7  m  (d)  6.500 × 10 –7  m  (2013)  2.  3.  4.  5.  The electrons identified by  quantum  numbers  n and  l:  (1)  n = 4, l  = 1  (2)  n = 4, l  =  0  (3)  n = 3, l  = 2  (4)  n = 3, l  =  1  9.  can be placed in order of  increasing  energy  as  (a)  (4) < (2) < (3) < (1)  (b)  (2) < (4) < (1) < (3)  (c)  (1) < (3) < (2) < (4)  (d)  (3) < (4) < (2) < (1)  (2012)  A gas absorbs a photon of 355 nm and emits at two wavelengths.  If one of the emission  is at  680  nm,  the  other is at  (a)  1035 nm  (b)  325 nm  (c)  743 nm  (d)  518 nm  (2011)  (c)  – 4.41 × 10 –17  J atom –1  (d)  – 2.2 × 10 –15  J  atom –1  7.  (2009)  The  ionization  enthalpy  of  hydrogen  atom  is  1.312 × 10 6 J mol –1 . The energy required to excite the electron  in the atom from  n = 1 to  n = 2 is  (a)  9.84 × 10 5  J mol –1  (b)  8.51 × 10 5  J mol –1  (c)  6.56 × 10 5  J mol –1  (d)  7.56 × 10 5  J mol –1  (2008)  Which of the following sets of quantum numbers represents the  highest energy of an  atom?  1  (a)  n = 3, l = 0, m = 0, s =  +  2 1  (b)  n = 3, l = 1, m = 1, s =  +  2 1  (c)  n = 3, l = 2, m = 1, s =  +  2 1  (d)  n = 4, l = 0, m = 0, s =  +  2 (2007)  The  energy required  to break  one  mole of  Cl—Cl bonds in  Cl 2  is 242 kJ mol –1 . The longest wavelength of light capable  10.  Uncertainty  in  the  position  of  an  electron  (mass = 9.1 × 10 –31 kg) moving with a velocity 300 m s –1 , accurate  of  breaking  a  single  Cl—Cl  bond  is  (c  =  3  ×  10 8  m  s –1  upto 0.001% will be (h = 6.6 × 10 –34  J s)  and N A  = 6.02 × 10 23  mol –1 )  (a)  19.2 × 10 –2  m  (b)  5.76 × 10 –2  m  (a)  494 nm  (b)  594 nm  (c)  1.92 × 10 –2  m  (d)  3.84 × 10 –2  m  (c)  640 nm  (d)  700 nm  (2010)  (2006)  Ionisation energy of He +  is 19.6 × 10 –18  J atom –1 . The energy  of the first stationary  state (n  = 1)  of Li 2+  is  11.  According  to  Bohr’s  theory,  the  angular  momentum  of  an  –17  –1  electron in 5 th  orbit is  (a)  8.82 × 10  J atom  (b)  4.41 × 10 –16  J atom –1  6.  (a)  1.52 × 10 –4  m  (b)  5.10 × 10 –3  m  (c)  1.92 × 10 –3  m  (d)  3.84 × 10 –3  m  (2010)  h (a)  25  p  h (b)  1.0  p  h (c)  10  p  h (d)  2.5  p  (2006)  Calculate the  wavelength (in  nanometre) associated with a  proton moving at 1.0  × 10 3  m s –1 .  12.  Which of the following statements in relation to the hydrogen  (Mass of proton = 1.67 × 10 –27  kg and h = 6.63 × 10 –34  J s)  atom is correct?  (a)  0.032 nm  (b)  0.40 nm  (a)  3s orbital is lower in energy than 3p orbital.  (c)  2.5 nm  (d)  14.0 nm  (2009)  (b)  3p orbital is lower in energy than 3d orbital.  In an atom, an electron is  moving with  a speed of 600 m/s  (c)  3s and 3p orbitals are of lower energy than 3d orbital.  with an accuracy of 0.005%. Certainty with which the position  (d)  3s, 3p and 3d orbitals all have the same energy.  of the electron can be located is (h = 6.6 × 10 –34  kg m 2  s –1 ,  (2005)  mass of electron, e m  = 9.1 × 10 –31  kg)  8  JEE MAIN CHAPTERWISE EXPLORER  13.  In  a  multi­electron  atom,  which  of  the  following  orbitals  17.  The orbital angular momentum for an electron revolving in an  described  by the  three  quantum  numbers  will  have  the  same  h  orbit  is  given  by  l( l + 1)  × .  This  momentum  for  an  energy in the absence of  magnetic  and electric  fields?  2 p  (i)  n = 1, l = 0, m = 0  s­electron will be given by  (ii)  n = 2, l = 0, m = 0  1  h  (iii)  n = 2, l = 1, m = 1  (a)  + × (b)  zero  2 2 p  (iv)  n = 3, l = 2, m = 1  h  h  (v)  n = 3, l = 2, m = 0  (d)  2  × (c)  2 p  2 p  (a)  (i) and (ii)  (b)  (ii) and (iii)  (2003)  (c)  (iii) and (iv)  (d)  (iv) and (v)  (2005)  18.  The de Broglie wavelength of a tennis ball of mass 60 g moving  with a  velocity of 10 metres per second is approximately  14.  The wavelength of the radiation emitted, when in a hydrogen  (Planck's constant, h = 6.63 × 10 –34  J s)  atom electron falls from infinity to stationary state 1, would be  –33  (a)  10  metres  (b)  10 –31  metres  (Rydberg constant = 1.097 × 10 7  m –1 )  –16  (c)  10  metres  (d)  10 –25  metres.  (a)  91 nm  (b)  192 nm  (2003)  (c)  406 nm  (d)  9.1 × 10 –8  nm  (2004)  19.  In Bohr series of lines of hydrogen spectrum, the third line from  the  red end  corresponds to  which  one of  the following  inter­  15.  Consider the ground state of Cr atom (Z = 24). The numbers of  orbit  jumps  of  the  electron  for  Bohr  orbits  in  an  atom  of  electrons with the azimuthal quantum numbers, l = 1 and 2 are,  hydrogen?  respectively  (a)  3 ® 2  (b)  5 ® 2  (a)  12 and 4  (b)  12 and 5  (c)  4 ® 1  (d)  2 ® 5  (c)  16 and 4  (d)  16 and 5  (2003)  (2004)  16.  Which of the following sets of quantum numbers is correct for  20.  Uncertainty in position of a  minute particle of mass  25  g  in  space  is  10 –5  m.  What  is  the  uncertainty  in  its  velocity  an electron in 4f  orbital?  (in m s –1 )?  (h = 6.6 × 10 –34  J s)  1  (a)  n = 4, l = 3, m = +4, s =  +  –34  (a)  2.1 × 10  (b)  0.5 × 10 –34  2 (c)  2.1 × 10 –28  (d)  0.5 × 10 –23  (2002)  1  (b)  n = 4, l = 4, m = –4, s =  -  2 21.  In a hydrogen atom, if energy of an electron in ground state is  1  13.6 eV, then that in the 2 nd  excited state is  (c)  n = 4, l = 3, m = +1, s =  +  2 (a)  1.51 eV  (b)  3.4 eV  1  (c)  6.04 eV  (d)  13.6 eV  (d)  n = 3, l = 2, m = –2, s =  +  (2004)  (2002)  2 Answer  Key  1.  (b)  7.  (c)  13.  (d)  19.  (b)  2.  (a)  8.  (a)  14.  (a)  20.  (c)  3.  (c)  9.  (c)  15.  (b)  21.  (a) 4.  (a)  10.  (c)  16.  (c)  5.  (c)  11.  (d)  17.  (b)  6.  (b)  12.  (d)  18.  (a)  9  Atomic Structure  1.  æ 1  ö -18 2  1 (b) : E =  -2.178 ´ 10  Z  ç 2 - 2 ÷ è n1 n2  ø  -18  E = -2.178 ´ 10  é 1 1  ù ê 2 - 2  ú (1)  û  ú ëê (2) E = +2.178 ´ 10 -18 ´ E= Þ  Þ \ 5.  2.  3.  hc  6.62 ´ 10 -34  J s ´ 3 ´ 10 8 m  = E 1.6335 ´ 10 -18 J  l = 1.214 × 10 –7  m  6.  l l1 Þ 1 1 1  = + 355 680 l 2  l 2 h  mv  Given, v  = 1.0 × 10 3  m s –1  355 ´ 680  = 742.769 nm » 743 nm  680 - 355 Energy required  to  break  1  bond  = We know that, E = hc l  Given, c = 3 × 10 8  m s –1  6.63 ´ 10 -34  = 3.9 ´ 10 -10  m  1.67 ´ 10 -27 ´ 1.0 ´ 103  l »  0.4 nm  \ l= or 7.  (c) :  Given, velocity of  e – ,  v  = 600 m s –1  Accuracy  of  velocity  =  0.005%  600 ´ 0.005  \ Dv = =  0.03  100 According to Heisenberg’s uncertainty principle,  h  Dx × mDv ³ 4p  6.6 ´ 10 -34  Þ Dx = 4 ´ 3.14 ´ 9.1 ´ 10 -31  ´ 0.03 8.  (a) : Energy required to break 1 mol of bonds = 242 kJ mol –1 \  (b) : According to de­Broglie’s  equation, = 1.92 × 10 –3  m  E = E 1  + E 2  or  hc = hc + hc 4.  -19.6 ´ 10 -18  ´ 9  = - 44.1 ´ 10 -18 J atom -1  4 l =  E =  hu = hc/l  \ l 2  = 242 ´ 10 3  = – 4.41 × 10 –17  J atom –1  (c) : We know that  1 1 1 = + l l1 l 2 6.63 ´ 10 -34 ´ 3 ´ 108 ´ 6.02 ´ 10 23  (c) :  I.E.(He + ) = 19.6 × 10 –18  J atom –1  = (a) :  (1)  n = 4, l  = 1 Þ  4p  (2)  n = 4, l  = 0 Þ  4s  (3)  n = 3, l  = 2 Þ  3d  (4)  n = 3,  l = 1 Þ  3p  Increasing order of  energy is 3p  <  4s <  3d  <  4p  (4)  < (2) < (3)  < (1)  Alternatively,  for  (1)  n +  l =  5 ;  n =  4  (2)  n +  l =  4 ;  n =  4  (3)  n + l =  5 ;  n  =  3  (4)  n + l =  4 ;  n  =  3  Lower  n  +  l  means  less  energy  and  if  for  two  subshells  n + l is same than lower  n,  lower will be the energy.  Thus correct order is (4) <  (2) < (3) <  (1).  Þ l= 6.63 ´ 10 -34 ´ 3 ´ 10 8  l E 1  (for H) ×  Z 2  =  I.E.  E 1  × 4 = –19.6 × 10 –18  E 1  (for Li 2+ ) =  E 1  (for H) × 9  l  = 12.14 × 10 –8  m  or 6.02 ´ 10 23  = = 0.494 × 10 –6  m = 494 nm  3  = 1.6335 ´ 10 -18  J  4  hc  l l= 242 ´ 10 3 242 ´ 10 3  6.02 ´ 10 23  J  (a) : The ionisation of H­atom is the energy absorbed when  the  electron in an atom gets excited from first  shell (E 1 )  to  infinity (i.e.,  E¥ )  I.E = E¥  –  E 1  1.312 × 10 6  = 0 –  E 1  E 1  = –1.312 × 10 6  J mol –1  6 6  E2  = - 1.312 ´2 10 = - 1.312 ´ 10  4  (2)  Energy of electron in second  orbit (n =  2) \  Energy required when an electron makes transition from  n = 1 to n = 2 6  DE =  E 2  –  E 1  = - 1.312 ´ 10  - ( -1.312 ´ 106 )  4 105 × 10 –33  metres. Thus option (b) is  = 2. total number of electrons = 12  [2p 6  and 3p 6 ]  For l  = 2..14 ´ 9.  (c) :  n = 3.1  ÷ö 2 2  l l è1 ¥ ø è n1 n2  ø \  D v = 2.6 ´ 10 -34  Dv = æ ö 4 ´ 3.  (d) :  For  hydrogen  the  energy  order  of  orbital  is  20.  E n  =  13.1 ´ 10 -28 m s -1  \ l = 91 × 10 –9  m = 91 nm  21.. 5.5  \  Angular momentum =  2 p p  correct  here.6  eV  or  E = 13.312 ´ 10 + 5.63 ´ 10 -34  \  Dx = 4 ´ 3. The third line from the  red end  2 p corresponds to yellow region  i. In order to obtain  less  when n = 5 (given) energy  electron  tends  to  come  in  1 st  or  2 nd  orbit. l = 2 represents 3d orbital  n = 4.  15.984 × 10  4 DE  = 9.92 ´ 10 -2  m 11.14 ´ 25 ´ 10 -5  (a) :  1 = R çç 12 .  mvr =  of orbit.  6. –1.05 × 10 –34  m = 1.  h  Dx ´ Dp = 4 p  h  h  Dx × (m × Dv ) = Þ Dx = 4 p 4 pm × D v 0. l = 1 represents 3p orbital  n = 3.1 2 ÷÷ Þ  1 = 1. l = 0 represents 4s orbital  The order of increasing  energy  of  the orbitals  is  3s <  3p <  4s <  3d.  (b) : The electron has minimum energy in the first orbit and  its energy increases as n increases.  (a) :  l = h  = 6.248 ´ 10  = 0.  (b) : The value of l (azimuthal quantum number) for s ­electron  is equal to zero.  (a) :  2 nd  excited state  will be  the  3rd energy level.  (b) :  24 Cr ® 1s 2  2s 2  2p 6  3s 2  3p 6  3d 5  4s 1  we know for p. 2 nd .  For l  = 1.  So  jump  5 h h may be involved either 5 ® 1 or 5 ® 2. total number of electrons = 5  16.84 ×10 5  J mol –1  9.  (c) :  According to  Heisenberg’s  uncertainty  principle.10  JEE MAIN CHAPTERWISE EXPLORER  6 6  6 =  -1.  1s  < 2s = 2p  < 3s  = 3p = 3d <  4s = 4p = 4d  =  4f  h  Dx × mDv = (d) :  Orbitals having  same  (n  +  l)  value  in  the  absence of  4 p  electric and magnetic  field  will  have  same  energy.1 ´ 10 -31 ´ 3 ´ 10 -3  = 1.63 ´ 10 ´ 1000  m  mv 60 ´ 10  = 11.  13. l = 0 represents 3s orbital  n = 3.  19. 0.  14.  10.e.6 = 1.  h  Orbital angular momentum =  l (l + 1) × 2 p  Substituting  the  value  of  l  for  s­electron  =  0(0 + 1) × h = 0  2 p  -34  18.  (c) :  For 4 f  orbital electrons. –3  s = ±1/2  17. +1.001  -3 ´ 300 = 3 ´ 10  m s -1  Here  Dv = 100  6.  (c)  :  According  to  Heisenberg  uncertainty  principle. +2.  n = 4  [3d 5 ]  s  p  d  f  l = 3 (because  0  1  2 3  )  m = +3.  12. 1 st .e. 3 rd  .  i.097 ´ 107 m -1 çæ 1 . –2.  l = 1 and for  d..51 eV  n 2  9  .  l  =  2. Here n represents number  nh  (d) :  Angular momentum  of  the  electron.  O 2 – . CO  C 2 2– . C 22–    .  The charge/size ratio of a cation determines its polarizing power.  sp 2 . Ca 2+ . NO + .  Which  of  the  following  species  exhibits  the  diamagnetic  are  respectively  behaviour?  2–  (a)  sp. Li    2  and Li 2  increases in the order  of  (a)  Li 2–   < Li 2  < Li +  2  (b)  Li 2  < Li + 2  < Li 2–   (c)  Li 2 –  < Li +  2  < Li 2  11. NO  –  NO + .  He 2 2+  (b)  H 2 + .  6. is present in  (a)  O 2+  (b) O + 2  2  (a)  benzene and  ethanol  (c)  O – 2  (d)  O 2–  (2009)  2  (b)  acetonitrile and  acetone  10. CN  . N 2  2–  CN – . sp 3 .11  Chemical Bonding and Molecular Structure  CHAPTER  4  1.  +  –  Stability of the species Li 2.  (2011)  (d)  K +  < Ca 2+  < Mg 2+  < Be 2+  (2007)  –  +  +  The hybridisation of orbitals of N atom in NO 3 .  7.  3  5. C 2 2–  Which one of the following pairs of species have  the  same  bond  order?  (a)  NO +  and CN +  (b)  CN –  and NO +  (c)  CN –  and CN +  (d)  O 2 –  and CN –  (2008)  (d)  H 2 2+ . O 2  .  sp 3  (a)  NO  (b)  O 2  +  (c)  sp. the bond order  has increased and the magnetic behaviour has changed?  The molecule  having smallest  bond angle  is  (a)  N 2 ® N 2+   (b)  C 2 ® C 2+   (a)  AsCl 3  (b)  SbCl 3  (c)  NO ® NO +  (d)  O 2 ® O 2 +  (2007)  (c)  PCl  (d)  NCl  (2012)  (c)  H 2 – . Be 2+ ?  (c)  AlF 6 3–  and SF 6  (d)  CO 3 2–  and NO 3 –  (2012)  (a)  Ca 2+  < Mg 2+  < Be +  < K +  The  structure of IF 7  is  (b)  Mg 2+  < Be 2+  < K +  < Ca 2+  (a)  square pyramid  (b)  trigonal bipyramid  (c)  Be 2+  < K +  < Ca 2+  < Mg 2+  (c)  octahedral  (d)  pentagonal bipyramid. sp.  dipole­dipole  as  the  major  the  shortest  bond length?  interaction. K + .  He 2 2–  4.  Which one of the following sequences represents the increasing  isostructural?  +  order of the polarizing power of the cationic species.  9.  sp  (2011)  (c)  O  (d)  O  (2007)  2  8. sp 2  (d)  sp 2 . both the  species are  not likely  to exist?  (a)  H 2 – .  Which of the following hydrogen bonds is the strongest?  (a)  O — H ­ ­ ­ F  (b)  O — H ­ ­ ­ H  Which one of the following molecules is expected to exhibit  (c)  F — H ­ ­ ­ F  diamagnetic  behaviour?  (d)  O — H ­ ­ ­ O  (2007)  (a)  S 2  (b)  C 2  (c)  N 2  (d)  O 2  (2013)  13.  Which  one  of  the  following  constitutes  a  group  of  the  isoelectronic  species?  (c)  KCl and water  (d)  benzene and  carbon tetrachloride.  In which of the following ionization processes.  Among  the  following  mixtures.  (a)  PCl 4  and SiCl 4  (b)  PF 5  and BrF 5  Mg 2+ . NO 2  and NH 4  15.  (2008)  12.  (2006)  .  In which of the following molecules/ions are all the bonds not  by  the  compound  equal?  (a)  FeCl 2  (b)  SnCl 2  (a)  SF 4  (b)  SiF 4  (c)  AlCl 3  (d)  MgCl 2  (2011)  (c)  XeF 4  (d)  BF 4–   (2006)  Using MO theory predict which of the following species has  17.  (a)  (b)  (c)  (d)  N 2 . sp 3  (b)  sp 2 .  CHEMICAL BONDING AND  MOLECULAR STRUCTURE 2  Among the following the maximum covalent character is shown  16. He 2 2–  3.  He 2  (2013)  3  In  which  of  the  following  pairs  the  two  species  are  not  14. N 2 .  sp 3 . CO. O 2–  .  2.  (d)  Li 2  < Li 2 –  < Li +  (2013)  2  In which of the  following pairs  of molecules/ions.   29.  15. d x 2  – y 2  Which one of the following has the regular tetrahedral structure?  (c)  s.  Number of sigma bonds in P 4 O 10  is  (a)  6  (b)  7  (2004)  (c)  17  (d)  16  The bond order in NO is 2.  22. p y  . d yz  (b)  s. p y  .  20.  In which of the following species is the underlined carbon having  (a)  Bond length in NO +  is greater than in NO.12  JEE MAIN CHAPTERWISE EXPLORER  18. ClO 4 –  (b)  CN – . d z2   (d)  s.  28. d xy  (a)  XeF 4  (b)  SF 4  (2002)  (c)  BF 4–   (d)  [Ni(CN) 4 ] 2–  (Atomic nos.: B =  5.  (d)  4.  16. p x  .  23. S =  16. IF 5  (2003)  19.  30.  (b)  (c)  (c)  (b)  (a) 8. p z  . p y  . BF 3  –1  (c)  NH 3. PCl 3  (d)  PF 5 .  26. Ni =  28. NH . NO 3  3–  2–  (d)  BO 3  .  Of the following sets which one does NOT contain isoelectronic  species?  (a)  PO 4 3– . SF 4  (b)  XeF 2 .  (a)  (b)  (d)  (b)  10.  27.  24.  A square planar complex is formed by hybridisation of which  (c)  dsp 2  hybridisation  atomic orbitals?  (d)  sp 3 d 2  hybridisation.  (c)  (a)  (c)  (b)  .  The  pair  of  species  having  identical  shapes  for  molecules  of  electrons?  both species is  (a)  O 22–   (b)  B 2  (a)  CF 4 .  (b)  (b)  (b)  (d)  12.  24.  (b)  5.  (d)  3.  (c)  2.  Which one of the following species is diamagnetic in nature?  (a)  He 2 +  (b)  H 2  (c)  H 2+  (d)  H 2–   (2005)    21.5 while that in NO +  is 3. (BF  (b)  (NH 4)    4)      .  Which of the following are arranged in an increasing order of  their bond strengths?  (a)  O 2–   < O 2  < O 2+   < O 22–    –  +  (b)  O 22–    < O 2  < O 2  < O 2  +  The maximum number of 90° angles between bond pair­bond  (c)  O 2–   < O 22–    < O 2  < O 2  +  –  2–  pair of electrons is observed  in  (d)  O 2  < O 2  < O 2  < O 2  (a)  dsp 3  hybridisation  (2002)  (b)  sp 3 d hybridisation  28.  (a)  CH 3 COOH  (b)  CH 3  CH 2  OH  (d)  Bond length is unpredictable. BF  (d)  (NH 2)  (2002)    3    . Which of  (2002)  the following statements is true  for these  two species?  30.  20.  31.  17.  (2004)  (d)  CH 2  CH – CH 3  (c)  CH 3 COCH 3  (2002)  The correct order of bond angles (smallest first) in H S.  (c)  (d)  (b)  (c)  9.  Which of the following molecules/ions does not contain unpaired  25.  3  (2004)  31.  (c)  (a)  (c)  (b)  11.  19.  14.  25. C 2 2–  2–  –  (c)  SO 32–    . CO 3  . N 2 . Xe = 54)  29.  (2004)  (a)  s.  2  BF 3  and SiH 4  is  (a)  H 2 S < SiH 4  < NH 3  < BF 3  (b)  NH 3  < H 2 S < SiH 4  < BF 3  (c)  H 2 S < NH 3  < SiH 4  < BF 3  (d)  H 2 S < NH 3  < BF 3  < SiH 4 . BF 3  Answer  Key  1.  (b)  6.  (d)  7.  sp 3  hybridisation?  +  (c)  Bond length in NO  is equal to that in NO. p y  . p x  . SO 4 2– .  Which one of the following compounds has the smallest bond  angle in its molecule?  (a)  SO 2  (b)  OH 2  (c)  SH 2  (d)  NH 3  (2003)  27. CO 2  (c)  N 2+   (d)  O 2  (2006)  (c)  BF 3 .  +  (b)  Bond length in NO is greater than in NO  . CO 3  .  18.  22. NO 3–   (2005)  26.  13. p x  .  In which of the following species the interatomic bond angle is  109°28¢?  –1  +  (a)  NH 3.  23.  21. = = (p 2py ) 2 (p * 2px ) 1  10 . extent of polarisation µ covalent character  (a)  :  According  to  MOT.O.  So  it  has  more  covalent  character.  it is paramagnetic. CO 3 2–  and NO 3 –  both are  trigonal planar.4  = 3  2 +  O2  : (s1s) 2 (s * 1s) 2 (s2s) 2 (s * 2s) 2 (s2pz) 2 (p 2px ) 2  \ B. Hence option  (c) is correct.  4. N 2 and CO are isoelectronic in nature.5  2 –  O2  : (s1s) 2 (s * 1s) 2 (s2s) 2 (s * 2s) 2 (s2pz) 2 (p 2px ) 2  = (p 2py) 2 (p * 2px) 2  = (p * 2py ) 1  \ B.  in ionic  bond.  \ B.  (c) :  Species  Bond  order  Li 2  1  –  Li 2  0.2  = 0  2 +  4  (c) : We know that.  11.O. .  (d) : The  structure is  pentagonal bipyramidal  having sp 3 d 3  hybridisation as given  below:  1  size of cation  Here the AlCl 3  is satisfying the above two conditions i.  lone  pair­bond  pair  repulsion also increases.  (c) :  Number of electrons in each  species are given  below  N 2  = 14  CN –  = 14  –  O 2  = 17  C 22–    = 14  +  NO  = 14  O 22–    = 18  CO = 14  NO = 15  It  is  quite  evident  from  the  above  that  NO + .e.  Bond length  2+  \  O2  has the shortest bond length.  the  molecular  orbital  electronic  configuration  of  O2 2+  : (s 1s) 2 (s * 1s) 2 (s 2s) 2 (s * 2s) 2 (s 2pz) 2 (p 2px) 2  = (p 2py ) 2  10 . = 7.  6.  (b) : As we move down the group the size of atom increases  and  as  size  of  central  atom  increases.  (b) : The structures of  NO 3 – . CN  .  Fajan’s rule states that  (i) the polarising power of cation increases.O.. = 10 .  greater is the stability.5  Higher the bond order.O.5  =  2.  (b) : In the given pair of species. number of electron in NO +  = number of electron in CN –  =  14 electrons.  –  C 22–    . µ .5  2 O2 2–  : (s 1s) 2 (s * 1s) 2 (s 2s) 2 (s * 2s) 2 (s 2pz) 2 (p2px ) 2  = (p 2py) 2 (p * 2px) 2  = (p * 2py ) 2  \ B.  Increasing order of  atomic  radius  :  N < P < As < Sb  Decreasing order of  bond  angle  :  NCl 3  > PCl 3  > AsCl 3  > SbCl 3  5.O.7  = 1. = 10 . Li 2+   0.5  8.  2.  (b) : PCl  and SiCl 4 Þ  both tetrahedral  PF 5 Þ  trigonal bipyramidal  BrF 5 Þ square pyramidal  AlF 6 3–  and SF 6  both are octahedral.8  = 1.  2 . Al  is  in  +3  oxidation  state  and also  has  small  size.  (d) : O 2  is expected to be diamagnetic in nature but actually  9. Thus  bond  angle decreases.  So  they are  isoelectronic in  nature.  H 2 2+  : s(1s) 0 \  Bond order =  0  He 2  : s(1s) 2 s*(1s) 2  Bond order  =  \  polarising power µ  3. NO 2 +  and  NH 4 +  is  sp 2  hybridisation  sp  hybridisation  sp 3  hybridisation  10. with increase in  magnitude  of  positive  charge  on  the cation \  polarising power µ  charge of cation  (ii) the polarising power of cation increases with the decrease  in the size of a cation.13  Chemical Bonding and Molecular Structure  1.0  2 1  Q B.  (d) : Species with  zero bond  order  does  not exist.   The  positive pole of one molecule is thus attracted by the negative  pole of the other molecule. polarising power decreases as  KK s(2s) 2 s*(2s) 2 s(2p z ) 2 p(2p x ) 2 p(2p y ) 2 s(2p z ) 1  K +  < Ca 2+  < Mg 2+  <  Be 2+  It contains  one  unpaired electron.5  =  2. which in turn depends upon  the electronegativities of the atoms present in the molecule  and  the  geometry  of  the  molecule  (in  case  of  polyatomic  molecules.O = 1/2 [N b  –  N a ] =  1/2 [10  –   4]  or  B.  hydrogen  bonding in F — H ­ ­ ­ F is strongest.6  =  2  2 + 109.4  Bond order  =   =  3  2 + N 2  Þ s1s 2 s*1s 2 s2s 2 s*2s 2 p2p x 2 p2p y 2 p2p z 1 Þ  paramagnetic  9 . The magnitude of dipole­dipole  forces in different polar molecules is predicted on the basis  of the polarity of the molecules.5  2 NO Þ s1s 2 s*1s 2 s2s 2 s*2s 2 s2p z 2 p2p x 2 p2p y 2 p*2p x 1 Þ  paramagnetic  Bond order  =   10 .  17.  (c) : Molecular  orbital  configuration  of  O 2 Þ s1s 2 s*1s 2 s2s 2 s*2s 2 s2p z 2 p2p x 2 p2p y 2 p*2p x 1 p*2p y 1 Þ  paramagnetic  F  16.  K +  > Ca 2+  > Mg 2+  > Be 2+  The  molecular  orbital configuration  of  N +  2  ion  is  Hence.  .O = 3  CN – ® s1s 2 s*1s 2 s2s 2 s*2s 2 p2p x2  p2p y2  s2p z 2  B.4  Bond order  =   =  2.  NO + ® s1s 2 s*1s 2 s2s 2 s*2s 2 s2p z 2 p2p x 2 p2p y 2  B.  1 1  KK s(2s) 2 s*(2s) 2 p(2p x)    p(2p y )  As the size of the given cations decreases  as  It contains 2 unpaired electrons.5°  O 2  Þ s1s 2 s*1s 2 s2s 2 s*2s 2 s2p z 2 p2p x 2 p2p y 2 p*2p x 1 Si  Þ paramagnetic  Bond order  =   10 .5  2 NO + Þ s1s 2 s*1s 2 s2s 2 s*2s 2 s2p z 2 p2p x 2 p2p y 2 Þ  diamagnetic  F  F  F  XeF 4 : sp 3 d 2  hybridisation.  The  molecular orbital  configuration of  O 2  molecule  is  15. Þ  paramagnetic  .14  JEE MAIN CHAPTERWISE EXPLORER  Hence bond order of  these  two  species  will  be  also  similar which is shown below.O  =1/2 [10  – 4]   or  B.  (d)  :  High  charge  and  small  size  of  the  cations  increases  The  molecular  orbital configuration  of  B 2  molecule  is  polarisation.  18.  Polar  molecules  have  permanent  dipoles.  SiF 4  :  sp 3  hybridisation and tetrahedral geometry.  (a) :  The  molecular  orbital configuration  of O 2 2–  ion  is  KK s(2s) 2 s*(2s) 2 s(2p z ) 2 p(2p x ) 2 p(2p y ) 2 p*(2p x ) 2 p*(2p y ) 2  10 4  Bond order  =   =  3  Here KK represents non­bonding molecular orbital of 1s orbital.  Þ  paramagnetic  2–  O 2 Þ s1s 2 s*1s 2 s2s 2 s*2s 2 s2p z 2 p2p x 2 p2p y 2 p*2p x 1 p*2p y 1 19.  14.5  =  2.  S  F  F  SF 4  molecule shows sp 3 d hybridisation but its expected trigonal  bipyramidal  geometry  gets  distorted  due  to  presence  of  a  lone  pair of  electrons and  it  becomes distorted  tetrahedral  or see­saw with the bond angles equal to 89° and 177° instead  of  the  expected  angles  of 90° and  180°  respectively.4  =  1. shape is square planar instead of  octahedral due to presence of two lone pair of electrons on  Xe atom.  2 O 22–    contains  no unpaired  electrons.  (b)  :  Dipole­dipole  interactions  occur  among  the  polar  molecules.  13.  (b) :  Molecular  orbital  configuration  is  KK s(2s) 2 s*(2s) 2 p(2p x ) 2 p(2p y ) 2 s(2p z ) 2 p*(2p x ) 1 p*(2p y ) 1  NO Þ s1s 2 s*1s 2 s2s 2 s*2s 2 s2p z 2 p2p x 2 p2p y 2 p*2p x 1 It contains 2 unpaired electrons.5  2 C 2 Þ s1s 2 s*1s 2 s2s 2 s*2s 2 p2p x 2 p2p y 2 Þ  diamagnetic  Bond order  =   8 .  F  Bond order  =   10 .5  2 N 2 Þ s1s 2 s*1s 2 s2s 2 s*2s 2 p2p x 2 p2p y 2 p2p Z 2 Þ paramagnetic  10 .  (a) :  F  .  F  F  Xe  F  BF 4 –  F  :  sp 3  hybridisation and tetrahedral  geometry. containing more than two atoms in a molecule).  (c) :  Because of  highest electronegativity  of  F.4  =  2  2 C 2+  Þ s1s 2 s*1s 2 s2s 2 s*2s 2 p2p x 2 p2p y 1 Þ  paramagnetic  Bond order  =   7 .O =  3  12.  (c) :  Number of electrons  in SO 3  = 16 + 8 × 3 + 2 = 42  Þ  paramagnetic  Number  of  electrons  in  CO 3 2– = 6 + 8 × 3 + 2=  32  O 22–  Þ s1s 2 s*1s 2 s2s 2 s*2s 2 s2p z 2 p2p x 2 p2p y 2 p*2p x 2 p*2p y 2 Number of electrons in NO 3 –  = 7  + 8 × 3 + 1=  32  Þ  diamagnetic  These are  not isoelectronic species as number  of electrons  O 2+  Þ s1s 2 s*1s 2 s2s 2 s*2s 2 s2p z 2 p2p x 2 p2p y 2 p*2p x 1 are not same. 5°  Due to absence of unpaired electrons.  (b) : Higher the bond order.5  2 F  P O (d) :  O P O O O P O O O  No.  (c) :  The  correct order  of  bond  angle  (smallest  first) is  H 2 S  <  NH 3  <    SiH 4  <   BF 3  and (d) (CH 2  CH — CH 3 ).  . of s  bonds  = 16  No.15  Chemical Bonding and Molecular Structure  20.  27. The expected bond angle  should be 109º28¢ but actual bond angle is less than  109º28¢  because of the repulsion between lone pair and  bonded  pairs  due to which contraction occurs.8  = 1  2 Hence  increasing order  of  bond order  is  O 2 2–  < O 2 –  < O 2  < O 2 +  F  regular tetrahedral  10 . = 10 .5  2 2 2 O 2–  Þ s(1s) 2s*(1s) 2s(2s) 2s*(2s) 2s(2p z)    p(2p x )  p(2p y ) 2p*(2p x ) 2p*(2p y ) 1 ;  B.  (b) :  He 2 + ® s(1s) 2 s*(1s) 1 . the carbon atom has a multiple  92.7  = 1.  (b)  :  Molecular  orbital  configuration  of  21.  F  P O Xe  F  F  28.  (b) : In molecules (a)  CH 3  C  OH  .  (d) :  O 2 Þ s(1s) 2s*(1s) 2s(2s) 2s*(2s) 2s(2p z ) 2p(2p x ) 2 1 1  p(2p y ) 2p*(2p x)    p*(2p y)    ;  B.  no  unpaired  electron  shape.5  = 2. B.  one  unpaired  electron  – 2 1  26.O. p y  and  d x 2  –  y 2  orbitals with bond angles of  90°.6°  S  107°  N  H  H 109°28 ¢  H  H  F  Si  B  H  H  H  F  120°  F  31.  30.  one  unpaired  electron.5°  104. = 2 2 O 22–  Þ s(1s) 2s*(1s) 2s(2s) 2s*(2s) 2s(2p z)    p(2p x )  sp 3 d 2 hybridisation  (twelve 90° angle between bond pair and bond pair)  –  F  2–  p(2p y ) 2p*(2p x ) 2p*(2p y ) 2 ;  B. H 2  will be diamagnetic.  (c) :  B  Ni  NC  CN  F  square planar  F  .  . (c)  CH  C  CH  3  3  24.  (b) : dsp 2  hybridisation gives square planar structure with s.  F  F  F  10 .O. shorter will be the bond length.  square planar  see­saw shaped  ××  H  92.O.  (b) : Central atom in each being sp hybridised shows linear  H 2 ® s(1s) 2 s*(1s) 0 .6  =  2  2 O +2  Þ s(1s) 2s*(1s) 2s(2s) 2s*(2s) 2s(2p z ) 2p(2p x ) 2 p(2p y ) 2p*(2p x ) 1.O. = CN  NC  22.  F  S  O 29. only (b) has  sp 3  hybridization. .  + F — Xe — F  O  C  O  1 0  H 2  ® s(1s)  s*(1s)  .  (a) : Both undergoes sp 3 hybridization..5°  106.  Bond angle :  119. .  one  unpaired  electron  25.  (c) :  SO 2  OH 2  SH 2  NH 3  H 2  ® s(1s)  s*(1s)  .  p x . of p  bonds = 4  23.5°  92..6° <  107° < 109°28¢  <  120°  bond. = dsp 2  hybridisation  dsp 2  sp 3 d  or  dsp 3  hybridisation  (four  90°  angles  between  bond  pair  and  bond  pair)  hybridisation  (six  90°  angles  between  bond  pair  and  bond  pair)  10 .  Thus NO +  is having  higher  bond  order  than  that  of NO so  O  O  NO +  has shorter bond  length.   Assuming that water vapour is an ideal gas.  wreversible  = - nRT ln Vf Vi DH ° .4  kJ  mol  respectively.  40 and  50 J K –1  mol –1 .  CHEMICAL  THERMODYNAMICS A piston filled with 0.  2. If standard enthalpy of combustion of methanol  is –726  kJ mol –1 .  –1  H 2O    (l)  and  CO 2(g)  are  –166.16  JEE MAIN CHAPTERWISE EXPLORER  CHAPTER  5  1.T DS° (b)  ln K =  RT (c)  (d)  3. w = + 208 J  (b)  q = + 208 J.  K =  e –DG°/RT  DGsystem  D Stotal = -T (2012)  The  entropy  change  involved  in  the  isothermal  reversible  expansion of  2 moles  of  an  ideal  gas  from a  volume of 10  9.  4. w = – 208 J  (c)  q = – 208 J. the  reaction  would  be  spontaneous  when  (b)  For a spontaneous process in an isolated system. the average  bond enthalpy of N—H bond  in NH 3  is  (a)  – 1102 kJ mol –1 (b)  – 964  kJ mol –1  (c)  + 352 kJ mol –1  (d)  + 1056 kJ mol –1  (2010)  (a)  – 22. –237.  8.  dm 3  to a volume  of  100  dm 3  at  27°C is  (a)  38.  6.88 kJ  (c)  + 228.3 J mol –1  K –1  (b)  35.  efficiency of the  fuel cell will  be  (a)  80 %  (b)  87%  (c)  90%  (d)  97%  (2009)  Standard entropy of X 2 .  D eg HCl  = –349 kJ mol –1 .  For  the  reaction.0°C.04 mol of an ideal gas expands reversibly  from 50.314 J/mol K) (ln 7.3 J mol –1  K –1  (2011)  is – 46 kJ mol –1 .32 kJ  change (DU) when 1 mol of water is vapourised at 1 bar pressure  1  H2( g ) + O2( g ) ® H 2O( l ) ; DH  = - 286.  On the basis of the following thermochemical data : (D f  G°  (d)  Exothermic processes are always spontaneous. DH and 10.  –  D hyd HCl –  = –381 kJ mol –1 )  will be  (a)  +120 kJ mol –1  (b)  +152 kJ mol –1  (c)  –610 kJ mol –1  (d)  –850 kJ mol –1  (2008)  For a particular reversible reaction at temperature T. (given : molar enthalpy of vapourisation of water at  2 1 bar and 373 K = 41 kJ mol –1  and R = 8.  (2007)  H + (aq)  = 0).3 J mol  K  (d)  42.  If  T e  is  the  temperature  at  criterion  for  spontaneity.  respectively. The  reaction is  3  CH 3 OH ( l ) +  O 2( g )  CO 2( g ) + 2H 2 O ( l ) . the change  (a)  T = T e  (b)  T e  > T  in entropy is positive. the temperature will be  (a)  1000 K  (b)  1250 K  (c)  500 K  (d)  750 K  (2008)  Oxidising  power  of  chlorine  in  aqueous  solution  can  be  determined  by the parameters indicated  below:  – –  1/2 Cl 2(g )  1/2 D diss H Cl (g )  D eg H –  Cl – ( g )  D hyd H Cl – (aq )  The energy involved in the conversion of 1/2 Cl 2(g) to Cl – (aq)  – – (using data.8 J mol –1  K –1  –1  –1  (c)  32.2.  5. Y 2  and XY 3  are 60.  (c)  T > T e  (d)  T e  is 5 times T  (2010)  (c)  Endothermic processes are never spontaneous.    1/2  X 2  +  3/2  Y 2 ®  XY 3 .0 mL to 375 mL at a constant temperature of 37.5 = 2. it absorbs 208 J of heat.2  and  –394.52 kJ  (2009)  In a fuel cell methanol is used as fuel and oxygen gas is used  as an oxidizer.88 kJ  (d)  – 343. w = – 208 J  (d)  q = – 208 J.2 kJ  and 100°C.  The standard enthalpy of formation of NH 3  If  the  enthalpy  of  formation  of  H 2  from  its  atoms  is  – 436 kJ mol –1  and that of N 2  is –712 kJ mol –1 . DH = –30 kJ.  2  At 298 K standard Gibb’s energies of formation for CH 3OH    (l) .3 J mol –1  K –1 ) will be  The  value of enthalpy  of  formation of  OH –  ion  at  25°C is  .88 kJ  (b)  – 228.  11.  w for the process will be (R = 8. to be  at equilibrium.  equilibrium. the internal energy  +  –  H 2O    (l) ®  H  (aq)  + OH  (aq)  ; D H = 57. The values of q  and  7. D diss HCl 2  = 240 kJ mol –1 .01)  (a)  q = + 208 J.  As it does so.  Identify the correct statement regarding a spontaneous process:  (a)  Lowering  of  energy  in  the  reaction  process  is  the  only  DS  were  found  to  be  both  +ve. w = + 208 J  (2013)  The  incorrect expression  among  the  following  is  (a)  in isothermal  process. 17  Chemical Thermodynamics  (a)  41.00 kJ mol –1  (b)  4.100 kJ mol –1  (c)  3.7904 kJ mol –1  (d)  37.904 kJ mol –1  (a)  100 kJ mol –1  (c)  800 kJ mol –1  (2007)  12.  In conversion of limestone  to  lime,  CaCO 3(s) ® CaO (s)  + CO 2(g)  the  values  of DH°  and DS°  are  +179.1  kJ  mol –1  and  160.2 J/K respectively at 298 K and 1 bar. Assuming that DH°  and DS°  do  not  change  with  temperature,  temperature  above  which conversion of limestone to lime will be spontaneous is  (a)  1118 K  (b)  1008 K  (c)  1200 K  (d)  845 K  (2007)  13.  (DH – DU) for the formation of carbon monoxide (CO) from its  elements at 298 K is  (R  = 8.314  JK –1  mol –1 )  (a)  –1238.78 J mol –1  (b)  1238.78 J mol –1  (c)  –2477.57 J mol –1  (d)  2477.57 J mol –1  (2006)  (b)  200 kJ mol –1  (d)  400 kJ mol –1  (2005)  18.  A schematic plot of ln K eq  versus inverse of temperature for a  reaction is shown in the figure. The reaction must be  6.0  ln  K eq  2.0  1.5 × 10 –3  2.0 × 10 –3  1/T (K –1 )  (a)  (b)  (c)  (d)  exothermic  endothermic  one with negligible enthalpy change  highly spontaneous at ordinary temperature.  (2005)  19.  Consider the reaction: N 2  + 3H 2 ® 2NH 3  carried out at constant  14.  The  enthalpy  changes  for  the  following  processes  are  listed  temperature and pressure. If DH and DU are the enthalpy and  below:  internal energy changes for the reaction, which of the following  Cl 2(g)  = 2Cl (g) , 242.3 kJ mol –1  expressions is true?  I 2(g)  = 2I (g) , 151.0 kJ mol –1  (a) DH = 0  (b) DH = DU  ICl (g)  = I (g)  + Cl (g) , 211.3 kJ mol –1  (c) DH < DU  (d) DH > DU  I 2(s)  = I 2(g) , 62.76 kJ mol –1  (2005)  Given that the standard states for  iodine and chlorine are I 2(s)  and Cl 2(g) , the standard enthalpy of formation for ICl (g)  is  20.  For a  spontaneous  reaction the DG, equilibrium  constant  (K)  (a)  – 14.6 kJ mol –1  (b)  –16.8 kJ mol –1  and E° cell  will be respectively  (c)  +16.8 kJ mol –1  (d)  +244.8 kJ mol –1  (2006)  (a)  –ve, >1, +ve  (b)  +ve, >1, –ve  (c)  –ve, <1, –ve  (d)  –ve, >1, –ve  (2005)  15.  An ideal gas is allowed to expand both reversibly and irreversibly  in an isolated system. If T i  is the initial temperature and T f  is the  21.  The enthalpies of combustion of carbon and carbon monoxide  final temperature, which of the following statements is correct?  are  –393.5  and  –283  kJ  mol –1  respectively. The  enthalpy  of  (a)  (T f ) irrev  > (T f ) rev  formation of carbon monoxide per mole is  (b)  T f  >  T i  for reversible  process  but  T f  = T i  for  irreversible  (a)  110.5 kJ  (b)  676.5 kJ  process  (c)  –676.5 kJ  (d)  –110.5 kJ  (2004)  (c)  (T f ) rev  = (T f ) irrev  22.  An  ideal  gas  expands  in  volume  from  1  ×  10 –3  m 3  to  (d)  T f  = T i  for both reversible and irreversible processes.  1 × 10 –2  m 3  at 300 K against a constant pressure of 1 × 10 5  Nm –2 .  (2006)  The work done is  16.  The  standard  enthalpy  of  formation  (DH f °)  at  (a)  – 900 J  (b)  – 900 kJ  298  K  for  methane,  CH 4(g)  is  –74.8  kJ  mol –1 .  The  additional  (c)  270 kJ  (d)  900 kJ  (2004)  information  required  to  determine  the  average  energy  for  23.  The internal energy change when a system goes from state A to  C – H bond formation  would be  B is 40 kJ/mole. If the system goes from A to B by a reversible  (a)  the dissociation energy of H 2  and enthalpy of sublimation  path and returns to state A by an irreversible path what would  of  carbon  be the net change in internal energy?  (b)  latent heat  of vaporisation  of  methane  (a)  40 kJ  (b)  > 40 kJ  (c)  the  first  four  ionisation  energies  of  carbon  and  electron  (c)  < 40 kJ  (d)  zero  (2003)  gain  enthalpy of  hydrogen  24.  In an irreversible process taking place at constant T and P and  (d)  the dissociation  energy  of  hydrogen  molecule, H 2.   in which only pressure­volume work is being done, the change  (2006)  in Gibbs free energy (dG) and change in entropy (dS), satisfy  17.  If the bond dissociation energies of XY, X 2  and Y 2  (all diatomic  the criteria  molecules) are in the ratio of 1 : 1 : 0.5 and DH f  for the formation  (a)  (dS) V, E  < 0, (dG) T, P  < 0  of XY is –200 kJ mol –1 . The bond dissociation energy of X 2  will  (b)  (dS) V, E  > 0, (dG) T, P  < 0 be  18  JEE MAIN CHAPTERWISE EXPLORER  (c)  (dS) V, E  = 0, (dG) T, P  = 0  (d)  (dS) V, E  = 0, (dG) T, P  > 0  (a)  (b)  (c)  (d)  (2003)  violates 1 st  law  of thermodynamics  violates 1 st  law of thermodynamics if Q 1  is –ve  violates 1 st  law of thermodynamics if Q 2  is –ve  does not violate 1 st  law of thermodynamics.  (2002)  25.  The enthalpy change for a reaction  does not depend upon the  (a)  physical states of reactants and products  (b)  use of different reactants for  the  same  product  28.  If an endothermic reaction is non­spontaneous at freezing point  (c)  nature of intermediate reaction  steps  of water and becomes feasible at its boiling point, then  (d)  difference  in  initial  or  final  temperatures  of  involved  (a) DH is –ve, DS is +ve  substances.  (b) DH and DS both are +ve  (2003)  (c) DH and DS both are –ve  26.  If  at  298  K  the  bond  energies  of  C  –  H,  C  –  C,  (d) DH is +ve, DS is –ve  (2002)  C and H – H bonds are respectively 414, 347, 615 and  C  29.  For the reactions,  435 kJ mol –1 , the value of enthalpy  change for  the reaction  C + O 2 ® CO 2  ; DH = –393 J  H 2 C  CH 2(g)  + H 2(g) ® H 3C – CH    3(g)  2Zn + O  ® 2ZnO ; DH = –412 J  2 at 298 K will be  (a)  carbon can  oxidise Zn  (a)  +250 kJ  (b)  –250 kJ  (b)  oxidation of carbon is not feasible  (c)  +125 kJ  (d)  –125 kJ.  (2003)  (c)  oxidation of Zn is not feasible  27.  A heat engine absorbs heat Q 1  at temperature T 1  and heat Q 2  at  (d)  Zn can  oxidise carbon.  (2002)  temperature T 2 . Work done by the engine is J (Q 1  + Q 2 ). This  data  Answer  Key  1.  7.  13.  19.  25.  (b)  (d)  (a)  (c)  (c)  2.  8.  14.  20.  26.  (b)  (d)  (c)  (a)  (d)  3.  9.  15.  21.  27.  (a)  (c)  (a)  (d)  (d)  4.  10.  16.  22.  28.  (c)  (b)  (a)  (a)  (b)  5.  11.  17.  23.  29.  (c)  (d)  (c)  (d)  (d) 6.  12.  18.  24.  (b)  (a)  (a)  (b)  19  Chemical Thermodynamics  1.  (b) : As it absorbs heat, \  7.  q = + 208 J  (d) : For the  given  reaction,  CH3OH ( l ) + æ V 2 ö w rev  =  -2.303 nRT log 10 ç V ÷ è 1 ø  also, DG°f  [CH3OH(l)] = –166.2 kJ mol –1 æ 375 ö w rev  =  -2.303 ´ (0.04) ´ 8.314 ´ 310  log 10 çè 50 ÷ø \  2.  DG°f  [H2O(l)] = –237.2 kJ mol –1  and DG°f  [CO2(g)] = –394.4 kJ mol –1  w rev  = – 207.76 »  –  208 J  Now, DG°reaction  = å DG of  products  - å DG of  reactants  (b) : DG° = –RT lnK DG° = DH° –  TD S° = [–394.4 + 2 × (–237.2)] – (–166.2)  DH° – TDS° = –RT  lnK  3.  = – 702.6 kJ mol –1  % Efficiency = (a) :  Entropy change for an  isothermal process  is  \  Efficiency »  97%  (d) :  1 X + 3 Y ®  XY 3  2 2 2  2 8.  1  DS = 2.303 ´ 2 ´ 8.314 ´ log æç 100 ö÷ è 10 ø  0 0 0  DS reaction = DS products - D S reactants  = 38.294 J mol –1  K –1 » 38.3  J mol –1  K –1  (c) :  1/2N 2  + 3/2H 2  B.E.   712  0 0 \  DS reaction  = DS XY - 1 DS X0 - 3  DSY 0  3 2 2  2 2  = 50 - 1 ´ 60 - 3  ´ 40 = – 40 J K –1  mol –1  2 2 NH 3  436  é1 3  ù \ ( DH °f ) NH = ê B.E.N + B.E.H - 3B.E. N - H ú 3 2 2  2 ë2 û Using equation, DG = DH – TDS  We have DH =  – 30 kJ, DS = – 40 J K –1  mol –1 and at equilibrium DG  = 0. Therefore  é1 3  ù - 46 = ê ´ 712 + ´ 436 - 3B.E. N - H ú 2 ë2 û T  = – 46 = 356 + 654  – 3B.E. N—H  3B.E. N —H  = 1056  B.E.N - H  = 5.  9.  D H  -30 ´ 1000  = = 750 K  DS -40  (c) :  1  Cl ® Cl ( g )  ;  2  2( g ) 1 - 240  DH1 = D diss H e  = = 120 kJ mol -1  Cl 2  2 2 1056  = 352 kJ mol -1  3 e  Cl( g ) ® Cl(-g ) ;  DH 2 = D eg H Cl  = -349 kJ mol-1  (c) : According to  Gibb’s  formula, DG = DH –  TDS  Cl(-g ) + aq ® Cl(-aq ) ;  Since DH  and DS,  both  are  +ve,  for DG  <  0,  the  value  of  T > T e .  6.  DG  -702.6  ´ 100 = ´ 100  DH  - 726 = 96.77% æ DH ° - T DS° ö ln K = - ç ÷ø  è RT æV ö DS = 2.303nR log ç 2  ÷ è V ø  4.  3  O ¾¾ ® CO2( g ) + 2H 2O( l ) ,  2 2( g ) DH  = –726  kJ mol –1  –  (b) :  The  reaction  for  the  formation  of  OH  (aq)  is  1  H 2( g ) + O 2( g )  H +( aq ) + OH ( – aq )  2  This is  obtained by adding  the two  given  equations. \ DH  for the  above reaction  = 57.32  +  (–286.2)  =  –228.88 kJ  DH 3  = D hyd H e  = -381 kJ mol -1  The required reaction is  1  Cl ® Cl(-aq ) ; D H 2  2( g ) 1  e e e  Then  DH = D diss H + D eg H + D hyd H 2  = 120 + (– 349) + (–381) = –610 kJ mol –1 20  JEE MAIN CHAPTERWISE EXPLORER  10.  (b) : In an isolated system, there is neither exchange of energy  16.  (a) : C + 2H 2 ® CH 4  ; DH° = –74.8 kJ mol –1  In order to calculate average energy for C – H bond formation  nor  matter  between  the  system  and  surrounding.  For  a  spontaneous process in an isolated system, the change in entropy  we should know  the following data.  is positive, i.e. DS >  0.  C (graphite) ® C (g)  ; DH° f  = enthalpy of sublimation of carbon  Most of the spontaneous chemical reactions are exothermic. A  H 2(g) ® 2H (g)  ; DH° = bond dissociation energy of H 2  number of endothermic reactions are spontaneous e.g. melting  17.  (c) : Let the bond dissociation energy of XY, X 2  and Y 2  be x kJ  of  ice (an  endothermic  process) is  a  spontaneous reaction.  mol –1 , x kJ mol –1  and 0.5x kJ mol –1  respectively.  The two factors which  are responsible  for the  spontaneity of  1 1  a process are  X 2 + Y2  ® XY ; DH f  = -200 kJ mol -1  2 2  (i)  tendency to acquire  minimum energy  DH reaction = [(sum of bond dissociation energy of all reactants)  (ii)  tendency to acquire  maximum randomness.  – (sum  of bond dissociation  energy of product)]  11.  (d) :  DU = DH – D nRT 1  é1 ù =  ê DH X 2 + DHY2  - DH XY  ú = 41000 – 1 × 8.314  × 373  2  ë2 û  = 41000 – 3101.122  x 0.5 x  = 37898.878 J mol –1  =  37.9 kJ mol –1  = + - x = -200  2 2  12.  (a) :  For DG° = DH° – TDS°  200  For a spontaneous process DG° < 0  x= =  800 kJ mol -1  \  0.25  i.e. DH° – TDS° < 0  K 2  DH  é 1 1  ù or DH° <  TDS°    or,  TDS° > DH°  18.  (a) :  ln  K = R ê T - T ú 1 ë 1 2  û  DH °  179.1 ´ 1000  i.e.  T >  or  T  >  6  DH  D S °  160.2  ln = [1.5 ´ 10-3 - 2 ´ 10 -3 ]  2  R or  T > 1117.9 K » 1118 K  DH  13.  (a) : DH – DU = Dn g  RT  ´ (-0.5 ´ 10-3 )  or,  ln3 = R 1  C +  O 2 ® CO  DH  of  reaction  comes  out  to  be  negative.  Hence  reaction  is  2  exothermic.  1  æ 1ö Dng  = 1 - ç1 + ÷ = è 2ø 2  19.  (c) : N 2  + 3H 2 ® 2NH 3 1  -1  DH – DU =  - ´ 8.314 ´ 298 = - 1238.78 J mol  2 14.  (c) :  1  1  I 2 (s)  +  Cl 2 (g) ® ICl (g) 2  2  1 1  é1 ù DH ICl (g)  =  ê DH I2 ( s ) ® I 2 ( g ) + DH I -I + DH Cl -Cl ú -[ D H I -Cl ]  ë2 2 2  û  1 1  é1 ù = ê ´ 62.76 + ´ 151.0 + ´ 242.3ú - [211.3]  ë2 2 2 û  = [31.38 + 75.5 + 121.15] – 211.3  = 228.03 – 211.3 = 16.73 kJ/mol  Dn = 2 – 4 = –2 DH = DU + DnRT = DU – 2RT \ DH < DU  20.  (a) : For spontaneous process, DG = –ve  Now DG = –RT ln K  When K > 1, DG = –ve  Again DGº = – nFEº  When Eº = +ve, DGº = –ve  21.  (d) : C (s)  + O 2(g) ® CO 2(g)  ; DH = –393.5 kJ mol –1  CO (g)  +  1  O  ® CO 2(g)  ; DH = –283 kJ mol –1  2  2(g) .. (i)  ... (ii)  On subtraction equation (ii) from equation (i), we get  15.  (a) : If a gas was to expand by a certain volume reversibly, then  C (s)  + O 2(g) ® CO (g)  ; DH = –110.5 kJ mol –1  it would do a certain amount of work on the surroundings. If  The enthalpy of  formation of carbon monoxide  per  mole  it was to expand irreversibly it would have to do the same amount  = –110.5 kJ mol –1  of work on the surroundings to expand in volume, but it would  22.  (a) : W = – PDV  also  have  to  do  work  against  frictional  forces.  Therefore  the  = –1 × 10 5 (1 × 10 –2  – 1 × 10 –3 )  amount of work have greater modulus but –ve sign.  = –1 × 10 5  × 9 × 10 –3  = –900 J W irrev.  > W reve.  ;   (T f ) irrev.  >  (T f ) rev.  21  Chemical Thermodynamics  H reversible path 26.  (d) :  23.  (d) :  A H B irreversible path  H C C +H–H H H H H–C–C–H H H DH Reaction  = å  BE reactant  – å  BE product  = 4 × 414 + 615 + 435 – (6 × 414 + 347)  = 2706 – 2831  = –125 kJ  We know  that for a  cyclic  process  the  net change  in internal  energy is equal to zero and change in the internal energy does  27.  (d) : It does not violate first law of thermodynamics but violates  not depend on the path by which the final state is reached.  second law of thermodynamics.  24.  (b) : For spontaneity, change in entropy (dS) must be positive,  28.  (b) : For endothermic reaction, DH = +ve  means it should be greater than zero.  Now, DG = DH – TDS  Change in Gibbs free energy (dG) must be negative means that  For non­spontaneous reaction, DG should be positive  Now DG is positive at low temperature if DH is positive. it should be lesser than zero. (dS) V, E  > 0, (dG) T, P  < 0.  DG is negative at high temperature if DS is positive.  25.  (c) : This is according to Hess's law.  29.  (d)  : DH  =  negative  shows  that  the  reaction  is  spontaneous.  Higher value for DH shows that the reaction is more feasible. 22  JEE MAIN CHAPTERWISE EXPLORER  CHAPTER  SOLUTIONS 6  1.  2.  3.  K f  for water is 1.86 K kg mol –1 . If your automobile radiator  7.  holds 1.0 kg of  water,  how  many  grams of  ethylene glycol  (C 2 H 6 O 2 ) must you add to get the freezing point of the solution  lowered  to –2.8°C?  (a)  93 g  (b)  39 g  (c)  27 g  (d)  72 g  (2012)  The  density  of  a  solution  prepared  by  dissolving  120  g  of  8.  urea (mol. mass = 60 u) in 1000 g of water is 1.15 g/mL. The  molarity of  this solution  is  (a)  1.78 M  (b)  1.02 M  (c)  2.05 M  (d)  0.50 M  (2012)  The degree of dissociation (a) of a weak electrolyte, A x B y  is  related  to  van’t Hoff  factor  (i)  by  the  expression  i - 1  i - 1  (b)  a = ( x + y + 1) ( x + y - 1) ( x + y - 1)  ( x + y + 1)  (c)  a = (d)  a = i - 1 i - 1 (a)  a = (2011)  9.  If sodium sulphate is considered to be completely dissociated  into cations and anions in aqueous solution, the change in freezing  point of water (DT f), when 0.01 mol of sodium sulphate is dissolved    in 1 kg of water, is (K f  = 1.86 K kg mol –1 )  (a)  0.0186 K  (b)  0.0372 K  (c)  0.0558 K  (d)  0.0744 K  (2010)  A binary liquid solution is prepared by mixing n­heptane and  ethanol.  Which  one  of  the  following  statements  is  correct  regarding the  behaviour of  the  solution?  (a)  The  solution  formed is an ideal solution.  (b)  The solution is non­ideal, showing +ve deviation from  Raoult’s law.  (c)  The solution is non­ideal, showing –ve deviation from  Raoult’s law.  (d)  n­heptane  shows  +ve  deviation  while  ethanol  shows  –ve  deviation from Raoult’s  law.  (2009)  Two liquids X and Y form an ideal solution. At  300 K, vapour  pressure of the solution containing 1 mol of X and 3 mol of  Y is 550 mm Hg. At the same temperature, if 1 mol of Y is  further added to this solution, vapour pressure of the solution  increases by 10 mm Hg. Vapour pressure (in mm  Hg) of  X  and Y in their pure states will be, respectively  (a)  200 and 300  (b)  300 and 400  (c)  400 and 600  (d)  500 and 600  (2009)  4.  Ethylene  glycol  is  used  as  an  antifreeze  in  a  cold  climate.  Mass of  ethylene  glycol  which should  be  added  to 4  kg of  water to prevent  it  from  freezing  at  –  6°C  will  be  : (K f  for  water = 1.86 K kg mol –1 , and molar mass of ethylene glycol  = 62 g mol –1 )  (a)  804.32 g  (b)  204.30 g  (c)  400.00 g  (d)  304.60 g  (2011)  5.  A 5.2 molal aqueous solution of methyl alcohol, CH 3 OH, is  supplied. What is the mole fraction of methyl alcohol in the  10.  The  vapour  pressure  of  water  at  20°C  is  17.5  mm  Hg.  If  18  g of  glucose  (C 6 H 12 O 6)  solution?    is  added  to 178.2  g  of  water  at  20°C, the vapour pressure of the resulting solution  will be  (a)  0.100  (b)  0.190  (a)  17.325 mm Hg  (b)  17.675 mm Hg  (c)  0.086  (d)  0.050  (2011)  (c)  15.750 mm Hg  (d)  16.500 mm Hg  On  mixing,  heptane  and  octane  form  an  ideal  solution. At  (2008)  373  K,  the  vapour  pressure  of  the  two  liquid  components  6.  (heptane  and octane)  are 105  kPa  and  45 kPa  respectively.  11.  At 80°C, the vapour pressure of pure liquid A is 520 mm of  Hg and that of pure liquid B is 1000 mm of Hg. If a mixture  Vapour pressure of the solution obtained by mixing 25.0 g of  solution of  A  and  B  boils at  80°C  and  1 atm  pressure,  the  heptane and 35 g of octane will be (molar mass of heptane =  –1  –1  100 g mol  and of octane = 114 g mol  )  amount of A  in the mixture is (1 atm = 760 mm of Hg)  (a)  50 mol percent  (b)  52 mol percent  (a)  144.5 kPa  (b)  72.0 kPa  (c)  34 mol percent  (d)  48 mol percent  (c)  36.1 kPa  (d)  96.2 kPa  (2010)  (2008)  Solutions  23  12.  A 5.25% solution of a substance is isotonic with a 1.5% solution  21.  Which one of the following statements is false?  of urea (molar mass = 60 g mol –1 ) in the same solvent. If the  (a)  Raoult’s law states that the vapour pressure of a component  densities  of  both  the  solutions  are  assumed  to  be  equal  to  over a solution is proportional  to its mole fraction.  1.0 g cm –3 , molar mass of the substance will be  (b)  The  osmotic  pressure (p)  of  a  solution  is  given  by  the  (a)  210.0 g mol –1  (b)  90.0 g mol –1  equation (p = MRT, where M is the molarity of the solution.  (c)  115.0 g mol –1  (d)  105.0 g mol –1  (c)  The  correct  order  of  osmotic  pressure  for  (2007)  0.01 M aqueous solution of each compound is  BaCl 2  > KCl > CH 3 COOH > sucrose.  13.  A  mixture  of  ethyl  alcohol  and  propyl  alcohol  has  a  vapour  (d)  Two  sucrose  solutions  of  same  molality  prepared  in  pressure of 290 mm at 300 K. The vapour  pressure of propyl  different  solvents  will  have  the  same  freezing  point  alcohol is 200 nm. If the mole fraction of ethyl alcohol is 0.6,  depression.  (2004)  its vapour pressure (in mm) at the same temperature will be  (a)  360  (b)  350  22.  Which of the following liquid pairs shows a positive deviation  (c)  300  (d)  700  (2007)  from Raoult’s law?  (a)  Water ­ hydrochloric acid  14.  The density (in g mL –1 ) of a 3.60 M sulphuric acid solution that  (b)  Benzene ­ methanol  is 29% H 2 SO 4  (molar mass = 98 g mol –1 ) by mass will be  (c)  Water ­ nitric acid  (a)  1.45  (b)  1.64  (d)  Acetone  ­  chloroform  (2004)  (c)  1.88  (d)  1.22  (2007)  23.  To neutralise completely 20 mL of 0.1 M aqueous solution of  15.  18 g  of glucose  (C 6 H 12 O 6 )  is  added  to  178.2  g  of  water. The  phosphorous acid (H 3 PO 3 ), the volume of 0.1 M aqueous KOH  vapour pressure of water for this aqueous solution at 100°C is  solution required is  (a)  759.00 torr  (b)  7.60 torr  (a)  10 mL  (b)  20 mL  (c)  76.00 torr  (d)  752.40 torr  (2006)  (c)  40 mL  (d)  60 mL  (2004)  16.  Density of a 2.05 M solution of acetic acid in water is 1.02 g/  24.  6.02 × 10 20 molecules of urea are present in 100 ml of its solution.  mL. The molality of  the solution  is  The concentration of urea  solution is  (a)  1.14 mol kg –1  (b)  3.28 mol kg –1  (a)  0.001 M  (b)  0.01 M  (c)  2.28 mol kg –1  (d)  0.44 mol kg –1  (2006)  (c)  0.02 M  (d)  0.1 M  (2004)  17.  Equimolal solutions in the same solvent have  25.  Which  one  of  the  following  aqueous  solutions  will  exhibit  (a)  same boiling  point but different freezing point  highest boiling point?  (b)  same freezing point but different boiling  point  (a)  0.01 M Na 2 SO 4  (b)  0.01 M KNO 3  (c)  same boiling and same freezing points  (c)  0.015 M urea  (d)  0.015 M glucose  (2004)  (d)  different boiling  and  different freezing points.  (2005)  26.  If liquids A and B form an ideal solution, the  18.  Two solutions of a substance (non electrolyte) are mixed in the  (a)  enthalpy of mixing is zero  following manner. 480 mL of 1.5 M first solution + 520 mL of  (b)  entropy of mixing is zero  1.2 M second solution. What is the molarity of the final mixture?  (c)  free energy of mixing is zero  (a)  1.20 M  (b)  1.50 M  (d)  free energy as well as the entropy of mixing are each zero.  (c)  1.344 M  (d)  2.70 M  (2005)  (2003)  19.  Benzene and toluene form nearly ideal solutions. At 20°C, the  27.  25 mL of a solution of barium hydroxide on titration with a 0.1  vapour pressure of benzene is 75 torr and that of toluene is 22  molar solution of hydrochloric acid gave a litre value of 35 mL.  torr. The partial vapour pressure of benzene at 20°C for a solution  The molarity of barium hydroxide solution was  containing 78 g of benzene and 46  g of toluene in torr is  (a)  0.07  (b)  0.14  (a)  50  (b)  25  (c)  0.28  (d)  0.35  (2003)  (c)  37.5  (d)  53.5  (2005)  28.  In a 0.2 molal aqueous solution of a weak acid HX, the degree  20.  If a  is  the  degree  of  dissociation  of  Na 2 SO 4 ,  the  vant  Hoff’s  of ionization is 0.3. Taking K f  for water as 1.85,  the  freezing  factor (i) used for calculating the molecular mass is  point of the solution will be nearest to  (a)  1 + a  (b)  1 – a  (a)  – 0.480°C  (b)  – 0.360 °C  (c)  1 + 2a  (d)  1 – 2a  (2005)  (c)  – 0.260 °C  (d)  + 0.480 °C  (2003)   27.86°C.d)  6.  (c)  A  –  B  interaction  is  weaker  than  A  –  A  and  (a)  0.0512°C  (d)  A  –  B  interaction  is  stronger  than  A  –  A  and  (c)  0.  22.  21.24  JEE MAIN CHAPTERWISE EXPLORER  29.  8.  (c)  13.  find the increase in boiling point. (a)  25.  18.  (a)  (a)  (c)  (b)  (a)  5.2372°C  (2002) (2002)  Answer  Key  1.  Freezing point of an aqueous solution is  (–0.  29.  24. (a)  2.  (b)  (a)  (c)  (b)  (b)  .  (a)  (c)  (d)  (d)  (b)  4.  (b) DH mix  < 0  K f  = 1.  (c)  (a)  (c)  (c)  (b.  (d)  0. Elevation  (a) DV mix  > 0  of  boiling  point  of  the  same  solution  is  K b  =  0. (b)  19.  17.  (a)  7.092°C  B – B interaction.  16.186°C  B – B interaction  (b)  0.  26.  11.  15.  9.  28.512°C.  (c)  (b)  (d)  (c)  (a)  3.  14.  In mixture A and B components show –ve deviation as  30.  20.  12.  30.  23.  10.186)°C. 55 » 72 kPa  7. K f  = 1.  0.25 + 0.05 M  1120 / 1.25  x heptane  = = 0.2  1000  5. solvent is water.15  = ´ 1000 = = 2.30 p Total  = x heptane  p° heptane  + x octane  p° octane  xA y+  +  yB x–  i .86 5.25 + 0.8) = 2.8°C  Mass of solvent =  1.8 = 1.15 1120 (a)  : A x B y  = 0.8  Þ w= = 93 g  62 1.2 ´ 18 93.30  114 0. P T  = Total presure  p° X  = Vapour pressure of  X  in pure  state  p°  Y  = Vapour pressure  of Y in pure  state  x X  = Mole fraction of  X  = 1/4  x Y  = Mole fraction of  Y = 3/4  (i)  When T = 300 K.15  g/mL  Total mass of solution =  1000 + 120  = 1120  g  35  = 0.54  0.1  \  a = ( x + y - 1) 25  = 0. i  = 3  m= 1 – a  xa  ya  i = 1 – a + xa + ya  = 1 + a(x + y  – 1) 4.86 ´ 5.  (c)  : P T  =  p° X  x X  +  p°  Y  x Y  w 1  and  w 2  = wt.25  Solutions  1.45  0.54 × 45  = 47.  N = 1000  18 \  Mole  fraction =  Weight of solute  Molecular mass of solute  m= ´ 1000  Mass of solvent (g) m= = 6.  (b) :  The solution containing  n­heptane and ethanol  shows  non­ideal  behaviour  with  positive  deviation  from  Raoult’s  law.25 + 24.  of  solute DT f  = 0 – (– 6) = 6  1.0 kg  Mass of solute = ?  Molecular mass of solute =  62 DT f  = K f  ×  m  Here  solute  is methyl alcohol.6 + 1000 1093.  Given n = 5.01  = 0.  (b)  : Given.2 +  18 where.0855 » 0.15 Mass of solute  Molecular mass of solute  Molarity =  ´ 1000  Volume of solution 120 / 60 2 ´ 1000 ´ 1.0558 K  w ´ 1000 (a)  : D T f  = K f  ×  m  =  K f ´ 2  w1 ´ m2  \ 6 = (c) :Depression in freezing point.86 K kg mol –1 \ DT f  = 3 × 1.  5.86 K kg mol –1 DT f  = 0 – (–2. as a result they easily vapourise  showing higher vapour presure than expected.30  = 0.  (c)  : Mole  fraction  of  solute  = n N + n n  = number of  moles  of  solute  N = number of moles  of  solvent  0. DT f  = i × K f  × m  For sodium sulphate.01 m.  8.3 = 71.86 ´ w2  ´ 1000  4000 ´ 62 6 ´ 62 ´ 4000  w 2  = = 800 g  1000 ´ 1.86 × 0.2.  (a) :  K f  = 1.  however  the  forces  between  n­heptane  and ethanol are not very strong.  1 kg Given. This is because the ethanol molecules are held together  by  strong H­bonds.01 = 0.6 p° octane  = 45 kPa  w / 62  w ´ 1000 = 1000 62 2.  w octane  = 35 g  w 62 ´ 2.86 nheptane  = (c)  : Mass of solute taken =  120 g  Molecular mass of  solute = 60  u  Mass of solvent =  1000 g  Density of solution  =  1.6  = = 0.  9.  p° heptane  = 105 kPa  w heptane  = 25 g  DT f  = K f  ×  m  2.45 × 105 + 0. of  solvent  and  solute  m 2  = molecular wt.086  93.25  100 noctane  = Volume of solution =  3.  P T  = 550 mm Hg .30 xoctane  = Mass 1120  =  mL  Density 1.   (c) : Molality.  (a) : Isotonic solutions  have same osmotic pressure.p s  18/180 1/10  directly  proportional  to  its  mole  fraction..4 × 200  or  P A o  = 350 mm  19.(1) (ii)  When at T = 300  K. at 1 atm pressure i..05  m= = P solution  =  17.02 . 1  mole of  Y is  added..p s = 99  \  P sol  =  P°X solvent  760 ´ 99  Let A be the  solute and  B the  solvent  = 752.  (d) : The  extent of depression in freezing point varies  with  the  number  of  solute  particles  for  a  fixed solvent  only  and  it’s  a  characteristic  feature  of  the  nature  of  solvent  also.6 M solution means 3.6 moles of H 2 SO 4  = 3. .  p 2 =  C 2 RT For isotonic solution. density =  Volume 1000 On solving equations (1)  and (2).6) + 200 ´ (1 - 0.6 × 98 g = 352.99 ps = ps  Þ  760 .  or.5 = 1.9  = 0. at 760  mm Hg  = 480 × 1.325  1000 ´ 1.5 20.344 M  Molarity of the  final  mixture  =  Þ  520 X A  + 1000 – 1000 X A  = 760 mm Hg 1000 1  Þ  X A  =  or   50  mol percent  2  12.8 g \ Mass of H 2 SO 4  in 1000 mL of solution = 352.  pressure is  760 .(2) 14.2 = 720 + 624 = 1344  PA° X A + PB° X B  = 760 mm Hg  Total volume = 480 + 520 = 1000  PA° X A + PB° (1 .  (d) :  p° Y  = 600 mm Hg and p° X  = 400 mm Hg  p s  N 10.2  n B  18  \ X B  = = M  n A + nB  18 + 178.8 g  Given.25 / M  =  V V [Where M = molecular weight of the substance]  1.  (c) : Total millimoles of solute  Then.4 torr  Þ  100p s  = 760 × 99 Þ  p s  =  100 178.  p1 = C1 RT .6)  or    290 =  0.  for  two different solvents the extent of depression may vary even  if number of solute particles be dissolved in them.216 g/mL  Now.05 ´ 60 897  11.  and mole  fraction of  B  in  solution  =  X B  18.26  JEE MAIN CHAPTERWISE EXPLORER \ 550 = p °X Þ ( 14 ) +  p  ( 43 )  ° Y  p°X + 3 p Y ° = 2200 .99  where M  = molarity.5/ 60 5.2.6 mole of H 2 SO 4  is present in  1000 mL  of solution.a + 2 a + a  = 1 + 2a  1 21.  =  or  M =  210  60  M 13..9  P solution  =   vapour  pressure of  its  pure  component  1  ×  mole  fraction  in  solution ps Þ 760 ´ 99 .  (b) : According to Raoult’s law. DT f  = k f  × m  For  different  solvents.p s  n  =  15.  (c) : According to Raoult’s law equimolal solutions of all the  and  PB °  = 1000 mm Hg  substances in the same solvent will show equal elevation in  Let  mole  fraction of A  in  solution =  X A  boiling points as well as equal depression in freezing point.  (d) : 3..  p1 = p 2 \  C 1  = C 2  1.  M 2  = molecular  mass  9.05 2. P A  °  = 520 mm Hg  = 2. \ Mass of 3.  (c) : Na 2 SO 4  ƒ  2Na +  + SO 42–    1                     0               0  1 – a  2a a  Vant Hoff factor (i) =  1 .5  ×  0.8 =  1216 g of  solution  29 Mass 1216  = = 1.2 /18 9.25  or.2  ´ 1000  16.X A ) = 760 mm Hg  1344  = 1.  So.  d = density.  (a) :  In  solution containing non­volatile  solute.  P = PA + PB  = PA° x A + PB° x B  or    290 =  PA ° ´ (0.. we get  p ° .8 g of H 2 SO 4  is present in  100  ´ 352..  P T  = (550 + 10) mm Hg \  x X  = 1/5  and  x Y  = 4/5 1 4  Þ 560 = p °X +  p Y ° 5 5 ( ) ( )  or p°X + 4 p Y ° = 2800 .6 × PA °  + 0.28 mol kg –1  17.94  Now P solution  =  P°X solvent  =  17.99  2.  (a) :  We  have.  value  of  k f  is  also  different. 29 g of H 2 SO 4  is present in 100 g of solution \ 352. P B  = P° B  X B  P°  B  =  75 torr  78 / 78 1 1  X B  =  (78/ 78) + (46 / 92) = 1 + 0.e.  = = ps  178.5 1  P B  = 75 ´ = 50 torr  1.  (a) : According to Raoult's law.28 × 10 –3  mol g –1  = 2.  m = 180 18  1000 d - MM 2  X B  = 9.5 5.5 + 520 × 1.   23.4810  Freezing point of solution = 0 – 0.  (b. Consequently the vapour pressure of  the solution is greater than the vapour pressure as expected  from Raoult’s law.405 × 0. i.186 1.  the escaping tendency of alcohol and benzene molecules from  the solution increases.  M1  = CH 3  CH 3  CH 3  ­ ­ ­ ­ O —– H ­ ­ ­ ­ O —– H ­ ­ ­ ­ O —– H ­ ­ ­ ­  On  adding benzene. of moles after dissociation  =  Kf  (experimental) no.e. DT b  = 0. of moles before dissociation  or. higher  the  extent  of  elevation  in  boiling  point. of moles of solute  10 -3 ´ 1000 = 0.  (b) : Moles of urea =  6.  p A  > pº  A x A;    p B  > p ºB x B ;  p > p A  + p B  In  solution  of  methanol  and  benzene.02 ´ 10 23  Concentration  (molarity)  of  solution  =  no.481° C  29.3  =  1.  methanol  molecules  are held together due to hydrogen bonding as shown below:  26.  K f  (observed) = 1.02 ´ 10 20  = 10 -3  moles  24.  the observed  vapour pressure of each component and total vapour pressure  are greater than predicted by Raoult's law.3  K f  (observed)  no.  N 1 V 1  (acid)  = N 2 V 2  (base)  0.3 = 1. of litres of solution  100 25. DV mix  < 0  and DH mix  < 0.1 ´ 2 ´ 20  \  V2  = 0.1 × 2 × 20 = 0.14  25  28.3 = 2. neither heat is evolved  nor absorbed during dissolution.1 ×  1 × V 2 0.512  or. Here A – B  attractive  force  is  greater  than  A – A and B – B attractive forces.85 × 1.27  Solutions  22.  Kf  (observed)  1.01 M  =  no.  (a) :  For ideal solutions.2 = 0.481 = – 0.1 × 35  or.  the  benzene  molecules  get  in  between  the molecules of methanol.  (a) : Elevation in boiling point is a colligative property which  depends upon  the  number  of  solute  particles. thus breaking the hydrogen bonds.  (a) : HX  H +  + X –  1                 0  0  1 – 0.  27.  As the resulting solution has weaker intermolecular attractions. from Raoult's law.3 + 0.3  Total number of moles after dissociation  = 1 – 0.85 1  or.  (b) :  In  solutions showing  positive deviation.  (c) : H 3 PO 3  is a dibasic acid.405 DT f  = K f  × molality = 2.  (b) : DT b  =  K b  DT f = K f  W B  ´ 1000  M B ´ W A  W B  ´ 1000  M B ´ WA  DTb K b DT b  = or  =  0.  Greater the number of solute particles  in  a  solution. DH mix  = 0.1 ´ 35  =  0.  (b) : Ba(OH) 2  HCl  M 1 V 1  =  M 2 V 2  M 1  × 25 = 0.86  .3   0.0512°C D T f K f  0.  Na 2 SO 4 ®  2Na +  + SO 4 2–  0.3 + 0. d) : For negative deviation.1 ´ 1  = 40 mL  6.  30.3          0. 034M solution of  the carbonic acid.3 atm  (d)  0.8 × 10 –11  Select the correct statement for a saturated 0.22  (b)  9.  will  Mg  ions  start  precipitating  in  the  form  of  Mg(OH) 2  from a solution of  0.  Four species are listed below :  +  (i)  HCO 3–   (ii)  H 3O    –  (2010)  (iii)  HSO 4  (iv)  HSO 3F    Which  one  of  the  following  is  the  correct  sequence  of  their  Three reactions involving  H 2 PO 4 –  are given  below:  acid strength?  (i)  H 3 PO 4  + H 2 O  H 3 O +  +  H 2 PO 4 –  –  2–  +  (a)  iii < i < iv < ii  (ii)  H 2 PO 4  + H 2 O  HPO 4  +  H 3 O  (b)  iv < ii < iii < i  (iii)  H 2 PO 4 –  + OH –  H 3 PO 4  +  O 2–  –  (c)  ii < iii < i < iv  In  which of the above does  H 2 PO 4  act as an acid?  (d)  i < iii < ii < iv  (2008)  (a)  (i) only  (b)  (ii)  only  (2010)  12.1 × 10 –5  M  (b)  5.28  JEE MAIN CHAPTERWISE EXPLORER  CHAPTER  EQUILIBRIUM 7  1. is 4.9 L  (d)  2. the solubility product of Mg(OH) 2  is 1.  (2010)  The correct order of increasing basicity of the given conjugate  bases (R = CH 3 ) is  (a)  RCOO –  < HC  C –  < NH 2 –  <  R –  (b)  RCOO –  < HC  C –  < R –  < NH 2 –  –  –  C  < RCOO –  < NH 2 –  (c)  R  < HC  –  (d)  RCOO  < NH 2 –  < HC  C –  <  R –  (2010)  A vessel at 1000 K contains CO 2  with a pressure of 0.  Solid Ba(NO 3 ) 2  is gradually dissolved in a 1.79  (d)  7. The value of  K c  for  the reaction.2 × 10 –10  g  (d)  6.  The pK a  of a weak acid.05M solution of silver nitrate to start the  (c)  4. K a  of this  acid is  9.1 × 10 –7  M  (2009)  which  pH.  4.  2.78.02  (2012)  The pH of a 0.18 atm  (2011)  to form?(K sp  for BaCO 3  = 5. If the total pressure at equilibrium is 0. At what concentration of Ba 2+  will a precipitate begin  (c)  0.0 × 10 –4 M  Na 2 CO 3  (a)  1.  (c)  The concentration of CO 3 2–  is greater than that of HCO 3 – .8 atm  (b)  3 atm  solution.1 × 10 –8  M  (d)  8.  (d)  (iii)  only  .  BOH  is  4. The pK b of a weak base. The quantity  corresponding salt. (HA).  11.0  3.  (b)  The concentration of CO 3 2–  is 0. BA.  (a)  9.0 × 10 –11 .  (a)  The  concentration of H +  is double that of  CO 3 2– .  1  1  N  +  O 2(g)  at  the  same  2  2(g)  2  (b)  4 ×  10 –4  (d)  0.5 atm.0 × 10 –13 .5 × 10 2  (c)  50.  5.  How many litres of water must be added to 1 litre of an aqueous  solution of HCl with a pH of 1 to create an aqueous solution  with pH of  2?  8.80.  Some  of  the  CO 2  is  converted  into  CO  on  the  addition  of  graphite. The value  of  the ionization constant.  (a)  1 × 10 –3  (b)  1 ×  10 –5  (c)  1 × 10 –7  (d)  3 × 10 –1  (2012)  (a)  5.  (d)  The  concentration  of  H +  and  HCO 3 –  are  approximately  equal.2 × 10 –5  g  (2010)  In aqueous solution the ionisation constants for carbonic acid  are  K 1  = 4.2 × 10 –7  and  K 2  = 4. will be  of potassium bromide (molar mass taken as 120 g mol –1 ) to be  (a)  9.1 × 10 –5  M  At 25°C.001 M  Mg 2+  ions?  (a)  8  (b)  9  (c)  10  (d)  11  6. the value  of K  is  10.0 × 10 –8  g  (c)  1.  The  pH  of  an  aqueous  solution  of  the  Solubility product of silver bromide is 5.1 × 10 –9  )  (a)  4.0 L  (2013)  The  equilibrium  constant  (K c )  for  the  reaction  N 2(g)  + O 2(g) ® 2NO (g)  at temperature T is 4 × 10 –4 . At  2+  (c)  8.1 molar solution of the acid HQ is 3.8 atm.1 L  (c)  0.58  added to 1 litre of 0.2 × 10 –9  g  (b)  1.    NO (g) ® temperature is  (a)  2.0 L  (b)  0.01  (2008)  precipitation of AgBr  is  (c)  (i) and (ii)  7.034 M. 29  Equilibrium  13.0 × 10 –5  and 5.0 × 10  (2007)  (R  =  0.x ø  (c)  K 2  K 3  = K 1  (d)  K 3  = K 1K  (2008)    2  æ x  ö æ x  ö 14.12  (b)  + 8. The pOH of  an aqueous  23. equilibrium  20.2 × 10 5  (b)  5.  Given the data at 25°C.84 atm. F 2  and ClF 3 ?  is K c  = 4. the partial pressure of PCl 3  Which of the following relation  is  correct?  will be  æ x  ö æ 2 x  ö (a)  K 3  ∙ K 23   = K 12   (b)  K1 K 2 =  K3  (a)  ç ÷ P  (b)  ç ÷ P  è x + 1 ø  è 1 .  (a)  1 : 9  (b)  1 : 36  Ammonium hydrogen sulphide decomposes to yield NH 3  and  (c)  1 : 1  (d)  1 : 3  (2008)  H 2 S gases in the flask.  For the following three reactions (i).  Phosphorus  pentachloride  dissociates  as  follows  in  a  closed  constants are given  reaction vessel. The overall dissociation  25.50 atm.800 V  (b)  K p  is less than K c  What is the value of log K sp  for AgI?  (c)  K p  = K c  RT  æ ö 2.5  24.98 × 10 8  (b)  3.8 × 10  c  (c)  5.  If degree of  dissociation of  X and Z  be equal  then  the  21.0 × 10 –5  –6  at 184°C)  15  –15  (K  = 1.88 × 10 6  17.  The equilibrium constant for  the  reaction.  constant of the acid will be  2NO 2(g)  2NO (g)  + O 2(g)  (a)  0.30  (b)  0.5.  (a)  – 8. the total pressure in the flask rises to 0.x ø  X  2Y and Z  P + Q.5  (d)  9.83 × 10 –3  g  (d)  1.0831  kJ/(mol.  An amount of solid NH HS is placed in a flask already containing  4  ratio of total pressures at  these equilibria is  ammonia gas at a certain temperature and 0.0 × 10 –10  respectively.  temperature is  If  the  solubility  product  constant  K sp  of  AgIO 3  at  a  given  (a)  0.  Cl 2(g)  + 3F 2(g)  2ClF 3(g) ; DH = –329 kJ  1  Which of the following will increase the quantity of ClF  3  in an  SO 3(g)  SO 2(g)  +  O 2(g)  2  equilibrium mixture of Cl 2 .9 × 10 –2  (2006)  (2007)  . pressure.  compared at 184°C it is found that  Ag + I – ® AgI + e –  ; E° = 0.0 × 10 –7  g  (c)  HCOOH  (d)  CH 3 CH 2 COOH  (2005)  (2007)  –  16.  The first and second dissociation constants of an acid H 2 A are  (c)  3.40 × 10 –3  (d)  Adding F 2  (2005) (c)  9.5  (c)  2.18  temperature is 1. The value of K c  for the reaction  (a)  Increasing the temperature  (b)  Removing Cl 2  2SO 2(g)  + O 2(g)  2SO 3(g)  will be  (c)  Increasing the volume of the container  (a)  416  (b)  2. The  AgIO 3  (molecular mass = 283) the equilibrium which sets in is  equilibrium  constant  for  NH 4 HS  decomposition  at  this  AgIO 3(s)  Ag + (aq)  +  IO 3 – (aq) .  What is the conjugate base of OH  ?  (a)  O 2  (b)  H 2 O  bufferd solution of HA in which 50% of the acid is ionized is  –  (c)  O  (d)  O 2–  (2005)  (a)  7. When the decomposition reaction reaches  15.  The exothermic formation of ClF 3 is represented by the equation:  19.17  (d)  0.  The  equilibrium  constants  K p 1  and  K P 2  for  the  reactions  (c)  ç ÷ P  (d)  ç ÷ P  (2006)  è x . less than or equal to K c depends  ç ÷ è ø  F upon the total gas pressure.0 × 10 –8 .9 × 10 –2 .612  (2005)  (c)  – 37.68 × 10 –6  (d)  3.1 ø  è 1 .4  will be  (a)  3.8 × 10 –2  (d)  4.059 V  (d)  whether K p is greater than.13  (2006)  26.  Among the following acids which has the lowest pK a  value?  (a)  1. (ii) and (iii). what is the mass of AgIO 3  contained  (c)  0.303 = 0.152 V  +  –  (a)  K p  is greater than K c  Ag ® Ag  + e  ; E° = – 0.0 × 10 –4  g  (b)  28.  In a saturated solution of the sparingly soluble strong electrolyte  equilibrium.0 × 10  (d)  5.  For the reaction.    When  K p  and  K c  are  18. respectively are in the ratio of  1  : 9.83  (d)  –16.  Hydrogen ion concentration in mol/L in a solution of pH = 5.  The pK a  of a weak acid (HA) is  4.11  (2005)  in 100 mL of its saturated solution?  22.  (i)  CO (g)  + H 2O  CO 2(g)  + H 2(g) ; K 1  PCl 5(g)  PCl 3(g)  + Cl 2(g)    (g)  CO (g)  + 3H 2(g) ; K 2  (ii)  CH 4(g)  + H 2 O (g)  If total pressure at equilibrium of the reaction mixture is P and  (iii)  CH 4(g)  + 2H 2 O (g)  CO 2(g)  + 4H 2(g) ; K 3  degree of dissociation of PCl 5  is x.3 × 10 –2  g  (a)  CH 3 COOH  (b)  (CH 3 ) 2 CH – COOH  (c)  2.0  (b)  4.K)).98 × 10 –6  (2005)  1.   37. The value of K c  (a)  s = (K sp /128) 1/4  (b)  s = (128K sp ) 1/4  for the reaction is  (c)  s = (256K sp ) 1/5  (d)  s = (K sp /256) 1/5  (2004)  (a)  3. the K p /K c  is equal  (a)  lowering of temperature as well as pressure  to  (b)  increasing temperature as well as pressure  (a)  1/RT  (b)  RT  (c)  lowering the temperature and increasing the pressure  (c)  (d)  1.  (b)  pH + pOH = 14 for all aqueous solutions. in  CuSO 4  solution deposits  1 gram equivalent of  copper at  –12  2+  the  cathode. The corresponding solubility product is K sp .  29.  (b)  3 × 10 –1  mol L –1  N 2(g)  + O 2(g)  2NO (g)  (c)  3 × 10 –3  mol L –1  –4  at temperature T is 4 × 10  .  The equilibrium constant for  the  reaction. s is given in  the  concentrations  of  N 2 O 4  and  NO 2  at  equilibrium  are  terms of K sp  by the relation  4.  Which one of the following statements  is not true?  –  2–  (a)  The conjugate base of H 2PO    4  is  HPO 4  .     (2005)  38.0 × 10 –4  M  and the corresponding equilibrium constant K c  is  (c)  1.  In which of the following reactions.500  coulombs  of  electricity  when  passed  through  a  (d)  SO 2 Cl 2(g)  2NH 3(g)  SO 2(g)  + Cl 2(g)  42. K p  /K c  is  (a)  RT  (b)  (RT) –1  –1/2  (c)  (RT)  (d)  (RT) 1/2  (2002)  (2002) .6 × 10 –4  M  (d)  4. increase in the volume at  [P4O10 ]  [P4O10 ]  constant  temperature  does  not  affect  the  number  of  moles  at  K =  K =  c  (a)  (b)  c  5[P ][O ]  [P4 ][O 2 ] 5  4 2  equilibrium?  1  K =  (a)  2NH 3 ® N 2  + 3H 2  5  (2004)  (c)  K c  = [O 2]  (d)  c  [O ] 5    2  (b)  C (g)  + (1/2) O 2(g) ® CO (g)  The conjugate base  of H 2 PO 4 –  is  (c)  H 2(g)  + O 2(g) ® H 2O    2(g)  (a)  PO 4 3–  (b)  P 2 O 5  (d)  None of  these.  What is the equilibrium expression for the reaction  (2003)  P 4  (s)  + 5O 2 (g)  P 4 O 10 (s) ?  40.  (d)  96.0 × 10 –6  M  (b)  1. In general  1.  Consider the reaction equilibrium:  (a)  (b)  50  2SO 2(g)  + O 2(g)  2SO 3(g)  ; DH ° = –198 kJ.  The solubility product of a salt having general formula MX 2 .  (2003)  water is  4 × 10  .  (c)  uninfluenced  by  occurrence  of  thunderstorm  (d)  which depends on the amount of dust in air.  33.  The  solubility  in  water  of  a  sparingly  soluble  salt  AB 2  is  respectively. CO (g)  + Cl 2(g)  COCl 2(g) .  The correct relationship between free energy change in a reaction  (a)  2.  34.  (2002)  2–  (c)  H 3 PO 4  (d)  HPO 4  (2004)  41.5 × 10 2  31.  Consider an  endothermic  reaction X ®  Y with the  activation  (c) DG° = RT ln K c  (d)  –DG° = RT ln K c  (2003)  energies  E b  and  E f  for  the  backward  and  forward  reactions.  For the reaction  CO (g)  + (1/2) O 2(g)  = CO 2(g) .0 × 10 –10  M  (2005)  (a) DG = RT ln K c  (b)  –DG = RT ln K c  28.  For the reaction equilibrium.30  JEE MAIN CHAPTERWISE EXPLORER  27.8 × 10 –2  and 1.3 × 10 2  mol L –1  30.0  (2004)  RT  (d)  any value of temperature and pressure.02  (2004)  for the forward reaction is  For the reaction.  32. The concentration of M  ions in the aqueous  solution of the salt is  36.  (c)  N 2(g)  + 3H 2(g)  (2003)  35.  Change in volume of the system does not alter the number of  When rain is accompanied by a thunderstorm.  On the basis of Le Chatelier's principle. the condition favourable  (c)  4 × 10 –4  (d)  0. The value of K c  for the reaction :  (d)  3 × 10 3  mol L –1  (2003)  NO (g)  1  1  N  +  O  at the same temperature is  2  2(g)  2  2(g)  39.0 × 10 –5  mol L –1 . the collected rain  moles in which of the following equilibria?  water will have a pH value  (a)  N 2(g)  + O 2(g)  2NO (g)  (a)  slightly lower than that of rain water without thunderstorm  (b)  PCl 5(g)  PCl 3(g)  + Cl 2(g)  (b)  slightly higher than that when the thunderstorm is not there  2.  (c)  The pH of 1 × 10 –8  M HCl is 8.2 × 10 –2  mol L –1  respectively.  The molar solubility (in mol L –1 ) of a sparingly soluble salt MX 4  N 2 O 4(g)  2NO 2(g)  is s. Its solubility product will be  (a)  E b  < E f  (b)  E b  > E f  (a)  4 × 10 –15  (b)  4 × 10 –10  (c)  E b  = E f  –15  (c)  1 × 10  (d)  1 × 10 –10  (2003)  (d)  there is no definite relation between E b and E f .   (d)  41.  8.  (a)  39.  (b)  23.  (a)  21.  (d)  17.  1 M NaCl and 1 M HCl are present in an aqueous solution.  (a)  26.  (2002)  (2002)  Answer  Key  1.  (d)  18.  (a)  29.  (d)  34.  (a)  35.  (d)  37.  (b)  (a)  4.  (d)  24.  (c)  (d)  3.  (d)  27.  (d)  33.31  Equilibrium  43.  (d)  6.  (a)  (c)  2.  (c)  44.  (d)  14.  (d)  22.  (d)  30.  (a)  20.  Let the solubility of an aqueous solution of Mg(OH) 2  be x then  its K sp  is  (a)  4x 3  (b)  108x 5  (c)  27x 4  (d)  9x  (2002)  44.  (a)  10.  (d)  19.  (c)  11.  7.  (b)  28.  Species acting as both Bronsted acid and base is  –1  (a)  (HSO 4)  (b)  Na 2CO      3  (c)  NH 3  (d)  OH –1  45. The  solution is  (a)  not a  buffer solution with pH < 7  (b)  not a  buffer solution with pH > 7  (c)  a buffer solution with pH < 7  (d)  a buffer solution with pH > 7.  (a)  32.  (b)  15.  (c)  16.  (c)  36.  (a) 40.  9.  (c)  .  (b)  5.  (c)  43.  (b)  12.  (a)  42.  (d)  13.  (c)  45.  (b)  31.  (d)  25.  (a)  38.  (ii)  1 ´ 10 -11  = 8.1 × 10 –9 \  5. (i)  .  KBr + AgNO 3  AgBr + KNO 3  Given.  nKBr  VSolution (L) Þ  w KBr  = 1 × 10 –11  × 120 = 120 × 10 –11  NO (g)  4. so H  CO  ionises more than HCO –  and hence. .  As  sp 3  carbon  is  less  electronegative than sp 3  nitrogen.5 atm  0.  – HSO 4  ®  1..  [H + ] =  K a C contribution of H +  is mostly due to ionisation of carbonic acid.  = 1 ´ 10 -11 M  [K + ] = [Br – ] = [KBr] Molarity  =  By  reversing the equation (ii).5 - 0.1 × 10 –5  M  Thus.  JEE MAIN CHAPTERWISE EXPLORER  (a)  : Initial concentration of aq.05 M –  Þ  [Ag + ] = [NO 3 ] = 0. (K sp ) AgBr  = 5.  (c) : N 2(g)  + O 2(g)  By  multiplying the equation (i) by  1  1  N  +  O  2  2(g)  2  2(g)  .]2 = Þ  –  10 -11  10 -3  = 10 -8  [OH  ] = 10 –4  9. K c  = 4 × 10 –4  2.3) 2  (c) : (K sp ) Mg(OH) 2  = [Mg 2+ ][OH – ] 2  –  1 × 10 –11  = [0. The  pK a  value  of other  species  are  given  below : –  HCO –3  ®  10. HCl solution with pH 1 = 10 –1  M  Final concentration of  this solution  after dilution =  10 –2  M  MV = M 1 (V 1  + V 2 )  10 –1  × 1 = 10 –2  (1 + V 2)    7. Its acidic strength is greater than    any given  species.5 – P +  2P =  0.  K  >> K  . H 2 PO 4  acts as a proton donor and thus.05 × [Br – ] = 5 × 10 –13  .25  (b) : In equation (ii).1 × 10 –9  = [Ba 2+ ] × [10 –4 ] Þ  [Ba 2+ ] = 5.  (b) : K sp  for BaCO 3  = [Ba 2+ ][CO 2– 3  ]  –4  given.  –  thus  the  concentrations  of  H +  and  HCO 3  are  approximately  equal. [CO 2–  3  ] = 1 × 10  M     (from Na 2 CO 3 )  K sp  = 5.  0.P ) (0.. wt.1  = 1 + V2  0.  –  H +  + HCO 3 ; K 1  = 4.  2  2  3  3  (a) : The order of acidity can be explained on the basis of the  acidity of the acids of the given conjugate base. RCOO –  is the weakest base.8  P = 0.5 .  + H 3O  ®  –1.  5 ´ 10 -2  \  [KBr] = 1 × 10 –11  M  K c¢  = K c = 4 ´ 10 -4 = 2 ´ 10 -2  1  1  N  +  O  2  2(g)  2  2(g)  1 1  K c¢¢ = = = 50.. . Due to  sp hybridised carbon.001][OH  ] 2  Þ [OH . BaCO 3 precipitate will begin  to  form.1 \  1  or 10 -6  = K a ´ 0. of KBr = 120)  1 w KBr  = 1.  (d) : HSO 3F is the super acid.0  K c¢ 2 ´ 10 -2  5 ´ 10 -13  Þ [Br .5 – P  2P  Total pressure = 0.  Since  RCOOH  is  the  strongest acid amongst all.  10 -3 = K a ´ 0.1 K a  = 10 –5  (a) : CO 2 (g)  + C (s)  2CO (g)  0.2 × 10 –9  g  H +  = 10 –pH  = 10 –3  5. acetylene is also acidic and hence a weak  base  but  stronger  than  RCOO – .05 M  –  [Ag + ][Br  ] = (K sp ) AgBr Þ  0. when [Ba 2+ ] = 5. Stronger is the  acid. we  get  3.  10.01  10 = 1 + V 2 Þ  V 2  = 9 L  2NO (g) .  w KBr  / 120  (Mol.  pH = 14 – 4 = 10  6. R –  is more basic than NH 2–  .8 × 10 –11 1  .6) 2  K P = CO  = = = 1..2 × 10 –7  (d) : H 2 CO 3  2–  –  (b) : pH = 3  Molarity  = 0.8 atm  PCO  (0.  weaker  is  the  conjugate  base.] = 1  2  NO (g)  (c) : Given. [AgNO 3 ] = 0. .1 M  HCO 3  H +  + CO 3  ; K 2  = 4.92 pOH = 4  Thus.  11.3  P 2  (2 P )2 (0. .  .74    acts as an acid.0 × 10 –13  The required  equation is.32  1.1 × 10 –5 M. 5 +  x             x  Total pressure = 0..8 and  pK b  = 4. Hence.5  0  At equi...5 = 9..33  Equilibrium  Lesser the pK a  value.0 ´ 10 –4  ´ 283 ´ 100  g/100mL  1000 = 28..952 =  log K sp  n 1  0.(ii)  1  1 .e.059  E °cell  =  log K  i.78 \  pH = 7 + 1/2 (pK a  –  pK b )  = 7 + 1/2 (4. (ii)  K 1  =  CH 4(g)  + 2H 2 O (g)  ‡ˆˆ  ˆˆ† CO 2(g)  + 4H 2(g)  K3  =  [CO 2 ][H 2 ] 4  .5 + 2x = 0. (ii) and (iii) ; K 3  =  K 1  ×  K 2  14.  (d) : For acidic buffer.a 2  K P  1  Given is  1  =  .a 2  = 1  Þ 4 P1 = 1 Þ P 1  =  1  P2 9 P2  36  a 2  P2  9  1 ..  -0.78) = 7. (i)  [CO][H 2 O]  CH 4(g)  + H 2 O (g)  ‡ˆˆ  ˆˆ† CO (g)  + 3H 2(g)  K 2  =  [CO][H 2 ]  [CH 4 ][H 2 O]  . .  pH =  pK a  + log  13..01  1.(iii)  K P  9  [H ] [A  ]  = K1× K 2 = 1×10 –5 × 5 ×10 –10 = 5 ×10 –15  [H 2 A ]  = 5 × 10 –15  Ag +  + IO 3 –  [S = Solubility]  S  S  = S 2  K sp  or.800 V  AgI (s) ® Ag +  + I – ..  (i)  1  1 . E° = – 0...a ö ç 1 + a P2 ÷ è ø  4 a 2  P 1  Þ K P  = .0 × 10 –4  ×  283   g/L  =  1.17) = 0.9 ´ 4.059  0.5 \ pH = 4.952 V  0.5  16.84 \ x = 0.9 ´ 10 -2  For 2SO 2(g)  + O 2(g)  ƒ  2SO 3(g)  [SO 3 ] 2  2 [SO 2 ] [O 2 ] 2  3  [HA – ]  2–  [SO 2 ][O 2 ] 1/ 2  = K c  = 4.a 2  4 a 2  P 1  Þ K P  = .  (d) :  NH 4 HS  ( s ) ƒ  NH 3   ( g ) + H 2S ( g )  Initial  pressure            0                     0.9 ´ 10 -4  10000  = =  416.  (b) : Higher the pK a  value.                     0  0.a 2  [H + ][A2– ]  18.17)(0. S = 1... strongest  acid has lowest pK a  value. E° = – 0.5 \  pOH = 14 – 4.0 ×  10 –4  mol/L  = 1.  (c) :  AgIO  H +  + A 2– ;  K 2  = 5 × 10 –10  =  H A –  From equations (i).3 × 10 –4  g/100 mL  = 2.  (i)  SO 3(g)  + 1/2 O 2(g)  ƒ  SO 3(g)  [SO3 ] [SO 2 ][O 2 ]1/ 2 = K c ¢ = 1  .0 × 10 –8  or.49  24....  (b) :  [H + ][H A – ]  = 1 × 10 –5  [H 2  A ]  + 2 [CH 4 ][H 2 O] 2  H +  + H A –  ;  17..83 × 10 –3  g/100 mL  =  [A – ]  [HA ]  When the acid is 50% ionised.01 (a) : Given PCl 5 (g)  ƒ  PCl 3 (g)  + Cl 2 (g)  t = 0                   1                    0            0  t eq  1 – x                x             x  Total number of moles = 1 – x + x + x  = 1 + x  æ x  ö ÷ P  Thus partial pressure of PCl 3  =  çè 1 +  x ø 21. (ii)  4.                1  At  equilibrium          1  –  a  0  2a  1  1–a  0      0  a        a  2  KP æ 2 a ö ç 1 + a P 1 ÷ è ø = = PX æ 1 .9 ´ 10 -2  [SO 3 ]  Substituting values of from equation (i) and (ii) into (iii). Hence  sequence of acidic strength  will be  +  > HSO  –  > HCO –  HSO 3F > H    3O    4  3  12. we  get  20..  (d) : Given that  pK a  =  4.  (d) :  H 2 A  3  = K c ¢ 2  = 1  4.0 ´ 10 – 4  ´ 283  g/L  1000 . weaker is the acid. (iv)  ˆˆ† 2 Y  ; Z ‡ˆˆ  ˆˆ† P + Q  X ‡ˆˆ  Initial  mol.135  or.11 atm 2  22.059  19. S 2  = 1..80 – 4.  (d) : AgI (s)  + e – ® Ag (s)  + I – . higher will be its acidic strength. E° = – 0.952  = - 16.  4 a 2  P1  1 . [A – ] = [HA]  or  pH = pK a  + log1    or   pH = pK a  Given pK a  = 4.  log K sp  = 0.5 + 0..a ö ç 1 + a P1 ÷ è ø  P Y 2  1 K= K P  = 2  P Z  æ a öæ a ö ç 1 + a P2 ÷ ç 1 + a P 2 ÷ øè ø = è K P  =  2  PZ  æ 1 .152 V  Ag (s) ® Ag +  + e – .  (d) : CO (g)  + H 2 O (g)  ‡ˆˆ  ˆˆ† CO 2(g)  + H 2(g)  [CO2 ][H 2 ]  K1  =  .  (a) : SO 3(g)  ƒ  SO 2(g)  + 1/2 O 2(g)  PP P Q  15.17 atm  K p  = p NH 3  × p H 2 S  = (0.   (ii)  Addition of reactants  or  removal  of  product  will farour  37.  (c) : The conversion of SO 2  to SO 3  is an exothermic reaction.3 × 10 –2  = 3 × 10 –3  mol L –1  39.2 ´ 10-2  mol L-1  K c  = [NO 2 ] 2  1. K sp  = (x) × (2x) 2  = x × 4x 2  = 4x 3  44.  (c) :  [N 2O4 ] = 4. As temperature  increases.  1  3 1  42.  (d) : M X 4  (solid)  1/ 5  \  s = 5  K sp æ K sp ö = 256 çè 256 ÷ø  30.303 RT logK c  Favourable conditions:  At equilibrium.2 ´ 10 -2  = [N 2 O 4 ]  4.= è 2ø 2 2 K p  .  – 2–  +  H 2PO    4  ® HPO 4  + H  24.  (a) : Mg(OH) 2 ® [Mg 2+ ] + 2[OH – ]  x  2x  K sp  = [Mg 2+ ] [OH – ] 2  or.  [P4 ( s ) ] [O 2 ( g ) ] 5  –  + (HSO 4)    + H  ®  H 2SO    4  We  know  that  concentration  of  a  solid  component  is  always  2–  –  + (HSO 4)    – H  ®  SO 4  taken as unity. Here we should also consider  [H + ] that comes from H 2 O.  (a) : HCl is a strong acid and its salt do not form bufter solution.  K c  =  1  [O 2 ]5  45.  (a) : Due to thunderstorm.æ1 + ö = 1 . increase in volume at  constant  temperature  does  not  affect  the  number  of  moles  at  equilibrium.e.  (c) : K p  = K c  (RT)Dn  ; Dn =  1 . hence pH decreases.  (a) : For endothermic reaction.0831 × 457) 1 \  K p  > K c  35.  As the resultant solution is acidic.  S = 1.  s = 1 × 10 –4  M Þ  [M 2 + ] = 1 × 10 –4  M  28. . DH = +ve DH = E f  – E b . where Dn = 0.34  JEE MAIN CHAPTERWISE EXPLORER  23.  = 1.  K c  =  (HSO 4 ) –  can accept and donate a proton.  OH –  ƒ  O 2–  +  H +  33.  (a) : According to Bronsted­Lowry concept.8 ´ 10 .  (d) : Conjugate base of OH –  is O 2– .  hence decrease the temperature will favour the forward reaction.  (d) : Cl 2(g)  + 3F 2(g)  ƒ  2ClF 3(g) ; DH = –329 kJ  36. (a) : CO (g)  + Cl 2(g)  Dn = 1 – 2 = –1  COCl 2(g) K p  1  -1  K p  = K c (RT)Dn .  hence  decrease  in  DG° = –2. [H + ] also increases. –ve) hence decrease in volume  K sp  = [A 2+ ] [B – ] 2  = 1.  (d) : pH = – log[H + ]  [H + ] = antilog (–pH) = antilog(–5. it means E b  < E f  M 4+  (aq)  + 4X –  (aq)  s  4s  Solubility product.  40.  (d) : P 4 (s)  + 5O 2 (g)  P 4O    10 (s)  and a Bronsted base is a substance which can accept a proton  [P4O10 ( s ) ]  from any other substance.  (d) : Conjugate base is formed by the removal of H +  from acid.  s 3  =  1 ×  10 –12  or.98 × 10 –6  34.  (d) : DG = DG° + 2.  (b) : N 2(g)  + O 2(g)  2NO (g)  2  [NO]  K c  = = 4 ´ 10 -4  [N 2 ] [O 2 ]  NO (g)  1  1  N  +  O  2  2(g)  2  2(g)  [N 2 ]1/ 2  [O 2 ] 1/2  1  = 1 = [NO]  Kc  4 ´ 10 -4  1 100  = = = 50  2 ´ 10 -2  2  K c ¢ = 31. K sp  = s × (4s) 4  = 256 s 5.  (a) : In this reaction the ratio of number of moles of reactants  to products is same i. hence pH is less than 7.4) =3.  There is also  a  decrease in volume or moles  in product side.0 × 10 –5 ) 2  or increase in pressure will favour the forward reaction. 2 : 2. temperature increases.0 × 10 –15  27.0 × 10 –5  mol L –1  (iii)  Here Dn = 2 – 4 = –2 (i. hence change in volume will not  alter the number of moles.  41.e.  Now [H + ] = [H + ] from HCl  + [H + ] from  H 2 O  = 10 –8  + 10 –7  = 10 –8  + 10 × 10 –8  = 11 × 10 –8 \  pH = –log(11 × 10 –8 ) = 6.1/ 2  \  K =  ( RT )  c  43.9587  26.  (Le­Chatelier's principle).  (a) : K p  = K c (RT)Dn Dn = 3 – 2 = 1  K p  = K c  (0.  (c) : AB 2  A 2+  + 2B –  the  forward reaction.  (b) : MX 2(s)  ƒ  M 2 + (aq)  + 2X – (aq)  s  2s  K sp  = s ∙ (2s) 2  = 4s 3  4 × 10 –12  = 4s 3  or. \  K = ( RT )  =  RT c  38..8 ´ 10-2  mol L-1  [NO 2 ] = 1. DG = 0 (i)  As  the  reaction  is  exothermic.  Thus  the  reaction  is  favoured  by  low  temperature  and  high  pressure.  (d) : For those reactions.  25.303 RT logK c  temperature  will favour  the forward  reaction.0 × 10 –5  × (1.  (c) : pH of an acid cannot exceed 7. 29. a Bronsted acid is  a substance which can donate a proton to any other substance  32.2 ´ 10 -2 ´ 1.2  = 0. 0  M  (b)  p(H 2) = 1 atm and [H    25°C are given below: + ] = 1.770 V  (d)  – 0.72 V.65 × 10 4  (c)  37.76.3  The reduction potential of hydrogen half­cell will be negative  (2007)  if  10.  The standard reduction potentials for Zn 2+ /Zn. respectively  (a)  5.  E° Fe 2+ /Fe  = –0.10 V)  was allowed to be completely discharged at 298 K.3  (b)  antilog(24.  of this solution is 1.5 V  (2007)  (c)  3. Ni 2+ /Ni.24 × 10 –4  S m 2  mol –1  The titration gives unsatisfactory result when carried out in  (d)  12.0 S cm 2 /equiv.2  M  of  the  same  solution  is  520 W.26 V  (b)  0.  5. D r G = + 966 kJ mol –1 .29 S m –1 . Resistance of the same cell when  Fe 3+ (aq)  + e – ®  Fe 2+ (aq)  will be  filled  with  0.0 V  (d)  2.42 V  The  potential  for  the  cell  Cr | Cr 3+ (0.51 V  E° Cr 2 O 7 2– /Cr 3+  = 1. 2 and 16  (c)  2.  4.  The Gibb’s energy for the decomposition of Al 2 O 3  at 500°C  What additional information/quantity one needs to calculate L°  is as follows :  of an  aqueous solution of acetic acid?  4/3Al + O 2 .036 V. The  reaction X +  Y 2+ ®  X 2+  +  Y will  be spontaneous  when  (a)  X = Ni.1 M is 100 W. 5 and 16  (2013)  2.  The  molar  (a)  – 0.01 M) | Fe  is  (a)  –0. The  conductivity  of  standard  electrode  potential  for  the  change.74 V;  E° MnO 4 – /Mn 2+  =  1.0  M  dilution in H 2 O (where ions move freely through a solution) at  + ] = 1. 2 and 8  (b)  5.35  Redox Reactions and Electrochemistry  CHAPTER  8  1.385 V  conductivity of 0.44  V  respectively.072 V  (b)  0.  2/3Al 2 O 3  (a) L° of chloroacetic acid (ClCH 2 COOH)  The potential difference needed for electrolytic reduction of  (b) L° of NaCl  (c) L° of CH 3 COOK  Al 2 O 3  at 500°C is at least  (d)  The limiting equivalent conductance of H +  (l° H +).  The equivalent conductances of two strong electrolytes at infinite  (a)  p(H 2 ) = 1 atm and [H + ] = 2.  Resistance  of  a  conductivity  cell  filled  with  a  solution  of  an  electrolyte of concentration 0. y  and  z  in  the reaction are. strongest oxidising agent will  be (a)  MnO 4 –  (b)  Cl –  9.08)  (d)  10 37.  Y = Fe  (2012)  oxidises oxalic  acid to carbon dioxide  and  water  gets oxidised by oxalic  acid to  chlorine  furnishes H +  ions in addition to those from oxalic acid  reduces permanganate to Mn 2+ . and  Fe 2+ /Fe  are  –  0.2 Scm 2 /equiv.  Zn | Zn 2+  (1 M) || Cu 2+  (1 M) | Cu (E° cell  = 1. + ] = 2.  3+  (c)  Cr  (d)  Mn 2+  (2013)  3. Y = Zn (b)  X = Fe.36 V  Based on the data given above.4 × 10 –4  S m 2  mol –1  (2006)  the  presence  of HCl. E° Fe 2+ /Fe  = – 0.  (2008)  Given E° Cr 3+ /Cr  = –0. The relative  æ [Zn 2 + ] ö concentration of Zn 2+  to  Cu 2+  ç 2+  ÷ is  è [Cu ] ø  (a)  9.33 V;  E° Cl/Cl –  =  1.0  M  (d)  p(H 2) = 2 atm and [H  (2011)    L° HCl  =  426.0 V  (b)  4.  (a)  5. The value  11. Y = Zn  (c)  X = Zn.  Given  E° Cr 3+ /Cr  = – 0.0  M  (c)  p(H 2) = 2 atm and [H    L° CH 3 COONa  = 91.  7.5 V  (2010)  Given : E° Fe 3+ /Fe  = – 0.  8.26 V  (c)  0.439 V.  –  0.339 V  (d)  –0. Y = Ni (d)  X = Ni.270 V  (2009)  (a)  124 × 10 –4  S m 2  mol –1  Amount of oxalic acid present in a solution can be determined  (b)  1240 × 10 –4  S m 2  mol –1  by its titration with KMnO 4 solution in the presence of H 2 SO 4 .23  and  –  0.  (c)  1. 5 and 8  (d)  2.  (a)  (b)  (c)  (d)  Consider  the  following  reaction.02 M solution of the electrolyte  will be  (c)  0.  because  HCl  .  REDOX REACTIONS  AND ELECTROCHEMISTRY xMnO 4 –  + yC 2 O 4 2–  + zH + ® xMn 2 + z  + 2 yCO2 +  H 2 O  2  The  values of x.339 V  (2008)  The  cell.1 M) | | Fe 2+ (0.  6. 12 kg of aluminium metal by this method would  (C) complexing.f.2 V  respectively.  (c)  10  (d)  1 × 10 10  (2003)  0.0 × 10 5  10  (b)  Ca(OH) 2  + H 2SO  ® CaSO  + 2H  O  (c)  1.  For the redox reaction:  the change in oxidation state from +2 to +3 is easiest?  Zn (s)  + Cu 2+  (0.36  JEE MAIN CHAPTERWISE EXPLORER  12. A  (d)  A.  (d)  2PCl 5  + H 2SO    4 ® 2POCl 3  + 2HCl + SO 2 Cl 2 .0 × 10 1  (b)  1. B  (b)  B.82 V  (2003)  addition of  H 2SO    4  to  cathode compartment.m. For which one of these metals  26.  Aluminium  oxide  may  be  electrolysed  at  1000°C  to  furnish  (d)  remove  adsorbed oxygen  from electrode  surface. The equilibrium  constant of the reaction at 25°C will be  (2005)  (a)  1 × 10 –10  (b)  29.  (c)  21.1 M chloroacetic acid  (c)  0.77 and +1. Fe and Co are – 0.2  (b)  552.1 M fluoroacetic  acid  (d)  0.  Consider the following E° values.0  126.0 × 10  (d)  1.  When during electrolysis of a solution of AgNO 3 .80 V  Zn (s)  + 2H  (aq) ® Zn  (aq)  + H 2(g)  (c)  1.63 V  (2004)  molar conductances  of the electrolytes  listed above  at infinite  22.08 g  (b)  10. the set of properties shown by CN –  ion towards  require  metal species is  7  (a)  A. The L° for NaBr is  calculate L° HOAc .10 volt.m. R = 8.5  (b)  1.0 V  and  –1.83 × 10 7  C of  electricity  (c)  5. E cell  for the cell will be (c)  Fe  (d)  Co  (2004)  2.41.  Mass  =  27  amu;  1  Faraday  =  96.314 JK –1 mol –1 )  (a)  2HI + H 2 SO 4 ® I 2  + SO 2  + 2H 2 O  (a)  1. the mass of silver  deposited on the cathode will be  (c)  increase the E and shift the equilibrium to the right  (a)  1.57.  The  E ° M 3 + / M 2 + values for Cr.14 V  (b)  1.2 S cm 2 /mol respectively.  Standard reduction electrode potentials of three metals A.  Among  the  properties  (A)  reducing  (B)  oxidising  To prepare 5.2  91. E° cell  is 1. of a cell.  Which of the following chemical reactions depicts the oxidising  found to be 0.  In a cell that utilizes the reaction.0591  F 18.0  426.f.40 V  (a)  1. +1.  E° 3+ 2+  = + 0. The equilibrium constant of the  behaviour  of  H 2 SO 4 ?  reaction is (F = 96.  For a cell reaction involving a two­electron change.  The limiting molar conductivities L° for NaCl.  –3.  to  (a)  517. the standard  e.1 M) ®  Zn + (1M) + Cu (s)  (a)  Cr  (b)  Mn  taking place in a cell.5 × 10 –2  17.7  (d)  217.  The molar conductivities L° NaOAc  and L° HCl  at  infinite dilution  19. B and  (d)  5.  aluminium  metal  (At. combustion of hydrogen occurs  dilution in H 2 O at 25°C. C  (a)  5.9  145.  25.  Electrolyte  KCl  KNO 3  HCl  NaOAc  NaCl  Sn (s)  + 2Fe 3+ (aq) ® 2Fe 2+ (aq)  + Sn 2+ (aq)  is  (S  cm 2  mol –1 )  149.5  (2005)  (b)  create potential  difference between  the two  electrodes  (c)  produce high purity water  15.91 V  (d)  0.5 V. the additional value required is  (a)  128 S cm 2  mol –1  (b)  176 S cm 2  mol –1  2  –1  (a) L° H 2 O  (b) L° KCl  (c)  278 S cm  mol  (d)  302 S cm 2  mol –1  (2004)  (c) L° NaOH  (d) L° NaCl.97 V respectively.  +  2+  (a)  2. To  126.49 × 10 10  C of  electricity  (2005)  C  are  +0.591 V at 25°C.500 C mol –1 .  (2004)  (b)  1. The  reducing  power of these metals are  16.  (2006)  20. The cathode  reaction is  3+  0  Al  + 3e ® Al  23. will  (a)  lower the E and shift equilibrium to  the left  27.6 g  (d)  108 g  (2003) (2004)  ( )  . involving one electron change is  13. C.1 M acetic acid  (b)  0.303 RT  = 0.49 × 10  C of  electricity  (c)  C. Mn.500  (2004)  Coulombs). B.14 V  Fe /Fe  Sn /Sn  (2006)  Under standard conditions the potential for the reaction  14.7  (a)  generate heat  (c)  390. 9650 coulombs  (b)  lower the E and shift the equilibrium to the right  of charge pass through the electroplating bath.295 V at 25°C.0 and 426. of the cell is found to be 0. KBr and KCl are  in water at 25°C are 91.8 g  (d)  increase the E and shift the equilibrium to the left.0 × 10 30  (2004)    4 4  2  (c)  NaCl + H 2SO    4 ® NaHSO 4  + HCl  21.68 V  Calculate molar conductance  of  acetic  acid using appropriate  (c)  0.  The standard e.49 × 10 4  C of  electricity  24.07 V  (d)  0. 152 and 150 S cm 2  mol –1  respectively.  The  highest  electrical  conductivity  of  the  following  aqueous  (a)  B > C > A  (b)  A > B > C  solutions is of  (c)  C > B > A  (d)  A > C > B  (2003)  (a)  0.77 V ;  E° 2+  = - 0.1 M difluoroacetic  acid.  In a hydrogen­oxygen fuel cell.  3.  (2002)  29. 5.  What will be the emf for the  given cell. 7  (c)  1.  (c)  F P 1  33.  When KMnO 4  acts as an oxidising agent and ultimately forms  [MnO 4 ] –1 .37  Redox Reactions and Electrochemistry  28. 7. 5  (d)  3.  32.  26.  27. 4.  MnO 2 .  21.  (c)  (d)  (a)  (c)  (b)  (d)  4.  31.  24. 3.  EMF of a cell in terms of reduction potential of its left and right  electrodes is  (a)  E = E left  – E right  (b)  E = E left  + E right  (c)  E = E right  – E left  (d)  E = –(E right  + E left )  (2002)  Answer  Key  1.  (d)  (d)  (a)  (a)  (d)  (a)  2.  16.  22.  12.  If f denotes reduction potential. 3.  (2002)  30.  28.  11.  23.  13.  14.  33.  20.  The heat  required to  raise  the  temperature  of  body  by  1ºC is  32.  Which of the following  reaction is  possible at  anode?  (a)  2 Cr 3+  + 7H 2 O ® Cr 2 O 7 2–  + 14H +  (b)  F 2 ® 2F –  (c)  (1/2) O 2  + 2H + ® H 2 O  (d)  None of  these. 5  (b)  1.  Pt | H 2  (P 1)  | H + (aq)  | | H 2  (P 2)  | Pt  RT  P 1  P  (b)  2 F log P  (a)  RT log  1  2  F P 2  RT log P 2  (d)  none  of  these.  15.  30.  25. 1  (2002)  35.  Mn 2 O 3 .  8.  19.  17.  (d)  (a)  (a)  (c)  (a)  (c) 6. then which  is true?  (a)  E° cell  = f right  – f left  (b)  E° cell  = f left  + f right  (c)  E° cell  = f left  – f right  (d)  E° cell  = –(f left  + f right )  (2002)  (a)  4. 5.  Which of the following is a redox reaction?  (a)  NaCl + KNO 3 ® NaNO 3  + KCl  (b)  CaC 2 O 4  + 2HCl ® CaCl 2  + H 2 C 2 O 4  (c)  Mg(OH) 2  + 2NH 4 Cl ® MgCl 2  + 2NH 4 OH  (d)  Zn + 2AgCN ® 2Ag + Zn(CN) 2  34.  Conductivity (unit Siemen's S) is directly proportional to area  of the vessel and the concentration of the solution in it and is  called  inversely proportional to the length of the vessel then the unit  (a)  specific  heat  of the constant  of proportionality is  (b)  thermal capacity  (a)  S m mol –1  (b)  S m 2  mol –1  (c)  water equivalent  –2  2  (c)  S  m  mol  (d)  S 2  m 2  mol –2  (2002)  (d)  none  of  these.  9.  35.  (a)  (b)  (c)  (c)  (c)  (b)  (2002)  3.  10.  18.  (c)  (d)  (c)  (a)  (b)  .  7.  34.  Mn 2+  then  the  number  of  electrons  transferred in each case respectively is  (2002)  31.  (c)  (b)  (d)  (b)  (b)  (c)  5.  29. 1. 059 [Zn 2+ ]  log  2  [Cu 2+ ]  When the cell is completely discharged.1  Zn 2+ or.1 – or  log  0.  is  the oxidising agent.2.3 - 0.  (d)  : The  ionic  reactions  are  :  –  4/3Al  2/3Al 23+    + 4e  2–  2/3O 3  O 2  + 4e –  Thus.  (b) : Cr ®  Cr 3+  + 3e –  E° red  = –0.  3.3  Cu 2+  10.  8.  E red  will only be negative when p H2  > [H + ]. CH 3 COOH is  or  Λ°CH3COOH = Λ°CH3 COONa + Λ°HCl -  Λ °NaCl  So.23 – (– 0. E cell  = 0  °  – Ecell = E cell  0 = 1.  Y = Ni  [Cr3+ ] 2  Zn + Ni 2+ ®  Zn 2+  + Ni  Ecell = E °cell - 0.3  0.059 log10 10 4  6  p H 2  0.059 log 10  2 + 3  ncell  [Fe ]  Alternatively.76) 6  (0. of electrons transferred  =  4 =  n DG = –nFE = – 4 × 96500 ×  E  or  966 × 10 3  = – 4 × 96500 ×  E  Þ 6. value of  Λ ° NaCl  should also be known..  0.1) 2  Ecell = 0.  (b) : According to Kohlrausch’s law.  4.059 [Zn 2+ ]  log  2  [Cu 2+ ]  [Zn 2+ ] 2+  [Cu ]  =  2 × 1.  According to  Nernst equation. So option (c) is  correct.  JEE MAIN CHAPTERWISE EXPLORER  + 2+  Titration cannot be done in the presence of HCl because KMnO 4  (d)  : 2MnO 4–   + 5C 2 O 42–    + 16H  ® 2Mn  + 10CO 2  + 8H 2 O being a  strong oxidizing agent oxidises HCl to Cl 2  and  get  \  x = 2.72)  Thus X should have high negative value of standard potential  E° cell  = 0.  COOH  2KMnO4 + 3H2SO4  + 5  COOH  K2SO4 + 2MnSO4  + 10CO2  + 5H2 O  9.42 V  2+ 3+  reduction potential are good reducing agents and can be easily  2Cr + 3Fe  ® 2Cr  + 3Fe  oxidised.01) 3  Þ  E° = + 0. for a spontaneous reaction E° must be positive.039 Ered = Ered  ° -  log  n [H + ]2  \  E c ell  = 0.38  1.  7.e. no.  Zn 2+  + Cu  2+  (d) :  Zn   +  Cu  0.3 -  0. \  MnO 4 –  is the  strongest oxidising agent.  X = Zn. the molar conductivity  at infinite dilution (L°) for weak electrolyte.5 V  4 ´ 96500 (c)  :  Given.036) – 2  × (–0.0591  E c ell  = 0.  E° = E° r educed species  – E° oxidised species  (0.  (a) :  k = 1 æ l  ö 1  æ l  ö ç ÷ i.059 log 10  = – 0.02 M  lm = k ´ 1000 1 ´ 129 1000  = ´ ´ 10-6  m 3  molarity 520 0.036 V  Fe 2+  + 2e –  Fe  ;  E°  2  = –  0.439)  E° 3  = – 0.  log = 37.261 V  Ered  = 0 -  5.77 V  (d) : Oxalic acid present in a solution can be determined by  its titration with KMnO 4  solution in the presence of H 2 SO 4 .  E° cell  =  E° c athode  –  E° a node  = –0.  Fe 3+  + 3e –  Fe  ;  E°  1  = –  0.  1.3  than Y so that it will be oxidised to X 2+  by reducing Y 2+  to Y.29 = ç ÷ R è a ø  100 è a ø  l/a  = 129 m –1  R = 520 W  for 0.439 V  Required equation  is  Fe 3+  + e –  Fe 2+  ;  E°  3  = ?  Applying DG° = –nFE° \ DG°  3  = D G°  1  – D G°  2  (–n 3 FE°  3 ) =  (–n 1 FE°  1 )  –  (–n 2 FE°  2 )  E°  3  = 3E°  1  – 2E°  2  = 3 × (–  0.059  Cu 2+  2+  Zn  = 10 37.3 – 0.108 + 0.  11. So  actual  amount  of oxalic  acid  in  (a)  : Greater the reduction potential of a substance.878 =  0.  2.53 V  H 2  (g)  (c)  : 2H +  (aq)  + 2e –  Ecell = 0.02  = 124 × 10 –4  S m 2  mol –1 . stronger  solution  cannot be  determined.72 V  2+  + 2e – ®  Fe  Fe  E°  (c)  :  The  elements  with  high  negative  value  of  standard  red  = –0.  C  = 0. z =  16  itself  reduced  to Mn 2+ . y = 5. for calculating the value of  Λ ° CH3 COOH  .0591 2  log  2  (1)2  E=- 966 ´ 10 3  = .2 M.42 – (–0.   thus this is a reduction reaction and H 2SO    4 behaves as oxidising  \  K c  = antilog 10 = 1 × 10 10  agent.10 V  its oxidation will be  easiest. 13..12 ´ 103  ´ 96500  7  E °  +0.295 =  log Kc  Mn 2+  | Mn 3+  = –1.41 V  0.0591 log10  2  E cell  = 1.0591  26.  Now  E = Z × F  [E = equivalent weight . more is the tendency  16.  L°CH3COONa + L°HCl = L°CH3 COOH + L° NaCl or.  (d) :  E °cell  =  c  I st  half reaction : 2HI ®  I 2  –1          0  n 17. therefore  0.0591  0.e.14 + 0. Furthermore.0591  0 = 0..0705 V .  (c) : L¥ AcOH  = L¥ AcONa  + L¥ HCl  – L¥ NaCl  Sn/Sn 2+ Fe /Fe  = 91.  The reducing power follows the following order:  Difluoroacetic  acid  will  be  strongest  acid  due  to  electron  B > C > A.  2 H I   +   H 2 S O 4 ®  I 2  +   S O 2  +   2 H 2 O  +1–1         +1  +6 –2               0          +4                   +1   –2  +  On adding H 2SO    4  the  [H  ] will increase  therefore E cell  will  also increase and the equilibrium will shift towards the right.e.  19.0  + 426.10 - 0. (iii)  Br –  Cl –  Equation (i) + (ii) – (iii)  L°NaBr  = L° + + L°  Na Br –  = 126 + 152 – 150 = 128 S cm 2  mol –1  0. F = Faraday]  the water obtained as a byproduct may be used for drinking by  E  or  W = ´ Q  the astronauts..  0.5  = 0.  (a) :  A  B  C  3 ´ 5.  (c) :  Ecell = E °cell  -  n log K c  In this reaction oxidation number of I increases by one.  (c) :  E °cell  = E ° + E °  3+ 2+  14.0295 = 1.  (a) :  From Faraday's 1st law. (ii)  + L°  .  0.L° NaCl Thus to calculate the value of  L° CH3 COOH one should know  the  value of  L° NaCl along  with  L° CH3 COONa and  L° HCl .0591 log K  maximum electrical  conductivity.57 V  2  2+  3+  Fe  | Fe  = – 0.2  – 126.0591  20.  (a) : Cr 2+  | Cr 3+  = +0.  21..  K c  = antilog 10   or  K c  = 1 × 10 10  More is the value of oxidation potential more is the tendency  1  to get  oxidised.  (c) :  Ecell = E °cell  -  n log  As Cr will have maximum oxidation potential value.  L°CH3COOH = L°CH3 COONa + L°HCl .  (b) :  Direct  conversion  of  chemical  energy  to  electric  energy  15.77 = 0.  Q  =  quantity  of  electricity]  using the heat to form steam for driving turbines. Z  =  electrochemical  the 40% maximum now obtainable through burning of fuel and  equivalent.2 V  =  = 5.5 V  –3.  or Q =  A  24.  (d) : CH 3COONa + HCl ® CH 3COOH +  NaCl      From the reaction.39  Redox Reactions and Electrochemistry  12.  (c) : Zn (s)  + 2H +  (aq)  Zn 2+  (aq)  + H 2  (g)  2 + 0.  18.0591 = 10  In  this  reaction  oxidation  number  of  S  decreases  by  two.059  [Zn ] ´ p H 2  Ecell = E °cell  -  log  2  [H + ] 2  Ecell  = 1.49  × 10  C  red  27 More is the value of reduction potential.  (a) :  L°NaCl  = L° L°KBr  = L° Na + K+ L°KCl  = L° K+ + L°  Cl –  .. thus  0.97 V  or.91 V  = 390. (i)  + L°  .  (c)  :  CN –  ions  act  both  as  reducing  agent  as  well  as  good  n ´ w ´ F  complexing  agent.  can be made considerably more efficient (i.77 V  or. i. upto 75%) than  W = Z × Q  [W =  weight.591 log Kc  this is an oxidation reaction and HI behaves  as a reducing  1  agent.  withdrawing effect of two fluorine atoms so as it will show  0.0 V  –1.1  Here n = 2.  (d) : Higher the acidity.  (a) :  In the  reaction.0591 log K c II nd  half reaction  :  H 2 SO 4 ®  SO 2  +6             +4  0. higher will be the tendency to release  to get reduced.295 = 0.591  Þ  log K c  = 0. less is the reducing power.10 – 0.  Þ  – 0.591 = – 0. E ° cell  = 1.  F At anode : 2H 2(g)  + 4OH – (aq) ® 4H 2 O (l)  + 4e –  W ´ F  W ´ F  – –  or  Q =  At cathode : O 2(g)  + 2H 2O    (l)  + 4e  ® 4OH  (aq)  or  Q =  A  E 2H 2(g)  + O 2(g) ® 2H 2 O (l)  n [A  = Atomic weight  .  protons and hence lighter will be the electrical conductivity.0295 logK c  Co 2+  | Co 3+  = – 1.7 S  cm 2  mol –1  22.  n =  valency  of  ion]  23..  25.   (d) : The oxidation states show a change only in reaction (d).  (a) : E cell  = E right  (cathode)  – E left (anode) .  –2 e –  H 2(P    2)    P P P  RT RT RT  E = E° log 2 = 0 og  2 =  log  1  nF P1 nF P1 nF P2  31.  (c) : E cell  = Reduction potential of cathode (right) – reduction  potential  of anode  (left)  = E right  – E left .8 g  96500 28.  (a) : Here Cr 3+  is oxidised to Cr 2O    7  . .  30.  (b) : 2H +  + 2e –  H 2 (P 1 )  H 2 (P 2 )  2H +  + 2e –  Overall reaction : H 2 (P 1 )  S × m × m 3  S × m  = 2  mol  m ´ mol m 2 ×  3  m  = S m 2  mol –1  33.  (b) : The mass of silver deposited on the cathode  =  108 ´ 9650 = 10.  0  +1  0  Zn + 2AgCN  –  +2 e  34.40  JEE MAIN CHAPTERWISE EXPLORER  27.  or  S =  K  or  K  = AC  [K  = constant of proportionality]  L SL  AC \  Unit of  K  =  2–  29.  S  µ  L +2  2Ag + Zn(CN) 2  –5 e  Mn 2+  [MnO 4 ] –1  –  –3 e  +4  MnO 2  35.  (c) :  +3  Mn 2 O 3  –4 e –  +7  e –  [KMnO 4 ]  – –  32.  (b) : It is also known as  heat capacity.  (b) : S µ A (A = area)  S µ C (C = concentration)  1  S µ ( L = length)  L AC  Combining we get.   (2010)  (log 2 = 0.  Cl 2  + H 2 S  –  –  Cl +  + HS  H +  + Cl  + S  (fast)  –  +  B. In this equation.25 h  (2010)  (a)  1  (b)  0  (c)  3  (d)  2  (2006)  The  half­life  period  of  a  first  order  chemical  reaction  is  6.  H 2S   Û H  + HS  (fast equilibrium)  –  –  Cl 2  + HS  2Cl  +  H +  + S  (slow)  (a)  A only  (b)  B  only  (c)  Both A and B  (d)  Neither A  nor  B  5. The  time required for the completion of 99%  11.6 minutes  (2009)  For a reaction  1  A ® 2 B rate of disappearance of A is related  2  to the  rate of appearance  of  B  by  the  expression  (a)  - d [ A] d [ B ]  = 4  dt dt (b)  .  The rate of a reaction doubles when its temperature changes  from 300 K to 310 K.301)  (a)  60. the rate increased by two  times.314 J  K –1  mol –1  and  log 2  = 0.5 kJ mol –1  (2013)  For a first order reaction.5 h  (d)  0.  equation?  –  –  +  +  H  + Cl  + Cl  + HS  (slow)  A. When  the initial  concentration  of NO with Br 2  to form NOBr.025 M in 40 minutes.  2.  Consider the  reaction  :  +  –  Cl 2(aq)  + H 2 S (aq)  S (s)  + 2H  (aq)  +  2Cl  (aq)  The  rate  of  reaction  for  this reaction  is  rate = k[Cl 2 ][H 2 S]  Which  of  these  mechanism  is/are  consistent  with  this  rate  9.3 minutes  (b)  23. (A) ®  products.0 mol L –1 . the half­life did not change.  Rate of a reaction can be expressed by Arrhenius equation as :  of  the chemical  reaction  will  be  k = Ae –E/RT .25 mol L –1  if  it  NOBr 2(g)  + NO (g) ® 2NOBr (g)  is a zero order reaction?  If the second step is the rate determining step.d [ A] =  d [ B ]  dt dt (2008)  Consider the reaction. 2A + B ® products. If the temperature is raised by 50°C.301)  (a)  230. The rate  of reaction when the  concentration  of A is 0. .  (2007)  The energies of activation for forward and reverse reactions for  A 2  + B 2  ƒ  2AB are 180 kJ mol –1  and 200 kJ mol –1  respectively. the concentration  of A changes from 0. E represents  .47 × 10 –5  M/min  (c)  1.  of the reactant A is 2.  The presence of a catalyst lowers the activation energy of both  (forward and reverse) reactions by 100 kJ mol –1 . how much time does it take  NO (g)  + Br 2(g)  NOBr 2(g)  for its concentration to come  from  0.  4.d [ A] =  1  d [ B ]  dt 2  dt (c)  .  6.73 × 10 –4  M/min  (d)  1. the order of the  (a)  1 h  (b)  4  h  reaction with respect to NO (g)  is  (c)  0.93 minutes.73 × 10 –5  M/min  (2012)  3.5 kJ mol –1  (b)  53.6 kJ mol –1  7. When the  concentration of A alone was doubled.  Activation energy of such a reaction  will be (R = 8.41  Chemical  Kinetics  CHAPTER  CHEMICAL KINETICS 9  1.03 minutes  (c)  46. The unit of rate constant for this reaction is  (a)  s –1  (b)  L mol –1  s –1  (c)  no unit  (d)  mol L –1  s –1 .06 minutes  (d)  460. When concentration  of B alone was doubled. the rate  of the  reaction increases  by  about  (a)  10 times  (b)  24 times  (c)  32 times  (d)  64 times  (2011)  8.  The following mechanism has been proposed for  the reaction  A  Products  is 1  hour.d [ A] =  1  d [ B ]  dt 4  dt (d)  .  (c)  48.6 kJ mol –1  (d)  58.1 M to 0.01  M is  (a)  3.47 × 10 –4  M/min  (b)  3.  The rate of a chemical reaction doubles for every 10°C rise  of temperature. The enthalpy  change of the reaction (A 2  +  B 2 ®  2AB) in the presence  of  a  catalyst will be (in kJ mol –1 )  (a)  20  (b)  300  (c)  120  (d)  280  (2007)  The  time  for  half  life  period  of  a  certain  reaction  10.50 to 0.   (d)  16.  (a)  s –1 .  the order of the reaction is  decreases from 0.  (2003)  For the reaction system:  2NO ( g)  + O 2(g) ® 2NO 2(g) .  R is Rydberg constant.4 M in 15 minutes.  A is adsorption factor.  In  a  first  order  reaction.  (c)  4.  (c)  18.  (b)  12.42  JEE MAIN CHAPTERWISE EXPLORER  (a)  the energy  above  which  all  the  colliding  molecules will  react  (b)  the energy below which colliding molecules will not react  (c)  the total energy of the reacting molecules at a temperature.  For the reaction A + 2B ® C.  (2006)  19.1 M to  0.  volume is suddenly reduced to half its value by increasing the  pressure on it.  (b)  2.8 M to 0.  (a) 6.  (d)  13.  (a)  21.  E a  is energy of activation.  (a)  10.  In the respect of the equation k = Ae –E a /RT  in chemical kinetics.  (d)  20. the rate of reaction will  (a)  diminish to one­fourth of its initial value  (b)  diminish to one­eighth of its initial value  (c)  increase to eight times of its initial value  (d)  increase to four times of its  initial value.  (a)  15.  (a)  23.  (2002)  21.  (c)  5.  Units of rate constant of first and zero order reactions in terms  of molarity M unit are respectively  17. If the reaction is of first order with respect to O 2  and second order with respect to NO 2 .  (c)  .025 M is  (c)  5  (d)  7  (2002)  (a)  30 minutes  (b)  15 minutes  (c)  7.  H 2  + I 2 ®  2HI is  15. rate is given by R = [A] [B] 2  then  16.  (a)  7. s –1  (d)  M.  (a)  17. the ratio of the new rate  reactant to drop to 3/4 of its initial value.  (c)  8. M  which one of the following  statements  is  correct?  (c)  Ms –1 .  A  reaction was  found  to  be  second  order  with  respect to  the  concentration of carbon monoxide.10/k  (b)  0.  the  concentration  of  the  reactant. If the rate constant for  to the earlier rate of the reaction will be as  a first order reaction is k.  (2006)  (a)  (b)  (c)  (d)  k is equilibrium constant.  t 1/4  can  be  taken  as  the  time  taken  for  the  concentration  of  a  A and halving the concentration of B.  (d)  22.  (b)  14. the t 1/4  can be written as  1 (a)  m + n (b)  (m + n)  (a)  0.  The formation of gas at the surface of tungsten due to adsorption  is the reaction of order  (a)  0  (b)  1  (c)  2  (d)  insufficient  data.  The rate law for a reaction between the substances A and B is  given by rate = k [A] n  [B] m .  (d)  -2 = -2  2  =  (2002)  dt dt dt (2004)  22. On doubling the concentration of  13.  (c)  19.  (2005)  20.  (a)  3.69/k  (d)  0. The time taken  (a)  3  (b)  6  for the concentration  to change from 0.75/k  (2005)  14.  (2004)  23. The correct statement in relation to this reaction  == -  (a)  dt dt dt is that the  –1  (a)  unit of k must be s  d [H 2 ] d [I2 ]  1 d [HI]  (b)  = =  (b)  t 1/2  is a constant  dt dt 2  dt (c)  rate of formation of C is twice the rate of disappearance  d [H 2 ] 1 d [I2 ]  d [HI]  (c)  1 = = -  of A  2 dt 2  dt dt (d)  value of k is independent of  the initial  concentrations of  d [H 2 ] d [I ]  d [HI]  A and B.  The differential rate law  for the reaction. with everything else kept the same.  The rate equation for the reaction 2A + B ® C is found to be :  d [H 2 ] d [I2 ]  d [HI]  rate = k[A] [B].  (2003)  12. If the concentration of carbon  monoxide is doubled.  A  reaction involving two different reactants  can never be  (a)  unimolecular  reaction  (b)  first order  reaction  (c)  second order  reaction  (d)  bimolecular  reaction. the  rate of reaction will be  (a)  remain unchanged  (b)  tripled  (c)  increased by a  factor of  4  (d)  doubled.29/k  2 (c)  (n – m)  (d)  2 (n – m)  (2003)  (c)  0. s –1  (2002)  Answer  Key  1. Ms –1  (b)  s –1 .  (b)  9. T  (d)  the  fraction  of  molecules  with  energy  greater  than  the  18.5 minutes  (d)  60 minutes.  activation energy  of  the  reaction.  (d)  11. 693  Now.693  6. t1/2  = 6.  (c) :  Rate at 50°C  = 2 10  = 2 5  = 32 times.  i.  (d) : NO (g)  + Br 2(g)  ƒ  NOBr 2(g)  NOBr 2(g)  + NO (g) ® 2NOBr (g)  [rate determining step]  Rate of the reaction (r) =  K [NOBr 2] [NO]    where [NOBr 2] =  K C  [NO][Br 2]      r =  K ∙ K C  ∙ [NO][Br 2][NO]    r = K¢ [NO] 2  [Br 2 ]  The order of the reaction with respect to NO (g)  =  2  11.693 l =  (for 1 st  order  reaction) t 1/ 2  - 1 d [ A] 1  d [ B ]  =  1/ 2 dt 2  dt d [ A] 1  d [ B ]  =  dt 4  dt (b) :  Rate  =  k [A] x  [B] y  When  [B]  is  doubled.200 = - 20 kJ mol –1  The  correct  answer  for  this  question  should  be  –20 kJ mol –1 . Hence the reaction is first order with respect to A.303  a log t a - x Þ a = 0.  Rate = - (a) : For the  first  order  reaction  k= 2.  unit  of  rate  constant  is  L mol –1  s –1 .  .93  ´ log(100)  0.0347 × 0.  keeping  [A]  constant  half­life  of  the  reaction does not change.43  Chemical  Kinetics  1.303 ´ 8.25  hours  6.e.  \ Rate = [ A]1 [ B ] 1  \ order = 2  Now  for  a  nth  order  reaction.  (c) :  Given.  the  rate  also  doubles.06 minutes.314 ´ 10  ë û  Ea  =  0.303 ´ 6.  Rate at T1 °C t1/ 2  =  (c) :  For this reaction. But no option given is correct.  unit  of  rate  constant  is  (L) n–1  (mol) 1–n  s –1  when  n  =  2.1 M.0347 min –1  40 0.303  [ A 0 ]  log  l  [ A ] (a) : DH R = E f .025 M.  keeping  [B]  constant.693 = 23.  The  given  rate  equation  is  only  consistent  with  the  mechanism A.303 R ë T1T2  û Ea  é 310 .300 ù log 2 = -3  ê 310 ´ 300 ú 2. for a  first order  reaction  t 1/ 2  =  k i.  9.25 M Þ  0.  t 1/2  = 1hr.303 0.47 × 10 –4  M  min –1  50  3.  (a) : The  rate  equation depends upon  the  rate  determining  step.T 1 ù k log 10  2 = ê ú k1 2.93 Since  reaction follows 1st order  kinetics.  5.  t = 40  min  8.  Hence  the  reaction  is  first  order  with  respect  to  B.025 40 [A]  product  Thus.  0. t 1/2  is independent of the concentration of the reactant.  or t = 7.303  k= log =  log 4 = 0.  (b) : k = Ae –E/RT  where  E  =  activation  energy.  E a  = 53.  a – x = 0.  [ A 0 ]  100  Q  Reaction is 99% complete.25 = –t  + 0.  [A 0 ] =  2M  \ k= 2  = 1 mol L-1  hr -1  2 ´ 1 Integrated rate law  for  zero  order reaction  is  [A] = –kt  + [A 0 ]  Here.03 × 2 log(10) = 46.5 M and [A] =  0.e.  the  minimum  amount  of  energy  required  by  reactant  molecules  to  participate  in  a  reaction. rate = k[A]  rate = 0.  0. t 1/2  is  given  as  [ A0 ]  2k or  k =  [ A0 ]  2t1/2  Given.Eb  = 180 .  2.303 ´ 8.  2. Hence we  can  ignore sign and select option (a).5 Þ  t  = 0.01 M min –1  = 3.93 min  0.3010 ´ 2.  10. [A 0 ] = 0.  \ =  [ A] 1  E a  é T2 .  (d) : For a zero order  reaction.  Now  when  [A]  is  doubled.1 2.314 ´ 10 -3 ´ 93 ´ 10 3  10  2.  4.6 kJ mol –1  2.  (b) :  As  r  =  k[A] n  r2 r1 =  k 2  =  k1  r 2  Since  r =  2  1  \ t  = (Given)  k 2  =  2  k1  where [A0] =  initial concentration  and  [A] = concentration of  A  at time t.  hence its concentration may be  Ratio between new and earlier rate  taken constant.44  JEE MAIN CHAPTERWISE EXPLORER  dx  =  k [CO] 2  dt r 2  = k[2CO] 2  =  4k[CO] 2  Thus. concentration  becomes  two  the  balanced  stoichiometric  equation.  (d) : Rate 1  = k [A] n  [B] m  Rate = k[RCl][H 2O] expected order = 1 + 1 = 2    On  doubling  the  concentration  of  A  and  halving  the  But actual rate law :  concentration  of  B  Rate = k¢[RCl]  actual order = 1  Rate 2  = k [2A] n  [B/2] m  Here water is taken in excess.  (a) : Generally. A = frequency factor  T = temperature.  where n = order of reaction  16.29  log  =  k 3  k Therefore.1  M  to 0.  20.  R = [A] × [B] 2  Thus.e.  Thus the rate may be expressed as -d [H 2 ] .  according  to  the  rate  law  expression  doubling  the  concentration of CO  increases  the rate  by a  factor of 4.  For  a  second  order  reaction.  (c) : Given  r1  = 13.d [ I 2 ] 1  d [ HI ]  = =  dt dt 2  dt The negative sign signifies a decrease in concentration of the  reactant with increase of time.  (c) : In Arrhenius equation.303 4 0.  (a) : Order is the sum of the power of the concentrations terms  in rate law expression.  i.  (b) :  t 1/ 4  = 2.  22.e.  2A + B ® C  1 d [ A] d [ B ] d [C ]  ==  Rate  = 2  dt dt dt i.  Here the molecularity of the reaction = 2 and the order of the  k[2 A]n [ B / 2] m  n 1  m  n .  rate  of  formation  of C  is  half  the  rate  of  disappearance  of B.  (a)  21. 2t 1/2  = 2 × 15 = 30 minutes.4 M in 15 minutes.  k is L mol –1  sec –1  for a second order reaction.  Rate 2  = k [2NO] 2  [2O 2 ]  But a reaction involving two different reactants can a first order  Rate1 k [NO]2  [O 2 ]  Rate 1  1  =  = reaction. E a  = energy of activation.e. 2 moles of HI are  formed in the same time interval.  (d) : H 2  + I 2 ® 2HI  When 1 mole of H 2  and 1 mole of I 2  reacts. For example.  14.  This equation can be used for calculation of energy of activation. the concentration of reactant will fall from 0.  n = 1 Þ first order reaction i.  1  t 1/ 2  =  ka i.  molecularity  of  simple  reactions  is equal to  18.  .025 M in two half lives.m  =2 ´ = 2  = 2  reaction = 1. k = Ae –E a /RT  k = rate constant.  12. R = gas constant.e. order of reaction = 1 + 2 = 3  23.  (d) : 2 A + B ® C  rate = k [A] [B]  The value of k (velocity constant) is always independent of the  concentration  of  reactant  and  it is  a  function  of  temperature  only.  Thus.  k [ A]n [ B ]m  ( )  15.  (c) : Rate 1  = k [NO] 2  [O 2 ]  the sum of the number of molecules of reactants involved in  When volume is  reduced to 1/2. t 1/2  is inversely proportional  to initial  concentration. t 1/2  = 15 minute.  (a) : Unit of K = (mol L –1 ) 1 – n  s –1 .8 M  n = 0 Þ zero order reaction  to 0.  (a) : The concentration of the reactant decreases  from 0.  17.  involving two different reactants can never be unimolecular.  unit  of  rate  constant.  a  reaction  times.  for  the  following  reaction  Rate 2  k[2NO]2  [2O 2 ]  or  Rate 2  8 RCl + H 2O ® ROH + HCl    \  Rate 2  = 8 Rate 1  Expected rate law :  19.  C and D are 0.  (2009)  Gold numbers of protective colloids A.  0.  Which of the following statements is NOT correct?  (a)  Magnesium chloride solution coagulates. the gold sol more  readily than the iron (III) hydroxide sol  (b)  Sodium sulphate solution causes coagulation in both sols  (c)  Mixing of the sols has no effect  (d)  Coagulation  in  both  sols  can  be  brought  about  by  electrophoresis  (2005)  (2013)  2.01. respectively.  (b)  More  easily liquefiable gases  are  adsorbed  readily. V c  as compared to the volume  of a solute particle in a true solution V s  could be  (a)  ~ 1  (b)  ~ 10 23  –3  (c)  ~ 10  (d)  ~ 10 3  (2005)  Which  one  of  the  following  characteristics  is  not  correct  for  physical  adsorption?  (a)  Adsorption on solids  is reversible  (b)  Adsorption increases with increase in temperature  (c)  Adsorption is spontaneous  (d)  Both enthalpy and  entropy of adsorption are  negative.  4.  protective  powers  is  (a)  B < D < A <  C  (b)  D < A < C <  B  (c)  C <  B <  D <  A  (d)  A < C <  B <  D  (2008)  The volume of a colloidal particle.  (2006)  6.  In Langmuir’s model of adsorption of a gas on a solid surface  (a)  the  rate  of  dissociation  of  adsorbed  molecules  from  the  surface does not depend on the surface covered  (b)  the adsorption at a single site on the surface may involve  multiple molecules at the same time  (c)  the  mass  of  gas  striking  a  given  area  of  surface  is  proportional to the pressure of the gas  (d)  the  mass  of  gas  striking  a  given  area  of  surface  is  independent of the pressure of the gas.  (c)  6.  (2012)  3.  which of the  following  is  correct?  (a)  x µ p1 m (c)  x µ p 0 m (b)  x µ p1/ n m (d)  All the above are correct for different ranges of pressure.50.  7. The correct order of their  8.  (c)  Under high pressure it results into multi molecular layer  on  adsorbent  surface.  (d)  5.  (2003)  Answer  Key  1.  (c)  .  (c)  (d)  2.10 and 0. B. Al 3+  and Ba 2+  for arsenic  sulphide  sol increases  in  the order :  (a)  Al 3+  < Na +  < Ba 2+  (b)  Al 3+  < Ba 2+  < Na +  (c)  Na +  < Ba 2+  < Al 3+  (d)  Ba 2+  < Na +  < Al 3+  5.  Which  of  the  following  statements  is  incorrect  regarding  physisorption?  (a)  It occurs because  of  van  der Waals forces.005.  (d)  (b)  3.  (d)  4.  The coagulating power of electrolytes having ions Na + .  (d)  Enthalpy of adsorption (DH a dsorption ) is low and positive.  8.45  Surface Chemistry  CHAPTER  SURFACE CHEMISTRY 10  1.  The  d+isperse  phase  in  colloidal  iron  (III)  hydroxide  and  colloidal gold is positively and negatively charged.  7.  According to Freundlich  adsorption isotherm. 0. respectively. e.  pressure.  (c) : For a negatively charged sol. Therefore adsorption always takes  place with evolution of heat. Smaller  the value of gold number greater will be protecting power of  8.  (d) : For true solution the diameter range is 1 to 10 Å and for  colloidal solution diameter range is 10 to 1000 Å.  (d) : The different protecting colloids differ in their protecting  powers.  it was  derived  by  Langmuir  that the mass of the gas adsorbed per gram of the adsorbent is  related to the equilibrium pressure according to the equation:  x aP  = m 1 + bP where x  is  the  mass  of  the  gas  adsorbed  on  m  gram  of  the  adsorbent.  the  protective  colloid.  3. P is the pressure and  a. greater the positive    charge on cations. 7. .46  JEE MAIN CHAPTERWISE EXPLORER  1.  Thus  1  protective power of colloid µ  Gold number 5. i.  4.  10 3  (b)  :  During  adsorption. it is an exothermic process and  since  the  adsorption  process  is  exothermic.  3  Vc (4 / 3) prc 3  æ r c ö = = Vs (4 / 3)p r s 3  çè r s ÷ø Ratio of diameters = (10/1) 3  = 10 3  V c  /V s  . like As 2S    3.  DH = –ve) but its value is quite low because the attraction of  gas molecules and solid surface is weak van der Waals forces.  Zsigmondy  introduced  a term  called Gold  number  to describe the protective power of different colloids.  (c) : Assuming the formation of a monolayer of the adsorbate  on  the surface  of the  adsorbent.  greater is the coagulating power.e.  the  physical  adsorption occurs readily at low temperature and decreases with  increasing temperature.  Hence  on  mixing  coagulation of two sols may be take place.  2. (Le Chatelier's principle). b are constants.  (c)  :  Opposite  charges  attract  each  other.  there  is  always  decrease  in  surface  energy which appears as heat..  (d)  :  Physical  adsorption  is  an  exothermic  process  (i.  (d) : According to Freundlich adsorption isotherm  x = kp1/ n m 1/n can have values between 0 to 1 over different ranges of  6. (2004)  (c)  5.  7.  (b)  8. Its  half­life period is 30 days.  the  mass  of  it  remaining  undecayed after 18 hours would be  (a)  4.084 g  (c)  3. when initial amount is 64 grams  is  (a)  16 g  (b)  2 g  (c)  32 g  (d)  8 g  (2002)  Answer  Key  1.  If  half­life  of  a  substance  is  5  yrs. If the initial mass  of the isotope was 200 g.  (c)  10. b­particle is emitted in radioactivity by  (a)  conversion  of  proton  to  neutron  (b)  form outermost  orbit  (c)  conversion  of  neutron  to  proton  (d) b­particle is not emitted.  (2005)  9.0 g  (2003)  The radionucleide  234  90 Th  undergoes two successive b­decays  followed  by  one a­decay.  (a)  3.  (c)  11.47  CHEMISTRY  CHAPTER  NUCLEAR CHEMISTRY*  11  1. The  atomic  number  and  the  mass  number respectively of the resulting radionucleide  are  (a)  92 and 234  (b)  94 and 230  (c)  90 and 230  (d)  92 and 230  (2003)  10.  (c)  . if one emission is an a­  particle.  isotope?  (a) b­particle emission  (b)  Neutron particle emission  (c)  Positron emission  (d) a­particle emission  (2007)  7.125 g  (d)  4.  the mass remaining after  24  hours  undecayed is  (a)  1.  (a)  100 days  (b)  1000 days  (c)  300 days  (d)  10 days  (2007)  234  In the transformation of  238 92 U to  92  U .  Which  of  the  following  nuclear  reactions  will  generate  an  6.0 g  (d)  16.167 g  (2004)  Consider the following nuclear reactions:  X A  + ® Y X N + 2 4  2 He  ;  Y N ® B  L + 2 b  The number of neutrons in the element L is  (a)  142  (b)  144  (c)  140  (d)  146  238 92 M The half­life of a radioactive isotope is three hours.  (2002)  11.0 g  (b)  8. after how many days will it be safe to enter  the  room?  8.  2.  (b)  2.  The half­life of a radioisotope is four hours.  23  (d)  the isobar of  11  Na  (2005)  Hydrogen bomb is based  on the  principle of  (a)  nuclear fission  (b)  natural radioactivity  (c)  nuclear  fusion  (d)  artificial  radioactivity.042 g  (b)  2.0 g  (c)  12. If the initial velocity is ten times the  permissible value.  (a)  9. what should be the other emission(s)?  (a)  Two b –  (b)  Two b –  and one b +  (c)  One b –  and one g  (d)  One b +  and one b –  (2006)  24  A photon of hard gamma radiation knocks a proton out of 12  Mg  nucleus to form  (a)  the isotope of  parent nucleus  (b)  the isobar of parent nucleus  23  (c)  the nuclide  11  Na  5.  (a)  4.  A radioactive element gets spilled over the floor of a room.  (d)  6.  (c)  * Not  included  in  the  syllabus of  JEE  Main  since  2008.  then  the  total  amount  of  substance left after 15 years.  3. If the initial  mass  of  the  iosotope  were  256  g.  4.   –  1  n  0  – a  A – 4  Z – 2  Y;  – b  A  X  Z  +b+  A  X  Z  A – 1  Z  X  A  Y  Z + 1  6.  A  X  Z  A  X  Z  2.  = 3.  (a) :  U  92  –a  234  2.693 238  ( 1 2 )  N = 200 ´  230 88 N 8.303  A  t  = log  0  = log  0  l  N l  A 6  or.  234  92 U  Thus in order to get 92 U 234  as end product 1a and 2b particles  should be emitted.  ( )  A  Z – 1 Y  (a) : Let  A be the  activity for  safe working. it is produced  by  the conversion  of  a  neutron  to  a  proton  at  the  moment  of  emission.  JEE MAIN CHAPTERWISE EXPLORER  (b)  :  The  atoms  of  the  some  elements  having  same  atomic  number but different mass numbers are called isotopes.69 days  » 100 days  0.303 10 A =  log  0.303  N  2.  24 (c) :  12 Mg + g ® 23 1  11 Na  +  1 p 5.  (c)  :  Hydrogen  bomb  is  based  on  the  principal  of  nuclear  fusion. The temperature produced  by the explosion of n  6  ( 2 )  ( )  N = N 0  1  2  Þ  2.303 ´ 30  = 99.  Given A 0  = 10 A  A 0  × N 0  and A × N  2.  2.  4.  1 H  and  1 H  releasing  huge amount  of energy. number of neutrons in element L  = 230 – 86 = 144  (a) : t 1/2  = 3 hours.  (c) : Since the nucleus does not contain b­particles.  (c) : t 1/2  = 4 hours  n  n = T  = 24 = 6  ; N =  N 0  1  t1/ 2  4  2  1  Þ N = 256  N = 4.0 g  -2 b -a ®  Th 230  (c) :  90 Th 234 ¾¾¾ ® 92 X 234 ¾¾¾ 90  Elimination of 1a  and 2b  particles give isotope.  10. In hydrogen bomb. n = T/t 1/2  = 18/3 = 6 9.48  1.125 g  (b) :  238 92 M ® 230 4  88 N + 2 2 He  230  +  86 L + 2 b  ® Therefore.  (d) : t 1/2  = 5 years. a mixture of deuterium oxide and  tritium oxide is enclosed in a space surrounding an ordinary  atomic bomb. 7.693 90 A  – b  234  B  91  –b  = 3.  1 1  0  0 n  ®  +1 p  +  –1 e  11.693/ 30  A the  atomic  bomb  initiates  the  fusion  reaction  between  3 2  .  n  n = 3  T  t1/ 2  = 1 1  n = N 0  æ ö = 64 æ ö = 8 g  è 2ø è 2 ø  15  = 3  5  .303 ´ 30  =  log 10  0.  S 2–  (d)  Li + .  Based on lattice energy and other considerations which one of  (c)  Na +  > Mg 2+  > Al 3+  > O 2–  > F –  +  –  2+  2–  3+  the  following  alkali  metal  chlorides  is  expected  to  have  the  (d)  Na  > F  > Mg  > O  > Al  (2010)  highest melting point?  Following statements regarding the periodic trends of chemical  (a)  LiCl  (b)  NaCl  reactivity of the alkali metals and the halogens are given.  Lattice energy of an ionic compound depends upon  (a)  charge on the ion only  in the halogens with increase in atomic number down the  (b)  size of the ion only  group. Ca 2+ .  electron gain enthalpy  of Na +  will  be  (a)  + 2. Se and Ar?  (a)  Ca < Ba < S  < Se < Ar  8. K + . Na + . S 2–  (c)  N 3– . Ca 2+ .55  eV  (c)  – 5.  (2006)  .  (2006)  The  increasing  order  of  the  first  ionisation  enthalpies  of  the  elements B.1 eV  (d)  – 10.  Which  among the  following  factors is  the  most important  in  (c)  Chemical  reactivity  increases  with  increase  in  atomic  making fluorine the strongest oxidising agent?  number  down  the  group  in  both  the  alkali  metals  and  (a)  Electron  affinity  halogens.  Which  of  the  following  represents  the  correct  order  of  increasing first ionization enthalpy for Ca.  13. K +  (b)  Ca 2+ .  The  first ionisation  potential of  Na  is  5.  species is  (a)  S 2– . Sr 2+ . Ba. Cl – .  49  The decreasing values of bond angles from NH 3  (106°) to SbH 3  (101°) down  group­15 of the periodic table is due to  (a)  increasing bond­bond pair repulsion  (b)  increasing p­orbital character in sp 3  (c)  decreasing lone pair­bond pair repulsion  (d)  decreasing  electronegativity. S 2–  (2012)  3.  (b)  Ionization  energy  (d)  In alkali metals the reactivity increases but in the halogens  (c)  Hydration  enthalpy  it  decreases  with  increase  in  atomic  number  down  the  (d)  Bond dissociation  energy  (2004)  group.  In  which  of  the  following  arrangements  the  order  is  NOT  (a)  Al 2O    3  < MgO < Na 2 O < K 2 O  according to the property indicated against it?  (b)  MgO < K 2O < Al    2 O 3  < Na 2 O  (a)  Al 3+  < Mg 2+  < Na +  < F ­  ­ increasing ionic size  (c)  Na 2 O < K 2 O < MgO < Al 2O    3  (b)  B < C < N < O  ­ increasing first ionisation enthalpy  (d)  K 2O < Na  (2011)    2 O < Al 2O    3  < MgO  (c)  I < Br < F < Cl ­ increasing electron gain enthalpy  The  correct sequence  which  shows  decreasing  order of  the  (with negative sign)  ionic radii  of the  element is  (d)  Li < Na < K < Rb ­ increasing metallic radius  (a)  O 2–  > F –  > Na +  > Mg 2+  > Al 3+  (2005)  (b)  Al 3+  > Mg 2+  > Na +  > F –  > O 2–  11. Ca 2+ . S. K + .  5.  4. S 2–  (c)  K + . Ca 2+ . O 2– .  6. Cl – .2  eV  (2013)  2. Cl –  (d)  Cl – . Ca 2+  (2006)  Which one of the following orders presents the correct sequence  of the  increasing basic  nature  of  the  given  oxides?  10. Mg 2+ .1  eV. S 2– . K + . P. Which  (c)  KCl  (d)  RbCl  (2005)  of these statements  gives  the correct picture?  (a)  The reactivity decreases in the alkali metals but increases  12. S and F (lowest first) is  (a)  F < S < P < B  (b)  P < S < B < F  (c)  B < P < S < F  (d)  B < S < P < F  (2006)  Which one of the following sets of ions represents a collection  of  isoelectronic  species?  (a)  K + .  (2005)  the group. F – .55 eV  (b)  – 2.  (b)  Ca < S < Ba  < Se < Ar  (c)  S < Se < Ca  < Ba < Ar  (d)  Ba < Ca < Se  < S < Ar  (2013)  The increasing order of the ionic radii of the given isoelectronic  9.  (c)  packing of the ion only  (b)  In both the alkali metals and  the  halogens  the chemical  reactivity decreases with increase in atomic number down  (d)  charge and  size of the ion. The  value of  7. Sc 3+  (b)  Ba 2+ . Cl – .Classification of Elements and Periodicity in Properties  CHAPTER  12  CLASSIFICATION OF ELEMENTS  AND PERIODICITY IN  PROPERTIES 1.   (a)  10. nos.  (d)  18. Cl = 17.  According  to  the  periodic  law  of  elements.  (a)  4. : Ce = 58.  19.  Which one of the following ions has the highest value of ionic  radius?  and then an endothermic step as shown below.  Al = 13. K = 19.  (c)  . Mg = 12. Yb = 70 and Lu = 71)  (2004)  (2002)  Answer  Key  1. Br­35)  (b)  oxygen has high  electron  affinity  + .  (b)  9. Cl – .  (a)  15.  (d)  12.  (a)  Li +  (b)  B 3+  –  –  –1  O (g)  + e  = O  (g)  ; DH° = –142 kJmol  2–  (c)  O  (d)  F –  (2004)  O –  (g)  + e –  = O 2–  (g)  ; DH° = 844 kJmol –1  17.  (d)  Sn > Yb > Ce > Lu. Cl –  (d)  Ca 2+ .  (a)  11. Sc 3+  (a)  Ce > Sn > Yb > Lu  (b)  Sn > Ce > Lu > Yb  (d)  Na + .  13.  Ca = 20. Ca 2+ .  (c)  (c)  (d)  (a) 2.  Which is the correct order of atomic sizes?  (c)  K + .  (b)  17.  (2003)  (b)  Na + .  (b)  16.  the  variation  in  (2004)  properties of elements is related to their  (a)  atomic masses  15. Mg 2+  (b)  N 3– . Al 3+ . Sc 3+ . Cs + . Na  = 11. Cl – .  The formation of the oxide ion O 2– (g)  requires first an exothermic  16.  (d)  14. Na +  (a)  Na  (c)  O –  ion will tend to resist the addition of another electron  (c)  Be.  (b)  6. Sc = 21)  (At.  Which one of the following groupings represents  a collection  This is because  of  isoelectronic  species?  (a)  oxygen is  more  electronegative  (At. F –  19. Br  (2003)  (d)  O –  ion has comparatively larger size than oxygen atom. Mg 2+ .  (d)  8. Nos. Sn = 50.  (c)  5.: Cs­55.  18.  (c)  3. Cl –  (d)  nuclear neutron­proton number ratios.  Which one of the following sets of ions represents the collection  (b)  nuclear masses  of  isoelectronic  species?  (c)  atomic numbers  (a)  K + . Al 3+ . Ca 2+ .: F =  9. Sc 3+ . Mg 2+ . F – . Ca 2+ .50  JEE MAIN CHAPTERWISE EXPLORER  14.  (c)  Lu > Yb > Sn > Ce  (Atomic nos.  7.   For  this  reason. So.E.  (c) :  NH 3  PH 3  AsH 3  SbH 3  15.  in atomic  number  due  to  following  reasons:  (a)  As the size increases.51  Classification of Elements and Periodicity in Properties  1.  The ionisation enthalpy decreases as we move down a group.4  11. ionization enthalpy of nitrogen is greater  Al 2O    3  < MgO < Na 2 O < K 2 O  than  oxygen. melting point decreases  size.  P(1s 2  2s 2  2p 6  3s 2  3p 3 )  has  a  stable  half  filled  electronic  configuration  than  S  (1s 2  2s 2  2p 6  3s 2  3p 4 ).5°  93.  (a) : Isoelectronic species are those which have same number  Bond angle    106. the  Thus all these  ions have  18 electrons in them.  ionisation enthalpy of P is higher than S.  (b)  Due  to decrease  in  electronegativity from  F to I.0  17. ionic radii decreases.  12.  they  have  a  strong  tendency  to  lose  the  single  valence  s­  NaCl is expected to have the highest melting point among  electron to form unipositive ions having inert gas configuration.  This reactivity arises due to their low ionisation enthalpies  And  while  descending  in  the  group  basic  character  of  corresponding oxides  increases.  (b) : As we move from left to right across a period. Order of nuclear charge:  Ca 2+  >  K +  >   Cl –  >    S 2–  Protons  :  20  19  17  16  Electrons  :  18  18  18  18  9.5°  91.  smaller the  11. But LiCl has lower melting  (d) : All the alkali metals are highly reactive elements since  point in comparison to NaCl due to covalent nature. (d) :  Element:  B  S  P  F  I.5°  91.  So  the  order  of  increasing  ionisation  enthalpy  should  be  \ B < C < N < O.  The  bond  angle  in  ammonia  is  less  than  109°  28¢  due  to  K +  = 19 – 1 = 18 ; Ca 2+  = 20 – 2 = 18  repulsion between lone pairs present on nitrogen atom and  Sc 3+  = 21 – 3 = 18 ; Cl –  = 17 + 1 = 18  bonded pairs of electrons.  Thus increasing  order of  ionic  radii  2+  +  –  2–  Ca  < K  < Cl  < S  (a)  : While moving from left to right in periodic table basic  character of  oxide  of  elements  will  decrease. Nuclear  charge is maximum  of the specie with maximum protons. anions generally have greater size than  cations.1 eV  2.  The weak F – F bond makes fluorine the strongest oxidising  electron by  the  nucleus  becomes  less. the  bond  between  halogen  and  other  elements  becomes  14.  (c) : Electron gain enthalpy = – Ionisation potential  = –  5.  and high negative values of their standard electrode potentials.  8. .  the  ionisation enthalpy increases with increasing atomic number.  Also greater.  \  10. the attraction for an additional  13.  But  N(1s 2  2s 2  2p 3 )  has  a  stable  half  filled  electronic  \ Correct order is  configuration.  (b)  : For  isoelectronic  species  as  effective  nuclear  charge  increases.  (a) :  K +  = 19 – 1 = 18  e –  Cl –  = 17 + 1 = 18  e –  Ca 2+  = 20 – 2 = 18 e –  Sc 3+  = 21 – 3 = 18 e –  Thus all the  species are  isoelectronic.  (d) : The bond dissociation energy of F – F bond is very low. ionisation  enthalpy increases  with increasing atomic  number.  4.  bond angles gradually decrease due to decrease in bond pair  lone  pair repulsion.  7. the  nuclear  charge (Z)  of  the  ion.  is B < C < O < N.  (d) : The value of lattice energy depends on the charges present  However. the reactivity of halogens decreases with increase  on the  two ions and the  distance between them.  halogen.  3. Thus the order  is  :  going down the group because lattice  enthalpies decreases  2–  –  +  2+  3+  O  > F  > Na  > Mg  > Al  as size of alkali metal increases. the correct order of increasing the first ionization enthalpy  isoelectronic species.  (c)  :  The  addition  of  second  electron  in  an  atom  or  ion  is  weaker and weaker. Thus.  always endothermic.3°  of  electrons. 5.  (b) : In case of halides of alkali metals.  Among  So.  given halides.  (a)  : All  the  given  species  are  isoelectronic.3  10.  6.4  In  general  as  we  move  from  left  to  right  in  a  period. As we move down the group. (eV):  8.  (d) : Ionization  enthalpy  decreases  from  top  to  bottom  in  a  group while  it increases  from left  to right  in  a  period.  O 2–  has highest  value  of  ionic  radius.  (b) :  Isoelectronic  species  are the  neutral atoms.  19.8  e 10  9  –  z  For F  .  the  size  decreases and when z/e ratio  decreases the  size increases. Number of electrons  in N 3–  = 7 + 3 = 10.  (c)  :  This  can  be  explained  on  the  basis  of  z/e nuclear charge  .  Number of electrons in F –  = 9 + 1 = 10  Number of electrons in Na +  = 11 – 1 = 10  18.9  e 10  For Li + .5  e 2  z  8  For O 2– .  = = 0. in case of lanthanides  Hence.  { }  z  3  = = 1.  (c) : According to modified modern periodic law. the properties  of elements are periodic functions of their atomic numbers.52  JEE MAIN CHAPTERWISE EXPLORER  16. But due to lanthanide contraction.  La > Sn.  (a)  :  Generally  as  we  move  from  left  to  right  in  a  period.  no. Thus  the  atomic  radius  of  Sn  should  be  less  than  lanthanides.5  e 2  z  5  For B 3+ . cations or  Ce > Sn > Yb > Lu. anions of different elements which have the same number of  .  electrons but different nuclear  charge.  there is regular decrease in atomic radii and in a group as the  atomic number increases the atomic radii also increases.  = = 0.  there is a continuous decrease in size with increase in atomic  number.  = = 2. Hence the  atomic radius follow the  given trend  :  17. of electrons  whereas  z/e  ratio  increases.   8.  (2002)  Answer  Key  1.53  General Principles and Processes of Isolation of Metals  GENERAL PRINCIPLES  AND PROCESSES OF  ISOLATION OF METALS CHAPTER  13  (a)  Magnetite  (c)  Galena  1.  (d)  6.  (2012)  Which of the following factors is of no significance for roasting  sulphide ores to the  oxides and  not subjecting  the sulphide  ores  to  carbon  reduction  directly?  (a)  CO 2  is more volatile than  CS 2 .  (d)  .  (c)  Ag and Au  (d)  Fe and Ni.  (b)  Metal sulphides are thermodynamically more stable than  CS 2 .  6.  (c)  CO 2  is thermodynamically more stable  than CS 2 .  7.  (2002)  The metal extracted by leaching with a cyanide is  (a)  Mg  (b)  Ag  (c)  Cu  (d)  Na.  (2002)  Cyanide process is used for the extraction of  (a)  barium  (b)  aluminium  (c)  boron  (d)  silver. the appropriate electrodes are  cathode  anode  (a)  pure zinc  pure  copper  (b)  impure sample  pure  copper  (c)  impure zinc  impure sample  (d)  pure  copper  impure sample.  Which method of purification is represented by the following  equation?  Ti (s)  + 2I 2(g)  TiI 4(g)  Ti (s)  +  2I 2(g)  2.  During the process of electrolytic refining of copper.  (d)  Metal sulphides are less stable than the corresponding  oxides.  Aluminium is extracted by the electrolysis of  (a)  bauxite  (b)  alumina  (c)  alumina mixed with molten cryolite  (d)  molten  cryolite.  (c)  (b)  2.  Which one of the following ores is best concentrated  by froth­  flotation  method?  When the sample of copper wth zinc impurity is to be purified  by electrolysis.  (a)  Cupellation  (b)  Poling  (c)  Van Arkel  (d)  Zone  refining  (2004)  5. These are  (a)  Sn and Ag  (b)  Pb and Zn  8.  (c)  (2002)  4.  (a)  (c)  3.  (2005)  4.  (b)  Cassiterite  (d)  Malachite.  (2008)  7. some metals  present as impurity settle as ‘anode mud’.  (c)  5.  3.   Zn ®  Ag 2 S + 2NaCN ®  Na[Ag(CN) 2 ]  ¾¾¾ Na 2 [Zn(CN) 4 ] + Ag ¯  (c)  :  Froth­flotation  method  is  used  for  the  concentration  of  sulphide ores. Ni.CS 2  is more volatile than  CO 2 . The method is based on the preferential wetting  (d) : The impure metal is made anode while a thin sheet of  pure metal acts as cathode. thorium and  uranium.  4.  The  reduction  process  is  on  the  thermodynamic  stability  of  the  products  and  not  on  their  7.  ores  to  their  oxides.  zirconium.  volatility. .54  1. dissolve  in  the  solution  and  less  electropositive  impurities  such  as  Ag.  (b) : Silver ore forms a soluble complex with NaCN from which  silver is precipitated using scrap zinc. So option (a) is of no significance for roasting sulphide  6.  argentocyanide  (soluble)  8. Au and Pt collect below the anode in the form of anodic  mud. Here galena (PbS)  is the only sulphide ore.  5.  (c)  :  In  the  electrolytic  refining  of  copper  the  more  electropositive impurities like Fe. Co.  JEE MAIN CHAPTERWISE EXPLORER  (c) : Van Arkel method which is also called as vapour­phase  refining is used for preparing ultrapure metals like titanium.  (c)  :  Aluminium  is  obtained  by  the  electrolysis  of  the  pure  alumina (20 parts) dissolved in a bath of fused cryolite (60 parts)  and fluorspar (20 parts). Zn.  (d) : Gold and silver are extracted from their native ores by  Mac­Arthur forrest cyanide process. the pure  metal is deposited on the cathode and equivalent amount of  the metal gets dissolved  from the anode.  3.  properties with the frothing agent and water. etc.  sod.  2. On passing the current.  (a) : Oxidising roasting is a very common type of roasting  in metallurgy and is carried out to remove sulphur and arsenic  in the form of their volatile oxides.   (b)  Saturation of water with MgCO 3 .  (2003)  .  (2008)  3.  (b)  Electrolysis  of  water.  In context with the industrial preparation of hydrogen from  water gas (CO + H 2 ).  (d)  Addition of Na 2 SO 4  to water.  Answer  Key  1.9%)  can  be  made  by  which  of the  following  processes?  (a)  Mixing natural hydrocarbons of high molecular weight.  (a)  3.  (d)  H 2  is removed through  occlusion with Pd.  (b)  CO and H 2  are fractionally separated using differences  in their densities.  (2012)  2.  (b)  2.  (c)  Which one of the following processes will produce hard water?  (a)  Saturation of water with CaCO 3 .  (c)  CO is removed by absorption in aqueous Cu 2Cl    2 solution.  (c)  Saturation of water with CaSO 4 .  which of  the  following is the  correct  statement?  (a)  CO is oxidised to CO 2  with steam in the presence of a  catalyst followed  by absorption of  CO 2  in  alkali.55  Hydrogen  CHAPTER  HYDROGEN 14  1.  (d)  Reaction of methane  with steam.  Very  pure  hydrogen (99.  (c)  Reaction of salt  like  hydrides with  water.   (c) : Permanent hardness is introduced when water passes over  rocks containing the sulphates or chlorides of both of calcium  and magnesium.  the gases and steam over  an iron  oxide  or cobalt oxide or chromium oxide catalyst at 673 K resulting  in  the  production  of  more  H 2 . .  The  entire reaction is called  water gas shift reaction.56  1.  JEE MAIN CHAPTERWISE EXPLORER  (b) :  Dihydrogen  of  high  purity  is  usually  prepared by  the  electrolysis  of water  using  platinum  electrodes  in  presence  of small amount of acid  or alkali.  Dihydrogen  is collected  at  cathode.  (a) : Carbon monoxide is oxidised to carbon dioxide by passing  3.  CO + H2 O  Fe2O 3 . CoO  673 K  CO2 + H 2  NaOH  CO 2  alkali  Na2CO 3  CO 2  is absorbed in alkali (NaOH).  2.   It  forms  an  insoluble  hydroxide  M(OH) 2  which  is  soluble  in  NaOH solution.  (a)  exhibiting maximum  covalency in compounds  (b)  forming polymeric  hydrides  (c)  forming covalent halides  (d)  exhibiting amphoteric nature  in their oxides. It forms its oxide MO which becomes inert on heating.  (c)  NH 4 NO 3  (d)  NaNO 3  (2012)  2.  (b)  (b)  2.  (2002)  Answer  Key  1.  (c)  prevent action of water and salt  (d)  prevent puncturing by under­sea rocks.  (b)  (b)  4. the two  elements differ in  9.  (2004)  11.  (c)  11.  (b)  .  5.  6. Then  M is  (a)  Mg  (b)  Ba  (c)  Ca  (d)  Be.  The  set  representing the  correct order  of  ionic  radius is  (a)  Li +  > Be 2+  > Na +  > Mg 2+  (b)  Na +  > Li +  > Mg 2+  > Be 2+  (c)  Li +  > Na +  > Mg 2+  > Be 2+  (d)  Mg 2+  > Be 2+  > Li +  > Na +  (2009)  3.  Beryllium  and  aluminium  exhibit  many  properties  which  are  similar.  7.57  s­Block  Elements  CHAPTER  s­BLOCK ELEMENTS 15  1.  (2003)  The solubilities  of carbonates decrease down  the  magnesium  group due to a decrease in  (a)  lattice energies of solids  (b)  hydration energies of cations  (c)  inter­ionic  attraction  (d)  entropy  of  solution  formation.  8.  (2003)  In curing cement plasters water is sprinkled from time to time.  (2003)  The substance not likely to contain CaCO 3  is  (a)  a marble statue  (b)  calcined gypsum  (c)  sea shells  (d)  dolomite.  (2003)  A  metal  M  readily  forms  its  sulphate MSO 4  which  is  water­  soluble.  KO 2 (potassium super oxide) is used in oxygen cylinders in space  and submarines because it  Several blocks of magnesium are fixed to the bottom of a ship  (a)  absorbs CO 2  and increases O 2  content  to  (b)  eliminates moisture  (a)  keep away the sharks  (c)  absorbs CO 2  (b)  make the ship lighter  (d)  produces  ozone.  Which of the  following  on  thermal  decomposition yields  a  basic as well as an acidic  oxide?  (a)  KClO 3  (b)  CaCO 3  7.  (2004)  10.  (a)  6.  (d)  5.  8.  9.  (2002)  One mole of magnesium nitride on the reaction with an excess  of water gives  (a)  one mole of ammonia  (b)  one mole of nitric  acid  (c)  two moles of ammonia  (d)  two moles of nitric  acid. But.  (a)  10.  This helps in  (a)  keeping it cool  (b)  developing  interlocking  needle­like  crystals  of  hydrated  silicates  (c)  hydrating sand and gravel mixed with cement  (d)  converting sand into silicic acid.  (b)  (b)  3.  The ionic mobility of alkali metal ions in aqueous solution is  maximum for  (a)  K +  (b)  Rb +  +  (c)  Li  (d)  Na +  (2006)  4.  Being a lighter element. hence the  increasing  order of  ionic mobility  is  Li +  < Na +  < K +  < Rb +  8. water insoluble Be(OH) 2  and BeO.  (b) : The composition of gypsum is CaSO 4∙2H    2O.  2.  3.  (b) : Water develops interlocking needle­like crystals of hydrated  silicates.g. Since the mobility of ions is inversely  proportional to the size of the  their hydrated  ions.  (b) :  7.  9.    11.58  JEE MAIN CHAPTERWISE EXPLORER  magnesium makes the ship lighter when it is fixed to the bottom  of the ship. The solubility of carbonate  of metals in water is generally low.  4. Also  Li  and Mg  are  diagonally related  and hence  the  order is  Na +  > Li +  > Mg 2+  > Be 2+ .  the  ionic  radii  decrease  due  to  increase  in  effective  nuclear  charge  as  the  additional electrons are added to the same shell.  (d) : Be forms water soluble BeSO 4 . However they dissolve in  water containing CO 2  yielding bicarbonates.  (a) : Beryllium has the valency +2 while aluminium exhibits  its valency as +3. The degree of hydration. and this solubility  decreases on going down in a group with the increase in stability  of carbonates of metals.  (a) : 4KO 2  + 2CO 2 ® 2K 2 CO 3  + 3O 2 .  10. The reactions involved are the hydration  of  calcium  aluminates  and  calcium  silicates  which  change  into  their  colloidal gels.  (b) : The alkali metal ion exist as hydrated ions M + (H 2O)    n  in  the aqueous solution. and decrease in hydration energy of the  cations.  (b)  :  Moving  from  left  to  right  in  a  period. Calcium  hydroxide binds the particles  of  calcium  silicate together while aluminium hydroxide fills the interstices  rendering the mass impervious.  (c) : Mg 3 N 2  + 6H 2 O ® 3Mg(OH) 2  + 2NH 3  6. Hence Li +  ion is mostly  hydrated e.  (b) : The stability of the carbonates of the alkaline earth metals  increases on moving down the group. At the same time. on account of its lightness. great affinity for  oxygen and toughness is used in ship.  1. however from  top to bottom the ionic radii increase with increasing atomic  number  and  presence  of  additional shells. [Li(H 2 O) 6 ] + . decreases with  ionic size as we go down the group. Be(OH) 2 is insoluble in NaOH giving sodium beryllate  Na 2BeO    2. some calcium hydroxide and  aluminium  hydroxides  are  formed  as  precipitates  due  to  hydrolysis. It does not    have CaCO 3 .  5.  (b) : Magnesium.   (b)  The  vapour at 200°C consists  mostly  of S 8  rings.  (d)  The oxidation state of sulphur is never less than +4 in  11.  In which of the following arrangements.5O2  (c)  2XeF2  + 2H2O ®  2Xe + 4HF + O2  (d)  XeF6 +  RbF ®  Rb[XeF7]  (2009)  8.  (b)  ONCl and ONO –  are  not  isoelectronic. Ge.   whereas there is no possibility of such interaction between  (c)  Silicon reacts with NaOH (aq)  in the presence of air to give  C and F in CF4.  Na 2 SiO 3  and H 2 O.  Identify the incorrect statement among the following.  (2011)  Which  of  the  following  statements  regarding  sulphur  is  10.  Which one of  the  following reactions of  xenon  compounds  is not  feasible?  (a)  XeO3  + 6HF ®  XeF6  + 3H2O  (b)  3XeF4  + 6H2O ®  2Xe + XeO3  + 12HF + 1.  7.  (c)  O 3  molecule  is  bent.  9.  (b)  Nitrogen  cannot  form  dp­pp  bond.  (d)  Chlorides of both beryllium and aluminium have bridged  chloride  structures in solid phase.  (c)  At 600°C the gas mainly  consists of S 2  molecules.  Boron cannot  form  which  one  of  the  following  anions?  3–  –  (a)  BF 6  (b)  BH 4  –  –  (c)  B(OH) 4  (d)  BO 2  (2011)  4.  (2011)  5. Sn and Pb increases steadily  in the sequence  (a)  PbX 2  =  SnX 2  =  GeX 2  =  SiX 2  (b)  GeX 2  =  SiX 2  =  SnX 2  =  PbX 2  (c)  SiX 2  =  GeX 2  =  PbX 2  =  SnX 2  (2007)  (d)  SiX 2  =  GeX 2  =  SnX 2  =  PbX 2 .  (c)  Single N — N bond is weaker than the single P – P bond.  The  correct  reason for higher B – F bond dissociation energy as compared  to that of C  – F is  (a)  smaller size of  B­atom as compared  to that of C­atom  12.  (2007)  (2009)  .  (b)  stronger s  bond between B and F in BF3  as compared to  (a)  Br 2 reacts with hot and strong NaOH solution to give NaBr  that between C and F in CF4  and H 2O.  (b)  Boric  acid is a  protonic  acid.59  p­Block  Elements  CHAPTER  16  p­BLOCK ELEMENTS 1.  Which of  the  following is  the  wrong  statement?  (a)  Ozone is diamagnetic gas.  (c)  Beryllium  exhibits coordination number of  six.  BF3  than that between C  and F in CF4.  Which of the following exists as covalent crystals in the solid  state?  (a)  Phosphorus  (b)  Iodine  (c)  Silicon  (d)  Sulphur  (2013)  2.  (2008)  Among the following substituted silanes the one which will  give  rise to  cross linked silicone polymer  on hydrolysis is  (a)  R 3 SiCl  (b)  R 4 Si  (c)  RSiCl 3  (d)  R 2 SiCl 2  (2008)  The stability of dihalides of Si.  The bond dissociation energy of B – F in BF3  is 646 kJ mol –1  whereas  that of  C  –  F in  CF4  is  515  kJ  mol –1 .  (d)  lower degree  of  pp­pp  interaction  between  B and  F  in  (d)  Cl 2  reacts with excess of NH 3  to give N 2  and HCl.  Which of  the following  statement  is  wrong?  (a)  The stability of hydrides increases from NH 3  to BiH 3  in  group 15 of  the  periodic  table.  6. the sequence is not  strictly according to the  property written against it?  (a)  CO2  < SiO2  < SnO2  < PbO2  : increasing oxidising power  (b)  HF < HCl < HBr < HI :  increasing acid strength  (c)  NH3  < PH3  < AsH3  < SbH3  :increasing basic  strength  (d)  B < C < O < N : increasing first ionization  enthalpy  (2009)  (2013)  3.  (d)  Ozone is violet­black in  solid  state.  incorrect?  (a)  S 2  molecule  is paramagnetic.  (d)  N 2 O 4  has two resonance  structure.  Which one of  the  following  is the correct statement?  (a)  B 2 H 6 ∙2NH 3  is known as ‘inorganic  benzene’.  its compounds.    (c)  significant  pp ­pp  interaction  between  B  and  F  in  BF3  (b)  Ozone reacts with SO 2  to give SO 3.   1  and  2  lone  pairs  of  electrons  on  the  (a)  It is used to fill gas in balloons instead of hydrogen because  central  atom respectively  it is lighter and non­inflammable.60  JEE MAIN CHAPTERWISE EXPLORER  13.  (2005)  15. it gives  (a)  one sigma. two pi.  SiO  .  Aluminium chloride exists as dimer.  Which of the following statements  is true?  25. This transformation is related to  (c)  HClO 4  is a weaker acid than HClO 3 .  (2004)  (c)  two sigma. Which  (a)  each silicon atom is surrounded by four oxygen atoms and  of  the following statements  about these chlorides  is correct?  each oxygen atom is bonded to two silicon atoms  (a)  MCl 2  is more volatile than.  The molecular shapes of SF 4 .  0  and  1  lone  pairs  of  electrons  on the  (a)  sp 2  and sp 2  (b)  sp 2  and sp 3  central  atom respectively  3  2  (c)  sp  and sp  (d)  sp 3  and sp 3 .  grey powder.  (a)  an  interaction  with  nitrogen  of  the  air  at  very  low  (d)  HNO 3  is a stronger acid than HNO 2 .  The structure of diborane  (B 2 H 6 )  contains  (a)  SO 2  < P 2 O 3  < SiO 2  < Al 2 O 3  (a)  four 2c­2e bonds and two 3c­2e bonds  (b)  SiO 2  < SO 2  < Al 2 O 3  < P 2 O 3  (b)  two 2c­2e bonds and four 3c­2e bonds  (c)  Al 2 O 3  < SiO 2  < SO 2  < P 2 O 3  (c)  two 2c­2e bonds and two 3c­3e bonds  (d)  Al 2 O 3  < SiO 2  < P 2 O 3  < SO 2 .  The states of hybridisation of boron and oxygen atoms in boric  20.  each oxygen atom is bonded to two silicon atoms  (c)  MCl 2  is more ionic than MCl 4 . Al 2 Cl 6  in solid state as well  as  in  solution  of  non­polar  solvents  such  as  benzene.  The correct order of the thermal stability of hydrogen  halides  (H – X) is  soil?  (a)  HI > HBr > HCl > HF  (a)  Ammonium sulphate  (b)  HF > HCl > HBr > HI  (b)  Potassium nitrate  (c)  HCl < HF > HBr < HI  (c)  Urea  (d)  HI > HCl < HF > HBr  (2005)  (d)  Superphosphate  of  lime  (2007)  23. M forms chlorides in +2 and +4 oxidation states.  (2005)  27.  (2006)  temperatures  17. White metallic tin buttons got converted  to  (b)  In aqueous medium HF is a stronger acid than HCl.  0  and  2  lone  pairs  of  electrons  on  the  (b)  It is used as a cryogenic agent for carrying out experiments  central  atom respectively  (2005)  at low temperatures. 1 and 1 lone pair of electrons on the central  atoms respectively  29.  Which  one  of  the  following  statements  regarding  helium  is  incorrect?  (c)  different  with  0.  (b)  each silicon atom is surrounded by two oxygen atoms and  (b)  MCl 2  is more soluble in anhydrous ethanol than MCl 4 .  (2004)  (d)  four 2c­2e bonds and four 3c­2e bonds  (2005)  28.  Which of the following oxides is amphoteric in character?  (a)  HClO 3  and Cl 2O  (b)  HClO 2  and HClO 4  (a)  CaO  (b)  CO 2    (c)  HCl and Cl 2 O  (d)  HCl and HClO 3  (2006)  (c)  SiO 2  (d)  SnO 2  (2005)  16.  (a)  zero  (b)  two  (d)  It is used in gas­cooled nuclear reactors.  A metal.  The number of hydrogen atom(s) attached to phosphorus atom  (c)  It is used to produce and sustain powerful superconducting  in hypophosphorous  acid is  magnets.  Regular use of the following fertilizers increases the acidity of  22. CF 4  and XeF 4  are  acid (H 3 BO 3 ) are respectively  (a)  the same  with 2.  Among Al O .  21.  When  calcium carbide are  dissolved in water.  (d)  different  with  1.  of their uniforms.  The number and type  of bonds  between two  carbon atoms in  26.  (2004)  (b)  the same with 1.  In silicon dioxide  14.  (c)  silicon atom is bonded to two oxygen atoms  (d)  MCl 2  is more easily hydrolysed than MCl 4 . one pi  (d)  two sigma.  (2005)  .  P  O  and  SO  the  correct  order  of  acid  2  3  2  2  3  2  strength is  19.  (2004) (c)  one  (d)  three.  The soldiers of Napolean  army while at Alps during freezing  winter suffered a serious problem as regards to the tin buttons  (a)  H 3 PO 3  is a stronger acid than H 2 SO 3 . one pi  3+  + 3Cl –  (a)  Al 3+  + 3Cl –  (b)  [Al(H 2O)  (b)  one sigma.  Heating an aqueous solution of aluminium chloride to dryness  (b)  a change in the crystalline structure of tin  will give  (c)  a change in the partial pressure of oxygen in the air  (a)  AlCl 3  (b)  Al 2 Cl 6  (d)  an interaction with water vapour contained in the humid  (c)  Al 2 O 3  (d)  Al(OH)Cl 2  (2005)  air. two pi    6]    3–  (c)  [Al(OH) 6 ]  + 3HCl  (d)  Al 2 O 3  + 6HCl.  (2006)  (d)  there are double bonds between silicon and oxygen atoms.  What products are expected from the disproportionation reaction  of  hypochlorous  acid?  24. MCl 4 .  (2004)  18.   (None)  3.  33.  (a)  13. The explanation for it is that  (a)  concentrated hydrochloric acid emits strongly smelling HCl  gas all the time  40.  In case of nitrogen. 3.  Graphite  is  a  soft  solid  lubricant  extremely  difficult  to  melt.  (a)  5.  35.  26.  (a)  4. It is due to  32. (b)  31.  In  XeF 2 .  (c)  (b)  (a)  (a)  (c)  (c)  12.  41.  (c)  7.  XeF 4 .  Which one of the following statements is correct?  (a)  Manganese salts give a violet borax test in the reducing  flame. PCl  as well as PCl  are possible. (a)  25.  29.  27.  (2003)  31.  ammonia  solution dissolves only AgCl.  XeF 6  the  number  of  lone  pairs  on  Xe  are  (b)  oxygen in air reacts  with the  emitted HCl gas to form a  respectively  (a)  2.  (2003)  37.  17. concentrated hydrochloric  (b)  Absolutely pure water does not contain any ions.  The reason for this anomalous behaviour is that graphite  (a)  is a non­crystalline substance  (b)  is an allotropic  form of  diamond  (c)  has molecules of variable molecular masses like polymers  (d)  has  carbon  atoms  arranged  in  large  plates  of  rings  of  strongly bound carbon atoms with weak interplate bonds.  When H 2 S is passed through Hg 2 S we get  (a)  The mixture only  cools  down  (a)  HgS  (b)  HgS + Hg 2 S  (b)  PCl 3  and HCl are formed and the mixture warms up  (c)  Hg 2 S + Hg  (d)  Hg 2 S.  Alum helps in purifying water by  (a)  forming Si complex with clay particles  (b)  sulphate part which combines with the dirt and removes  it  (c)  coagulating the mud particles  (d)  making mud water soluble. 3  cloud of chlorine  gas  (c)  strong  affinity  of  HCl  gas  for  moisture  in  air  results  in  (c)  4.  Which one of the following  is an  amphoteric  oxide?  (a)  ZnO  (b)  Na 2O    (c)  SO 2  (d)  B 2O  (2003)    3  5  availability of vacant d orbitals in P but not in N  lower electronegativity of P than N  lower tendency of H­bond formation in P than N  occurrence of P in solid while N in gaseous state at room  temperature.  (d)  (d)  (b)  (d)  (c)  (a)  10.  (b)  From  a  mixed  precipitate  of AgCl  and AgI. 2.  acid  pulls  moisture  of  air  towards  itself.  15.  22. 1.  (c)  (c)  (d)  (b)  (c)  (c)  9.  14.  30.  24.  (d)  On boiling a solution having K + .  (d)  due to strong affinity for water. Ca 2+  and HCO 3 –  ions we  get a precipitate of K 2 Ca(CO 3 ) 2 .  This  moisture  (c)  Chemical  bond  formation  takes  place  when  forces  of  forms droplets of  water and  hence the  cloud.  (d)  6. (d)  37.  42.  38.  (c)  2.  What may be expected to happen when phosphine gas is mixed  (2002)  with chlorine gas?  42.  28.  36.  Which one of the following substances has the highest proton  (2002)  affinity?  (a)  H 2 O  (b)  H 2 S  39.  (c)  PCl 5  and HCl are formed and the mixture cools down  (d)  PH 3∙Cl  (2003)    2  is formed with warming up.  40.  34.  39.  33. 1.  36.  Concentrated hydrochloric acid when kept in open air sometimes  produces a cloud of white fumes.  (2002)  35.  (c)  (d)  (b)  (b)  (a)  (d)  11. 1  (b)  1.  (2002)  Answer  Key  1.  (2003)  (d)  In covalency transference of  electron takes  place. 2  (d)  3.  (2003)  38. NCl 3  is possible but not NCl 5  while in case  (c)  NH 3  (d)  PH 3  (2003)  of phosphorus.  (c)  Ferric  ions  give  a  deep  green  precipitate  on  adding  potassium ferrocyanide  solution.  attraction  overcome the  forces of  repulsion.  21.  (2002)  forming of droplets of liquid solution which appears like  41. 2.  (d)  (b)  (d)  (b)  (c)  (c) .  Which  one  of  the  following  pairs  of  molecules  will  have  permanent dipole moments for  both members?  (a)  SiF 4  and NO 2  (b)  NO 2  and CO 2  (c)  NO 2  and O 3  (d)  SiF 4  and CO 2  (2003)  3  (a)  (b)  (c)  (d)  34.  Glass is a  (a)  micro­crystalline  solid  (b)  super­cooled  liquid  (c)  gel  (d)  polymeric  mixture.  16.  Which of the following statements is true?  a cloudy smoke  (a)  HF is less polar than HBr.  32. (a)  19.  20.61  p­Block  Elements  30.  23.  18. (b)  8.   and +6 are more  common.  the  basic character decreases on  going down the group due to decrease in the availability of  the  lone pair of electrons  because of  the  increase in size of  elements from N to  Bi.  6.  3.  (d) : 3Br 2  + 6NaOH ® 5NaBr + NaBrO 3  + 3H 2 O  O 3  + SO 2 ® O 2  + SO 3  Si + 2NaOH + O 2 ® Na 2SiO    3  + H 2O   . proton  donor)  but behaves as a  Lewis acid by accepting a pair of electrons from OH –  ions.  Borazine  The size of the central atom increases from N to Bi therefore.  the tendency to form a stable covalent bond with small atom  like  hydrogen decreases  and  therefore.  it is called inorganic  benzene.  2B3N3H6 + 12H 2  Borazine  has  structure similar to benzene  and therefore.  (c) :  RSiCl 3  on hydrolysis gives a cross linked  silicone.C – F.  (None) :  All the statements are  correct. Hence option (d)  is  correct.  stability decreases.  (a) : Due to non­availability of d­orbitals.  –  +  B(OH) 3  + 2H 2 O ® [B(OH) 4]    +  H 3O    BeCl 2  like Al 2Cl    6  has a bridged polymeric structure in solid  phase generally as  shown below. +4. It is a notable part that boric acid does not  act as a protonic acid  (i. +6 oxidation states but +4  Cl  .  3B2H6 + 6NH 3  3[BH2(NH3)2 ]+ BH 4 –  or  B2H6  ∙ 2NH 3  or  Cl  11. +2.  So the stability also decreases.E. 450 K  room  temp.  (c) :  In group15 hydrides.  (c) : Due to the inert pair effect (the reluctance of ns 2  electrons  of  outermost  shell  to  participate in  bonding)  the  stability  of  M 2+  ions (of group IV elements) increases as we go down the  group.  Also.e. B is sp 2  hybridised and has a vacant 2p­orbital  (c) : In BF3 which overlaps laterally with a filled 2p­orbital of F forming  strong  pp­pp  bond.  8.  12. the maximum covalency of boron  cannot  exceed  4.  Beryllium exhibits coordination number of four as it has only  four available orbitals in its valency shell.62  JEE MAIN CHAPTERWISE EXPLORER  1.  1300°C 440°C  280°C  150°C  7.  Cl  Cl –  Be  Cl  Cl  Cl  Cl  Cl  Al  Al  Cl  Polymeric structure of Al2 Cl 6  OH  R  (ii)  HO – Si – OH + H O – Si – OH + H O – Si –  OH  OH  OH  R  –H2 O  R  HO – Si – O – Si – O – Si –  OH  OH  R  R  OH  R  OH  OH  R  (iii)  HO – Si – O – Si – O – Si – OH  –3H2 O  OH  OH  OH  H O  H O  H O  HO – Si – O – Si – O – Si – OH  R  R  R  R  R  R  – O – Si – O – Si – O – Si – O –  O  O  O  – O – Si – O – Si – O – Si – O –  R  R  R  Cross linked silicone  Be –  Cl  Polymeric structure of BeCl 2  OH  R – Si –  OH  R  (a) : The reaction is not  feasible because XeF6  formed will  XeF6  + 3H2O  9.  3H2 O  (i)  R – Si –  Cl  –3HCl  Cl  R  XeO3  +  3H2F2.B – F  > B. Thus. Therefore.  Decomposition  temperature  5.  (a) : Thermal stability decreases gradually from NH 3  to BiH 3 .  NH 3  PH 3  AsH 3  SbH 3  BiH 3  4.  10..  (d) : Boric acid is a  weak monobasic  acid  (K a  = 1.  (c)  2.  C  does  not  have  any  vacant p­orbitals to  undergo p­bonding.  Thus B.  However  in  CF4. boron is unable to  expand its octet.  correct order  of  basicity is  NH3  > PH3  > AsH3  >  SbH3.  further produce  XeO3  by  getting  hydrolysed.0 × 10 –9 ).  B2H6 ∙ 2NH 3  Heat.  The  formation can be  explained  in three  steps  (d) : Sulphur exhibits –2.E.  thermal stability  of  HX  The crystallisation of AlCl 3  from aqueous  solution.  the four orbitals of each of the boron atom overlap with two  terminal hydrogen atoms forming two normal B – H s­bonds. Due to successive decrease in the strength  +  –  2AlCl 3  + aq.  H  .  (b) : Hypophosphorous acid  (in ClO 3 – ) and –1 (in Cl – ).  2– . SiCl    4.  XeF 4  (sp 3 d 2 .  (b) : As the  size  of the  halogen  atom increases  from  F to I.  H – F to H – I (H – F < H – Cl < H – Br < H – I).  2NH 3  + 3Cl 2  N 2  + 6HCl  6NH 3  + 6HCl  6NH 4 Cl  8NH 3  + 3Cl 2  N 2  + 6NH 4 Cl  13.  O  O  18.  Now.  H  H  .  .  F  (XeF 4 )  O  16.  molecules also decreases from HF to HI  (HF > HCl > HBr >  yields  an  ionic  solid  of  composition  [AlCl 2 (H 2 O) 4 ] +  HI).  state are ionic  in nature and behave as reducing agent.  (d): CaO­basic. e.  HO  H  H  Hence. CO 2  and SiO 2­acidic.  > H – I).  F  F  Further as we move down the group. trigonal bipyramidal with one equatorial position  SnCl 4.  .  . 1s orbital of hydrogen atoms (bridge  atom) and one of hybrid orbitals of the other boron atom overlap  to form a delocalised orbital covering the three nuclei with a  pair of electrons.  .  Number of hydrogen atom(s) attached to phosphorus atom = 2. B  B  B  B  .  SnCl 2 . two lone pairs).  .  Such a  bond is known  as three  centre  two  electron (3c – 2e) bonds.  H .  (a) :  (NH 4 ) 2 SO 4  + 2H 2 O  (2H +  + SO 4 2– ) + 2NH 4 OH  Strong acid  Weak base  (NH 4)    2SO    4  on hydrolysis produces strong acid H 2SO    4. Hence H 2 SO 3  22. whereas those which show +2 oxidation    occupied by 1 lone pair). no lone pair). which    increases the acidity of  the soil.  (a) : According  to  molecular  orbital  theory.  (c) : The elements of group 14 show an oxidation state of +4  Structure  of  diborane  and  +2.g. H  H  14. therefore.  20.  H .  2–  .  (d) : 3HClO 4(aq) ® HClO 3(aq)  + 2HCl (aq)  (CF 4 )  (SF 4 )  It is a disproportionation reaction of hypochlorous acid where  –  the oxidation number of Cl changes from +1 (in ClO  ) to +5  21.  (d) : SF 4 (sp 3 d. SnO    2­amphoteric.  H  –  X  bond  length  in  HX  molecules  also  increases  from  is a stronger acid than H 3 PO 3 . .  Al 2 Cl 6  + H 2 O  .  . greater  P  is the acidity.g.  1 s  .  (b) : Aluminium chloride in aqueous solution exists as ion  from the fact that  H – X bond dissociation energies decrease  pair. Two of  reacts both with acids and bases. PbCl 2 .  H  .  from H – F to H – I.  .63  p­Block  Elements  Cl 2  reacts  with  excess  of  ammonia  to  produce  ammonium  chloride and nitrogen.  One of the remaining hybrid orbital (either filled or empty) of  one of the boron atoms. tetrahedral.  Due to higher dissociation energy of H – F bond and molecular  The increase in  H –  X bond length decreases  the  strength  of  association due  to  hydrogen  bonding in  HF.  It is  a  network  190°C to give the non­ionic dimer Al 2Cl    6. as it    three have one electron each while the fourth is empty. square planar. etc. e.  .  24. .  HNO 3  is a stronger  acid than HNO 2 . greater is the electronegativity and higher is the oxidation  state of the central atom. H  . CF 4  (sp 3 .    heat  solid in which each Si atom is surrounded tetrahedrally by four  +  –  [AlCl 2(H    2 O) 4 ]  [AlCl 4 (H 2 O) 2 ]  ∙ xH 2 O  190°C  oxygen  atoms.    23. PbCl    4.  H  H  H . the tendency of the element  F  F  F  to  form  covalent  compound  decreases  but  the  tendency  to  C  S  Xe  form  ionic  compound  increases. The  compounds  showing  an  oxidation  state  of  +4  are covalent compound and have tetrahedral structures. greater is the acidity.  Ca 2+  [  C      C  ]  2 p  – O – Si – O – Si – O –  O  O  19.  (b) : Calcium carbide is ionic carbide having  [  C      C  ]  .  each  of  the  two  boron atoms is in sp 3  hybrid state. etc.  HF  is a weaker  H – X bond from H – F to H – I  (H – F > H – Cl > H – Br  acid than HCl.  (d) : Higher is the oxidation state of the central atom.  . The decrease in the strength of H – X bond is evident  17.  (a)  :  Silicon dioxide  exhibits polymorphism.  . HClO 4  is a stronger  acid than HClO 3 .  –  [AlCl 4 (H 2O)  This  compound  decomposes  at  about    2]    ∙ xH 2O.  . Of the four hybrid orbitals.  F  F  F  F  F  F  15. ® [AlCl 2(H    2O)    4]    (aq)  + [AlCl 4(H    2O)    2]    (aq)  of H – X  bond from H  – F  to H  –  I.   accepting  proton  to  form  ammonium ion as it has tendency to donate an electron pair.  be  decanted  off.  Grey tin  White tin  (cubic)  (tetragonal)  33. AgBr and AgI at the room  temperature are 2.0 × 10 –13 and 8.  each carbon atom makes a use of sp 2  hybridisation.  since  the  bonding  between  the  layers  XeF 6  sp 3 d 3  1 lone pair  involving only weak van der Waal's forces.  (a) : Helium is twice as heavy as hydrogen.  3 O  .  +  H – N   + H  . its lifting power is  92 percent of that of  hydrogen.  AgCl + 2NH 4OH ®  [Ag(NH 3)      2]Cl + 2H    2O    Diammine  silver  chloride  30.  (b) :  Grey tin  is very  brittle  and  easily  crumbles  down to  a  powder in very  cold climates. .  sp  .  26.  . This is called tin  disease  or tin  plague.  34. SiO 2  is slightly acidic whereas P 2O    3  and  35.  sharing of electrons between two non­metal atoms takes place. Ba.  (c)  :  Ammonia  is  a  Lewis  base.  and  cryogenic  research  where  temperatures  close  to  absolute  zero are needed.8 × 10 –10 . .  (b) : Al 2 Cl 6  + 12H 2 O  2[Al(H 2 O) 6 ]  +  6Cl  ZnO + H 2 SO 4 ®  ZnSO 4  + H 2 O  27.  (c) : Due to the higher electronegativity of F.  H  O  .  H  .  36. In covalency.  having a general formula R 2 O∙MO∙6SiO 2  where R = Na or K  39. This gives softness.  H  H  +  H – N ® H  H  The change of white tin to grey tin is accompanied by increase  in  volume.5 × 10 –17 respectively. Zn or Pb.  (b) :  H  . sp 3  29.  (c) : HCl gas in presence of moisture in air forms droplets of  SO 2  are the anhydrides of the acids H 3 PO 3  and H 2 SO 3 .  37.  (c) : The negatively charged colloidal particles of impurities get  neutralised by the Al 3+  ions and settle down and pure water can  solid  solution  (supercooled  liquid)  of  silicates  and  borates.  PH 3  +  4Cl 2 ®  PCl 5  +  3HCl  2  B sp  28.  Further.  2  2  6  2  3  15 P = 1s  2s  2p  3s  3p  31.  32.  they are even less soluble than AgCl.  (c) have permanent dipole moment.  Thus..  (d)  : Acidity  of  the  oxides  of  non  metals  increases  with  the  zinc sulphate  electronegativity and  oxidation  number  of  the  element.  ZnO + 2NaOH ®  Na 2 ZnO 2  + H 2 O  Al 2O    3  < SiO 2  < P 2O    3  < SO 2  sodium zincate  Al 2O    3  is amphoteric..  phosphorus  pentachloride. 5.64  JEE MAIN CHAPTERWISE EXPLORER  SnO 2  + 4HCl ® SnCl 4  +  2H 2O    SnO 2  + 2NaOH ® Na 2 SnO 3  + H 2 O  25...  (b) : Glass is a transparent or translucent amorphous supercooled  38.  (c) : Phosphine burns in the atmosphere of chlorine and forms  sp 3  O .  (d) : Graphite has a two­dimensional sheet like structure and  In phosphorus the 3d­orbitals are available. Due to greater solubility  of AgCl than AgI.  character of graphite.  (a) : ZnO is an amphoteric oxide and dissolves readily in acids  3+  –  forming corresponding zinc salts and alkalies forming zincates.  (a) :  7 N = 1s 2  2s 2  3p 3  and M = Ca.  40.  (c) : NO 2  and O 3  both have unsymmetrical structures.  H  .  (d) :  XeF 2  sp 3 d  3 lone pairs  The above layer structure of graphite is less compact than that  XeF 4  sp 3 d 2  2 lone pairs  of  diamond. HF is more polar  than HBr pure water contains H +  and OH –  ions.  Consequently. so they  42.  liquid solution in the form of cloudy smoke. these layers can slide  over each other. greasiness and lubricating  41. ammonia solution dissolves only AgCl and  forms a  complex.  Helium has the lowest melting and boiling points of any element  which makes liquid helium an ideal coolant for many extremely  low­temperature applications such as superconducting magnets.  (b) : The solubility product of AgCl. AgI is the least soluble silver halide.  The lattice energies of AgBr and AgI are even higher because  of greater  number of  electrons in  their anions.   Four successive members of the first row transition elements  are listed below with atomic numbers.65  d­ and f­Block Elements  CHAPTER  17  d­ and f ­BLOCK ELEMENTS 1.  (2011)  (d)  more energy difference between 5f and 6d than between  4f and 5d­orbitals.  Which of the following arrangements does not represent the  7.  Fe  and Co  is  (a)  Cr > Mn > Fe > Co  (b)  Mn > Cr > Fe > Co  11. Which of the following  statements about iron  is incorrect?  (a)  Ferrous  compounds  are  relatively  more  ionic  than  the  8.  The actinoids exhibit more number of oxidation states in general  (c)  Cr > Fe > Mn > Co  (d)  Fe > Mn > Cr > Co  than the lanthanoids. the main reason  being  (b)  All the  members exhibit  +3  oxidation state. will the magnitude of D oct  be  the highest?  (a)  [Co(NH 3 ) 6 ] 3+  (b)  [Co(CN) 6 ] 3–  3–  3+  (c)  [Co(C 2O  (d)  [Co(H 2O)  (2008)    4)    3]      6]    In context of the lanthanoids.  (2013)  2. all the 4s and 3d electrons are used  for  bonding.  (2009)  In  which  of  the  following  octahedral  complexes  of  Co (At. Mn. the transition metals show  basic character  and form  cationic complexes.  (d)  Ferrous oxide is more basic in nature than the ferric oxide.  (a)  more reactive nature of the actinoids than the lanthanoids  (c)  Because of similar properties the separation of lanthanoids  (b)  4f orbitals more  diffused than the 5f  orbitals  is not easy.  (b)  The  ionic  sizes  of  Ln(III)  decrease  in  general  with  increasing atomic number.  (d)  Once the d 5  configuration is exceeded.  The  correct  order  of  E° M2+  values  with  negative  sign  for    /M  (2008)  the  four successive  elements  Cr.  Larger number of oxidation states are exhibited by the actinoids  with increasing  atomic number in the  series.  (c)  In the highest oxidation states of the first five transition  elements (Sc to Mn).  (2012)  3. (2009)  Knowing that the chemistry of lanthanoids (Ln) is dominated  by its +3 oxidation state.  correct order of the  property stated  against  it?  (a)  Sc < Ti < Cr  < Mn  : number of oxidation  states  (b)  V 2+  < Cr 2+  < Mn 2+  < Fe 2+  : paramagnetic behaviour  (c)  Ni 2+  < Co 2+  < Fe 2+  < Mn 2+  : ionic size  (d)  Co 3+  < Fe 3+  < Cr 3+  < Sc 3+  : stability in aqueous solution.  (d)  Ln(III) hydroxides are mainly basic in character.  In context with the transition elements. Which one of them is  expected to have  the  highest  E° M3+    /M 2+  value?  (a)  Co (Z  = 27)  (b)  Cr (Z  = 24)  (c)  Mn (Z = 25)  (d)  Fe (Z = 26)  (2013)  Iron exhibits +2 and +3 oxidation states. which of the following statement  is  not  correct?  (a)  There is a gradual decrease in the radii of the members  10. which of the following  statements is incorrect?  (a)  In  addition  to  the  normal  oxidation  states.  corresponding  ferric  compounds. the tendency to  involve all the 3d  electrons in  bonding decreases. This is because  (2010)  .  than those by the lanthanoids.  (c)  Ferrous compounds  are  more  easily  hydrolysed than  the  corresponding  ferric  compounds. 27).  (c)  Ln(III) compounds are  generally colourless.  6.  (c)  lesser energy difference between 5f and 6d than between  (d)  Availability of 4f  electrons  results  in  the formation  of  4f  and 5d  orbitals  compounds in +4 state for all the members of the series.  The  outer electronic  configuration  of  Gd  (Atomic  No :  64)  is  (a)  4f  3 5d 5 6s 2  (b)  4f 8 5d 0 6s 2  4  4  2  (c)  4f  5d  6s  (d)  4f  7 5d 1 6s 2  (2011)  9.  (b)  Ferrous  compounds  are  less  volatile  than  the  corresponding  ferric  compounds. no.  (b)  In the highest oxidation states.  the  zero  oxidation  state  is  also  shown  by  these  elements  in  complexes.  4. which of the following statements  is  incorrect?  (a)  Because of the large size of the Ln(III) ions the bonding  in its compounds is predominantly ionic in character.  5.   (c)  sodium  (d)  magnesium.  (d)  Cerium (IV) acts as an oxidising agent.  The lanthanide contraction is  responsible for  the fact that  (c)  Manganese nitrate  (a)  Zr and Y have about the same radius  (d)  Silver nitrate.  (a)  Poor  shielding  of  one  of  4f­electron  by  another  in  the  These are obtained by floating molten glass over a liquid metal  subshell.  on strong heating?  (2006)  (a)  Ferric nitrate  (b)  Copper nitrate  16.  (2007)  (b)  The +3 oxidation state of cerium is more stable than +4  13.  oxidation state.) among is  (a)  [MnCl 4 ] 2–  > [CoCl 4 ] 2–  > [Fe(CN) 6 ] 4–  (b)  [MnCl 4 ] 2–  > [Fe(CN) 6 ] 4–  > [CoCl 4 ] 2–  12.  Identify the incorrect statement among the  following:  (c)  [Fe(CN) 6 ] 4–  > [MnCl 4 ] 2–  > [CoCl 4 ] 2–  (a)  4f­and 5f­orbitals are equally shielded.  The oxidation state of chromium in the final product formed by  27. (d)  Evolved I 2  is reduced.  (c)  +2  (d)  +3  (d)  CrO 42–  (2003)   is oxidised to +7 state of Cr.  (2004)  15.  (a)  The common oxidation states of cerium are +3 and +4.40 Å  (c)  1.  The  correct  order  of  magnetic  moments  (spin  only  values  in  B.  Which of the following statements about cerium is incorrect?  (d)  The chemistry of various lanthanoids is very similar.  respectively  (a)  one.66  JEE MAIN CHAPTERWISE EXPLORER  (a)  the 5f orbitals extend further from the nucleus than the 4f  orbitals  (b)  the 5f orbitals are more buried than the 4f orbitals  (c)  there  is  a  similarity  between  4f  and  5f  orbitals  in  their  angular part of the  wave function  (d)  the actinoids are more reactive than the lanthanoids.  Nickel (Z = 28) combines with a uninegative monodentate ligand  –  2–  (a)  Cu 2I  (b)  CuI 2  is formed.  The  number  of    2  is formed.: Mn = 25.  Of the following outer electronic configurations of atoms.73  (2006)  solution is added to it. The metal used can be  (b)  Effective shielding of one of 4f­electrons by another in the  (a)  mercury  (b)  tin  subshell.  (m B )] of Ni 2+  in aqueous solution would be (atomic number of  (c)  The +4 oxidation state of cerium is not known in solutions.  Heating mixture of Cu 2 O and Cu 2 S will give  (a)  Cu + SO 2  (b)  Cu + SO 3  (c)  CuO + CuS  (d)  Cu 2SO    3  (2005)  20.  Which one of the following nitrates will leave behind a metal  (d)  Greater shielding of 5d electrons by  4f­electrons.  Calomel (Hg 2 Cl 2 ) on reaction with ammonium hydroxide gives  (a)  HgNH 2 Cl  (b)  NH 2  – Hg – Hg – Cl  (c)  Hg 2 O  (d)  HgO  (2005)  of Lu 3+  (Atomic number of Lu = 71)?  (a)  1.  What would happen when a solution of potassium chromate is  treated with an excess of dilute nitric acid?  the  reaction  between  KI  and  acidified  potassium  dichromate  2–  (a)  Cr 3+  and Cr 2O    7  are formed.  partially filled  orbitals.  solution is  2–  (b)  Cr 2 O 7  and H 2 O are formed.  (b)  d­Block  elements  show  irregular  and  erratic  chemical  (Atomic nos.  Excess  of  KI  reacts  with  CuSO 4  solution  and  then  Na 2 S 2 O 3  (c)  0  (d)  1.  For making good quality mirrors. Fe = 26. plates of float glass are used. the  (c)  one.  (2005)  .90  22.  Cerium (Z  = 58) is an important member of the lanthanoids.84  (b)  4. tetrahedral  23.  (2003)  (c)  Poorer shielding of  5d  electrons  by  4f­electrons.  X  to  form  a  paramagnetic  complex  [NiX 4 ]  .  (2004)  (c)  La  and  Lu  have  partially  filled  d­orbitals  and  no  other  21.  (2007)  19. Which of the statements is incorrect for  this  reaction?  14.06 Å.  (2003)  (b)  Zr and Nb have similar oxidation state  (c)  Zr and Hf have about the same radius  26.  The radius of La 3+  (Atomic number of La = 57) is 1.  which does not solidify before glass.  (2004)  Ni = 28)  (a)  2.  25.06 Å  (d)  0. square planar  (d)  two.  (a)  +4  (b)  +6  (c)  CrO 42–    is reduced to +3 state of Cr.M.  (d)  [Fe(CN) 6 ] 4–  > [CoCl 4 ] 2–  > [MnCl 4 ] 2– . square planar.85 Å  (2003)  18.  unpaired electron(s) in the nickel and geometry of this complex  2  2  3  (2004)  ion are.60 Å  (b)  1.  (c)  Na  S  O  is oxidised. Which  (d)  Zr and Zn have the same oxidation state. tetrahedral  (b)  two.  The “spin­only” magnetic moment [in units of Bohr magneton.  highest oxidation state is achieved by which one  of them?  (2006)  (a)  (n – 1)d 8 ns 2  (b)  (n – 1)d 5 ns 1  3  2  (c)  (n – 1)d  ns  (d)  (n – 1)d 5 ns 2 . Co = 27)  properties among themselves.  Which of the  following  factors  may  be  regarded  as  the main  cause of  lanthanide  contraction?  24.  (2005)  one of the following given values will be closest to the radius  17.   34.  13.  (2003)  Answer  Key  1.  Fe = 26)  ions is  (a)  3  (b)  4  (c)  5  (d)  6  (2003)  32.  How do we differentiate between Fe 3+  and Cr 3+  in group III?  (a)  By taking excess of NH 4 OH solution.  35.  29.  23.  9.  14.  17.  32. +5  (2002)  results in liberation of some violet coloured fumes and droplets  of a metal appear on the cooler parts of the test tube.  10.  12.  (2003)  (2002)  35.  25. La 3+ .  (d)  Both (b) and (c).  chromium  (Cr).  20.  The number of d­electrons  retained  in Fe 2+  (At.  (b)  (a)  (d)  (a)  (c)  (a) .  33. 24. Pm 3+  and Yb 3+  in increasing order of their  ionic radii.  (b)  (b)  (a)  (a)  (d)  (b)  2. +3  (b)  +2.  22.  The  atomic  numbers  of  vanadium  (V).67  d­ and f­Block Elements  28. However it becomes soluble  (a)  +2.  Which  of  the  following  ions  has  the  maximum  magnetic  moment?  solid is  (a)  Mn 2+  (b)  Fe 2+  (a)  (NH 4 ) 2 Cr 2 O 7  (b)  HgI 2  2+  (c)  Ti  (d)  Cr 2+  (2002)  (c)  HgO  (d)  Pb 3 O 4 .  8. +4  (d)  +3.  (d)  (c)  (c)  (b)  (d)  (a)  5.  A red solid is insoluble in water.  16.  (a)  Yb 3+  < Pm 3+  < Ce 3+  < La 3+  (b)  Ce 3+  < Yb 3+  < Pm 3+  < La 3+  (c)  Yb 3+  < Pm 3+  < La 3+  < Ce 3+  (d)  Pm 3+  < La 3+  < Ce 3+  < Yb 3+  30.  (c)  By decreasing OH –  ion concentration.  24.  18. Heating the red solid in a test tube  (c)  +3.  The most stable ion is  (a)  [Fe(OH) 3 ] 3–  (b)  [Fe(Cl) 6 ] 3–  second ionisation  enthalpy?  3–  3+  (c)  [Fe(CN) 6]  (d)  [Fe(H 2O)  (2002)      6]    (a)  V  (b)  Cr  (c)  Mn  (d)  Fe  (2003)  34.  (a)  (c)  (b)  (a)  (d)  (d)  3.  31.  27.  11.  A reduction in atomic size with increase in atomic number is a  characteristic of  elements  of  (a)  high atomic masses  (b)  d ­block  (c)  f ­block  (d)  radioactive series. +4  if some KI is added to water.  28.  no.  Arrange Ce 3+ .  (2002)  manganese (Mn) and iron (Fe) are respectively 23.  (c)  (b)  (a)  (c)  (b)  (b)  4.  (d)  (a)  (a)  (b)  (b)  (c)  6.  19. The red  36.  29.  36.  21.  7. 25 and  26.  15. Which  one of these  may  be  expected  to  have  the highest  33.  Most common oxidation states of Ce (cerium) are  31.  30.  26.  (b)  By increasing NH 4+   ion concentration.  Hence shielding of 4f is  Cl  {  more than 5f.  the total  lanthanide contraction.  5.  etc;  possess  almost  the  same  electron decreases in actinides. Therefore all these three subshells can participate.  17. An  4f  orbital is  NH 2  Hg 2Cl  ®  Hg  + Hg + NH 4 Cl + 2H 2 O    2  + 2NH 4OH   nearer to the nucleus than 5f orbitals. The black substance is a mixture of mercury and mercuric  nucleus on the valency electrons due to presence of electrons  amino  chloride. atomic  number  weaker field ligand  than  NH 3  and C 2 O 42–    therefore 3+  3–  3+  increases by one unit and the addition of one electron occurs  D oct  [Co(H 2 O) 6 ]  < D oct [Co(C 2 O 4 )]  < [Co(NH 3 ) 6 ]  at  the  same time  in  4f­energy  shell.  (a)  :  Calomel  on  reaction  with  ammonium  hydroxide  turns  12.91 V  E° Fe 2+ /Fe  =  – 0.  4. Thus.28 V  (b) : When the transition metals are in their highest oxidation  state.  [NiX 4 ] 2– ®  4p  × ×  × ×  × ×  × ×  sp 3  hybridisation  Number of unpaired electrons = 2  Geometry = tetrahedral. 6d and 7s energy levels are of comparable  16. +4. +5. +6 and +7.  (a)  :  The  decrease  in  the  force  of  attraction  exerted  by  the  black.  3. This brings the valence shell nearer to the nucleus and  10. The  pairs  of  elements  4f orbitals (lanthanides) hence the hold of nucleus on valence  such  as  Zr­Hf. Colour appears due to presence  of unpaired  f­electrons  which undergo  f­f  transition.  of second and third transition series resemble each other more  11.  2.  6. the elements  energies.  to the fact that the 5f.  (c) : Actinoids show  different  oxidation  states  such  as +2.  Black .18 V  E° Cr 2+ /Cr  = – 0.e.  Nb­Ta.  hence the size of atom or ion goes on decreasing as we move  +3. For this reason the actinoides  properties. the 4f­electrons shield each other  are  given below :  quite poorly from the nuclear charge.  they no longer have  tendency to give away electrons.  (a) : As we proceed from one element to the next element in  the lanthanide series. However +3 oxidation state is most  in the series.  On  account  of  the  very  Common ligands in order of increasing crystal field strength  diffused shapes of  f­orbitals. i.  in  the  inner shells  is called  shielding  effect.  The wide range of oxidation states of actinoids is attributed  9.  (b) :  Strong field ligand  such as  CN – .  14.  3d  4p  4s  (a)  (c) :  Ferrous  oxide  is  more basic. The sum of the successive reactions  is equal to  common among all the actinoids. the nuclear charge.  (c) : In each vertical column of transition elements.  H 2 O  is  a  15.  7.44 V  Ni 2+ ®  E° Co 2+ /Co  = – 0.  thus they are  not basic but show  acidic  character and form  anionic  complexes. the effect of nuclear  I –  < Br –  < Cl –  < F –  < OH –  < C 2 O 4 2–  < H 2 O < NH 3  < en < NO 2 –  charge increase is somewhat more than the changed shielding  < CN –  effect.  exhibit more number of  oxidation states  in general.68  1.  8.  less  volatile  and less easily hydrolysed  than  ferric  oxide.  (b) : Ni ®  (b) : E° Mn 2+ /Mn  = – 1.  (c) : Ln 3+  compounds are generally coloured in the solid state  as well as in aqueous solution.84  54  7  1  2  64 Gd = [Xe]  4f  5d  6s  (d) : Availability of 4f electrons does not result in the formation  3d  4s  of compounds in +4 state for all the members of the series.  JEE MAIN CHAPTERWISE EXPLORER  (b) : Number of unpaired electrons in Fe 2+  is less than Mn 2+ .  more  ionic.  (a)  :  As  the  distance  between  the  nucleus  and  5f  orbitals  closely than the elements of first and second transition series  (actinides) is more than the distance between the nucleus and  on  account  of  lanthanide  contraction.  Number of unpaired electrons (n) = 2  (d) :  The  electronic  configuration  of  m = n(n + 2) = 2(2 + 2) = 8 »  2.  13.  usually produce low  spin  complexes  and  large  crystal  field  splittings.  Mo­W.  (a) :  28 Ni ® [Ar] 3d 8  4s 2  so Fe 2+  is less paramagnetic than Mn 2+ .   Magnetic moment =  n n + 2  31.  .  i.  21.    2K 2CrO    4  + 2HNO 3 ®  K 2 Cr 2O    7  + 2KNO 3  + H 2 O  or. This  causes a reduction in the size of the entire 4f n  shell.  .  .  .  36.  As we move through the lanthanide series.  19.  [Fe(CN) 6 ] 4– ®  .  (d) :  26 Fe = 1s 2  2s 2  2p 6  3s 2  3p 6  3d 6  4s 2  Fe 2+  = 1s 2  2s 2  2p 6  3s 2  3p 6  3d 6  The number of d ­electrons retained in Fe 2+  = 6.  hence  solubility  product  of  Fe(OH) 3  is  attained.  potassium iodide forming a complex.  (c) : With increase  in  atomic number  i.85 Å) is the ionic radius of Lu 3+ .  .  .  .  (d) : When heated at red heat.69  d­ and f­Block Elements  28.e. The mutual shielding effect of f electrons  is  very  little.  .  (a) : Mercury is such a metal which exists as liquid at room  temperature.  144244 3  sp 3  Number of unpaired  electrons = 3  . Cl –  is most basic amongst all. Nos. Ce 4+  is  stable  due to 4 f 0  configuration.  .  .  .  Hg   +   I 2  HgI 2  –1  22. This is  because of the electronic  configuration of Cu +  which  is  3d 10  (completely filled) and  of Cr +  which is 3d 5  (half­filled).  (b) : The second ionisation potential values of Cu and Cr are  sufficiently higher than those of neighbouring elements. (d) :  +6  +3  2–  Cr 2 O 7  +  14H + + 6I –  2Cr 3+  + 7H 2 O + 3I 2  29.  33.  (a) : According to their positions in the periods.e.  (d) : NH 4 +  ions are increased to suppress release of OH –  ions. Thus the lowest  35.  (b) :  4KI + 2CuSO 4  0  0  I 2  + Cu 2 I 2  + 2K 2 SO 4  +2.  (a) :  [MnCl 4 ] 2– ®  - 3d  4s  - . greater the number of unpaired electrons.  2CrO 4 2–  yellow  H +  2–  Cr 2 O 7  + H 2 O  orange  these oxidation states are only stable in those cases where stable  4 f 0 .  .  144244 3  sp 3  Number of unpaired  electrons = 5  .  70  61  58  57  silver.  (b) :  violet vapours  32.  (c) : The common stable oxidation state of all the lanthanides  value (here 0.  . .  . the electron is to be removed  from very  stable configurations.  .  (n – 1)d 5  ns 2  can achieve the maximum oxidation state of +7.  .  - - -  4p  .  [CoCl 4 ] 2–®  .  (a) : Mn 2+  (3s 2 3p 6 3d 5 ) has the maximum number of unpaired  electrons (5) and therefore has maximum moment.5  +2  I 2  + 2Na 2 S 2 O 3  –1  Na 2  S 4 O 6  + 2NaI  (n – 1)d  ns  23. the number of the principal shell increases and therefore.  (lanthanide ions) decreases from La 3+  to Lu 3+ .. however increases by one at each step.  (b) : A more basic ligand forms stable bond with metal ion. greater will be the  HgI 2  + 2KI ® K 2 HgI 4  paramagnetic character.  i.  . The oxidation states of +2 and +4 are also exhibited and  27. K 2HgI    4 .  . 4f electrons are being  added one at each step. 4 f 7  or 4 f 14  configurations are achieved.  .  for the second ionisation potentials.  Yb 3+  <  Pm 3+  <  Ce 3+  < La 3+  25. This  is  due  to  the  shape  of  the  f  ­orbitals.  Colour  of  precipitate  is  different.  the inward pull experienced by the 4f electrons increases.  . AgNO 3  decomposes to metallic  At.  The  nuclear charge.  the  ionic  radii  of  Ln 3+  contraction.  18.  in moving  down  a  group.  is +3.  .  (c) : +4 oxidation state of cerium is also known in solutions.  (b) : Dilute nitric acid converts chromate into dichromate and  H 2O.  Ionic radii (pm)  86  98  103  106  2AgNO 3 ®  2Ag + 2NO 2  + O 2  Ionic  size  decreases  from  La 3+  to  Lu 3+  due  to  lanthanide  26.  34.  (d)  :  Due  to  lanthanide  contraction.  these values  are in the order:  24. But in case of f ­block elements  there is a steady decrease in atomic size with increase in atomic  number due to lanthanide contraction.  20.  HgI 2  on heating liberates I 2  gas. .e.  the size of the atom increases.  144244 3  d 2 sp 3  Number of unpaired  electrons = 0  30. Hence.  (a) : Cu 2S + 2Cu    2O   ® 6Cu + SO 2  This is an  example  of auto­reduction.  (b) : The precipitate of mercuric iodide dissolves in excess of  where n = number of  unpaired electrons. 5 g mol –1 ) is passed through a cation exchanger. Pt = 78)  The  magnetic  moment  (spin only)  of  [NiCl 4]    (2007)  (a)  1.  The IUPAC name for the complex [Co(NO 2 )(NH 3 ) 5 ]Cl 2  is  (a)  nitrito­N­pentaamminecobalt(III) chloride  Which one  of the  following  has  an  optical isomer?  (b)  nitrito­N­pentaamminecobalt(II) chloride  (a)  [Zn(en) 2 ] 2+  (b)  [Zn(en)(NH 3 ) 2 ] 2+  (c)  pentaammine nitrito­N­cobalt(II) chloride  (c)  [Co(en) 3 ] 3+  (d)  [Co(H 2 O) 4 (en)] 3+  (2010)  (d)  pentaammine nitrito­N­cobalt(III) chloride.82 BM  4.  exhibit optical  isomerism?  (a)  [Co(en)(NH 3 ) 2 Cl 2 ] +  (b)  [Co(en) 3 ] 3+  (c)  [Co(en) 2 Cl 2 ] +  (d)  [Co(NH 3 ) 3 Cl 3 ]  (2013)  2. respectively  (a)  6 and 3  (b)  6 and 2  (c)  4 and 2  (d)  4 and 3  (2008)  (2011)  11.  (c)  2.  Which among the following will be named as dibromidobis  (ethylene diamine) chromium(III  bromide?  10.  (2011)  13.  In Fe(CO) 5 .5 g mol  ).: Fe = 26.84 BM.  Which of the following has a square planar geometry?  2–  2–  (a)  [Cr(en) 2 Br 2]Br  (b)  [Cr(en)Br 4 ] –  (a)  [PtCl 4]  (b)  [CoCl 4]        (c)  [Cr(en)Br 2 ]Br  (d)  [Cr(en) 3 ]Br 3  (2012)  (c)  [FeCl 4 ] 2–  (d)  [NiCl 4 ] 2– . the Fe – C bond possesses  (b)  The  complex is  paramagnetic.  5.70  JEE MAIN CHAPTERWISE EXPLORER  CHAPTER  18  COORDINATION COMPOUNDS 1.  Which of the following compounds shows optical isomerism?  Which of  the  following  has  an  optical isomer?  2+  2–  (a)  [Cu(NH 3)  (b)  [ZnCl 4]    4]      (a)  [Co(NH 3 ) 3 Cl] +  (b)  [Co(en)(NH 3 ) 2 ] 2+  (c)  [Cr(C 2 O 4 ) 3 ] 3–  (d)  [Co(CN) 6 ] 3–  (2005)  3+  3+  (c)  [Co(H 2O)  (d)  [Co(en) 2(NH  (2009)    4 (en)]    3)    2]    .78 g of AgCl (molar mass = 143.  6. Ni = 28.675  g  of  CoCl 3 ∙6NH 3  (molar  mass  = 267.  (2006)  (d)  The complex gives white precipitate with silver nitrate  solution.  7. The  14.  How many EDTA (ethylenediaminetetraacetic acid) molecules  Which of the following facts about the complex [Cr(NH 3 ) 6 ]Cl 3  are required to make an octahedral complex with a Ca 2+  ion?  is  wrong?  (a)  Six  (b) Three  (a)  The  complex  involves  d 2 sp 3  hybridisation  and  is  (2006)  (c)  One  (d) Two  octahedral in shape.  2–  is  (At.  Which of the  following  complex  species  is  not  expected to  9.  12.82 BM  (b)  5.  (d)  1.46 BM  3.  The value of the ‘spin only’ magnetic moment for  one of  the  following configurations is 2.  (b)  d 4  ( in weak ligand field)  The formula  of the complex is  (At.  8.  (2006)  A  solution  contains  2. mass  of Ag  =  108 u)  (c)  d 3  ( in weak as well as in strong fields)  (a)  [CoCl(NH 3 ) 5 ]Cl 2  (b)  [Co(NH 3 ) 6 ]Cl 3  (d)  d 5  (in strong ligand field)  (2005)  (c)  [CoCl 2 (NH 3)  (d)  [CoCl 3 (NH 3 ) 3 ]  (2010)    4 ]Cl  Which of the  following  pairs  represents  linkage  isomers?  15.  (c)  ionic character  (d) s­character  only.  (a) p­character only (b)  both s and p characters  (c)  The  complex  is  an  outer  orbital  complex.41 BM  The coordination number and the oxidation state of the element  E in the complex [E(en) 2  (C 2 O 4 )]NO 2  (where (en) is ethylene  diamine)  are.  Which one of the following cyano complexes would exhibit the  (a)  [Cu(NH 3 ) 4][PtCl  lowest value of paramagnetic behaviour?    4 ]  and  [Pt(NH 3 ) 4 ][CuCl 4 ]  3–  3–  (b)  [Pd(PPh 3 ) 2 (NCS) 2 ] and  [Pd(PPh 3 ) 2 (SCN) 2 ]  (a)  [Cr(CN) 6]  (b)  [Mn(CN) 6]      3–  3–  (c)  [Co(NH 3 ) 5 (NO 3 )]SO 4  and [Co(NH 3 ) 5 (SO 4 )]NO 3  (c)  [Fe(CN) 6 ]  (d)  [Co(CN) 6 ]  (2005)  (d)  [PtCl 2 (NH 3 ) 4 ]Br 2  and  [PtBr 2 (NH 3 ) 4 ]Cl 2  (2009)  16. The correct one  is  chloride ions obtained in solution were treated with excess of  –1  (a)  d 4  ( in strong ligand field)  AgNO 3  to give 4. Co = 27. nos.   11.71  Coordination Compounds  17.  (c)  (c)  (b)  (a)  6.  9. One mole of the same complex  reacts with two moles of AgNO 3  solution to yield two moles of  (2005)  AgCl (s).  Answer  Key  1. the oxidation state  complex?  of nickel is  4–  4–  (a)  [Fe(CN) 6]  (b)  [Mn(CN) 6]      (a)  –1  (b)  0  (c)  [Co(NH 3 ) 6 ] 3+  (d)  [Ni(NH 3 ) 6 ] 2+  (c)  +1  (d)  +2  (2003)  [Atomic nos. In this  context which  of  the  following  statements is  (a)  In acidic solutions hydration protects copper  ions.  21.  (b)  Haemoglobin is the red pigment of blood and contains iron.  (d)  Copper hydroxide is an amphoteric substance.  24.  14.  8.  19.  21.  (2002)  (a)  the number of ligands around a metal ion bonded by sigma  bonds  27.  13.  The oxidation state of  Cr in [Cr(NH 3 ) 4 Cl 2 ] +  is  (a)  +3  (b)  +2  (c)  +1  (d)  0  19.  (c)  In  alkaline  solutions  insoluble  Cu(OH) 2  is  precipitated  (c)  Cyanocobalamine is B 12  and contains cobalt.  15.  22.  incorrect?  (b)  In  acidic  solutions  protons  coordinate  with  ammonia  (a)  Chlorophylls  are  green  pigments  in  plants  and  contain  molecules forming NH 4+    ions and NH 3  molecules are not  calcium.  7.  (d)  (b)  (d)  (d)  (b)  2.  available.  (2004)  23. Co =  27. The structure of the complex is  (a)  [Co(NH 3 ) 5 Cl]Cl 2  Which one of the following has  largest number  of isomers?  (b)  [Co(NH 3 ) 3 Cl 2 ]∙2NH 3  +  2+  (a)  [Ru(NH 3)  (b)  [Co(NH 3)    4Cl    2]      5Cl]    (c)  [Co(NH 3 ) 4 Cl 2 ]Cl∙NH 3  2+  +  (c)  [Ir(PR 3)  (d)  [Co(en) 2Cl    2H(CO)]      2]    (d)  [Co(NH 3 ) 4 Cl]Cl 2 ∙NH 3  (2003)  (R =  alkyl group. en =  ethylenediamine)  (2004)  24.  27.  17.  (2002)  18.  (c)  (a)  (c)  (a)  5.  22. gives 3 moles  of ions on dissolution in water. Fe =  26.  (b)  (b)  (a)  (b)  .  12.  26.  The IUPAC name of the coordination compound K 3 [Fe(CN) 6 ]  is  (a)  potassium hexacyanoferrate  (II)  (b)  potassium hexacyanoferrate (III)  (c)  potassium hexacyanoiron (II)  (d)  tripotassium hexacyanoiron (II)  (2005)  (c)  the number of ligands around a metal ion bonded by sigma  and pi­bonds both  (d)  the number of only anionic ligands bonded to  the  metal  ion.  10.  25.  One mole of the complex compound Co(NH 3 ) 5 Cl 3 .  In the coordination compound.  Ammonia forms the complex ion [Cu(NH 3 ) 4 ] 2+  with copper ions  in  alkaline  solutions  but  not in  acidic  solutions.  CH 3  – Mg – Br is an organometallic compound due to  (b)  the number of ligands around a metal ion bonded by pi­  (a)  Mg – Br bond  (b)  C – Mg bond  bonds  (c)  C – Br bond  (d)  C – H bond.  23. Ni = 28]  (2004)  26.  (d)  Carboxypeptidase­A is an enzyme and contains zinc.  16.  (a)  (d)  (a)  (a)  (b)  3.  20.  What  is  the  Coordination compounds  have great  importance in biological  reason for  it?  systems.  The type of isomerism present in nitropentamine chromium (III)  chloride is  The coordination number of a central metal atom in a complex  (a)  optical  (b)  linkage  is determined by  (c)  ionization  (d)  polymerisation.  which is soluble in excess of any alkali. K 4 [Ni(CN) 4 ].  18.  (2004)  (2003)  Which  one  of  the  following  complexes  is  an  outer  orbital  25.:  Mn = 25.  20.  (c)  (a)  (d)  (d)  (b) 4.  in the complex.  5.  . This overlap is also called back donation of electrons  In[Pd(PPh3)2(NCS)2].  (b) : In a metal carbonyl.  en  en  Ni 2+ .  [Co(en)2(NH3)2] 3+  exhibits  optical isomerism.  The  hybridisation  scheme  is as shown in figure.  about  the  central  2­valent  platinum  ion  in  a  square  planar  configuration.03 moles  atom is formed when a vacant hybrid bond of the metal atom  of AgCl on treatment with AgNO 3.  (b) :  No. the nickel is in +2 oxidation state and the ion has  the  electronic  configuration  3d 8 .  2.  2.  Formation of p­bond is caused when a filled orbital of the metal  (b) : Linkage isomerism is exhibited by compounds containing  atom overlaps with a vacant antibonding p* orbital of C atom  ambidentate ligand.01  267. it is an octahedral complex.  (c) : EDTA.  [M(AA) 3 ] n± . the metal carbon bond possesses both  the s­ and p­character.  Optical isomerism is common in octahedral complexes of the  general  formula.  by metal atom to carbon. where (AA) is a symmetrical  bidentate ligand. the formula of the  containing  a  lone pair of electrons. It is an inner  orbital complex.  Thus  among  the  given  options.  [M(AA) 2 a 2 ] n± .  (c)  :  In  the  paramagnetic  and  tetrahedral  complex  2–  [NiCl 4]    .  (c) : The complex [Cr(NH 3 ) 6 ]Cl 3  involves d 2 sp 3  hybridization  9. the linkage of NCS and Pd is through N.  complex is  [Co(NH 3 ) 6 ]Cl 3 .  Ca 2+  + [H 2 EDTA] 2– ® [Ca(EDTA)] 2–  + 2H +  12. Thus. it implies that 3 chloride    overlaps  with  an  orbital  of  C  atom  of  carbon  monoxide  ions are ionisable. In [Pd(PPh3)2(SCN)2].78  =  0.e.03  143. [Ar]3d 8  :  NH3  3+  Co  [NiCl 4 ] 2–  :  3+  Co  NH3  m= H3N  H3N  en  en  n( n +  2) BM = (2(2 + 2) =  8 Non­superimposable  mirror images  = 2. of moles  of AgCl  = 7.  of CO.  Therefore  co­ordination  number  of  the  complex  is 6 i.  (c) : Optical isomers rarely occur in square planar complexes  due  to the  presence  of  axis  of  symmetry.5 11.  (d)  :  Optical  isomerism  is  usually  exhibited  by  octahedral  compounds of the type [M(AA)2B2].  (a) : In 4­coordinate complexes Pt. A s­bond between metal  and  carbon  Since 0.  Oxidation number of  E in the  given complex is  x  + 2 × 0 + 1 × (–2) = +1 \  x = 3  4. the linkage of SCN and Pd is through S.82 BM  (a)  :  In  the  given  complex  [E(en) 2 (C 2 O 4 )] + NO 2 –  ethylene  diamine is a bidentate ligand and  (C 2 O 4 2– ) oxalate ion is also  bidentate  ligand.  of moles  of  CoCl 3 ∙6NH 3  = No.  The  free  acid  H 4 EDTA  is  insoluble  and  the  disodium  salt  Na 2 H 2 EDTA is the most used reagent.01 moles of the complex CoCl 3∙6NH    3 gives 0.  6.    presence  of  plane  of  symmetry.  [Ma 2 b 2 c 2 ] n± .675  =  0. Square planar complexes rarely show optical  isomerism on accout  of  presence  of axis  of  symmetry.5 4. only  [Co(en) 3 ] 3+  shows  optical  isomerism. among the  10. Thus.  as it involves (n – 1)d orbitals for hybridization..  [Mabcdef]  n± .72  JEE MAIN CHAPTERWISE EXPLORER  1. [M(AA) 2 ab] n±  and [M(AB) 3 ] n± .  (d) : [Co(NH 3 ) 3 Cl 3] will not exhibit optical isomerism due to  8.  (a)  3. which has four donor oxygen atoms and two donor  nitrogen atoms in each molecule forms complex with Ca 2+  ion. the four ligands are arranged  given options. 73  Coordination Compounds  –  M  +  +  +  C  C  M  O  O  ox 3–  (a)  +  –  .  . C  –  +  –  +  M  ox  .  O .  –  +  Cr  Cr  ox  (b)  3–  ox  ox  ox  –  –  M  C  .  O .  +  +  (a) The  formation  of  the  metal ¬  carbon s­bond  using  an  unshared  17.  (b) : K 3 [Fe(CN) 6 ]  Potassium hexacyanoferrate(III)  18.  (a) : Let the  oxidation state of Cr in  [Cr(NH 3 ) 4 Cl 2 ] +  =  x  The p­overlap is perpendicular to the nodal plane of s­bond.  x + 4(0) + 2(–1) = +1  13.  (d) : [Co(NO 2 )(NH 3 ) 5 ]Cl 2  x – 2 = +1   or,  x = +1 + 2 = +3  pentaaminenitrito­N­cobalt(III)  chloride  19.  (d)  :  [Co(en) 2 Cl 2 ] +  shows  geometrical  as  well  as  optical  14.  (a) : Spin only magnetic moment  = n(n + 2)  B.M.  isomerism.  Where n =  no. of unpaired  electron.  20.  (a)  :  Chlorophyll  are  green  pigments  in  plants  and  contains  Given,  n(n + 2) = 2.84  magnesium instead of calcium.  or,  n (n + 2) = 8.0656  pair  of  the  C  atom.  (b)  The  formation  of  the  metal ®  carbon p­bond.  or,  n = 2  21.  (d) :  Complex  ion  Hybridization  of central  ion  In an octahedral complex, for a d 4  configuration in a strong field  [Fe(CN) 6 ] 4–  d  2 sp 3  (inner)  4–  ligand, number of unpaired electrons = 2  [Mn(CN) 6 ]  d  2 sp 3  (inner)  3+  [Co(NH 3 ) 6 ]  d  2 sp 3  (inner)  15.  (d) : [Co(CN) 6 ] 3–  2+  [Ni(NH 3)  sp 3 d 2  (outer)  Co ® [Ar] 3d 7  4s 2    6]    3+ 6  0  Co  ® [Ar] 3d  4s  22.  (a) : The number of atoms of the ligands that are directly bound  3d  4s  4p  In presence of strong field ligand CN – pairing of electrons takes  place.  ××  ××  ××  ××  ××  ××  14444444444442 444444444444434  d 2 sp 3  to the central metal atom or ion by coordinate bonds is known  as the coordination number of the metal atom or ion.  Coordination number of metal = number of s bonds formed by  metal with ligands.  23.  (a) : Given reactions can be explained as follows:  [Co(NH 3 ) 5 Cl] 2+  + 2Cl – Þ 3 ions.  [Co(NH 3 ) 5 Cl]Cl 2  [Co(NH 3 ) 5 Cl]Cl 2  + 2AgNO 3 ® [Co(NH 3 ) 5 Cl](NO 3 ) 2  + 2AgCl  There  is  no  unpaired  electron,  so  the  lowest  value  of  24.  (b) : In acidic solution, NH  forms a bond with H +  to give NH +  3  4  paramagnetic behaviour is observed.  ion which does not have a lone pair on N atom. Hence it cannot  act as a ligand.  16.  (c) : Optical isomers rarely occur in square planar complexes  on  account  of  the  presence  of  axis  of  symmetry.  Optical  25.  (b) : Let the oxidation number of Ni in K  [Ni(CN)  ] = x  4  4  isomerism  is  very  common  in  octahedral  complexes  having  1 × 4 + x × (–1) × 4 = 0 Þ  4 + x – 4 = 0 Þ  x = 0  general formulae:  [Ma 2 b 2 c 2 ] n± ,  [Mabc def] n± ,  [M(AA) 3 ] n± ,  [M(AA) 2 a 2 ] n± ,  26.  (b)  :  The  nitro  group  can  attach  to  metal  through  nitrogen  n±  n±  as (–NO 2 ) or through oxygen as nitrito (–ONO).  [M(AA) 2ab]  and  [M(AB) 3]      (where  AA  =  symmetrical  bidentate  ligand  and  AB  =  27.  (b) : Compounds that contain at least one carbon­metal bond  unsymmetrical bidentate ligand).  are called organometallic compounds. 74  JEE MAIN CHAPTERWISE EXPLORER  CHAPTER  19  ENVIRONMENTAL CHEMISTRY 1.  The  gas  leaked  from  a  storage  tank  of  the  Union  Carbide  plant in Bhopal gas tragedy was  (a)  phosgene  (b)  methylisocyanate  (c)  methylamine  (d)  ammonia  (2013)  2.  Identify the  wrong statement  in  the  following.  (a)  Acid rain is mostly  because  of  oxides  of  nitrogen and  sulphur.  (b)  Chlorofluorocarbons  are  responsible  for  ozone  layer  depletion.  (c)  Greenhouse  effect is responsible  for  global  warming.  (d)  Ozone layer does not permit infrared radiation from the  sun to reach the earth.  (2008)  3.  Answer  Key  1.  (b)  2.  (d)  3.  (c)  The smog is essentially caused by the presence of  (a)  O 2  and O 3  (b)  O 2  and N 2  (c)  oxides of sulphur and  nitrogen  (d)  O 3  and N 2 .  (2004)  75  Environmental  Chemistry  1.  (b)  violet  rays from sun  to reach the earth.  2.  (d) : The thick layer of ozone called ozoneplanket which is  3.  effective in absorbing harmful ultraviolet  rays given out by  the sun acts as a protective shield. It does not permit the ultra  (c) : Photochemical smog is caused by oxides of sulphur and  nitrogen. 76  JEE MAIN CHAPTERWISE EXPLORER  PURIFICATION AND  CHARACTERISATION OF  ORGANIC COMPOUNDS CHAPTER  20  1.  2.  29.5  mg  of an  organic  compound  containing  nitrogen was  digested  according  to  Kjeldahl’s  method  and  the  evolved  ammonia was absorbed in 20 mL of 0.1 M HCl solution. The  excess of the acid required 15 mL of 0.1 M NaOH solution  for  complete  neutralization.  The  percentage  of  nitrogen  in  the  compound is  3.  (a)  29.5  (b)  59.0  (c)  47.4  (d)  23.7  (2010)  The  ammonia  evolved  from  the  treatment  of  0.30 g of an organic compound for the estimation of nitrogen  was passed in 100 mL of 0.1 M sulphuric acid. The excess of  Answer  Key  1.  (d)  2.  (c)  3.  (c)  acid required 20 mL of 0.5 M sodium hydroxide solution  for  complete neutralization. The organic compound is  (a)  acetamide  (b)  benzamide  (c)  urea  (d)  thiourea.  (2004)  In  a  compound  C,  H  and  N  atoms  are  present  in  9 : 1 : 3.5 by weight. Molecular weight of compound is 108.  Molecular formula of compound is  (a)  C 2H  (b)  C 3H    6N    2    4N    (c)  C 6 H 8 N 2  (d)  C 9 H 12 N 3 .  (2002)  77  Purification and Characterisation of Organic Compounds  1.  (d) : The % of N according to Kjeldahl’s method  =  w N 1  = Normality of the standard acid =  0.1 N  w = Mass of the  organic  compound  taken  = 29.5 mg = 29.5 × 10 –3  g  V = Volume of  N 1  acid neutralised  by ammonia  = (20 – 15) = 5 mL.  Þ 2.  %N = 1.4 ´ 0.1 ´ 5  29.5 ´ 10 -3  Percentage of nitrogen in urea (NH 2 ) 2 CO  14 ´ 2  =  60 ´100 = 46.6  \  The compound must be urea.  1.4 ´ N1  ´ V = 23.7  (c) : Equivalents of  NH 3  evolved  100 ´ 0.1´ 2 - 20 ´ 0.5 =  1  =  1000 1000 100 Percent  of  nitrogen  in  the  unknown  organic  compound  =  1 ´ 14  ´ 100 = 46.6  100 0.3 3.  (c) :  C         H         N  9    :    1    :    3.5  9  1  3.5  :  :  12  1  14  3  1  1  :  :  4  1  4  3     :     4    :     1  Empirical formula = C 3 H 4 N  (C 3 H 4 N) n  = 108  (12 ´ 3 + 1 ´ 4 + 14) n  = 108  54n = 108 Þ n = 108/54 = 2  Molecular formula = C 6 H 8 N 2 78  JEE MAIN CHAPTERWISE EXPLORER  CHAPTER  21  1.  SOME BASIC PRINCIPLES  OF ORGANIC CHEMISTRY The  order of  stability  of  the  following  carbocations is  CH 2  CH 2  II  III  (a)  III > I > II  (c)  II > III > I  2.  3.  4.  5.  (b)  III > II > I  (d)  I > II > III  The  correct  decreasing  order  of  priority  for  the  functional  groups  of  organic  compounds  in  the  IUPAC  system  of  nomenclature  is  (a)  – CONH 2 , – CHO, – SO 3 H, – COOH  (b)  – COOH, – SO 3 H, – CONH 2 , – CHO  (c)  – SO 3 H, –COOH, –CONH 2 , – CHO  CH—CH 2 ; CH 3 —CH 2 —CH 2 ;  I  9.  (2013)  How many chiral compounds are possible on monochlorination  10.  The electrophile, EÅ  attacks the benzene ring to generate the  of  2­methyl  butane?  intermediate s­complex. Of the following, which s­complex  (a)  2  (b)  4  is of lowest  energy?  (c)  6  (d)  8  (2012)  NO 2  NO 2  Identify the  compound  that exhibits  tautomerism.  +  (a)  (b)  (a)  2­Butene  (b)  Lactic  acid  +  H  (c)  2­Pentanone  (d)  Phenol  (2011)  E  H  E  Out of the following, the alkene that exhibits optical isomerism  is  NO 2  H  (a)  2­methyl­2­pentene  (b)  3­methyl­2­pentene  H  (c)  4­methyl­1­pentene  (d)  3­methyl­1­pentene  (2010)  (c)  (d)  +  E  +  E  The IUPAC name of neo­pentane  is  (a)  2­methylbutane  (b)  2,2­dimethylpropane  (c)  2­methylpropane  (d)  2,2­dimethylbutane  (2008)  11.  The  absolute  configuration  of  HO2 C  (2009)  6.  7.  8.  (2008)  (d)  – CHO, –COOH, – SO 3 H,  –CONH 2  The number of stereoisomers possible  for a  compound  of  the  molecular formula CH 3  – CH  CH  – CH(OH) –  Me  is  (a)  3  (b)  2  (c)  4  (d)  6  (2009)  CO2 H  is  HO  H  H  (a)  S,  R  (c)  R,  R  (b)  S,  S  (d)  R,  S  OH  (2008)  The alkene that exhibits  geometrical isomerism is  12.  Which one of the following  conformations of cyclohexane is  (a)  propene  (b)  2­methylpropene  chiral?  (c)  2­butene  (d)  2­methyl­2­butene  (2009)  (a)  Boat  (b)  Twist boat  Arrange the  carbanions,  (c)  Rigid  (d)  Chair  (2007)  (CH3 )3 C , CCl3 , (CH 3 ) 2CH , C6 H 5CH 2  13.  Increasing order of stability among the three main conformations  in order of  their decreasing  stability  (i.e. eclipse, anti, gauche) of 2­fluoroethanol is  (a)  eclipse, gauche, anti  (a)  C6 H5 CH 2 > CCl3 > (CH3 )3C > (CH 3 ) 2 CH (b)  gauche, eclipse, anti  (c)  eclipse, anti, gauche  (b)  (CH3 ) 2 CH > CCl3 > C6 H5 CH 2 >  (CH3 )3 C (d)  anti, gauche, eclipse.  (2006)  (c)  CCl3 > C6 H 5 CH 2 > (CH 3 ) 2CH > (CH 3 )3 C 14.  The increasing order of stability of the following free radicals  (d)  (CH3 )3C > (CH 3 )2 CH > C6 H 5 CH 2 > CCl3  (2009)  is  79  Some Basic Principles of Organic Chemistry  .  .  .  .  (a)  (CH 3 ) 2 CH < (CH 3 ) 3 C < (C 6 H 5 ) 2 CH < (C  CH < (C 6 H 5 ) 3 C  .  .  .  .  (b)  (C 6 H 5 ) 3 C < (C 6 H 5 ) 2 CH  < (CH 3 ) 3 C < (CH 3 ) 2 CH  .  .  .  .  (c)  (C 6 H 5 ) 2 CH < (C 6 H 5 ) 3 C <  (CH 3 ) 3 C < (CH  C < (CH 3 ) 2 CH  .  .  .  .  C < (C 6 H 5 ) 2 CH  (d)  (CH 3 ) 2 CH < (CH 3 ) 3 C < (C 6 H 5 ) 3 C < (C  H  CH 3  H – C Å  C 2 H 5  – CH – C 2 H 5  H  (III)  (2006)  (IV)  it is true that  (a)  all four are chiral compounds  (b)  only I and II are chiral compounds  (c)  only III is a chiral compound  (d)  only II and IV are chiral compounds.  15.  CH 3 Br + Nu  ® CH 3  – Nu + Br  The  decreasing  order  of  the  rate  of  the  above  reaction  with  nucleophiles (Nu – ) A to D is  [Nu –  = (A) PhO – , (B) AcO – ,  (C) HO – , (D) CH 3 O – ]  (a)  D > C > A > B  (b)  D > C > B > A  (c)  A > B > C > D  (d)  B > D > C > A.  (2006)  20.  The reaction :  – –  16.  The IUPAC name of the  compound shown below is  Cl  (a)  (b)  (c)  (d)  Br  (a)  (b)  (c)  (d)  (CH 3 ) 3 C – Br  2­bromo­6­chlorocyclohex­1­ene  6­bromo­2­chlorocyclohexene  3­bromo­1­chlorocyclohexene  1­bromo­3­chlorocyclohexene.  H 2 O  (2003)  (CH 3 ) 3 C – OH  elimination  reaction  substitution reaction  free radical  reaction  displacement reaction.  (2002)  21.  Which of the following does not show geometrical isomerism?  (a)  1,2­dichloro­1­pentene  (2006)  (b)  1,3­dichloro­2­pentene  17.  The decreasing order of nucleophilicity among the nucleophiles  (c)  1,1­dichloro­1­pentene  is  (d)  1,4­dichloro­2­pentene.  (2002)  (1)  CH 3 C – O –  (2)  CH 3 O –  22.  A similarity between optical and geometrical isomerism is that  O  (a)  each forms equal number of isomers for a given compound  O  (b)  if in a compound one is present then so is the other  –  H  C  S – O  –  (c)  both are included in stereoisomerism  (3)  CN  (4)  3  (d)  they have no similarity.  (2002)  O  (a)  1, 2, 3, 4  (b)  4, 3, 2, 1  23.  Racemic mixture is formed by mixing two  (c)  2, 3, 1, 4  (d)  3, 2, 1, 4  (a)  isomeric  compounds  (2005)  (b)  chiral  compounds  18.  Due to the presence  of an unpaired  electron, free  radicals are  (c)  meso compounds  (a)  chemically  reactive  (d)  optical isomers.  (2002)  (b)  chemically  inactive  (c)  anions  24.  Arrangement of (CH 3 ) 3 C–, (CH 3 ) 2 CH–, CH 3 CH 2  – when attached  (d)  cations  (2005)  to benzyl or an unsaturated group in increasing order of inductive  effect  is  19.  Among the following four  structures I to IV,  (a)  (CH 3 ) 3 C – < (CH 3 ) 2 CH – < CH 3 CH 2  –  CH 3  O  CH 3  (b)  CH 3 CH 2  – < (CH 3 ) 2 CH – < (CH 3 ) 3 C –  C 2 H 5  – CH – C 3 H 7  CH 3  – C – CH – C 2 H 5  (c)  (CH 3)    2CH – < (CH    3)    3C – < CH    3CH    2  –  (I)  (II)  (d)  (CH 3)  C – < CH  CH  – < (CH  )  CH –.  (2002)    3  3  2  3  2  Answer  Key  1.  (a)  7.  (c)  13.  (a)  2.  (a)  8.  (c)  14.  (a)  3.  9.  15.  (c)  (c)  (a)  4.  (d)  10.  (c)  16.  (c)  5.  11.  17.  (b)  (c)  (d)  6.  (c)  12.  (b)  18.  (a)  19.  (b)  20.  (b)  21.  (c)  22.  (c)  23.  (d)  24.  (b)   (d) : 3­Methyl­1­pentene exhibits optical isomerism as it has  an asymmetric C­atom  in the  molecule. only 2­butene exhibits  cis­trans  isomerism.  (c) :  5.  (c) : The given compound has a C  (*)  carbon.  (b) :  neo­pentane  or  2. Also. exhibit geometrical isomerism.  Out of four possible  isomers only  I  and  III  are  chiral. greater is its stability.  (a) :  *  H3 C – HC  CH – C H(OH) – Me  H  C  H3 C  C group and one chiral  H  H C  C  CH(OH)Me  Me(OH)CH  C  H  CH3  d.  2­pentanone  exhibits  tautomerism.  7.2­dimethylpropane  +  E  This  structure  will  be  of  lowest  energy  due  to  resonance  stabilisation of +ve charge.  2.  (a) : Greater the number of resonating structures a carbocation  6.  possess. the  presence of electron­withdrawing NO 2  group will destabilize  the  +ve  charge and hence  they will have greater  energy.  dynamic equilibrium is known as tautomerism. 2° carbanions are more stable.  3. . In all other three structures.  (c) : The groups having +I effect decrease the stability while  groups having –I effect increase the stability of carbanions.  Compounds  in  which the  two  groups  attached  to  a  double  bonded carbon are different. out of  2° and 3° carbanions.  thus. thus the  decreasing order of stability is :  CCl3 > C6 H5 CH 2 > (CH3 ) 2 CH > (CH 3 )3 C.  4.80  JEE MAIN CHAPTERWISE EXPLORER  1.  (c) : When two groups attached to a double bonded carbon  atom are  same.  9. the compound does not exhibit geometrical  isomerism.  (c) : The  type  of isomerism in  which  a  substance  exists in  two readily interconvertible different structures leading to a  8.  (c) : The order of preference of functional groups is as follows:  –SO 3 H > –COOH > – COOR > – COX > –COCl > –CONH 2  > –CHO > –CN >  C  O  > –OH > –SH > –NH 2  >  C  C  > –C  C– > –NO 2  >  –NO > –X  H  10.  Benzyl carbanion is stabilized due to resonance. l  isomers of cis ­form \  Total stereoisomers = 4.   more  basic  the  species  increases the inductive  effect increases.  . but different spatial arrangement of atoms  phenyl  group  attached  to  the  carbon  atom  holding  the  odd  or groups.  (d) : Strong bases are generally good nucleophile.  2  (1)  16. Hence basicity as well as nucleophilicity order is  2  O  5  1  CH 3 O –  > CH 3  4  6  C  O –  > H 3 C  S  O  O  3  O –  –  Now CN –  is a better nucleophile than CH 3O    .  (c) : Both involves compounds having the same molecular and  structural formulae. dextro and laevorotatory  accordingly.  If the nucleophilic atom or the centre is the same. Further as the number of  22.e.  (c) :  C     C – CH 2  – CH 2  – CH 3  that the steric strain is maximum.  Benzyl free radicals are stabilised by resonance and hence are  more stable than alkyl free radicals.  H  19.e.  electron  increases.e.  15.  the  order of stability of  free radical is  as  follows:  Identical  groups (Cl)  on C  ­ l  will give  only one  compound.  .  In fully eclipsed conformation F and OH groups are so close  21.  Only I and II are chiral compounds.  as  number  of  – CH 3  group  nucleophilicity parallels  basicity.  3  2  3  3  6  5  2  6  5  3  mixture.e. stronger is  the  active as it does not have  any  plane of symmetry.  (b)  :  –CH 3  group  has  +I  effect. R )  17.  (a)  :  If  the  nucleophilic  atom  or  the  centre  is  same.  optical isomers is termed as racemic mixture or dl form or (±)  (CH  )  CH < (CH  )  C < (C  H  )  CH < (C  H  )  C  i. This  order  can  be  easily  explained  on  the  general  concept that a weaker acid has a  stronger conjugate base.  (b) : A chiral object or compound can be defined as the one that  is not superimposable on its mirror image..  .  13.  the  stability  of  a  free  radical  increases  23.  (a)  :  Free  radicals are  highly reactive  due to  presence  of  an  unpaired electron. They readily try to pair­up the odd electrons.  nucleophile.  i. or we can say that  all the four groups attached to a carbon atom must be different.  Hence it does not show geometrical isomerism.  (b) : This is an example of nucleophilic substitution reaction.  (d) : An equimolar mixture of two i.  H  skew or gauche  CH 3  CH 3  *  H  *  H  (II)  CH  CO – C – C  (I)  C 2 H 5  – C – C  3  2  5  3  7  H  H  20.  tertiary > secondary > primary.  (a) :  Hence decreasing order  of nucleophilicity is  CH 2  – CH 2  F  CN –  > CH 3 O –  > CH 3  OH  C  O –  >  H 3 C  O  S  O –  2­fluoroethanol  F  H  F  H  H  H  H  HO  HO  H  OH  H  H  H  eclipsed  anti or staggered  F  OH  H  H  H  H  O  O  F  H  18.  (c) :  COOH  COOH  HO2 C  CO2 H  HO  HO  H H  H  OH  C  C  2  HO  H  H  1  OH  Cl  3  OH  H  1  3  COOH  COOH  –  > HO –  > PhO –  > AcO –  CH 3O    Here.  –  Br  +  OH –  (CH 3 ) 3 C  OH  +  Br  (CH 3 ) 3 C  The anti conformation is most stable in which F and OH groups  Nucleophile  Leaving group  Substrate  are far apart as possible and minimum repulsion between two  Cl  H  groups  occurs. more basic the species.  O  is  the  same  in  all  these  species.  fully eclipsed  . stronger is the  nucleophile.  (c) :  (6) (2)  (5)  (3)  (4)  3­bromo­1­chlorocyclohexene  Br  (R . nucleophilicity  12.81  Some Basic Principles of Organic Chemistry  11.  . i.  24.e.  the  nucleophilic  atom  i. hence this conformation is  Cl  most unstable.  (a) : On the basis of hyperconjugation effect of the alkyl groups.  bond. The order of stability of these conformations is  Condition for geometrical isomerism is presence of two different  anti > gauche > partially eclipsed > fully eclipsed  atoms of groups attached to each carbon atom containing double  14.  (b) : The twist boat conformation  of cyclohexane is optically  parallels basicity.   (2007)  One mole  of a symmetrical alkene on ozonolysis  gives two  moles of an aldehyde having a molecular mass of 44 u.  A gaseous hydrocarbon gives upon combustion 0.  Which  branched  chain  isomer  of  the  hydrocarbon  with  molecular mass 72 u gives only one isomer of mono substituted  alkyl halide?  (a)  Neopentane  (b)  Isohexane  9.  5.  Which  of  the  following  molecules  is  expected  to  rotate  the  (b)  CH 3 CH 2CH  C  CCH 2CH  plane­polarised light?    2    2 CH 3  (c)  CH 3 CH 2C  CH    CHO  COOH  CHCH 3  (d)  CH 3 CH  (2008)  H  (b)  HO  H  (a)  H 2 N  In  the  following  sequence  of  reactions. The empirical formula of the hydrocarbon    is  (a)  C 7 H 8  (b)  C 2 H 4  (c)  C 3 H 4  (d)  C 6 H 5  (2013)  2.2­dibromopropane?  H  6.  (2007)  (a)  CH 4  (b)  CH 3  – CH  CH 2  11.  the  alkene  affords  CH 2 OH  H  the compound  B  O  H O  B.  (2012)  Toluene is nitrated and the resulting product is reduced with  tin and hydrochloric acid.  Presence of a  nitro group in a  benzene ring  (a)  deactivates the ring towards electrophilic substitution  (b)  activates the ring towards electrophilic substitution  (c)  renders the ring basic  (d)  deactivates the ring towards nucleophilic substitution.82  JEE MAIN CHAPTERWISE EXPLORER  CHAPTER  HYDROCARBONS 22  1.72 g of water  8.  (c)  Neohexane  (d)  Tertiary  butyl  chloride  (2012)  3.  and 3.  4. The reaction mixture  so formed  contains  (a)  mixture  of  o­ and  m­bromotoluenes  (b)  mixture  of  o­ and  p­bromotoluenes  (c)  mixture  of  o­ and  p­dibromobenzenes  (d)  mixture  of  o­ and  p­bromoanilines.  (2010)  (c)  benzyl  chloride  C  – H produces  The treatment of CH 3MgX with CH    3C    (d)  o­ and  p­chlorotoluene.08 g of CO 2.  The  reaction  of  toluene  with  Cl 2  in  presence  of  FeCl 3  gives  predominantly  alkene is  (a)  m­chlorobenzene  (a)  ethene  (b)  propene  (b)  benzoyl  chloride  (c)  1­butene  (d)  2­butene.  (2008)  (a)  CH 3  – CH  H  (2008)  C  –  CH 3  (b)  CH 3  – C  CH 2  +  HBr ®  CH  +  2HBr ®  (c)  CH 3 CH  CHBr  + HBr ®  The  hydrocarbon  which  can  react  with  sodium  in  liquid  (d)  CH  CH  +  2HBr ®  (2007)  ammonia is  (a)  CH 3 CH 2 C  CCH 2 CH 3  12. The  10.  Which  of  the  following  reactions  will  yield  (c)  CH 3 C  C –  CH 3  2. The product so obtained is diazotised  and then heated with cuprous bromide.  2­Hexyne  gives trans­2­hexene  on  treatment  with  (a)  Li/NH 3  (b)  Pd/BaSO 4  (c)  LiAlH 4  (d)  Pt/H 2  (d)  CH3   –  C  7.  CH3 CH  CHCH 3  3  A  2  Zn  The compound  B  is  (a)  CH 3CHO  (b)  CH 3CH      2CHO    (c)  CH 3 COCH 3  (d)  CH 3 CH 2COCH    3  (c)  (2008)  (d)  SH  H 2 N  NH 2  H  Ph  H  Ph  (2007)  .   (2003)  monochlorinated compounds  is  (a)  n­hexane  (b)  2.2­dimethylpentane  4.4­dimethyl­5.  The IUPAC name of  (a)  (b)  (c)  (d)  19.  (b)  18.  (2007)  14.3­dimethylbutane  24.  Which of these will not react with acetylene?  conditions  (a)  NaOH  (b)  ammonical AgNO 3  (d)  1­bromo­2­butene under kinetically controlled conditions.  (2002)  conditions  (c)  3­bromobutene  under  thermodynamically  controlled  26.  (d)  16.  (b)  17.  Which one of the following has the minimum boiling point?  (a)  n­Butane  (b)  1­Butyne  formation  of  (c)  1­Butene  (d)  Isobutene  (2004)  (a)  primary alcohol  (b)  secondary or tertiary  alcohol  23.  25.  19.  13.  14.  (b)  (d)  (d)  (a) 9.  26.5­diethyl­4.  (a)  15.83  Hydrocarbons  13.  This  (d)  mixture of secondary  and tertiary  alcohols.  On mixing a certain alkane with chlorine and irradiating it with  (c)  mixture of primary and  secondary alcohols  ultraviolet  light.5­diethylpentane  5. c)  7.  (d)  10.  What is the product when acetylene reacts with hypochlorous  (a)  3­bromobutene  under  kinetically  controlled  conditions  acid?  (b)  1­bromo­2­butene  under  thermodynamically  controlled  (a)  CH 3 COCl  (b)  ClCH 2 CHO  (c)  Cl 2 CHCHO  (d)  ClCHCOOH.  (a)  2.4­dimethylpentane.  Acid catalyzed hydration of alkenes except ethene leads to the  22.  (c)  12.  (d)  21.  it  forms  only  one  monochloroalkane.  (b)  24.  (2007)  15.  Which  one  of  the  following  is  reduced  with  zinc  and  hydrochloric acid  to give the  corresponding hydrocarbon?  (a)  Ethyl acetate  (b)  Acetic acid  (c)  Acetamide  (d)  Butan­2­one  (2004)  21.  (a)  (a)  (b)  (c)  8.  20.  The compound formed as a result of oxidation of ethyl benzene  by KMnO 4  is  (a)  benzyl  alcohol  (b)  benzophenone  (c)  acetophenone  (d)  benzoic  acid.  Reaction of  one molecule  of  HBr  with one  molecule of  1.  (2005)  alkane could be  (a)  propane  (b)  pentane  17.  (b)  22.  Amongst the following compounds.  (a)  11.  (a.  the  isomer which  can give two  (c)  isopentane  (d)  neopentane.  (2005)  is  3­ethyl­4­4­dimethylheptane  1.  Butene­1 may be converted to butane by reaction with  (c)  2. 3­  butadiene at 40°C gives predominantly  25.  Me  Å Me N  Et  OH  n­Bu  D  20. the optically active alkane  having lowest molecular mass is  (a)  CH 3  – CH 2  – CH 2  – CH 3  CH 3  (b)  CH 3  – CH 2  – CH – CH 3  The alkene formed as a major product in the above elimination  reaction is  (a)  (b)  CH 2  Me  Me  (c)  CH 3  – C –  CH 2  C 2 H 5  Me  (d)  CH 3  – CH 2  – C  (2006)  (d)  (c)  H  CH  (2004)  16.  2­Methylbutane  on  reacting  with  bromine  in  the  presence  of  sunlight gives mainly  (a)  1­bromo­2­methylbutane  (b)  2­bromo­2­methylbutane  (c)  2­bromo­3­methylbutane  (d)  1­bromo­3­methylbutane.  (a)  3.  (d)  .  Of the five isomeric hexanes.  (c)  4.  (2003)  18.1­diethyl­2.  (d)  5.  (c)  Na  (d)  HCl  (2002)  (2005)  Answer  Key  1.  (2005)  (a)  Zn ­ HCl  (b)  Sn ­ HCl  (c)  Zn ­ Hg  (d)  Pd/H 2 .  (d)  6.2­dimethylbutane  (d)  2­methylpentane.  (b)  23. 08  CH 3  O  O  Mono­ozonide  x  0. mass of R = 44 –  29 =  15  CH 3  CH 3  CH 3 This is possible.  (a) :  The  complete  reaction sequence is as follows  H  H  y = 0.  JEE MAIN CHAPTERWISE EXPLORER  (a) :  Moles of water produced =  Moles of CO 2  produced =  0.07 7  = = y 0.04  18  5. FeCl 3  Cl  \  The aldehyde is CH 3 CHO and the  symmetrical alkene is  + Cl 2  +  CH 3 HCCHCH 3 . CuBr/D  NH 3  or EtNH 2  at low temperature or LiAlH 4 as reducing agent  (anti­addition).  sodium alkylides.  CH 3  HNO3/H 2SO 4  CH 3  Sn/HCl  Nitration  Toluene  NO 2  NH 2  p ­nitrotoluene  CH 3  3. only  when R is —CH 3  group.07  44  Equation for combustion of an unknown hydrocarbon.  is  CMgX  (c) : Terminal alkynes react with sodium in liquid ammonia  to yield ionic compounds  i.07  and  2  H  O 3  C  C  CH 3  H3 C  H  O  C  C  H3 C  \  y = 0.  3.  (a) :  As the molecular  mass indicates  it should be  pentane  8.  Þ R + 12 + 1 + 16 =  44  10.  2.  æ yö y  C x H y  + ç x + ÷ O 2 ® xCO 2 + H 2 O  è 4ø 2  7.  (b) : The  reaction sequence  is as follows :  and  neopentane  can  only  form  one  mono  substituted  alkyl  CH 3  halide as all the hydrogens are equivalent in  neopentane.08  = 0.  (d) :  Mol. NaNO2 /HCl  2.  Br  +  O –  O  N  N  –  O  O  N  –  O  O  O  N  +  From  the  resonating  structures  +  of  it  can  be  seen  that  the  Molecular mass of RCHO =  44 nitrogroup  withdrawn  electrons  from  the  rings  and  hence  it  deactivates the benzene ring for further electophilic substitution.04  Þ  x = 0. C x H y  (a) : Grignard reagent reacts with compounds having active  C–H ® CH 4  + CH 3 C  CH 3 MgX + CH 3 C  6.  O  N    O  (d) :  o ­bromotoluene  (a) :  O  4.  or acidic hydrogen  atom  to give  alkane.72  = 0.84  1.  CH 3  Br  p ­bromotoluene  9.  (a.e.  o­chlorotoluene  Cl  p­chlorotoluene  .08 8  Zn–H2 O  Cleavage  2CH3 CHO + ZnO  \  The  empirical  formula  of  the  hydrocarbon  is  C 7 H 8 . c) : For trans products we should take Na or Li metal in  1.  it is the less sterically hindered b­hydrogen that is removed and hence less substituted alkene  is the major product.  (a) :  CH 3  – CH 2  – CH 2  – C – C – CH 2  – CH 3  H 3 C – C  –  C – CH 3  CH 3 CH 2 – CH 3  3­ethyl­4. 2­dibromopropane  7  C  CH 3  OH  (secondary alcohol)  HBr  CH 3  CH 3  CH 3  (3º)  –  HBr  CH  1  CH 3  CH 3  13. regardless of  length is oxidised to –COOH. the    1.(CH 3)    2CHCH    2CH    3  127°C  2­methylbutane  (CH 3)    2C(Br)CH    2CH    3  2­bromo­2­methylbutane . 2­dibromopropane  CH 3 CH    CHBr  OH –  CH 3  CH 3  C  OH  CH  CH 3  CH 3  Br  CH 3  CH 3  CH 3  (tertiary alcohol)  17.  (d) :  H 3 C – C  –  C – CH 3  Ethyl benzene  Benzoic acid  Cl  H  When oxidises with alkaline KMnO 4  or acidic Na 2Cr  18.85  Hydrocarbons  11.  CHO  12.  6  CH  CH 3  – CH – CH 2 Br  1.  at a­carbon.  Br 2 /hu  So.  (b)  :  The  reactivity  order of abstraction  of  H atoms  towards  bromination of alkane is 3°H > 2°H > 1°H.  CH 3  5  4  3  2  H  2.3­dimethylbutane  Due to the presence of chiral carbon atom. it is optically active. 1­dibromoethene  CH 3  CH 3  – CH    CH 2  HBr  CH 3  – CH – CH 3  CH 3  Br  + C  CH  CH 3  migration  +  CH 3  C  (2º)  CH 3  2­bromopropane  CH 3  OH  Br CH 3  –  C    CH  HBr  CH 3  – C    CH 2  CH 3  – C – CH 3  Br  Br  2.  (b) :  CH 3  C  H +  CH 2  CH  CH 3  Br  1. 4­dimethylheptane  H  Cl 2  CH 2 Cl  CH 3  H 3 C – C  —  C – CH 3  H  H  COOH  CH 2  – CH 3  H  +  CH 3  CH 3  KMnO 4  14.  (b) :  HO – C* – H  CH 3  CH 3  H 3 C – C  –  C – CH 3  CH 2 OH  H  2.  (d) :  CH 3  D  CH – CH      CH 2  HBr  40°  CH 3  Me  N  Et  –  OH  nBu  In Hofmann elimination reaction.2­addition  Therefore.  CH 2  more hindered b­H 15.3­dimethylbutane  has  two  types  of  hydrogen  atoms  so  on  monochlorination gives only two monochlorinated compounds.4­addition product is thermodynamically controlled product  entire side chain (in benzene homologues) with atleast one H  and formed at  comparatively higher temperature.  19.  (b) : CH 3  Br  HC    CH  HBr  HBr  H – C      CH 2  H – C – CH 3  Br  16.  (b) : 1.  (b) : The number of monohalogenation products obtained from  any  alkane  depends  upon  the  number  of  different  types  of  hydrogen it contains.4­addition  +  CH 3 CH(Br) – CH     CH 2  +  less hindered b­H  CH 2 (Br)CH     CHCH 3  1.  hence it is expected to  rotate plane  of polarized  light. 1­bromo­2­butene will be the main product under  thermodynamically  controlled  conditions.2­addition product is kinetically controlled product while    2O    7.  1.   C 2 H 5  Optically active due to  presence of  chiral carbon  atom.86  JEE MAIN CHAPTERWISE EXPLORER  20. the lower is  the boiling point.  CH(OH)  CHOH  2  HOCl  HOCl  Then­alkanes have larger surface area in comparison to branched  25.  (c) :  CH 3  – C *  three types of hydrogen  (three monohalogenation  product)  one type of hydrogen  (one monohalogenation product)  neopentane  Thus the given alkane should be neopentane. Thus.  Compound  containing  only  one  type  of  hydrogen  gives  only  one  monohalogenation  product. .  Acetylene reacts with the other three as:  CH  CNa  Na  NH 3(l)  CH  HCl  CH  CH 2  HCl  CHCl  CH 3  CHCl 2  [AgNO 3  + NH 4 OH]  CAg  CAg + NH 4 NO 3  white ppt.  (d) : The number of monohalogenation products obtained from  26.  CH 3 CH 2 CH 3  two types of hydrogen  (two monohalogenation product)  propane  CH 3 CH 2 CH 2 CH 2 CH 3  pentane  two types of hydrogen  (two monohalogenation product)  be the stronger acid H 2 O and the stronger base  CH 3  C  C  .  (d) : Among the isomeric alkanes.  O  CH 3  – CH 2  – C – CH 3  Zn ­ Hg  Butan­2­one  HCl  CH 3  – CH 2 CH 2  – CH 3  Butane  CH 3  CH 3  – CH – CH 2  – CH 3  isopentane  CH 3  H 3 C – C – CH 3  CH 3  H  21. therefore.  (c) :  CH    CH  CHCl  CHCl  2  chain isomers (as the shape approaches that of a sphere in the  CHO  branched chain isomers). they have lower boiling  CHCl 2  points in comparison to straight chain isomers. intermolecular forces are weaker  –H 2 O  in branched chain isomers. acid and amide.  24.  (d)  :  Butan­2­one  will  get  reduced  into  butane  when  treated  with zinc and hydrochloric acid following Clemmensen reaction  whereas Zn/HCl do not reduce ester. The greater the  Butane  branching of the chain.  23.  (d) :  H 3 C – CH 2  – CH     CH 2  Pd/H 2  Butene ­ 1  22. the normal isomer has a higher  H 3 C – CH 2  – CH 2  – CH 3  boiling point than the branched chain isomer.  (a) : Acetylene does not react with NaOH because product would  any  alkene  depends  upon  the  number  of  different  types  of  hydrogen it contains.   Oxidation  of  (A)  gives  an acid (B).  What is DDT among the following?  (2006)  (a)  A fertilizer  (b)  Biodegradable  pollutant  12. secondary or tertiary.  The structure of the compound that gives a tribromoderivative on  (c)  Non­biodegradable  pollutant  treatment with bromine water is  (d)  Greenhouse gas  (2012)  CH 3  Consider the  following  bromides  :  CH 2 OH  (a)  (b)  OH  The  correct  order of  S N 1  reactivity  is  (a)  A >  B >  C  (b)  B >  C >  A  (c)  B >  A > C  (d)  C >  B > A  CH 3  CH 3  OH  (c)  (2010)  (d)  OH  (2006)  . C 8H  7.87  Organic Compounds Containing Halogens  ORGANIC COMPOUNDS  CONTAINING HALOGENS CHAPTER  23  1.  4.  A solution of (–) – 1 – chloro – 1 – phenylethane in toluene  racemises slowly in the presence of a small amount of SbCl 5 .  is  (a)  CH 3 Cl  (b)  (C 2 H 5 ) 2 CHCl  (c)  (CH 3 ) 3 CCl  (d)  (CH 3 ) 2 CHCl  (2008)  9.  (c)  1­phenylcyclopentene  Which alcohol reacts  fastest and by what mechanism?  (d)  3­phenylcyclopentene.  Which of the following is the correct order of decreasing S N 2  reactivity?  (a)  R 2CH X > R 3C X > RCH      2 X  (b)  RCH X > R 3 C X > R 2 CH X  (c)  RCH 2  X > R 2CH X > R 3C X      (d)  R 3 C X > R 2 CH X > RCH 2  X  (X is a halogen)  (2007)  CH 2 Br  CH 2 Br  (a)  (b)  CH 3  CH 3  CH 2 Br  C 2 H 5  (c)  (2013)  (d)  Br  CH 3  2.  5.  The  organic  chloro  compound.  C 8 H 6 O 4 .  Compound (A).  (2006)  (a)  Tertiary  alcohol  by  S N 2. Identify the  compound (A).  (a)  by heating phenol with HF and KF  (d)  Secondary alcohol  by  S N 2.  Fluorobenzene (C 6 H 5 F) can  be synthesised in the laboratory  (c)  Tertiary  alcohol  by  S N 1. gives a white precipitate when warmed    with alcoholic AgNO 3 .  (b)  Secondary alcohol  by  S N 1.  6.  Which of the following on heating with aqueous KOH produces  acetaldehyde?  (a)  CH 3COCl  (b)  CH 3CH      2 Cl  (c)  CH 2 ClCH 2Cl  (d)  CH  CHCl  (2009)    3  2  8.  (2013)  (b)  from  aniline  by  diazotization  followed  by  heating  the  Iodoform  can  be  prepared  from  all  except  diazonium salt with HBF 4  (a)  isopropyl  alcohol  (b)  3­methyl­2­butanone  (c)  by direct fluorination of benzene with F 2  gas  (c)  isobutyl  alcohol  (d)  ethyl  methyl  ketone.  (2012)  (d)  by reacting bromobenzene with NaF solution.  which  shows  complete  stereochemical inversion during a S N 2 reaction.  due  to  the  formation  of  10.  Reaction  of  trans­2­phenyl­1­bromocyclopentane  on  reaction  (a)  free  radical  (b)  carbanion  with alcoholic KOH produces  (c)  carbene  (d)  carbocation.    9Br. (B) easily forms anhydride on heating.  11.  (2013)  (a)  4­phenylcyclopentene  An unknown alcohol is treated with the “Lucas reagent” to  (b)  2­phenylcyclopentene  determine whether the alcohol is primary.  3.   (d)  13.  The  structure  of  the  major  product  formed  in  the  following  reaction is  NaCN  DMF  Cl  CN  (b)  Cl  NC  CN  I  Cl  (c)  CN  (d)  (2006)  I  CN  (2004)  18.  Bottles containing C 6 H 5 I and C 6 H 5 CH 2 I lost their original labels. The end  (a)  alkenes  (b)  alkyl copper halides  solution in each tube was made acidic with dilute HNO 3  and  (c)  alkanes  (d)  alkenyl halides.88  JEE MAIN CHAPTERWISE EXPLORER  13.  Alkyl halides react with dialkyl copper reagents to give  taken in a test tube and boiled with NaOH solution.  (d)  (d)  11.3­dichloropentane  (d)  2­hydroxypropanoic  acid  (2004)  14.  (2003)  (a)  1­chloropentane  (b)  2­chloropentane  Answer  Key  1. A and B were separately  15.  (d)  19.  (c)  5.  (c)  .  15.  (2005)  then  some AgNO 3  solution  was  added.  Tertiary  alkyl  halides  are  practically  inert  to  substitution  by  yellow  precipitate.  21.  (c)  (c)  (a) 10.  Acetyl  bromide  reacts  with  excess  of  CH 3 MgI  followed  by  treatment with a saturated solution of NH 4 Cl gives  (a)  acetone  (b)  acetamide  (c)  2­methyl­2­propanol  (d)  acetyl  iodide.  Substance  B  gave  a  16.  Which of the following will have a  meso­isomer also?  (a)  2­chlorobutane  (b)  2.  17.  (c)  4.  (2004)  (c)  predominantly  1­butene  (d)  predominantly  2­butyne.  (2005)  21.  Which of the following  compounds  is not chiral?  (d)  Addition of HNO 3  was unnecessary.  (b)  3.  16.  (a)  18.  (b)  9.  (b)  20.  The compound formed on heating chlorobenzene with chloral  formation  of  in the presence of concentrated sulphuric acid is  (a)  equimolar mixture of  1 and 2­butene  (a)  gammexene  (b)  DDT  (b)  predominantly  2­butene  (c)  freon  (d)  hexachloroethane.  They were labelled A and B for testing.  (2005)  (b)  A was C 6 H 5 CH 2 I  (c)  B was C 6 H 5 I  17. Which  one  of  the  following  statements  is  S N 2 mechanism because of  true for this  experiment?  (a)  insolubility  (b)  instability  (a)  A was C 6 H 5 I  (c)  inductive  effect  (d)  steric hindrance.3­dichlorobutane  (c)  2.  (a)  14.  (d)  8.  (2004)  I  (a)  (c)  1­chloro­2­methylpentane  (d)  3­chloro­2­methylpentane  19.  Elimination  of  bromine  from  2­bromobutane  results  in  the  20.  (b)  2.  (b)  (a)  12.  (c)  6.  (a)  7.  in the transition state there will be five groups  attached  to  the  carbon  atom  at  which  reaction  occurs.  and hence can give  tribromoderivative.  (d) :  KOH  5. Compound (B).  step.  (b)  :  S N 1  reaction  rate  depends  upon  the  stability  of  the  3­phenylcyclopentene  carbocation. turbidity appears immediately with tertiary  alcohol by S N 1 mechanism. and the bulkier  the  group. as we move from  1° alkyl halide to 3° alkyl halide.  4.  (c) : In Lucas test.  a d–  HO  –  or  d  d– X  C  X  b  d  (transition state)  a  C  + X –  b  d  HO  group  In S N 2 reaction. .  (c)  H  Ph  Ph  6.  (c) : The compounds with  form  iodoform.  the  order  of  reactivity  is  thus.  H  10.  the  more  the  reaction  will  be  hindered  sterically. because it has  +I effect increases  which  makes  the  carbon  bearing  halogen  less positively polarised and hence less readily attacked by the  two ortho and one para position free with respect to OH group  nucleophile. the nucleophile attacks from back side  resulting in the inversion of molecule.  (c) :  S N 2 mechanism occurs as  a  1.  Thus  there will be crowding in the transition state. as carbocation formation is the rate determining  It follows E2 mechanism. Also. So the decreasing order of reactivity  of halide  is  RCH 2 X  >  R 2 CHX  >  R 3 CX  (primary)  (secondary)  (tertiary)  Thus  all  the  compounds  except  isobutyl  alcohol  will  form  Br  H  iodoform.  (a) : Since the compound on treatment with Br 2 ­water gives a  tribromoderivative.  (d) :  +  N      NCl –  NH 2  0°C  HBF 4  NaNO 2  + HCl  –HCl  aniline  diazotization  benzene  diazonium  chloride  +  – C 6 H 5 N 2 BF 4  benzene diazonium  tetra fluoroborate  D  C 6 H 5 F   +   BF 3  +  N 2  fluorobenzene  8. the next stable carbocation is formed from  reactions take place when the two groups to be eliminated are  (C).89  Organic Compounds Containing Halogens  9.  it  is  a  2°  carbocation.  (b) :  7.  11.  they are attached i.  (a) : In S N 2 reactions.  (a) :  OH  +      C  b  (nucleophile)  2. therefore it must be m­cresol.e.  (d) : A carbocation intermediate is formed during racemisation.  Hence S N 2 reaction is favoured by small groups on the carbon  atom attached to halogens.  (A)  forms  the  least  stable  1°  trans and lie in one plane with the two carbon atoms to which  carbocation. E2 reactions are stereoselectively trans. forms a 2° allylic carbocation which is  Hughes  and  Ingold  proposed  that  bimolecular  elimination  the  most stable.  3. the crowding increases and  12.   alkyl halide  reacts  with lithium dialkyl cuprate.  Nu  ­­­­­­ C ­­­­­­ X –  H – C – Cl  3  OH  H – C – Cl  4  CH 3  – C – COOH  H – C – H  H  CH 3  2.3­dichlorobutane have meso isomer due to the presence of plane  of  symmetry.3­dichloropentane  2­hydroxypropanoic acid  2.2 bis  (p­chlorophenyl) ethane  or  DDT  b  Cl  H  H  H  H  O Mg I  CH 3  OH  CH 3  1  1­butene (20%)  C  C  H – C – Cl  CH 3 CH      CHCH 3  a  CH 3  NH 4 Cl  H – C – Cl  2­butene (80%)  d  CH 3  CH 3  a  CH 3  C  2­methyl­2­propanol  19.1­trichloro­2.  17.  20.  (d) :  Nu  +  O  OH  Br  I  –  C  Cl  2.  R 2 CuLi + R¢X ® RR¢ +  RCu + LiX  X  1  2  2  In elimination reaction of alkyl halide major product is obtained  according to Saytzeff’s rule. B  C 6 H 5 CH 2 I  NaOH  C 6 H 5 CH 2 ONa  HNO 3 /H +  C 6 H 5 CH 2 OH  AgNO 3  Yellow precipitate  .90  JEE MAIN CHAPTERWISE EXPLORER  CH 3  CH 3  Br 2 ­H 2 O  Br  OH  m­cresol  18.  Thus there will be crowding in the transition state.  (b) :  14.  (d) :  CH 3  KOH (alc.  the  alkene  which  is  most  substituted  one  predominates. in the transition state.  d  H  a  C  Nu  +  X –  d  Cl  CCl 3 CH O +  H  Cl  In an S N 2 reaction.  (a)  : To  be  optically  active  the  compound  or  structure  should  possess chiral or asymmetric centre.6­tribromo­3­methylphenol  CH 2  CH 3  No yellow precipitate  Thus A must be C 6 H 5 I.  (b) :  DDT  is  prepared by  heating  chlorobenzene  and  chloral  with concentrated sulphuric acid. there will be five groups  attached to the carbon atom at which  reaction  occurs.  H  Cl  H  H  H  *  H – C – C – C – C – C – H  H – C – C – C – C – C – H  H  H  H  H  H  H  H  H  H  H  1­chloropentane  2­chloropentane  A  C 6 H 5 I NaOH  C 6 H 5 ONa  HNO 3 /H +  C 6 H 5 OH  AgNO 3  Cl  CH 3  H  H  H  H  CH 3 Cl  H  H  *  *  *  H – C – C – C – C – C – H  H – C – C – C – C – C – H  H  H  H  H  H  H  H  H  H  H  1­chloro­2­methylpentane  3­chloro­2­methylpentane  Cl  H 2 SO 4  – H 2 O  CCl 3 CH  Cl  1.  (c)  :  In  Corey House  synthesis  of  alkane. (b) : CH 3 CH 2 CHCH 3  C  CH 3  CH 2 CN  NaCN  Cl  DMF  Br  b  CH 3  CH 3 Mg I  CH 3 Mg I  I  16.3­dichlorobutane  CH 3  15.)  2­bromobutane  Cl  O Mg I  +  CH 3 CH 2 CH      CH 2  b  1  2  CH 3  H – C – H  H – C – Cl  3  CH 3  CH 3  2­chlorobutane  2.4.  (a) :  of  bulky groups make  the reaction sterically hindered.  (c) :  Br  13.1. which states that when two alkenes  may  be  formed. and presence  21.   4.  (2010)  7.  4.  The  (c)  n­butyl  alcohol  (d)  n­propyl  alcohol.  HBr reacts with CH 2  CH – OCH 3 under anhydrous conditions  (c)  ethyl  chloride  (d)  ethyl  ethanoate.  4.  .  The  major  product  obtained  on  interaction  of  phenol  with  sodium hydroxide  and  carbon dioxide is  (a)  benzoic  acid  (b)  salicylaldehyde  (c)  salicylic  acid  (d)  phthalic acid.  (2012)  Phenol.  (d)  H 3 C – CHBr – OCH 3 .91  Alcohols.  6­tribromophenol.  OH  NO 2  (III)  OCH 3  (IV)  (b)  II > IV > I > III  (d)  III > I > II > IV  (2013)  In the following sequence of reactions.  CH 3 CH 2 OH  P + I 2  A  Mg  ether  B  HCHO  C  H 2 O  D  the compound D is  (a)  propanal  (b)  butanal  Sodium  ethoxide  has  reacted  with  ethanoyl  chloride.  Ortho­nitrophenol  is  less  soluble  in  water  than  p­  and  m­nitrophenols  because  (a)  o­nitrophenol shows  intramolecular  H­bonding  (b)  o­nitrophenol shows  intermolecular  H­bonding  (c)  melting  point  of  o­nitrophenol  is  lower  than  those  of  9.  Arrange the following compounds in order of decreasing acidity.  Phenyl magnesium bromide reacts with methanol to give  (a)  a mixture of anisole and Mg(OH)Br  (b)  a mixture of benzene and Mg(OMe)Br  (c)  a mixture of toluene and Mg(OH)Br  (d)  a mixture of phenol and Mg(Me)Br.  6. is  (a)  1­Butanol  (b)  2­Butanol  (c)  2­Methylpropan­2­ol  (d)  2­Methylpropanol.  3.  (2011)  at room temperature to give  Phenol is heated with a solution of mixture of KBr and KBrO 3 .  OH  OH  .  OH  .  + CHCl 3  + NaOH  CHO  The  electrophile involved in the above  reaction  is  (a)  Å  (a)  dichloromethyl  cation  (CHCl 2 ) (b)  dichlorocarbene  (: CCl 2)    (b)  (c)  trichloromethyl  anion  (CCl 3 )  Å  (d)  formyl cation  (CHO) (c)  (d)  (2010)  (2006)  12.  (2006)  .HCl and anhydrous ZnCl 2 .  5.  (a)  CH 3 CHO and CH 3 Br  The  major product obtained  in  the  above  reaction is  (b)  BrCH 2 CHO and CH 3 OH  (a)  2­bromophenol  (b)  3­bromophenol  (c)  BrCH 2  – CH 2  – OCH 3  (c)  4­bromophenol  (d)  2.  From amongst the following alcohols the one that would react  fastest with conc.  (2008)  Cl  CH 3  (I)  (II)  (a)  IV > III > I > II  (c)  I > II > III > IV  2. Phenols and Ethers  ALCOHOLS.  (2009)  8. 6­trinitrobenzene  (c)  o­nitrophenol  (d)  p­nitrophenol. when it first reacts with concentrated sulphuric acid  and then with concentrated  nitric acid.  (2006)  (2011)  The  main product  of  the  following reaction  is  –  ONa +  OH  11. PHENOLS  AND ETHERS CHAPTER  24  1.  m­ and p­isomers  (d)  o­nitrophenol  is  more  volatile  in  steam  than  those  of  m­ and p­isomers.  (2007)  compound that is  produced in  this reaction  is  (a)  diethyl  ether  (b)  2­butanone  10. gives  (a)  nitrobenzene  (b)  2.   (None  of  the  option  is  correct)  (b)  14. This is due to  (a)  dipolar character of ethers  (b)  alcohols having resonance structures  (c)  inter­molecular hydrogen bonding in ethers  (d)  inter­molecular hydrogen bonding in alcohols.  (d)  4.  For which of the following parameters the structural  isomers  C 2 H 5 OH and  CH 3 OCH 3  would  be expected to have  the  same  (d)  (c)  values? (Assume ideal behaviour)  CH 2 COOH  (2005)  OH  (a)  Heat  of  vaporisation  OH  (b)  Vapour pressure at the same temperature  14.92  JEE MAIN CHAPTERWISE EXPLORER  13.  (a)  8.  The  latter  on  acidic  hydrolysis  gives  chiral  carboxylic acid.  (a)  7.  During  dehydration  of  alcohols  to  alkenes  by  heating  with  concentrated H 2 SO 4  the initiation step is  (a)  protonation  of  alcohol  molecule  15.  (d)  5.  (2003)  (b)  CH 3 CH 2 CH 2 CHCH 3  CH 3  (c)  CH 3 CH 2 CCH 2 CH 3  OH  Answer  Key  1.3­dimethyl­1­hydroxy  cyclohexane  1.  (b)  18.  Among the following compounds which can be dehydrated very  (b)  formation  of  carbocation  easily?  (c)  elimination of water  (a)  CH 3 CH 2 CH 2 CH 2 CH 2 OH  (d)  formation of an  ester.  15.  (d)  2.  (b)  6.  (c)  9.1­dimethyl­3­hydroxy  cyclohexane  3.  13.  An  ether  is  more  volatile  than  an  alcohol  having  the  same  molecular formula.  18.  (d)  (d) 3.  (d)  (c)  11.  16. The  structure  of  the carboxylic  acid is  (d)  CH 3 CH 2 CHCH 2 CH 2 OH  16.  19.  (c)  12.  p­cresol reacts with chloroform in alkaline medium to give the  compound  A  which  adds  hydrogen  cyanide  to  form  the  compound  B.1­dimethyl­3­cyclohexanol.  (d)  .  The IUPAC name of the compound  (a)  (b)  (c)  (d)  CH(OH)COOH  (a)  (b)  CH(OH)COOH  OH  OH  is  HO  CH 3  CH 3  (2004)  CH 3  3.  (b)  17.  (2004)  CH 3  17.3­dimethyl­1­cyclohexanol  1.  (d)  (c)  10.  (2003)  (2005)  OH  19.  The  best  reagent  to  convert  pent­3­en­2­ol  into  (c)  Boiling points  pent­3­en­2­one is  (d)  Gaseous densities at the same temperature and pressure  (a)  acidic permanganate  (2004)  (b)  acidic dichromate  CH 3  CH 2 COOH  (c)  chromic anhydride in glacial acetic acid  (d)  pyridinium  chlorochromate. 93  Alcohols.  7.HCl and anhydrous ZnCl 2 is Lucas reagent.  1° alcohol + Lucas reagent  No reaction.  Mg  H – C  – OH  CH 2 CH 3  n ­propyl  alcohol  (D)  H 2 O  HCHO  H – C  – OMgI  CH 2 CH 3  (C)  .  Thus. conc.  (d) :  CH 3 CH 2 OH  P  CH 3 CH 2 I  I 2  CH 3 CH 2 MgI  (B )  (A )  H H  The  preferential  formation  of  this  compound  is  due  to  conjugation  in  the  compound.6­tribromophenol.  2° alcohol + Lucas reagent  Turbidity after  5  mins. H2 SO 4  373 K Phenol  5.  :  (None  of the  option is  correct)  OH  OH  SO3 H  +  Conc. 2° and 3° alcohols. Phenols and Ethers  1.e.  (d) : Electron donating groups (—CH 3  and —OCH 3 ) decrease  while electron withdrawing groups (—NO 2 and —Cl) increase  the acidity.4..  therefore  order  of  decreasing  acidity  is  III > I > II > IV. The  product  formed  is  salicylic  acid.  (d) : KBr (aq)  + KBrO 3(aq)  Br 2(aq)  This bromine reacts with phenol gives 2. .  (b) :  OH  2­Phenol sulphonic  acid  SO3 H  4­Phenol sulphonic  acid  OH  Conc.  8.  6. i.  (a) : o­Nitrophenol is stable due to intramolecular hydrogen  bonding.  (d) :  4.  (c) : The reaction of phenol with NaOH and CO 2  is known as  Kolbe­Schmidt or  Kolbe’s  reaction.  Since  —OCH 3  is  a  stronger  electron  donating  group  than  —CH 3  and  —NO 2  is  stronger  electron  withdrawing  group  than  —Cl.  which is used to distinguish between 1°.  3.  2. HNO 3  D  NO 2  O2 N  NO 2  Picric acid  9.  3° alcohol + Lucas reagent  Immediate  turbidity.  It is difficult to break the H­bonding when dissolved in water  thus less soluble.  (c) : The reagent. the required  alcohol is 2­methylpropan­2­ol.   19.3­dimethyl­1­cyclohexanol  Molecular weight  2 As both the compounds have same molecular weights. the initiation step is protonation of alcohol.  (b) : CH 3OH  +  C  ®  C 6H    6H    5MgBr     6  +  Mg(OCH 3)Br    CH 3  13. . It is hydrolysed  rapidly by dilute acids  at room  temperature  to give  methanol  and aldehyde. Hence.  (d) : The reason for the lesser volatility of alcohols than ethers  is the intermolecular association of a large number of molecules  due to hydrogen bonding as – OH group is highly polarised.  (c) :  Br  CH 3 CH 3  CH(OH)COOH  OH  14.  (d) : Vapour density  =  18.  hydrogen bonding  No such hydrogen bonding is present in ethers. boiling point and heat of vaporization  will differ as ethanol has hydrogen bonding whereas ether does  not.  OH –  + CHCl 3  ƒ  HOH +  : CCl 3–  ® Cl –  +  : CCl 2  12.  :  CCl 2  generated    from  chloroform  by the action  of  a  base.  R  R  R  R  ­ ­ ­ ­ O – H ­ ­ ­ ­ O – H ­ ­ ­ ­ O – H ­ ­ ­ ­ O –  H  15.  (d)  :  Pyridinium  chlorochromate  oxidises  an  alcoholic  group  selectively in the presence  of  carbon­carbon  double  bond.  (b) :  +  ONa  + CHCl 3  + NaOH  CHO  (Reimer­Tiemann reaction)  The  electrophile  is  dichlorocarbene.  17.  (a) : Dehydration of alcohol to alkene in presence of concentrated  H 2 SO 4  involves following steps :  H +  – C – C –  –H 2 O  H  OH  H  OH 2  alcohol  protonated  alcohol  – C – C –  +  +  H  carbonium  ion  –H +  – C    C –  alkene  CH(CN)OH  OH  2  CH 3  H + /H 2 O  HCN  1  OH  CH 3  Å  The more stable carbocation is generated thus more easily it will  be dehydrated. The rest of these three  properties; vapour pressure.  (b) :  CH 3  CHO  OH  3  ®  3.  However.  Thus.  (c)  :  The  ease  of  dehydration  of  alcohols  is  tertiary > secondary > primary according to the order of stability  of  the  carbocations. both will  have the same vapour density. it  undergoes many addition  reactions at  the  double bond.  – C – C –  + CHCl 3  + KOH  CH 3  – CH 2  – C – CH 2  – CH 3  OH  HO  H 3 C – CH – OCH 3  H +  CH 3  – CH 2  – C – CH 2  – CH 3  16.  H 2 C     CH – OCH 3  H +  Å  H 2 C – CH – OCH 3  H Br –  OH  11. gaseous density of both  ethanol  and  dimethyl  ether  would  be  same  under  identical  conditions of temperature and pressure.94  JEE MAIN CHAPTERWISE EXPLORER  10. under anhydrous conditions at room temperature.  (d) : Methyl vinyl ether is a very reactive gas.   A liquid was mixed with ethanol and a drop of concentrated  H 2 SO 4 was added.  The compound formed as a result of oxidation of ethyl benzene  by KMnO 4  is  (a)  benzyl  alcohol  (b)  benzophenone  (c)  acetophenone  (d)  benzoic  acid.  (a)  A < B < C < D  (b)  D < B < C < A  trichloroacetate  ion  and  another  compound. Ketones and Carboxylic Acids  ALDEHYDES.  (a)  (b)  (c)  (d)  CO 2 H  is  (C)  CF 3 CO 2 H  (D)  Me  Ozonolysis of an organic compound  gives formaldehyde as  one  of the  products.  PhCOCH 3  D.  (2006)  (b)  a vinyl group  11.  which  of  the  following  is  the  most appropriate  reagent?  (a)  Zn­Hg/HCl  (c)  NaBH 4  3.  (2007)  Silver  mirror  test  is  given  by  which  one  of  the  following  compounds?  10.2­trichloroethanol  trichloromethanol  2.  KETONES  AND CARBOXYLIC ACIDS CHAPTER  25  1.  5.  CH 3COCH    3  by using NaOH.  (b)  Na.  6.  An organic compound A upon reacting with NH 3  gives B. The mixture of the products contains sodium  C.2­trichloropropanol  chloroform  In Cannizzaro reaction given below  –  2 PhCHO  :OH  PhCH 2 OH  +  PhCOO –  8.  OH –  (2012)  2.95  Aldehydes.  The liquid was  (a)  CH 3 OH  (b)  HCHO  (c)  CH 3 COCH 3  (d)  CH 3 COOH  (2009)  9. A compound with a fruity smell was formed.  Among the following the one that gives positive iodoform test  (c)  an isopropyl group  upon reaction with I 2  and NaOH is  (d)  an acetylenic  triple  bond.2.  (2011)  (a)  CH 3 CH 2 CH(OH)CH 2 CH 3  (b)  C 6 H 5 CH 2 CH 2 OH  The  strongest acid  amongst  the  following  compounds is  CH 3 (a)  CH 3 COOH  (c)  H 3 C  (d)  PhCHOHCH 3  (2006)  (b)  HCOOH  OH  (c)  CH 3 CH 2 CH(Cl)CO 2 H  12.  (2009)  CH 3  2.  (c)  CH 3 CH 2 CH 2 COOH  (d)  CH 3 —CH—COOH  (2013)  In  the  given  transformation.  The correct order of increasing acid strength of the compounds  (a)  Acetaldehyde  (b)  Acetone  (A)  CH 3 CO 2H  (B)  MeOCH 2 CO 2H      (c)  Formaldehyde  (d)  Benzophenone  (2011)  Me  4.  The increasing order of the rate of HCN addition to compounds  (d)  ClCH 2 CH 2 CH 2 COOH  (2011)  A ­ D is  Trichloroacetaldehyde was subjected to Cannizzaro’s reaction  A. NH 3  (d)  NH 2  – NH 2 .  The  other  (c)  D < C < B < A  (d)  C < D < B < A  (2006)  compound is  . This  confirms  the  presence  of  (a)  B < D < A < C  (b)  D < A < C < B  (a)  two  ethylenic  double  bonds  (c)  D < A < B < C  (d)  A < D < C < B.2.  HCHO  B. A is    (a)  CH 3 CH 2 COOH  (b)  CH 3COOH    7.  (2011)  the  slowest step is  (a)  the  attack of :OH –  at the  carboxyl group  (b)  the  transfer of  hydride  to  the carbonyl  group  (c)  the  abstraction  of  proton  from  the  carboxylic  group  (d)  the  deprotonation  of  PhCH 2 OH. C in presence of KOH reacts with Br 2  to  give CH 3CH    2 NH 2. liq. On  heating.  PhCOPh. B gives C.   (a.  When  CH 2  CH  –  COOH  is  reduced  with  LiAlH 4 .  The IUPAC name of CH 3 COCH(CH 3 ) 2  is  (c)  Butanol  (d)  Benzoic  acid  (2004)  (a)  isopropylmethyl  ketone  14.  CH 3  O  O  –  R – C  CH 3  R – C  + Nu  Z  –  (d)  + Z  Nu  is fastest when Z is  (a)  Cl  (b)  NH 2  (c)  OC 2 H 5  (d)  OCOCH 3 .  14.  In the anion HCOO  the two carbon­oxygen bonds are found to  (2002)  be of equal  length.  15.  10.  The general  formula C n H 2n O 2  could be  for open  chain  (a)  diketones  (b)  carboxylic  acids  (c)  diols  (d)  dialdehydes.  (c)  (d)  (c)  (c) 6.  (d)  The  anion  is  obtained  by  removal  of  a  proton  from  the  ® ethyl butanoate  acid  molecule.  (2003)  (b)  CH 3 COONa + C 2 H 5 OH  21. What is the  reason for it?  (a)  Electronic orbitals  of carbon  atom are  hybridised.  8.  the  (b)  2­methyl­3­butanone  composition of the resultant solution  is  (c)  4­methylisopropyl  ketone  (a)  CH 3 COOC 2 H 5  + NaCl  (d)  3­methyl­2­butanone.  Cl 2  22.  Which of the following compounds has wrong IUPAC name?  O bond is weaker than the C – O bond.  (b)  (c)  (a)  (c)  5.  Rate of the reaction.  19.  (a)  12. KOH  (2002)  B  What is B ?  (a)  CH 3 CH 2 COCl  (b)  CH 3 CH 2 CHO  (c)  CH 2  CHCOOH  (d)  ClCH 2 CH 2 COOH  (2004)  –  17.  (b)  CH 3  – CH – CH 2  – CHO  ® 3­methylbutanal  (2003)  CH  3 18.  (2003)  (c)  CH 3  – CH – CH – CH 3  ® 2­methyl­3­butanol  OH CH 3  O 19.  22.  13.  17.  (d)  (d)  (a)  (d)  3.  (c)  CH 3 COCl + C 2 H 5 OH + NaOH  (CH 3 ) 2 C  CH – CH 2  – CHO gives  (d)  CH 3 Cl + C 2 H 5 COONa.  (a)  (b)  (b)  (b)  2.  CH 3 CH 2 COOH  red P  A  alc.  20.  23.  On  vigorous  oxidation  by  permanganate  solution.  CH 3  COOH + CH 3 CH 2 COOH  CH – OH + CH 3 CH 2 CH 2 OH  C      O + CH 3 CH 2 CHO .  Consider the acidity  of  the  carboxylic  acids:  (i)  PhCOOH  (ii)  o­NO 2C    6H    4COOH    (iii)  p­NO 2C    6H    4COOH    (iv)  m­NO 2C    6H    4COOH    Which of the following  order  is  correct?  (a)  i > ii > iii > iv  (b)  ii > iv > iii > i  (c)  ii > iv > i > iii  (d)  ii > iii > iv > i  (a)  CH 3  – C  –  CH – CH 2 CH 3  CH 3  CH 3  (b)  CH 3  (2004)  (c)  CH  3  16.  21.  (b)  The C  (a)  CH 3  – CH 2  – CH 2  – COO – CH 2 CH 3 (c)  The anion HCOO –  has two resonating structures.  the  compound obtained will be  (a)  CH 3  – CH 2  – COOH  (b)  CH 2  CH – CH 2 OH  (d)  CH  – CH – C – CH  – CH  ®  2­methyl­3­pentanone 3  2  3  CH 3  (2002)  Answer  Key  1.  (b)  .  (c)  18.  (2004)  OH  OH  15.  Which one of the following undergoes reaction with 50% sodium  (c)  CH 3  – CH 2  – CH 2 OH  hydroxide solution to give the corresponding alcohol and acid?  (d)  CH 3  – CH 2  – CHO. c)  (d)  (d)  (b)  4.  16.96  JEE MAIN CHAPTERWISE EXPLORER  13.  7.  (2003)  (a)  Phenol  (b)  Benzaldehyde  20.  11.  23.  On  mixing  ethyl  acetate  with  aqueous  sodium  chloride.  9.  H2SO4  are  added must be an acid. Ketones and Carboxylic Acids  O  O  1.  R  R  10.  tend  to increase  the  acid  strength  of  substituted acid.  Mechanism :  –  C6 H 5 – C  O  +  OH  CH 3 —CH 2 —C—NH 2  ( C )  Fast  H  2.  3. c) : Formaldehyde  and  acetaldehyde  can  be  oxidised  by  Tollen’s reagent  to give  silver  mirror.  (c) : Effect of substituent on the acid strength of aliphatic acids:  (i)  Acidity  decreases  as  the  +I­effect  of  the  alkyl  group  increases.  \  CH 3 COOH  +  C 2 H 5 OH  conc.  (ii)  Acidity decreases as the  –I­effect as well  as  number  of  6. H­transfer to the carbonyl group  is the rate  determining step and hence the  slowest.  (d) :  H  H  –  C6 H 5 – C – O  +  O  C – C6 H 5  Slow. etc. regardless of length is oxidised to – COOH.97  Aldehydes. –  NH 2  etc.    (d) :  Ethyl benzene  4.  5.  (a) :  In  Cannizzaro’s  reaction  one  molecule  is  oxidised  to  halogen atoms decreases. it is an  ester. –  OH.  On the basis of given information the relative order of increasing  acid strength of the  given compounds is  (CH 3 ) 2 COOH < CH 3 COOH < CH 3 OCH 2 COOH < CF 3 COOH Vinyl group (CH 2  CH –) on ozonolysis gives formaldehyde.  COOH  CH 2  – CH 3  KMnO 4  9.  Hydride transfer  OH  H  C6 H 5 – C  O  +  O –  C – C6 H 5  OH  to  –  CHOH  only.  (a) :  CH 3 —CH 2—C—OH  NH 3  ( A )  7.  Rearrangement  H  –  –  OH  group  and  alkene  are  acid‑sensitive  groups  so  Clemmensen reduction cannot be used and NaBH 4  reduces  –  8.  In  case of Cannizzaro reaction.  (c) : CH 3 CH 2 CH(Cl)COOH is the strongest acid due to –I effect  HCOOH > CH 3COOH > (CH    3)    2CHCOOH > (CH    3)    3CCOOH    of  Cl. tend to decrease while electron withdrawing substituents  like –NO 2 .  –CHO.  . the  entire side chain (in benzene homologues) with atleast one H  at a­carbon.  C6 H 5 –  CH 2 – OH  +  C6 H5 COO  (d) : Since the compound formed has a fruity smell.  (a.  FCH 2COOH > ClCH    2COOH > BrCH    2COOH >    ICH 2COOH > CH    3COOH    F 3 CCOOH > F 2 CHCOOH > FCH 2 COOH > CH 3 COOH  (iii)  Electron donating substituents like  – R.  (b) :  Benzoic acid  When oxidises with alkaline KMnO 4  or acidic Na 2 Cr 2 O 7 .H 2 SO 4 CH 3 COOC 2 H 5 + H 2 O.  –  + CH 3 —CH 2 —C—ONH 4  ( B )  D  O  CH 3 —CH 2 —NH 2  Br 2 /KOH  (b) :  Rate  determining  step  is  always  the  slowest  step. thus the liquid to which ethanol and conc.  carboxylate  ion  and  the  other is  reduced  to  alcohol.   (d) :  H – C –  C – C  –   C – H  H  CH 3  H  3­methyl­2­butanone  21.  (d) :  NO 2  1  2  3  Cl 2  red P  –HCl  CH 3 CHClCOOH  (A)  alc.  From the above information.  (c) : HCOO –  exists as  C      O  O–  O H–C O H – C – O– The aromatic aldehydes and ketones are less reactive than their  So. it reduces carboxylic  the rate of HCN addition to compounds HCHO.  NO 2 )  at  p­position  have  more  pronounced  electron withdrawing effect than as – NO 2  group at m­position  (–I  effect) \ Correct order of acidity is ii > iii > iv > i.  D PhCHOHCH 3  + 4I 2  + 6NaOH ¾¾ ®  CHI 3  + PhCOONa + 5NaI + 5H 2 O  p  isomer due to ortho­effect.  13.e.  (d) : Iodoform test is given by only the compounds containing  CH 3 CO  – or CH 3 CHOH –  group.  (b) : Aldehydic group gets oxidised to carboxylic group.  (b) : LiAlH 4  is a strong reducing agent. This is due to the +R effect of the benzene  ring.  (c) : CH 3 CH 2 COOH  chloride.  decreases. KOH  –HCl  CH 2  (B)  4  NO 2  23. the two carbon­oxygen bonds are found to be of equal length. the reactivity of the ketones further  overlapping between the 3p orbital of Cl and 2p orbital of carbon.98  JEE MAIN CHAPTERWISE EXPLORER  11.  aliphatic analogous.  18.  OH  CH 3  o­isomer  will  have  higher  acidity  then  corresponding  m  and  3­methyl­2­butanol CHCOOH  .  Double  bond  breaks  and  carbon  gets  oxidised  to  carboxylic  group.  group into primary alcoholic group without affecting the basic  PhCOCH 3  and PhCOPh is  skeleton  of  compound.  14.  PhCOPh < PhCOCH 3  < CH 3 COCH 3  < HCHO. The basicity of the leaving group  H  R  H  increases as  The  lower reactivity  of  ketones  over  aldehydes  is  due to +I­  –  Cl –  < RCOO –  < RO –  < NH 2  effect of the alkyl (R) group and steric hindrance.  CHO  CH 2 OH  50% NaOH  Benzaldehyde  Benzyl alcohol  COONa  +  Sodium benzoate  CH 2  CH – COOH  H  O  H  1  2  3  LiAlH 4  [H + ]  CH 2  CH – CH 2 OH  H  4  20. As the size  Secondly  least  stabilization  by  resonance  due  to  ineffective  of the alkyl group increases. it is clear that increasing order of  19.e.  As  M  group  (i.  CH 3  CH 3  (CH 3 ) 3  (CH 3 ) 2 CH  C      O  >  C      O  >  (CH 3 ) 3  (CH 3 ) 2 CH  17. Dialdehydes ­ C    n H n O 2 .  12.  (a) :  R – C  R – C  + Z  + Nu  Order  of  reactivity:  Nu  Z  H  R  Reactivity of the acid derivatives decreases as the  basicity  of  R  C      O  >  C      O  C      O  >  the leaving group increases.  (b) : Diketones ­ C nH    2n – 2 O 2..  (a) : CH 3COOC    2H    5  + NaCl (aq) ® no reaction  i.  16.  (c) :  CH 3  – CH – CH – CH 3  Electron withdrawing group increases the acidity of benzoic acid.  the  resultant  solution  contains  ethyl  acetate  and  sodium  22.  (b)  :  Benzaldehyde  will  undergo  Cannizzaro  reaction  on  treatment  with  50%  NaOH  to  produce  benzyl  alcohol  and  benzoic acid as it does not contain a­hydrogen.  (c) : Addition of HCN to carbonyl compounds is a characteristic  O  O  –  –  nucleophilic  addition  reaction  of  carbonyl  compounds. CH 3 COCH 3 . Carboxylic acid ­ C    nH    2n O 2  PhCHO > PhCOCH 3  > PhCOPh  Diols ­ C n H 3n O 2.  COOH  COOH  COOH  COOH  NO 2  <  <  <  15.   When primary amine reacts with chloroform in ethanolic KOH  synthesis nor for separation of  amines?  then the product is  (a)  Hinsberg method  (b)  Hofmann  method  (a)  an  isocyanide  (b)  an aldehyde  (c)  Wurtz reaction  (d)  Curtius reaction  (2005)  (c)  a cyanide  (d)  an  alcohol.  CH 3 CH 2 NH 2  + CHCl 3  + 3KOH ® (A) + (B) + 3H 2 O.  8. On heating it gives NH 3  alongwith a solid residue.  9.  (b)  11.  (2002)  Answer  Key  1.  (a)  (d)  3.  (2003)  the reaction is continuously removed.  (a)  6.  CH 3 NH 2  and (CH 3 ) 2 NH is  (2005)  (a)  CH 3 NH 2  < NH 3  < (CH 3)    2 NH  (b)  (CH 3 ) 2 NH < NH 3  < CH 3 NH 2  Amongst the following the most basic compound  is  (a)  benzylamine  (b)  aniline  (c)  NH 3  < CH 3NH    2  < (CH 3)    2NH    (c)  acetanilide  (d)  p­nitroaniline  (2005)  (d)  CH 3 NH 2  < (CH 3 ) 2 NH < NH 3  (2003)  Which one of the following methods  is  neither  meant for the  13.  Ethyl isocyanide on hydrolysis in acidic medium generates  (c)  (NH 2 ) 2 CO  (d)  CH 3 CH 2 CONH 2  (2005)  (a)  ethylamine salt and methanoic acid  (b)  propanoic acid and ammonium salt  Reaction of cyclohexanone with dimethylamine in the presence  (c)  ethanoic acid and ammonium salt  of catalytic amount of an acid forms a compound if water during  (d)  methylamine salt and ethanoic acid.  the compounds (A) and (B) are respectively  (a)  C 2 H 5 NC and 3KCl  (b)  C 2 H 5 CN and 3KCl  10.  12.67% and  N  =  46. The compound  is  (a)  CH 3 NCO  (b)  CH 3 CONH 2  11. The compound formed is  generally known as  (a)  a  Schiff’s base  (b)  an enamine  (c)  an imine  (d)  an amine  6.  5.  A  compound  with  molecular  mass  180  is  acylated  with  8. The  number of amino groups present per molecule of the former  compound is  (a)  6  (b)  2  (c)  5  (d)  4  (2013)  Which of the following is the strongest base?  (a)  NH 2  (b)  (c)  NH 2  (d)  CH 3  NHCH 3  CH 2 NH 2  (2004)  In the chemical reaction.  3.67%  while rest is  oxygen.  2.  (c)  13.  (c)  10.  9.99  CHEMISTRY  ORGANIC COMPOUNDS  CONTAINING NITROGEN CHAPTER  26  1.  CH 3 COCl to get a compound with molecular mass 390.  The correct order of increasing basic nature for the bases NH 3 .  (a)  12.  7.  (d)  5. The  solid residue gives violet colour with alkaline copper sulphate  NC  (d)  H 3 C  (2003)  solution.  (c)  CH 3 CH 2 CONH 2  and 3KCl  (d)  C 2 H 5 NC and K 2 CO 3 .  (a)  2.  (2007)  Which one of the following does not have sp 2 hybridized carbon?  (a)  Acetone  (b)  Acetic acid  (c)  Acetonitrile  (d)  Acetamide  (2004)  Which  one of  the following  is  the  strongest  base  in  aqueous  solution?  (a)  Methylamine  (b)  Trimethylamine  (c)  Aniline  (d)  Dimethylamine  (2007)  (a)  H 3 C  CN  (b)  H 3 C  N 2 Cl  The reaction of chloroform with alcoholic KOH and p ­toluidine  forms  An  organic compound  having molecular mass  60  is found to  NHCHCl 2  (c)  H 3 C  contain C = 20%.  4.  (c)  7.  (d)  (c)  4.  (c)  .  H  =  6. 66  1.00  6. the  non­bonding electron pair of nitrogen is delocalized into benzene  ring by resonance.  (a) : Due to resonance of electron pair in aniline.  In  benzylamine  electron  pair  is  not  involved  in  resonance. This  anomalous  behaviour  of  tertiary amines is due to steric factors i. crowding of alkyl groups  cover nitrogen atom from all sides and thus makes it unable for  protonation.  NHCOCH 3  NH 2  CH 2 NH 2  NH 2  ethanolic  KOH..  (c) : In Wurtz reaction alkyl halide reacts with sodium metal  in the  presence of dry ether to give alkane.  8. the covering of alkyl groups  3  2    3  3  Acetonitrile  Acetamide  over  nitrogen  atom  from  all  sides  makes  the  approach  and  bonding  by  a  proton  relatively  difficult.  Electron  withdrawing  (C 6 H 5  –)  groups  decreases  electron density on nitrogen atom and thereby decreasing basicity.67  6.  (c) :  CH 3  – CO – CH 3  ;  CH 3  – COOH  Acetic acid  electrons to proton increases and hence the basicity of amines  Acetone  3  3  also increases.  Thus the relative strength is in order  (CH 3 ) 2 NH > CH 3 NH 2  > NH 3 .  then  a  bad  smell  compound  isocyanide  is  formed.  (c) : Except the amines containing tertiary butyl group.  4.  . This is called carbylamine reaction and this reaction is  used as a test of primary amines.  3.  H 3 C  (c) :  Element  Simplest  Percentage  Relative  no.  13. due to which basicity decreases.  (d) : The increasing order of basicity of the given compounds  is  (CH 3 ) 2 NH > CH 3 NH 2  > (CH 3 ) 3 N > C 6 H 5 NH 2  Due to  the  +I effect  of  alkyl groups.  (b) :  6. Further the presence of electron donating groups in  the  benzene  ring  increase  the  basic  strength  while  electron  withdrawing group decrease the basic strength of substituted  aniline.e.67  D  C 2 H 5 N       C + 2H 2 O  Ethylisocyanide  NH 2 CONHCONH 2  + NH 3-  biuret  Biuret gives violet colour with alkaline copper sulphate solution.100  JEE MAIN CHAPTERWISE EXPLORER  390 - 180  =  5  42 1..  (a)  :  When  a  primary  amine  reacts  with  chloroform  with  . the non­bonding electron pair of nitrogen  does not take part in resonance. The alkyl groups. basic strength  decreases. . So aliphatic amines are more basic than aniline.  sp  sp 2  sp  sp  ;  CH  – CONH  CH  – C      N  In case of tertiary anine (CH 3)  N.  .  hence  the  basicity  10. which are electron releasing  groups. as a result the electron density on the N atom  decreases.  (d) : H 3 C  NH 2  + CHCl 3  + 3KOH  decreases.  .33  1.  Primary  amine  Isocyanide .  11.  5.67  46.  N(CH 3 ) 2  O  + (CH 3 ) 2 NH  The  above  reaction  is  known  as  carbylamine  reaction  and  is  generally used to convert primary amine into isocyanide. of amino groups  =  2.  7.  D RNH 2  + CHCl 3  + 3KOH ¾¾   ® RNC + 3KCl + 3H 2O  NO  (ii) dehydration  enamine  2  I  II  III  IV  Decreasing order of basic strength is II > I > IV > III. increase the electron density around the nitrogen thereby  increasing the availability of the lone pair of electrons to proton  or Lewis acids and making the amine more basic. the compound is H 2 NCONH 2 . all lower  aliphatic amines are stronger bases than ammonia because of +I  (inductive) effect.67  3. In other three compounds.  (d) : In this compound.  2NH 2 CONH 2 NC + 3KCl + 3H 2 O  (i) H +  H +  C 2 H 5 NH 2  + HCOOH  Ethylamine  Methanoic  acid  12.  So.  1  4  2  1  The molecular formula  is  CH 4 N 2 O.  ratio  of  atom  C  H  N  O  20.  (a) : CH 3CH    2NH    2  + CHCl 3  + 3KOH  C 2H    5NC + 3KCl + 3H    2O    This is called carbylamine reaction.  (a) : Alkyl isocyanides are hydrolysed by dilute mineral acids  to form primary amines. The observed  order  in  the  case  of  lower  members  is  found  to  be  as  secondary  >  primary  >  tertiary.67  26..  (c) : No.  the  electron  density on  sp 3  sp 3  sp 2  sp 3  sp 2  nitrogen increases and thus the availability of the lone pair of  9. .  (a)  4.  2.  (b)  6.  polymerization  is  (a)  HNO 3  (b)  AlCl 3  (c)  BuLi  (d)  LiAlH 4  (2012)  6.  (b)  (a)  2.6  (d)  polystyrene.  (2002)  Answer  Key  1.  hydrogen bonding  is  (a)  natural rubber  (b)  teflon  (c)  nylon­6.101  Polymers  CHAPTER  POLYMERS 27  1.  (c)  .  (c)  3.6  (c)  Terylene  (d)  Bakelite  (2005)  Nylon threads are made of  (a)  polyvinyl  polymer  (b)  polyester  polymer  (c)  polyamide  polymer  (d)  polyethylene  polymer.  Bakelite  is  obtained  from phenol  by  reaction  with  (a)  HCHO  (b)  (CH 2 OH) 2  (c)  CH 3 CHO  (d)  CH 3 COCH 3  (2008)  4.  (2010)  3.  (b)  5.  Which of  the following  is  fully  fluorinated  polymer?  (a)  Neoprene  (b)  Teflon  (c)  Thiokol  (d)  PVC  (2005)  7.g.  The  polymer  containing  strong  intermolecular  forces  e.  7.  Which of the following is a  polyamide?  (a)  Teflon  (b)  Nylon­6.  (2003)  Polymer formation from monomers starts by  (a)  condensation reaction  between monomers  (b)  coordinate reaction  between monomers  (c)  conversion of monomer to  monomer ions by  protons  (d)  hydrolysis of  monomers.  The species which can best serve as an initiator for the cationic  5.   Cl  4.  acids such  as H 2SO    4.6 involves amide (CONH) linkage therefore.  n  (a)  :  Polymerisation  takes  place  either  by  condensation  or  addition reactions.  (b) : Neoprene :  — CH 2  – CH      C – CH 2  —  (c) : Nylon threads are polyamides.102  JEE MAIN CHAPTERWISE EXPLORER  1.  n  n HOOC(CH 2 ) 4 COOH + n H 2 N(CH 2 ) 6 NH 2  Teflon  :  — CF 2  – CF 2  —  Adipic acid  n  Thiokol  :  Nylon (polyamide)  n  Cl  n  Hexamethylene  diamine  HO – OC – (CH 2 ) 4  – CONH(CH 2 ) 6 NH  – CH 2  — CH 2  – S – S – CH 2  — CH 2  – S – S – CH 2 CH 2  –  PVC :  — CH 2  – CH —  280°C  high pressure  7.  (c) : Nylon­6.  Nylon­6. SnCl 4  or  BF 3  in  H 2O. 6  6.  ( HN – (CH 2 ) 6  – NHCO – (CH 2 ) 4  – CO ) n  (a) : Bakelite is a thermosetting  polymer which is made by  reaction between phenol  and  HCHO. AlCl    3 . HF.  3. . They are the condensation  polymers of diamines and dibasic acids.    2. it will  also have very strong inter molecular hydrogen bonding between  (b) : Polymers having amide linkages (– CONH) are known as  polyamides.  (b) : Cationic polymerisation is initiated by use of strong Lewis  5.  n(H 2 N(CH 2 ) 6 NH 2 ) + n(HOOC(CH 2 ) 4 COOH)  Hexamethylene  diamine                       Adipic  acid  amide linkage  group  of two  polyamide chains.   Insulin production and its action in human body are responsible  for the level of diabetes. heterocyclic base and phosphate ester  (a)  Sugars  (b)  Amines  linkages are at  (c)  Primary alcohols  (d)  Nitro  compounds  (2012)  (a)  C 5¢ and C 2¢ respectively of the sugar molecule  Which one of  the  following  statements is  correct?  (b)  C 2¢ and C 5¢ respectively of the sugar molecule  (a)  All amino acids are  optically active.  a­D­(+)­glucose and b­D­(+)­glucose are  (b)  Enzymes are normally heterogeneous catalysts that are very  (a)  enantiomers  (b)  conformers  specific in  action.  (2006)  14.  (2003)  .  (a)  fixed  configuration  of  the  polypeptide  backbone  (d)  Enzymes are specific biological catalysts that possess well­  (b) a­helical  backbone  defined active sites.  6.  Which base is present in RNA but not in DNA?  are  (a)  Uracil  (b)  Cytosine  (a)  –OH and –COOH  (c)  Guanine  (d)  Thymine  (2004)  (b)  –CHO and –COOH  13.103  Biomolecules  CHAPTER  28  BIOMOLECULES The term anomers of glucose refers to  (a)  isomers of glucose that differ in configurations at carbons  one and four (C­1 and C­4)  (b)  a mixture of (D)­glucose and (L)­glucose  (c)  enantiomers of  glucose  (d)  isomers of glucose that differ in configuration  at  carbon  one (C­1).  5.  In both DNA and RNA.  (2012)  11. 7.  4.  (2007)  The pyrimidine bases present in DNA are  (a)  cytosine and  adenine  (b)  cytosine and guanine  (c)  cytosine and  thymine  (d)  cytosine and  uracil.  (d)  C 5¢ and C 1¢ respectively of the sugar molecule  (c)  All amino acids except glutamic acid are optically active.  (c)  epimers  (d)  anomers.  8.  Synthesis  of  each  molecule  of  glucose  in  photosynthesis  9.  (c)  hydrophobic  interactions  (2004)  (d)  sequence of a­amino  acids.  The reason for double helical structure of DNA is operation of  (a)  van der Waal's forces  (b)  dipole­dipole  interaction  (c)  hydrogen  bonding  (d)  electrostatic  attractions. This compound belongs  to which  of  The presence or absence of hydroxy group on which carbon  the  following  categories?  atom of sugar differentiates  RNA and  DNA.  The two functional groups present in a typical carbohydrate  12.  (c)  C 1¢ and C 5¢ respectively of the sugar molecule  (b)  All amino acids  except  glycine  are  optically active.  (c)  C     O and –OH  (a)  Enzymes are specific biological catalysts that can normally  (d)  –OH and –CHO  (2009)  function at very high temperatures (T ~ 1000 K).  (a)  1 st  (b)  2 nd  (a)  A co­enzyme  (b)  A hormone  rd  th  (c)  3  (d)  4  (2011)  (c)  An  enzyme  (d)  An antibiotic  (2004)  3.  Identify the correct statement regarding enzymes.  Which  of  the  following  compounds  can  be  detected  by  Molisch’s  test?  10.  (2006)  1.  (2008)  (c)  Enzymes are specific biological  catalysts that  cannot be  The secondary structure  of a  protein refers  to  poisoned.  (2005)  (d)  All amino acids except lysine are optically active.  involves  (a)  6  molecules of ATP  (b)  18  molecules  of ATP  (c)  10  molecules  of ATP  (d)  8  molecules of ATP  (2013)  2.   (c)  3. which is found in  amino acid is  (a)  – COOH group  (b)  – NH 2  group  (c)  – CH 3  group  (d)  both (a) and (b).  (b)  10.  (c)  16.  The functional group.  (c)  14.  (b)  9.  (b)  7.  (d)  2.  (b)  6.  (d)  5.  (d)  12.  (2003)  16.  Complete hydrolysis  of  cellulose  gives  (a)  D­fructose  (b)  D­ribose  (c)  D­glucose  (d)  L­glucose.  (b)  17.  (b)  13.  (c)  11.104  JEE MAIN CHAPTERWISE EXPLORER  15.  RNA is different from DNA because RNA contains  (a)  ribose sugar and thymine  (b)  ribose sugar and uracil  (c)  deoxyribose sugar and thymine  (d)  deoxyribose sugar and uracil.  (2002) Answer  Key  1.  (2002)  17.  (a)  8.  (c)  4.  (d)  15.  (a)  .   Thus  the  two  functional  groups  present are  C     O (aldehyde  or ketone) and  –OH.  (b)  : The sugar molecule found in RNA is D­ribose while  the  sugar  in  DNA  is  D­2­deoxyribose. a form in which the OH at C 1  in aldoses and C 2  in ketoses  lies towards the right and b form in which it lies towards left.  (c) :  NH 2  N  OH  a ­D­(+)­glucopyranose  H  OH  b ­D­(+)­ glucopyranose  A  pair  of  stereoisomers  which  differ  in  configuration  at  C­1 are known as anomers.  6. fructose.  (b) :  6CO 2  + 18ATP + 12NADPH + 6RuBP ®  6RuBP + Glucose + 18ADP + 18P + 12NADP +  One  molecule of glucose  is formed from  6CO 2  by utilising  18ATP and 12NADPH.  4.  Thus glucose..  The  sugar  D­2­deoxyribose differs from ribose only in the substitution  of  hydrogen  for  an  –  OH  group  at  2­position  as  shown  in  figure. a­D­(+) glucose  ƒ  equilibrium mixture  ƒ b­(D)­ (+) glucose  As a result of cyclization the anomeric (C­1) becomes asymmetric  and the newly formed – OH group may be either on left or on  right in Fischer projection thus resulting in the formation of two  isomers (anomers).  9.  (c) : DNA contains cytosine and thymine as pyrimidine bases  and guanine and adenine as purine bases. all exist in a and b form.  3. based on the dehydration of carbohydrate  by sulphuric acid to produce an aldehyde.  (b) : Secondary  structure of  proteins is  mainly of  two types. .  (b) : Insulin is a proteinaceous hormone secreted by b­cells by  islet of Langerhans of pancreas in our body.  (d) : Due to cyclic hemiacetal or cyclic hemiketal structures. ribose.  7.  amino acid coils as a right handed screw (called a­helix)  because of the formation of hydrogen bonds between amide  groups of the same peptide chain.  with two molecules of phenol resulting in red or purple coloured  compound.  (d) : Structures of a­D­(+)­glucose and b­D­(+)­glucose are :  6  5  H  4  HO  H  OH  3  O  H  H  a 4 H  1  OH  HO  2  H  OH  3  H – C – OH  H – C – OH  H – C  H – C  CH 2 OH  b­D­glucose  a­D­glucose O  OH  H  1  OH  2  10.  HO – C – H H – C – OH  H – C – OH  H – C – OH  5. which condenses  8.e.  all the pentoses and hexoses exist in two stereoisomeric forms  i.  Glucose exists in two forms a­D­glucose and b­D glucose. The isomers having – OH group to the left  of  the  C­1  is  designated b­D­glucose  and  other  having  – OH group on the right as a­D­glucose.105  Biomolecules  1. etc.  (i) N  OH  5¢  H  O  HO – C – H  CH 2 OH  6  5  O  and  HO – P – O – CH 2  O  4¢  C  H  O  H  3¢  N  N  1¢  H  C  C  C 2¢  HO  OH  H  a­helix : This  structure  is formed when the  chain of a­  11.  (b) : Glycine is optically inactive while all other amino acids  are  optically active.  (c) :  Carbohydrates  are  essentially  polyhydroxy  aldehydes  HO – C – H  and  polyhydroxy  ketones.  (a) : Molisch’s test is a sensitive chemical test for the presence  of carbohydrates.  2.  (ii) b­plated  sheet  :  In  this  structure  the  chains  are  held  together by a very large number of hydrogen bonds between  C  O  and NH  of different  chains.  –NH 2 .  Cellulose  is  a  straight  chain  polysaccharide  composed  of  D­glucose  units  which  are  joined  by b ­glycosidic  linkages. guanine and adenine.  14.  (c) : (C 6H    10 O 5)    n  + nH 2O    Cellulose  three hydrogen  bonds  H +  nC 6H    12 O 6  D­glucose  derivatives  (b)  Purine  derivatives  (c)  Sugar  Thymine  Adenine  Guanine  Deoxyribose  Uracil  Adenine  Guanine  Ribose . as well as an amino  group.  13.e.106  JEE MAIN CHAPTERWISE EXPLORER  12.  (a)  Pyrimidine  Cytosine  Cytosine  Adenine     Thymine  Cytosine     Guanine  two hydrogen  bonds  15. Both have the same purine bases  i.  Hence cellulose on hydrolysis produces only D­glucose units.  (d)  :  An  amino  acid  is  a  bifunctional  organic  molecule  that  which normally  functions  effectively  at  body  temperature.  (b) :  DNA  RNA  of  their nucleotide  monomers.  (a) : RNA contains cytosine and uracil as pyrimidine bases while  DNA has cytosine and thymine.  contains both a carboxyl group. –COOH.  (d) : Enzymes are shape selective specific biological  catalysts  16.  (c) : The two polynucleotide chains or strands of DNA are linked  up by hydrogen bonding between the nitrogenous base molecules  17.   Which one of the following types of drugs reduces  fever?  (a)  Analgesic  (b)  Antipyretic  (c)  Antibiotic  (d)  Tranquiliser  (2005)  4.  (c)  5.  Aspirin in known as  (a)  phenyl  salicylate  (b)  acetyl  salicylate  (c)  methyl salicylic acid  (d)  acetyl salicylic acid  (2012)  2.  (2003)  (b)  OCOCH3  (c)  H2 C  CH – CN  and  H2 C  CH – CH  CH2  5.  is used as  (c)  (2002)  .  Which of the following could act as a propellant for rockets?  (a)  Liquid hydrogen + liquid nitrogen  (b)  Liquid oxygen + liquid argon  (c)  Liquid hydrogen + liquid oxygen  (d)  Liquid nitrogen + liquid oxygen.  (d)  2.  Cl  Answer  Key  1.  (c)  3.  (b)  4.107  Chemistry in Everyday Life  CHEMISTRY IN  EVERYDAY LIFE CHAPTER  29  1.  Buna­N synthetic  rubber is  a  co­polymer of  (a) 3.  and  (d)  H2 C  CH – CN  H2 C  CH – C  CH2  (a)  antiseptic  (c)  analgesic  (2009)  COOH The compound  (b)  antibiotic  (d)  pesticide.   (c) : Buna­N is a co­polymer of butadiene and acrylonitrile. .  (b) : An antipyretic is a drug which is responsible for lowering  temperature of the feverish organism to normal but has no effect  1.  (c)  :  The  compound  is  acetyl  salicylic  acid  (Aspirin).  4.  (c) : Liquid hydrogen (because of its low mass and high enthalpy  of combustion) and liquid oxygen (as it is a strong supporter of  combustion) are used as an excellent fuel for rockets.108  JEE MAIN CHAPTERWISE EXPLORER  3.  Drugs  which relieve or decrease pain are termed analgesics.  5.  (d) : Aspirin ­  2.  on normal temperature states.   PRINCIPLES RELATED TO  PRACTICAL CHEMISTRY Which of the following reagents may be used to distinguish  3.109  Principles Related to Practical Chemistry  CHAPTER  30  1.  (a)  The compound formed in the positive test for nitrogen with the  Lassaigne solution of an  organic compound is  (a)  Fe 4 [Fe(CN) 6 ] 3  (b)  Na 3[Fe(CN)    6]    (c)  Fe(CN) 3  (d)  Na 4[Fe(CN)  (2004)    5NOS.  (d)  2.  2.  (b)  3.  between  phenol  and  benzoic  acid?  (a)  Aqueous NaOH  (b)  Tollen’s  reagent  (c)  Molisch reagent (d)  Neutral FeCl 3  (2011)  Biuret  test is not given by  (a)  proteins  (b)  carbohydrates  (c)  polypeptide  (d)  urea  (2010)  Answer  Key  1.    .   (a) : 3Na 4 [Fe(CN) 6 ] + 4Fe 3+ ® Fe 4 [Fe(CN) 6 ] 3  + 12Na +  Prussian blue .  solution.  Benzoic acid gives buff coloured (pale dull yellow) precipitate  3.  (b)  :  Biuret  test  is  used  to  characterise  the  presence  of  —CONH group in a compound.110  1.  JEE MAIN CHAPTERWISE EXPLORER  (d) : Phenol gives violet colouration with neutral ferric chloride  2.  with  neutral ferric  chloride  solution. MATHEMATICS .  Z Í X and Y Ç Z  is empty is  (a)  2 5  (b)  5 3  (c)  5 2  (d)  3 5  (2012)  3. y) :  y  =  x + 1 and 0 <  x  < 2}  T = {(x.  let  f  (x)  =  x  +  5x  +  1.  B  and  C  are  three  sets  such  that  A Ç B  =  A Ç  C  and  A È  B  =  A È  C.  4.  then  (a)  A =  C  (c)  A Ç  B = f  (b)  B =  C  (d)  A =  B  (2009)  3  For  real  x.  (2011)  (a)  (– ¥.  x ³  –1.  (c)  Statement­1 is true. Relations and Functions  SETS.  Statement­2  : f  is bijection.  Then  (a)  R is an equivalence relation but S is not an equivalence  relation  (b)  neither  R nor  S is an  equivalence  relation  (c)  S is an equivalence relation but R is not an equivalence  relation  (d)  R and  S both  are  equivalence  relations  (2010)  Let  f  (x)  =  (x  +  1) 2  –  1.  Show that  f  is invertible  and its inverse is  (a)  g ( y ) =  y . y) :  x  – y is an integer}.  Statement­2 : B = {(x.  n. ¥)  5. Statement­2 is true; Statement­2 is  a  correct  explanation  for  Statement­1.  Which one  of the  following  is  true?  (a)  T is an equivalence  relation on  R  but  S  is  not  (b)  Neither S  nor  T  is an equivalence  relation  on  R  (c)  Both S  and  T are  equivalence relations on  R  (d)  S  is an equivalence  relation on  R  but  T  is  not  (2008)  Let  f  :  N ®  Y be a function defined as  f  (x) = 4x  + 3 where  Y = {y Π N :  y = 4x  + 3 for some x ΠN}. y)|x.  2. Statement­2 is false.  Statement­2 is  true.  Statement­1  : The set {x : f  (x) = f  –1 (x)} = {0. ÷ m. 5}. y) ΠR × R : x = ay for some rational  number a} is an equivalence  relation  on  R. n.  Let  R  be the set of  real  numbers.  (b)  Statement­1  is false. –1}.  7. ¥) – {0}  (d)  (0. y are real numbers and x = wy for some rational  number w};  ì æ m p ö S = í ç . 3.  (c)  Statement­1 is true.  q ¹  0  and  qm  =  pn}. Statement­2 is  true; Statement­2 is  not  a  correct  explanation  for  Statement­1. Statement­2 is true; Statement­2  is  a  correct  explanation  for  Statement­1.  The  number  of  subsets  of  A  ×  B  having  3  or  more elements is  (a)  211  (b)  256  (c)  220  (d)  219  (2013)  Let X = {1.  (a)  Statement­1  is true.  respectively. ¥)  The  domain  of the  function  f ( x ) = (2011)  Consider  the  following  relations:  R = {(x. 2.  then  (a)  (b)  (c)  (d)  f  is onto  R  but not one­one  f  is one­one  and onto  R  f  is  neither one­one  nor  onto  R  f  is one­one  but  not onto  R  (2009)  Let R be the real line.  1  is  | x | -  x (b)  (– ¥. 0)  (c)  (– ¥. p  and q  are integers such that  î è n q ø 10. RELATIONS  AND FUNCTIONS CHAPTER  1  1.1  Sets. Statement­2 is true;  Statement­2  is  not  a  correct explanation  for  Statement­1.3  4  (c)  g ( y ) = 4 +  y + 3  4  (b)  g ( y ) =  3 y + 4  3  (d)  g ( y ) =  y + 3  4  (2008)  . Z) that can be formed such that Y Í X.  (2009)  If  A. The number of different ordered pairs  (Y.  y) ΠR  ×  R  :  y  –  x is an integer} is an  equivalence  relation  on  R.  9. 4.  (b)  Statement­1 is false. Consider the following subsets of the  plane R ×  R  :  S = {(x.  Let A and B be two sets containing 2 elements and 4 elements  6.  (d)  Statement­1 is  true. Statement­2 is true.  (a)  Statement­1 is true.  Statement­2 is  false.  Statement­1 : A  = {(x .  (d)  Statement­1 is true.  8.   2.  not  symmetric  and  transitive.n .. ¥)  x 3  –  3x 2  +  3x  +  3  24.  The  function  f ( x) = log( x + x 2  + 1)  is  14.  5.  5}.  (2003)  é0. (4.    defined  by  f ( x ) = sin x .  (2006)  (a)  f (x) =  f (–x)  (b)  f (2 +  x) =  f (2  –  x)  (c)  f (x  +  2) =  f (x  –  2)  (d)  f (x) =  –f (–x).  Let R = {(3.  (2004)  13. 4).. (6.  p ö p  (b) 0.  2). (2.  è 1 -  x 2 ÷ø (c)  neither  an  even  nor  an  odd  function  then  f  is  both  one­one  and  onto  when  B  is  the  interval  (d)  an  even  function. 3) (6.  (2004)  12!  (c)  3!(4!) 3  12!  (d)  3!(4!) 4  .  –4]  x 3  +  6x 2  +  6  f ( x ) = 3  2  + log10 ( x3  . ¥)  (d)  (1.  2)  ( )  ( )  16.  3. (3. The number of ways to partition  17.  2). 3.  0) È  (1. 2).  A  function  is  matched  below  against  an  interval  where it is  supposed  to  be  increasing.  then  the  interval  of  S  is  letter  in  common}.  4}.  The  domain  of  the  function  f ( x )  = sin ( x . 9).  3)  (c)  [2.2  JEE MAIN CHAPTERWISE EXPLORER  11. The relation  .  Let R = {(1.  2. The  relation  R  is  12!  12!  (a)  not  symmetric  (b)  transitive  (a)  (4!) 3  (b)  (4!) 4  (c)  a  function  (d)  reflexive.  (2007)  18. ¥)  (c)  (–1. ¥)  2x 3  –  3x 2  –  12x  +  6  (d)  one­one  but  not  onto.  2.x ). (12.  R =  {(x.3 cos x + 1. 6) (9.  Which  of  the  following  pairs  is  (a)  onto  but  not  one­one  incorrectly  matched?  (b)  one­one  and  onto  both  Interval  Function  (c)  neither  one­one  nor  onto  (a)  [2.  3]. 9)..  (2005)  22.  symmetric  and  transitive  (b)  reflexive.  3]  (d)  [1. 1)} be a  relation on  S is  the  set  A  =  {1. .  Let  f  :  (–1. is  1 ù 4 - x æ (d)  ç -¥ .  1]  (b)  [–1. 3).  (3.  2. 2 û  (2005)  is  by  f (n ) = í 2  ú ï . 12).  3}  (d)  {1.  (b)  an  equivalence  relation  (2004)  (c)  reflexive  only  (d)  reflexive  and  transitive  only.  C  of    equal  size.  B. (3.  (c)  f (–x)  (d)  f (a) +  f (a  –  x). (3. 12}. 9.  Thus  A È B È C  =  S.3)  is  9 - x 2  (a)  reflexive  and  symmetric  only  (a)  [1.  (2003) . 3.  (2004)  relation  R  by  :  19. 0) È  (1.  (2005)  A Ç B = B Ç C = A Ç C = f.  symmetric  and  transitive  line  x  =  2.  Let W denote the  words in the English dictionary.  y) Π W ×  W |  the  words  x  and  y  have  at  least  one  is  onto. when  n  is even  î  2  15. when n  is odd  p p é p p ù .  1]  Then  R  is  (c)  [0.  3.  If  f  :  R ®  S.2 .  2.  (2003)  (b)  (–¥. 12).  2 (a)  ê 2 ÷ 23.12} is to be  partitioned into three  (a)  f (x)  (b)  –f (x)  sets  A. 3). 2.  then  (d)  reflexive.  A function f  from the set of natural numbers to integers defined  ë ø  ì n .  2]  (b)  [2.  The set S  = {1.  4. (2.  6}  12.  3 ú 3x 2  –  2x  +  1  (2005)  è û  (a)  (–1. 12).  A  real valued  function  f (x)  satisfies  the  functional  equation  f (x  – y) =  f (x) f (y)  –  f (a  –  x) f (a  +  y)  where  a is  a  given  constant and  f (0)  =  1.  f (2a  –  x)  is  equal  to  (b)  (1. 6. 2) È  (2. Define the  (c)  {1.  be  a  function  defined  by  (a)  an  odd  function  æ 2 x  ö (b)  a  periodic  function  f ( x ) = tan -1 ç . 2 (d)  ëê .  symmetric  and  not  transitive  20.  4.  The  range  of  the  function  F(x)  =  7  –  x P x  –  3  is  (a)  {1. .  3]  (d)  [–1..  (2004)  (a)  not  reflexive.  (a)  [0.  The graph of the  function y =  f (x) is symmetrical about  the  (c)  reflexive.1 .1  is  21. 2) È  (2.  3.  Domain  of  definition  of  the  function  (c)  (–¥.  4}  (b)  {1.  ï (c) - 2 . 6)} be a relation on the set A = {3.  1) ® B.   21. (d)  2  (2003)  26.  (b)  (b)  (c)  (c)  .  1]  (d)  [–9.  Which  one  is  not  periodic?  (a) |sin3x|  +  sin 2 x  (b)  cos x + cos 2  x 2  (c)  cos4x  +  tan  x  (d)  cos2x  +  sinx.  (d)  (b)  (d)  (d)  (c)  2.  10.  7.  18.  (a)  (a)  (b)  (a)  (a) 5.  14.  13.  then  å  f ( r ) is  r = 1  7( n + 1)  (a)  2  7n(n + 1)  (c)  2  ( )  (b)  7n(n  +  1)  x ù 28.  15.  The  period  of  sin 2q  is  (a) p 2  (b) p  (c) p 3  (d) p/2.  12.  (a)  [1.  22.  (2002)  and  f  (1)  =  7.  9]  (c)  [–9.  26.  19.  27.  28.  20.  The  domain  of sin -1 éê log 3  is  ë 3  úû  7 n .  23.  17.  (c)  (a)  (a)  (b)  6.  If f : R ®  R satisfies  f (x + y) = f (x)  + f (y).  8.  9.  11.  (2002)  (2002)  Answer  Key  1. y Π R  n 27.  9]  (b)  [–1. for all x.  –1].  25.  (d)  (b)  (c)  (c)  (b)  3.  (a)  (a)  (d)  (b)  (b)  4.  16.  24. Relations and Functions  25.3  Sets.   Y Ç Z = f  Þ  x Π B È  A Þ  x Π B  Number of ways  = 3 5 .  y) ΠZ  and (y. ÷ø Î S n b  k ΠN.  ÷ø Î S  since  mn =  mn.  4. z) ΠA Þ  (x. y) ΠA and (y.  x  – x  = 0 Π Z  n n symmetric – for (x.  5.  2.  ÷ Î S .  f ¢(x) = 3x 2  + 5 > 0 \  f  is strictly increasing  (c)  : We have (x.  The  number of subsets having atleast 3  elements  =  8 C 3  +  8 C 4  +  8 C 5  +  8 C 6  +  8 C 7  +  8 C 8  = 2 8  – ( 8 C 0  +  8 C 1  +  8 C 2 ) = 256 – 1 – 8 – 28 = 219  (b) : Let  x Π C  Suppose x Π A Þ  x Π A Ç  C Þ  x Π A Ç  B  (Q  A Ç  C  =  A Ç  B)  Thus  x Π B  Again suppose x Î/  A Þ  x Π C È  A (d) :  X = {1.e.(2)  \  (x. which gives (x + 1) 2  – 1 =  x The  function. 0 <  x  < 2} is not  æm pö æ p a ö reflexive for (x. then  y =  f  (x) Þ  4k  + 3 = 4x  + 3 Þ  x =  k Π N  This means S  is transitive. because it’s a polynomial function  Thus x must be –ve. Thus R is not  Hence f  is one­one  also.  x) Π Z  i. corresponding to any y Π co­domain there $ some x Π (a) :  f ( x ) = | x | -  x domain such that  f  (x) =  y. Thus  S is symmetric.  n = q and  q =  b .  symmetric  and  transitive. (x.  Thus corresponding to any y Î Y we have x Î N. The function  (b) : The solution  of f  (x) =  f  –1 (x)  are given  by  then is  onto. Z Í  X.  |x| –  x  > 0 Þ  |x| >  x  Also f is  continuous on R.4  1.3  y = 4x + 3 Þ  x =  \  x = –1. y) Π Z Þ  x  –  y Π Z æ m p ö Þ  y –  x Π Z i. z) Combining (1) and (2) we get B = C.  x) ΠS  would imply x =  x  + 1 Again.3  or  g ( y ) =  is the  inverse  of the  function.  w) Π Z. 0) Ï R for any w but (0.  10. being  both one­one  and  onto is  invertible.  x 1.  .  =  Þ  4x 1  + 3 = 4x 2  + 3 Þ  x 1  =  x 2  n b Thus f (x 1 ) = f (x 2)  Þ  x 1  = x 2 . x) Î R for w = 1 implying that R is reflexive.e.  For a ¹ 0. y) is in B but (y.(1)  Þ  z – x  = integer Similarly we  can show that B Í  C  .  (b) : The function is  f :  R ®  R  Also (x.  As a polynomial of odd degree has always at least one  real  1  root. x) Þ  Symmetric  Let y Π R  then y =  x 3  + 5x + 1 Hence A is a equivalence  relation  but  B  is  not.  q ÷ø Î S Þ qm = pn  transitive – for (x. about f  being  ONTO  or not. giving  ç . 0) is not in B.  ç .3  x .  nothing  can be  said  y . y) :  x –  y Π Z} is  æ m m ö As  çè . 0  \ f -1 ( x ) =  4  4  But as no co­domain  of f  is  specified. giving  p m ö æ x –  w Π Z i. 4. Hence f  is  ONTO..  3. y) :  y  = x  + 1. a) ΠR.  S is reflexive.  symmetric. x) Π A  is true Þ  Reflexive  f (x) =  x 3  + 5x + 1  As (x.  ÷ Î S  èn qø è q b ø  Þ  0 = 1 (impossible)  Thus  S  is not an equivalence  relation  means qm = pn and bp = aq. w) Π Z But this can be written  as np = mq. 5};  Y Í X  ..  Þ  x 3  + 5x  + 1 – y  = 0  (0.  Thus in both cases x Π C Þ x Π B  (a) : y – x = integer and z –  y =  integer Hence C Í  B  .  6.   x 2 Π N m p p a  m a  i.e.  \  T  is an equivalence  relation on  R. 3.  i.e.  T = {(x. (y.  reflexive – for (x.  reflexive. \  x Π(– ¥.  f (x) = x.  4  7. Hence the function is one­  æm aö one.. y) Π A Þ  (y.  Þ Transitive  8. ÷ Î S and ç .. (a..  (a) :  Let  f (x 1 ) =  f  (x 2 ). 2. for some  Thus çè .  Þ  x –  y ΠZ and y – w ΠZ.  9. Let y ΠY be a number of the form y = 4k + 3.e.  (a) : To be an equivalence relation the relation must be all –  Hence R  is not  an equivalence  relation.  0). x) Π Z  çè n .  è q n ø  S = {(x..  JEE MAIN CHAPTERWISE EXPLORER  (d) : A × B will have  2 ×  4 =  8 elements.  Þ  (x + 1) 2  – (x  + 1) =  0 Þ  (x + 1)x  =  0 y .  But  (x.  1)  we  have  2 – x  x  = 2  2 + x f ( x ) = tan -1 çæ 2 x 2  ÷ö è 1 .  b) Î R  . (9. 3) Π R  but (3.  (c):  If  y  =  f (x)  is  symmetrical  about  the  line  x  = a  then  Þ  (a.  2  ÷ 2 è 1 .  3) Ï  R 2 2  ú ëê û  \  relation is not symmetric which  means our choice (a) and  =  2  sin  (x  –  60°) (b) are out of court.  3}  is  required  range  \  Let x = DON.  q(x) ¹  0  =   tan –1  tan2q  =  2  tan –1 x  Þ - f (x  +  2)  =  f (x  –  2)  p p £  sin –1 (x  –  3) £  2 2 p p  Þ –  sin  £  x  –  3 £  sin  2 2 now  D f  of  p(x)  is  - p 2 x  ö p < tan -1 æç < .  (d)  :  For  (3.  (iii)  Let  (x. Relations and Functions  R  is  not  reflexive  as  (1.  5 letter in common. f (0) = 1 )  17.  (b) :  Given relation  R  such  that  R = {(x.  9) Π R. a) Î R which is hold i.  16. 3) Π R and (3. (a. Þ  R  is  symmetric.e.  (a) : Number of  ways  = 12 C4 ´ 8 C4 ´ 4 C4  =  12!  .  6)}.  x) has every  letter  common \  R  is  reflexive  \  from  (i). y) Π R and  (y.  4.  12.  (d):  Let  f (x)    =  g(x)  +  1  \  R is reflexive. 12).  14.  z =  SHE  then (x. R is reflexive. y  = NEST.  (9. 6).e.  é1 ù 3  where  g(x)  = 2 ê sin x cos x ú 13.  \  F(3)  =  4 P 0 .  symmetric  but  not transitive.  9).  F(4)  =  3 P 1 .x ù úû  ëê é 1 + x 2  + é ù 1  =  –log ê ú =  –  log ê 1  ê êë 1 + x 2  . y) Π W ×  W | the  word  x and  y  have  atleast  18. 4} and  (2.(i)  Þ  x 2  <  9  |x| < 3  i.(*)  f ( a ) = 0.  \  –2 £  2  sin  (x  –  60°) £  2  For reflexivity a Î R.  R  is not  symmetric  as  (2.e  (–¥..  z) Π R. 4) Π R and  (2.  (3. We need to prove reflexivity and transitivity.  (b) :  f (x) =  (By x = tan q )  p ( x )  (say)  q ( x )  then  Domain  of  f (x)  is  D f  p(x) Ç  D f  q(x). z) Ï  R. 3)(6.  (b..  –1 £  2  sin  (x  –  60°) +  1 £  3  Again.  (a)  :  R is a function as A = {1.  (d)  :  f (x) =  x 3  +  6x 2  +  6  f ¢( x ) = 3 x 2  + 12 x = 3 x ( x + 4)  –3  f ¢( x ) > 0 Þ x < -4 È x >  0  the  interval  x  <  –  4  i. 12). (6.  2.  (ii).  2) Ï  R  (4!) 3  R is not transitive as (1.  F(5)  =  2 P 2 But  R  is not transitive \  {1.  –3  <  x  <  3  . 1) Ï  R.tan  q ø  \  21. 19. x ) Î R " x Î W (iii)  x  –  3 £  7  –  x Þ  x £  5 \  (x. y) Π R  then  (y.x ø  2 tan q ö \ f (tan q ) = tan -1 æç 2  ÷ è 1 .  c) Î R 20.  (b)  : Given f (x –  y)  = f (x) f (y) – f (a  –  x) f (a  +  y)  let  x  =  0  =  y  f  (0) =  ( f  (0)) 2  –  ( f  (a)) 2  1  =  1  –  ( f (a)) 2 Þ  f (a)  =  0 \ f (2a  –  x)  =  f (a  –  (x  –  a))  = f (a)  f (x  –  a)  –  f (a  +  x  –  a)f (0)  By  using  (*)  = 0  –  f (x)(1)  =  –  f (x) ( Q  ..  one  letter in  common}  (i) \  7  –  x  >  0 Þ  x  <  7  where W denotes  set  of  words  in  English  dictionary  (ii)  x  –  3 ³  0 Þ  x ³  3  Clearly  ( x . (12.  (c)  :  F(x)  to  be  defined  for  x Π N.x úû  ë xù ú ú û .  – 4]  matched  correctly  and  after  checking  others  we  find  that  f (x)  =  3x 2  –  2x  +  1 Þ  f ¢(x)  >  0  for  x  >  1/3  which  is  not  given  in  the  choice.5  Sets.  (3. 2.  (c)  :  For  x Π (–1.  (3. x) Π R  as  x and  y  have  atleast one  x  =  3.. 9).x ø  2  15. 3..  (a)  :  f (x)  =  log é x 2  + 1  + êë xù úû 2  \  f (–x)  =  log é 1 + x . 12).. 1) Π R but (1.  1) Ï  R  11. 3) Π R  2  3  4 Þ  2 £  x £  4  Again  9  –  x 2  >  0 .(ii)  From  (i)  and (ii)  we  have \  2 £  x  <  3  is  correct  Domain  22.  for  transitivity  of  (a.  c) Π R  f (x  + a) =  f (x – a)  which is also true in R = {(3.   (a)  :  If  y  =  sin –1 a.1 - 1  Þ  1  =  2  2  –  +  f (3)  =  f (1) +  f (2)  \  x ¹  ±  2 cos  x  let  us  take.  (b)  :  If  n  is  odd.  f (x)  =  cos  x  Let  f (x  +  T) =  f (x) Þ  cos  T +  x =  cos  x Þ  T +  x =  2np  ±  x  which  gives  no  value  of  T  independent  of  x \  f (x)  cannot  be  periodic  Now say g(x) = cos 2 x + cos  x  which is sum of a periodic  and non periodic function and such function have no  period..  24.. d) Similarly we  can  say that cos 4x  + tan 2 x  and  cos2x  +  sin  x  are  periodic  function. Finally f (n)  is  one­one  onto  function.  n  is  even)  Let  f (2k 1 ) =  f (2k 2 ) 2 k1  2 k = ..6  JEE MAIN CHAPTERWISE EXPLORER  =  – f (x) Þ f (x)  +  f (–x)  =  0 Þ f (x)  is  an  odd  function.  then  –1 £  a £  1 é é æ x ö ù ù æxö \  –1 £  log 3 ç ÷ £  1  ê as y = sin -1 ê log 3 ç ÷ ú ú è 3 ø û û è 3 ø ë ë Þ  x 1  £  £  31 3 3  Þ  1 £ x £  9 .  y  =  1 \  f (1  +  1)  =  2f (1)  =  2(7) \  f (2)  =  2(7)  27.2  2  2  Þ k 1  =  k 2 Þ f (n)  is  one­one  if  n  is  even  Þ -  Again f (n) =  n - 1  2  (b)  Now  cos 2 x  is  periodic  with  period p  and  for  period  of  1  f ¢ (n)  =  >  0  [n Π N  if  n  is  odd  2  - 1  and  f ¢ (n)  =  <  0  [n Π N  if n  is  even  2 Now all such function which are either increasing or decreasing  in the stated domain are said to be onto function.C.M of their periods LCM (p.p )  æp ö \  L.  (c)  :  Let x  =  0  =  y \  f (0)  =  0  and  x  = 1.  Þ  k 1  =  k 2 26.C.  0) È  (1.  So..  let  n  =  2k  –  1  Let  n  f (2k 1  –  1)  =  f (2k 2  –  1) \  =  7(1 +  2  +  3 +  .1 .  cos  x  +  cos 2  x  is  non  periodic  function.x 2  \  D f  g(x)  =  R  –  {–2.e.  (c)  :  Let  g(x)  =  3  4 ..+  n)  =  7n(n + 1)  2  p  and period of sin 2 x is p  3 1 - cos 2 x (a)  Same as the period of |sin x| or  whose period  2  is p  Now period of |sin 3x| + sin 2 x is the L. ¥ )  25.  0) È  (1.1  2k . 23.  x  = 1.  2) È  (2.  (b) : Period of |sin 3x| is  Þ  f (n)  is  one­one  functions  if  n  is  odd  Again. ¥ ) –1  0  1 \  Domain of  f (x)  is  (–1.M  of  ç .    If  n  =  2k  (i.  +  f (n)  r  = 1  2 k 2  .  \  x 3  –  x  >  0  +  \  x Π (–1.  2}  f (x)  =  log 10 (x 3  –  x) x(x  + 1)  (x  –  1)  >  0  –  å  f (r ) =  f (1) +  f (2) +  f (3) +  .  (b)  :  Let  f (q)  =  sin 2q  =  |sin q|  Period  of  |sin q|  is p  28. p ÷ =  = p  HCF (3.  y  =  2 \  =  7 +  2(7)  =  3(7)  and  so  on.1) è 3 ø  (c.  y =  0 \  f (1 +  0) = f (1)  +  f (0) =  7  (given)  x =  1.  B)  equals  (a)  (1.  If  the  cube  roots of  unity  are 1. + ç z  + 6  ÷ is  z ø  è z ø z ø  è zø è è (a)  18  (a)  –p  (b)  54  (b) p/2  (c)  6  (d)  12.  1)  12.  –1 +  2w.  1 –  2w.  then  (1 - i )  (a)  x  =  2n. w.  æ 1 + z ö ÷ equals  then  arg çè 1 + z  ø p .  0)  (b)  (–1.  1)  (c)  (0.  (2005)  13.7  Complex Numbers  CHAPTER  COMPLEX NUMBERS 2  1. and Arg(z) – Arg(w)  = p/2.  where  n  is  any  positive  integer  (b)  x  =  4n  +  1. then the  maximum value of | Z  |  is equal to  Z (c)  2 +  2 (d)  3 + 1 5 + 1 (b)  2  (2009)  If  Z  (a)  6. then the value  If z is a complex  number of  unit modulus  and argument q.  3.  1 +  2w 2  (d)  –1.  1)  (d)  (1.  where  n  is  any  positive  integer  (c)  x  =  2n  +  1. where z is a complex number.  Then  arg  z  equals  (a)  3p/4  (b) p/2  (c) p/4  (d)  5p/4.  then (a)  2  (b)  –1  (c)  1  ( )  is  equal  to  x  y  + p q  2 2  ( p + q  ) (d)  –2.  2..  are  (a)  –1.  where  n  is  any  positive  integer  (d)  x  =  4n.  If z 2  + z + 1 = 0.  (2007)  8.  (2003)  .  (2012)  10.  (2004)  15.  then  arg  z 1  –  argz 2  is  equal  to  If w(¹  1)  is  a  cube  root  of  unity.  If z and w are two non­zero complex numbers such that |zw|  =  1. Then  (A.  If  w = z - (1/ 3) i (a)  a  circle  (b)  an  ellipse  (c)  a  parabola  (d)  a  straight line.  If |z + 4| £ 3.. then  zw  is  equal  to  (a)  –1  (b)  i  (c)  –i  (d)  1.  then  z  lies  on  11.  (2003)  17.  (2006)  2 2  2 2  æ 1 ö æ 2  1  ö æ 3 1 ö æ 6  1  ö of  ç z + ÷ + ç z  + 2  ÷ + ç z + 3 ÷ + .  –1. then the point represented by the  z - 1  complex number  z lies  (a)  either on the real axis or on a circle not passing through  the  origin. w  be  complex  numbers  such  that  z + i w =  0 and  zw  = p.  If  | z 2  –  1 |  =  | z | 2  +  1.  and  (1  + w) 7  =  A +  Bw. 1  + 2w.q  (b) q  (a)  (c) p – q  (d)  – q 2 (2013)  The conjugate of a complex number is  number is  1  (a)  i - 1  (b)  -1  i - 1  (c)  1  .  (c)  either on the real axis or on a circle passing through the  origin.  9.  (2004)  1/3  14. then the maximum value  of |z + 1| is  (a)  6  (b)  0  (c)  4  (d)  10.  (2005)  (2011)  The  number of complex  numbers  z  such  that  |z – 1| = |z + 1| = |z – i|  equals  (a)  0  (b)  1  (c)  2  (d) ¥  (2010)  5. Then that complex  i - 1  1  i + 1  (d)  -1  i + 1  (2008)  7.  1  –  2w 2 . w 2  then  the  roots  of  the  equation  (x  –  1) 3  +  8  =  0.  –1  (b)  –1.  (2006)  (c)  –p/2  (d)  0.  4  = 2.  (2005)  z  and  |w|  =  1.  Let  z.  If  z 1  and  z 2  are  two  non­zero  complex  numbers  such  that  |z 1  +  z 2 | =  |z 1 | +  |z 2 |.  (b)  on the imaginary axis.  then  z  lies  on  (a)  a  circle  (b)  the  imaginary  axis  (c)  the  real  axis  (d)  an  ellipse.  z 2  If z ¹  1 and  is real.  If 1 + i  = 1 ..  (d)  on a circle with  centre  at the origin.  (2004)  x  16.  If z  =  x  – iy  and  z  =  p  +  iq.  The  value  of  2k p 2 k p ö æ + i cos  ÷ is  11 11  ø k =1 è (b)  1  (c)  –1  10  (a)  i  å ç sin (d)  –i.  4.  where  n  is  any  positive  integer.  –1 –  2w 2  (c)  –1.   then  (c)  Re(z) >  3  (d)  Re(z) >  2.  (d)  5.  (b)  14.4 < z - 2 .  (c)  7.  (b)  .  z  20.  The  locus  of  the  centre  of  a  circle  which  touches  the  circle  (c)  a 2  =  4b  (d)  a 2  =  b.  z and w are two nonzero complex number such that  | z | = | w | numbers) will  be  and Arg  z  + Arg w = p then  z  equals  (a)  an  ellipse  (b)  a  hyperbola  (a) w  (b)  – w  (c) w  (d)  – w  (c)  a  circle  (d)  none  of  these  (2002)  (2002)  Answer  Key  1.8  JEE MAIN CHAPTERWISE EXPLORER  18.z1  =  a and  z .  its  solution  is  given  by  being complex further.  (a)  11.  (c)  6.  If  z .  (2003)  z . z 1  and z 2  form  (a)  Re(z) >  0  (b)  Re(z) <  0  an  equilateral  triangle.  (d)  20.  (a)  8.  (d)  12.  (2002)  (a)  a 2  =  2b  (b)  a 2  =  3b  21.  (c)  3. z 1 & z 2 are complex  19.  (d)  16.  9.  (d)  17.  (d)  18.  (b)  10.  15. assume that the origin.  Let z 1  and  z 2  be two  roots of  the equation z 2  +  az +  b  = 0.  (d)  (d)  (b)  (b) 4.  (b)  2.  21.  (d)  13  (a)  19.z 2  =  b externally (z. 2 £ r - z 2  is real.  4.. 0) and radius 3 units. 1).(ii)  6.2r .  1 1 -1  = = i - 1  ..5 £ r £ 1 +  5  (i) and (ii) gives  Þ  ( z .  3. These points are non­collinear.1) 2 +  y 2  Þ  – y(x 2  – y 2 ) + 2xy(x – 1) = 0 Þ  y(x 2  +  y 2  – 2x) = 0 Þ  y = 0  or x 2  + y 2  – 2x = 0 \  \  z lies on real axis  or on  a circle passing through origin.  If z  is a real number.  (a) : We have for any two complex numbers a  and b  ||a| – |b|| £  |a  – b|  5 ..£2 Þ r 2  .  1.4 £ 0  r The  corresponding  roots  are  2 ± 20  r= = 1 ±  5  2  Thus  1 .1 Let z = x + iy 2 4  £ 2  r 10  = å  æç sin 2 k p + i cos 2 k p ö÷ = . hence  r £ 1 +  5  .  2 nd  solution  :  We   resort  to definition  of  modulus.1)( z .z .1 i + 1  Im  (–1. then  r - 4  £ 2  r The  left inequality  gives  r 2  + 2r – 4 ³  0  The  corresponding  roots  are  Þ .  (b) : Note that  Set |Z| = r > 0. hence the  desired  number  is the  centre  of  the  (unique)  circle  passing  through  these three  non­collinear  points. 0) is 6 units. Þ zz .1 £ r £ 5 + 1  We have  z = ( z )  giving  z = Þ z + z = 0  (z  being purely imaginary)  Þ  1 + y 2  = y 2  – 2y  + 1 \  y = 0  r £ -1 -  5  So. 1 and  i.  Hence maximum distance of z from  (–1. the  greatest value  is  5 + 1..  (d) : (1 + w) 7  = (–w 2 ) 7  = –w 14  = –w 12 w 2  = –w 2  = 1 + w  = A + Bw  given  Hence on comparison.iy )  is real ( x .  9.  5.  z - 1  2  ( x . |z – 1| 2  = |z – i| 2 Thus. w 2 \  æ z + 1 ö + æ z 2 + 1 ö + .  (b) : 1 st  solution :  |z  – 1|  =  |z  + 1|  =  |z  –  i|  reads  that  the  distance  of  desired  complex number z is same from three points in the complex  plane –1. we  have (A.9  Complex Numbers  1 + z 1 + z  = = z  1 + z  1 +  1  z 2  Observe that  | z | = 1 = zz 1 + z  Then the  arg of the  number  is  just the argument of z  1 + z and that’s q. 0)  n 2 k p ö æ 2kp (d)  :  å ç sin n + 1 + i cos n + 1 ÷ ø k =1  è \  Þ  (x – 1) 2  + y 2  = x 2  + (y – 1) 2 4 4 4  Now | Z | £ ZÞ | Z  | £ 2  |z| Z | Z |  1  i - 1  (a) : z lies on or inside the circle with  centre (–4.  (c) : z ¹  1.  Again.  |z – 1| = |z + 1| Þ  |z – 1| 2  = |z + 1| 2 . 0)  (–4..1) = ( z + 1)( z + 1)  Thus x = 0  -2 ± 20  = ±  5  2  Thus  r ³ 5 . 0) is the desired  point.  B)  =  (1.(i)  But r  > 0.y + 2 xiy ) (( x .1 (As r > 0)  Again consider the  right inequality  4  r..1) .  8...z + 1 = zz + z + z + 1  Þ  1 + y 2  = (y – 1) 2  (because x = 0) or implies that  r ³ 5 .  z - 1  2.i .i 11  11 ø k =1  è (d)  :  z 2  +  z  +  1  =  0 Þ  z  = w. then  r= z 2  is real.  (d) :  z  = 7. æ z 6  + 1  ö ç ÷ ç ÷ z ÷ø çè è z2 ø z 6  ø  è 2  2 = 4  (w  + w 2 ) 2  +  2(w 3  + w 3 ) 2  = 4  (– 1) 2  +  2(2 2 )  =  4 +  8  =  12 2  O  Real  . (0.   =(x 2  +  y 2 ) +  1 Again  consider  x  =  –1.  2.  1  –  2w 2  are  roots  of  (x  –  1) 3  +  8 =  0  but  on the  other  hand  the other  roots  does  not  satisfies  the  equation  (x  –  1) 3  +  8  =  0. –1 cannot be the proper choice.  1 –  2w 2  (i  = 1.i ø  Þ  (x  + 1) 3  + 2(x +  1) 2 (w 2  – w)  –  4(x  +  1)  =  0  x  æ 2 i ö which cannot be expressed in the form of given equation (x –  Þ ç ÷ =  1 1) 3  +  8  =  0.  (d)  :  Method  I  :  Let  z 1  =  cosq 1  +  i  sinq 1 ..  z 1 z 2  =  b  Þ x 3  – 3x 2  + 3x – 1 + 8 = 0 Þ (x – 1) 3  + 8 = 0  which matched  Again  0.  13  (a) :  Þ  Þ Þ  Þ  3  z  3 z \ w = 3 z .  11.  (d) : Method (1)  : (By  making  the  equation  from the given  x y  roots)  +  =  –2(p 2  +  q 2 )  Let  us  consider  x  =  –1.  –1  –2w 2 Þ  2x  =  0  \  Required  equation  from  given  roots  is x  =  0 Þ  (x  +  1)(x  +  1  –  2w)(x  +  1  +  2w 2 )  =  0 Þ  z  lies  on imaginary  axis.  p  q  Adding  the  equations  of  (*)  we  get  12. Now  consider  the  roots  è 2  ø  x i  =  –1.  z 2  =  cosq 2  +  isinq 2 \  z 1  +  z 2  =  (cosq 1  +  cosq 2 )  +  i(sinq 1  +  sinq 2 )  Now |z 1  +  z 2 | =  |z 1 |  +  |z 2 |  Þ (cos q1 + cos q2 )2 + (sin q1 + sin q2 )2  = 1 + 1 Þ  2(1 +  cos(q 1  – q 2 ))  =  4  (by  squaring) Þ  cos(q 1  – q 2 )  =  1 Þ q 1  – q 2  =  0 (Q cos0° = 1)  Þ  Arg z 1  – Arg z 2  =  0.  Method  2  (by  taking  cross  checking)  As  (x  –  1) 3  +  8  =  0  . –1.  z 1 .i =  3  z Þ  x  –  iy  =  p 3  –  3pq 2  +  i(3p 2 q  –  q 3 ) Þ 3( x ) + i (3 y .  (c)  :  | zw | = 1 Þ | z || w | = 1 So | z | = – Product of roots  taken  all at  a  time  =  0  | w | Now sum of  roots x 1  + x 2  + x 3  p Again  Arg ( z ) – Arg (w)  = 2  =  –1 + 1 – 2w  +  1 –  2w 2  = 3  z  2  z  Product  of  roots  taken  two  at  a  time  \  = i = z i  from  (1) w w  =   –1 +  2w  – 1  +  2w 2  +  1  +  2(w 2  + w)  +  4w 3  =  3  z  1  Product  of  roots  taken  all  at  a  time  = z z i Þ  z w = = - i ..  For real axis y  = 0  Þ  (x + 1)[(x  + 1) 2  +  (x +  1)(2w 2  –  2w)  –  4w 3 ]  =  0 x  1  + i  æ ö Þ  (x  +  1)[(x + 1) 2  +  2(x  +  1)(w 2  – w)  –  4  ]  =  0 16.10  JEE MAIN CHAPTERWISE EXPLORER  10. 1  –  2w.i  3 z - i z + iw  =  0 z  =  -iw Þ  z  =  iw w  =  –iz \  arg  (–iz 2 )  = p arg  (–i) +  2arg (z)= p 2arg(z) = p  + p/2 =  3p/2  arg(z) =  3p/4  x  = 0 Imaginary axis 14.  (–2) 3  +  8  =  0 Þ 0  =  0  Similarly  for  1  –  2w  we  have  (x  –  1) 3  +  8  =  0 Þ  (1 – 2w  – 1) 3  +  8  =  0 Þ  (–2w) 3  + 8 = 0 Þ  –8  +  8 =  0  and  for  1  –  2w 2  we  have  (1  –  2w 2  –  1) 3  +  8  =  0 0  z 1  z 2 \  0 2  +  z 1 2  +  z 2 2  =  0z 1  +  z 1 z 2  +  z 2 0  =  0  Þ  z 1 2  +  z 2 2  =  z 1 z 2 (z 1  +  z 2 ) 2  =  3z 1 z 2  a 2  =  3b .  n Π I +  and  the  equation  with these  roots  is  given  by  3  2  x  – (sum of the roots)x  + x(Product of roots taken two at a time)  1  .(*)  and  x  =  –1  satisfies  (x –  1) 3  +  8  =  0  i.  –1  +  2w.  (b) :  |z  –  1|  =  |z|  +  1  Þ  Let  z  =  x  +  iy (x +  1) 3  =  0 which  does  not  match  with  the  given  equation  2  +  y 2  Þ  (x  –  1)  (x – 1) 3  + 8 = 0 so x = –1.  1  –  2w.  z 2  are  vertices  of  an  equilateral  triangle  with  given  equation.  \  w i 2  =    (–1)[(1  –  2w)(1  –  2w  )]  =  –7 3  2  \  Required equation  is  x  –  3x  +  3x  +  7  =  0 18.e.  z 2  are  roots  of  z 2  +  az  +  b  =  0 \  z 1  +  z 2  =  –a.  (1)  17.  –1 p q \  Required  equation  from  given  roots  is  2  2  (x +  1)(x  +  1)(x  +  1)  =  0  15.  –1..  3)  Þ  i x  =  1 Þ  i x  =  (i) 4n Þ x  =  4n.1) = 3( x + iy )  ( z = x + iy )  Þ  x  =  p 3  –  3pq 2  and  y  =  –(3p 2 q  –  q 3 )  x  y  Þ (3x) 2  + (3y – 1) 2  = 9(x 2  + y 2 ) Þ 6y – 1 = 0  which is straight  =  p 2  –  3q 2  and  =  –(3p 2  –  q 2 )  (*)  line.  (d) :  z 1/3  =  p  +  iq Þ  x  –  iy  =  (p  +  iq) 3 Þ 3 z . (b) : As  z 1 .  (d)  :  Given  w = Þ w 6 (–8)  +  (8) =  0 Þ  0  =  0 \  –  1..  (d)  :  Given  ç ÷ =  1 è 1 .   (c) : |z –  4| <  |z –  2|  or  |a  – 4  +  ib| <  |(a  –  2)  +  ib|  by  taking  z  =  a+ib Þ  (a  –  4) 2  +  b 2  <  (a  –  2) 2  +  b 2 Þ  –8a +  4a  <  –16  +  4 Þ  4a  >  12 Þ  a >  3 Þ  Re(z) >  3  21. . A hyperbola is the locus of a point which moves in such  a way that the difference of its distances from two fixed points  (foci)  is  always  constant.  (b) : Let  |z|  =  |w| = r \  z =  re ia  and w  =  re ib  where a  + b  = p (given)  - =  re  Now  Z e ia  =  re i(p  – b)  =  re ip ×  e –ib  =  – re –ib  =  – w  20.  which  represent  a  hyperbola  Since.11  Complex Numbers  19.  (b)  :  r  a  z 1  z 3  b  z 2  z 1 z 3  –  z 3  z 2  =  (a  +  r) –  (b  +  r)  =  a - b =  a  constant.   6. where I is 2 × 2 identity matrix. Statement­2 is true; Statement­2 is  (a)  Statement­1 is true. Statement­2 is true;  Statement­2  is  not  a  correct  explanation  for  Statement­1.  Statement­2: |adj A| = |A|  (d)  Statement­1 is true.1 ö ç 1 ÷ ç 1 ÷ (c)  ç ÷ (d)  ç ÷ è 0 ø  è -1ø  2 2  (b)  –  1  (c)  –  2  2  (d)  1  (2012)  Let A  and B  be  two  symmetric  matrices  of  order  3.  Statement­2 is  false.  b.12  JEE MAIN CHAPTERWISE EXPLORER  MATRICES  AND DETERMINANTS CHAPTER  3  1.  If P 3  = Q 3  and 2 5.1 ÷ (a)  ç ÷ è 0 ø  æ 1 ö ç .  (2010)  The number of 3 × 3 non‑singular matrices.  then u 1  + u 2  is  equal  to  ç ÷ ç ÷ è0ø è 0 ø  4.  The  system  has  (2013)  The number of values of k. with four entries  as  1  and  all  other  entries  as  0.  Let  a.  If  .  (2011)  not  a correct explanation for Statement­1  (b)  Statement­1 is true. has no solution. for which the system of equations  (k + 1)x + 8y = 4k.  Statement­2 is  true.1 ÷ (b)  ç ÷ è -1ø  æ . Define Tr(A) = sum  matrix A. is  (a)  1  3.  é1 a 3 ù If  P = ê1 3 3 ú is  the  adjoint  of  a  3  ×  3  matrix A  and  ê ú êë 2 4 4 úû  7.  then a  is  equal  to  (a)  11  2.  Statement‑1  : Tr(A)  =  0. kx  + (k +  3)y = 3k  –  1.  The  number  of  values  of k  for  which the  linear  equations  (c)  Statement­1 is false. Statement­2 is true.  (a)  (b)  (c)  (d)  (2013)  æ1 ö æ 0 ö that  Au1 = ç 0 ÷ and Au 2  = ç 1 ÷ .  (b)  5  (d)  at least 7  (2010)  Let A  be  a  2  ×  2  matrix  with  non‑zero  entries  and  let A 2  = I. Statement­2 is true;  Statement­2  is  not  a correct explanation for Statement­1  (c)  Statement­1 is true.  c  be  such  that  b(a  +  c) ¹  0.  is  (a)  less than 4  (c)  6  9.  P  Q  = Q  P.  (b)  5  (d)  4  (b)  2  (c)  3  (d)  infinite  æ 1 0 0 ö Let  A = ç 2 1 0 ÷ . Statement­2 is false.  Let  A  be  a  2  ×  2  matrix  (c)  Statement­1 is true.  (d)  Statement­1 is false. Statement­2 is true; Statement­2  is  a  kx  +  4y  + z  =  0  correct  explanation  for  Statement­1.  (c)  0  Consider  the  system  of  linear  equations x 1  +  2x 2  + x 3  =  3  2x 1  +  3x 2  + x 3  =  3  3x 1  +  5x 2  +  2x 3  =  1  Statement‑2  :  |A|  =  1.  Statement 2 : AB is symmetric matrix if matrix multiplication  of A  with B  is  commutative. Statement­2 is true; Statement­2 is  Statement­1: adj (adj  A) = A  a  correct explanation  for  Statement­1.1 ö æ .  (b)  Statement­1 is false.1 ö ç .  |A|  =  4.  (a)  Statement­1 is true.  2x +  2y  + z  =  0  (2009)  possess  a  non‑zero  solution  is  (a)  1  (b)  zero  (c)  3  (d)  2  (2011)  11. Statement­2 is false.  then  determinant  of  (P  + Q  )  is  equal  to  (a)  0  8.  (2010)  (a)  Statement­1 is true.  (b)  Statement­1 is true.  10. Statement­2 is true; Statement­2  is  a  correct  explanation  for  Statement­1.  Statement‑1 : A (BA )  and  (AB )A  are  symmetric  matrices. If u 1  and u 2  are  column matrices  such  ç ÷ è 3 2 1 ø  æ . Statement­2 is true.  infinite  number  of  solutions  exactly  3  solutions  a unique  solution  no  solution  of  diagonal  elements  of A  and  |A|  =  determinant  of  (2012)  Let P  and Q  be  3  ×  3  matrices  with P ¹ Q.  4x  + ky  +  2z  =  0 (d)  Statement­1 is true. a 3 . . y ¹ 0 then D is  1 1 1 +  y (a)  (b)  (c)  (d)  divisible  by x but  not  y  divisible  by y but  not  x  divisible  by  neither  x  nor  y  divisible by  both x  and  y.  If a  +  b  +  c  =  –  2  and  cx and  z = bx  + ay.  (c)  If det A ¹ ± 1. c be any real numbers. Let I be the 2 × 2  (2006)  identity matrix.  Let A be a 2 × 2 matrix with real entries. y = az  +  21.  Statement­2 : If A ¹ I and A ¹ – I.. then the value of the  log a n log a n +1 log a n + 2  log a n +3 log a n + 4 log a n + 5  . then A –1  exists and all its entries are non­  x + ay +  z = a  –  1. by the principle of mathematical  (c)  Statement­1 is true.1 0  ÷÷ .  Then  -b b + 1 b . then tr(A) ¹ 0. then A –1  exists but all its entries are not  (a)  0  (b)  1  (c)  2  (d)  3.  a...  z not all zero such that  x = cy + bz.13  Matrices and Determinants  æ1 2ö æ a  0 ö 18.  (2005)  Statement­1 :  If A ¹ I and A ¹ –I.  Let  A = ê0 a ê êë0 0 (a)  1/5  (b)  (2007)  aù A 2  | = 25. Assume that A 2  = I. Suppose that there are real  2  2  2  numbers x.  (a)  A  (b)  A +  I  (c)  I  – A  (d)  A  –  I.  then  which  of  the  following  will  be  always  true?  (a)  A =  B  (b)  AB  =  BA  (c)  either  A or  B  is  a  zero  matrix  (d)  either  A or  B  is  an  identity  matrix.  (2004) .  If  A  and  B  are  square  matrices  of  size  n  ×  n  such  that  A 2  –  B 2  =  (A –  B)(A  +  B).  The only correct statement  about  the  ç .1 0 0  ÷ è ø  matrix A  is  (a)  A –1  does  not  exist  (b)  A  =  (–1)I.  then  which  one  of  the  (a)  Statement­1  is true.  Statement­2 is  false  1 1  ë û  ë û  (b)  Statemen­1  is false.  integers  if a  is  (d)  If det A = ± 1.  é1 0 ù é1 0 ù 20.  If D = 1 1 + x 1 for x ¹ 0.  where  I  is a  unit  matrix  (c)  A  is  a  zero  matrix  (d)  A 2  =  I..then det A =–1. the sum of diagonal entries  2  19. then A –1  exists and all its entries are integers  (a)  either  –2  or  1  (b)  –2  (2008)  (c)  1  (d)  not  –2. are G. Denote by tr(A).1  1  15.  x  +  y  + az  = a  –  1 has  no  solutions. Statement­2 is true; Statement­2 is  induction  a correct explanation  for  Statement­1  (a)  A n  =  2 n  –  1  A  –  (n  –  1)I  (d)  Statement­1 is true.  log a n + 8  (c)  0  (d)  –2.  è 3 4ø è 0  b ø  (a)  there  cannot  exist  any  B  such  that  AB  =  BA  c c .  If | A  ú 5  úû  5  (c)  5 2  (d)  1. a 2 .  b Π N.  then |a |  equals  5a ú .  then  the  inverse  of  A  is  of A.  (2005)  a a +1 a -1 1 1 a +1 b +1 c .  then A  need not  exist  then  f (x)  is  a  polynomial  of  degree  (b)  If det A  = ± 1.1 c + 1  (-1)n + 2 a (-1)n +1 b (- 1) n c (b)  there exist more than one but finite number B’s such that  then  the  value  of  n  is  AB  =  BA  (a)  any even integer  (b)  any  odd  integer  (c)  there  exists  exactly  one  B  such  that  AB  =  BA  (c)  any integer  (d)  zero  (2009)  (d)  there  exist  infinitely  many  B’s  such  that  AB  =  BA.  Let  A be  a square  matrix all  of  whose  entries  are  integers.  is  (a)  2  log a n + 6 (a)  2  (b)  –1  log a n + 7 (b)  1  (c)  –2  (d)  5.  then a is  (2004)  determinant  25.  (2007)  17. y.  (2005)  13.  Statement­2 is  true  following holds for all n ³  1.P.  Let  A = ç 0 .1 ö ç 23.  é5 5 a 16.  Let  A = 2 1 -3  and  10( B ) = -5 0  a ÷ ..  Let a.  Let  A = ç ÷ and  B = ç ÷ .  (1 + a 2 ) x (1 + b 2 ) x 1 + c 2 x Then  which  one  of the  following  is  true?  –1  (a)  If det A = ± 1.  (2005)  necessarily integers  22.  12. Statement­2 is true; Statement­2 is  (b)  A n  =  nA  –  (n  –  1)I  (2008)  not a correct explanation for Statement­1 (c)  A n  =  2 n  –  1 A  +  (n –  1)I  (d)  A n  =  nA  +  (n  –  1)I.  If B is the  ç ÷ ç ÷ è 1 -2 3 ø  1 ø  è1 1 inverse  of  matrix  A.  The  system  of  equations a x  +  y  +  z  = a  –  1. . b.  (2004)  2 2 ö æ 4 æ 1 -1 1 ö ç ç ÷ 24.  If a 1 . a n .  If  A = ê ú and  I = ê0 1 ú ...  (2006)  æ 0 0 .1 b -1 c + 1 = 0. Then a 2  +  b 2  +  c 2  + 2abc is equal to  1 + a 2 x (1 + b 2 ) x (1 + c 2 ) x  (a)  1  (b)  2  (c)  –  1  (d)  0 (2008)  f ( x ) = (1 + a 2 ) x 1 + b 2 x (1 + c 2 ) x  14.1 + a .  If  A  –  A  +  I  =  0.   (a)  5.  (d)  29. then  (2003)  log l log m p  1  q  1  log n r  1  (a)  –1  equals  (b)  2  (c)  1  ­(d)  0.  (d)  21.  then  a b ax + b  bx + c  is  ax + b bx + c  0 is  equal  to  b c w  (c) w 2  (2003)  (d)  0.  11.14  JEE MAIN CHAPTERWISE EXPLORER  26.  7.  If  1.  (2002)  (a)  +ve  (c)  –ve  29.  (a)  19.  9.  (d)  16.  (d)  15.  (c)  20.  then  b a ë û  ë û  (a) a  =  a 2  +  b 2 .  8.  (a)  10.  (b)  28.  (d)  24.  (b)  (ac –  b 2 )(ax 2  +  2bx  +  c)  (d)  0.  (d)  (2002)  .  (a)  (d)  3.  Answer  Key  1. all positive.  (c)  (d)  6.  (a)  22.  (b)  (c)  4.  (c)  27. q th  and r th  term of a GP.  (c)  25.  (b)  26. b  =  a 2  –  b 2  (c) a  =  2ab. b  =  ab.  (b)  12.  (d)  13.  é a b ù 2  é a b ù 27.  If  A = ê ú and  A = ê b a ú .  (a)  (d)  2.  (a)  14. m.  1  n then  D = w w 2 n (a)  1  wn w2 n  w2 n  1  1  n (b) w  28.  (a)  17. w 2  are  the  cube  roots  of  unity. n are the p th .  (b)  23. w.  If l.  If  a  >  0  and  discriminant  of  ax 2  +  2bx  +  c  is  –ve. b  =  2ab  (b) a  =  a 2  +  b 2 . b  =  a 2  +  b 2  (d) a  =  a 2  +  b 2 .  (d) 18. \  k  = 2.  Number of non­singular matrices =  6 C 1  ×  6 C 1  =  36  We can also exhibit more  than 6 matrices to  pick  the  right  choice. 3  For k = 1.  P 2 Q  = Q 2 P.  3. Au = ê0 ú .  Hence. A(BA) = (AB)A by associativity. \ a  =  11  Remark :  det(adj  A)  =  (det  A) n  – 1.  4 k  2  we have  k 4 2 =  0  2 2 1  é1 0 0 ù é1 ù é0 ù ê 2 1 0 ú . 3x  + 6y = 8  which are parallel lines.1û  ë For no solution  of  AX  = B  a  necessary  condition is  det A = 0.  é a ù Let u1 = êb ú ê ú êë c úû  é1 ù Þ a = 1.  3x 1  + 5x 2  + 2x 3  = 1  As such the system is inconsistent and hence it has no solution.  which on expansion gives  k 2  – 6k  + 8 = 0 Þ  7.  (k – 2)(k – 4) = 0.  Thus P is symmetric. x + 4y  = 2  which is just a single  equation in  two variables. |P 2  +  Q 2 | = 0  5.  where  A  is  a  matrix  of  order  n.  Au 2  = ê1 ú ê ú 3 p     +  2q   +  r   =  0   Þ   r   = -2  êë0 úû  é 1ù é 0 ù é 1 ù u1 + u 2  = ê -2ú + ê 1ú = ê -1 ú ê ú ê ú ê ú êë 1úû êë -2 úû êë -1 úû  (d) :  For the  system to possess non­zero solution.  Also  (AB) T  = B T A T  = BA = AB  (Q A and B are commutative) Þ  6.  For k = 3. .  8. So no solution in this case.  (a) :  P 3  = Q 3 .e.  2a   +  b   =  0  Au 1 = ê 0 ú ê ú Þ b = -2. then det A will be non­zero  and all the elements of that term will be   1 each.  x + 4y  = 2  It has infinite  solutions.15  Matrices and Determinants  1. Au = ê1 ú A = (b) :  1 ê ú 2  ê ú ê ú êë3 2 1 úû êë0 úû êë0 úû  é p ù Let u2  = ê q ú ê ú êë r úû  AB is also symmetric. det(adj  A) =  (det A) 2 Þ  2a – 6 =  16 Þ  2a = 22.  3a   +  2b   +  c   =  0 Þ  c   =  1  êë 0 úû  é0 ù Þ p = 0.  (d) :  Let  A(BA) =  P  Then P T  = (ABA) T  = A T B T A T  (Transversal rule)  = ABA =  P  ék + 1 8 ù éxù é 4 k ù (a) : The equation  is ê úê ú= ê ú k k + 3û ëyû ë3k  .  4.  k + 1 8  Þ = 0  k k + 3  Þ  (k + 1)(k  + 3) – 8k = 0 Þ k 2  + 4k + 3 – 8k = 0 Þ  k 2  – 4k +  3 =  0 Þ (k – 1)(k – 3) = 0 \ k = 1.  2 p   +  q   =  1   Þ   q   =  1.  Again. 4  (d) : x 1  + 2x 2  +  x 3  = 3  2x 1  + 3x 2  +  x 3  = 3  3x 1  + 5x 2  + 2x 3  = 1  A quick observation tells us that the sum of first two equations  yields  (x 1  + 2x 2  +  x 3 ) + (2x 1  + 3x 2  +  x 3 ) = 3 + 3 Þ  3x 1  + 5x 2  + 2x 3  = 6  But this contradicts the third equation.  é1 a 3 ù (a) :  P = ê1 3 3 ú ê ú ëê 2 4 4 úû  Let P = 1(12 – 12) – a(4 –  6) + 3(4  – 6)  = 2a – 6  Also.  é a1 a2 A = ê b1 b2 ê (d) :  êë c1 c2 a 3 ù b 3 ú ú c3 úû  Let A = (a 1b    2c    3  + a 2 c 1 b 3  + a 3 b 1 c 2 ) – (a 1 c 2 b 3  + a 2 b 1 c 3  + a 3c    1b    2 )  If any of the terms be non­zero.  PQ 2  =  P 2 Q Þ  P(P 2  +  Q 2 ) = (Q 2  +  P 2 )Q Þ  P(P 2  +  Q 2 ) = (P 2  +  Q 2 )Q  P ¹ Q Þ  P 2  +  Q 2  is singular.  2. the  equation becomes  2x + 8y = 4. the  equation becomes  4x + 8y = 12. i.. 1  a  Þ D+ ( -1)n  b + 1 b .  (d) : D = 1 1 + x  1  = .1 -b  = 0  c é a 2  + bg b ( a + d ) ù A2  = ê ú 2  ú ëê g ( a + d )  d + bg û  é a 2  + bg b ( a + d )ù é1 A2  = I = ê ú=ê 2  ú ë0 ëê g ( a + d )  d + bg û  2  2  giving a  + bg  = 1 = d  + bg  and g(a + d) = b(a  + d) = 0  As A ¹ I.  (d) : Each entry of  A  is an integer.c .1  15.1 + c c -1 c +1 a a +1 a -1 a +1 b +1 c .  tr (A) = a  + d = 0   {a  = – d}  Statement­2 is false because  tr (A) = 0  13.1 = 0  14.1 = 0  c c .1 + ( -1) a .bg b g .bg = -1  Statement­1  is therefore  true. so the  cofactor of  every  c c + 1 c - 1  entry is an integer.1 c + 1  a - b a + 1 a .  which  gives  |adj  A|  =  |A|  Thus  statement­2  is  also  true. we have a  = – d  det A = 1 .1  n  .b  Þ  c -1  a  = 0  b a -1  c + 1 c - 1  c (Changing rows to columns)  Þ 1(1 – a 2 ) +  c(–  c – ab) – b(ca  + b) = 0 a a + 1 a . A ¹ – I.  (a) : A2  = êê0 a 5a úú êê0 a 5 a úú êë0 0 5 úû êë0 0 5  úû . C 3 ®  C 3  –  C 1 )  a+c 0  a + c  1 0 0  = -b + c 0  b + c  R 1 ® R 1  + R 3 .b b + 1 b . And then each entry of adjoint is integer.b 2 b .  But  statement­2  doesn’t  explain  statement­1.1 b .1  a -1 b -1 c .1  Þ a 2  +  b 2  +  c 2  + 2abc = 1  n  Þ D + ( -1) -b b + 1 b .  a 11.  10.1 = 0  c c .  (a) :  System of equations  x –  cy  – bz = 0  cx –  y +  az  = 0  bx +  ay  – z  = 0  has non trivial solution if the determinant of coefficient matrix  is  zero 1  .bg  0 ù 1 úû = -1 + bg .0)  =  xy  c -2 c + 1  1 0  y Expanding along 2nd column  Hence D is divisible by both x and y.1  1 1 1 +  y C 2 ® C 2  – C 3  c -2 c + 1  (Apply C 2 ®  C 2  –  C 1 .1  This means all entries in A –1  are  integers.1 c  .  (a) : Let  A = éê ú .  (Changing columns in cyclic order doesn’t change the deter­  Also det A  = ± 1 and we know that  minant) n  n  Þ  D + (–1)  D = 0 Þ  {1 + (–1)  }D =  0  1  A -1  =  ( adj A )  det  A a a + 1 a .  (a) :  We  have  adj(adj  A)  =  |A| n–2 A  Here  n  =  2.  JEE MAIN CHAPTERWISE EXPLORER  éa b ù (c) : Let  A = ê ú ë g d û  a bù 12.  (b) :  Þ a +1 a -1 -b b + 1 b .  é5 5a a ù é5 5 a a ù 16.1 .1 = 0  ( -1) n + 2 a ( -1)n +1 b (- 1) n c a + 1 b + 1 c .  which  gives  adj(adj  A)  =  A  The  statement­1  is  true.  Now D = -b b + 1 b .  R 2 ® R 2  + R 3  = 1 x 0 = 1( xy .  Again  |adj  A|  =  |A| n  –  1  Here  n  =  2.  D = 2{(a + c)(b + c) – (a + c) (c – b)}  = 2(a + c)2b  = 4b(a + c) ¹ 0  (By  hypothesis)  Now {1 + (–1) n }D = 0 Þ  1 + (–1) n  =  0  Which mean n = odd integer. We have  ë g d û  é a 2  + bg b (a + d) ù é1 0 ù A2  = ê ú=ê ú ú ë0 1 û ëê g (a + d )  d + bg û  Which gives a + d  = 0 and a 2  + bg  = 1  So we  have Tr(A) =  0  det A  = ad – bg = –a 2  – bg = – (a 2  + bg) = –1  Thus statement­1  is true but statement­2  is false.16  9.1 c + 1  1 1 1  a 2 a . 17  Matrices and Determinants  22.  (b)  :  For  no  solution  | A |  =  0  and  (adj  A)(B) ¹  0  é 25 25a + 5a 2 5a + 5a + 25 a 2 ù ê ú A2 = ê 0 a2 25a + 5 a 2  ú ê ú 0 0 25  ú ëê û  1  2  2  Given |A  | = 25, 625a  = 25 Þ  | a |=  .  5 a 1 1  Now  |  A  | =  0  Þ 1 a 1 = 0  1 1 a Þ a 3  –  3a  +  2  =  0 Þ (a –  1) 2  (a +  2)  =  0 Þ a =  1,  –2.  But  for a  =  1,    | A |  =  0  and  (adj  A)(B)  =  0 Þ  for a  =  1  there  exist  infintiely  many  solution.  Also  the  each  equation  becomes  x  +  y  +  z  =  0 again  for a  =  –2  |A|  = 0  but  (adj  A)(B) ¹  0  Þ $  no solution. 17.  (b) :  Give A 2  –  B 2  =  (A  +  B)(A  –  B) Þ  0 =  BA –  AB Þ  BA =  AB  æ 1 2ö æ a  0 ö 18.  (d)  :  A = ç and B = ç ÷ è 3 4ø è 0 b ÷ø  23.  (d)  :  (i) |A|  =  1 \  A –1  does  not  exist  is  wrong  statement  æ 1 2ö æ a 0ö æ a 2 b ö = Now  AB = ç è 3 4÷ø çè 0 b÷ø çè 3a 4b ÷ø  ...  (i)  and  æ a 0ö æ 1 2ö æ a 2 a ö BA = ç = è 0 b÷ø çè 3 4÷ø çè 3b 4b ÷ø  ...  (ii)  As  AB =  BA Þ 2a  =  2b Þ a  =  b \  æ a  0 ö B=ç = aI 2  Þ $ infinite  value  of  a  =  b Π N  è 0 a ÷ø  æ -1 0 0  ö ç (ii) (–1)  I  =  0 -1 0  ÷ ¹  A Þ  (b)  is  false  ç ÷ è 0 0 -1 ø (iii)  A  is  clearly  a  non  zero  matrix \  (c)  is  false  We  left  with  (d)  only.  24.  (d)  :  Given  A –1  =  B  =  10  A –1  =  10  B 19.  (c)  :  A 2  –  A  +  I  =  0 Þ  I  =  A  –  A ×  A  IA –1  =  AA –1  –  A(AA –1 )  ,  A –1  =  I  –  A.  æ 1 0 ö 20.  (b)  :  A = ç ÷ è 1 1 ø  æ1 0ö 3  æ 1 0 ö n  æ 1 0 ö \ A2 = ç ÷, A = ç ÷ so  A  = ç ÷ 2 1 3 1 è ø è ø è n 1 ø  æ n 0 ö æ n - 1 0  ö and  nA  –  (n  –  1)I  = ç ÷-ç ÷ n - 1 ø  è n nø è 0 æ 1 0 ö n  =ç ÷ = A  .  n 1  è ø  æ ç Þ ç è æ Þ  ç ç è æ Þ  ç ç è 4 -5 1 4 -5 1 2 2  ö 0  a ÷ =  10  A –1 . ÷ -2 3 ø  2 2  ö 0  a ÷ (A)  =  10I ÷ -2 3 ø  4 -5 1 2 2 öæ 1 0 a ÷ç 2 ÷ç -2 3 ø è 1 æ 10 0 0  ö =  ç 0 10 0  ÷ ç ÷ è 0 0 10 ø  -1 1  ö 1 -3  ÷ ÷ 1 1 ø  ...(*) Þ  –5  + a =  0  (equating  A 21  entry  both  sides  of  (*)) Þ a  =  5  a2 a3  a  = = ...  n  =  r  a1 a2 an  - 1  which  means  a n ,  a n  +  1 ,  a n  +  2 Π G.P. Þ  a n  +  1 2  =  a n  a n  +  2 Þ  2  log  a n  +  1  –  log  a n  –  log  a n  +  2  =  0...(i)  Similarly  2  log  a n  +  4  –  log  a n  +  3  –  log  a n  +  5 =  0 ...(ii)  and  2  log  a n  +  7  –  log  a n  +  6  –  log  a n  +  8  =  0 ...(iii)  Using C 1 ®  C 1  +  C 3  –  2C 2  we get D  =  0  26.  (d)  : As w  is  cube  root  of  unity \ w 3  = w 3n  =  1 25.  (c) :  21.  (c)  :  Applying  C 2 ®  C 2  +  C 3  +  C 1  f ( x ) = 1 + 2 x + x ( a 2 + b2 + c 2 )  1 + a2 x 1 (1 + c 2 ) x  (1 + a 2 ) x 1 (1 + c 2 ) x  (1 + a 2 ) x 1 1 + c 2 x Applying  R 1 ®  R 1  –  R 2 ,  R 2 ®  R 2  –  R 3  and  using  a 2  +  b 2  +  c 2  =  –  2  we  have  (1 + 2 x - 2 x ) 1 - x  0 0 0 0  x - 1  =  (1  –  x) 2  (1 + a 2 ) x 1 1 + c 2 x =  x 2  –  2x  +  1 \  degree  of  f (x)  is  2.  1  n w \  w2 n wn w2 n  w2 n  1  1  w n =  (w 3n  –  1)  – w n (w 2n  – w 2n )  + w 2n (w n  – w n )  =  0 18  JEE MAIN CHAPTERWISE EXPLORER  æ a b ö æ a b ö 27.  (a)  :  A = AA = ç ÷ç ÷ è b a ø è b a ø  2  æ a 2 + b2  2 ab  ö æ a b ö =ç ÷= ç 2 ab a 2 + b2  ÷ çè b a ÷ø è ø  \  28.  (c) :  C 1 ®  xC 1  +  C 2  –  C 3  = 1  x  0  0  b c ax 2  + 2bx + c bx +  c l  =  t p  =  AR p  –  1 Þ  log  l  =  log  A  +  (p  –  1)  log  R  Similarly,  log  m  =  log  A +  (q  –  1)  log  R  and  log  n  =  log  A  +  (r  –  1)  log  R \  ax + b  bx + c  0  (ax 2  + 2bx + c )  2  [b  x  +  bc  –  acx  –  bc]  x =  (b 2  –  ac)  (ax 2  +  2bx  +  c)  =  (+ve)  (–ve)  <  0  =  29.  (d)  :  Let  A  be  the  first  term  and  R  be  the  common  ratio  of  G.  P. log l log m log n p  1  q  1  r 1  = log A + ( p - 1) log R log A + ( q - 1) log R log A + ( r - 1) log R = log A - log R log A - log R log A - log R c 1 µ  c 3  =  0  +  0  =  0 p  1  q  1  r 1  p 1 p log R q 1 + q log R r 1 r log R p  1  q  1  r 1  c 1 µ  c 2  19  Quadratic Equations  CHAPTER  4  1.  2.  3.  4.  5.  QUADRATIC EQUATIONS The  real number  k for  which  the  equation  2x 3  + 3x + k = 0 has two distinct  real roots in  [0, 1]  (a)  lies between  2 and  3  (b)  lies between  –1 and  0  (c)  does not  exist  (d)  lies between  1 and  2  (2013)  If the equations x 2  + 2x + 3 = 0 and ax 2  + bx + c = 0, a, b,  c ΠR have a common  root, then  a  :  b :  c is  (a)  3 : 2 : 1  (b)  1 :  3 :  2  (c)  3 : 1 : 2  (d)  1 :  2 :  3  The equation  e sinx  –  e –sinx  –  4 =  0 has  (a)  exactly  one  real  root.  (b)  exactly  four  real  roots.  (c)  infinite  number of  real  roots.  (d)  no real  roots.  (2013)  (2012)  9.  If the roots of the quadratic equation x 2  + px + q = 0 are tan30°  and  tan15°,  respectively  then  the  value  of  2  +  q  –  p  is  (a)  2  (b)  3  (c)  0  (d)  1.  (2006)  10.  All  the  values  of  m  for  which  both  roots  of  the  equation  x 2  –  2mx +  m 2  –  1 =  0 are greater  than  –2 but less  than  4,  lie  in  the  interval  (a)  –2  <  m  <  0  (b)  m  >  3  (c)  –1  <  m  <  3  (d)  1 <  m  <  4.  (2006)  11.  The value of a  for which the  sum  of the  squares  of  the  roots  of the equation x 2  – (a – 2)x – a – 1 = 0 assume the least value  is  (a)  0  (b)  1  (c)  2  (d)  3.  (2005)  12.  If the roots of the equation x 2  – bx + c = 0 be two consecutive  integers,  then  b 2  –  4c  equals  (a)  3  (b)  –2  (c)  1  (d)  2.  (2005)  Let a, b  be  real  and  z  be  a  complex  number.  13.  If  both  the  roots  of  the  quadratic  equation  If  z 2  + az  + b  =  0  has  two  distinct  roots  on  the  line  x 2  –  2kx  +  k 2  +  k  – 5  =  0  Re z = 1, then it is  necessary that  are  less  than  5,  then  k  lies  in  the  interval  (a)  |b| = 1  (b) b Π(1, ¥)  (a)  (6, ¥)  (b)  (5,  6]  (c) b Π (0, 1)  (d) b Π(–1, 0)  (2011)  (c)  [4,  5]  (d)  (– ¥,  4).  b are the roots of the equation x 2  – x + 1 = 0, then If a and a 2009  + b 2009  =  (a)  –2  (b)  –1  (c)  1  (d)  2  (2010)  a n x n  (2005)  a n  –  1 x n  –  1  14.  If  the  equation  +  +  ...  +  a 1 x  =  0,  a 1 ¹  0,  n ³  2,  has  a  positive  root  x  = a,  then  the  equation  na n x n – 1  + (n – 1)a n – 1  x n – 2  + ... + a 1  = 0 has a positive root,  which  is  (a)  smaller than a  (b)  greater  than a  (c)  equal  to a  (d)  greater  than  or  equal  to a.  (2005)  15.  Let  two  numbers  have  arithmetic  mean  9  and  geometric  mean  4.  Then  these  numbers  are  the  roots  of  the  quadratic  equation  (a)  x 2  + 18x – 16 = 0  (b)  x 2  – 18x + 16 = 0  (c)  x 2  + 18x + 16 = 0  (d)  x 2  – 18x – 16 = 0.  (2004)  6.  If the roots of the equation  bx 2  + cx +  a = 0  be  imaginary,  then  for  all  real  values  of  x.  The  expression  3b 2 x 2  + 6bcx  + 2c 2  is  (a)  less than 4ab  (b)  greater than  – 4ab  (c)  less than – 4ab  (d)  greater than 4ab  (2009)  7.  The quadratic equations x 2  – 6x + a = 0 and x 2  – cx + 6 = 0  have one root in common. The other roots of the first and second  equations are integers in the ratio 4 : 3. Then the common root  is  2  (a)  2  (b)  1  (c)  4  (d)  3  (2008)  16.  If (1 – p)  is a  root  of  quadratic equation  x  + px + (1  – p) = 0  then  its  roots  are  If  the  difference  between  the  roots  of  the  equation  (a)  0,  –1  (b)  –1,  1  (c)  0,  1  (d)  –1,  2.  (2004)  x 2  +  ax  +  1  =  0  is  less  than  5 ,  then  the  set  of  possible  2  17.  If  one  root of  the  equation  x  +  px  +  12  = 0  is  4,  while  the  values of a  is  equation x 2  +  px +  q  = 0 has  equal  roots, then  the  value  of  (a)  (3, ¥ )  (b)  (– ¥ , –3)  q  is  (c)  (–3, 3)  (d)  (–3, ¥ ).  (2007)  (a)  3  (b)  12  (c)  49/4  (d)  4.  (2004)  8.  20  JEE MAIN CHAPTERWISE EXPLORER  (a)  3x 2  –  25x  + 3  =  0  (b)  x 2  +  5x  – 3  =  0  18.  The  value  of a for  which  one  root  of  the  quadratic equation  2  2  (c)  x 2  –  5x  + 3  =  0  (d)  3x 2  –  19x  +  3 =  0.  (2002)  (a  – 5a + 3)x  +  (3a –  1)x +  2 =  0 is twice as large as the  other  is  22.  Difference  between  the  corresponding  roots  of  (a)  –2/3  (b)  1/3  (c)  –1/3  (d)  2/3.  (2003)  x 2  + ax + b = 0 and x 2  +  bx + a = 0 is same and a ¹  b, then  19.  If  the  sum  of  the  roots  of  the  quadratic  equation  (a)  a  +  b  +  4  =  0  (b)  a  +  b  –  4  =  0  ax 2  + bx + c = 0 is equal to the  sum of  the squares of their  (c)  a –  b  –  4  =  0  (d)  a –  b  +  4  =  0.  (2002)  a , b  and  c  2  23.  Product  of  real  roots  of  the  equation  x  + | x |  +  9  =  0  reciprocals,  then  c a b  are  in  (a)  is  always  positive  (b)  is  always  negative  (a)  geometric  progression  (c)  does  not  exist  (d)  none  of  these.  (2002)  (b)  harmonic  progression  2  (c)  arithmetic­geometric  progression  24.  If p and q are the roots of the equation x  + px + q = 0, then  (d)  arithmetic  progression.  (2003)  (a)  p  =  1,  q  =  –2  (b)  p  =  0,  q  =  1  (c)  p  = –2,  q  =  0  (d)  p  =  –2,  q  =  1.  (2002)  20.  The  number  of  real  solutions  of  the  equation  2  x  –  3|x|  + 2  = 0  is  25.  If a, b, c are distinct +ve real numbers and a 2  +  b 2  +  c 2  = 1  (a)  4  (b)  1  (c)  3  (d)  2.  (2003)  then ab  +  bc  +  ca  is  (a)  less than  1  (c)  greater  than  1  21.  If a ¹ b  but a 2  =  5a  –  3  and b 2  =  5b  –  3  then  the  equation  whose  roots  are a/b  and b/a  is  (b)  equal  to  1  (d)  any  real  no.  Answer  Key  1.  (c)  2.  (d)  7.  (a)  8.  (c)  13.  (d)  19.  (b)  25.  (a) 14.  (a)  20.  (a)  3.  9.  15.  21.  (d)  (b)  (b)  (d)  4.  10.  16.  22.  (b)  (c)  (a)  (a)  5.  11.  17.  23.  (b)  (b)  (c)  (c)  6.  12.  18.  24  (b)  (c)  (d)  (a)  (2002)  21  Quadratic Equations  7.  (a) :  Let a  and 4b  be the  root of  1.  (c) : Let  f  (x) = 2x 3  + 3x + k  ,  f ¢(x) = 6x 2  + 3  > 0  Thus f is strictly increasing. Hence it has atmost one real root.  x 2  – 6x  +  a = 0  But a polynomial equation of odd degree has atleast one root.  and a and 3b  be those of the equation  x 2  –  cx  + 6  =  0  Thus the equation has exactly one root. Then the two distinct  From  the  relation between roots  and  coefficients roots, in any interval whatsoever is an impossibility. No such  a + 4b = 6 and 4ab =  a (c) exists.  2  a + 3b =  c  and 3ab  = 6  2.  (d) : In the equation x  + 2x + 3 = 0, both the roots are imaginary.  we obtain ab  = 2 giving  a = 8  a b c  Since a,  b, c Π R,  we  have  = =  The first equation is  x 2  – 6x + 8 = 0 Þ x  = 2,  4  1 2 3 For a  = 2, 4b  = 4 Þ  3b  = 3  Hence a : b : c : :  1 :  2 :  3  For a = 4, 4b  = 2 Þ  3b = 3/2 (not an integer)  sinx  –sinx  3.  (d) :e  – e  –  4 =  0 So the common root is a  = 2.  Þ  (e sinx ) 2  – 4e sinx  – 1 = 0 Þ  t 2  – 4t –  1 =  0  4 ± 16 + 4  Þ t  = = 2 ±  5  2 8.  i.e., e sin x  = 2 + 5 or 14 2 4244 -  53  (neglected) | a - b | = (a + b )2  - 4ab  ,  | a - b | = a 2  - 4  - ve  Since,  | a - b | < 5 Þ  a 2  - 4 <  5 Þ  a 2  – 4 < 5 Þ a 2  < 9 Þ – 3 < a <  3.  sin x = ln (2 + 5) > 1 \  No  real  roots.  4.  (b) : Let  roots be  1 +  ai, 1  + bi,  then we  have,  (a Π R)  (1 + ai) + (1 +  bi) =  – a Þ  2 +  (a + b)i  = – a  (1 + ai)(1 +  bi) = b  Comparing we have, a  =  – 2  and a  = – b  Now (1 + ai)(1 –  ai)  = b Þ  1 +  a 2  = b Þ b  = 1  +  a 2  As a 2 ³  0 we  have b Π (1, ¥)  1 ± i  3  5.  (b) : We have x 2  –  x + 1 =  0 giving  x =  .  2 Identifying these  roots  as w  and w 2 ,  we have a  = w, b  = w 2 . We can also take the other way round  that  would not  affect the  result.  Now a 2009  + b 2009  = w 2009  + w 4018  = w 3k + 2  + w 3m + 1  (k, m Π N)  = w 2  + w = –1.  ( . . . w 3k  =  1)  6.  (b) : The  roots  of  bx 2  +  cx  +  a  =  0  are  imaginary  means  c 2  – 4ab  < 0 Þ  c 2  <  4ab  Again the  coeff.  of  x 2  in  (c) : x 2  + ax + 1 = 0  Let roots be a  and b, then a  + b  =–a and ab  = 1  9.  (b)  : a  =  tan  30°, b  =  tan  15°  are  roots  of  the  equation  x 2  +  px  +  q  =  0 \  tan a  + tan b  =  –  p  and tan a  ∙ tan b  =  q  using tan a  +  tan b  =  tan  (a  + b)  (1  –  tan a  tan b) Þ  –  p  =  1  –  q Þ  q  –  p  =  1 Þ  2  +  q  –  p  =  3  10.  (c)  :  Let a, b  are  roots  of  the  equation  (x 2  –  2mx  +  m 2 )  =  1  –3  –1  3  5 Þ  x  =  m  ±  1 =  m  + 1,  m  –  1  Now  –2  <  m  + 1  <  4  ......  (i)  and  –2  <  m  –  1  <  4  .......  (ii)  ......(A)  ì Þ  -3 < m < 3 í and 1 < m < 5 .......(B)  î By  (A)  &  (B) we  get  –1  <  m  <  3  as shown  by  the  number  line.  3b 2 x 2  +  6bcx  +  2c 2  is  +ve,  so  the  minimum  value  of  the  11.  (b)  :  Let  f (a)  = a 2  + b 2  =  (a + b) 2  –  2ab  expression  =  (a  –  2) 2  +  2(a  +  1) 2 2 2 2 2 2  36b c - 4(3b )(2c ) 12 b c  \  f ¢(a)  =  2(a  –  2)  +  2  == = - c 2  For Maxima  |  Minima  f ¢(a) =  0 (3b 2 ) 12b 2  Þ  2[a  –  2  +  1]  =  0 Þ  a  =  1  2  2  As c  < 4ab  we have –c  > –  4ab  Again f ¢¢(a) =  2,  Thus the  minimum value is  – 4ab.  f ¢¢(1)  = 2  > 0 Þ at  a  = 1,  f (a)  will  be  least. 22  JEE MAIN CHAPTERWISE EXPLORER  By  (i)  and  (ii)  we  have  12.  (c)  :  Let a, a  +  1  are  consecutive  integer \  (x  + a)(x  + a  +  1)  =  x 2  –  bx  +  c  Comparing  both  sides  we  get Þ  –b  =  2a  +  1  c  = a 2  + a \  b 2  –  4c  =  (2a  +  1) 2  –  4(a 2  + a)  =  1.  9 a 2  13.  (d)  :  Given  x 2  –  2kx  +  k 2  +  k  –  5  =  0  Roots  are  less  than  5 Þ D ³  0 Þ  (–2k) 2 ³  4(k 2  +  k  –  5) Þ  k £ 5  ...(A)  Again  f (5) > 0 Þ  25 –  10k  +  k 2  +  k  –  5  >  0 Þ  k 2  –  9k  +  20    >  0 Þ  (k  –  4)(k  –  5)    >  0 Þ  k  <  4 È  k  >  5  ...(B)  ...(C)  Also  sum of roots  < 5  Þ  k  <  5  2 from  (A),  (B),  (C)  we  have  k Π (–¥,  4) as  the  choice  gives  number  k  <  5 is  (d).  14.  (a)  :  If  possible  say  f ( x ) = a0 x n + a1 x n -1  + ... + an x a 2  - 5 a + 3  ´ 2  2a 2  ( a 2 - 5a + 3) 2  2  2 Þ  9(a  –  5a  +  3)  =  (1  –  3a)  2  Þ a  =  3  19.  (b)  :  Given a  + b  =  1 (1 - 3a ) 2 =  + 1 = ( a + b ) 2  - 2 ab a 2 b2 a 2b 2  Þ 2a 2 c  =  bc 2  +  ab 2 2a  c b  Þ c , a , b  Π A.P =  + Þ  a b c b  a c Þ  reciprocals  are  in  H.P  20.  (a) :  Given  x 2  –  3|x|  +  2  =  0  If  x ³  0  i.e.  |x| =  x \  The  given  equation  can  be  written  as  x 2  –  3x  + 2  =  0 \ f (0) =  0  (x  – 1)  (x  –  2)  =  0 x  =  1,  2  Similarly for  x  <  0,  x 2  –  3|x|  +  2  =  0 Þ  x 2  +  3x  + 2  =  0 Þ  Now  f ( a ) = 0 (\  x  = a  is  root  of  given  equation) Þ  n -1 n - 2  + ... + a1  = 0  has at least  \ f ¢( x ) = nan x + ( n - 1) an -1 x one  root  in  ]0, a [ Þ  na n  x n–1  +  (n  –  1)  a n–1  x n–2  +  ....  +  a 1  =  0  has  a  +ve root  smaller  than a.  Þ  Hence 1,  –1, 2,  –2  are  four  solutions  of  the  given  equation.  a b and which  21.  (d) : We need the equation whose roots are b  a  are reciprocal of each other, which means product of roots is a b = 1. In our choice (a) and (d) have product of roots 1,  b a  so choices (b) and (d) are out of court. In the problem choice,  None  of  these  is  not  given.  If  out  of  four  choices  only  one  choice  satisfies that  product of root is 1 then  you  select that  choice for correct answer. Now for proper choice we proceed  as, a ¹ b,  but a 2  =  5a  –  3  and b 2  =  5b  –  3,  Changing a, b  by  x \ a, b  are  roots  of  x 2  –  5x  +  3  =  0 Þ a  + b  = 5, ab  =  3  a b a 2 + b 2  19  a b + = and product  .  = 1 now,  S = =  ab  3 b a  b a \  Required  equation,  x 2  –  (sum  of  roots)  x  +  product  of  roots  =  0 19  Þ  x 2  –  x  +  1  =  0 3  2  Þ  3x  –  19x  +  3  =  0  is  correct  answer.  15.  (b)  :  Let  the  two  number  be a, b a + b  =  9  and ab  =  4 \  2 \  Required  equation  2  x 2  –  2(Average  value  of a, b)x  +  G. M  =  0  x 2  –  2(9)x  +  16  =  0  16.  (a)  : As  1  –  p  is  root  of  x 2  +  px  +  1  –  p  =  0 Þ  (1  –  p) 2  +  p(1  –  p)  +  (1  –  p)    =  0  (1  –  p)  [1  –  p  +  p  +  1]  =  0 Þ  p =  1 \  Given  equation  becomes  x 2  +  x  =  0 Þ  x  =  0,  –  1  17.  (c) : As  x 2  +  px  +  q  =  0  has  equal  roots \  p 2  = 4q  and  one  root  of  x 2  +  px  +  12  =  0  is  4. \  16 +  4p  +  12  =  0 \  p  =  –7 49  \  p 2  =  4q Þ  q  =  4  18.  (d)  :  Let a,  2a  are  roots  of  the  given  equation \  sum  of  the  roots 1 - 3 a  a  +  2a  =  3a  =  2  a - 5a + 3  and  product  of  roots 2  a(2a) =  2a 2  =  2  a - 5a + 3  x  =  –1,  –2  ...(i)  ...(ii)  22.  (a)  :  Let a, b  are  roots  of  x 2  +  bx  +  a  =  0 \ a  + b  =  –b  and ab  =  a  again let g, d  are  roots  of  x 2  +  ax  +  b  =  0 \ g  + d  =  –a  and gd  =  b  Now  given 23 Quadratic Equations  Now  if  q  =  0  then  p  =  0 Þ  p  =  q  If  p  =  1,  then  p  +  q  =  –  p  q  =  –  2p  q  =  –  2(1)  q  =  –  2 Þ  p  =  1  and  q  =  –  2  a  – b = g  – d Þ  (a  – b) 2  =  (g  – d) 2 Þ  (a  + b) 2  –  4ab  =  (g  + d) 2  –  4gd Þ  b 2  –  4a  =  a 2  –  4b Þ  b 2  –  a 2  =  –4(b  –  a) Þ  (b  –  a)  (b +  a  +  4)  =  0 Þ  b  +  a  +  4 =  0  as  (a ¹  b)  23.  (c) :  x 2  +  |x| +  9  =  0 Þ  |x| 2  +  |x| +  9  =  0 Þ \  no  real  roots  (³  D  <  0)  24  (a)  :  Given  S  =  p  +  q  =  –  p  and  product  pq  = q Þ  q(p  –  1)  =  0 Þ  q  =  0,  p  =  1  25.  (a) : In such type of problem if sum of the squares of number  is known and we needed product of numbers taken two  at a  time or needed range of the product of numbers taken two at  a  time. We  start  square  of  the  sum  of  the  numbers  like  (a +  b  +  c) 2  =  a 2  +  b 2  +  c 2  +  2(ab  +  bc  +  ca) Þ  2(ab  +  bc  +  ca) =  (a  +  b  +  c) 2  –  (a 2  +  b 2  + c 2 ) ( a + b + c) 2  - 1  Þ  ab  +  bc  +  ca  =  <  1 2    (b)  Statement­1 is false.  (2010)  10.  Let T n be the number of all possible triangles formed by joining  6.  (d)  Statement­1 is false.  Statement­1  :  The  number  of  different  ways  the  child  can  buy the six ice­creams is  10 C 5 .  3.24  JEE MAIN CHAPTERWISE EXPLORER  PERMUTATIONS  AND COMBINATIONS CHAPTER  5  1.  20 C  –  20 C  +  20 C  –  20 C  + .. then  the  value of  n is  (a)  5  (b)  10  (c)  8  (d)  7  (2013)  2. If a voter votes for at least  has 9 distinct blue balls.  Statement­2 is  false. then the number of ways in which he can vote  out at random and then transferred to the other..  statement­2  is  true; statement­2 is  a  0  1  2  3  10  correct  explanation  of  statement­1. the number of ways in which one or more balls can  be selected from 10  white.  (a)  0  (b)  20 C 10  (c)  – 20 C 10  (d)  1  20 C 10 .1) 10C j .  (2011)  4. The number  is  of ways in which  this can be  done is  (a)  5040  (b)  6210  (c)  385  (d)  1110.  (a)  3  (b)  36  (c)  66  (d)  108  (2006)  (2010)  . 9 green  and 7  black balls is  (a)  630  (b)  879  (c)  880  (d)  629  (2012)  7.  At an election.  The  sum of  the  series  Statement­2 : S 1  = 90 × 2 8  and S 2  = 10  × 2 8 . Statement­2 is true; Statement­2 is  a  correct explanation  for  Statement­1..  not  greater  than  the  number  to  be  elected.. – . a voter may vote for any number of candidates. Then  the number of such arrangements is  (a)  at least 500 but less than 750  (b)  at least 750 but less than 1000  (c)  at least 1000  (d)  less than 500  (2009)  In a shop there are five types of ice­creams available.  There  are  10  There are two urns. 4 novels  and 1 dictionary are to be selected and arranged in a row on  a shelf so that the dictionary is always in the middle. Statement­2 is true  (c)  Statement­1 is true.. A child  buys six ice­creams.  (2007)  (c)  Statement­1 is true.  Statement­2  :  The  number  of  different  ways  the  child  can  buy  the  six  ice­creams  is  equal  to  the  number  of  different  ways of arranging 6 A’s and 4  B’s in a row.  (a)  Statement­1 is true.  å  j 10 C j  j =1  8.  (b)  Statement­1 is true.  vertices  of  an  n­sided  regular  polygon. Urn A has 3 distinct red balls and urn B  candidates and 4 are to be elected. Statement­2 is true; Statement­2 is  not  a correct explanation for Statement­1  (2008)  How  many  different words  can  be  formed by  jumbling  the  letters in the word MISSISSIPPI in which no two S are adjacent?  (a)  7 ∙  6 C 4  ∙  8 C 4  (b)  8 ∙  6 C 4  ∙  7 C 4  j =1  (c)  6 ∙ 7 ∙  8 C 4  (d)  6 ∙ 8 ∙  7 C 4  (2008)  Statement­1 : S 3  = 55  × 2 9 . statement­2 is true; statement­2 is not  2  a correct  explanation  for  statement­1.  Statement­2 : The number of ways of choosing any 3 places  from  9 different  places  is  9 C 3 . From each urn two balls are taken  one candidate.  From 6 different novels and 3 different dictionaries. statement­2  is true.  (a)  Statement­1 is true..  (c)  Statement­1 is true.  If T n  +  1  – T n  = 10.  Let  S1 = 10 å 10  j ( j . statement­2  is false.  å  j 2 10 C j  .  Statement­2 is  true. Statement­2 is false  (b)  Statemen­1 is false. Statement­2 is true; Statement­2 is  a correct explanation for Statement­1  (d)  Statement­1 is true.  Statement­1  :  The  number  of  ways  of  distributing  10 identical balls in 4 distinct boxes such that no box is empty  is  9 C 3 .  9.  (d)  Statement­1 is true..  S 2  = j =1 10  and  S3  = 5..  Assuming  the  balls  to  be  identical  except  for  difference  in  colours.. +  20 C  is  (a)  Statement­1 is  true. Statement­2 is true; Statement­2 is  not  a  correct  explanation  for  Statement­1.   If  C r  denotes the number of combinations of n things  taken  (a)  602  (b)  603  (c)  600  (d)  601. Total number of such  numbers  are  14.  by (2002)  (a)  30  (b)  5 ! ´  4 ! (c)  7 ! ´  5 ! (d)  6 ! ´  5!.  (2005)  r at a time.  (a)  5.  (b)  (a)  (b)  (b)  3.  The sum of integers from 1 to 100 that  are divisible  by  2 or  15.  (c)  (d)  (d)  (d) 4.  2.  3.  13.  6  50 56 .  13.  If the letters of the word SACHIN are arranged in all possible  (a)  196  (b)  280  (c)  346  (d)  140.  5.25  Permutations and Combinations  11.  The number of ways  in which 6 men and 5 women can dine  5  is  at  a round table if  no two women  are  to sit together is given  (a)  3000  (b)  3050  (c)  3600  (d)  3250.  (a)  18.  15.  (2002)  (2004)  20.  4  repetition  allowed  is  (2005)  (a)  125  (b)  105  (c)  128  (d)  625.  (a)  6.  How many ways are  there  to  arrange  the  letters  in the word  (2002)  GARDEN  with  the  vowels  in  alphabetical  order?  (a)  360  (b)  240  (c)  120  (d)  480.  7.  1.  The  value  of  C4 + å  (2003)  r =1  18.  1.  3.  (c)  .  Total  number  of  four  digit  odd  numbers  that  can  be  formed  using  0. (2003)  21.  2.  A student is to answer 10 out of 13 questions in an examination  (a)  216  (b)  375  (c)  400  (d)  720.  the  digits  0.  2.  (2002)  The  number  of  choices  available  to  him  is  Answer  Key  1.  14.  (a)  (b)  (a)  (d)  2.r  C3  is  12.  8.  then  (2003)  the  word  SACHIN  appears  at  serial  number  n  17. then the expression n C r + 1  + n C r – 1  + 2 × n C r  equals  (a)  n +  2 C r +  1  (b)  n +  1 C r  (c)  n +  1 C r +  1  (d)  n +  2 C r  .  19.  9.  such that he must choose at least 4 from the first five questions.  (a)  16.  21.  20.  Number greater than 1000 but less than 4000 is formed using  (a)  56 C 4  (b)  56 C 3  (c)  55 C 3  (d)  55 C 4 .  ways  and  these  words  are  written  out  as  in  dictionary.  (c)  10.  3.  19.  (d)  17.  7  are  16.  Five digit  number divisible  by  3 is  formed  using  0.  (c)  12.  Then  number  of  ways  of  distributing  8  identical  balls  in  3  distinct  boxes  so  that  none  of  the  boxes  is  empty  is  (a)  312  (b)  3125  (c)  120  (d)  216.  (2004)  4.  (d)  11.  (a)  3 8  (b)  21  (c)  5  (d)  8 C 3 . 6 and 7 without repetition. .2)  = 10  6 6 Þ Then the  number of ways =  6 C4  ∙  3 C1  ∙ 4!  3n(n – 1) =  60 Þ n(n  – 1) = 20 Þ  n 2  –  n – 20 =  Þ  Þ  (n – 5)(n + 4)  = 0 \  n =  5  7. ..26  1. 1 dictionary can be selected in 3 C 1  ways. P.20 C11 . we have 7 letters M.  10(10 .20 C1 + 20 C 2 . Statement­1 is false.  + 20 C10 . I.  6.. P... \ Statement­2 is true  So..20  C3  + .  7  black balls  is  = (10 + 1) (9 + 1) (7 +  1) – 1  = 880 – 1 = 879 ways  (c) : x 1  + x 2  +  x 3  +  x 4  = 10  The number of positive  integral solution  is  6 + 4 – 1 C 4 –  1 =  9 C 3  8..  (a) :  A  voter can vote one candidate or two or three  or four  candidates \  Required  number  of  ways  =  10 c 1  +  10 c 2  +  10 c 3  +  10 c 4  =  385  Fixed  11.  3..  9 green.1) (d) :  Q  20 C0 + 20 C1 x + . Statement­2 is true. I.  of  word  start  with  H  =  5!  No. we only have to  arrange 4 novels which can be  done in 4! ways..  Also out of 3 dictionaries.  Then statement­1  is true and statement­2  is false..1)  = 10  2 2  Þ  n  – n  – 20  = 0.1)(n . \  n =  5  Here we have used  n C r  +  n C r + 1  =  n +  1 C r +  1  n  Þ 2.1) 10 C j  9.1) n(n ....20 C9 ) + 20  C10  = 0  20 C0 ..1) + j ) × 10 C j  j =1  10 = å j =1 9  10  j ( j .  5.  of  word  start  with  I  =  5!  No. I.1)  8  j ( j ..  C2  = 10 Þ (b) :  Number of ways in which  one  or  more balls can be  selected from 10  white.. + 20 C20 x 20 = (1 +  x ) 20  After putting x = –1.  å j( j .20 C12 + .20 C 9 + 20  C10 =  1  C10  2  20  10... + 20 C10 x10  + å  10  = 9 ´ 10 .. + 20  C20  = 0  j = 2  10 S2 = 10  å j× 10 C j = 10 j =1 10 S3  = å j =1 9 å  C j -1 = 10 ´ 2  j =1  10  j 2 × 10C j = å ( j ( j ...1) 10C j + å j × 10 C j  j =1  = 90 ∙ 2 8  + 10  ∙ 2 9  =  (45 + 10)2 9  =  55 ∙ 2 9 .  (d)  :  S A C H I N  No..  of  word  start  with  C  =  5!  No..  of  word  start  with  A  =  5!  No.  (d) : The  number of  ways = ( 3 C 2 ) × ( 9 C 2 )  = 3 ´ 2(20 C0 .  The required number of ways = 7 ∙ 5 ∙ 3 ∙  8 C 4  = 7 ∙  6 C 4  ∙  8 C 4 ..  2 nd  solution :  n  +  1 C 3  –  n C 3  =  10  n (n .2  j ( j .  JEE MAIN CHAPTERWISE EXPLORER  (a)  1 st  solution :  n  +  1 C 3  –  n C 3  =  10  :  Since the dictionary is fixed in the middle. 4 novels can be selected in  6 C 4  ways..  7  = 7 × 5 × 3  way of arranging them =  24 And four  S  can be put in 8 places in  8 C 4  ways..2  = 90 ´ 2 8  C0 .  of  word  start  with  N  =  5!  Total  words  =  5!  +  5! +  5!  +  5!  +  5!  =  5(5  !)  =  600 . we get  20 å  8C j .  Number  of different  ways  of  arranging 6A’s  and  4B’s  in  a  row  10  10  = = C4  = Number of different ways the child can buy  6 ´  4  the  six ice­creams. .  (c) :  S1 = = 6∙5  0 ∙ 3∙ 24 = 1080 = 1080  2 (b) : We have to find the number of integral solutions  if x 1  + x 2  + x 3  + x 4  + x 5  = 6  and that equals  5+6–1 C 5–1  =  10 C 4  Thus Statement­1 is false.  9 ´ 8  = 108  2 (c) : Out of 6 novels.20 C1 + 20 C2 .20 C1 + 20 C2 -20 C3 + .20 C 3 + .1) × × C j .  ( n + 1) n(n ...  4.  = 0 (a) : Leaving S.. I..  It is the same as the number of ways of choosing any 3 balls  from  9  different places. 100  =  9 +  6  ×  2  =  21  Set  of  numbers  divisible  by  10  are  10..  S 20  =  [5  +  100]  = 10  ×  105  =  1050  2  m =  y  – 1.  (b)  :  (i)  Each  box  must  contain  at  least  one  ball since  no  =  n  +  2 C r  +  1  box  remains  empty  so  we  have  the  following  cases  18.  10.  .27  Permutations and Combinations  Now add the rank of  SACHIN so required rank of SACHIN  =  600  +  1  =  601.  n  =  z  –  1  10  i. of question 8  No.  2.  4. of question 5  No..  where  H  =  Hundred  place  =  2 C 1  ×  4 ×  4  ×  4  T  =  Ten’s  place  III  6.  .  5.e.  15. I  1.100  As  1. 1.  (b)  :  Set  of  numbers  divisible  by  2  are  2.E  these  alphabets  can  be  arrange  themselves  by  2!  ways 17.  (d)  1 ´ 3!  3  ×  +  3!  ×  2  20..  7  Number  of  ways  of  6  men  at  a  M  We  have  to  fill  up  four  places  like  TH  H  T  U  round Table  is n  –  1!  =  (6  – 1)!=  5!  (Case:  If  repetition  is  allow)  Now we  left with  six places  between  the  men  and there  are  5 C  6 2  4 C  =  5  ×  6 2  ×  4  6  5  women.  (c)  :  Let  number  of  digits  formed  x.  ..  (a) :  Case  (i)  :  Required  ways  for  first  case  =  5 C 4  ×  8 C 6  =  140  Case (ii):  No.  3. 5  1.  2.  3. =  128  U  =  Unit  place  \  Number  of  ways  19. which means left extreme digit  will be  either  2  or  3.  1  we  get  50 C4 + 50 C3 + 51 C3 + 52 C3 + 53C3 + 54 C3 + 55  C3  ( Q  nCr + n Cr +1 = n + 1 Cr +1 )  =  51 C 4  +  51 C 3  +  52 C 3  +  53 C 3  +  54 C 3  +  55 C 3  =  52 C 4  +  52 C 3  +  53 C 3  +  54 C 3  +  55 C 3  =  53 C 4  +  53 C 3  +  54 C 3  +  55 C 3  =  54 C 4  +  54 C 3  +  55 C 3  =  55 C 4  +  55 C 3  =  56 C 4  16.  Similarly.  y.  1.  2. Box  Number of balls  \  1000 <  x <  4000. z respectively  S 50  =  25[102]  then x +  y +  z =  8  and  no  box  is  empty  so  each x..  \  Required  numbers  =  2 C 1  ×  H  T  U  I  1.  1. y. 3.  3. 2. 4  have  case  ways  and  Now  sum  of  numbers  divisible  by  2  is  given  by  1... 4  2.  2. 3. of question 5  No. of question 4  No. 6  2..  (a)  50 6  C4 + å  56 .  4..  6.  2. (a) :  Consider  n C r  –  1  +  2 n C r  +  n C r  +  1  6! =  ( n C r  –  1  +  n C r )  +  ( n C r  +  n C r  +  1 )  \  Number  of  words  =  =  360  2!  = n  +  1 C r  +  n  +  1 C r  +  1  14.  4.  4.  30.  (d)  :  Number  of  women  =  5  M  M =  25  ×  122  =  3050  Number  of  men  =  6  M  M  21. 2.  5.100  2! Set  of  numbers  divisible  by  5  are  5. (l + 1) + (m + 1) + (n + 1) = 8 are non negative integers and  S 10  =  [10  +  100]  =  5  ×  110 2  \  Required  number  of  ways  =  n  +  r  –  1 C r  \  Required sum  =  25  ×  102 +  1050  –  550  =  3  +  5  –  1 C 5  =  7 C 5  =  7 C 2  =  25[102  +  42  –  22]  M  15.  3.  3.r  C3  r = 1  Putting  r  =  6. 4  have equal number of ways of arranging the  50  n  S 50  =  [2  +  100]  using  S n  =  [a  +  l]  balls  in  the  different  boxes.  20.  2  2  (ii): Let the number of balls in the boxes are x.  z ³  1 20  Þ  l  +  m  +  n  +  3  =  8  where  l  =  x  –  1. =  720 \  Required  number  of  ways  =  5!  ×  6 P 5  =  5!  × 6!  . of question 8  5  8  C 5  C 5 \  Required  ways  for  case  (ii)  =  5 C 5  ×  8 C 5  8 ´ 7 ´ 6  =  56  3 ´ 2 ´ 1 Total  number  of  ways  =  140  +  56  =  196  =  13.  5. of question 6  No.  12.  these  5  women  can  be  arranged  themselves  by  P 5  1  1  =  5 ×  36 ×  4  way.  (a)  :  Number  of  letters  =  6  Number  of  vowels  =  2  namely  A.  (d) :  Odd  numbers  are  1.   MATHEMATICAL INDUCTION  AND ITS APPLICATION 2. having  n  radical  signs  then  by  methods  of  mathematical  induction  which  is  true  (a)  a n  >  7. "  n ³  1  (b)  a n  >  3..  Statement­2  is  true;  Statement­2  is  not  a  correct  explanation  for  Statement­1  (2008)  Answer  Key  1.  (b)  Let  S(k) = 1  + 3 + 5 +  .  Then  which  of  the  following  is  true?  (a)  S(k) Þ  S(k  –  1)  (b)  S(k) Þ  S(k  +  1)  (c)  S(1)  is  correct  (d)  principle of mathematical induction can be used to prove  the  formula. Statement­2 is true; Statement­2 is a  correct  explanation  for  Statement­1  (d)  Statement­1  is  true..  (a)  Statement­1  is  true.  Statement­2  is  true  3.28  JEE MAIN CHAPTERWISE EXPLORER  CHAPTER  6  1..  1 1 1  + + .. + (2k  –  1) =  3 +  k 2 .  n ( n + 1) < n + 1.. "  n ³ 1  (c)  a n  <  4..  1 2  n Statement­2 : For every  natural number  n ³  2.  (d)  2.  (c)  Statement­1 is true..  (b)  3. + >  n . "  n ³ 1  (d)  a n  <  3.  Statement­1 : For every  natural number  n ³ 2.  (2002)  .  Statement­2  is  false  (b)  Statemen­1  is  false. "  n ³ 1.  (2004)  If  a n  = 7 + 7 + 7 + . S  of  S(k)  So  S(1)  is  not  true.  we get  1 1 1  + + ..  i.(i)  ..(iii)  1 1 1 1 1  + + + ....  L. + >  k  1 2  k Þ k> Þ k+ ( k  + 1) ....1 Þ k +1 Step 1 : For n  =  2... + >  k  1 2  k 1  Adding  on  both sides.  (b)  :  S(k)  =  1  + 3  +  .(i)  When  k  =  1.. + + > k  + 1  1 2 3 k k + 1  By Assumption step..e.........S  of  S(k) ¹  R...(ii)  Let  S(k)  is true \  1 +  3  + 5  +  .  Now  S(k  + 1);  1  + 3  + 5  +  .  P (2) : Statement­2  For n = k  k ( k + 1) < k + 1  Þ Q  k k + 1 > k k < k + 1  k  k + 1  . + >  n  1 2  n 1 1  + >  2  is true  1 2  Step 2 : Assume P(n) is true  for n =  k. + + > k  + 1  1 2 3 k k + 1  hence (ii) is true  for n =  k  + 1  hence P(n) is true  for  n ³  2  So. + (2k  – 1)  = k 2 + 3 Þ  1  + 3  +  5  + . + + > k  + 1 2 k k +1 k + 1  k > k  + 1 - \  a n  =  =  1± 1 +  28  2 1 ±  29  >  3 2 ..... +  (2k  – 1)  + (2k  +1)  = 3 + (k  + 1) 2  .  (b) :  a n  =  k + 1 < k + 1 k + 1  Þ 1  k + 1  From (iii) & (iv)  Step 3 : For  n =  k  + 1..  we get  k + 1  1 1 1 1 1  + + .(iv)  .....H. Statement­1 and Statement­2  are  correct but  Statement­2 is  not explanation  of Statement­1  2.  (d) :  Statement­1  1 1 1  Let  P ( n ) : + + ..  1 1 1  + + .29  Mathematical Induction and Its Application  1..H.  +  (2k  –  1) =  3  +  k 2  .. we  have to  show that  1 1 1 1 1  + + + .(ii)  .  +  (2k  –  1)  +  (2k  +  1)  =  3  +  k 2  +  2k  +  1  =  (k +  1) 2  +  3 Þ  S(k  +  1)  true \  S(k) Þ  S(k  +  1)  7  + a n  Þ  a n 2  –  a n  –  7  =  0 For  k ³  2  k Þ k  > k +1 Multiplying by  k  Þ 1 > 1  > k  + 1  k + 1  3..   If x is so small that x 3  and higher powers of x may  be neglected.  (b)  132  (c)  144  (d)  –132  (2011)  The remainder left out when 8 2n  – (62) 2n +1  is divided by 9 is  (a)  2  (b)  7  (c)  8  (d)  0  n  Statement­1 :  å ( r + 1) n Cr  = ( n + 2)2 n -1  r = 0  n  Statement­2 :  ( )  (2006)  (2006)  11  é 1  ù If the coefficient of x 7  in ê ax 2  + equals the coefficient  bx úû  ë 11  é æ 1  ö ù in  ê ax .  (2005)  (c)  ab  =  1  (d)  b of  x –7  10.  (2004)  n - 5  n - 4  5  6  n  (a)  6  (b)  (c)  n - 4  (d)  n - 5 .  (2009)  5.  2.  then  a n  is  1  (1 .  7.ax )(1 - bx)  (a)  n  –  1  (b)  1  n - 1  2  (c)  1  n  2  (d)  2n - 1  .1  ö æ çè 2/3 1/3 ÷ is  x .  (a)  å ( r + 1) n Cr  xr = (1 + x ) n + nx (1 + x ) n -1  ( 3  )  (1 + x )3/ 2  .  Statement­2  is  false  (d)  -  x 2 .  n)  is  (a)  (20..  The  term  independent  of  x  in  expansion  of  10  x +1 x .  20)  (c)  (45.a n +1  (c)  (d)  ..  then  (m.  then  a  and  b  satisfy  the  relation  è bx ø û  ë (a)  a  +  b  =  1  (b)  a  –  b  =  1  a  =  1.ç 2  ÷ ú .  3.  n  if  (1  –  y) m  (1  +  y) n  = 1  +  a 1 y  +  a 2 y 2  +  . the sum of 5 th  n  –  1  2  n  (a)  (–1)  (n  –  1)  (b)  (–1)  (1  –  n)  and 6 th  terms is zero.  3  (c)  x - 3 x 2  (a)  Statement­1  is  true. Statement­2 is true; Statement­2 is a  11.b n  (b)  b - a b - a a n +1 .  (d)  an odd positive  integer.b n +1  b n +1 .  n  1  r  t  5  and  t n  =  å n  .  The coefficient of the middle term in the binomial expansion  in  powers  of  x  of  (1  + ax) 4  and  of  (1  – ax) 6  is  the  same  if correct  explanation  for  Statement­1  a equals  (d)  Statement­1  is  true.  The  coefficient  of  x  in  expansion  of  (1  +  x)(1  –  x)  is  In the binomial expansion of (a – b) n .  45)  (b)  (35. n ³  5.  (c)  an irrational number..1 + 1 x  2  then 1/ 2  (1 - x )  3  2  (a)  3 x +  x 8  may be approximated  as  3  2  (b)  1 -  x 8  r = 0  6..  7  The  coefficient  of  x  in  the  expansion  of  (1 – x – x 2 + x 3 ) 6  is  (a)  –144  4..  (2005)  2 8  8  (b)  Statemen­1  is  false.  then  n  is  equal  to  13.  2  (2004)  .  b - a b - a For  natural  numbers  m.  then ( 3 + 1)2 n .  Statement­2  is  true;  Statement­2  is  (a)  –3/10  (b)  10/3  (c)  –5/3  (d)  3/5.  If  s n  =  å n  s n  r = 0  C r  r = 0  C r  (2007)  If the expansion in powers of x of the function  is  a 0  +  a 1 x  +  a 2 x 2  +  a 3 x 3  +  .  35)  (d)  (35.  If n  is a positive  integer.  not  a  correct  explanation  for  Statement­1  (2004)  (2008)  n  n  12.  (2012)  9.x + 1  x - x1/2 ø (a)  120  (b)  210  (c)  310  (d)  4  (2013)  8.  (b)  a rational number other  than positive  integers..( 3 - 1) 2 n is  (a)  an even  positive  integer. then  a/b  equals  (c)  (n  –  1)  (d)  (–1) n  –  1  n.  45).  and  a 1  =  a 2  =  10.  b n .  Statement­2  is  true  (c)  Statement­1 is true.30  JEE MAIN CHAPTERWISE EXPLORER  CHAPTER  BINOMIAL THEOREM 7  1.a n  a n .   (a)  .  The number of integral terms in the expansion of  ( 3 + 8  5) is  (a)  33  (b)  34  (c)  35  (d)  32.  (c)  6.  19.  If x is positive. the first negative  term in the  expansion of (1  + x) 27/5  is  (a)  5 th  term  (b)  8 th  term  (c)  6 th  term  (d)  7 th  term.  13.  (b)  12.  (b)  (d)  (c)  (c) 2.  r  and  n  are  positive  integers r  >  1.  (a)  9.  (c)  11.  (a)  17.  (c)  15.  (d)  16.  The  positive  integer  just  greater  than  (1  +  .  If  the  sum  of  the  coefficients  in  the  expansion    of  (a  +  b) n  is  4096.  (d)  14. n  >  2  and  coefficient  of  (2002)  th  th  2n  (r + 2)  term and 3r  term in the expansion of (1 + x)  are  Answer  Key  1.  then  n  equals  (a)  3r  (b)  3r  +  1  (c)  2r  (d)  2r  +  1.  (c)  5.  (a)  4.  (a)  10.  (2003)  256  15.  then  the  greatest  coefficient  in  the  expansion  is  (a)  1594  (b)  792  (c)  924  (d)  2924.31  Binomial Theorem  14.  The  coefficients  of  x p  and  x q  in  the  expansion  of  (1  +  x) p  +  q  are  (a)  equal  (b)  equal  with  opposite  signs  (c)  reciprocals  of  each  other  (d)  none  of  these.  (2002)  19.0001) 1000  is  (a)  4  (b)  5  (c)  2  (d)  3.  (2002)  equal.  17.  (2003)  16.  (d)  3.  (b)  18.  (2002)  18.  7.  (c)  8. .ax )(1 - bx )  =  (a 0  +  a 1 x  +  ...x -1/2 ) 10  \ 20 .x1/3 + 1) ( x + 1)( x  . ù ....( 3 - 1) 2 n = 2 é 2n C1∙( 3) 2n -1 + 2 nC3 ∙( 3) 2n .)  Þ  (a 0  +  a 1x    +  ...2 b 2 + .  (c) :  n n  å ( r + 1) n Cr = å r × n Cr + å n Cr  r =0 r =0 r = 0  n n  n  = å r × × n -1 Cr -1  + å n C r  r r =0 r = 0  = n × 2n -1 + 2n = 2n -1 ( n + 2)  Thus Statement­1 is  true...  +  a nx    =  1  +  x  (a  +  b)  +  x 2  (a 2  +  ab  +  b 2 )  +  x 3  (a 3  +  a 2 b  +  ab 2  +  b 3 )  +  .....  45)  ( (c)  : Tr + 1  of ax 2 + 1 bx ) 11 1 Tr + 1  of æç ax .....r  1  Cr  ( ax ) r  æç .4  = .  +  a n–1 x n–1  +  a n x n  +  ..  (***)    +  .3 + ..  +  x n  n  (a  +  a n–1  b  +  a n–2  b 2  +  .b ) = - Cr -1 x r + å n Cr x r  = nx (1 + x )n -1  + (1 + x ) n  Substitute x  = 1 in the above  identity  to get  å ( r + 1) n Cr  = n × 2n -1  + 2 n  Statement­2 is also  true &  explains Statement­1  also.  +  a  b n–1  +  b n  (a) : (1 – x – x 2  +  x 3 ) 6  = ((1 –  x)(1 – x 2 )) 6  8 = –1 (modulo 9)  ( On  comparing  the  coefficient  of  x n  both  sides  we  have  an irrational number........  n n  n r n r n r  Again  å ( r + 1)  Cr x = å r × Cr x + å  Cr x r =0 r = 0  n = nå r=0 a n .bx ) -1  (1 .r.....  ìï ( x1/3 + 1)( x 2/3 ..  (a) : Using Modulo Arithmetic  Also  62  = –1 (modulo 9) = [(–1) 2n  –  (–1) 2n + 1 ]mod  9  = (1 + 1)mod 9 = 2 mod  9 Þ Remainder =  2  n 5..  x +1 x .  n)  =  (35..  (m..  3..a n +1  .........  (*)  Differentiating  w.... Þ  n(1  +  y) n–1 (1  –  y) m  –  m(1–  y) m–1 (1  +  y) n  =  a 1  +  2a 2 y  +  3a 2 y 2  +  4a 4 y 3  +  ...t.1  ö æ (b) :  ç 2/3 1/3 è x ..(**)  Again  differentiating  (**)  with  respect  to  y  we  have  [n(n  –  1)(1  +  y) n–2 (1  –  y) m  +  n(1  +  y) n–1 (–m)  (1  –  y) m–1 ]  –[m(1 +  y) n (m  – 1)(1 – y) m–2 (1–y) m–1 n(1 + y)+n–1]  = 2a 2  +  6a 3y  ...ax ) -1 (1 .)  (1  +  bx  +  b 2 x 2  .5 r 6  = ( x1/3 ......r  ( )  1  bx 11  )  1  bx  = 11  C 5  a 6  b5  .  y  both  sides  of  (*)  we  have  –m(1  –  y) m–1 (1  +  y) n  +  (1  –  y) m  n(1  +  y) n–1  =  a 1  +  2a 2 y  +  3a 3 y 2  +  4a 4 y 3  +  ..  n n -4 4 (b) : C4 a ( .........1) üï =í ý x ( x  ...  +  ......  ë û  = (1 –  6 C 1 x +  6 C 2 x 2  –  6 C 3 x 3  +  6 C 4 x 4  –  6 C 5 x 5  +  6 C 6 x 6 )  (1 –  6 C 1 x 2  +  6 C 2 x 4  –  6 C 3 x 6  +  6 C 4 x 8  –  6 C 5 x 10  +  6 C 6 x 12 )  Coeff.  b - a (d)  :  (1  –  y) m  (1  +  y) n  =  1  +  a 1 y  +  a 2 y 2  +  a 3 y 3  +  ... +  a n x n  +  .......  r = 4  (c) :  ( 3 + 1)2 n ..x + 1  x - x1/2 ÷ø 6.2  ö÷ è bx ø 11  2 \  Coeff.  +  ab n–1  +  b n )+  .(B)  Solving  (A)  and  (B)  n  = 45......32  JEE MAIN CHAPTERWISE EXPLORER  10  1.2 ö÷ bx ø è ( 11 = =  11Cr (ax 2 ) r  11 ..1)  þ  x 2/3 ..  m  =  35 \  n  n -1  r = 0  ( a n + a n -1b + a n .  = 6 ∙ 20 – 20 ∙ 15 + 6 ∙ 6 = 120 – 300 + 36 = –144  4.  10  7.. of x 7  = (– 6 C 1 )(– 6 C 3 ) +  (– 6 C 3 )( 6 C 2 )  +  (– 6 C 5 )(– 6 C 1 )  = 8.  Now  putting  y  =  0  in  (**)  and  (***)  we  get  n  –  m  =  a 1  =  10  (A)  and  m 2  +  n 2  –(m  +  n) –2  mn  =  2a 2  =  20    .)  (1  –  bx) –1  =  (1 +  ax +  a 2 x 2  +  .  +  ......5 r  =0 6  Thus  2......  = = (1  – x) 6  (1 – x 2 ) 6  Þ  8 2n  – (62) 2n +  1  n a n  =  a n  +  a n–1  b  +  a n–2  b 2  +  .  .  +  .) n  +  ...... +  b n x n  +  .  of  x 7  in ax + 11 ....x1/3 + 1  ï îï 20 . + abn -1  + b n )( b .  b 5  )  C5 a n -5 ( - b ) 5  Þ  (d)  :  From  given  1  = (1 ...  b n +1 .  Tr + 1  = ( - 1)r 10 Cr x  Þ \  Term  =  10 C 4  = 210.a )  b - a (Multiplying  and dividing  by b  –  a)  9..   16.3r + 1 = r + 1  ï Þ 2 n = 4 r  Þ  3r  –  1  =  r +  1  and í ï n = 2 r î 2r  =  2  n .. =  11 ×  3×4 ×  7  =  924 6 5 4 3 2 1 3 . 8 7  11 ´ 9 .  8.  (a) : Coefficient of  middle term in (1  + ax) 4  =  coefficient of  middle  term  in  (1  – ax) 6 3  10 12.å  n C r  t n  =  n  å 1  n Cr n  - 10  9 10  \  The  positive  integer  just  greater  than  is  2......  (c) :  Consider  (a  +  b) n  =  C 0  a n  +  C 1 a n  –  1 b  n  +  C 2 a n–  2 b 2  +  .1 + 3 × 1 x + 3 × 2 × 1 x 2  + .33  Binomial Theorem  11  1  ö æ and  coefficient  of  x –7  in  ç ax .3 x 2 é1 + 1 x + 1 × 3 × 1  x 2  + .  8  11. +  ç 4  ÷ ç ÷ 4  è 10 ø  è 10 ø 2  è 10 ø  \  ab  =  1.(ii) \  By  (i)  and  (ii)  Coefficient  of  x p  in  (1  +  x) p  +  q  =  Coefficient  of  x q  in  (1  +  x) p  +  q  19.  9  17. (n ..  (a)  :  (3 1/2  +  5 1/8 ) 256  r  T r  +  1  =  256 C r  ( 3) 256 - r 5 8  256 - r r ...  (b):  (1  +  x)  (1 –  x) n  =  (1  –  x) n  +  x(1  –  x) n \  Coefficient  of  x n  is  =  (–1) n  +  (–1) n  –  1  n C 1  =  (–1) n  [1  –  n]  \  4 C 2a 2  =  6 C 3 (– a) 3 Þ a  = -  n  13.  (d)  : 1/ 2  (1 . 7  .r )  n  C n .  (a)  :  In  the  expansion  of  (1  +  x) p  +  q  T r  +  1  =  p  +  q C r x r \  Coefficient  of  x p  =  p  +  q C p  å  n C r  n  <  1  +  .  n  >  2  and  Coefficient  of  T r  +  2  =  Coefficient  of  T 3r  in  (1  +  x) 2n Þ 2n C r  +  1  =  2n C 3r  –  1 R  <  \  ì 2 n .. 2 .  3  ( )  (1 + x )3/ 2  .  . 8 ..1) .x 2 (1 - x ) -1/ 2  = . 1 .  (c) :  Given  r  >  1. ..  =  ´  ..  are  both  positive  integer 2 8  \ r  =  0.....( n .r  r  = 0  1  n  n Þ  t n  =  n  r =0 r  å nC r .r  =  r  = 0  replacing  n  –  r  by  r  r  = 0  t n  =  ns n  –  t n t n  n  \ =  s n  2  =  ( p + q )!  q !( p + q - q )!  =  ( p +  q )!  q ! p !  27  n( n .  (c):  t n  =  r  r  = 0  t n  =  å n ..  (d)  :  General  term  in  the  expansion  of  (1 +  x ) 5  For  integral  terms  10  1 1 1  + +  3  +  .r  å  n C n .x )  3 1 + x + 3 1 1 x 2 + .2 ÷ = bx ø  è 11 Now  a6 = b5 C5 11  C 6  a 5  b 6  11  C 6  a 5  b6  1000  1  ö æ 16.. ù 8  úû  8 êë 2 2 2 2!  3  = .r  + 1)  r  x  r !  32  27  \ n  –  r  + 1  <  0 Þ  +  1  <  r Þ  r  >  5 5  T r  +  1  =  Þ  r  >  6  15.¥ =  9 10  10 2 10 ìQ nCx = n  C y  ï r =  1  í Þ x + y = n  ï or  x = y î 18.  2 2 2 2! 2 2! 4  = (1 - x ) 1/ 2  ( )( )  3  = ..  +  C n  2 n  = 4096  =  2 12 Þ  n  =  12  (even)  Now (a  +  b) n  =  (a  +  b) 12  as  n  =  12  is  even  so coefficient  of  greatest  term  is  C n  =  12C12 =  12 C6  n  2  =  2  12 11 10 9 .x 2 + higher  powers  of  x 2 .(i)  Also  coefficient  of  x q  in  (1  +  x) p  +  q  is  =  p  +  q C q  14.  (c)  :  Let  R  =  ç 1  + 4  ÷ 10 ø  è 10 3  1 2  999 æ 1  ö æ 1 ö æ 1 ö = 1 + 1000 ç 4 ÷ + 1000  + .256 \ 256  =  0  +  (n  –  1)8  using  t n  =  a  +  (n  –  1)d 256 256 \ =  n  –  1 \  n  =  +  1  8  8  n  =  32  +  1 Þ  n  =  33  ( p + q)! ( p + q )!  = p !( p + q -  p)! p ! q !  .1 + 1 x  2  10......  +  C nb    Putting  a =  b  =  1 \  2 n  =  C 0  +  C 1  +  C 2  +  ..... .  n Statement 2 : å ( k 3 .  (2012)  3..  is  (a)  150  (b)  zero  (c)  – 150  (d)  150 times its 50 th  term  (2012)  10. .1 + 1  - . Statement 2 is  false. Statement 2 is  true....( k .  12.77.777...P....  If  the  coefficients  of  r th ..   . = a 10  = 150 and a 10 .. If the terms  of  the  geometric  progression  are  alternately  positive  and  negative... p ¹ q ..P.  If  a1 + a2  + .P.  (a)  Statement 1 is  true. Then the common  ratio of this progression is equals  (a)  (b) 1  5 - 1  2 ( 5  1  (c) 2 1 -  5  ( )  )  1  (d)  2  5.34  JEE MAIN CHAPTERWISE EXPLORER  CHAPTER  8  1. + a p  p 2  a  = .  The sum of the third and the fourth terms is 48. each  term equals the sum of the next two terms.  a n  are  in  H..  a 2 ..  (2005)  .. + (361 + 380 + 400) is 8000.  then  the  expression  a 1a    2  +  a 2a    3  +  . .  correct  explanation  for  Statement  1.  then  6  equals  a1 + a2  + .  SEQUENCES AND SERIES The  sum  of  first  20  terms  of  the  sequence    0. then  the first term is  (a)  4  (b)  – 4  (c)  – 12  (d)  12  (2008)  1 .P.  If 100 times the 100 th  term of an A... +  a n  –  1 a n  is  equal  to  (a)  n(a 1  –  a n)  (b)  (n  –  1)(a 1  –  a n)      (c)  na 1 a n  (d)  (n  –  1)a 1 a n . Statement 2 is true; Statement 2 is  not a  correct  explanation  for  Statement  1.  then the  150 th  term of this A.  4.P.  2.  be  terms  of  an  A..  (r  +  1) th  and  (r  +  2) th  terms  in  the  If  a 1  = a 2  = .  a 3 . with  binomial expansion of (1 + y) m  are in A.. is  (a)  7  (99 - 10 -20 )  9 (b)  7  (179 + 10-20 )  81 (c)  7  (99 + 10 -20 )  9 (d)  7  (179 - 10-20 )  81 7. + aq  q 2  a 21  (a)  41/11  (b)  7/2  (c)  2/7  (d)  11/41.  (2013)  Statement  1  :  The  sum  of  the  series  1  +  (1  +  2  +  4)  +  (4 + 6 + 9) + (9 + 12 + 16) + .  His  total  saving  from  the  start  of  service  will  be  `  11040  after  11.  (2006)  If  a 1.  6..  (c)  Statement 1 is false.1)3 ) = n 3  for any natural number n .  k = 1  8. then m and r satisfy  common difference –2..  0.  A man saves  `  200  in each  of  the  first three months of his  service.  upto infinity  is  The  sum of  the  series  2! 3! 4! 1  (a)  e -  2  (b)  e + 1  2  (c)  e –2  (d)  e –1 .   a 2..7...  (b)  Statement 1 is true...  .  0..  (d)  Statement 1 is true.  (a)  20 months  (b)  21 months  (c)  18 months  (d)  19 months  (2011)  The sum to infinity of the series  1 + 2 + 6 + 10 + 14  + . a 11 . is  3  32 33 34  (a)  3  (b)  4  (c)  6  (d)  2  (2009)  The first two terms of a geometric progression add up to 12. are in an A. Let a n  denote the  number  of  notes  he  counts  in  the  n th  minute. Statement 2 is true; Statement­ 2 is a  9.  (2007)  Let  a 1 .  (2006)  A person is to count 4500 currency notes.. In each of the subsequent months his saving increases  by ` 40 more than the saving of immediately previous month.  5. then the time taken by him to count  the  equation  all notes is  (a)  m 2  –  m(4r  –  1)  +  4r 2  + 2  =  0  (a)  24 minutes  (b)  34 minutes  (b)  m 2  –  m(4r  +  1)  +  4r 2  – 2  =  0  (c)  125 minutes  (d)  135 minutes  (c)  m 2  –  m(4r  +  1)  +  4r 2  + 2  =  0  (2010)  (d)  m 2  –  m(4r  –  1)  +  4r 2  –  2  =  0. with non­zero common  difference equals  the  50  times its  50 th  term.  (2007)  In a geometric progression consisting of positive terms.P. . log 3[4 ×  3 x  – 1] are in A.  are  in  G.  1  1  (b)  G.  13.  R 1 .  y 1 )..  21.  (d)  e - 1  ..P.  (b)  H.  m  and  T n  =  .  (2005)  14.  n n n  13.  then  the  points  (x 1 ..  25.  (2003)  21.  (c)  Arithmetic­Geometric  Progression  (d)  A.  then  a  –  d  equals  n m (c)  H.).P.  8.  24.  27.  a 3 .P.  14.  b. x 3  and y 1 .  R.  is  equal  to  log an + 8  (a)  0  (b)  1  (c)  2  (a)  log e  2  –  1  (c)  log e  (4/e)  (2005)  15.  n.  19. c are in A. y 2 .  If  f  (1)  =  f  (–1)  and  a.G.  a n . When  n  is  odd.  (d)  (d)  (a)  (d)  (b)  3. with the same common  ratio.  Let  R 1  and  R 2  respectively  be  the  maximum  ranges  up  and  down on an  inclined plane and R be the maximum range  on  16.  (d)  G.  then  f ¢(a).  If x 1 .  26.P.  y 3 )  (a)  lie  on  an  ellipse  (b)  lie  on  a  circle  (c)  are  vertices  of  a  triangle  (d)  lie  on  a  straight  line.  (2003)  23.  (b)  (d)  (b)  (c)  6.P.  whose  first  term  is  a  and  the  horizontal  plane.  (2003)  22. x 2 . upto ¥ is equal to  1 × 2 2 × 3 3 × 4 (b)  log e  2  (d)  2log e  2.  (b)  are  in  H.  17.  (c)  satisfy a + 2b + 3c  =  0  (d)  are  in A.  2  2  n ( n +  1)  n ( n +  1)  (2002)  (a)  (b)  4  2  26.  and |a| <  1..  (x 2 ..1 + 1  .  18.  (a)  256  (b)  512  (c)  1024  (d)  none.  +  9  =  n  is  even.  23..  (b)  (b)  (c)  (c)  (b)  4.  (2002)  2  4  1 2  + 2 × 2 2  + 3 2  + 2 × 4 2  + 5 2  + 2 × 6 2  + .  15..  Then..  .  (c)  (d)  (2004)  e 2  28.  1 1  +  .....P.P.  (b)  Arithmetic­Geometric  progression  (c)  A. then x equals    (a)  log 3 4  (b)  1  –  log 3 4  17.  then  the  determinant  log an log an + 1 log a n + 2  D =  log an + 3 log an + 4 log an + 6 log an + 7 log a n + 5  .  (2002)  then  a.  The  sum  of  first  n  terms  of  the  series  (c)  1  –  log  3  (d)  log 4 3.P. x +  4cy + cz =  0 has  a  non­zero solution.  22.  10.  |b| <  1.  (b)  (d)  (c)  (d)  (b) 5.P.35  Sequences and Series  ¥ ¥ ¥ n=0 n=0 n = 0  (a)  are  in  G.  Let  f  (x) be a  polynomial  function  of second degree.  then  the  product  of  its  9  terms  is  19.  7.  z  are  in  (a)  H.P.  Fifth  term  of  a  GP  is  2.  ¥  is  4 × 2! 16 × 4! 64 × 6! e - 1  (a)  e + 1  (b)  (c)  e + 1  e e 2  e 20.P.  12.P. y 3  are both in G..  The  sum  of  series  2! 4! 6! 27.  If  x = å a .  x + 3by + bz = 0.  (2004)  (c)  (d)  éê square  is  100. c  are  in A. The  common  ratio  of  GP  is  ú ë 2  û  2  (a)  5  (b)  3/5  (c)  8/5  (d)  1/5.  z = å  c where  a.  The  sum  of  the  series  1 + 1 + 1 + 1  + .  1 1 1  (2002)  + + +  .  11. b. y = å b .  y...  Let  T r  be  the  r th  term  of  an  A.P.  The  value  of  21/ 4 × 41/8 × 81/6  .  If  a 1 ..P..  Sum of infinite number of terms in GP is 20 and sum of their  2  3n(n + 1)  n(n + 1) ù .  R 2  are  in  common  difference  is  d.P.  f ¢(b)  and  f ¢(c)  are  in  (a)  G. is  n( n + 1)  when  3  3  3  3  3  2  25.  T  =  .  9.  28.  20..  If  the  system  of  linear  equations  x  +  2ay  +  az = 0. log 9 (3 1 –  x  + 2). ¥  is  (e - 1) 2  (e 2  - 1)  (a)  (b)  2 e 2 e (a)  1  (b)  2  (c)  3/2  (d)  4.P..  .  If  for  some  positive  integers  m.  |c|  <  1  then  x.  1  –  2  +  3  –  4  +  .  (b)  (c)  (a)  (b)  (a)  2.  2  e (2005)  (2003)  1 .  2  - 2)  2  ( e  (2002)  (e - 1)  .  a 2 .  If 1.  c  Answer  Key  1..  y 2 )  and  (x 3 ..  the  sum  is  (a)  425  (b)  – 425  (c)  475  (d)  – 475.P.  (2003)  (a)  1/mn  (b)  1  (c)  0  (d)  m n (2004)  24.P.  The sum of the series  (d)  4.  is  18.P.  b..  16.  (d)  Arithmetic­Geometric  Progression  (A. .  (a)  (b)  (a)  (c)  .  m ¹  n.  (a)  A.   3 3  3 3 3  7  (179 + 10-20 )  81 2.1)( -21)) = 3000  2  Let n – 10 =  m Þ  m × 148 –m(m – 1) = 3000 Þ  m 2  – 149m + 3000 =  0 Þ  (m – 24)(m – 125) = 0 9.(1)  .600  = 10440 è 2  ÷ø  Þ  n 2  + 5n – 546  = 0 Þ  (n +  26)(n – 21)  = 0 8.3 ö Þ ç (480 + 40n ..  (d)  :  Given  a 1 .  n ..7  14444244443  Hence.4)40} = 11040  2  Now  a = æ n ...to ¥ ú = ∙ = ∙ = 2  3 êë 3 û 3 1 - 1  3 2 / 3  3 2  Þ S = 2 \ S = 3  3  7. n  = 34 is the only  answer..  a1 + a2  + .1) d q \  m = 24..160) = 11040 ...  (b) : We have  a 1  + a 2  + .  ar 3 . + r = (1 ...  T 3  = 19 = 27 – 8 Þ  T n  =  n 3  –  (n  – 1) 3 4é 1 4 1  ù 4 1 1 + + .(1)  .P.. +  a n  = 4500  – 10 × 150  = 3000 Þ  148 + 146 + ..10-20 ) ý 9 íî 9  þ 1 + (1 + 2 + 4) + (4 + 6 + 9) + .. ..77777.  12 12 12  = = = -12 From (1)  1 + r 1 .1) d  p  = 2 a1  + ( q .. we  have  a 135  = 148 + (135 – 1)(–2)  = 148 – 268 < 0  But a 34  is positive....e.x  = 1 .10 .  a = ar + ar 2 Þ  r 2  +  r – 1 = 0 -1 + 5  .36  JEE MAIN CHAPTERWISE EXPLORER  1..  (a)  : Let S = 1 + 7ì 1  ü 20 ..  ..  (b) :  tr  = 0.. ar 2 ...... be  a.  we  have a +  ar  = 12  ar 2  + ar 3  = 48  on  division we  have  = \  Statement 2 is  a  correct explanation  of  statement 1..+ - .  2! 3! 4!  upto  infinity  \  n = 21.  (b) :100 (a + 99d) = 50 (a  + 49d) Þ  a + 149d = 0 i.  r  terms  7 7 7 7  = + + . T 150  =  0  3 ´ 200 + .....  (b) : Let it happens  after  n months.x + x  .  . upto  infinity.+ ..x + x .  3  32 33 3 4  1 1 2 6 10  S = + + + + . \ r  = – 2  3.  3 3 31 3 1  3  Subtracting (2) from (1).(2)  ar 2 (1 + r ) 48  = Þ r 2  = 4  a (1 + r ) 12  \  r = ± 2  But the  terms are alternately positive and negative.  giving n = 34.  (d) :Statement 1 :  . a p  p 2  = 2  a1 + a2  + ....  ar...3  {2 ´ 240 + ( n .  a 3 ..  5... T 2  = 7 = 8 – 1...  a 2 ... 135  But for  n  = 135. = 3000 n ......2 - 1  2 3 4  (d) :  Q  e .(2)  2 1 4 4 4  S = 1 + + 2 + 3 + 4  + .  Then put x = 1.. a q  q Þ  r =  Þ  p  [ 2 a1  + ( p .10...r )  10 102  10  9  20 S 20  = å r =1 = 7æ tr  = ç 20 9è 20  ö 10.. + (361 + 380 + 400) is 8000  n  3 3 3  Statement 2 :  å  ( k ..1) ) = n k  = 1  Statement 1 : T 1  =  1.  be  terms  of A.1) d ]  2  Þ  2 a 1  + ( p . we get  6...  2  10.  4..  1! 2! 3! 4!  (b) : Given..10  Þ × (2 ´ 148 + (n ..(k .(1 . 125......  .1) d ]  p 2  2  = 2  q  q  [ 2 a1  + ( q .r ÷ = ø r =1  å  2 6 10 14  + + + + .  (c) :  Let the  G. we get  1 1 1 1  e -1  = 1 .P.. +  a n  =  4500 Þ  a 11  + a 12  + . 1 .  c  =  x y z log b + ( r + 7)log R Þ  using  (applying  C 2 ®  2C 2  –    (C 1  +  C 3 ))  log b + ( r .  a n  –  1  a n  =  (n  –  1)a 1a    n ...    a 2... å bn = ..(ii)  ..1  d -  a n  d .37 Sequences and Series  Þ  [2a 1  +  (p  –  1)d]q  =  p[2a 1  +  (q  –  1)d] as  a.  b.  z = 1.  (b)  :  T r +  1  =  m C r  y  r ...P.r + 1)!  Þ  log b + ( r + 1)log R  = M  an -1 a n  = log b + r log R log b + ( r + 2)log R log b + ( r + 3)log R log b + ( r + 4)log R  1  n  1  m  1 1  Now  T m  –  T n  =  -  =  (m  –  n)  d n m 1  1  Þ  d  =  and  a  =  mn  mn \ a  –  d  =  0  T n  =  a  +  (n  –  1)  d  =  17.r )  + ( r + 1)!( m . ¥ ÷ ú ç ÷ 2 ê ø úû  ë è 2 2! 2 (4!) 2 (6!) = öù 1 é æ x 2 x 4 x 6  + + + . y .  a.. ¥  4(2!) 16(4!) 64(6!) 1 1 1  = 1+ 2 + 4 + + . \ log r = log b + ( r - 1)log R 11. y= .  | b |  <  1..  å c n  = 1...x  ùû  2  where  x  =  1/2 = 1 [ e1/ 2 + e -1/ 2 ]  =  e + 1  2  2  e 16..  (b): As  S n  is  needed  for  n  is  odd  let  n  =  2k  +  1 \  S n  = S 2k +  1  =  Sum up to  2k terms + (2k + 1) th  term  2k (2k + 1) 2  + last  term  2  ( n - 1) n 2  n 2 ( n + 1)  =  +  n 2  as  n =  2k  +  1  =  2  2  =  .    . ..r + 1)! ( r + 1)!( m .  (n)  Adding  (i).r + 1)( m .a n = 0 1 ...b 1 - c Þa = log b +( r + 6)log R y ..a2 a a  a1 a 2  = 1 = 1 -  2  d d d a 2  a 3  a2 a 3  = -  d d Þ  .r + 1)! ( r + 1)!( m .P.  (a) :  Let  t r  denote  the  r th  term of  G.1)!( m .  b..  (i)  .P.  ¥ and  n å a = n=0 ¥ ¥ 1 1 1  .r - 1)!  r !( m - r )!  Þ r ( r + 1) ( m ..1 ö 2 1 1  x .  (d)  :  Given  a 1..  \  12....(i)  ..  13.a 1.P. \  m C r  –  1  +  m C r +  1  =  2  ×  m C r  Þ m! m !  m !  + = 2  ( r .  a n  are  in  H..1)log R log b + ( r + 5)log R a .b n = 0  1 - c 1 1 1  \ x = ...r + 1)!  r (r + 1) +  (m  –  r  +  1)(m  –  r)  =  2(r  +  1)(m  – r  +  1) Þ  r (r + 1)  +  (m  –  r) 2  +  m  –  r –  2(r  +  1)  (m  –  (r  –  1))=0 Þ  r (r + 1) + m 2  + r 2  – 2mr + m – r +  2 (r 2  –  1)  – 2m(r + 1)  =  0 Þ  m 2  –  m  (4r  +  1)  +  4r 2  –  2  =  0.  (a)  :  Given | a |  <  1. b= . c Π AP \  2b =  a +  c  Þ  2a 1 (q  –  p)  =  d[(q  –  1)p  –  (p  –  1)q] Þ  2a 1 (q  –  p)  =  d(q  –  p) Þ  2a 1  =  d æ y .1  x -1 z . z Π H. Î A.  2 1 1 1  + + + .r +1)  = ( r + 1)!( m ...  (n)  equations  we  get  a 1  a n  -  a 1a    2  +  a 2 a 3  +  a 3 a 4  +  .1) log R 0 log b + log R ( r + 1)  1  log b + ( r + 2) log R 0 log b + ( r + 4) log R  2  log b + ( r + 5) log R 0 log b + ( r + 7) log R 1  = ´ 0 =  0.  (ii)  .1 z . Þ  1 1 1  ..P.  (ii)  a n .  | c |  <  1...  14.  a n  –  1  a n  =  d d 1 1  Also  = + ( n ..1  .a n  = ( n .  (c)  :  1 + = öù 1é æ 1 1 1  ê 2 ç1 + 2 + 4 + 6  + .. ¥ ÷ ú ê 2 ç1 + 2ë è 2! 4! 6!  ø û  = 1 éë e x + e .  (c) :  T m  =  a  +  (m  – 1)  d  =  2( r + 1)( m ..1  2 ç ÷ = x + z Þ y = x +  z è y ø  Þ  a 6  a + 5d a + 10 a 1  = 1 = 1 a21 a1 + 20 d a1 + 40 a1  Þ  a 6  =  11 a21  41  Þ x .1) a1 a n  d Þ  a 1a    2  +  a 2 a 3  +  ...  a1 a 2  a n  Þ  1 1  = d  a2 a1  Now  from  given  determinant  we  have  log b + ( r . ¥  2 2! 2 (4!) 26 (6!) 15..1) d  an  a1  a1  ..... with  first  term  b  and  common  ratio  R  \ tr  = bR r  ..  c Π A. 1 2 n 1 1  ö å çè 2 n .1 .  1 1 1  + –  ...  R 2 Π H...P. 2  Þ log 3 (3 1  –  x  +  2)  =  log 3 (4∙3 x  –  1)  +  1 Þ  3 1  –  x  +  2  =  (4∙3 x  –  1)  ×  3  Q  log 3 3  =  1.  +  (2n  –  1) 3 ] n  =  odd  =  5  –  2 3 [1 3  +  2 3  +  .  (c) :  Let q  be  the  angle of  inclination of  plane  to horizontal  and  u  be  the  velocity  of  projection  of  the  projectile u 2  u 2  ..  1 3 b b  =  0 1 4 c c  Þ  (3bc  –  4bc)  –  2a(c  –  b)  +  a(4c  –  3b)  =  0 2ac  =  b a + c Þ a. Þ  f ¢ (a)... + 9 3 ) – (2 3  + 4 3  + 6 3  +  8 3 )  = (1 3  + 3 3  + 5 3  + .38  JEE MAIN CHAPTERWISE EXPLORER  =  –[log e 2  –  1]  =  1  –  log e 2  Now s  =  s 1  –  s 2  =  (A)  –  (B)  =  log e 2  –  1  +  log e 2  =  log(4/e)  é ù x 2 x 4  + + .  t  =  3/4 Þ  3 x  =  –1/3  which  is  not  possible  3  3  Þ 3 x  =  and  t  =  4 4 Þ x  log 3 3 =  log 3 3 –  log 3 4  ........  C  are  collinear..  B(ar.P.÷ + ç .r )  b  = and  a (1 - r )  a br (1 ..  24.+ .(B)  25...  (d) :  Let  x 1  =  a \  x 2  =  ar.  (a)  :  e x  +  e –x  =  2 ê1 + 2! 4!  úû  ëê -1  1 1  e + e –  1  =  + + ..  2ab.. Þ  a 2 .  C(ar 2 ..¥  2 × 3 4 × 5 6 × 7 å 1  log 3  (3 1  –  x  +  2). c Π A.  f ¢ (b). ¥  2  2! 4! 1 1 1  (e - 1) 2  + + + ..(A)  (By taking logarithm at  the  base  3  both  sides) Þ  x  =  1  –  log 3 4  1  (2n) (2n + 1)  s 2  = å  t n¢  =  t¢ n  =  1 = (2n )(2 n + 1) R 1 .¥  1 × 2 2 × 3 3 × 4 1 1 1  + + Let  s 1  =  +  .P..  ac Π A.  f ¢ (c)  =  2ac  as  a.... ¥ ú 2 3 4 5 ë û  =  [1 3  +  3 3  +  .P.  (c):  s  =  \  s n = å t n  =  22.  B. 1  –  x  Þ  3  +  2  =  12∙3 x  –  3 x  1  Þ 3  [(3  –  x )  +  2]  =  12∙3 2x  –  3∙3 x  (multiplying  3 x  both  side) 2  x Þ 12t  –  5t  –  3  =  0  where  t  =  3  Þ  (3t  +  1)  (4t  –  3)  =  0 Þ  t  =  –1/3..  R. ¥ ú 18.  y 3  =  br 2  Now  A(a..1)(2n) 2n .  f ¢ (b)  =  2ab... .  but  point  B  is  common  so  A.r )  b  = slope  of  BC  =  ar (1 - r )  a as  slope  of  AB  =  slope  of  BC \  AB  ||  BC...  br).  Now  slope  of  AB =  Þ  21.  b.  +  n 3 ] n  =  even  =  4  = [2n(n  + 1)  (n  +  2)  (n  +  3) –  12n(n  + 1)  (n  +  2)  +  13n(n  é n 2 ( n + 1) 2  ù +  1)  –  n] n  =  5  (odd)  –  2 3  ê ú 4  êë úû n = 4  (even)  (Remember  this  result) .. ¥ è 2 3 ø è 3 4 ø  é 1 1 1 1  ù =  .  R 2  =  \  R 1  =  g (1 .  br 2 )  23.sin q)  g (1 + sin q )  2 g  2  1 1  +  \  =  2  = R  u R1 R2  20.  x 3  =  ar 2  and  y 1  =  b \  y 2  =  br..  f ¢ (c) Π A.  log 3 (4∙3 x  –  1) Π A.  \ t n  =  .  (b) : For non trivial solution the determinant of the coefficient  of  various  term  vanish  1 2 a a  i.2 n ÷ø 1 1 1 1  + .÷ + ..  c Π H.P.¥ 1 × 2 3 × 4 5 × 6 æ b(1 . Þ 2a 2 .  Þ 1 1 1  = (2n ...ê .  2ac Π A.  ab.P..  b.  (d)  :  Let  the  polynomial  be  f (x)  =  ax 2  +  bx  +  c  given f (1)  =  f (–1) Þ  b  =  0 \  f (x) =  ax 2  +  c  now  f ¢ (x) =  2ax \  f ¢ (a) = 2a 2 .  (c)  :  As 1.+ + ...  b).  (a)  : (1 3  + 3 3  + 5 3  + ..¥  2 3 4 5 =  log e 2  =  1 - Again s 2  =  1 1 1  + + +  .  + 9 3 )  – 2 3 (1 3  + 2 3  + 3 3  +  4 3 )  1  ö æ 1 ç ÷ è 2n 2 n + 1 ø å  æ 1 1 ö æ 1 1 ö =  ç . ¥  =  2! 4! 6! 2 e 19..P.+  .e. ..  r  =  common  ratio  1 - r S¥  =  20  a  According  to  question  =  20 1 - r Þ a  =  20(1  –  r)  .(*)  ...(i)  2  Also  a  1 - r 2  =  100 Solving  (i)  and  (ii)  we  have  r  =  3/5  =  2l (say)  1 2 3 4  + + +  +  .r 1 + r Þ  a  =  5(1  +  r)  27.ar 8  i = 1  8 ´ 9  =  a9 r 2  .P  is  a  and  common  ratio r \  t 5  =  ar 4  =  2 9  \  a a  ...P..  (b)  :  Let  terms  of  G.  ar.....  (b)  :  Let  first  term  of  a  G.r 4 1  2 so  S¥  =  2 1  Where l  =  28..  Now (B)  –  (A) Þ  = + + 2 4 8 16 32 64 a  1 2  l  = ´ =  \ l  =  1  1 .  Þ  =  100 1 .¥  4 8 16 32 l  1 2 3 4  + +  +  ...  are  a..  ar 2 ......39  Sequences and Series  1 2 3 4  + + + + ... ¥  8 16 32  =  [2  ×  5  ×  6  ×  7  ×  8  –  12  ×  5  ×  6  ×  7  +  13  3 æ 16 ´ ×  5  ×  6  –  5]  –  2  ç è =  [3750  –  5(505)]  –  2  ×  16  ×  25  25 ö ÷ 4 ø  =  1225  –  800    =  425  26.  (b)  :  S¥  =  2 4 Õ ai  =  a ×  ar  ar 2  ..¥  = 0  +  + 2 8 16 32 64 l 1 1 1 1 1  + + + ..(ii)  =  a 9 r 36  =  (ar 4 ) 9  =  2 9  =  512 ..  (A)  . a  \  S¥  =  where a  =  first  term.  (B)  .  .. 1 ö f(x) = [x] cos ç p .r.  differentiable  in  (2.  (b)  Statement 1 is true.  ì sin( p + 1) x + sin x .  Statement  2  is  true;  Statement  2  is  a  correct  explanation  for  Statement  1. a firm is manufacturing 2000 items. Statement 2 is  false.  DIFFERENTIAL CALCULUS (b)  1  (c)  2  Statement 2 :  a = (d)  –1/4  (2013)  At present.  Statement  2  is  true;  Statement  2  is  not  a  correct  explanation  for  Statement  1. x>0 x  = – 1 and x  = 2.  5. x Î R  8.  Consider the function. x ¹  0  has  extreme  values  at ï x+x . then the new level of production of items is  7.  (d)  Statement 1 is  true.cos2 x )(3 +  cos x )  lim  is equal to  x ® 0  x tan4 x (a)  1/2  2. 5) and f(2) = f(5).  If f  : R ® R  is  a  function  defined  by æ 2 x .  The  values of p  and q  for  which  the  function  continuous  for  every  real x.  (2012)  A spherical balloon is filled with 4500 p cubic metres of helium  gas. Statement 2  is true; Statement 2  is a  correct explanation  for  Statement 1.  (a)  3000  (b)  3500  (c)  4500  (d)  2500  (2013)  3.  continuous only at x =  0. It is estimated  that the rate of change of production P w.  (2012)  x ï f (x) = ï q . then f  is  dy 2  equals to  æ d 2 y ö æ dy ö -2  (a)  çç 2 ÷÷ ç ÷ è dx ø è dx ø æ d 2 y ö æ dy ö-3  (b)  çç 2 ÷÷ ç ÷ è dx ø è dx ø -1  æ d 2 y ö (c)  çç 2 ÷÷ è dx ø  -1  (d)  æ d 2 y ö æ dy ö-3  .40  JEE MAIN CHAPTERWISE EXPLORER  CHAPTER  9  1.çç ÷÷ ç ÷ (2011)  è dx 2 ø  è dx ø æ 1 .  is continuous for all x  in R .  (b)  Statement  1  is  true.2)} ö ÷ lim  çç ÷ x ® 2 è x-2 ø  (a)  equals -  2 (b)  equals  1  2  equals 2  (c)  does not  exist (d)  discontinuous  only at  non­zero integral  values of x.  10.  (d)  Statement  1  is  true.  ïî  x 3/2  Statement 1 : f has local maximum at x = – 1 and at x = 2.  (2012)  9. f(x ) = |x  – 2| + |x  – 5|.  Statement  2  is  false. are  (a)  (b)  (c)  (d)  6. Statement 2 is true; Statement 2 is  not a  correct explanation  for  Statement  1.  5].  then (a)  1  2  (b)  1  dy at x  =  1 is  equal to  dx  (c)  2  (d)  1  (a)  Statement  1  is  true.  Statement 1 : f ¢(4) =  0  Statement  2  : f  is  continuous  in  [2.  If y  = sec(tan –1x).t. b Î R  be  such that  the  function f  given  by í ï 2 f ( x )  =  ln  | x |  + bx 2  + ax . x < 0  ï discontinuous  only  at x =  0.x .  (c)  Statement  1  is  false.  d 2 x (2011)  .  (1 .  (c)  Statement 1 is false.  Statement  2  is  true. If the firm employs  dx 25 more workers. additional number  of workers x is given by  dP = 100 - 12 x .cos {2( x . Statement 2 is  true.  (a)  Statement 1 is  true. x = 0  Let a. where [x ] denotes the greatest integer  è 2 ÷ø  function. then the rate (in metres  per minute) at which the radius of the balloon decreases 49  minutes after the leakage  began is  (a)  2/9  2  1 -1  and b =  2 4 (b)  9/2  (c)  9/7  (d)  7/9  (2012)  (2013)  4. If a leak in the balloon causes the gas to escape at the  rate of 72p  cubic metres per minute.   Then  which  one  of  the  following  holds?  p  p  (a)  The  cubic  has  maxima  at  both  and  –  3  3  p  p  (b)  The cubic  has  minima  at  and  maxima  at  –  3  3  p  (c)  The  cubic  has  minima  at  –  and  maxima  3  p  at  3  p  p  (d)  The  cubic  has  minima  at  both  and  –  3  3  (2008)  If f  has a local minimum at x  = –1.  (a)  5  (b)  7  (c)  Statement­1  is  true.  1) ® R  be  a  differentiable  function  with f  (0) = –1 and f ¢(0) = 1. then  in  the interval  [–1.  Statement­2  is  true. if x > -1 (a)  1  (b)  log 2  (c)  –log 2  (d)  –1  (2009)  18. q =  2 2 17.  (2009)  (c)  2  (d)  –1.  Statement­2  is  true;  Statement­2  is  22.  1]  :  (a)  (b)  (c)  (d)  .  (c)  f  is differentiable at x  = 0 and at x = 1  1  (d)  f  is differentiable at x = 0 but not at  x  =  1  Statement­2 :  0 < f ( x ) £ .1)sin if x ¹ 1  (2010)  19.  Let f : R ® R  be  a  positive  increasing function  with  lim x ®¥ (a)  1  f (3 x ) f (2 x )  = 1.  (2010)  2  2  21.  1 2  (b)  Statement­1  is  true.  Statement­2  is  true;  Statement­2  is  x 7  + 14x 5  + 16x 3  + 30x – 560 = 0  have?  not  a  correct  explanation  for  Statement­1. for all x ΠR.  Let f  : R ® R  be  defined  by  ì k .  (2007) (2009)  real root  of P¢(x) = 0.1  ïî  0 if x = 1  14. q = -  2 2 (a)  p = .  (2007)  4  3  2  16.  Statement­2  is  true. q =  1 3  2 2 5 1  (d)  p = .  Statement­2  is  false.  Let  f  (x)  =  x|x|  and  g(x)  =  sin  x. then a possible value of k  is  (a)  1  (b)  0  (c)  –1/2  (d)  –1  1  ì ï( x .41  Differential Calculus  3 2 1  2 1 3  (c)  p = .  e x + 2 e .  Let  y  be  an  implicit  function  of  x  defined  by  x 2x  –  2x x  cot  y  –  (b)  p = .  2 2 (2008)  (a)  Statement­1 is true.  If p and q are positive real numbers such that p  + q  = 1. if x £ -1  f ( x ) = í î 2 x + 3.  2  2  Statement­2  :  gof  is  twice differentiable  at x  = 0.  Given P(x)  =  x  +  ax  +  bx  +  cx  +  d  such that  x = 0 is the only  23. Then lim  =  f (x) x ®¥ f ( x) (b)  2/3  (c)  3/2  (d)  3  (2010)  13. q =  1= 0.  (2007)  (a)  Statement­1  is  true. Statement­2 is true; Statement­2 is a  20.  Suppose  the  cubic x 3  –  px  +  q  has  three distinct  real  roots  where  p  >  0  and  q  >  0.  (c)  1  (d)  3  (2008)  (d)  Statement­1  is  false. Then  y¢(1)  equals  (2011)  11.  the maximum value of (p + q) is  Statement­1  :  gof  is  differentiable  at  x  =  0  and  its  1  1  (a)  (b)  derivative is continuous  at  that point.  Let f  :  (–1. then  15. g (x ) = [f  (2f  (x ) +  2)] 2 .  f ( x ) = .x (b)  f  is neither differentiable at  x  = 0 nor at  x  =  1  Statement­1 : f  (c) =  1/3.2 x . Then which of the following is  P(–1)  is  not  minimum  but  P(1)  is  the  maximum  of P  true ?  P(–1) is the  minimum but P(1) is  not the  maximum of P  neither P(–1) is  the minimum  nor P(1)  is  the  maximum  (a)  f (x)  is  differentiable  everywhere  of  P  (b)  f (x) is not differentiable  at  x  = 0  P(–1)  is  the  minimum  and  P(1)  is  the  maximum  of  P  (c)  f (x) ³  1  for all  x Π R  (d)  f (x) is not differentiable  at  x  =  1.  (c)  2  (d)  2. for some c Î R .  Let f  : R ® R  be  a continuous  function  defined by  Then  which  one  of the  following  is  true?  1  (a)  f  is differentiable at x = 1 but not at  x  =  0  f (x) = .  If P(–1) < P(1)..  The  function  f  :  R –  {0} ®  R  given  by  not  a  correct  explanation  for  Statement­1.  How many real solutions  does the  equation  correct  explanation  of  Statement­1.  Let  f ( x ) = í x . |x| + 1}.  (b)  Statement­1  is  true. Then g¢(0)  =  (a)  4  (b)  –  4  (c)  0  (d)  –2  (2010)  12.  Let  f  :  R ®  R  be  a  function  defined  by  f (x) = min {x + 1.  Statement­2  is  false.2 x  x  e - 1  (c)  Statement­1  is  false.  can be made  continuous at  x  = 0 by defining  f (0)  as  (d)  Statement­1 is true; Statement­2 is true; Statement­2 is a  (a)  0  (b)  1  correct  explanation  for  Statement­1.   (d)  (2005)  (c)  ( a .  then  function  in  1 .  (c)  (d)  (2005)  2 2  6p  54p  (2004) 3 x 2  + 9 x + 17  is  3 x 2  + 9 x + 7  (c)  1  (d)  17/7.  1)  and  at  that  point the tangent to the graph is y = 3x – 5.  (2005)  fence are  of same  length x.  x ¹  .  3).  –1) È  (–1.  (2007)  2 2  (b)  it  passes  through  the  origin  m  n  m  +  n  26  If x  ∙  y  =  (x +  y)  .b )2 .  2  (c)  ç 4 2 ÷ (d)  ç 2 4 ÷ (2007)  è ø  è ø  1  2  .  A function y = f (x) has a second order derivative f ¢¢(x) = 6(x  – 1).  If  2a  +  3b  +  6c  =  0.2 .  Let a and b  be  the  distinct  roots  of  ax 2  +  bx  +  c  = 0.  If  f  is  a  real­valued  differentiable  function  satisfying  y  | f ( x ) .  (2006)  )  2  x 2  y  + =  1  is  a 2 b 2  (a)  ab  (b)  2ab  (c)  a/b  (d)  ab  (2005)  39.  p .  0)  and  (3.  then  f (1)  27.tan x .b ) 2  (a)  0  (b)  æp.  (2006)  31.  (2006)  2  37. ¥)  (b)  (– ¥.  The  function  f (x)  =  tan –1  (sin  x  +  cos  x)  is  an  increasing  33.  6].b )  2 2  25.  The  set  of  points  where  f ( x ) = is  differentiable.cos( ax 2  + bx + c )  lim  is  equal  to  æ pö æ p pö x ®a ( x .  If  x  is  real.  The  normal  to  the  curve  x  =  a(cos q  + q sin q ).  A triangular park is enclosed on two sides by a fence and on  equals  the third side by a  straight river bank.  then  at  least  one  root  of  the  equation  ax 2  +  bx  +  c  =  0  lies  in  the  interval  (a)  (2.  (2004)  3  of ice of uniform thickness that melts at a rate of 50 cm  /min.  (2006)  38. pö æ .  (2004)  40.  2 ÷ è ø  è ø  a 2  ( a .  A value of c for which conclusion of Mean Value Theorem  holds for the function f (x) = log e  x  on the interval [1.  then  (b)  (a)  x  (a)  f (6) < 8  (b)  f (6) ³  8  2  8  (c)  f (6) = 5  (d)  f (6) <  5. - a . The  two sides having  (a)  1  (b)  2  (c)  0  (d)  –  1. If f (1) = –2 and f ¢(x) ³ 2  3  2  x 3  for  [1.  y Π R  and f (0) =  0.  3)  (b)  (1.  p When the thickness of ice is 5 cm.  Let  f ( x ) = 1 .42  JEE MAIN CHAPTERWISE EXPLORER  24.    is  p p  1  cm/min  1  cm/min  f ( x )  is  continuous  in  éê 0. x Î éëê0. 3] is  34.  Angle between  the  tangents  to  the  curve y  =  x 2  –  5x  + 6  at  the  points  (2. The  maximum area  enclosed by  the  park  is  36.  Suppose  f (x)  is  differentiable  at  x  =  1  and  x  28.p  thickness  of  ice  decreases.  The  function  g ( x ) = (a)  x =  2  (c)  x =  0  x  2  +  has  a  local  minimum  at  2  x (b)  x =  –2  (d)  x =  1.  1)  (d)  (1.  then  f ¢(1)  equals  1+ | x |  h ® 0 h (a)  4  (b)  3  (c)  6  (d)  5.  ùú . then the function  (a)  1/4  (b)  41  is  (a)  (x  +  1) 3  (b)  (x  –  1) 3  2  32.  (2005)  1  2  (c)  x  (d) px 2 .a 2  ( a .  (2005)  (a)  (– ¥.  then f is  ë 2 û  4  (b)  (a)  18p  36p  1  1  5  cm/min  1  cm/min.  (2006)  30.  the  maximum  value  of  ( ) .  ùû  2 ú 4  4 x . ¥). 2 ÷ (a)  ç 0.  then  dy/dx  is  (c)  it  is  at  a  constant  distance  from  the  origin  x +  y  y  p (a)  (b)  xy (2005)  (d)  it  passes through a 2 . 0) È  (0.  is  lim 1  f (1 + h ) =  5.  Let f  be the differentiable for "  x.  y  =  a(sinq  – qcosq)  at  any  point q  is such  that  (a)  log 3 e  (b)  log e 3  p 1  (a)  it makes  angle  + q  with  x­axis  (c)  2 log 3 e  (d)  log e  3. then the rate at which the  41.p . ¥)  (d)  (0.  (a)  -  (b)  (c)  1  (d)  –1. x.  Area of the greatest rectangle that can be inscribed in the ellipse  ( 29. ¥)  (c)  (– ¥.  0)  is  (a) p/2  (b) p/3  (c) p/6  (c) p/4.  x  x  (c)  xy  (d)  .  2)  (c)  (0. p ö .f ( y ) | £ ( x - y )2 .a ) 2  (b)  ç .  A  spherical iron ball 10  cm  in radius  is  coated  with  a  layer  (c)  (x  –  1)  (d)  (x  +  1) 2 .  (2006)  35. If its graph  passes through the point (2.   If  lim ç1 + x  + 2  ÷ = e  .  (c)  f(x)  is  continuous  everywhere  (2003)  (d)  f(x)  is  discontinuous  everywhere.  æ at ö y = a cos t .  ì x.  If  f  (x)  =  x n .  (2003)  53.g ¢¢ ( x ) = 0.  (2003)  48.. then  f ¢(5) is  (a)  0  (b)  1  (c)  6  (d)  2.  The  maximum  distance  from  origin  of  a  point  on  the  curve  value  of  the  sum  at  x  equal  to  (a)  1  (b)  –1  (c)  –2  (d)  2.  æ at ö x =  a sint  –  b sin è ø  (2003)  b ì -æç 1 + 1 ö÷ | x|  x ø ï ..  2]  such  (a)  1  (b)  2  (c)  1/2  (d)  3.2 f ( x )  52. x = 0  (a)  continuous  for  all  x. + n  is  n®¥ n ®¥ n n (a)  zero  (b)  1/4  (c)  1/5  (d)  1/30.  (2002)  1 - cos 2 x  is  2 x (a)  1  (c)  0  55.  y and f (5) = 2.  If 2a + 3b + 6c = 0 (a.  Let f (2) = 4 and  f ¢(2) = 4 then  lim  equals  x - 2  x ® 2  f (a ) g ( x ) . b..tan( x / 2) ][1 .  If  f (1)  =  1.  then  f  (x)  is  47.  (2003)  54.  (2002) .  both  a..g (a ) f ( x ) + g (a )  lim = 4.  b  >  0  is  b (a)  a  –  b  (c)  2 a + b (b)  a +  b  2  (d)  57.  except  x  =  0  50.  1)  (a)  a Π R.b cos è ø ..  If  f ( x ) = í xe è ïî  0 .  f ( x ) .  (2002)  46. where a > 0.  (2002)  attains  its  maximum  and  minumum  at  p  and  q  respectively  2  such  that  p  =  q.  f ¢(1) f ¢¢(1) f ¢¢¢ (1) (-1)n f n (1)  (2002)  is  f (1) + + .  b Î R(d)  a =  1  and  b  =  2.  5]  (2004)  (d)  none  of  these  (2002)  43.f ( a ) .43  Differential Calculus  2 x  51.  but  not  differentiable  at  x  =  0  (b)  neither  differentiable  not  continuous  at  x  =  0  (c)  discontinuous  everywhere  (d)  continuous  as  well  as  differentiable  for  all x.  then  a  equals  58.  g n  (a)  exist  and  are  not  equal  for  some n.  lim  æ x + 5 x + 3 ö ç ÷ 2  x ®¥ è x + x + 3  ø 1 .x )  (a)  f(x)  is  continuous  at  every x.  if x  is rational and  f ( x ) = í (2003)  î .  (2003)  45. is  3  3  x ®p / 2 [1 + tan( x / 2) ] [ p - 2 x ]  (c)  e  (d)  1..  The  value  of  4 4 4 3 3 3  lim 1 + 2 + 3 5+ .  If f (x + y) =  f (x) ×  f (y) "  x. are  ax 2  +  bx  +  c  = 0  has  x ®¥ è x ø  (a)  At  least  one  in  (0.log(3 . c Π R) then the quadratic equation  æ a b  ö 2  42.  Further  if  x f (2) ..  if x  is rational.  If the function f (x) = 2x 3  – 9ax 2  + 12a 2 x + 1..  lim  x ® 0  (b)  –1  (d)  does  not  exist.  (2002)  (a)  0  (b)  1/32  (c) ¥  (d)  1/8.  Let  f  (a)  =  g(a)  =  k  and  their  n th  derivatives  f  n  (a).  b  =  2  (b)  a =  1.  f (2) = 3g (2) = 9  (2003)  then  f (x)–  g(x) at  x  =  3/2  is  (a)  0  (b)  2  49.  The real number x when added to its inverse gives the minimum  56.  f (x)  and  g (x)  are  two  differentiable  function  on  [0.  5]  as  (a)  2 n  –  1  (b)  0  (c)  1  (d)  2 n .  except  x  = 0  (a)  –1/3  (b)  2/3  (c)  –2/3  (d)  0. f ¢(0) = 3.  g ( x ) -  f ( x )  (a)  2  (b)  – 2  x ® a  (c)  – 4  (d)  3.  then  the values  of  a  and b.  f ¢ (1)  =  2. x ¹ 0 .  the  value  of  k  is  x x ® 0  (b)  f(x)  is  discontinuous  at  every  x.1  x - 1  is  (b)  4  (d)  1/2. f ¢(1) = 2 g ¢(1) = 4  .  b Π R  (b)  At  least  one  root  in  [2.  then  Lt  x ®1  (a)  2  (c)  1  (2002)  a 2 - b 2  . +  1! 2! 3! n ! 59. Then  log(3 + x) . + n -  lim 1 + 2 + 3 5 + .  If  lim = k .  (2002)  then  the  value  of  k  is  1  (a)  2  (b)  1  2  x  (c)  0  (d)  4.  t h a t  f ¢¢ ( x ) .  3]  (c)  a Π R.  then  the  value  of  (c)  10  (d)  5..  (c)  At  least  one  root  in  [4.x.sin x ]  [ (a)  e 4  (b)  e 2  lim  44.  f  is  defined  in  [–5.   (b)  (b)  (a)  (c)  (a)  (a)  (b)  (b)  (b)  (a)  3.  45.  16.  37.  13.  44.  56.  54.  34.  32. ([x] denotes greatest integer less than  [ x ]  x ® 0  or  equal  to  x)  (a)  has value  – 1  (b)  has  value  0  (c)  has  value  1  (d)  does  not  exist  (2002)  60.  43.  42.  29.  25.  26.  35.  19.  15.  (d)  (c)  (b)  (c)  (c)  (b)  (c)  (c)  (a)  (a)  4.  20.  58.  9.  27.  52.  Answer  Key  1.  If  y = ( x + 1 +  x 2  ) n .  48.  14.  18.  47.  8.  lim  2  61.  59.  60.  50.  33.  57.  11.  39.  40.  31.  (a)  (a)  (a)  (b)  (c)  (a).  28.  41. (c)  (b)  (a)  (c)  (d)  5.  61.  22.  (a)  (a)  (b)  (d)  (a)  (b)  (b)  (b)  (c)  (d)  (2002)  .  then  (1 + x )  (a)  n 2 y  (c)  –y  d 2 y dy  +  x  is  dx  dx 2  (b)  –n 2 y  (d)  2x 2 y.  51.  12.  17.  49.  23.  21.  24.  (c)  (b)  (d)  (a)  (a)  (c)  (a)  (a)  (d)  (b)  6.  36.  46.44  JEE MAIN CHAPTERWISE EXPLORER  log x n  - [ x ]  .  10.  30.  (c)  (a)  (d)  (b)  (c)  (b)  (d)  (d)  (b)  (a)  (a) 2.  55.  7.  n ΠN .  38.  53.   (c) : Let  x = 2 +  h  lim h®0 x ® n+ è dx ø 1 .  (a) :  P(0) = 2000 = l.  x  = 2 \  Statement 1 : f has local maxima at x = –1. True  íç ÷ ý 8. \ l = 2000  P(25) =100 × 25  – 8 × 25 3/2  + 2000 =  3500.1 ö p 2 ÷ø  5. LHL = –1. radius decreases at a rate of  m/min  9  Þ 972 p = 2  ì -1 ü d x d æ dx ö d ï æ dy ö ï Statement­1 : f ¢(4) =  0.45  Differential Calculus  (1 . x = 0  ï 2  x+x . x>0 ïî x 3/2  .  íç ÷ ý × = =-ç ÷ ×ç ÷ ï dy dx ïî è dx ø þ  æ 2 x .  dP = (100 - 12 x ) dx 2  3 / 2  ×x + l  3 and 1 1  + 4 b + a = 0  Þ a =  2 2 1 x2 + 2b = - 1 x2 - æ 1 1 ö 1 = -ç + < 0  è x 2  2 ÷ø  2 1 2 + l  dy 1  = sec(tan -1 x ) × tan(tan -1 x ) × dx 1 +  x2  dy 1 1  = 2 × 1 × =  dx x = 1  2  2 4.7.72 p m 3  / min. lim f ( x ) = 0 x ® n- \  f(n) = 0 Þ  f(x) is continuous  for  every  real  x. x>5 î  1  4 \ Statement 2 :  a = .  f ( x ) = [ x ] cos çè æ pö = [ x ] cos ç px .2 b + a = 0  Þ b = -  4 æ 2sin 2 x ö æ x ö = lim ç ÷ (3 + cos x )  ÷×ç x ® 0 è x2  ø  è tan4 x ø P = 100 x .x ï .  ì ï sin( p + 1)x + sin x . b = -  7.  (a) : f(x) = |x – 2| + |x – 5|  ì7 .8 x f ¢(– 1) = 0 and f¢(2) = 0  f ¢¢( x) = - 1  = 2 ´ ´ 4 =  2  4 3 / 2  1  + 2bx + a x 1  Þ .  (a) :  f ( x ) = í q. x ¹  0 has extreme values at  x = – 1.  dx 2 è dx ø  -3  æ dy ö d 2 y =-ç ÷ è dx ø  dx 2 9. v  =  v 0  + 49∙ = 4500p  – 49 × 72p  dt  v= = 4500p  – 3528p = 972p  4  3  pr Þ r 3  = 243 × 3 = 3 6 Þ  r  =  9  3 dr dr 18 2  \ .72 p = 4 p ´ 81 ´  Þ == -  dt dt 81 9 2  Thus. 2 £ x £ 5  ï 2 x .12 × Þ f ¢ ( x ) = dv = .1 .  differentiable  in  (2. we  have.  6.  (c) :  xlim  ® 0  1 . v0  = 4500 p  dt 4 3 dv 4  dr pr \ = p ´ 3 r 2  ´  3 dt 3 dt dv After 49 min.  lim f ( x) = 0.2 x . Thus limit doesn’t exist.cos 2 h |sin h |  =  lim h h h ® 0  RHL = 1.  5].  (d) : y = sec (tan –1 x)  Þ [Given]  for all  x Π R – {0} Þ  f  has a local maximum at  x = – 1. True  -1 -2 -1  d ïì æ dy ö ïü dx æ dy ö d 2 y æ dy ö But Statement 2 is not a correct explanation for  statement 1.÷ =  [x] sin px  è 2 ø  Let  n be an integer. x < 0  x ï ï 10.cos2 x )(3 +  cos x )  x(tan4 x) 1.  (a) : f(x) = ln|x| + bx 2  +  ax.  (c) : f :  R ® R. 5) and  f(2) = f(5).  (b) :  dx P = 100 x . x = 2 dP = 100 - 12 x 2.cos2 x = lim (3 + cos x )  x ® 0  2  æ tan4 x ö x ×ç ÷ è x ø  Integrating. x = 2.  3.  (b) :  2 = dy çè dy ÷ø  = dy ï dy îï è dx ø þ  Statement­2  :  f  is  continuous  in  [2. x < 2  ï f ( x ) = í 3.  we have  D of 4x 2  + 3ax  + 2b  is less than zero  i.  1 – 2 cot  y  – 1 = 0 15. of  P(x) =  P(1)  But minimum of P(x) doesn’t occur at  x  = –1. e + 2e ³  2 2 2 1 1  £ Then  x e + 2 e .  = lim h ® 0  11. x  = 0  ï 2 x cos x 2  .sin x .  x.  12.cosec 2  y ) + cot y × x x (1 + ln x )] =  0  dx Let  the composite  function  gof (x)  be  denoted  by H(x). x  < 0  Again H ¢¢ ( x )  = í 2 2 2  . \ . x > 0  î  H¢(x) is continuous at x = 0 for  H¢(0) = LH¢(0) =  RH¢(0)  ì-2 cos x 2 + 4 x 2 sin x 2  .  (a) : Using A.M..e. P(–1)  is  f (c) = 1/3. b 2  – 4ac < 0 then ax 2  + bx + c > 0 " x ΠR)  So P¢(x) < 0 if x Π[–1. x ³ 0  î  2 cos x . we  have  f (x) < f (2x) < f  (3x)  Dividing by f  (x) leads to 1  < As  lim x ®¥ f (2 x ) f (3 x )  <  f ( x) f ( x) f (3 x )  = 1.  (b) : gof (x) = g(f (x)) = sin(x|x|)  Þ  cot  y  = 0 \  y = p/2  2  Differentiating (i) w.e..  17.  Let x = 0  g¢(0) = 2f  (2f (0) + 2) ∙ f ¢(2f  (0) + 2) ∙ 2f ¢(0)  = 2f  (0) ∙ f ¢(0) ∙ 2f ¢(0) = (–2)(1)(2) = – 4. x ³ 0  Again. we have by  Squeez  theorem  f ( x) f (2 x )  or Sandwich  theorem.h) .x x -x ³ e x × 2 e .× 2  × h  = 1 × 0 =  0  - h h h ® 0  RH ¢ (0) = lim  g¢(x) = 2f  (2f (x) + 2) ∙ f ¢(2f (x) + 2) ∙ 2f ¢(x)  H (0 .x .e.­G.  (d) :  lim f ( x) = 1 x ®1 + h ® 0 + As f  has a local miminum at x = –1  f (–1 + ) ³ f  (–1) ³ f (–1 – ) Þ  1 ³ k  + 2 k £  –1  Thus k = –1 is a possible value. Thus such that  Thus H(x) is NOT twice differentiable at  x  = 0  16.  lim = 1. x  < 0  Then  H ( x )  = ìí 2  î sin x .  14. p + 2 = q =  1/2 \  p = –3/2..  Thus. decreasing  and P¢(x) > 0 if x Π (0. inequality.sin h 2  sin h 2  = lim. 0)  i.H (0)  - h 1  2 2  Observe that  f  (0) = 1/3..2[ x x ( .  not the minimum.  (a) : P(x) = x 4  + ax 3  + bx 2  + cx + d  P¢(x) = 4x 3  + 3ax 2  + 2bx  +  c  P¢(0) = 0 Þ  c  = 0  Also P¢(x) =  x(4x 2  + 3ax  + 2b)  As P¢(x) = 0 has no real roots except  x  = 0. x  < 0  =í 2  dy  î sin x . we can say that as f is continuous.H (0)  h æ sin h 2  ö sin h 2  . Here  we are  able to  At x  = 1 we have  identify the  point  as  well. (3a) 2  – 4 ∙ 4 ∙ 2b < 0  then 4x 2  + 3ax  + 2b > 0 "  x Π R  (If a > 0..  (d) : x 2x  – 2x x  cot  y  – 1 = 0  .x 1 x x e + 2 e -x is always positive.e.0  = lim  ç 2  ÷ h  ø  h h ® 0 + h ® 0 + è h =  lim  = 1 ∙ 0 = 0  Thus  H(x) is differentiable at  x  = 0  ì-2 x cos x 2 .  lim f ( x ) = - x ® 0  sin( p + 1) + sin x = p + 2 x LH ¢ (0) = lim  h ® 0 - Now. x ³ 0  2 x 2 x (1 + ln x ) . p/2) we  have .t. increasing  Max.  e x + 2 e .46  JEE MAIN CHAPTERWISE EXPLORER  lim f ( x ) =  + x ® 0  1  2 sin x 2 .  x ®¥ f ( x ) 13.  (a) : As f is a positive increasing  function.  i.  Using extreme­value theorem.  (b) :  g(x) = {f (2f  (x) + 2} 2  We have  on differentiation  with  respect  to x..M. x  < 0  ï Also H ¢ ( x) = í 0 .. q  = 1/2.  At P(1.(i)  f  will attain a  value  1/3  at some  point..x 2 2  As  0 < 1  e + 2 e .r. we have  ì. 1]  i. we have  £ H (0 + h) .4 x sin x LH¢¢(0) = –2 and RH¢¢(0) = 2  As f  (–1) =  k + 2  Þ  k + 2 £ 1. M.ny ö mx + nx .f ( a ) f (3) .e.2 æ 0  ö ç form ÷ x ®0 ( e ø . f  (x) is an strictly increasing  function.  + 0] = 0  P  = 0  2  ù é1 22.47 Differential Calculus  2(1 + ln1) .my ..M ³ G.  (b) : Denote x 3  –  px +  q by f  (x)  i.1 . Hence it is not differentiable at  x = 0.1) sin .my  ÷= dx çè y(x + y) x ( x + y )  ø  Þ dy æ nx .  (c) : Let  2x 24. x ΠR  Hence f (x) is differentiable for all  x Î R. i.0  1  (1 + h .2 x  æ 0  ö ç form ÷ x ®0  x ( e2 x  - 1)  è 0  ø = lim P  18.1)  = lim =  lim sin  h h h®0 h ® 0  As the limit dosen’t exist.  x  we  get  i.  (c) : Let p = cos q.  f ¢( c ) = f (b ) . f (x) = x 3  – px  + q  Now for expression.f  (1)  f  ¢ (1) = lim . 4 ÷ .2 .e.r.  (d) :  f ¢( x ) = f ¢( x )  = 1  .(cos x .f  (0)  Again  f  ¢ (0) = lim  .  By using A.cos x > sin x 2 4  æ p pö Hence y = f (x) is increasing in  ç . 3  f (1 + h ) .2[1(-1) Þ 2+ 2 \ ( )  dy dx ( )  dy dx ( )  dy dx 1  p 2 + q 2  ³  pq Þ  pq £ 2  2  (p + q) 2  =  p 2  +  q 2  + 2pq Þ  ( p + q ) £  2.t.sin x  2 + sin 2 x If f ¢(x) > 0 then  f (x) is increasing function  For  .  21.  19.  è ø  25. if  the limit  exists h h ® 0  1  ( h .  (a) : f (x) = min {x + 1. f ¢(x) =  0. 3 3  f ¢¢(x) = 6x  By using L’ Hospital rule  2 e 2 x  .  c 2 2log 3 e 26.log e 1  1  = log e 3  2 2  Þ  1 1 1  = log e 3 = \ c = 2 log 3 e.e. |x| + 1} f (0) = lim p ö ÷ > 0  3 ø  p and minima at  3  p  .  (b) :  f (0) = lim ê x  .my ö y y dy y  = = Þ = .  x ®0  4 e + 4 xe2 x  23.1 û  P  = - 1  e2 x  . q = sin q  Þ dy æ n m + n ö m + n m  = dx çè y x + y ÷ø  x + y x Þ dy æ nx + ny .  dx çè nx .  f (x) =  x 7  + 14x 5  + 16x 3  + 30x  – 560 2x Þ f (x) = x + 1.mx .1) sin .  3x 2  –  p  =  0  p p  .  if the limit exists.  m n dy m + n  æ dy ö + = 1 + x y dx x + y çè dx ÷ø  so it can have at the most one solution. h h ® 0  f (1 + h ) -  f  (1)  \  lim  h h ® 0  1  (1 + h .  (c) : By LMVT.p < x < p .sin1  f ( h ) f  (0)  h .1) + 2 xe2 x  è 0  f (0) = lim x = -  Again use L’ Hospital rule  æ p ö æ f ¢¢ ç ÷ < 0  Þ  f ¢¢ ç è è 3 ø Thus maxima at  -  4 e 2 x  = 1. \it is not diffentiable at x = 1  f ( h ) . It can be shown that  it has  exactly one  solution.  (a)  :  x m  ×  y n  =  (x  +  y) m  +  n  Taking  log  both  sides  we  get  \ f ¢(x) = 7x 6  + 70x 4  + 48x 2  + 30 m  log  x  +  n  log  y  =  (m  +  n)  log(x  +  y)  Þ f ¢(x) > 0 "  x ΠR  Differentiating  w.sin x )  1 + (sin x + cos x ) 2  cos x .1  \  lim =  lim  h h h® 0 h ® 0  But this limit dosen’t exist.f (1)  = b-a 3 - 1  f ¢( c ) = log e 3 .  (b) :  By definition  20.2 x  ú x ® 0 ë e .my ÷ø x x dx x 0 £ q £ p/2  p + q = cos q + sin q Þ maximum value of  (p +  q) =  2  Second  method  . b ) ö ù ÷ú ê sin ç 2 2  2  ø ú ´ a ( x .  3 x 2 + 9 x + 7  px 2  + qx +  r [find  the  solution  of  the  inequality  A  y 2  +  B  y +  K ³  0]  where A  =  q 2  –  4pr  =  –3. x  < 0  ï1 .b ) 2 .b )  ù a ê ú 4  ú ëê û  2  30.a )( x .5 = 1  è dx ø  y = 0  since m 1 m 2  =  –1 Þ  tangents are  at  right  angle  i. (c)  :  [ y .  p  2 x  2  + 2  x 1 2  g ¢( x )  = -  2  \  2  x for  maxima  and  minima  g¢(x)  =  0 Þ  x  =  ±  2  4  Again  g ¢¢( x ) = 3  > 0 for x  =  2  x <  0  for  x  =  –2 \  x  =  2  is  point  of  minima  31.b) ù 2sin 2  ê úû a 2 ( x .a ) 2  é a ( x .a )( x .  (b)  :  As a is  root  of  ax 2  +  bx  +  c  =  0 \  aa 2  +  ba  +  c  =  0.  Now  equation  of  normal  to  the  curve 34.  B  =  4ar +  4PC  –  2bq  =  126  é æ a ( x .  (b)  :  For  the  range  of  the  expression  = y  = ax 2  + bx + c  æ 2  ö 2sin 2 ç ax + bx + c ÷ 2  è ø = lim  x ® a ( x . x ³ 0  î 1 + x A T  C  ì 1  .a (sin q .b )  = lim  ê è a ( x .  (a).  solve  –3y 2  +  126  +  y  –  123 ³  0 Þ  3y 2  –  126y  +  123 £ 0 Þ  y 2  –  42y  +  41 £  0 Þ  (y  –  1)(y  –  42) £  0 Þ  1 £  y £  42 Þ  maximum  value  of  y  is  42  4  3  32.b ) ú 2  x ® a ê ú 2  ëê û  =1´ a 2  ( a .1 £ sin 2 a £ 1  2 2  1  2  \ Maximum are of  DABC =  x 2  x  28. 0). .a ) 2  x ® a 29.  (c) :  AT =  x sin a  BT  =  x  cos a  Area  of  triangle  1  ABC  =  base × height  2  1  =  (2 BT )( AT )  2  a  1  2  = (2 x cos a sin a )  B  2  1 2 1  2  =  x sin 2 a £ x  as  .b ) 2  2 ë = lim  ´ 2 2  4  x ®a 2  é ( x .a (cos q + a sin q)) sin q  Þ x cos q + y sin q =  a (1)  Now  distance  from  (0..  18p  33.  (c)  :  Given  f ( x )  = 1+ | x |  ì x  . x ³ 0  ïî (1 + x ) 2  f ¢ (x )  is  finite  quantity  " x ΠR \ f ¢ (x )  is  differentiable  " x Î ( -¥ . x  < 0  ï (1 .e. (3.q cos q ) ]  =- cos q ( x .    Now  ( lim  1 . given points (2.  (a)  :  Let  g ( x )  = 3 x 2 + 9 x + 17  )  .  (a) : Given equation y = x 2  – 5x +  6.a )( x .  2  dy dy  d q = × = tan q = slope of tangent  dx d q dx \  Slope  of  normal  to  the  curve  =  –  cotq  =  tan  (90  + q).  0)  dy  = 2 x - 5  dx æ dy ö = 4 .cos( ax 2  + bx + c )  ( x .a )  distance (d) =  1  \  distance  is  constant =  a.x ) 2  ï Þ f ¢ ( x )  = í ï 1  .  è dt øat  y  = 5  4 p (15) 2  dt = 1  cm/min.a ) ( x .5 = -1  say  m 1  = ç dx ÷ è ø x = 2  \ y = 0  and  m2 = æç dy ö÷ at  x = 3 = 6 .48  JEE MAIN CHAPTERWISE EXPLORER  27. ¥ )  K  =  b 2  –  4ac  =  –123  i.  (a)  :  v = p( y + 10)  where  y  is  thickness  of  ice  3  10  10 + y  Þ dy  dv  = 4 p (10 +  y ) 2  dt dt æ dy ö 50  dv  = ç ÷ as  = 50 cm 3  / min.e.  0)  to  x  cosq  +  y  sinq  =  a  is  (0 + 0 .x  Þ f ( x )  = í ï x  .   2y  (1 .  (b)  :  Any  point  on  the  ellipse  2  x 2  y  + 2  =  1  2 a b is  (acos q .p  41.  1) 1  =  (2 –  1) 3  +  c 1 Þ  c 1  =  0 f (x) =  (x  –  1) 3  Þ  Lt  Lt  x ® p/4  2x  æ d 2 A ö dA p = 0 Þ q =  and   ç 2  ÷ = -8 ab sin 2 q d q 4  è d q ø q = p / 4  2  as  d A  < 0.  (b)  :  Let  if  possible  f ¢(x)  =  2  for Þ f (x)  =  2x +  c  (Integrating  both  side  w. \  sides  of  rectangle  are  2 2  Required  area  =  2ab.tan x  putting  4x  – p  =  t x ® p/4  4 x .  \  Area  is  maximum for q  = p/4.f ( y ) £ ( x -  y ) 2  lim x® y =  f ( x ) .f ( x )]  ax 3 bx 2  Þ  Lt  =  4 Þ + +  cx 39.H.  1)  40.x ÷ ú ø û  ë è 4  æp ö tan ç .  (b)  :  Given  f ² (x)  =  6(x  –  1) f (x) =  (by  integration) Q  f (0) = 0  Þ f ( x ) (" x Î R ) = 0  \  f (1)  =  0.  bsinq )  so  the  area  of  rectangle inscribed  in the  ellipse  is  given  by  A =  (2acosq) (2bsinq)  \ A = 2 ab sin 2 q Þ dA  = 4 ab cos 2 q d q  Now  for  maximum    area  6( x - 1) 2  +  c 2  é Q  f ( x ) = y = 3 x + 5 ù 3 =  3 +  c ê f ¢ ( x ) = 3 " x Î R ú c  =  0  ë û 2 f ¢ (x) =  3(x  –  1)  f (x) =  (x  –  1) 3  +  c 1  as  curve passes  through  (2.x ÷ è 4  ø  2 x  a b  ö æ 42.÷ a b  è 4 2 ø f (0) =  0 and  f (1)  =  + + c 3 2  Lt  f (a)  =  4  x ®  a .  æ p x ö ax 3 bx 2  tan ç . f  (x)  =  0 ( f ¢ ( x ) < 0.sin x )  + + cx called  Here  integration  of  ax 2  +  bx  +  c  is  è 4 2 ø 3 2  44. not possible )  Þ f ( x )  =  k Þ f (x)  =  0  ax 3 bx 2  + + cx =  0 3 2  \ at least one root of the equation  ax 2  +  bx +  c = 0  lies  in  (0.cos ç .f ( y )  £ lim  x .2 x ö 2  4.  (c)  :  Given  f ( x ) . d q 2  2 a 2 b  .2 x )  given  condition  then  at  least  one  root  of  the  equation  ax 2  +  è 4  ø  bx  +  c  =  0  must lies  in  that  interval.  f ¢ (x) =  Þ  Þ  so  Þ  Þ  \  37.  (d)  : As  f (x)  is  differenatiable  at  x  =  1  f (1 + h )  5 =  lim  assumes  0/0  form  h h ® 0  f ¢ (1)  \  f ¢(1)  =  5.  –2  =  2  +  c Þ c  =  – 4 \  f (x)  =  2x  –  4 \ f (6)  =  2 ×  6  –  4  =  8 \ f (6) ³  8.f (a) .  Now  use  the intervals  in f (x) if f (x)  satisfies  the  x ®p/2  æ p .t.x ÷ ax 3 bx 2  è 4 2ø è 2  ø Now  f (x) =  + + cx Lt  3 2  x ®p/2  æ p x ö ( p 2 x ) 2  4.÷ (1 . ç .49  Differential Calculus  2a + 3b + 6 c 6  =  0 given  2a  +  3b  +  6c  =  0 \  x  =  0  and  x  =  1  are  roots  of  35.  (c)  :  Let  f (x)  =  x ® a  g ( x ) -  f ( x )  3 2  Note : In such type of  problems we always consider f (x)  as  k  =  4  the integration of L.x ÷ ´ (1 + tan x )  4  ø è Lt  =  –1/2  x ® p/4  æp ö 4 ç .  x) \ f (1) =  2 +  c.r.  (a)  :  \  38.tan x ) ´ (1 + tan x )  é æp öù (1 + tan x) ê -4 ç .  (b)  :  Lt  it by f (x).  36.  æ p x ö æp ö tan ç .  5 = lim  h ®0  1  1 .  (d)  :  Lt  x ® a  f (a) g ( x ) .g (a) f ( x ) + g (a )  =  4 g ( x ) -  f ( x )  f (a)[ g ( x) .y  x -  y x ® y  Þ f ¢( x ) £  0 .S of the given  equation without constant. ç ÷ ( p .÷ 1 .  (b)  :  e 2  =  ç 1 + + 2  ÷ x  x ø  è e 2  =  e (1¥  form)  é a b  ù Lt ê1+ + -1ú (2 x )  x  x 2  û  x ®¥ ë e 2  =  e 2a Þ  2a = 2 \  a  =  1  and  b Π R  43. log(3 -  x )  =  k x xö x ö æ æ log ç 1 + ÷ ..  (b)  :  =  \  f ¢ (x) =  1 –  1/x 2  and  f ² (x)  =  2  x 3  now  f ¢ (x) =  0 Þ x  =  ±  1 \  f ²(1)  >  0 Þ  x  =  1  is  point  of  minima.  6  As f (0) =  f (1) = 0 and f (x) is continuous and differentiable  . + n 3  n5  2 n( n + 1)(2 n + 1)(3n 2  + 3n .  ì -æ 1 + çè | x |  ï 47.÷ 4 2 æ p .  +  C n x n  .2 f (2)  x - 2  ( x ..  +  1! 2! 3!  n !  n C  –  n C  +  n C  . ç .(i)  Putting  x  =  –1  in  both  side  of  (i)  we  get  0 =  C 0  –  C 1  +  C 2  –  C 3  +  .(A)  2a + 3b + 6 c =  =  0  given.1)  n  =  f ² (x) =  n(n  –  1)x n  –  2  so  =  C 2  2! 2!  f n (x) =  n(n  –  1)  .1 \  2  =  a 2  =  2a Þ  a 2  –  2a  =  0 a(a  –  2) =  0 Þ  a  =  0.2) f (2) . x = 0  =  0  é 1 1 ù ê .H.÷ 3ø 3 ø  è è k =  Lt  x ® 0  x xö x ö æ æ log ç 1 + ÷ log ç 1 .2 x ö ç ÷ è 4 2 ø è 4  ø  14 + 24 + 34 + .50  JEE MAIN CHAPTERWISE EXPLORER  æp xö æ p x ö tan ç .h )e  è h h ø .log ç 1 .+  (–1) n C  0  1  2  n  Now  (1  +  x) n  =  C 0  +  C 1 x  +  C 2 x 2  +  ....1) ù =  ú 30  úû  50.2[ f ( x ) .D  at  x  =  0  Lt  (0 + æ 1 1 ö -ç + ÷ h) e  è h h ø x ® 0 + æ 1 1 ö -ç .x + x ú û ë and  Lt ..  2a and  f ² (a)  <  0  and  f ² (2a)  >  0  Now  p =  a  and  q  =  2a  and  p 2  =  q f n (1)(- 1) n  =  (–1) n  n C n n!  f ¢(1) f ¢¢(1) f ¢¢¢(1)  (- 1) n f n (1)  + -  =  +  . x ® 0  =  0  As  LHL  =  RHL  \  f (x)  is  continuous  at  x  =  0  x ® 0  x ® 0  Again RHD  at  x  =  0  is  - 0  =  0  h also  we  have  L.  (b) : For maximum  and  minima  f ¢ (x)  =  0 Þ  6x 2  –  18ax  +  12a 2  =  0  and  f ² (x)  =  12x  –  18a  f ¢ (x) = 0 Þ  x  =  a.  (a) :  Given  f (x)  =  í x e ï0 î  Lt x ® 0+ f ( x ) =  Lt  xe .  xf (2) ... \  By  Rolle’s  theorem  f ¢ (x)  =  0 Þ  ax 2  + bx + c = 0 has at least one root in the  interval  (0.  (c) :  Lt  n  ® ¥ n  ® ¥ n5  Þ  Þ  13 + 23 + .÷ 4 2 1 2  1  è ø è 4 2 ø Lt  =  ´  =  2  p x  x ®p/2  æ ö 4 16 32  4..  (a) :  Let  us  consider  f (x)  =  +  cx 3 2  a b \  f (0) =  0 and  f (1) =  +  +  c  3 2  ..(B)  also  in  [0.÷ 3ø 3 ø è è + Lt  k  =  Lt x x x®0 x ® 0  ´3 .  1].  (b)  :  f (x) =  x n \  f (1) =  1 =  n C 0 \  f ¢ (x) =  nx n  –  1  so  –f ¢ (1)  =  –  n  =  – n C 1  f ¢¢(1) n(n .2/ x  x ® 0 + 1 ö x ÷ø .0  is  =  1  - h so  L.H..  (a)  :  f (x) =  x +  1/x  49...  \  é ..  1).  (d)  :  We  have  lim ç 2  ÷ x ®¥ è x + x + 3  ø  1 / x  æ 1 + 5 + 3  ö ç x  x 2  ÷ = lim ç 1 3  ÷ x ®¥ ç 1 + + 2  ÷ è x  x ø = 10  = 1  ..D  at  x  =  0 \  f (x)  is  non  differentiable  at  x  =  0  48..f ( x ) =  Lt ..0 êUsing  1 4  +  2 4  +  . + n 4  -  Lt  45.H.÷ 2 sin 2  ç .Lt  n (n + 1)  n ® ¥ 4 ´ n 5  n ® ¥ 30 × n 5  =  Lt  n( n + 1)(2 n + 1)(3n 2  + 3n - 1)  30  3 n( n + 1)(6n + 9 n 2  + n .D ¹  R.f (2)]  Lt  x ® 2  x - 2  52...÷ (0 ....´ 3  3 3  1 1 2  k  =  + =  3 3 3 ax 3 bx 2  +  51.  a  =  2  f (1) –  Lt  x ® 0  log(3 + x ) .e x®0 x  ¹ 0  ..2 f ( x ) + 2 f (2) .1)  ..  +  n 4  ë  6  - 0  30 1  =  5  46.  (c) :  Lt  x ® 2  lim  [f (2)  –  2  f ¢(x)]  =  x®  2  =  4 –  2  ×  4 =  –  4  1/ x  æ x 2  + 5 x + 3 ö 53. ÷ b bø è ø ø  è =  a 2 + b 2  .  x  at  ì ï a sin t .b cos  = y  b î x 2 +  y 2  =  AB  =  at ö æ at ö ö æ + cos2  ç ÷ ÷ .  (d) : As  f ¢¢ (x) –  g ¢¢ (x)  =  0 Þ  f ¢ (x) –  g¢(x) =  k  f ¢ (1)  –  g¢(1)  =  k \  k  =  2  So  f ¢ (x) –  g¢(x) =  2 Þ  f (x) –  g(x) =  2x  +  k 1  f (2) –  g(2)  = 4  +  k 1  k 1  =  2  So  f (x) –  g(x) =  2x  +  2 [ f ( x) -  g ( x)]  3  =  2 ´ 3  + 2  =  5  \  x = 2 2  59.  B(x.  (a)  :  Lt x ®0 56.  (c) :  Given  f (x  +  y)  =  f (x)  f (y) \ f (0 +  0) =  (f (0)) 2 Þ  f (0) =  0 or  f (0)  =  1  but  f (0) ¹  0  f ( x ) f (h ) -  f ( x )  f ( x + h ) -  f ( x )  Now  f ¢(x) =  Lt  =  h Lt  ® 0  h h ® 0  h f (h ) .1 é x  ù ê1 + ú êë 1 + x 2  úû  n  1  2  y 1  =  n éê x + 1 + x  ùú .  (a)  :  Lt  x ® 1  f ( x ) .1  x - 1  1 ´  2 ´ 1 ´ 2  =  2  2 58.  (a)  :  Let  2 sin 2  x x 2  = Lt  sin x  = 1.2 ab cos ç t  .1  h f ¢(5)  =  f (5)  ×  3  =  2  ×  3  =  6  Now  f ¢(x) =  f (x)  Lt  h ® 0  \  55.  ë û 1 +  x 2  ny  dy ö æ y 1  =  ç y 1  = ÷ dx ø 1 +  x 2  è Þ  y 1 2 (1  +  x 2 )  =  n 2 y 2 (0/0  form)  Þ  y 1 2 (2x)  + (1  +  x 2 )  (2y 1 y 2 )  =  2yy 1 n 2 Þ y 2 (1  +  x 2 ) +  xy 1  =  n 2 y .2 ab cos a  (since  |cos a| £  1) £  a 2 + b 2  - 2 ab =  a  –  b.0).  57.  (d)  :  n log  x  log x n  -  [ x ]  -  1  =  x Lt ® 0  [ x ]  x  ® 0  [ x ]  Lt  log  x  does  not  exist  x  ® 0  [ x ]  which  does  not  exist  as  Lt  2  61.51  Differential Calculus  54.  y)  =  í at  ï a cos t .  (a)  :  y 1  =  n éê x + 1 + x  ùú ë û n .1  f ¢(x) =  f (x)  Lt  h ® 0  h f (h ) - 1  \  f ¢(0) =  f (0)  Lt  h ® 0  h f (h ) - 1  3 =  Lt  (³  f (0)  =  1)  h ® 0  h f (h ) .b sin  b  = x  A(0.  (b)  =  x ® 0  æ æ at  ö a 2 (sin 2 t + cos2 t ) + b 2 ç sin 2  ç ÷ è b  ø è \  2  x  ×  f ¢ (x)  x ® 1 2 f ( x)  1  Lt  60. x ­axis and lying  in  the  first quadrant is  (a)  36  (b)  18  (c)  27  4  9.5p(t ) - 450 .ò x3 y( x 3 ) dxùû + C 3ë 7.k (T - t).t ) 2  2 I 2 (c)  T -  k (d)  –  2  (2012)  (b)  e –kT  (a)  I -  10.3 ò x3 y( x3 ) dx + C 3 (a) g(x) – g(p)  (b)  1  3 x y( x3 ) .  Statement­II  is  true. x Î R .  (b)  Statement­1  is  true.  The  area  (in  sq.  dt then the  time at which  the  population becomes  zero is  3.  (c)  Statement­I  is  false.  Statement­II  is  false.ò x y( x ) dxùû + C 3 2. then y(ln 2) is equal to  dx (a)  13  (c)  7  (b)  – 2  (d)  1  11.  The  value V (t )  depreciates  at  a  rate  given  by  differential  equation  k (T . If p (0) = 850.  6. a (b)  ln 18  dV ( t )  = .  y = ò | t | dt .2 cos x | + k .  b 1  ln18  2  (c)  2 ln 18  (d)  9  (2013)  4.  units)  bounded  by  the  curves  y =  x . Statement­II is a  correct  explanation  for  Statement­I.  The value  of  (b)  2  (d)  ln 9  the equipment is  Statement­II :  ò f ( x )dx = ò f ( a + b . which are parallel to the line y = 2x.ò x2 y( x 3 ) dx + C 3 (c)  g ( p ) (c)  1 é 3 x y( x3 ) . 2y – x  + 3 = 0. Then the scrap value V(T) of  is  equal  to p/6.x )dx. then g(x + p) equals  0  (a)  1  3 x y( x3 ) .  (d)  10 2  3  (2012)  The population p(t) at time t of a certain mouse species satisfies  the differential equation  (2013)  dp(t )  = 0.52  JEE MAIN CHAPTERWISE EXPLORER  CHAPTER  INTEGRAL CALCULUS 10  1. then tan x .  Statement­II  is  true.  (d)  Statement­I is true Statement­II is true.  Statement­II  is  not  a  correct  explanation  for  Statement­I.  (a)  p  (a)  log 2  2 ò  0  8 log (1 + x )  1 + x 2  dx is  (b)  log2  5  (2011)  .  If  g( x ) = ò cos 4t dt .  If the integral ò  5 tan x dx = x + a ln|sin x .  (2013)  5. are  0  (b)  ± 3  (d)  ± 1  20 2  3  (b)  10 2  (c)  20 2  x equal to  (a)  ± 2  (c)  ± 4  (b) g(x)∙g(p)  g( x )  1  3 3 2 3  (d)  éëx y( x ) .2 a  is equal to  (a)  1  (c)  –  1  (2012)  Let I  be the purchase value of an equipment and V(t) be the  value  after  it  has  been  used  for t  years.  If  (d)  I -  kT 2  2 (2011)  dy = y + 3 > 0 and y(0) = 2.  (d) g(x) + g(p)  (2012)  2  The  area  bounded  between  the  parabolas  x  =  y  and 4 x 2  = 9y  and the straight line y  = 2 is  (2013)  (a)  The  intercepts  on x ­axis  made  by  tangents  to  the  curve.  x If ò f ( x )dx = y( x ) then ò x 5 f ( x 3 ) dx is equal  to  8.  Statement­I : The  value  of  the integral  p/3 ò  1 + p / 6  dx tan x b a (a)  Statement­I  is  true. where k > 0 is a constant and T is the total  dt life in years of the equipment.   where [x] denotes the greatest  (2.  (2006)  0  6  28.  (2007)  x  log t  1  dt .  local minimum at p  and 2p.  (2011)  1  sin x cos x  dx and J = ò  dx ..  (2006) .  ò  [( x + p)3 + cos 2 ( x + 3p)] dx is equal to  . +  f  (a)}  a f  ([a]) – {f  (1) +  f  (2) + .÷ + C è 2 12 ø  (d)  Then  ò p( x ) dx  equals 1 log tan æ x  .ø  + c  (d)  x + log sin è x .  the  tangent  to  the  parabola  at  the  point  a  25....  The  value  of the  integral.  The  area of the region  enclosed  by  the  curves y  = x .log cos è x . y  = 1/x  and the positive x­axis  is  dx  equals  22.. x  = e. +  f  ([a])}  [a] f  (a) – {f  (1) +  f  (2) + ..  ç 2 12 ÷ 2 è ø  (2007)  x  0  (a)  41  (b)  21  (2008)  (c)  41  dt  = p is  2  t t . +  f  (a)}.ø  + c  4  4  18.  ò [cot x]dx. (a)  (b)  (c)  (d)  (d)  p  log 2  8 (2011)  1 x 5 p ö ÷ .  (c)  log tan æç + ö÷ + C 2 è 2 12 ø  1  æ x  p ö (b)  log tan ç .  The  solution  for  x  of the  equation  ò  15.  24.  The  area  of  the  region  bounded  by  the  parabola  (2007)  (y  –  2) 2  =  x  –  1.  Let  I = ò Then  which  one  of the  following  is  true?  2  2  (a)  I > and  J <  2  (b)  I > and  J >  2  3  3  2  2  (d)  I < and  J >  2  (c)  I < and  J <  2  3  3  21.  16. where  [.  Let  F ( x ) = f ( x ) + f  çæ ÷ö .  ò x f (sin x ) dx is equal to  0  pö pö æ æ (c)  x .3 p / 2  (a)  p 4  32 (b)  p 4  p +  32 2 (c)  p  2 (d)  p - 1 . 2 (2006)  p 27.  is  0  equal  to  (a)  1  (c)  – p /2  (b)  –1  (d) p/2  (2009)  sin x dx  2 ò  p sin æ x . p (0) =  1 and p (1) = 41. 1]. a > 1.  The area enclosed between  the curves y 2  =  x and  y = |x|  is  (a)  1/6  (b)  1/3  (c)  2/3  (d)  1.ø  + c  4  4  (2008)  19..p ö + C .x + x (c)  2  dx  is  (d)  1.p / 2  26.]  denotes  the  greatest  integer  function.ø  + c  (b)  x + log cos è x .  local maximum at p and 2p. where  f ( x ) = ò  1  +t è x ø  1  (c)  4 2 - 1 (d)  4 2 + 1 (2010)  Then  F(e)  equals  (a)  1  (b)  2  (c)  1/2  (d)  0.  ò  (a)  3/2 square  units  (b)  5/2  square  units  cos x + 3 sin x (c)  1/2 square  units  (d)  1  square  units  (2011)  (a)  log tan æç x  + p ö÷ + C è 2 12 ø  14.  The area bounded by the curves y = cosx and y = sinx between  3  2  (a)  3 p the ordinates x  = 0  and  x =  is  2 (b)  2 2  (c)  2  (d) p .  For x Î ç 0.  x x 0 0  20.  Let p ( x )  be  a  function  defined  on R  such  that 1  x  p p¢(x) = p¢(1 – x ). for  all x Π [0.  local maximum at p and local minimum at 2p.  The value  of   is  a f  (a) – {f  (1) +  f  (2) + . 3) and the x­axis is  (a)  4 2 - 2 (b)  4 2 + 2 (a)  6  (c)  12  (b)  9  (d)  3  1  integer not  exceeding  x  is  (2009)  (a)  (b)  (c)  (d)  p 17..  The  area  of  the  plane  region  bounded  by  the  curves  x + 2y 2  = 0 and  x + 3y 2  = 1  is equal to  4  (a)  3  5  (b)  3  1  (c)  3  p p (a)  p ò f (cos x ) dx 0  0  p/2 p p/2 (c)  ò  f (sin x ) dx 2  0  (d)  p ò  3  (2008)  (a)  1/2  (b)  3/2  ò  f (cos x ) dx .ö è 4 ø pö pö æ æ (a)  x .  (2006)  .  2  (d)  3  (b)  p ò f (sin x ) dx x  9 .log sin è x .  define  f ( x ) = ò  t sin t dt Then f  has  2 ø  0  local minimum at p and local maximum at 2p. +  f  ([a])}  [a] f  ([a]) – {f  (1) +  f  (2) + .  The value of  ò [ x ] f ¢( x) dx.1  2  2  (d)  42  (2010)  23.  (2007)  13.53  Integral Calculus  (c) p log2  æ è 12.   The area enclosed between the curve y = log e (x + e) and the  .F (1).  lim ú equals  n ® ¥ ê n  ë n2 û  n2 n2 n2  1  1  (b)  sec1  (a)  cosec1  2  2  1 tan1  (c)  (d)  tan1.  (a)  2  (b)  1  (c)  4  (d)  3.  then  value  of  p p (b) 4 + 2 - 1  (a) 4 .  If  ò sin( x .2  coordinate axes  is  (a)  7/3  (b)  14/3  (c)  28/3  (d)  1/3...  (2004)  31.  The parabolas  y 2  =  4x  and  x 2  =  4y divide  the square  region  2  p / 2 (sin x + cos x  )  bounded by the lines x = 4.  If  f ( x ) = f ( a )  ò xg{ x(1 - x )} dx and  f ( .  (c)  (log x ) 2  + 1 + C  (d)  (2005)  (c)  64  (d)  15. I3  = ò 2  dx and  I 4  = ò  2  dx 1  0 0 1  1  log tan  x  . x > 0.  I 1  = 1 + e x  38.  a >  0.2 + 1  (A.a )  f ( a )  I 2  = (2005)  I 2  g{ x(1 - x)} dx.  Let  be  a  non­negative  continuous  function  such  that  the area bounded by the curve  y =  f ( x ).  then the value of  I  is  1  f ( .  then one of the possible values  If  ò  x e log x  1  x  + C  (a)  2  + C  (b)  of k is  (log x ) 2  + 1  x + 1  (a)  16  (b)  63  x  xe x  + C .  ò 2  3  1 1 3 2  2  cos x - sin x x  x2 x x  33.  (2005)  3  2  41. If  40.  (2005)  (c)  (sina.  The value of  ò (a) p/2  p p / 2  39. B) is  p p (a)  (–sina .  (2004)  (2005)  dx  is equal  to  42. cosa )  (d)  (–cosa .  The  area  of  the  region  bounded  by  the  curves  0  46.  (a)  1 : 2 : 3  (b)  1 : 2 : 1  (2004)  (c)  1 : 1 : 1  (d)  2 : 1 : 2.  Let  F ( x) = ç ÷ .54  JEE MAIN CHAPTERWISE EXPLORER  é 1 sec2 1 + 2 sec2 4 + . sina )  (c) 1 .  The value of  ò |1 -  x | dx is  32.  (d) (2004)  p  2 8  2  f is  2  sin x  43.  x­axis and the ordinates  1  log tan  x  -  p + C (c) p 2 8  p 2  p x = b >  x =  and  is b sin b + cos b + 2b  . S 2 .  If  I1 = ò 2 dx .3 p + C then  (a) 2  8  2  (a)  I 1  > I 2  (b)  I 2  > I 1  (c)  I 3  > I 4  (d)  I 3  = I 4 .  sin  x  (2005)  æ e  ö d 45. y = 4 and the coordinate axes.  n lim  is  f ( x )  3  n ®¥ r  = 1  4  t  1  Then  lim  dt  equals  f (2) = 6.  ò í 4  2  ý dx 3  sin x  îï 1 + (log x ) þï dx = F (k ) . I 2 = ò 2 dx . sina )  (2004)  n  1  35.  ò (a)  1 – e  (b)  e – 1  (c)  e  (d)  e + 1. x = 3 and the x­axis  is  x ®0  x sin x (a)  3  (b)  2  (c)  1  (d)  4.  (2004)  p 30.  1  (2005)  log cot  x  + C (b) 2  2  f ( x  )  34.1)  ïü is equal to  36.  2  cos 2  x  dx .p 1 + a (b)  ap  (c)  2p  e x  .4 -  2 .  (2003)  1 +  x 2  ( () ( ( )  )  ( )  ( )  ( )  ( )  )  ( ( )  )  ( )  3  x 2  2  ò  sec  tdt  37.a ) dx = Ax + B logsin( x .  2  dx è x ø  ïì (log x .a )  ò (a)  –1  (b)  –3  (c)  2  (d)  1.  is  x  ..  The value of  I = ò dx  is  S 1 . + 1 sec2 1 ù 29.  If  ò x f (sin x) dx =  A ò f (sin x )dx.4 +  2  (d) 1 . then A is  (d) p/a  0 (2005)  0  (a) p/4  (b) p  (c)  0  (d)  2p.a ) + C . f ¢ (2) =  . cosa )  (b)  (cosa.  Let  F  :  R ®  R  be  a  differentiable  function  having r / n  å  e  44.  The value of  lim  is  y = |x – 2|.  (a)  2  (b)  1  (c)  0  (2004)  (d)  3  (2003) . S 3  are  respectively  the  areas  of  these parts numbered  1 + sin 2 x 0  from top to bottom; then S 1  :  S 2  :  S 3  is  (a)  2  (b)  1  (c)  0  (d)  3.  x = 1.  48  x ® 2  6  x - 2  (2004)  (a)  36  (b)  24  (c)  18  (d)  12.  Then 4  4  4 1  log tan  x  +  3 p + C .   If f (y) = e  y .p - 1  (2003)  48.  2 x (1 + sin x )  dx  is  1 + cos 2  x (a) p 2 /4  (b) p 2  ò  -p (2003)  (c)  0  (d) p/2.  If y =  f (x) makes +ve intercept of 2 and 0 unit x and  y and  encloses  an  area  of  3/4  square  unit  with  the  axes  then  t F( t ) = ò f ( t . Then  2  (a)  e + e  - 3  2 2  57. units  (d)  none  of  these  (2002)  a  a + b  b  ò  f ( x ) dx 2  a  is  1  (a)  p + 1  1 1  (c)  p . units.e  - 5 .a  b  ò  f ( x) dx 2  a  p 54.  then lim  n [ I n +  I n .  The area bounded by the curves y = lnx.  40.  41.  16.  46.  I n  = ò  tan n  x dx .  24.  38.  37.  56.  2  a  1  (b)  1 -  p 1  (d)  p + 2 . y = |ln x| and  y = |ln|x|| is  (a)  4 sq.  29.  (2003)  is  0  (a)  3/2  (b)  1  ò  sin x dx (a)  20  1  2  (d)  e .  11. then  ò x f ( x) dx is equal to  (a)  n p + 1  n ®¥ 0  1  (a)  n + 2  1 1  (c)  n + 1 + n + 2  1 p + 2 p + 3 p + . c)  (b)  (2002)  .  57.  43. units  (c)  6 sq.x ) n dx is  52.  20.  19.  (b)  (b)  (c)  (*)  (c)  (c)  (c)  (b)  (a)  6.  48.  26.(1 + t )  (c)  F (t) = te –t  ò x f ¢ ( x ) dx (b)  F (t) = t e  t  (d)  F (t) = 1 – e  t (1 + t).  50.  58..  (b)  (a)  (a)  (a)  (b)  (c)  (c)  (b)  (a)  (d)  2.  (2002)  (2003)  51.x ) dx 2  a  (b)  b .  54.  ò [ x 2  ] dx is  0  (a)  2 -  2 (b)  2 +  2 (c)  (d)  2 - 1 2 - 2.  b  (d)  a + b  ò  f (b .  (2002)  p/ 4 0  2  (c)  e + e  + 5  2 2  (d)  –3/4.  22. units  (b)  4 sq. units  (c)  10 sq.  (2003)  2  58.  b  (c)  a + b  b  ò  f (a + b .  53.  28.  44.  34. y = ln |x|.  30.  10.  39. + n p  3.  If f (a + b – x) = f (x).  8.  The value of the integral  I = ò x(1 .  23.  45.  51.  (2002)  49.  21.2 ]  equals  n ®¥ 0  (a)  1/2  (b)  1  (c) ¥  (d)  0.  36.  15.  12.  47. Answer  Key  1.  (c)  (c)  (b)  (c)  (b)  (c)  (a)  (b)  (a)  (c) 5.  17.  9.  The area of the region bounded by the curves y = |x – 1| and y  = 3 – |x| is  (a)  3 sq. g(y) = y ; y > 0 and  55.  (a.  52.  31.  7.  (d)  (d)  (a)  (a)  (d)  (a)  (b)  (c)  (b)  (b)  4.  25. then  2  0  (a)  F (t ) = et  .55  Integral Calculus  1  47.  14.  (2002)  is  p the value of the integral  ò  f ( x ) g ( x) dx is  2  (b)  e .  49. units  (b)  6 sq.x) dx.  32. units  (d)  2 sq.  (d)  (c)  (b)  (c)  (c)  (b)  (c)  (b)  (b)  (d)  (2002)  53.n + 2  1  (d)  n + 1 .y ) g( y )dy .  18.  33..  55. d)  (b)  (d)  (c)  (a)  (c)  (d)  (a.  and g(x) be a function that satisfies f (x) + g(x) = x 2 .  lim  1 1  (b)  n + 1 .  13.  27.  2 2  (b)  8  (c)  10  (d)  18.  35.  Let f (x) be a function satisfying f ¢(x) = f (x) with  f (0) = 1  56.e  - 3  2 2  (c)  5/4  10 p 50.  42.  we  have  2 x .56  JEE MAIN CHAPTERWISE EXPLORER  1.  (c) :  p ò 850 3  2  Area =  ò [(2 y + 3) .ò x 2 y( x 3 ) dx + C 3 Þ Þ  a = 2  dy = | x | = 2.900 ò  - 50 0  3  Þ p = 900 - 50∙ e t /2  0  If p  =  0.  1  3 x y( x3 ) .2)  5sin x = sin x .  (b) : Let x 3  = u. x 2  = 9y  Area bounded by the parabolas and y  = 2  2æ 2  yö = 2 ´ ò ç3 y dy = 5 ò  ydy ÷ è 2  ø 0 0  = 5´ \  x = 1.  (d) : Solving y =  x with 2y – x  + 3  =  0.  9.  ò | t | dt = . \ x = ± 2 (d)  :  dx x 6.  (a) :  x 2  =  .2 sin x - 2 cos x 1  = éëu ò f ( u)du .  we get  1  u f (u) du 3 ò  Now  ò x 5 f ( x 3 )dx = a (cos x + 2 sin x )  5 tan x = 1 + tan x .5 p( t) - 450 dt t 2 dp p .3)( x + 1) = 0 y 4 7.  (d) :  1  900  t /2  =e Þ t = 2 ln 18  50 dV = .x + 3 = 0 Þ ( x .  y1  = ò| t | dt = ò tdt = 0 0  -2 and  y2  = t2  = 2  2 0  Þ g( x + p) = .900  = dt Þ 2ln  =t p . then 3x 2 dx  =  du  5 tan x dx = x + a ln|sin x . then  = 9 + 9 – 9 = 9  p/ 3 4.ò tdt = .y2 ] dy = y + 3 y -  0  3  y 3 ( y )3/2  10 20 2  = ´ 2 2  = 3/2 3 3 d( p(t ))  = 0.2 cos x p 6 Again Statement­II is true.2 cos x | + k tan x .2 5.  We then get x = ±1  3.  (b) : ò  Also suppose ò f ( x )dx = y( x ) Differentiating both sides.ò ( ò f ( u)) duùû 3 = 2.2  0  Tangents are y – 2 = 2(x – 2) and y + 2 = 2(x +  2)  Then the x intercepts are obtained by putting y = 0. we have V(T) = a  =  I 2 2  . d) :  g( x ) = ò cos 4t dt 0  We can solve  for  y  to  get  2 sin x (1 + 2 a) + cos x ( a .k(T - t) dt + cot x On integration.  2 I = tan x p / 3æ ò  çè 1 + p / 6  p/ 3 = æp pö = ò  p / 6 1 1 tan x ò  1 × dx = çè 3 .  V = + tan x ö ÷ dx 1 + tan x ø \ I =  p 12 At t = 0.2cos x sin x .2 0 x é sin 4t ù sin 4 x Þ g ( x) = ê = ë 4 úû 0  4 2  2  (a.  V(t) = I \ a = I -  kT 2 k(T . 9  8.t) 2  + a  2 kT 2  + a  Þ  I = 2 2  kT As t  =  T.6 ÷ø = p / 6  sin 4( x + p) sin 4 x = 4 4 Þ  g(p) = 0 Þ  g(x + p)  = g(x) + g(p) or g(x) –  g(p).  (c) :  I = ò p / 3  dx p/ 61 + Adding. sin x )dx æ 1 1 ö æ 1 1 1 1  ö = 2ç + .  (cos x .cos x - sin x ] p / 4  8  2 .  3).  I = ò p( x )dx = ò p(1 .x )]dx = ò [ . we have ln5 =  C  Now ln(y + 3) = x + ln5  As x = ln2 we have  ln(y + 3) = ln2 + ln5  = ln10 Þ  y + 3 = 10 Þ  y  =  7.y ) 2 dy = ò t 2 dt = ê ú = = 9  3 ë û 0  3  0 0  p 17.x )dx Adding  we  have  On adding we get  2 I = ò {[cot x ] + [ .q ÷ø ÷ø dq 0  ì æ æp ö öü = ò  ln í(1 + tan q) ç 1 + tan ç .  (a) :  Area = 1 + ò  dx = 1 + ln x 1  = 3  2 1  x 2 2 14. x 12.  Set x = 0 to obtain p(0) =  –p(1) +  k Þ  1  = – 41 + k.t.1 ÷ + ç + + + ÷ è 2 ø è 2 2 2 2 2 ø  ln(1 + tan q)dq 0  p/ 4 p /4 ò 5 p /4  (As the  first and third integrals are equal in magnitude)  Let  x = tan q Þ  J = Adding 2J =  ò 3 p /2  (sin x .  (a)  :  dy 10.cot x] dx 0  Now.  3)  is  y  –  3  =  1  (x  –2) Þ  x  –  2y  +  4  =  0  2  Þ I = 8 J = p ln 2.6 y + 9) dy = ò ( y .57  Integral Calculus  dy 15.  (b) :  I = ò  2  0  1 + x 1  Let  J = ln (1 + x )  ò  1 + x 2  0  ò p/4 = 2 [ sin x + cos x ]0 dx ò  æp ö ò  ln(1 + tan çè 4 .2 = 4 2 - 2  16.3) 2  dy 0  0  (Let  3  –  y  =  t)  3  3 3  é t 3 ù 3 3  = ò (3 .  (b) :  f ( x ) = ò  t sin tdt 0  f ¢ ( x) = x sin x f ¢¢ ( x ) = x cos x + 1  -1/ 2  x sin x 2 f ¢¢ ( p) = .  x Ï  Z.  p(x) = –p(1 – x) +  k.  y¢  =  1/2  The  equation  of  the  tangent  to  the  parabola  at  (2.  k being the  constant  of  integration.  we  have  2(y  –2)y¢  =  1  y ¢ = Þ 1  2( y - 2)  at  (2.q) ÷ø dq = 0  p/ 4 æ æp öö ò  ln(1 + tan q) + ln çè 1 + tan çè 4 .4)] dy 0  3  e e 13.sin x )dx + 0 0  0  p 2 I = ò ( -1) dx = -p \  I  =  –p/2  0  Note  that  [x]  +  [–x]  =  0.  (c) :  dx = y + 3  Þ y + 3 = dx As y(0) = 2.  x Π Z  =  –1.p < 0 ;  f ¢¢ (2 p) = 2 p > 0 Thus at p  maximum and at 2p  minimum.(2 y .2) 2  + 1 .  (b) : (y  –  2) 2  =  x  –1  Differentiating  w.  The desired  area =  1  p/ 4 8 ln (1 + x )  dx 11.  (b) : p¢(x) =  p¢(1 –  x)  On integration.r.q ÷ ÷ ý dq è4 ø øþ è î 0  p /4 2J = ò  (ln 2)dq = 0  p p ln 2 Þ 8 J = 4 ln 2  4 4 5 p / 4  + [ . \ k =  42  1 3  = ò ( y 2  .x ) dx = ò kdx = ò 42 dx = 42.  (c) :  I = ò [cot x] dx 0  p 0 1  p I = ò [cot(p .cot x ]} dx 0 1 p 0  1 1  2 I = ò p( x) + p(1 .  The  area  of  the  bounded  region  3  = ò [( y .cos x )dx + p /4 0 p /4 Now J = 5 p /4 (cos x . 0 Thus I = 21. .  x.   (c) : 1 ò  = log tan æç + ö÷ + c ...ö + c  è 4 ø  a  25.2 ï 2 2  =í ý n 1 n  3 2  ï ..  (d) :  2 ò  p sin æ x .. if n = 2 m + 1 ï î n n-2 2  þ  p =  2 p 27.x ) dx 0  1  é2 x 2  ù 2 1 1  = ê x3/ 2  ..  There  is  no correct option..  (c) :  F ( x ) = ò 1 1  p = 2× x+ 2× ln sin æ x .  2  (a) : Solution x +  2y 2  = 0  and x + 3y 2  =  1 we  have  1 – 3y 2  = – 2y 2 2 (– 2..  (a) : Required area  Y  y = |x |  x = y 2  (1.ú = .  +  f ([a])]  -p 2  26..  2 2 2 12 ø p è sin  x + 6  ( )  X  1  p = 0 + 2  ´  2 2 p ì 2  ï n  í using fact  ò sin  x dx 0  ï î  ì n .1) f ¢( x ) dx + ò  Kf ¢( x ) dx =  f (2) –  f (1) +  2[ f (3) –  f (2)] +  3[ f (4)  –  f (3)] + .....  (c)  :  Let  I = ò éë( x + p )3 + cos 2 ( x + 3p ) ùû  dx -3 p 2  Putting  0  x  + p  =  z  -p p -3 p -p Þ z = and  x = Þz= 2 2 2 2  \  dx =  dz  and  x  +  3p  =  z  +  2p also  3 ù1  é y  = 2 ò (1 .ö + c  è 4 ø  2 2  19.58  JEE MAIN CHAPTERWISE EXPLORER  x  p -1  23.. – 1)  x + 3y 2  = 1  x + 2y 2  = 0  y¢  1  3  = ò 1 f ¢( x ) dx + ò 2 f ¢( x ) dx + .  (*) :  éësec  t ùû  = 2  2  sin x  dx  18....  p = x + ln sin æ x .  (c) :  In the interval of integration  sin  x <  x  1 K  = –  [ f (1) +  f (2) +  . 0)  dx 1  x  p 22.3 y ) . + y 2  = 1 Þ  \  y =  ± 1  y = – 1 Þ  x =  – 2  y = 1 Þ  x = – 2  The bounded  region is  as  under  x¢  1/ x  ln t ln t  dt + ò  dt  1 + t 1  +t 1 1  24. +  f (K)]  +  K  f (a)  O  1  1 a  ò ( K . 1  ´ p if  n = 2 m  ü ï n n ...ö sin  ú dx  è 4 4 û 4 ø ë x = -  2.sec -1  2  = p p sin æ x .  (d)  :  Let  I =  ò xf (sin x) dx 0  .ö è 4 ø sec -1 x .= 3 2 ë û 0  3 2 6.+ ö è 4 4 ø = 2 ò dx  p æ sin  x .  (i) .ú 3  û 0  ë 0  2 4  =  sq...  (b)  :  ò [ x ] f ¢( x ) dx .  p p p 3 p . 1)  A  1 =  [a]  f (a)  –  [ f (1)  +  f (2)  +  ...1 × n .ö è 4 ø p pù é p = 2 ò ê cos + cot æ x .  say  [a] =  K  such  that  a  >  1  c  being a constant  of integration.  x x x  1  æ ln t 2  ln t ö ln t  F ( x ) = ò ç + dt = ò  dt  =  (ln x )  ÷ 2  1 + t (1 + t )  t t è ø 1 1  F(e) = 1/2... 0)  x  B  (– 2. 1)  1  = ò ( x .y 2 ) dy = 2 ê y .1  Þ  sec  x = + =  2  2 4 4  l= ò  [z3 + cos 2 (2p + z )] dz = ò -p 2 -p 2  1  1  sin x x  2  2  I=ò dx < ò dx = ò  xdx = é x 3/ 2 ù = êë 3  ú 0  3  û x x 0 0 0  p 2 z 3dz p 2  +  ò cos 2  z dz -p 2  p 2  =  0  (an  odd  function)  +2 ò cos 2  z dz 2  \ I <  3  0  1 1  cos x  1  dx < ò dx = [2 x ]1 0  = 2  x x 0 0  Also  J = ò \  J < 2  21.  (K  – 1)  [ f (K)  –  f (K  –1)]  +  K[ f (a)  –  f (K)]  y  2 2  The desired area =  2 ò [(1 ..3 ...  (0. units  3 3 x= p 2  \  20.( -2 y )] dy = 2 ´ K 2  K -1  (1...... .p sin B + 2  4  p p \ f = 1 ..  (ii)  0  a 32.x )ö dx = p ò  f (cos x ) dx è ø 2  \  e  éë z (log e  z .. 1)  = ò log z dz 0  2 a y = loge (x + e )  e  using  ò f ( x ) dx = ò  f ( a .  (b)  :  Required  area  =  ò  log e ( x + e ) dx a  p = pp p 2  \  I = ò f (sin x )dx =  2 ò f (sin x ) dx 20 2  0  0 0  p 0 2 2 2 x >  2 x \ 3  2 b  f ( x )  = ò f ( x ) dx  = b .  x 3  >  x 2  28...  (c)  :  Total  area  =  4  ×  4  =  16  sq.  B  p \ f ( x )  = ò 1  2 34.3 1  p ú = × .  (a)  :  For  0  <  x  <  1. + sec 2 1  29.log x ) 2  ö \ òç dx = ò f ¢( x ) dx = f ( x )  ç 1 + (log x ) 2  ÷÷ è ø  2  C  y = 4  2  x  16  dx = = S1  4 3  0  16 16  \ S2  = 16 ´2 = .p 1 + a a a  p 3  I 1  >  I 2 . ´ if  n  is even ú n n-2 2 2  ëê û  f ( x )  =  p 2  31.  (c)  :  Method  by  cross  check  x  Consider  f ( x )  = (log x )2  + 1  2 x log x  1 + (log x ) 2  x  \ f ¢ ( x ) = (1 + (log x )2 ) 2  \ f ¢ ( x )  = 1 + (log x ) 2  .1)ùû 1  33...  48  36.  = 4 f ¢(2) ´ ( f (2))3  = 1  ´ 4 ´ 6 ´ 6 ´ 6 = 18.e  O  A  n = 4  æ 1 .  [using  ò f ( x ) dx = 2 ò  f ( x ) dx if  f  (2a  –  x)  =  f (x)]  b O  1 – e  1  By  (i)  &  (ii)  on  adding p 2 (0..  (c) :  n lim n  ®¥ n 2 n n è n ø  2  n 2 æ n  ö 1 1 2 4  = lim 2 sec 2 2 + 2 sec 2 çæ 2  ÷ö + .  (2)  2 f ( x ) = ò cos 2 x dx = 2 ò cos 2  x dx = 2 ´ 2 ò  cos 2  x dx .  2 =  lim  x ® 2  -p p p -p 0  .+  2.3 = 3  ò 2 2  3  a .log x )  ö = ç ç (1 + log x ) 2  ÷÷ è ø  æ (1 .  2 f ( x ) = 4  ´ 1  ´  p 2 2  0  4  (0/0)  form.  (a)  :  Let  f ( x ) = ò cos  x x  dx ( a  > 0)  .p 1 + a p b b  cos 2  x  \ f ( x )  = ò dx  \ ò f ( x ) dx = ò  f ( a + b .  (c)  :  lim  ò x ® 2  6  1 tan1.x ) dx 0 Y  1 .59  Integral Calculus  p 0  I = ò ( p -  x) f (sin x ) dx .2 if x  > 2  ï y = if x  = 0  37.  S3 = ò  4 t 3  dt  x - 2  f ¢ ( x ) ´  4( f ( x )) 3  1  p/2 p/2 é ù n  ê By using  ò sin  x dx  ú 0  ê ú ê n ..e.  (c)  :  According  to  question r = n  2  1  r sec 2  r  å  = lim å æç r2  ö÷ sec 2  r  = n lim n n n ®¥ r  = 0  n  n ®¥ r = 1  è n ø 1  3  0 \ 1 1 2 4 1  sec 2 2 + 2 sec 2 æç 2  ö÷ + .2 log x (1 + log 2 x ) 2 2  æ (1 .  3 3  \  S 1  :  S 2  :  S 3  is  1  :  1  :  1.  (c) :  í 0 ï 2 .log x  ö x  \ òç dx  = 2 ÷ 1 + (log x ) 2  è 1 + (log x ) ø  Hence (c) is correct answer and we can check the other choices  by the  similar argument.(1)  .x  ..1 n .  and  for 1  <  x  <  2.  units  ( B sin B + p4 cos B + B 2 )  ( )  2  ( ) ( )  a x  cos 2  x  dx  1 + a x  ò  f (b) = sin B + B cos B ..  2 x < 2 x  Þ I 3 < I 4  0  a = 1.  (b)  :  Using  fact  ò X  a  2  p = p ò f æ sin( .  ì x .x +  x r = n  3 \ 2 x >  2 x 3  as  2 x >  2 x 6  Area  of  x 2  >  x 3  i.  2 4  2  30....x )  ..a  f ( a + b + x ) + f ( x ) a  2  1 = ò x sec 2 ( x 2 ) dx =  0  0  B ( > p / 4)  ò f ( x ) dx = p/ 4 p / 4  f ( x )  35..x if x < 2  î  Required  area  = Area  of DLAB  + Area  of DMBC . + 2 sec  ç 2  ÷ n n ®¥ n èn ø n n è n ø  ( )  2  \ ò 2 x dx > ò 2 x  dx x  dx  = 6 . 2  -1 ò =  1 ( x 2 .x \  2  )  dx .a) dx  = Ax  +  B  log sin  (x  – a)  + C p Þ  Differentiating  w.  (c)  :  M(3.  –1.x)} dx = if | x |< 1  2  ïî(1 .a  and f (–a)  =  \  f (a) =  1 + e a  1 + e .  (d)  :  f ( .x 2  ìï \  |1  – x 2 |  =  í 1  1  [AL  ×  AB +  BC  ×  CM]  =  [1  ×  1  +  1  ×  1]  =  1  2  2  e x  38.x )) if  x < -1 and x ³ 1  =  ò  ò  (1 .  (a)  :  ò  0  =  (sin x + cos x ) 2  (sin x + cos x) 2  cos x - 1  sin x 2  dx  1 1  dx  2  cos( x + p/4)  ò  1 1  dx  2  sin æ p + x + p ö ç ÷ 4 ø è2 ò  1 1  dx  2  2 sin æ x + 3p ö cos æ x + 3 p ö ç ÷ ç ÷ è2 8 ø è 2 8  ø ò  æ p x ö sec 2 ç 3  + ÷ 1  è 8 2 ø dx  =  æ x 3 p ö 2 2  tan ç + ÷ è 2 8  ø 0  p 2  ò  1  ò  p 2  Þ  A f (sin x ) dx =  0  0  ò  1 2 f ( .2  A(1.  x  =  3 Now  1 .  0)  3  3   2  x  – y  = 3  ò  (1 .  x  >  x    2  – 41.a )  b using  b  f ( x)dx = ò f (a + b .a)  =  A  +  sin( x .a )  sin ( x .x) g {(1 .60  JEE MAIN CHAPTERWISE EXPLORER     2 .  3 y  = L(1.  1)  (0.1) dx ò  -1 1  4 28  æ 2 ö 20  =  =  + 2 ç ÷ + 3 3 3 è ø  3  42.  x  =  2.  (c)  : As  f (x) =  1 + e x  e a  e .a )  1  ò a cos x .  0)  C(3.a )  p 1 2  ò g {(1 .  0)  B(2.a  \  f (–a) +  f (a)  =  1  f ( a )  x g {x(1 .x ) dx ò a f ( a )  f ( a )  ò  2 Þ Þ  2I 1  =  I 2 \  I 2  2  =  I1  1  =  =  p 2  ò x f (sin x ) dx ò  =  A f (sin x ) dx p 2  ò  or  A f (sin x ) dx =  0  p 2  p p ò ò 0 0  p f (sin x) dx = xf (sin x) dx 2  p =  ´ 2 f (sin x ) dx 2  ò  ò 0  p 2  ò 0  0  =  Þ  A f (sin x )  =  p f (sin x ) dx Þ  A  = p  p/2  40.r.(1 .  1.b sin x dx  where  a  =  b  =  1  a =  r  cos q  =  1  b =  r sin q  =  1 \  r  =  2 q  =  tan –1 (b/a)  let  f ( a )  ò  (1 .  0)  x  =  1.  (b)  :  0  sin x  ò sin( x .x)( x) } dx f ( .x 2 ) dx + ( x 2  .x) x} dx x g { x (1 .  x  both  sides æ cos x  ö 2  + sin x ÷ =  ç è -1  ø 0  Þ  =  1  – (–1)  =  2  Þ  sin x  B cos ( x .x 2  dx =  Putting  1 –  x 2  =  0 \  x  =  ±  1  Points  –2.x )(1 + x )  dx -2  .a)  sin  x  =  A  sin  (x  – a) +  B  cos  (x  – a)  .  1)  ò  1 .  (b)  :  =  a  1 æ x  3 p ö ´ 2 log tan ç + ÷ +c 8  ø  2 2  è2 1 æ x  3 p ö log tan ç + ÷ +c 8  ø  2  è2 p/2  dx =  ò (sin x + cos x) dx 43.a )  39.x ) } dx = f ( .1) dx + -2 ò 3  (1 .t. .2][(2n )(2n .  (b)  :  ò x (1 ..x)  f (a  +  b  –  x)  dx  t  x sin x tan x 2  =  Lt  x ® 0  x sin x tan x 2  =  Lt  x ® 0  2  sin x  x  x tan x 2  1  =  1  ×  1  =  1  =  Lt  sin x  x  ® 0  x 2  Lt  x  ®  0  x Putting  ò x f ( x) dx dx ....  (c)  :  Let  I  =  a  b  3  esin x  dx  =  F(k)  –  F(1) I  =  1  64  sin z  Þ  ò  e  z 1  dz =  F(k)  –  F(1)  where  (x 3  =  z) 64  F ( z )  1  ]  Þ [ Þ  Þ  F(64)  –  F(1)  =  F(k)  –  F(1) k =  64  =  F(k)  –  F(1) 2  46.2  2 ´ 2 ´ 1  (2n + 4)(2n + 2)  1  =  ( n + 2)(n + 1)  1 1  =  (by  partial  fraction)  n + 1 n + 2  =  b  4  3  sin x 3  ò x e dx  =  F(k)  –  F(1) 1  45.y  y dy (By  replacing  y ®  t  –  y  in  f (y))  0  t  0  q q F(t)  =  .  B)  =  (cos a .2)... q  = p/2 and  1  n  p/2  ò x (1 . q  =  0  x =  1.2)(2n .2] ù =  ú (4 n + 2)(4 n)(4 n ..  (a)  :  From  given  F(t) =  0  =  e ò  t .2]  =  (2n + 4)(2n + 2)(2n)(2n - 2)..e x )  dx 0  1  2 x òx e 0 [(2n )(2n .2).  (b) : As  f (x) =  f ¢ (x) and  f (0) =  1 f ¢ ( x )  =  1 Þ  f ( x )  Þ  log(f (x))  =  x Þ  f (x) =  e x  +  k Þ  f (x) =  e x  as  f (0) =  1  Now  g(x) =  x 2  –  e x \ 1  ò  f ( x ) g ( x )  dx =  0  1 q cos 2 n  + 1 q d q 0  =  ò e x ( x 2  .2).q) e d q ò ò  t  0  =  (t  eq ) 0 t  –  [(q  –  1)  eq ] 0 t  =  t(e t  –  1)  –  (t  –  1)e t  –  1  =  e t (t  –  t  +  1)  –  t  –  1  =  e t  –  (t  +  1)  50.e 2 x dx ò  0  1  –  = [(x 2  –  2x  +  2)e x ] 0  1  æ e 2 x  ö çç ÷÷ è 2  ø 0  . 4.q)e d q = (t .4) .61  Integral Calculus  sin  x  =  A  (sin  x  cosa –  cos  x  sina) + B (cos  x  cos a +  sin  x  sin a)  sin  x  = sin  x  (A  cos a  +  B  sin a)  +  cos  x(B  cos a  –  A  sin a)  Now solving  A  cos a  +  B  sin a =  1  and  B  cos a  –  A sin a  =  0  (A.x) n  dx 0  x =  sin 2q  dx  =  2  sin q  cos q  dq  x =  0.x )  dx =  ò  sin 2 q cos 2 n q (2 sin q  cos q)dq  \  0  p 2  I  =  0  I  =  ò x f (a + b .  (b)  :  n Lt  ®¥ 1  x  =  e dx =  e  –  1  ò  0  p/2  \ 2 ò  sin 3 q cos 2 n  + 1 q d q 0  2[2 ´ (2n)(2n .  (c):  Given  4  Þ  2  3 x  ò  x3  48.x ) dx \ I  =  ò 2 a 2  ò  a  49.  sin a)  r  r = n  1  n  e  n r  = 1  å  44.ò xf ( x) dx a a  b b  a+b a + b  f ( x ) dx = f (a + b ..  (b)  : ( tan t ) 0 x  Lt  x ® 0  1  47.2  û ò f (t .4.(t .x) dx a  b  ò (a + b) f ( x)dx .y) g ( y) dy t  0  2n + 1 b  ò (a + b)  f (a  +  b  –  x)  dx  –  a  b p/2  ò  sin a  b  1  3 2 n  + 1  qdq =  2 ò sin q cos  é ê Using êë ò (a + b ..  (a).  .  (b)  :  I n  =  n  ò  tan  x dx 0  1  (–1..  0  ò  51..+ (–1)  f n (x)]  55.x) ..  ( ò I n  –  2  =  =  0  +  ò 1 dx 1  =  2 -1 ò [ x 1  2  ] dx x dx .62  JEE MAIN CHAPTERWISE EXPLORER  æp ö æ e 2  .  (a)  :  Required Area  =  dx p ò p sin x dx - 0 ò  sin  x dx 0  = 18     (Using period  of  |sin  x| = p)  =  10 ×  2  –  1 ×  2  f(x)  =  |log|x||  p/4 57. 0) ò 2(1 + x ) dx + ò 2 dx + ò (4 .0 ÷ = p 2  -  =  (e  –  2)  –  çç ÷÷ =  e  –  4 è ø  2 2  è 2  ø  2  n  x  x  n  n  n  n  Using f  (x)e  dx = e  [f  (x) – f 1  (x) +  f 2  (x) + .  units..(.ò ç ..x + 1) dx + (3 .( x ....  (c)  :  sin x  cos x ) 0 p  n  . units  \  52..]  =  0  –  10 p 56.. f n  are  derivatives of  first.÷ . 0)  2  3  [\  f(2)  =  0.  (d)  :  Given  f ( x )  dx =  3/4  where f 1 . f 2 .x + 1) + (3 . second  .2 0  I n  +  I n  –  2  =  1  n +  1 \  n(I n  +  I n  –  2 )  =  1  1 +  1/n \  n Lt ® ¥  n(I n  +  I n  –  2 )  =  1  1 2  ò  0  2  x sec 2  x dx [ x 2 ] dx =  ò 2  [ x 2 ]dx + 0 0  2  (By  putting cos  x  = t)  n .  0)  p/4  (1. x dx  0  è xø 0  1  =  2[(1 –  0) +  (x) 0  ]  =  4  sq..2  ò  tan  p/4  =  x dx 0  0  58.  + ç ÷ ú ènø è n ø úû  êë è n ø r=n =  n Lt  ®¥ 1  ´  n  2  ò x f ¢( x) dx =  xò f ¢( x) dx –  0  1 p + 2 p + .  (d)  :  ò  sin  x 10 p 53.  curve  having  intercept  4  =  –3/4  2  units  on  x­axis.(.2 x )  dx -1 0 2  1  = 4  sq.  (b)  :  Required  area  0 =  ò 2  ò  ò  (3 + x) .2  x dx 0  1  æ 1 ö = 2 éë x log  x ùû ..  (a)  :  Lt  n p  n ® ¥ 1 =  n Lt ®¥ n 1  æ r  ö 1  p  å  n çè n ÷ø =  r  = 1  x p dx =  ò  0  0  1  p +  1  ò f ( x) dx 0  3  =  2f(2)  –  4  2  ( x ))0  - 3/4  =  ( x × f p  é æ 1 ö p æ 2  ö p æ n  ö ù ê ç ÷ + ç ÷ + .2  ò  tan  tan n .2  x ×  (sec 2 x  –1)  dx  +  p p ò p/4  x sin x  p p/4  tan n x dx + 0 -p p p/4 \  I n  +  I n  –  2  =  ò 1 + cos 2  x dx  +  2 ò 1 + cos 2  x dx  54..n th  order..x) .  0)  =  2ò log  x dx 0  )  p 1  p x  -p x sin x  x sin x  ò 1 + cos 2  x dx =  2 ×  2 ò 1 + cos 2  x =  0  +  2  -p 0  =  p sin x  x sin x  ò 1 + cos 2  x dx  =  4  ×  2 ò 0 1 + cos 2  x dx  0  p p æ ö p by using  xf (sin x ) dx = f (sin x)  dx ÷ ç ò ò  2  0  è ø 0 =  4  p =  4 ´ 2  ×  2 = 4p  (tan -1 p/2  ò  p/4  ò 1 + cos 2  x dx  0  ò  tan n . + n p  (2.1 ö e 2  3  =  4p  ×  ç .1) dx -1 0 =  1  1  0  1 2  (0.  (b)  :  2  tan n . 2) = ke (2003)  . x  >  0  then  dx dx  dy  1 - x 1  (c)  x 2 = y 2  +  xy  (a)  (b)  dx x x  x  1 + x dy  2 2  .  where A and  B  are  arbitrary  constants  is  of  (a)  second  order  and  second  degree  (b)  first  order  and  second  degree  (c)  first  order  and  first  degree  (d)  second  order  and  first  degree.  (2006)  12.  degree  as  follows  (a)  order  1.  6.e tan  y ) = 0  is  dx (a)  2 xe tan y = e 2 tan  y  + k -1  -1 tan (b)  xe -1  (c)  xe 2 tan -1  y  -1 = tan -1 y + k y -1  = e tan  y  + k -1  .  (2004)  dx dy  y  + .  If  x =  e y + e  ..tan  y  .  then the solution of the equation  dx is  8.  degree  3.log x + 1).  The differential equation for the family of curves x 2  + y 2  – 2ay  The differential equation of all circles passing through the  =  0.  (d)  ( x .  (b)  order  1.  (d)  x = y + 3 xy  .  is  (a)  y¢¢ =  y¢y  (b)  yy¢¢  =  y¢  (c)  yy¢¢  =  (y¢) 2  (d)  y¢  =  y 2  (2009)  The  solution  of  the  differential  equation  5.  3.  degree  2  (d)  order  1.  7. 0  < x  < p/2  is  (a)  sec x = (tanx + c)y  (b) y secx = tanx + c  (c) y tanx = secx  + c  (d)  tanx = (secx + c)y  (2010)  The differential equation which represents the family of curves  y  =  c 1 e c 2 x .  where c > 0.  (2005)  The  solution  of  the  differential  equation  ydx  +  (x +  x 2 y)dy  = 0  is  1  1  (a)  xy + log y =  C  (b)  .xy =  C  (d)  logy  =  Cx. to  ¥ dy (b)  y 2 = x 2  - 2 xy  is  11.  (2004)  10.  y  (a)  x log æç ö÷ = cy  è x ø  æ x ö (b)  y log ç ÷ = cx  è y ø  æ x ö (c)  log ç ÷ = cy  è y ø  y  (d)  log æç ö÷ = cx .  DIFFERENTIAL EQUATIONS Solution  of  the  differential  equation  cos xdy  = y(sinx  – y )dx .  degree  1  (c)  order  2.  degree  2  (2008)  The  differential equation  of the family of  circles with fixed  radius 5 units and  centre on the line y  = 2  is  (a)  (x  – 2) 2  y¢ 2  = 25  – ( y –  2) 2  (b)  (x  – 2) y¢ 2  = 25 – ( y – 2) 2  (c)  ( y – 2) y¢ 2  = 25 – ( y  – 2) 2  (d)  ( y – 2) 2  y¢ 2  = 25  – (  y  – 2) 2  (2008)  9.  where  c 1  and  c 2  are  arbitrary  constants.  è x ø  (2005)  The  differential  equation  representing  the  family  of  curves  y 2  = 2 c ( x +  c ). is of order and  dy x + y  =  dx x satisfying the condition  y(1)  = 1  is  (a)  y =  x ln x +  x  (b) y  =  ln x +  x  (c)  y =  x ln x +  x 2  (d) y  =  x e (x  – 1)  4.xy + log y =  C  1  (c)  .63  Differential Equations  CHAPTER  11  1.  If  x dy  = y (log y . where  a  is  an  arbitrary  constant is  origin and having  their centres  on the  x­axis  is  (a)  (x 2  –  y 2 )y¢  =  2xy  (b)  2(x 2  +  y 2 )y¢  =  xy  dy  (a)  y 2 = x 2  + 2 xy  (c)  2(x 2  –  y 2 )y¢  =  xy  (d)  (x 2  +  y 2 )y¢  =  2xy. is a parameter.  The  solution  of  the  differential  equation  dy  (1 + y 2 ) + ( x ..  2.  (c)  (d)  (2004)  (2007)  1 +  x x dx The  differential  equation  whose  solution  is  Ax 2  +  By 2  =  1.   The  solution  of  the  equation  (c)  2.  (a)  5.  (a)  (2002)  (2002)  .2 x  e + c + d .  (2003)  dx 2  14.  (d)  14.  1  (c)  3.  (c)  8.  4.  are  respectively  (a)  1.  (a)  2.64  JEE MAIN CHAPTERWISE EXPLORER  13.  2  (b)  3.2 x  e + cx 2  + d 4  (d)  1  .2 x  15.  (a)  6.  (d)  13.  (d)  12.  (c)  3.  3  (d)  2.  1.  (a)  11.  (b)  3.  (a)  7.  The degree and order of the differential equation of the family  2  (a)  1.  (b) (d)  1.  2  d 2 y  =  e .2 x  e + cx + d 4  (c)  1  .  (d)  10.  3  3  of  all  parabolas  whose  axis  is  x­axis.  4  Answer  Key  1.  (b)  15.  2.  (a)  9.  The  order  and  degree  of  the  differential  equation 2  3  (1 + 3 dydx )  =  4  d 3 y  are  dx 3  (a)  1  - 2 x  e 4  (b)  1  .   dx 2  2  (d)  :  Given  A  x  +  B  y  =  1  As  solution  having  two  constants.sec 2  x dx + k Þ v sec x = .t..( y - 2)  dx From (1) and (2) on eliminate ‘a’  .  - y  = ln x + k  x Þ  y =  x ln x + kx  As before. so that  dx dx dv x + vx  = 1 + v  We have  v + x = dx x dv  dx  Þ x  = 1  Þ dv =  dx x dx  x  dy  ( y . .  x.. Then y =  x ln  x  +  x  2nd Method  (Inspection) :  Rewriting the  equation  dy x + y  =  as  dx x xdy – ydx = xdx  dx = -  cos 2 x On integration. we get  2( x ...  (i)  B x dx Differentiating  (*)  w. = e ò xdy .tan x = - sec x Þ  y 2  dx y 4.y )  = y tan x .r.  y ¢ = c1c2ec2 x  = c2 y .r. (i) .. y  = 1 giving  1 = 0 + (ln k) \  ln  k  = 1..(1)  ...tan x + k Þ 2..t.  only  choices  (a)  and  (b)  are  possible  Again A  x 2  +  B  y 2  =  1  . which is linear in v .  curves.  dx = e ln sec x = sec x The  solution is  v ´ sec x = ò ..h ) + 2 y dy  = 0  dx dy  h = x +  y  to  eliminate  h.ydx (y – 2) 2 (y¢) 2  = 25 – (y – 2) 2  (a) : General equation of all such circles is  (x – h) 2  + (y – 0) 2  = h 2  .a ) + ( y ..a = .2) 2  = 25  è dx ø sec x = .  (a) : 1st solution:  cos x dy  = y (sin x – y )dx Þ  cos x dy  = y  sin x dx  – y 2 dx Þ  cos x dy  – y  sin x dx  =  –y 2 dx Þ d(y cos x) = –y 2 dx Þ  d( y cos x)  ( y cos x )2 Þ v = ln x + ln k As v =  y/x we have y =  x  ln x + (ln k)x  At x  = 1.(ii)  y¢ y  = Þ y ¢¢y = ( y ¢ )2  y ¢¢ y ¢  Which  is  the  desired  differential  equation  of  the  family  of  6.y 2 sec x = Þ dx dx cos x dy .C Þ sec x = y(tan x + C ) y Differentiating w.  we  get  3.  (*) A  y dy  ..y tan x = ...  x  y¢¢ = c 2 y¢  From (i) and (ii) upon  division  .t.  y  =  x ln  x  +  x  (d) :  The  equation  to circle is  (x  – a) 2  + (y – 2) 2  = 25  Differentiation  w.  putting  value  of  h  in  dx equation .2) 2 æ ö + ( y ..y 2 sec x Þ  dx 1 dy 1  . evaluating constant. \  order  of  differential  equation is 2 so our choices (b) & (c) are discarded from the  list.  dv + (tan x )v = - sec x .r..=  Þ  .t.tan x .65  Differential Equations  1. we have I.F..(2)  2  Þ  (c) :  y = c1 e c2 x  5.  =  On integration  1  tan x dx x 2  y  dx  d æ ö = è x ø  x  Setting.2) = 0  dx dy  Þ x .  x  dy  ( x .  (a) : 1st Method  (Homogeneous equation):  dy dv  = v +  x  Let y =  vx.. dy  \  we get  y 2 = x 2  + 2 xy  ..  x  Again  on  differentiating .(i)  Again differentiating  w.y = v . (i)   where h is parameter Þ  (x – h) 2  +  y 2  =  h 2  Differentiating.. we  have Þ  – sec x  = –y  tan x  + yk Þ  sec x  = y (tanx  + C) where C  is a  constant  2nd  solution:  We have  dy dy y(sin x .r. 66  JEE MAIN CHAPTERWISE EXPLORER  y .  (a)  :  x  =  e Þ  x  =  e y  +  x  Differentiate  w. dy 1  æ y ö dv dx  = Þ log çè ÷ø  = cx ..  to  x) Þ  order 2 and degree 1 (Concept: Exponent of highest order  derivative is called degree and order of that derivative is called  order  of  the  differential  equation.F.  (ii) æ dy ö + ç ÷ =  0  Þ  2  è dx ø  dx (by  differentiating  (ii)  w.  v log v x x 2 dx tan -1  y  +  1x  =  e dy  e tan  y  dx  1  + x  Þ x ∙ I.  (a)  :  Given  family  of  curve  is  x 2  +  y 2  –  2ay  =  0  Þ  2a  =  Þ  e 2 tan  y  +c 2  -1  = e 2 tan  y  + k =- y  2  x  (a .  (a)  :  From  the  given  equation  (1  +  y 2 )  dy  = y (log y ..  x) dy  y  Þ  =  2a  dx  - 1  Þ  ò 1 + y 2  dy  -1  - 1  tan  Þ  x e 2  2  ò y × I..t.  =  Þ =  dy  1 +  y 2  1 + y 2  ö dy yæ æ y ö y  \ = ç log ç ÷ + 1 ÷ Now  put  = v  dx xè è xø x ø  \  v  logv  dx  =  x  dv  Þ 11.r.(1) 2  x +  y  y Also  from  (1).  (c) :  ç 1 + 3  ÷ =  4 çç 3  ÷÷ dx ø  è è dx ø 3  2  é d 3 y ù dy ö æ Þ  çè 1 + 3  ÷ø  =  ê 4  dx3  ú dx ë û \  highest  order  is  3  whose  exponent  is  also  3.t. (b) :  Given  \  \  dx 2  =  e –2x dy e -2 x  =  +  c dx  - 2  e - 2 x  y  =  +  cx  +  d 4  .  (d)  :  x 12..÷ = è xy ø y Þ  1  -  =  –log  y  +  C xy y  -1 y tan Þ  x e -  d ( xy )  2  ( xy )  10.r.1  = e tan  13.  degree  3.  2x  +  2yy¢  –  2a  y¢ =  0 Þ  y  =  d 2 y 1  + log  y  =  C  xy 2 ..  xy d 2 y æ dy ö æ dy ö + x ç ÷ = y ç ÷ 2  dx dx è ø è dx ø Þ  order  2  degree  1.  Þ  order  1.dy Þ y Þ  dy  æ 1  ö d ç .  (i) ..r..(i) \  yy 1  =  c  2 3  2  y = 2 × yy1 x + yy1 Þ ( y .) dy  y  tan  Þ 2 x e y  - 1  2 tan  =  e y  2  .2 xyy1 ) = 4 ( yy1 )  2  3  3 1  Þ  (y  –  2xyy 1 )  =  4y y  9..  x  after  taking  logarithm  both  sides dy 1  dy 1 -  x  Þ \  =  1 + =  dx x  dx  x æ x 2 + y 2  ö ç ÷÷ y¢ =  0 2x +  2yy¢  –  ç y è ø  2xy  +  y¢(2y 2  –  x 2  –  y 2 )  =  0 Þ  y¢(x 2  –  y 2 )  =  2xy  +  k  æ d 3 y ö dy ö 3  æ 14.log x + 1)  dx 8.  d 2 y  15..= B .  (a): As  axis of  parabola  is  x­axis  which  means focus lies on x­axis.(ii)  2  7.....F = e (d)  : y = 2 c ( x +  c )  \  2yy 1  =  2c Now putting  c  =  yy 1  in  (i)  we  get ( ) . Equation of such  parabola’s  is  given  by  y 2  =  4a(x  –  k)  Þ  2yy 1 =  4a  (by  differentiating  (i)  w.  (b)  :  y  dx  =  –(x 2 y  +  x)  dy Þ  ydx +  xdy  =  –x 2 y  dy Þ  ydx +  xdy  2  ( xy )  =  ....  -1  where  I.F.  y + e y + e  2  æ d 2 y ö æ dy ö y ç 2  ÷ + ç ÷ è dx ø  è dx ø By  (i)  and  (ii)  we  get  A  . 0)  .  –2)  and touching  the  axis of x  at  7.  Statement­2  is  not  a  correct  explanation  for  Statement­1. Statement­2 is a  correct  explanation  for  Statement­1. If the centre  of the ellipse is at the origin and its axes are the coordinate  axes.  Statement  2  is  false.  Statement­2  :  If    the  line. then the  (2013)  slope of the  line PQ  is  The x­coordinate  of  the  incentre  of  the  triangle  that  has  the  (a)  – 2  (b)  – 1/2  coordinates of mid points of its sides as (0.  3y = x -  3 3y = x - 1 (b)  y = 3x -  3 (d)  y = x +  3 (2013)  The  equation  of  the  circle  passing  through  the  focii  of  the  x 2 y2  + =  1 and having centre at  (0.  Statement­2  is  true.  tangent to the parabola  y 2  = 16 3 x the ellipse  2x 2  + y 2  =  4. 0)  (c)  – 1/4  (d)  – 4  (2012)  is  2  2  2  2  2  11. 4) in the  12.  (d)  Statement  1  is  true.67  Two Dimensional Geometry  CHAPTER  12  1. 5  (m ¹  0) is  their  m common tangent.  (c)  Statement­1  is  false.  (2013)  3.  y 2  = 16 3 x and the ellipse  2 x 2 + y 2  = 4 is y = 2 x + 2 3 Statement 2 : If the line  y = mx +  4 3 . then  the equation  of the ellipse is  (a)  4x 2  + y 2  = 8  (b) x 2  + 4y 2  = 16  (c)  4x 2  + y 2  = 4  (d) x 2  + 4y 2  = 8  (2012)  10. The  bisector  of  the  ratio 3 : 2. (m ¹  0) is a common  m The  length  of  the  diameter  of  the  circle  which  touches  the  x­axis at the point (1.  (3.  Statement­2  is  true.  The lines L 1  : y  – x  = 0 and L 2  : 2x  + y  = 0 intersect the line L 3  : y  +  2 =  0 at P  and Q  respectively. 3) is  16 9 2  2  (a) x  + y  – 6y  +  7 =  0  (b) x 2  + y 2  –  6y  –  5 = 0  ellipse  5. 0) also passes  through the  point  (a)  (2. (1.  (d)  Statement­1 is true. then m  satisfies m 4  –  3m 2  +  2 = 0. the  equation  of  the reflected  ray  is  (a)  (c)  4. Statement­2 is true.  Statement  2  is  true;  Statement  2  is  a  correct  explanation  for  Statement  1. where O  (c) x 2  + y 2  – 6y  +  5 =  0  (d) x 2  + y 2  –  6y  –  7 = 0  is the origin. 2x 2  +  2y 2  = 5  and a  parabola y2  =  4 5x .  (b)  Statement  1  is  true.  –2)  (c)  (–2.  (b)  Statement­1  is  true.  9. 5)  (d)  (–5.  (a)  6  (b)  11/5  (c)  29/5  (d)  5  Statement­1 : The ratio PR  : RQ  equals  (2012)  . 0) and passes through the point (2.  2.  (a)  Statement  1  is  true. 1) and (1.  (2012)  curves is  y = x +  5. y = mx + (a)  Statement­1  is  true.  Statement  2  is  true. 2)  (2013)  Statement 1 : An equation of a common tangent to the parabola  Given : A circle.  TWO DIMENSIONAL  GEOMETRY The  circle passi  ough (1.  Statement­1  :  An  equation  of  a  common  tangent  to  these  then m  satisfies m 4  + 2m 2  = 24.  8. 3) is  (a)  6/5  (b)  5/3  (c)  10/3  (d)  3/5  (2012)  An  ellipse  is  drawn  by  taking  a  diameter  of  the  circle  (x – 1) 2  + y 2  = 1 as its semi­minor axis and a diameter of the  circle x 2  + (y  – 2) 2  = 4 as its semi­major axis.  (c)  Statement  1  is  false. then k  equals  acute angle  between L 1  and L 2  intersects L 3  at R . 2) to meet the coordinate  axes at P  and Q  such that it forms a triangle OPQ.  Statement  2  is  true;  Statement  2  is  not  a  correct  explanation  for  Statement  1.  1)  and  (2. If the area of the triangle OPQ  is least. 1).  A line is drawn through the point (1.  6. –5)  (b)  (5.  Statement­2  is  false.  A ray of light along x + 3y =  3 gets reflected upon reaching x-axis.  The two circles x  + y  = ax and x  + y  = c  (c > 0) touch each  (a)  2 -  2 (b)  1 +  2 other  if  (c)  1 -  2 (d)  2 +  2 (2013)  (a)  a =  2c  (b)  |a| = 2c  (c)  2  |a| =  c  (d)  |a| =  c  (2011)  If the line 2 x + y  = k  passes through the point which divides  the  line  segment  joining  the  points  (1.   The point diametrically opposite to the point P(1. – 4)  (2008)  16.  is  (c)  exactly  one  value  of  p  (a) y = 0  (b) y = 1  (d)  all values of p  (2009)  (c) y = 2  (d) y = 3  (2010)  (a)  3x 2  + 5y 2  – 15 =  0  (c)  3x 2  + 5y 2  – 32 =  0  24. then  20. Then a possible value  19. 0)  (d)  (0. 4) and Q(k. Statement­2 is true; Statement­2 is a  triangle ABC  is at the  point  correct  explanation  for  Statement­1. 0  .  Equation of the ellipse whose axes are the axes of coordinates  of the  ellipse is  and  which  passes  through  the  point  (–3. 0) is equal to 1/3. 0  3  ( )  (d)  (0.  The  shortest  distance  between  line y  – x  =  1  and  curve x  = y 2  is  (a)  8  (b)  4  (c)  3  4  (d)  (c) ( )  5  .  The equation of the tangent to the curve  x 2  (b)  all except two values  of  p  parallel  to the x ­axis.  The  line L  given  by  + = 1  passes  through  the  point  (2008)  5 b (13. which in turn is inscribed in another  (2011)  ellipse that passes through the point (4. y) meets the x­axis at G.  If  P  and  Q  are  the  points  of  intersection  of  the  circles  (b)  5x 2  + 3y 2  –  32 = 0  x 2  + y 2  + 3x + 7y + 2p – 5 = 0 and x 2  + y 2  + 2x + 2y – p 2  = 0.  32). 3) has y­intercept – 4. – 4)  23  17  23  (a)  (b)  17  (c)  (d)  (c)  (– 3.  (2011)  ( )  13.  2  2  (d)  5x  + 3y  – 48 = 0  (2011)  then there is a circle passing through P.  then  the locus  of P  is  (a) x = 1  (b)  2x + 1 = 0  (c) x = –1  (d)  2x – 1 = 0  (2010)  15  15  x 2  + 17  (2010)  26.  A focus of an ellipse is at the origin. Then the vertex of the parabola is at  m  (a)  (2.  0)  to  the  distance  from  the  (a)  Statement­1  is  true.  The shortest distance between the line y – x = 1 and the curve  of  k  is  x =  y 2  is  (a)  – 4  (b)  1  (c)  2  (d)  –  2  2 3  3 2 3  3 2 (a)  (b)  (c)  (d)  (2008)  8  5  4  8  (2009)  28.  bisector  of an  angle divides  21.68  JEE MAIN CHAPTERWISE EXPLORER  Statement­2 :  In  any  triangle.  Then the length of the semi­  2  major axis is  5  8  2  4  (b)  (c)  (d)  (a)  x y 3  3  3  3  17. The directrix is the line  1  x = 4 and the eccentricity is  . 4)  (b)  (3.  The circle at two distinct points if  (a)  –85 < m  < –35  (c)  15 < m < 65  (b)  –35 < m <  15  (d)  35 < m <  85  (2010)  27.  where c1  and c2  are arbitrary  constants.  Statement­2  is  true;  Statement­2  is  . If the  distance of G from the origin is twice the abscissa of P.  Q and  (1. 0)  (2009)  3 2 8  22. 0). 0  (a) (b) 4  2  not  a  correct  explanation  for  Statement­1. 0) on the  x y + = 1 .  The normal to a curve at P(x. 2)  (c)  (1.  (2007) (c)  yy ¢¢  = (y¢ ) 2  (d)  y¢  =  y 2  (2009)  . Then the equation  14. 0)  (b)  (0. is  (a)  circle  (b)  hyperbola  (a)  y¢¢  = y ¢ y  (b)  yy ¢¢  = y ¢  (c)  ellipse  (d)  parabola.  one  of them  from  the  point (1.  5  5  (d)  Statement­1  is  true. Then  the distance  between L  and K  is  circle x  2  + y  2  + 2x + 4y – 3 = 0 is  c 3 (a)  (3.  Statement­2  is  true.  If  two  tangents  drawn  from  a  point P  to  the  parabola y 2  = 4x  are  at right angles.  Three distinct points A. 1)  (2008)  y 2  = 4x  + 8y + 5 intersects the line 3x  – 4y  = 18.  1)  and  has  (a)  x 2  + 12y 2  = 16  (b)  4x 2  + 48y 2  = 48  2  (c)  4x 2  + 64y 2  = 48  (d)  x 2  + 16y 2  = 16  (2009)  eccentricity  is  3 2  3  5  23.  point ( –1. 1) for  4  y = x +  .  (b)  Statement­1  is  false.  The differential equation which represents the family of curves  the  curve  is a  y = c1 ec2 x . Then the  circumcentre of  the  (c)  Statement­1 is true. (a)  all  except  one value  of  p  that is  15.  The ellipse  x 2  + 4y 2  = 4 is inscribed in a rectangle aligned  with the coordinate axes.  Statement­2  is  false.  The  perpendicular  bisector  of  the  line  segment  joining  P(1. 4)  (d)  (– 3.  A  parabola  has  the  origin  as  its  focus  and  the  line  x = 2 as the directrix. B and C are given in the 2­dimensional  coordinate  plane  such  that  the  ratio  of  the  distance  of  any  the triangle  into two similar triangles. The  line K  is  parallel  to L  and  has  the  equation  25.   31.  The locus of a point P (a. which of the following  2 cos a sin  a  remains constant  when a  varies  ?  (a)  abscissae  of  vertices  (b)  abscissae  of  foci  (c)  eccentricity  (d)  directrix.  then  the  equation  of  the  locus  of  è 2 ø  its  centre  is  1 ö æ 1  ö æ (c)  ç . 3 ÷ (d)  ç -3.  2). 0) and  R =  (3.  Its  equation  is  angled triangle with AC as its hypotenuse.  For the hyperbola  2  y 2  line y = ax + b  is a tangent to  the hyperbola  x 2 .  If a circle passes through the point (a. 1) and are tangent to x­axis.69  Two Dimensional Geometry  29. b) and cuts  the  circle  æ 1 ö (a)  ç 0. (a)  (2.  The locus  of  the centre  of  the  circle  is  2  (a)  a  circle  (b)  an  ellipse  27  9  (c)  x 2 + y 2  =  (d)  x 2 + y 2  =  .  x  >  0.  k). 0).  2}.  Let C be the circle with centre (0.  An ellipse has OB as semi minor axis.2  = 1.  (2007)  such  that  the  area  of one  of  the  sectors is  thrice  the  area  of  33.  1)  be  the  vertices  of a  right  between  the  axes  is  bisected  at  A. ¥)  2  2  2  x  +  y  =  p  orthogonally.  then  m  is  (a)  1  (b)  2  (c)  –1/2  (d)  –2. 3}  (b)  {–3.  (d)  xy =  105  (b)  xy =  (2006)  40.2  = 1  is  a b (a)  a  circle  (b)  an  ellipse  (c)  a  hyperbola  (d)  a  parabola.  (2007)  34. 3)  and  radius 2.  (2006)  (c)  0 £ k £  1  (d)  k ³  1 .  0)  (2005)  (c)  (–1. Then the  eccentricity of  the  x  ellipse  is  2  35.  then  a  belongs to  44. If the area of the  (a)  x  +  y  =  7  (b)  3x  –  4y  + 7  =  0  triangle is 1 square unit.  (2006)  (a)  2ax  +  2by  –  (a 2  –  b 2  +  p 2 )  =  0  è2 ø  è 2 ø  (b)  x 2  +  y 2  –  3ax  –  4by  +  (a 2  +  b 2  –  p 2 )  =  0  36.  (d)  x + 2  3 x + y = 0  (2007)  is  8. then the set of values which ‘k’ can  (c)  4x  +  3y  =  24  (d)  3x  +  4y  =  25. then the equation  of  the  of  the  centre  of  the  circles.  Let P = (–1. If  (h. Q = (0. x > 0  1  1  2  1  1  (b)  (c)  (d)  .  The  equation  of  a  tangent  to  the  parabola  y 2  =  8x  is  another  sector  then  y = x + 2. b) moving under the condition that the  y 2  x 2  .  (2005)  C  that  subtend  an  angle  of  2p/3  at  its  centre  is  45. k) are the coordinate  of a circle of area  49p  square  units.  (2007)  2  2  38.  A circle touches the x­axis and also touches the circle with centre  3  2 2  (a)  x + y =  (b)  x 2  +  y 2  =  1  at  (0.  (2007)  42.  .  Consider a family of circles which are passing through the  37.  (a)  3/5  (b)  1/2  (c)  4/5  (d)  1/ 5 .  ÷ (b)  (3.  If one of the lines of my 2 + (1 – m 2 )xy – mx 2 = 0 is a bisector  of the angle  between  the lines  xy  = 0.  Let  A(h. (2005)  (a)  2  4  2  3  and  y  =  3x.  In an ellipse.  1)  and  C(2.  then  the  set  of  values  of  k  is  circle  is  given by  the  interval  (a)  x 2  +  y 2  +  2x  –  2y  –  47  =  0  (b)  x 2  +  y 2  +  2x  –  2y  –  62  =  0  1 1  1  k £  (a)  . Then  its  eccentricity  is. 0) and radius 3 units. The  (c)  2ax  +  2by  –  (a 2  +  b 2  +  p 2 )  =  0  equation of the locus of the mid points of  chord of the circle  (d)  x 2  +  y 2  –  2ax  –  3by  +  (a 2  –  b 2  –  p 2 )  =  0.  If the lines 3x – 4y – 7 = 0 and 2x – 3y – 5 = 0 are two diameters  point (–1. 3}  (d)  {0. –2}  diameters  of  a  circle  and  divide  the  circle  into  four  sectors  (c)  {1.  (2005)  43.  A straight line through the point A(3.£ k £  (b)  2 2  2  (c)  x 2  +  y 2  –  2x  +  2y  –  62  =  0  (d)  x 2  +  y 2  –  2x  +  2y  –  47  =  0.  (2006)  take is  given by  2  2  41. 4)  (b)  (–2. the distance between its focii is 6 and minor axis  30.  (2006)  The  locus  of  the  vertices  of  the  family  of  parabolas  a 3 x 2 a 2 x  + - 2 a is  3 2  105  (a)  xy =  64  35 (c)  xy =  16  y= 3  4  64  .  (2007)  39. 3 3) be three points. F and F ¢  its focii and  the angle FBF ¢  is a  right angle. 1)  (d)  (0. The point on this line from which the other tangent  (a)  3a 2  –  2ab  +  3b 2  =  0  (b)  3a 2  –  10ab  +  3b 2  =  0  to the  parabola is  perpendicular  to  the given  tangent is  (c)  3a 2  +  2ab  +  3b 2  =  0  (d)  3a 2  +  10ab  +  3b 2  =  0. a  ) falls inside the angle made by the lines  y = .  If  the  pair  of  lines  ax  +  2(a  +  b)xy  +  by  =  0  lie  along  (a)  {–1.  The  equation  of  the  bisector  of  the  angle  PQR  is  3  x + y = 0  (a)  2  (b)  x + 3 y = 0  (c)  3  y =  0.  (c)  a  parabola  (d)  a  hyperbola. 4) is such that its intercept  32.  (2005) (2006)  4  4  .  B(1.  If (a.÷ .  b) and cuts  the  circle  x 2  +  y 2  =  4  orthogonally. then the centroid  A(p.  If non­zero numbers a.  If the sum of the slopes of the lines given by x 2  – 2cxy – 7y 2  = 0  is  four  times  their  product.  (2004)  1 7  7  (c) 3 .  (2004) (2004)  . then the equation of the  ellipse  is  (a)  4x 2  +  3y 2  =  12  (b)  3x 2  +  4y 2  =  12  (c)  3x 2  +  4y 2  =  1  (d)  4x 2  +  3y 2  =  1. 3)  and  making  intercepts  on  the  co­ordinate  axes  whose  sum is –1  is  x y  y  + =  1  and  x + =  1  2 3  2 1  x y  y  x + = 1  (b)  . is  1/2. y = a sinq at q always  is  AB.  (2005)  51.  1)  and  the  mid  points  of  two  55.P.  (a) (b) - 1.  (2004)  59.  If  the  circles  x 2  +  y 2  +  2ax  +  cy  +  a  =  0  and  54.  then  (a)  d 2  +  (2b  –  3c) 2  =  0  (b)  d 2  +  (3b  +  2c) 2  =  0  the  locus  of  the  vertex  C  is  the  line  (c)  d 2  +  (2b  +  3c) 2  =  0  (d)  d 2  +  (3b  –  2c) 2  =  0. The locus of the other end of  the  of  the  triangle  is  diameter through  A  is  - 1 7  7  (a)  (y  –  p) 2  =  4qx  (b)  (x  –  q) 2  =  4py  .  The  equation  of  the  straight  line  passing  through  the  point  (4. b.=  1  and  2 3  - 2 1  (a)  (2004)  52.  If  the  lines  2x  +  3y  +  1  =  0  and  3x  –  y  –  4  =  0  lie  along  diameters of a circle of circumference 10p.  –2)  (b)  (–1. with its  centre at the origin.  Equation  of  the  circle  on  AB  as  a  diameter  is  2  2  2  2  passes  through  the  fixed  point  (a)  x  +  y  +  x  +  y  =  0  (b)  x  +  y  –  x  +  y  =  0  (a)  (0.1  (c)  2 3  - 2 1  x y  y  x + = 1.  (2004)  53.  then  the  locus  of  its  centre  is  48.  Let A(2.  (2005)  57.  (d)  .  If one of the lines given by 6x 2  – xy + 4cy 2  = 0 is 3x + 4y = 0.  (2004)  58.  The normal to the curve x = a(1 + cosq).  If  a  vertex  of a  triangle  is  (1.  -  2 (d)  (1. then the straight line  (a)  2ax –  2by  +  (a 2  +  b 2  +  4)  =  0  x  y  1  + + =  0  always passes through a fixed point. That point  a b c (b)  2ax +  2by  –  (a 2  +  b 2  +  4)  =  0  is  (c)  2ax +  2by  +  (a 2  +  b 2  +  4)  =  0  (a)  (–1.  (c)  (a.  ( )  ( )  )  below  the  x­axis  at  a  distance  of  2/3  from  it  below  the  x­axis  at  a  distance  of  3/2  from  it  above  the  x­axis  at  a  distance  of  2/3  from  it  above  the x­axis  at  a  distance  of  3/2  from  it. 0) and Q a point on  the locus y 2  = 8x.  a)  (c)  x 2  +  y 2  –  x  –  y  =  0  (d)  x 2  +  y 2  +  x  –  y  =  0.  The eccentricity of  an ellipse. 2) and (3.  then  c  equals  49.  A  variable  circle  passes  through  the  fixed  point  sides through this vertex are (–1.  a).  (2004)  47.  2)  (d)  2ax –  2by  –  (a 2  +  b 2  +  4)  =  0.  The intercept on the line y =  x by  the circle x 2  +  y 2  – 2x = 0  61.  –2).  of the lines ax + 2by + 3b = 0 and bx – 2ay – 3a = 0.= - 1  and  2 3  - 2 1  x y  y  + = - 1  and  x + = .  0)  (b)  (0.70  JEE MAIN CHAPTERWISE EXPLORER  46. q) and touches x­axis. –3) and B(–2.  The  locus  of  mid  point  of  PQ  is  (a)  x 2  –  4y  + 2  =  0  (b)  x 2  +  4y  +  2  =  0  2  (c)  y  +  4x  + 2  =  0  (d)  y 2  –  4x  +  2  =  0.  (2004)  1  (c) 1. then  the equation  x 2  +  y 2  –  3ax  + dy  – 1  = 0  intersect in two distinct points  P  of  the  circle  is  and Q then the line 5x + by – a = 0 passes through P and Q for  (a)  no  value  of  a  (a)  x 2  +  y 2  +  2x  +  2y  –  23  =  0  (b)  exactly  one  value  of  a  (b)  x 2  +  y 2  –  2x  –  2y  –  23  =  0  (c)  exactly  two  values  of  a  (c)  x 2  +  y 2  –  2x  +  2y  –  23  =  0  (d)  infinitely  many  values  of  a.  3  (d) 1.  (2005)  56.  60. 2).  (2005)  (d)  x 2  +  y 2  +  2x  –  2y  –  23  =  0. b) ¹ (0. 0)  is  ( )  ( )  ( (a)  (b)  (c)  (d)  50.  The line parallel to the x­axis and passing through the intersection  (a)  3  (b)  –  1  (c)  1  (d)  –  3. c are in H.  If a circle passes through the point (a. 1) be vertices of a triangle ABC. 3  .  0)  (d)  (a. If the  then  centroid  of this  triangle moves  on  the line  2x  +  3y  =  1. If one of the directrices is x = 4.. where  (2004)  (a.  then  c  has  the  value  (a)  2  (b)  –1  (c)  1  (d)  –2.  (a)  3x +  2y  =  5  (b)  2x –  3y  =  7  (2004)  (c)  2x +  3y  =  9  (d)  3x –  2y  =  3.  3 3 3 2  (c)  (x  –  p)  =  4qy  (d)  (y  –  q) 2  =  4px.  (2005)  Let P be the point (1.  If a ¹  0 and  the line 2bx +  3cy +  4d =  0 passes  through the  2  2  points of intersection of the parabolas y  = 4ax and x  = 4ay.   A point on the parabola y 2  = 18x at which the ordinate increases  69. units.  A triangle with vertices (4.  The  normal  at  the point (bt 12   .  The centre of the circle passing through (0. 0).  then  - 9 9  2  . 0) and (1. (3.  The  foci  of  the  ellipse  x  + 2  = 1  and  the  hyperbola  16  b (a 1  –  a 2 )x  +  (b 1  –  b 2 )y  +  c  =  0. (b sint.  72. each of radius 3.    b 2)    is  70. (–1.  If  the  pair  of  lines  68. where t is a parameter.  If  the  pairs  of  straight  lines  x 2  –  2pxy  –  y 2  =  0  and  2 2  2 3 1  1 3  x 2  – 2qxy –  y 2  =  0 be  such that  each  pair  bisects the  angle  . Then the equation of the circle is  76.a22 + b12 - b2 2  144 81 25  1  (a)  5  (b)  7  (c)  9  (d)  1. then  intersect  on  the  y­axis  then  (a)  r <  2  (b)  r  =  2  (a)  2fgh =  bg 2  +  ch 2  (b)  bg 2 ¹ ch 2  (c)  r >  2  (d)  2  <  r  <  8.  If  the  two  circles  (x  –  1) 2  +  (y  –  3) 2  =  r 2  and  ax 2  +  2hxy  +  by 2  +  2gx  +  2fy +  c  =  0  x 2  + y 2  – 8x + 2y + 8 = 0 intersect in two distinct points.  The lines 2x – 3y = 5 and 3x – 4y = 7 are diameters of a circle  (2002)  having area as 154 sq.  (2003)  1 1  1  .71  Two Dimensional Geometry  62.  (b)  ( a12 + a22 + b12 + b2 2 )  2  (2003)  ( )  ( (c)  (a12 + b12 .  (d) (2002)  (c) 2 2  2 2  between  the  other  pair. 5) is  (a)  isosceles  and  right  angled  (b)  isosceles  but  not  right  angled  (c)  right  angled  but  not  isosceles  (d)  neither  right  angled  nor  isosceles  (2002)  64. –b cost) and (1.  .  Then  the  value  of  b 2  is  (a)  a12 . lie on the circle  (a)  x 2  +  y 2  +  2x –  2y  =  47  x 2  +  y 2  =  25. -  2  (b) (a) 65.  The  locus  of  any  point  in  the  set  is  (b)  x 2  +  y 2  –  2x +  2y  =  47  (a)  4 £ x 2 + y 2  £  64  (b)  x 2 + y 2  £  25  (c)  x 2  +  y 2  –  2x +  2y  =  62  2 2  (c)  x + y ³  25  (d)  3 £ x 2 + y 2  £  9  (2002)  (d)  x 2  +  y 2  +  2x –  2y  =  62.  If the equation of the locus of point equidistant from the points  2  y 2  (a 1 .a12 - b1 2 ).  x y p x y p The equation  of  its  diagonal  not  passing  through  the origin  is  75.  2bt 1) on a  parabola  meets  the    2  at  twice  the  rate  of  the  abscissa  is  parabola  again  in  the  point  (bt 2  .  then  74.  (2004)  (c)  (2.  (2003)  4  (a)  x 2 + y 2  =  2  (b)  x 2  +  y 2  =  4p 2  66.  (2003)  (c)  abc  =  2fgh  (d)  none  of  these  (2002) ( ( )  )  ( ( )  )  . 0) and  (c)  (3x +  1) 2  +  (3y) 2  =  a 2  –  b 2  touching  the  circle  x 2  +  y 2  =  9  is  (d)  (3x  –  1) 2  +  (3y) 2  =  a 2  –  b 2 .  b 1)    and  (a 2.a22 - b2 2 )  (d)  1  2 ( a + b22 .  –4)  (a)  t2 = -t 1  +  2  (b)  t2 = t 1  -  8 2 t1  t1  9 9  . –1).  The  equation of a  circle  with origin as a centre  and  passing  through equilateral triangle  whose median is  of length  3a  is  is  (a)  x 2  +  y 2  =  9a 2  (b)  x 2  +  y 2  =  16a 2  2  2  2  2  (a)  (3x –  1)  +  (3y)  =  a  +  b  2  2  2  (c)  x  +  y  =  4a  (d)  x 2  +  y 2  =  a 2 .  (2002)  (b)  (3x +  1) 2  +  (3y) 2  =  a 2  +  b 2  73.  The centres of a set of circles.  (2003)  77.  A square of side a lies  above the x ­axis  and  has one  vertex  p at  the  origin.  The  point  of  lines  represented  by  (a)  y (cosa  +  sina)  +  x  (sina  –  cosa)  =  a  3ax 2  +  5xy  +  (a 2  –  2)y 2  =  0  (b)  y (cosa  +  sina)  +  x  (sina  +  cosa)  =  a  and ^  to  each  other  for  (c)  y (cosa  +  sina)  +  x  (cosa  –  sina)  =  a  (a)  two  values  of  a  (b) "  a  (d)  y (cosa  –  sina)  –  x  (sina  –  cosa)  =  a.  2bt 2 ).  Locus  of  mid  point  of  the  portion  between  the  axes  of  (a)  p  =  –q  (b)  pq  =  1  x  cosa  +  y  sina  =  p  where  p  is  constant  is  (c)  pq  =  –1  (d)  p  =  q.  2  2 )  (2003)  71.  (a) (b)  (2.  . a sint).  4)  (d) 8 2  (c)  t2 = t 1  +  2  (d)  t2 = -t 1  - 2 .  Locus  of  centroid  of  the  triangle  whose  vertices  are  (a cost.  (2002)  angle a (0 < a  < p/4)  with the  positive  direction of x  ­axis. The  side  passing  through  the  origin makes  an  1 1 2  (c)  2 + 2 =  2  (d)  12 + 12 =  4 2  .  (2003)  (c)  for  one  value  of  a  (d)  for  no  values  of  a.  then  c  =  2  2  y  x 1  =  coincide.  67. 0).  (2003)  t1  t1  63.   48.  68.  47.  21.  16.  39.  36.  20.  54.  51.  71.  30.  8.  56.  69.  57.  59.  17.  40.  43.  67.  63.  41.  61.  26.  31.  18.  64.  14.  70.  24.  74.  12.  10. c)  (b)  (c)  (a)  (c)  (a)  (a)  (b)  (a)  5.  79.  25.  29.  42.  77.  13.  72.  78.  (a)  (d)  (d)  (a)  (d)  (c)  (c)  (d)  (c)  (d)  (c)  (a)  (a)  6.  60.  19.  9.  62.  75.  52.  (b)  (d)  (c)  (d)  (d)  (c)  (d)  (b)  (b)  (c)  (c)  (b)  (b)  (b) 2.  46.  (a)  (c)  (c)  (c)  (c)  (a)  (a)  (c)  (d)  (b)  (d)  (d)  (d)  3.  58.  35.  50.  (a)  (a)  (b)  (b)  (a)  (d)  (c)  (d)  (c)  (c)  (c)  (c)  (c)  .  22.  (a)  (b)  (d)  (a)  (a)  (b)  (a)  (c)  (b)  (d)  (d)  (d)  (a)  4.  49.  65.  38.  53.  23.  66.  7.  27.  (2002)  (d)  none  of  these  (2002)  (c)  -1 ±  2 Answer  Key  1.  15.  34.  44.  32.  28.72  JEE MAIN CHAPTERWISE EXPLORER  78.  37.  55.  Two common tangents to the circle  x 2  + y 2  = 2a 2  and parabola  y 2  =  8ax  are  angle of measure 45° at  the major segment of the circle then  value  of  m  is  (a)  x  =  ±(y  +  2a)  (b)  y  =  ±(x  +  2a)  (a)  2 ±  2 (b)  -2 ±  2 (c)  x  =  ±(y  +  a)  (d)  y  =  ±(x  +  a).  11.  If the chord y =  mx +  1 of the  circle x 2  + y 2  = 1  subtends an  79.  33.  73.  45.  (d)  (a)  (c)  (a)  (b.  76.   3 \ x . 3) \  1 + 9 + k 2  – 6k = k 2  Þ k= 5 10  Þ diameter =  3 3 9.  2.  0)  As a 2e 2  = a 2  – b 2  =  7  we  have equation  of  circle as  (x  – 0) 2  + (y  –  3) 2  = ( 7 ... r = 2 Þ b = 4 Þ x 2  y 2  + = 1 Þ x 2 + 4 y 2  = 16  16 4 10.  ç m ÷ = çç .3y .  then m  satisfies m 4  + 2m 2  – 24 = 0  From (1).4 + 2 è ø m è 1  Þ 1  ( x -  3)  .2  è ø  2 4 x 2  1 .  8 14  + = k Þ k = 6  5 5 (d) :  Statement 1 :  y 2  = 16 3x .  (b) :(x  – 1) 2  + y 2  = 1.  1 st  solution  : x ­coordinate  of  incentre  ax + bx2 + cx3  2 ´ 2 + 2 2 ´ 0 + 2 ´ 0  = 1 = a+b+c 2 + 2 +  2 2 4 2  = = = 2 . y = m x +  x 2  y 2  2  + = 1 . 1) ; B  (2.  y 1) divides line segment AB  in the ratio 3 :  2    1 + m2 Þ (1 + m2 )m2  = 2  which gives m 4  + m 2  – 2 = 0 Þ  (m 2  – 1)(m 2  + 2) = 0  As m Î R. 1).  (3.0)2 + (0 . 0) is given by {(x  – 3) 2  + y 2 )} + ly  =  0  As the circle passes through (1. we have  5  2 x1  =  3(2) + 2(1)  8  3(4) + 2(1)  14  =  y1  =  =  5 5 5 5 2x + y = k  passes through P(x 1 .3 =  0 4 3 m 48 m2 = 4+ 2  m1 2 2  ö ÷ ÷ø  = 4 + 2m 2  Þ 24  = 2 + m 2  m 2  m 4  + 2m 2  – 24 = 0  .2 2 ÷ tan = 2 .  (d) : Foci are  given by  (± ae.  (a) : y = mx + c Þ  2  = m + c  Co­ordinates of P  & Q : P (0.  (a) : Let a tangent to  the  parabola be  As it is a tangent to the circle x 2  + y 2  = 5/2. –2). r  = 1 Þ a  = 2  and x 2  + (y  – 2) 2  = 4.73  Two Dimensional Geometry  1.(1) (m 2  + 6) (m 2  – 4) = 0 Þ m = ± 2  Þ Þ  Statement  2  :  If  y = mx +  4 3  is  a  common  tangent  to  m y 2  = 16 3 x and  ellipse  2 x 2  + y 2  =  4. (1.3)2  \ x 2  + y 2  – 6y  – 7  =  0  5.x = .x -  5 3. 4) P(x 1. 1)  happen to be a right  angled triangle with  vertices as shown.  8.3. c). m 2  = 1 \ m  = ±  1  Also m  = ± 1 does satisfy m 4  –  3m 2  +  2 =  0  Hence common  tangents are  7. 0) and  (1. –2) lies  on the circle. y 1 )  \ 2´ 5  y = mx + ( m ¹  0) m æ 5ö çè ÷= m ø  (a) : A  (1.  (b) : The system of circles touches the line y  = 0 at the point  6. 0)  . 4.    A  simple  calculation  shows that (5.  The  point  of  incidence  is  ( 3 . we can determine l which  gives 4 + 4 – 2l  = 0 \ l  =  4  The  circle  is (x  –  3) 2  + y 2  +  4y  =  0.  Hence  the  equation  of  reflected ray is y = \ 3y . Q(–c/m .  x = m1y + 4 m1  + 2 2 4 Þ y= y = x + 5  and  y  = .  (c) : Let the  equation of the circle is (x  – 1) 2  +  (y  – k) 2  = k 2  It passes through (2.2  4+2 2 2 +  2 A 2 nd  solution  : r = ( s -  a)tan  2 æ 4 + 2 2  ö p =ç . statement 2 is a correct explanation for statement 1.4 +  m =  m1  m1  m1 2  2  æ æ4 3ö 2  Now.  (a) : As the slope of incident ray is -  ray has to be  1  3  1  so the slope of reflected  3 .  0)  2  1  c ´ | c | ´ =  A Þ c = A 2 m 2m .  (a) : The triangle  whose  sides midpoints  are  given  to be  (0.   (c) :  Let P  be (y 2 . 3) is the point of contact and equation of the tangent  is y  = 3.  1 1  .  (b) :  The  circle is x 2  + y 2  – 4x  – 8y  – 5 = 0 Þ  (x  – 2) 2  + (y  – 4) 2  = 5 2  Length of perpendicular from centre (2.0)  = 2 2 Þ Þ a a = ±c 2 2 4 2 + 12  12. Þ  2 3  4 Then the equation to line k  is 4x – y  =  –3  The  distance between lines k  and c  is  20 + 3 a = c .  dx x 3  As the tangent is  parallel to x­axis.  The equation of directrix is x = –1.2 + =  A 2m 2m 2 m dA =0 dm 1 2 1 2  Þ =0Þ = Þ m 2  = 4 Þ m = ± 2  2 m2 2 m 2  Q  16. 1) lies on it Þ  2  x 2  y + = 1 2 2  a b 9 + 1  = 1 a 2 b 2  Also  b 2 = a2 æç 1 . 4 x .m |  < 5 Þ (10 + m) < 25  5 Þ  –25 < 10 + m  < 25 Þ  –35 < m  < 15  19..= 1.  x 2 + y 2 = c 2  çè x .  (d) :  y = x +  Let  P(t 2 .  17... we  have  8  1= 0 Þ x 3  = 8. 0) & radius = a and c è 2 ø  2 Thus  .1 / 2 t = - 2 t  The shortest distance between the curves occur along the common  normal. 4) on the line 3x  – 4y  – m  = 0 should be  less than radius. O  divides PQ in the ratio of OP : OQ  which is  2 2 : 5  but it fails to divide triangle into two similar  triangles.  (d)  : The centres and radii are  2  2  æ aö a ±c çè ÷ø  + (0 .(i)  Again  differentiating  w.  we  get  y ¢ = c1c2 ec2 x  = c2 y .  t)  be  a  point  on  y 2  =  x  The  slope  of  normal at  P  = -  1  1  the slope of tangent at  P = .  y = 2 + 2  = 2 + 1 = 3  2 Thus (2. i.t.  x.= 4. 0 ö÷  and (0. hence this is the locus of point P. \ c = -  .74  JEE MAIN CHAPTERWISE EXPLORER  Þ (2 .  (c)  :  y = c1 ec2 x  Differentiating  w. \ | a| = c.b 2 )  Minimum value  =  1  ×  = 23  17  18.  (d)  :  4 a 2 1 4-1 3  = × = 4 ×  1  2 4 2 14. Thus  –2t  =  –1 Þ  t  =  1/2  4 Thus  the  point  P  is  (1/4.÷ø  + y = 2 4 13 32 32 13 8  + =1 Þ = 1=Þ b = - 20  5 b b 5 5 x y Thus the  line is  .(ii) .+ 1 )  ( = 4 2  = 2 3 3 2  =  8  4 2 20..4 m + 4 m 2  =  A Þ =A Þ .y = 20  5 20 The  equation of line parallel  to 4x  – y  = 20 has  slope  4.r.. we have  æ aö a 2  2 . the perpendicular tangents  intersect at the directrix.  Centre æç a .  (d) : As the line  passes through (13.e.a .  (c) : From a property of the parabola. 11.  (c) : Let  the ellipse  be  (–3.  b 2  =  32  7 5 2 The  equation to  ellipse  becomes  3x 2  + 5y 2  = 32  15.  13.  1/2)  x 2  The  required  shortest  distance dy 8  = 1 -  On  differentiation.16 . 32). \ x = 2 3  x 4  So.t.2 ö÷ Þ 5b 2 = 3 a 2  è 5 ø  Upon solving we  get  a = 32 .y + 1|  y  + 1| = y 2  – y  + 1  (Q y 2  – y  + 1 > 0 )  (4 ac .  2 2 As |y 2  – 3 c |6 .r.m) 2  m 2  . y )  Perpendicular distance from P  to x – y  +  1 =  0  is  | y 2  .  (a)  : In triangle OPQ..  x  y¢¢  =  c2y¢  . k)  then  P¢ º  (2h  – a.  0)  As  points  A.  1)  lies  on  the  above  ellipse  4 1  1 1 3  + = 1  Þ = 1 .  is  (2.  (c) :  The vertex is  the  mid  point of  FN.  Q  and  (1.  1). we have  23.  the  corner  of  the  Þ  x +  5y  +  2p  –  5  +  p 2  =  0  a  >  ae  )  e Remark : The question should have read “The corresponding  directrix” in place of “the directrix”.e ö = 4  Þ a æ 2 .  of  the  circles  S1  :  x 2  +  y 2  +  3x  +  7y  +  2p  –  5  =  0  and  7ö k  + 1 ö æ 15 k 2  .ö = 4  è e ø  è 2 ø  Þ a× Þ  9[(x  –  1) 2  +  y 2 ]  =  (x  +  1) 2  +  y 2  Þ a  ..y = .  1)  doesn’t  lie  on  PQ. 0)  F  (0.. the coordinate of  P¢  is given by  P¢ º  (2 × – 1 – 1..  1  +  5  +  2p  –  5  +  p 2 ¹  0 x+ y =  | 2 x |  Þ y dy = x or y dy  = - 3 x  dx dx dx 2  2 Þ  p  +  2p  +  1 ¹  0 Þ  (p  +  1)  ¹  0 \  p ¹  –1  ydy = xdx or ydy = –3xdx  Thus for all values of p except ‘–1’ there is a circle passing through  After integrating.  the  circumcentre  of  triangle  ABC  is  (5/4.  C  lie  on  this  circle.  so  its  equation  is  x 2 y 2  + = 1  16  b2  x = 2  1  the slope of the original line PQ l  Bisector  1  = ( k  .ae =  4  e y2 x2 y 2  3 x 2  = + c or  =+c 2 2 2 2  Þ  x 2  – y 2  = –2c  or 3x 2  + y 2  = 2c Þ x 2  – y 2  =  c 1  or 3x 2  + y 2  =  c 2 .  The midpoint =  è 2 2 ø  A(2. 3)  .  (a)  : The  radical  axis. b) and C(h.4  k - 1  P(1.(i)  P. –2)  Remark : If P be (a. 0)  V  l  = -  rectangle  in  1st  quadrant.1) æ 0 4 Þ = è è 2ø 2  ø  2 2  Þ k 2  – 1 = 15 Þ  k 2  = 16 \  k = ± 4.75  Two Dimensional Geometry  From  (i)  and  (ii)  upon  division  y¢ y  = y ¢¢ y ¢  Þ Þ y ¢¢y = ( y ¢ ) 2  which is the desired differential equation of the family of curves. As C is the mid point P and P¢.  22.  Q  and  (1. – 2)..  21.  1).1  = ( k .e. 1)  it’s necessary  Y .( X -  x )  Þ G º ç x + y × .  (b. we get  .  (a) :  The  slope  of  i.  S2  :  x 2  +  y 2  +  2x  +  2y  –  p 2  =  0  is  S1  –  S2  =  0 28. y) is  dy  ö æ dx  If  there is  a circle passing through  P. 4)  æ k + 1 7ö .  which  reduces  to  x 2 + y 2  - (note that  1  1  Þ a æ .  that is. 0)  N  27.1) æ x è y .  y)  such  that  PM  1  =  where  M º  (1. 0)  (–1.  (a)  : The  given  ellipse  is  3  8  =  4 \ a =  2  3  25. y  = – 4 satisfies it.  Again  the  ellipse  circumscribing  the  rectangle  passess  =- through  the  point  (4. vertex = (1.  0).  (a)  : Let  P  be  a  general  point  (x.2ø è 2  ø  x 2  3  2  + y = 1  Þ  x 2  + 12y 2  =  16  16 4  As x  = 0.  B.  0).  dy  i. 2 × – 2 – 0) C  P¢  P  º (– 3.e.0 ÷ dy dx ø  è and  sufficient    that  (1.1) 2 + y 2  1  = we  have  ( x + 1) 2 + y 2  3  Þ x 2 + y 2  - 5  x + 1 =  0  2  The  locus  is  a  circle  with  centre  (5/4.  0)  and  N º  (–1.= 16  b 2  4 4  b 2  Þ  b 2  = 4/3  Thus the  equation to the desired ellipse is  The equation to the bisector  l  is  Þ 7ö k  + 1 ö æ = ( k  .1)  3 .  (b) : Obviously the major  axis is along the x­axis  The distance between the focus and the corresponding directrix  a  =  - ae  = 4  e (2.  24.  (d)  :  The  centre  C  of  the  circle  is  seen  to  be  (– 1.  the  point  A. Q (k .  0)  PN 3  ( x . 2k – b)  8x 2  +  8y 2  –  20x  +  8  =  0  10  x + 1 =  0  4  x 2 y 2  + = 1  4 1  26. – 4)  (1. c) : Equation of normal at  P(x.  which  in  the  case  of  intersection  of  the  circles  is  the  common  chord.   (i)  .  k)  be  mid  point  of  AB Р AOM =  60°  Now  OA =  OB =  3  and  OM ^  AB  (By  properties  of  circle)  Now  OA = h 2 + k 2  .  B (1. 1)  33.> 0  . 3  3)  120°  60°  Q(0.0).  16  ÷ è ø \  M(h.  (d) :  Let  AB  is  chord  of  circle  and  M(h.0)  Þ  if a  varies then the abscissa  of foci remain constant.  we get  R (3..  (ii)  Solving  (i)  and  (ii)  we  get  1  < a < 3  2  36..  (ii)  (o. 1)  C (2..  k =  y) . 0)  2  x 2  + y  = 1 ( a > b )  a 2 b2  Given  2b  =  8  and 2ae  =  6  38.  –1) \  equation  of  circle  is  (x  –  1) 2  +  (y  +  1) 2  =  7 2 Þ  x 2  +  y 2  –  2x  +  2y  –  47  =  0  Þ Þ  x 2  = }  3 3 35 a  y . o )  b  4  By  (i)  and  (ii)  we  have  ae = 3  b 2  = 16 e 2  Þ  a 2  9  16  1 .  then  P  lies  on  directrix  x = –2 of y 2  = 8x  Hence  P º ( - 2. k )  Þ  3 x + y = 0  2 Q  pr 2  = 49 p  r  =  7  Point  of  intersection  of  AB  and  PQ  is  (1.  (a)  :  Let  (1 – m 2 )x 2  = 0 Þ  m = ± 1.  (c)  :  Given  lines  are  y = 2 ( x > 0)  and  y  =  3x  (x  >  0)  using  (a. k be the locus of the vertex of family of parabola  3 2 2  y = a x + a x  .  (b) :  Q  b 2  = a 2  (e 2  – 1) Þ  sin 2 a  = cos 2 a (e 2  – 1) Þ  tan 2 a  + 1 = e 2 Þ e 2  = sec 2 a  Vertices  º ( ± cos a ..e. where x = h + ...0) º ( ±1.0) Coordinate  of focii  are  ( ± ae. 0)  xy = 0 are y = ± x  Put y = ± x in  my 2  + (1 – m 2 )xy –  mx 2  = 0. y = k  + 4 a  16  a 3  hk  = çæ -3 ÷ö çæ -35 a ÷ö è 4 a ø è 16  ø 105  hk = 64  105  xy =  64  2  (taking  h  =  x. 3.1 | ´ 1 = 1  2  k = –1.  34.  k )  A  F¢  (a.76  JEE MAIN CHAPTERWISE EXPLORER  29.  (i)  2  2  and  a  –  3a  <  0  ..  Angle bisector  Second  method  Equation  of  bisectors  of  lines  P (–1.  a 2 )  in  these  lines  a  a 2  .  32.e2 =  e 2  ( Q  b 2  =  a 2  (1  –  e 2 )  as  a  >  b) Þ  9  3  e =  Þ  5  Þ  O  (o.1 ³ 0 Þ k ³ ..  D ³  0  1  2 k .  2  30. o )  31. 1) then  (–1 – h) 2  + (1 –  k) 2  = k 2 Þ  h 2  +  2h –  2k +  2 =  0.  (a) :  Let h.m )  = 0  m B  Þ  m = ± 1.  (b)  :  Let  P  is  the  required  point. { Þ  æ -3 -35 a ö vertex  is  ç 4 a .2 a 3 2  \  Þ  a 3h 2 a 2 h  + = 2 a 3 2  3k 3h  6  = h 2  + 2 a  a 2  a3 k= 2  Þ  3æ 35a ö æ 3  k+ = h + ö 16 ø è 4 a ø  a 3 è i.  (d) : Equation of circle (x – h) 2  +  (y – k) 2  = k 2  It is passing through (–1. o )  2  .  x  35.  (d)  :  Let  OA  =  r  Given  area  =  49 p  A  P  co­ efficient of  xy co­ efficient of  y 2  3x – 4y  = 7  2x – 3y = 5  O  2  \  Sum of slopes  = - (1 . b )  (o .  (c) :  Slope of the  required  angle bisector is  tan120° = -  3 Hence equation of  the  angle  bisector  is  y = .  (a) : Sum of the  slopes  = - Þ  h 2 + k 2  = 9  9  2 2  Þ  x + y =  4  4  37.  OM = r cos q  h + k = 3cos60 °  3  h 2 + k 2  = 2  A¢  B  F  A  39..3( x - 0)  A (1.  (a) :  1 ´ | k . 2  ÷ a ø è B  (a. if  r 1 .  0)  b  c . 2  44.1  =  m 1  (say)  Slope  of  BF = Þ  a 2  +  a  +  1  =  0    and  ae b a b  2  = m 2  (say)  Slope  of  BF ¢ = 1 3  Þ a+ + =  0  and  –(c  – d  +  b)  =  b/a  - ae 2 4  Now  m 1  ×  m 2  =  –1  Þ b ´ b  = .  (c) :  Let  locus  of  the  centre  of  circle  be  (a..  (c) :  Given  y  = ax  + b  and  1  .a2 a 2 )( -b2 - b 2 )  Now 5ax  +  (c  –  d)y  +  a  +  1 =  0  and  Þ a 2 a 2b2 = . It means given line is the equation  if  (*)  are  equal of common chord and the equation of common chord of  two  \  keeping  D  =  0  in  (*)  we  have  intersecting  circle  is  S 1  –  S 2  =  0  4a 2 a 4b2 = 4( b2 .  (c) :  Let locus of centre be a. b)  is  given  by  x + y  = 1  a b  where a  =  2a  =  6  and b  =  2b  =  8 x  y  \  required  equation  be  + = y 6 8  Þ  4x +  3y  =  24  tan q = If C 1 . 0) Þ  1  –  e 2  =  e 2 . 3) and radius is 2.  41.. b )  B  Mid point (a. b  then according to  given. e = ±  at  A(a.  0).b 4 + a 2 a 2 b 2  5x  +  by  –  a  =  0  represent  same  equation.  (c) :  ax 2 + 2( a + b ) xy + by 2  = 0  (*)  Let q  be  the  angle  between  the  lines  represent  by  *  \ tan q = F¢  y 2  x  =  1  a 2 b 2  Þ a 2  + b 2  –  6b  + 9  = b 2  +  4  +  4b  and a 2  + b 2  –  6b  + 9  = b 2  –  4b  +  4 Þ a 2  –  10b  +  5  =  0  and  x 2  =    2b  +  5 \ b2 x 2 .d  + 1  = .  ae .  0)  B  (0.y =  b .    2e 2  =  1 .  \ 5 a = c .  Þ a2 a 2 b2 = b2 (b 2 + b2 )  Þ a 2 a2 = b2 + b 2  2 2 2 2  Þ a x . 0)  45. 0)  x  Internal  touch.d = a + 1  5  b -a 43.  F(ae.77  Two Dimensional Geometry  40. b ) (a. A  C 1 C 2  =  r 2  ±  r 1 Þ  (a  –  0) 2  +  (b  –  3) 2  =  |b ±  2| 2  42.ab  a + b y  2 ( a + b ) 2  .  (a.  r 2  are  radii  of  circles  then  2  h 2  .  Þ a 2 a 2b2 = ( b2 .  (a)  :  As the line passes through P and Q which are the point  Now the line y = ax + b  will be tangent to circle if both roots  of intersection of two circles. b ) æ b 2  ö 2  ç Q e  = 1 .  (0.a 2 y 2 =  a 2 b2  Þ  x 2  =  10y  –  5  and  x 2  =  2y  –  5  Þ b2 x 2 . C 2  are centres of the circles with radii r 1 . b). This  case  does  not exist  as  centre  of circle is (0.(*)  46. 4)  o  F  x  A (a . 5)  C  ¼ =   Area of  DBC ¼ }  {3 area  OAB (a.a2 ( ax + b ) 2 =  a 2 b2  Both  are  parabolas  but  x 2  =  2y  –  5  does  not  exist.a 2 a 2 )( -b2 a 2 - a 2 b2 )  =  5ax  +  (c  –  d)y  +  a  +  1  =  0.  (b)  :  F ¢(–ae. \  No value of a exist. b )  (0. b) = M (3.ae ( )  .  by using y = aa  + b 2  2  2 2  2 2  2  2  2  Þ x (b  –  a a  )  –  2a abx  +  (–b  a  –  a b )  =  0  .b2b2 + b2 a 2 a 2 .ab  \ tan45 ° = a+b Þ  3a 2  +  2ab  +  3b 2  =  0. r 2  respectively  then  (C 1 C 2 ) 2  =  r 1 2  +  r 2 2 Þ a 2  + b 2  =  p 2  +  (a  –  a) 2  +  (b  –  b) 2 Þ  p 2  + a 2  + b 2  – 2aa  – 2bb  =  0 Þ  2ax + 2by – (a 2  + b 2  + p 2 )  =  0.  b ) B  Case (i)  Now q =  45  {Q area  of  one  sector  =  3  time  the  area  of  another  sector}  q  D 2 ( a + b) 2  .  (c) : Now the equation of  line which  meet the x­axis and y­  axis  y  Þ  b 2  =  a 2 e 2 Þ  a 2  –  a 2 e 2  =  a 2 e 2  (0.ab  a +b (0.1  d – b  – c = +b/a has no solution..   so  \  equation  of  directrix  x  =  e a 2  +  b 2  +  2ga  +  2fb  +  4  =  0  a  Now  replacing  g.  (b)  :  Intercepts  made  by  the  lines  with  co­ordinate  axis  is  y  (–3b/a  ..  y¢)  be  point  of  locus  mid  point  of  PQ..  0)  and  Common  intercept  is  (0. y 3 )  æ x1 + x2 + x3 y1 + y2 + y3  ö . 2)  x 3  = 3  ×  2  – 1  =  5  G  y 3  = 2  ×  2  – 1  =  3 P(1.  –1)  Also  circumference  of  the circle  be  given  2pr  =  10p \  r =  5 \  Required  equation  of  circle  is  (x  –  1) 2  +  (y  +  1) 2  =  5 2  or  x 2  +  y 2  –  2x  +  2y  –  23  =  0  (0.. 0)  52..  (d)  :  The  equation  ax 2  +  2hxy  +  by 2  =  0  a 2  ´ 3  2  Further  b  =  by  (*)  =  (y  –  m 1 x)  (y  –  m 2 x) 4  1  2h  4 ´ 3  Þ  m 1  +  m 2  =  -  =  .  (–3b/a.33+ 5 . 0)  Þ = h. 37 ) . –3/2)  55.(*)  Þ  ç ÷ =  1  –  1/4  =  3/4  \  2g 1 g 2  +  2f 1  f 2  =  c 1 c 2  (where  (–g.  –3/2)  and  (3a/b..  1 \  A(0.  c  =  1/3  and a  =  1/2. 1 + 33 + 3 ) =  (1..  (2)  x  B    Solving  (1)  and  (2)  we  get  y  = C  A  x  = 0..  B(4a.2  a +  y 2  =  4  is  2  æ b ö x 2  +  y 2  +  2gx +  2fy  +  c  =  0  ...  (c) : As per  given condition  centre of  the  circle  is  the  point  of  intersection  of  the  2x  +  3y  +  1  = 0  and  3x  –  y  –  4  =  0 \  centre  is  (1.. k )  Þ  (2k) 2  =  8(2h  –  1) 2  2  Þ  y  =  2(2x  –  1) Þ y  –  4x  +  2  =  0..  (c)  :  The  point  of  intersection  of  parabola’s  y 2  =  4ax  and  x 2  =  4ay  are A(0.  1 \  y  =  0. y ¢ )  . 0).  –3/2).  (1)  and  line be  y  =  x  . y 2 )  47.  –3/2)  and (0.  0).  (b) :  Equation  of  directrix  x  =  4  which  is  parallel  to y­axis  x­axis so  axis  of  the  ellipse  is  x­axis. b ®  y \  (p –  x) 2  = 4q(y) which required locus of one end point of  x 2 y 2  +  = 1  (a  >  b)  the  diameter.  0).  y  =  –2  (1.  b  =  1/2.  (c)  :  Given  circle  x 2  +  y 2  –  2x  =  0  .. 1) and equation of circle in the diameter form  is (x  – 0)  (x  –  1) +  (y  – 0)  (y  – 1) = 0 Þ  x 2  +  y 2  –  (x +  y)  =  0  54.  q)  Þ  (p  – a) 2  =  4qb  51.  2 2  \  x¢  =  2h  –  1.  ÷ \  Centroid  G  =  ç 3 3  è ø = (1 ..  b).    (0..  y  respectively \  4  =  e \  2ax  +  2by  –  (a 2  +  b 2  +  4)  =  0  Þ  a =  2  (³  e  =  1/2)  57. B(1.  (b) : Let the equation of circle cuts orthogonally the circle x 2  Again e =  1/2 and  e 2  =  1 .  (c)  :  Equation  of  circle  AB  as  diameter  is  given  by  (x  –  p)  (x  – a)  +  (y –  q)  (y  – b)  =  0 Þ  x 2  +  y 2  –  x(p  + a) –  y  (q  + b) +  pa  +  qb  = 0 .(1)  Now  (1)  touches  axis  of  x  so  put  y  =  0  in  (1)  we  have  x 2  –  x(p + a)  +  pa  +  qb  =  0  .. 1)  M 2 (3. 0)  (–3a/b. (d) : \ x 2  = 2(– 1) – 1 = – 3  y 2  = 2  ×  2  – 1  =  3  M(–1.  f  by  x.(2)  and  D  =  0  in  equation  (2) B(a. 2)  R(x3.  c  =  1/4  Putting  these  value  in  the  given  equation  we  get  x +  2y  +  3 =  0  and    2x  +  3y  +  4  =  0  ..  y ¢ + 0  x ¢ + 1  P(1.  Let  equation  of  ellipse  be  Now a ®  x. = k .(i) .  b  =  1/3.78  JEE MAIN CHAPTERWISE EXPLORER  Q(x2.  –f)  are  point  of  locus) è a ø  Also  the  equation  of  one  directrix  is  x  =  4 Þ  c  =  – 4  a  Again  circle  (i) passes  through  (a.  a 2 b 2  2  b  56.  (d)  :  Let  M(x¢.(*)  b 2  =  =  3  4c  b 4 3  x 2 y 2  m 1 m 2  =  c  Hence  equation  of  ellipse  is  2 +  2  =  1 2  a b and    3x  +  4y  =  0 Þ  m 1  =  –3/4 2 2  x y 2  +  Þ  =  1  or  3x 2  +  4y 2  =  12  \  m 2  =  -  4 3  c 50. b)  \ (p  + a) 2  =  4[pa  +  qb] A(p.  Q(x¢.  49.  4a)  as  the  line 2bx  +  3cy  +  4d  =  0  passes  through  these  points \  d  =  0  and  2b(4a)  +  3c(4a)  =  0 Þ  2b  +  3c  =  0 Þ  (2b  +  3c) 2  +  d 2  =  0  53.  y¢)  lies  on  y 2  =  8x M( h.  48.  –2)  is  required  point  on  the  line.(*)  Solving  the  equations  of  (*)  we  have  x  =  1.  y¢  =  2k  Now  (x¢.  (d)  :  Let  us  take  the  two  set  of  values  of  a  = 1.   (c)  : According  to  the  problem  square  lies  above  x­axis  y –  y 1  =  -  (x  –  x 1 ) dy Now  equation  of  AB  using  two  point  form..  k)  by  (x.  (c)  :  Given  equations  are  2 3  - 2 1  x 2  –  2qxy  –  y 2  =  0  . b)  lies  on  2x +  3y  =  1 \  2a  +  3b  =  1  Joint equation of angle bisector of the line (i) and (ii) are same Now  centroid  of  ABC  is  x 2 .  Þ  2y  =  dt  2  dt Again pR 2  =  154.  (b)  :  Co­ordinate  of  centre  may  be  (1.(i)  x  4 3  O A(a.  y)  we  get  choice  (a)  is  correct.  61.79  Two Dimensional Geometry  Now  by  (*)  we  have  æ 3 2 ö 1  .b cos t + 0  B  (b.2 ö æ 2 + (-2) + h -3 + 1 + k ö \  =  .  y 2  81 9  = =  18 72 8  9  So  the  required  point  is  x  =  .(1)  (c)  :  Let  locus  of  point  C(h.sin a)  Þ  y  –  a  sin q =  (x  –  a(1  –  cos q))  (y  –  a  sin a)  =  [x  –  a  cos a] cos q  a (cos a + sin a )  which  passes through  (a...(ii)  Þ  =  1 a 1 + a By  squaring  (i)  and (ii)  then  adding  we  get  Þ  a  = ±  2  (as  a  =  2  b  =  –3  (3h  –  1) 2  +  (3k) 2  =  a 2 (cos 2 t  +  sin 2 t)  +  b 2 (cos 2 t  +  sin 2 t)  and  a  =  –2  b  =  1)  Replacing  (h. equidistant  from (a 1 .  0)  3  k  =  a  sin  t  –  b  cos  t  .  taking  ratio  of  their  coefficients 3  è 3 ø  1  -p Þ  2h  +  3k  =  9 Þ  2x +  3y  =  9  \  =  q  1  (c)  : The  equation  of  normal  at q  is  Þ  pq  =  –1  1  66.  (a)  :  Let  (h..  62.  \  h  =  (4. so  dy  18  dy  choices  (a)  and  (d)  are  out  of  court.  b 2 )  is  given  by  (a  –  a 1 ) 2  +  (b  –  b 1 ) 2  =  (a  –  a 2 ) 2  +  (b  –  b 2 ) 2 (a) : If m 1  and m 2  are slope of the lines then by given condition  Þ  a 1 2  +  b 1 2  –  2a 1a  –  2b 1b  –  a 2 2  –  b 2 2  +  2a 2a  +  2b 2b  =  0 m 1  +  m 2  =  4m 1 m 2 Þ  2(a 2  –  a 1 )a  +  2(b 2  –  b 1 )b  +  a 1 2  +  b 1 2  –  b 2 2  –  a 2 2  =  0 2 c 4  -  =  Þ  Þ  (a 2  –  a 1 )x  +  (b 2  –  b 1 )  y  +  7  7 1  Þ  c  =  2  (a  2  +  b 1 2  –  a 2 2  –  b 2 2 )  =  0 2  2  2  1  By  using  ax  +  2hxy  +  by  =  0 1  - 2h a  Þ  c  =  -  [a 1 2  +  b 1 2  –  a 2 2  –  b 2 2 ]  Þ  m 1  +  m 2  =  and  m 1 m 2  =  2 b b  64.  We  get  dx y  –  y 1  =  m(x  –  x 1 )  sin q a (cos a .  60..(2)  As  (a.  a +  b  =  –1 . –1 satisfies the given equations of diameter..y 2  xy  æ h k .  (d) : Let a.  R 2  =  49 \  R  =  7  .  k)  be  the  co­ordinate  of  centroid (d) :  Given  OA  +  OB  =  –1  y  a cos t + b sin t + 1  i.. b  is the point of locus.  k)  and  centroid  (a.  –1)  or  dt  dt (–1.ç + ÷ =  è 4  c ø  4c 1 2  1 8  3  3  + + -  =  Þ  Þ  -  =  4c c 4c 4 c 4 4 9  3  -  =  Þ  \  c  =  –3  4c 4 58..  x y x y + =  1  so  equation  are  -  =  1  and  65. 1 + 1  -q ç ÷ or  çè 3 3  ÷ø 3 3  è ø  Þ  qx 2  +  2xy  –  qy 2  =  0  . b)  x 2  –  2pxy  –  y 2  =  0  .e..  b 1 )  and  (a 2 .  3)  3  \  equation  of  the  line  be  a sin t .  0)  k  =  x y  3  =  1 a 1 + a Þ  3h  –  1  =  a  cos  t  +  b  sin  t  .  y  =  9/2  8  Þ  y  =  9/2 \  x  =  63. 1) but 1..  59.  0)  Þ y(cos a  +  sin a)  +  x(cos a  –  sin a)  = a sin a(cos a  + sin a) + a cos a  (cos a  – sin a)  dy  dx (d)  :  Given  y 2  =  18x  and  =  2  =  a(sin 2a  +  cos 2a)  dt  dt =  a(1)  dy  dx  \  2y  =  18  67.(3)  æ h ö 3( k - 2)  \  2 ç ÷ +  =  1 Now  (2)  and  (3)  are  same.   –1)  and  r 2  =t 2 = 3  Again  product  of  the  slope  of  AC  and  AB  1  =  × (–5)  =  –1 5  Þ  AC  b  AB Þ  right at  A  72. ç .  (a)  :  AB =  26 .0)  æ 2  ö = 4  ç + f ÷ 12 15  ´  è 4  ø  =  3 Again  foci  =  a 1 e 1  =  x 2  +  y 2  =  9  x 2  +  y 2  +  2gx  +  2fy  =  0  5 12 2 9 =  1 +  4f  Common  tangent  \  focus  of  hyperbola  is  (3.  5)  A  O  C(–1. 2 2  x y \ equation of common tangent is 2gx + 2fy –9 = 0 and distance  -  and  eccentricity  for  =  1  is  144 81  from the centre  of circle x 2  +  y 2  = 9 to the common  tangent  is  25 25  equal  to  the  radius  of the  circle x 2  +  y 2  =  9 2 2  ì a1 + b 1  0 + 0 .  2bt 1 ). k)  p  sin a =  2 k sin 2a  +  cos 2a  (4.  –1)  \  ABC is  isosceles  X  Þ  =  p2 4h 2 +  p 2  4 k 2  y  0.  74.80  JEE MAIN CHAPTERWISE EXPLORER  \  Required  equation  of  circle  be  (x –  1) 2  +  (y  +  1) 2  =  49 Þ x 2  +  y 2  –  2x  +  2y  =  47  68.  (c)  :  Given  median  of  the  equilateral  triangle  is  3a.  (d)  :  (x –  1) 2  +  (y  –  3) 2  =  r 2 \  C 1 (1.  AC =  –f  =  ±  2 26 Y  B(3.  (d)  :  Since  the  normal  at  (bt 1 2 .  2 h  \  b 2  9  =  1  –  e 2  =  1  –  or  16  16 Þ  b 2  =  7  71.  (b) :  Let  equation of  circle be x 2  +  y 2  +  2gx +  2fy  +  c =  0.9  ï e 1  = \  =  3 ï a 1 2  4 g 2 +  4 f 2  í 81 15  ï Þ 3 2  =  4(g 2  +  f 2 )  9  3  1 + = 3  ïî \ e1  = 144 12  1  (0.  0)  As  their  foci  are  same \  4e  =  3 \  e  =  3/4 f  =  ±  2 2  æ b ö b 2  =  1  –  ÷ è a ø  16  \  \  e 2  =  1  –  ç æ1 Centre of the required circle be  ç .  t1  As  it passes  through (0.1) 2 + (3 + 1)2  =  5  In D  LMD. 0)  so  –g  =  .  (LM) 2  =  (LD) 2  +  (MD) 2  Now  for  intersecting  circles  2  r 1  +  r 2  >  C 1 C 2  and  |r 1  –  r 2 |  <  C 1 C 2 LM ö 2  =  9a 2  +  æ (LM)  ç ÷ \ Þ r +  3 >  5 and  |r  –  3|  <  5 è 2  ø Þ  r  >  2  and  –5  <  r  –  3  <  5 3  Þ  r  >  2 and  –2  <  r  <  8 \  (LM) 2  =  12a 2  Þ (LM) 2  =  9a 2 4  Þ  r Π (2. 0 ÷ ç è cos  a ø =  1 x 2 +  1  y 2  =  4  p 2  x  .  2bt 2 ) L  2  =  (3a  –  R) 2  +  R  ç ÷ M  R \  t 1 x +  y  =  2bt 1  +  bt 1 3  passes  through  (bt 2 2 .  (b)  : Eccentricity  for  2 +  2  =  1  1  a b and as  it passes  (1.  3)  and  r 1  =  t 1  =  r  (x –  4) 2  +  1(y  +  1) 2  =  9 \  C 2 (4.  on  parabola  y 2  =  4bx  2  æ LM ö meet  the  parabola  again  at  (bt 2 2 .  so  C 1 C 2  =  (4 .  k)  is  ç ÷ 2 cos a 2 sin  aø è p  \  cos a  =  .  0)  =  (4e.0)  æ p ö .  (OM) 2  =  (OD) 2  +  (MD) 2  69.  0)  =  (ae.  2bt 2 ) è 2  ø O  Þ R 2  =  9a 2  +  R 2  –  6aR  +  3a 2 Þ  t 1 bt 2 2  +  2bt 2  =  2bt 1  +  bt 1 3 D  Þ 6aR  =  12a 2  Þ  t 1 (t 2 2  –  t 1 2 )  =  2(t 1  –  t 2 ) R  =  2a  N Þ  t 1 (t 2  +  t 1 )  =  –2 So  equation  of  circle  be  2  (x  –  0) 2  +  (y  –  0) 2  =  R 2  =  (2a) 2 Þ  t 2  +  t 1  =  Þ x 2  +  y 2  =  4a 2  t1  2  Þ  t 2  =  -  –  t 1  73. .  8)  Again in  triangle OMD.  0) so  c  =  0  x 2 y 2  70. è2 p p ö æ .  0)  2gx +  2fy =  9  \  f 2  =  2  so  focus  of  ellipse  (ae.  0)  ö æ 1  ö 2 ÷ .  p sin  a  Midpoint (h.2  ÷ ø è2 ø  (0.  (d)  : \  (h. 2  is b 2  =  a 2 (1 –e 2 )  Now x 2  + y 2  + 2gx + 2fy = 0 and x 2  + y 2  = 9 touches each other.  0) to the tangent line = Radius of circle î by1  + f = 0  a h g h b f g  f  c  On  compairing  the  equation  given  in  (*)  we  get  bg  =  fh and  bg 2  =  fgh  .(1)  Þ  m =  1  ±  2  and –1  ±  2  Now say point of intersection on y axis be (0..(1)  a  ¶s  ¶s  =  mx  +  = 0 = m  ¶ x  ¶ y and \  y 2  =  8ax  =  4(2a)x ¶s  \  Equation  of  tangent  to  parabola  \ = 0 Þ  ax  +  by  +  g  =  0 Þ  ¶ x  2a  ... b = 0  a +  b tan  45° =  ±  2 m 2  ... Þ  x 2 (1  –  m 2 ) +  2mxy  =  0  Now tan q  =  ± 77..  (a) : As s = ax 2  + 2hxy + by 2  + 2gx + 2fy + c = 0 represent a  pair  of  line ìï a = 1 ....(3)  Now  using  (2)  and  (3)  in  equation  (1)  we  have  1(1  –  m 2 )  \  Þ  Þ  Þ  2  a  =  ±  2 a  ´ m  1  (0 0)  1  + m 2  2  2  m  (1  +  m  )  –  2  =  0 (m 2  – 1)  (m 2  +  2)  =  0 m  =  ±  1  Required  equation  of  tangent  y  =  mx  +  = ±  (x  +  2a) y = mx  +  2a  m  2a  m .  (a) : Using fact : Pair of lines Ax 2  + 2hxy + By 2  = 0 are b to  each  other  if  A  +  B  =  0 Þ  3a +  a 2  –  2  =  0 Þ  a 2  +  3a  –  2  =  0 Þ  There  exist  two  value  of a  as  D  >  0  -3 ± 17  2  76. \  Equation  of  such  circle  be  (x – a) 2  +  (y – b) 2 = 3 2 Þ a 2  + b 2  –  2ax –  2by  +  25  =  9 Þ  x 2  +  y 2  –  2x 2  –  2y 2  +  25  =  9 Þ  x 2  +  y 2  =  25  –  9 Þ  x 2  +  y 2  =  16  and  x 2  +  y 2  =  25 Þ  4 £  x 2  +  y 2 £  64  \ a =  abc  +  2fgh  –  af 2  –  bg 2  –  ch 2  =  0 Þ  (abc  –  af 2 ) +  (fgh –  bg 2 ) +  fgh –  ch 2  =  0 Þ  0 +  0 +  fgh  –  ch 2  =  0 \  ch 2  =  fgh  Now adding  (2)  and  (4)  2fgh =  ch 2  +  bg 2  .81  Two Dimensional Geometry  75.  (b)  :  Let  common  tangent  to  the  curves  be  intersection of pair of line be obtained by solving the equations y  =  mx  +  c  . b) is  the centre  of  the  circle  whose radius is 3.  (c)  :  Equation  of  chord  y  =  mx  +  1  Equation  of  circle  x 2  +  y 2  =  1  Joint equation of the curve through the intersection of line and  circle  be  given  by x 2  +  y 2  =  (y  –  mx) 2 .m 2  2  h 2  .. y 1 ) and point of  79.(2)  2  2  Again ax  +  2hxy  +  by  +  2gx +  2fy  +  c =  0  meet  at  y­axis \  x  =  0 Þ  by 2  +  2fy  +  c  =  0  whose  roots  must  be  equal \  f 2  =  bc  af 2  =  abc  ..(2)  y =  mx  +  ¶s  m  and = 0 Þ  hx +  by  +  f  =  0 Þ  ¶ y  which  is  also  tangent  to  the  circle  2  x 2  +  y 2  =  2a 2  =  ( 2a )  ì hy1  + g  = 0  (*)  í Now Distance from (0.ab  where  í ïî h = m.0  1 -  m2  Þ  = ±  2m 2  =  0  \  Þ  m  ±  2m  – 1  =  0 Þ  m  =  ±  1  ±  2 or  abc +  2fgh  –  af 2  –  bg 2  –  ch 2  =  0  ..  (a) : Let (a.(4)  78.   If  the  straight  lines  x -1 y .  (a)  2/5  (c)  2/3  x . 3. then  the integer  k  is equal to  (a)  – 2  (b)  – 5  (c)  5  (d)  2  (2008)  .2 z . . 7)  (c)  (–5. 6. 6.  Statement­2 : The plane x – y + z = 5 bisects the line segment  joining A (3.  5 5 5 (b)  6 - 3 2  . 5)  (b)  (5.  If the angle between the line  x = y .  –3. Statement­2 is true; Statement 2 is not  a  correct  explanation  for  Statement­1.  If the lines  x . 4) in the plane x – y + z = 5.  (b)  5/3  (d)  3/2  (2011)  A line AB in three­dimensional space  makes angles  45°  and  120° with the positive x­axis and the positive y­axis respectively. 6) and  B(1. 3). –15)  (d)  (6.  Statement­2  is  true;  Statement­2  is  not  a  correct  explanation  for  Statement­1.  (a)  Statement­1  is true.  Statement­2  is  true.  (c)  Statement­1  is  true. 1. 6) is the mirror image of the  point B (1.5  = =  If the lines  x .1 z .  7 7 7 (c)  -6 - 3 2  .  Statement­2 is  false. b)  equals  (a)  (–6.1 y .  If  AB  makes  an  acute  angle q  with  the  positive  z­axis.  Statement­2 is  true.  Then  (a.2 y .  7 7 7 (d)  6.1  = = and  = =  k 2 3 3 k 2  intersect at a  point.2  = =  bisects  the  line  1 2 3 segment joining A(1.1 z .  (c)  –  1  (d)  2/9  (2012)  Statement­1 : The point A(1.  (d)  Statement­1  is  true.  5  7  9  3  (b)  (c)  (d)  (a)  2  2  2  2  (2013)  3.  coplanar.  (c)  Statement­1 is true.1 y + 1  z .  (b)  Statement­1  is false. . 0. –3.3 z .2 y .  then  k  can  have  (a)  exactly  three  values  (b)  any  value  (c)  exactly  one  value  (d)  exactly  two  values  (2013)  2.  (b)  Statement­1 is true. 0.1 z + 2  = = line    lie  in  the  plane  3 - 5 2  x  +  3y  – az  + b  =  0.  An equation of a  plane  parallel  to  the  plane  x – 2y + 2z – 5 = 0 and at a unit distance from the origin is  (a)  x  –  2y  +  2z  –  1  =  0  (b)  x  –  2y  +  2z  +  5  =  0  (c)  x  –  2y  +  2z  –  3  =  0  (d)  x  –  2y  +  2z  +  1  =  0  (2012)  4. 1. . 4).  (2010)  Statement­1 : The point A(3.  The  direction  cosines  of  the  vector  are  (a)  6 - 3 2  . 3) in the  line x  y .  then q  equals  (a)  30º  (b)  45º  (c)  60º  (d)  75º.k z = =  = =  intersect. 3.  2  respectively. 2  (2009)  11.4  and  are  1 1 2 k - k 1 7.4 z .3  y . 7) is the mirror image of the  point B(1.  (2011)  6.2  = =  1 2 3 Statement­2  :  The  line  : x  y .  and  2 3 4 1 2 1 then k  is equal to  (a)  9/2  (b)  0  5. Statement­2 is true; Statement­2 is a  correct  explanation  for  Statement­1.  The  projections  of  a  vector  on  the  three  coordinate  axis  are  6.  (d)  Statement­1  is  false. –17)  (2009)  10.82  JEE MAIN CHAPTERWISE EXPLORER  THREE DIMENSIONAL  GEOMETRY CHAPTER  13  1.  (2010)  Let  the  x .3 x .  (a)  Statement ­1 is  true.1 z .2 = y .  Distance  between  two  parallel  planes  2x  +  y  +  2z  =  8  and  4x + 2y  + 4z + 5 = 0  is  8.1  x .  7) and B(1.3  and the plane  = 2 l  æ 5  ö then l  equals  x + 2 y + 3 z = 4 is cos -1 ç è 14 ÷ø  9.3 = z . Statement­2 is  true;  Statement­2  is  a  correct  explanation  of  Statement  1.  Statement­2  is  false.   a)  p  p  p  p  (b)  2 (c)  6 (d)  3 .4  x .  Then  2 2 ø  14. (2005)  28. z = c ¢y +  d will be perpendicular.  then  cos 2q  equals  18.  The angle between  the lines  2x =  3y =  –z and  6x = –y  = –4z  is  (a)  90º  (b)  0º  (c)  30º  (d)  45º.  a.kˆ .  then  the  angle  that  the  line  intersection  are  given  by  makes with  the  positive direction  of  the z­axis is  (a)  (3a.  z  =  cy  +  d  and  x =  a ¢y +  b ¢.  –3)  (c)  x  –  y  –  z  =  1  (d)  2x  –  y  –  z  =  1.1 y .  (a..  Distance  between  two  parallel  planes  x  –  2y  =  0  is  2x +  y  +  2z  =  8  and  4x  +  2y +  4z  +  5 =  0  is  (a)  7/2  (b)  5/2  19  ö æ 17 (a)  ç . with each of the x and z axis. If the vectors  c  lies in the plane of  a and  x 2  +  y 2  +  z 2  –  10x  +  4y  –  2z  =  8  then  a  equals  r  b  .  is  such  that  è 3 ø  3 sin 2b  =  3sin 2q. . 9.  2a. If L makes an angle a  (b)  (a)  9  3 3  with the positive x­axis.  The  co­ordinates  of  each  of  the  points  of  of  each  of  x­axis  and  y­axis.  15.5  = = = =  and  1 1  - k k 2 1  are  coplanar  if  (a)  k  =  1  or  –1  (b)  k  =  0  or  –3  (c)  k  =  3  or  –3  (d)  k  =  0  or  –1.  z  =  cy  +  d  and  (a)  3/5  (b)  1/5  x =  a¢y  +  b¢.  The  two  lines  x  =  ay  +  b.  z  =  c¢y +  d¢  are  perpendicular to  each other  if  (c)  2/3  (d)  2/5.  (2007)  (2004)  (a)  a = 8.1  z .  a. b  = 2  (c)  a = 4. then x  equals  (a)  1  (b)  –1  (c)  2  (d)  –2.  The  distance  between  the  line  r  r = 2iˆ .  a). 3. 1 ÷ (d)  ç 5 .  2a)  (b)  (3a.  3a).2  = =  and  the  1 2 2  1  plane  2 x .  1. then the coordinates  of  one  of  the  sphere  and  the  plane  of  the  other  end  of  the  diameter  are  (a)  x  –  y  –  2z  =  1  (b)  x  –  2y  –  z  =  1  (a)  (4. 3..ˆ  j + 4 kˆ )  and  the  plane  (2008)  r  ˆ r × ( i + 5 ˆ  j + kˆ ) = 5  is  Let  L  be  the  line  of  intersection  of  the  planes  10  10  2x + 3y + z = 1 and x + 3y + 2z = 2.  The  two  lines  x  =  ay  +  b.  2  are  coplanar.  2a. –3)  (d)  (4.  (2004)  (a)  aa¢  +  cc¢  =  –1  (b)  aa¢  +  cc¢  =  1  a c  a c  + = -1  + = 1 .  (2007)  25.  2  meets  each  of  the  lines  x  =  y  +  a  =  z  and  x  +  a  =  2y  =  2z.  The  image  of  the  point  (–1. . 4 ÷ .  3).5 .j + 2 k and  spheres  x 2  +  y 2  +  z 2  +  6x  –  8y  –  2z  =  13  and  r  ˆ r  r  c = xi + ( x .  which  it  makes  with  y­axis.  16.  if and only  if .  (b)  5  3  (c)  - 4  3 (d)  3 .y + l z + 4 = 0  is  such  that  sin q =  .  3.. b =  4  17.  z  =  1  + ls  and  t  x =  .  (2006)  è ø  If  the  angle b.2 y .  (2004)  (c)  (4.  The  line  passing  through  the  points  (5.  b.  a.  a.  3.4  z .  the  value  3 of l  is  21.  3a).  If  the  angle q  between  the  line  x .  (a.  a)  (d)  (2a.  3a)..  y = 1 + t.  4)  (c)  3/2  (d)  9/2  (2004)  è 3 ø  3 19  ö 13  ö æ 17 æ9 27. b  = 6  13.  A line makes the same angle q.  The  x 2  +  y 2  +  z 2  +  7x  –  2y  –  z  =  13  and  If  (2.  If  the  plane  2ax  –  3ay  +  4az  +  6  =  0  passes  through  the  r  ˆ ˆ ˆ r ˆ ˆ  ˆ  midpoint  of  the  line  joining  the  centres  of  the  Let  a = i + j + k .  (d)  (2006)  ¢ ¢  a c a¢ c¢  19.  then  cos a  equals  10  3  .  (2005)  (a)  – 4  (b)  –2  (c)  0  (d)  1.  (2003)  30.  (2a. 5)  (b)  (4.  (a.  3  (a)  -  5 (b)  a = 2.  1)  æ 17 -13 ö .83  Three Dimensional Geometry  12. .  intersection  of  the  spheres  (2007)  24.  (c)  (d)  (2005)  1  1  1  3  10  .  3a).3  z .  (a)  4 (c)  (3a.  (c)  ç .2 ˆj + 3kˆ + l (iˆ . –3.  a)  and  (3. . with parameters s and t respectively. 4 ÷ (b)  (15.  4  (2005)  22.2) ˆ  j .  1.  4)  in  the  3  plane  26.  3a.  then l  equals  (a)  –1/2  (b)  –1  (c)  –2  (d)  0.  11.  If  the  straight  lines  x  = 1  +  s.  The  plane x  +  2y  –  z  =  4  cuts  the  sphere  x 2  +  y 2  +  z 2  –  x  +  z  –  2  =  0  in  a  circle  of  radius  (a)  1  (b)  3  (c)  2  (d)  2.  The  lines  (c)  x + 1 y .  (2004)  20.  3a. .  y  =  –3  – ls. b =  8  (d)  a = 6.  A  line  with  direction  cosines  proportional  to  If a line makes an angle of p /4 with the positive directions  2.  crosses the  yz­plane at the  point  è 0.  5)  is  one  end  of  a  diameter  of  the  sphere  x 2  + y 2  + z 2  – 3x + 3y + 4z = 8 is the same as the intersection  x 2  + y 2  + z 2  – 6x – 12y – 2z + 20 =  0. z = 2 – t. b = i .  (a)  1  (b)  (c)  (d)  2  2  3  (2007)  23.  (2005)  29.   0).  A  tetrahedron  has  vertices  at  O  (0.  36.  (c)  (a)  (c)  (b)  (a)  (b)  4. 2  (c)  4  (d)  1.84  JEE MAIN CHAPTERWISE EXPLORER  (a)  (b)  (c)  (d)  aa¢  +  bb¢  +  cc¢  =  0  (a +  a¢)  (b  +  b¢)  +  (c  +  c ¢)  =  0  aa¢  +  cc¢  + 1  =  0  aa¢  +  bb¢  +  cc¢  +  1  =  0.1 + 1 .  b ¢.1 .  a 2 b 2 c 2 a¢2 b¢2 c¢ 2  (2003)  33.  27. c .  0. 1.1 .  21.  23.  8.1  = 0  a 2 b 2 c 2 a¢2 b¢ 2 c¢ 2  (b)  1 .1  = 0  a 2 b 2 c 2 a¢2 b¢2 c¢ 2  (c)  1 + 1 + 1 .  7.  b.  If  a  1 + b 3  = 0  and  vectors  (1. 1. Then the angle between  the  faces  OAB  and  ABC  will  be  (a)  cos –1 (17/31)  (b)  30°  (c)  90°  (d)  cos –1 (19/35).  A (1.  (0.  2  (d)  2.1 .  18.  26.  9.  (b)  (b)  (b)  (a)  (a)  (d)  3. c 2 )  are  non­coplanar. 3) and C (–1.  (2003)  32. 1. b .  (2002)  (d)  Answer  Key  1. 1  (b)  1.  35.  11.  12. 1.  1.  29.  c ¢  from  the  1 + c3  origin.  14.  1.  31.  13.  Two  systems  of  rectangular  axes  have  the  same  origin.  then  the  product  abc  equals  (a)  –1  (b)  1  (c)  0  (d)  2.  0.  (c)  (d)  (a)  (d)  (c)  (b) .  The  d. a 2 ) .r. (1.  33.  24.  (d)  (c)  (c)  (a)  (b)  (a)  2.  20.1 + 1 + 1 .  0). a . 2).1 .  30.  (a)  (b)  (b)  (a)  (c)  (b)  5. B (2.1  = 0  a 2 b 2 c 2 a¢2 b¢2 c¢ 2  1 + 1 + 1 + 1 + 1 + 1  = 0.  If  b b 2 c c2 34.  32.  25. b 2 )  and  plane  cuts  them at  distance  a.  c  and  a ¢.  0)  x +  2y +  2z  +  7  =  0  is  which makes  an  angle p/4  with  plane x  +  y  =  3  are  (a)  2  (b)  3  (a)  1.  15.  (2003)  (c)  1.  17. 2. 1).  then  a a 2 1 + a 3  31.  19.  28.  The  radius  of  the  circle  in  which  the  sphere  x 2  +  y 2  +  z 2  +  2x  –  2y  –  4z  –  19  =  0  is  cut  by  the  plane  36.  (d)  (b)  (d)  (d)  (b)  (c)  6. 2.  The  shortest  distance  from  the  plane  12x  +  4y  +  3z  =  327  to  the  sphere  x 2  +  y 2  +  z 2  +  4x  –  2y  –  6z  =  155  is  3  (b)  13  (a)  11  4  (c)  39  (d)  26.  (2003)  (2003)  r r  r r  35.  of  normal  to  the  plane  through    (1.  10.  22. 1.  34.  (2003)  (a)  1 + 1 .  16.  r r  (1. 3  = 0  we have  the lines  as perpendicular  ö æ 5 + 3 l ö -1  ÷ = sin  ç ÷ ø è 14 5 + l 2  ø 2  So. 2. 3. 2. so statement  1 and  statement  2  are  true  but  statement  2  is  not  a  correct  explanation  of statement  1.3  y .  Again  (2.  2.  4x  + 2y  + 4z  = –5  Distance between planes =  16 .  x .1 .  (c) : Equation of  a  plane  parallel to  x – 2y + 2z – 5 = 0 and at a unit distance from origin  is x –  2y + 2z + k = 0 | k |  = 1 Þ | k |= 3  3 Þ  \ x – 2y + 2z – 3 = 0  or  x – 2y + 2z + 3  = 0  4.1  = = = r1  2 3 4 x . 6.  –2)  lies  on  the  plane x .5 ö = = = -2 ç ÷ø è 1 -1 1 3 Þ  a .(.1 = b .  As 1.3 c.0 + 6. 4r 1  + 1  = r 2 Þ 2r 1  – r 2  = 2. we  have  a-1 b. 4)  is  (2.  2 8. But it does not explain statement­1. we get 1(1 + 2k) + 1(1 + k 2 ) – 1(2 – k) = 0 Þ  k 2  + 1 + 2k + 1 – 2 + k = 0 Þ  k 2  + 3k  = 0 Þ  k(k  + 3) =  0 \  k =  0.4 æ 1 .0 y . .  Also the midpoint of AB  lies on the given line.  (c) :  1 2 l  x + 2y + 3z = 4  we  have  (3)(1)  +  (–5)(3)  +  2(–a)  =  0 Þ  –12 – 2a  =  0.1 = . 3r 1  – 1 = 2r 2  + k.1 z + 2  9.  Statement ‘2’ holds even if the line is not perpendicular. This  situation is possible.  (b) : The planes are  4x + 2y + 4z = 16.5) 2 2 4 + 2 +  4 2  = 21 7  = 6 2  3. 1.  (a) : The  line  is  = = 3 - 5 2  The  direction  ratios  of  the  line  are  (3.  As  the  line  lies  in  the  plane  x  +  3y  – az  + b  =  0.  1. 6)  Again the midpoint of A(3. 6) and B(1. so the plane bisects the segment  AB.  As the point lies on the plane.  3).10 + k Þ k = 10 =  2 2 2 5.1  1 1 .3 + 4 .  – 4). 7) and B(1.85  Three Dimensional Geometry  1. 6.3  and r2  = - 5  2 9 11 9  .k z = = = r2  1 2 1 or  2r 1  + 1 = r 2  + 3.  7).  c)  Thus by image formula.  (5 + 3 l )  + 5  = 1 (sin 2 q + cos 2 q = 1)  2  14(5 + l  ) Þ (5 + 3 l ) 2  5 + l 2  14  + 5 = 14  Þ  (5 + 3l) 2  + 5(5 + l 2 ) = 14(5 + l 2 ) Þ  25 + 30l + 9l 2  + 25 + 5l 2  = 70 + 14l 2 Þ  30l + 50 = 70 Þ  30l  = 20 \ l  = 2/3  7.1  z .  (b) :  Let the image be (a.4  = 2  1 -1 1 \  (a.  –5. 0.2 y .  (d) : For the  lines to  be  coplanar  Angle between line and  plane  (by definition)  æ = sin -1 ç 1 × 1 + 2 × 2 + l × 3 è 1 + 4 + 9 1 + 4 + l2 1 . so  n =  Þ c os q = 1 1  Þ n =  4 2 1  2 1  . 5) & the equation of the plane is x – y  + z  =5. m = -  2  As l 2  +  m 2  +  n 2  = 1.  (a) :  and  x .1 y + 1  z . \ q = 60° . –3  So there  are  two  values  of k.  3) is  (0.k =  0  k 2 1 Expanding.  2). 1.  (d) : The  direction  ratios  of  the  line  segment  joining  A(1. and 4r 1  – r 2  = – 1  – 2r 1  = 3  Þ r1 = \ - . b)  is  (–6.  b.2 – 4. c) = (3. b.3 = c . we have  n 2  = We take positive  values.3  = = 6. \ a  =  –6  Þ  2 +  3  +  2a  + b  =  0 Þ b  =  –2a  –  5  =  12 –  5 =  7  Hence  (a.  (c) : We have  l = 1 2 1  .  The  direction ratios  of  the  given  line  is (1.   5  . 1) and (5.  (b) :  a = i + j + k .y2 )2 + ( z1 -  z2 ) 2  62 + 32 +  2 2  = 36 + 9 + 4 =  7 uuur  The  direction  cosines  of  PQ are  = x1 .z 1  = =  a1 b1 c 1  x .  z¢)  in  ax +  by  +  cz  +  d  =  0 is  given  by  x .a = -9 Þ a = 6  2  \  9 -13  x= .e.  3 1  = .b ) = -3 \ b = 4  Þ  1(1 – 2x + 4) –1 (–1 – 2x) + 1 (x – 2 + x) = 0 Þ  5 – 2x + 1 + 2x + 2x – 2 = 0 Þ  x = – 2.  we  have  which on solving given  2k 2  +  5k –  25 =  0 Þ  2k 2  + 10k – 5k – 25  = 0 Þ  2k(k + 5) – 5(k  + 5) = 0 Þ  (2k – 5) (k + 5) = 0  \ k = - 5.  l = m = n 3 - 3 3  1 1 1 1  \ l:m:n = : .5 y .y2 z1 -  z 2  .  (d)  :  Image  of  point  (x¢.2) ˆ  j .(1 .  y¢. 1)  Let the other end  of diameter is (a.  (b) : As the  lines intersect.j + 2 k and  r  c = xiˆ + ( x .  17.x2 ) 2 + ( y1 .z ¢ -2( ax ¢ + by ¢ + cz ¢ + d )  = = = a b c  a 2 + b 2 + c 2  13  Again  z = m ( a .1  z .: Þ cos a = .  y1  –  y2.b a - 1  The line  crosses the  yz  plane where x  = 0.  (d) : The  equation  of  the  line  passing through  (3.kˆ  r r r  [a b c ] = 0  Then  cos a = 5  2  12.x2 y .  (b) : Let required angle is q  p p Q l = cos .a + = -  2 2  2 2 2  Þ  x + 1 y . we get.3 ˆ  j + 3 kˆ .6)  = = = 1 -2 0 5  3  Þ . .  L =  2 3 1  1 3 2  = 3iˆ .4  -2( -1 .  (b) : Let  the  vector  PQ  be  (x1  –  x2.  i. 6 3 2  .1) + a = -  2  5 13  3 5 13  Þ .b ) =  2 2  2 2  Þ (1 .  b = i .  9 + 9 + 9 3 Second  method  If direction cosines of L be  l.y 2 z .86  JEE MAIN CHAPTERWISE EXPLORER  uuur  10.x2 y1 . b. n.  15.y1 z . g)  b + 3  3 = a + 2  Þ  a = 4 .  7 7 7  1 1 1  1 -1 2 = 0  x x .5 = 2 m \ m = -  2 17  Again  y = m (1 .  z1  –  z2)  we  have  x1  –  x2  =  6  y1  –  y2  =  –3  z1  –  z2  =  2  Length  of  PQ  PQ = ( x1 .  (a) :  Fact  :  Two  lines  and  x ..a  = = = m (say)  2 1 .  (c) : Direction of  the  line.  2 16.y ¢ z .( a . a)  is  x .  6 = Þ  b = 9  2 2 g + 5  1= Þ g = -3. m = cos then n = cos q 4 4  We know that l 2  +  m 2  + n 2  = 1 p p cos2 + cos 2 + cos2  q = 1  Þ  4 4 Þ  1 1  + + cos 2  q = 1  2 2 Þ  cos 2 q  = 0 Þ q  = p/2  Thus required angle is p/2. 1. z = 4  5 5  iˆ ˆ  ˆ  j k  13.x1 x .x ¢ y .  18.3 z . 6.1) + a = -  Þ . then  2l + 3m +  n = 0. m.b ) + 1 =  2  5 17  5 15  Þ . b.2 -1  11.z 2  = =  a2 b2 c2  are ^  if  a 1 a 2  +  b 1  b 2  +  c 1 c 2  =  0 .  3 3 3 3  r  ˆ ˆ  ˆ  r  ˆ ˆ  ˆ  14.y = . -  .b ) + 1 =  Þ .  l + 3m + 2n = 0  After solving.e.  PQ PQ PQ i.  (c) : Centre of sphere º (3.(1 .  4.  –1/2  y  z x+a = =  2 1 1  Let P º  (r.  0. b.( -5)  12 + 52 + 1 2  10  ..  d = 2 2 2  6  1 + 2 + 1  (ii ) (iii)  (ii) and (iii) Þ  r – l = 2a  .  (a)  :  Let  x 1 . 1.2 j + 3 k ) × ( i + 5 j + k ) .  1)  and  (5.9  = 1.  6 6  + 2x 21.  –2.  CD :  20. –2. a).(ii)  2 3 + 2 + ( -6) 2 2 2 2 + ( -12) + ( -3) 2  = 0  Þ  10x  –  5y  –  5z  =  5 Þ  2x  –  y  –  z  =  1  y + a  z x = =  1 1 1  25. r – l  According to question  direction  ratios  of  PQ are (2.  1 y  1    r ..  (a)  :  d  = \ d = d = r r  a × n .(i)  M  (1... 1.    y  + z  =  2 r r  a × n1 = d1    )   z . z axis respectively  then  l 2  +  m 2  +  n 2  =  1 Þ  2l 2  +  m 2  =  1  or  2n 2  +  m 2  =  1 Þ  2  cos 2q  =  1  –  cos 2b  (a  = g  = q)  2  cos 2q  =  sin 2b Þ  2 cos 2q  =  3  sin 2q  (given  sin 2b  =  3  sin 2q) Þ  5  cos 2q  =  3  28.. 2  d = 2 . r – l  – a.  1)  1 1  5  + + 2  \ R =  4 4 2  d  = ^  distance  from  centre  to  the  plane  is  equal  to  \ r .d  = =  a 1  c .4  3  .  z 1  be  any  point  on  the  plane  \  Radius  of  the  circle  2x  +  y +  2z  – 8  =  0 =   Radius of sphere – perpendicular distance from centre of  sphere  to  plane \  2x 1  +  y 1  +  2z 1  –  8 =  0 2  ( )  æ ö = ç 5÷ .9 6 è 2 ø  = 15 .d ¢ = = a¢ 1  c ¢  As  lines  are  perpendicular \  a  a¢  + 1  +  c  c¢  =  0 Þ  a  a¢  +  c  c¢  =  –1  and  .  (d)  :  Centre  of spheres  are  (–3. 1)  using  mid  point  in  the  equation  2ax –  3ay  +  4az  + 6  =  0 Þ  2a  –  3a  +  4a  +  6  =  0 Þ  a  =  –2.  y 1 .  (d)  :  Equation  of  the  plane  of  intersection  of  two  spheres  S 1  =  0  =  S 2  is  given  by S 1  –  S 2  =  0 6 .  (b) : Angle between the line and plane  90– q  is same as the angle between the line  line q  and  normal  to  the  plane  a1 a2 + b1b2 + c1c 2  Plane  \ cos(90 . r – a.d  n (2 i .1 y + 3 z . r) and  Q = (2l  – a.(1)  (i) and (iii) Þ l  =  a  r = 3a. l. 2)  R  =  Radius  of  sphere  is  g 2 + f 2 + w 2  - c = C 2  (5. l  = a \  p º  (3a..  3 22. 1)  x .87  Three Dimensional Geometry  Given  lines  can  be  written  as  x .  (c)  :  From  the  given  lines  we  have  x .  (A)  1  l l  .2 l + a r -l-a r-l = =  2 1 2  (i ) 1 + 0 + 1  .  26.24 + 18  2 C 1  (–3.. a × n2 = d 2  ( x 1     8 Normal of  plane  + 4x   2 y  +    +   4z 5  =  0  2(2 x + y + 2 z . 3a) and Q º  (a.8) + 21  \  d  =  2 2 2  = 21 7  = 6 2  4 + 2 + 4  27.  19.  4..  (a)  :  From  given    line  y  x y z x z = = and  = = 3 2 -6 2 -12 - 3  a1a2 + b1b2 + c1c 2  cos q = 2 2  a1 + b12 + c12 a22 + b22 + c2  cos q = 23. y.  3 3  . g with x.b¢ y  z . l )  Direction ratios of PQ are r – 2l + a.  (a)  :  Centre  of  sphere  is  1/2.  (a) : If a line makes the angle a. 2a. a.  (b) : Given AB =  \ q  =  90º.b y  z . 1)  24.1  = = =  s  .q ) = 2  a12 + b12 + c12 a22 + b22 + c2  1  (1 ´ 2 + 2 ´ ( -1)) + 2  l Þ = 3  12 + 22 + 22 22 + 12  + l  5  Þ l =  .   we  get 1  a Þ  (1  +  abc)  1  b 1  c a 2  b 2  =  0 c 2  Þ  (1  +  abc)  =  0  by  using  (A)  \  Centre  of  sphere  is  (–2. b .. 3  12x + 4y + 3z – 327 = 0  aa¢  +  cc¢  + 1  =  0  r r  r r  r r  31..d  = =  and  a 1  c x .x1 a1 a2 y2 .  1.. c .j .  2)  PB ^  from  centre  to  the  plane  z2 -  z 1  c 1  =  0 c2  . a 2 ) .  two  lines  cos q  =  x ..(*) x­Intercept y ­Intercept z ­Intercept  x y z  + +  =  1  Þ  a b c So  the  distance  from  (0.5 j - 3 k 12 + 12  + 4 + 19 =  5  and  centre  (–1.  (c)  :  Given  lines  can  be  written  as  x . b 2 ) . (i .z 2  = =  are  coplanar  if  a2 b2 c2  x2 .  0.  (b)  :  The  radius  and  centre  of  sphere  x 2  +  y 2  +  z 2  +  2x  –  2y  –  4z  –  19  =  0  is  29.  (b)  :  Using  fact.1  1  = d 1  =  1 1 1 1 1 1  + 2 + 2 + 2 +  2  2 2 a b c a b c Þ . P  2  1  a a  1  b b 2  1  c c 2  A  \  a 2 a  + 1  b b 2 b 3  + 1  =  0  c c2 a now  ¹  0  3  c 3  + 1  On  solving.  (1.  35.  (A)  2 12 + 4 + 3 2  .1  z-2 =  =  1 2  - 2  As  lines  (A)  and  (B)  are  coplanar and  32.b ¢ y .  (1.  b..  3)  Required distance  is  .0 y . 0.88  JEE MAIN CHAPTERWISE EXPLORER  x .3k i .  0) to  this  plane  to  the  plane  (*)  is  given  by  0 + 0 + 0 .x1 y .(2) 2 + 12 + 32  + 155  =  26 –  13 = 13  units.j .  we  find  the  distance  from  the  centre  of  sphere  to  the  plane­Radius  of  sphere  k  =  0  or  k  =  –3  30.5 j .327  2 B .3k ) . c 2 )  are  non  coplanar. 1.  (d) :  Concept using  angle between  the  faces  is  equal  to  the  angle  between  their  normals.  c)  is x y z  + + = 1  .y2 z .y1 z .24 + 4 + 9 .  0..y1 b1 b2 1 -1 -1  1  - k  =  0 2 1  Þ  1 k Þ  (5i .(B)  =  5i  –  j  –  3k  and  vector  b  to  the  face  ABC  is  uuur uuur  AB ´ AC =  i  –5j  –  3k Þ  1 -4 -1  1  -l l =  0 1 2 - 2 (2l  –  2l)  +  4(–2  –l)  –1(2  + l)  =  0 Þ  5l  =  –10 \ l  =  –2  \  cos q  =  \  \  Let q  be  the  angle  between  the  faces  OAB  and  ABC 19 19  \q = cos -1 æ ö è 35 ø  35  33.z 1  = =  and  a1 b1 c1  x .1 + 2 + 4 + 7  =  4  1 + 2 2 + 2 2  Now  (AB) 2  =  AP 2  –  PB 2  =  25 –  16  =  9 \  AB  =  3  k 2  +  3k  =  0  34.0  z .  (c)  :  Now  equation  of  the  plane  through  (a.  1.x2 y .b y . uuur uuur  \  Vector  b  to  the  face  OAB  is  OA ´ OB . a .  0)  (0.0  z .  (a)  :  As  vectors  (1.3k )  5i .d  = = a¢ 1  c¢ \  Required  condition  of  perpendicularity  is  –2.  (b)  :  In  order  to  determine  the  shortest  distance  between  the  plane  and  sphere.  0)  (0.  c )  or (0.89  Three Dimensional Geometry  Similarly. 0)  or (0.  1.  b. 0)  \  Also angle between (*) and x + y + 0z = 3 is p/4 p  \  cos  =  4 a + a 2 a  = 2 2 2 2 2 1 +1 a + b + c 2 2 a 2 + c 2  Þ  2a 2  +  c 2  =  4a 2 Þ  c  =  ±  \  2  a DR’s  a.2 .  c 1  + 2 1 b¢ 2 + 1  2  c ¢  (0.  a.  a.  ±  2a \  Required  DR’s  are  1.2 .  1.(*) (by  using  (0. 0)  d = ?  k  y  x  y  z  + + = 1  a  b  c  Now  d 1  =  d 2  given  (as  origin  is  same) Þ  1  1 1 1  =  1  1 1 1  + + a 2 b 2 c 2  a ¢ 2 b ¢ 2 c ¢ 2  1 1 1 1 1 1  Þ  2 + 2 + 2 . 0)  (0.  2  match  with  choice  (b) .  0)  in  (*)) (0. 0. c¢ )  (a .  (b)  :  Let  DR’s  of  normal  to  plane  are  a. b . 0.b¢ .  1.  2  or  1.  c  i. 0.e.2  =  0  a b c a¢ b¢ c ¢  + +  a(x  –  1)  +  b(y) +  c(z)  =  0  Þ  a(0  – 1)  +  b(1)  +  c(0)  =  0  Þ  – a  +  b  =  0 Þ  a  =  b  .  d 2  =  1 a¢ z  36. 0)  or (a¢ .  b.  –  2  Hence  1..  1. 0. 0.. v .j .k .k^ and c = i^ .  33  ® ^ ^  Let  AB = 3 i + 4 k be  two  unit  vectors.  r  r r  r The  vectors  a and b are  not  perpendicular  and  c and d are  r r r r r r (a) p  (b)  0  (c)  p  4 (d)  two vectors satisfying : b ´ c = b ´ d and  a × d = 0.j^ + 2 k^  (a)  .  –3)  r r r r (c)  (–2.  Then the angle  between  a and c  is  (2011)  with the altitude directed from the vertex B  to the  side AD.[ pv w qu ] .  q)  If  a = çè 3 i + k ÷ø  and  b = 7 çè 2 i + 3 j .  –2)  (2010)  r r 3( p × q ) r r r æ p×q ö r (b)  r = -3 q + r r p (a) r = q .  q)  r r r é r r r (d)  exactly  one  value  of  (p.j^ + 2 k^  (c)  i^ .  then the angle between  is  (a)  p  3 (b)  p  4 (c)  p  6 (d)  8.  r r æ a×c ö r a × b r r r æ a×c ö r (d) c + ç r r ÷ b è a × b ø r (b) c . b = 2 i + 4 j + 4 k and  c = l i + j + m k are  mutually orthogonal.q + ç r r ÷ p (2012)  r r r r r r r r r  ( p × p) [ 3u pv pw ] .  (b)  2 i^ . m)  =  (a)  (–3.  If r is  the  vector  that  coincides  r  (d)  i^ + j^ - 2 k^  ^ ^ ^ r ^ ^ ^  (2010)  r  ^ ^ ^ If the vectors  a = i .  If  u . b  = 1  (c) a = 1.  r (2010)  r  Let  a = j^ .  p  2 (2012)  3.j^ .  The vector  a = a i^  + 2  j + b k lies in the plane of the vectors  r  ^  ^  r  ^  ^  b = i + j and  c = j + k and  bisects  the  angle  between  r r  b and c  . then the length of the median through  A is  (a)  (b)  45  (c)  18  (d)  72  7. b = 2 i + 4 j + 4 k and  c = l i + j + m k are  mutually orthogonal.6 k ÷ø .ç r r ÷ c (2011)  è a × b ø r  r ^ ^ r  ^ ^ ^  Let  a = j .  ^ ® ^  ^ ^ ^  sides of a triangle ABC. b  = 2  (b) a = 2.  then  the  value  10 (c)  all  values  of  (p.  3)  (d)  (3.  r r r r then the  equality r r 3( p × q ) r r r æ p×qö r (c)  r = 3 q .  uuur r uuuur r Let ABCD  be a parallelogram such that  AB = q . –2)  (2013)  2.2 j + 4 k are  the  9.90  JEE MAIN CHAPTERWISE EXPLORER  CHAPTER  VECTOR ALGEBRA 14  ® 1. Then the vector b satisfying  r r r r r  a ´ b + c = 0 and a × b = 3 is p  2 (2008)  ^  ^  r  12.i^ + j^ - 2 k^  (b)  2 i^ . AD =  p and r  ÐBAD  be  an  acute  angle. –3)  (c)  (–2.k and c = i .  q)  r 1 æ ^ r 1  æ ^ ^ ö ^ ^ ö (b)  more  than  two  but  not  all  values  of  (p  .  then (l.  c = a^  + 2 b and  d = 5 a^  + 4 b are  perpendicular  to  each  other.j + 2 k .i^ + j^ - 2 k^  (c)  i^ .r r p (d) r = . b  = 2  (d) a = 2.  q)  (2009)  of ( 2 a .  5. w are non­coplanar vectors and p. Then which one of the following gives possible values  of a and b?  (a) a = 1.  2)  (b)  (2. Then the  r  vector d is  equal  to  r r r æ b ×c ö r (a) b + ç r r ÷ c è ø 6.  Then  the  vector    satisfying  r r r r  r a ´ b + c = 0 and a × b = 3 is (a)  .[ 2w qv qu ] = 0 holds  for  è p × p ø  (a)  exactly  two  values  of  (p.j^ - 2 k^  If the  vectors  AB = 3 i + 4 k and  AC = 5 i .k^  . 3)  (d)  (3.  If  the  vectors  r r ^  ^  .  The  non­zero  vectors  a . b  = 1  (2008)  .b ) × ë( a ´ b ) ´ ( a + 2b ) ùû is  r r r  11.ç r r ÷ p r r r  ( p × p) è p × p ø  10. m)  =  (a)  (–3. b  and c  are  related  by  (a)  5  (b)  3  (c)  –5  (d)  –3  r r r r  r r  a = 8b  and c = -7b .ç r r ÷ b è ø a × b r r æ b × cr ö r (c) b . q are real numbers.j^ - 2 k^  (d)  i^ + j^ - 2 k^  (2010)  r  ^ ^ ^ r ^ ^ ^  r  ^ ^ ^ If the vectors  a = i . 2)  (b)  (2.  then (l.  r  then r is given  by  4.j + 2 k .  C  with  position  (a)  14  (b)  7  (c)  2  (d)  14.  The  values of  a.k .  (a)  a  (b)  3a  r 2  r 2  (2003)  (c)  4a  (d)  2 a  .v ) ´ (v .  For  any vector  a . b = xiˆ + ˆ  j + (1 .3 ˆ  j - 5 kˆ  and  aiˆ .  If  a .  w  (d)  more  than two  values  of q  (2007)  r r r  are  perpendicular  to  each  other  then  u . .  w  be  such  that  u = 1. b .  r r  r r  r r r r  r r r r  c = yiˆ + xjˆ + (1 + x .  a . If the vectors  of  these are collinear.  then  r r r r r r r  r r r 1  r r  r (u + v .  (2006)  then the vectors  a + 2b + 3c .w) × (u . The  length  of  the  median  through  A  is  is  equal  to  r 2  r 2  (a)  72  (b)  33  (c)  288  (d)  18.  (d)  equal  to  zero.  Let  u .  v = iˆ .  for  which  the  points  A.  Let  a . iˆ . v  and  w  are  three  non­coplanar  vectors.  (2005)  (2003)  uuur  uuur  r  r ˆ2 r ˆ  2 r  ˆ  2  19.  (2005)  r  r  r  ˆ is  a  unit  j and  w = iˆ + 2 ˆ  j + 3kˆ . b and c be distinct non­negative numbers. b × c ¹ 0.  then  a × b + b × c + c × a is  equal to  (a)  only x  (b)  only y  (a)  –7  (b)  7  (c)  1  (d)  0. b and  c  be  non­zero  vectors  such  that  29.  28.x ) k and  26. c  are  three  vectors.  c  are non­coplanar vectors and l is  a real number.  c  are  non­coplanar  vector  and l is  a  real  number  3iˆ + ˆ  j -  kˆ  which  displace  it  from  a  point  iˆ + 2 ˆj + 3 kˆ  to  r r r r r r r r  then  éë l ( a + b ) l 2 b lc ùû = éë a b + c b ùû for  j +  kˆ .  j + kˆ  respectively are  vectors  2iˆ .  If  C  is  the  mid  point  of  AB  and  P is  any  point  outside AB.  If  a .  Let  a = i . then  2uˆ ´ 3 vˆ  is  a  unit  vector  for  1  2 2  (a)  no  value  of q  .3 ˆ  (2004)  the  vertices  of  a  right­angled  triangle  at  c  are  r r r  23. b . l b + 4 c and  (2l .  If  u  ˆ  and  v  ˆ  are unit vectors and q is the acute angle between  (a)  (b)  3  3  them.  (b)  exactly  one  value  of l  (2004)  (c)  exactly  two  values  of l  r r r  (d)  exactly  three  values  of l.  Then  éë a .  (2005)  (2004)  r  ˆ ˆ r r r ˆ  r r  r r  18.ˆ  r  r  r  then  vector  such that  u × nˆ = 0  and  v × nˆ = 0 .  Let  a . v .  Let a.  then  | w × nˆ |  is  equal  uur uur uuur  uur uur uuur r  to  (a)  PA + PB + PC = 0  (b)  PA + PB + 2 PC = 0  uur uur uuur  uur uur uuur  (a)  1  (b)  2  (c)  3  (d)  0.  (2005)  25.  the  value  of  ( a ´ i ) + ( a ´ j ) + ( a ´ k )  27.  (2006)  24.  The  work done  in standard  units  by  the  point  5iˆ + 4 ˆ  (a)  no  value  of l  the  forces  is  given  by  (a)  25  (b)  30  (c)  40  (d)  15.  B.  (2003) .  iˆ + kˆ  and  ciˆ + cjˆ + bkˆ  lie  in  a  plane.  such  that  a + b + c = 0.  A particle is acted upon by constant forces  4iˆ + ˆ  j -  3 kˆ  and  r r r  16.  where  a .  w = 3.  (c)  (2004)  (d)  3  3  (b)  exactly  one  value  of q  r r  r  r r r  22.1)c are non­  r r r r r r  r r r  coplanar  for  15.  The vectors  AB = 3iˆ + 4 kˆ  and  AC = 5iˆ .  (a)  2  and  1  (b)  –2  and  –1  r r r r r  r  (c)  –2 and  1  (d)  2  and  –1.  If  the  (c)  exactly two  values  of q  r  r  r  r  r r  projection  v  along  u  is equal to that of  w  along  u  and  v . b .  If  u . v = 2.  (c)  PA + PB = PC (d)  PA + PB = 2 PC (2005)  (2003)  r r r  r r r  21.ˆj + kˆ. If the vector  a + 2 b is  collinear with  r r  aiˆ + ajˆ + ckˆ .y ) kˆ .  then  sinq  equals  (c)  3u × u ´ w (d)  0.  (2004)  (c)  perpendicular  (d)  parallel. b .  If q  is the acute angle between  the  3  (a)  ur × vr ´ wr  (b)  ur × wr ´ vr  r r  r r  vectors  b and c . b   and  c  be  three  non­zero  vectors  such  that  no  two  r r  17.  then  c  is  r  r  c  and  b + 3 c is  collinear  with  a (l  being  some  non­zero  (a)  the  arithmetic  mean  of  a  and  b  r r r  scalar)  then  a + 2 b + 6 c equals  (b)  the  geometric  mean  of  a  and  b  r  r  r  (c)  the  harmonic  mean  of  a  and  b  (a)  l c (b)  l b (c)  l a (d)  0.  then  a and c  are  (b)  all  except  one  value  of l  (a)  inclined  at  an  angle  of p/3  between  them  (c)  all  values  of l  (b)  inclined  at  an  angle  of p/6  between  them  (d)  no  value  of l. | c | = 3.  If  n  20.v + w equals  14.  (c)  neither  x  nor  y  (d)  both x  and  y. b  and c  are  any  three  r r r r  r r  (a)  all  except  two  values  of l  vectors  such  that  a × b ¹ 0.  Let  u = iˆ + ˆj.2 ˆ  j + 4 kˆ  are the sides  of  a  triangle  ABC.  c ùû depends  on  | b | = 2.  If  ( a ´ b ) ´ c = a ´ (b ´ c ) .w)  equals  ( a ´ b ) ´ c = b c  a .Vector Algebra  91  2  2  13.   9. b = 2  and the angle between  a  and  b  is  6 then r 2  ( ar ´ b )  is  equal  to  (a)  48  (b)  16  r  (c)  a  (d)  none  of  these  (2002)  Answer  Key  1.  19.ˆ  j + 5 kˆ  respectively.  (2002)  r r r r  r r  36.  iˆ .  22.  If  a.  (d)  (a)  (b)  (d)  25.  (2002)  r r r  r r r  35.  8.  (a)  (d)  (a)  (d)  26.  17.  (d)  31.  (b)  (a)  (b)  (a)  29.  If  a.  Consider A. c = 3  then angle between vector  b  and  c  is  (a)  60º  (b)  30º  (c)  45º  (d)  90º.  7.  (a)  (a)  (c)  (c)  30.  (*)  36.  3l c + 2 m ( a ´ b ) = 0  then  (a)  3l  +  2m = 0  (b)  3l =  2m  (c) l = m  (d) l + m  =  0.  -iˆ .5 ˆ  j and  b = 6iˆ + 3 ˆ  j are two vectors and  c  is a vector  r r  r r  r r such  that c = a ´ b  then  a : b : c  = (a)  34 : 45 : (c)  34  :  39  :  45  39  (b)  34 : 45 : 39  (d)  39  :  35  :  34.  (b)  33.  given  that  a + b + c = 0  (a)  25  (b)  50  (c)  –25  (d)  –50. b .  (c)  34.  c  are  vectors  such  that [ a b c ] = 4  then r r [ar ´ b b ´ cr cr ´ ar ] = (a)  16  (b)  64  (c)  4  (d)  8.  (a)  35.  13.  (b)  3. Then  ABCD  is a  (a)  rhombus  (b)  rectangle  (c)  parallelogram  but not  a  rhombus  (d)  square.  If a ´ b = b ´ c = c ´ a  then a + b + c  = (a)  abc  (b)  –1  (c)  0  (2003)  (d)  2.  (a)  32.92  JEE MAIN CHAPTERWISE EXPLORER  30.  (c)  (a)  (a)  (a)  28.  (b)  4.  a = 3i .  C  and  D  with  position  vectors  7iˆ .  20.3 ˆ  j + 4 kˆ  and  5iˆ .  (a)  .  12.  10.  (c)  37. b = 5.  (b) 2. | c | = 3  thus  what  will  be  the  value  of  r r r r r r r  r r  a × b + b × c + c × a . b .  15.  c  are  vectors  show  that  a + b + c = 0  and  r r  r r  r  a = 7.  (d)  (a)  (d)  (d)  27.  r r  r 33.  24.  21.  B. | b | = 4.6 ˆ  j + 10kˆ .  14.4 ˆ  j + 7kˆ .  r r r r r r r  r r  31.  (a)  5.  (2002)  (2002)  r r r  34.  (2002)  r  r  ˆ r  32.  (a)  6.  23.  16.  If  a = 4.  If  | a | = 5.  (2002)  r r  r  p  r  37.  11.  18.  m.r  r + c ( a . m  = 2.pq[u v w] + 2 q 2 [u v w] = 0  r r r  Þ (3 p 2 .k = 0 r  ^ ^ ^  \ b = .4b$ ) = 5 .b )d rr r r r r r r r  Þ ( a.b )d = -( a.  r r  Thus  a and c  are antiparallel. 4) + (5.c )b + ( a.^j + 2 k^  .  b = 2 i^ + 4 j^ + 4 k^  .j - k r  Thus.  rrr r rr r r  r Also [ a b c ] = [b c a ] = [c a b ] (cyclic)  rrr r r  r And [a b c ] = -[ a c b ]  ( Interchange  of any  two  vectors)  r r r r r r r r r  [3 u pv pw] .  r 9.2b .b × b = - 5 r  r (b) :  a × b ¹ 0 (given) r  r a×d = 0 r r r r r r r r  r r Now.3 = 0 Þ b$ ∙ a$  = \q = 2 3 r  uuur uuuur (d) : r = BA + AQ r uuur uuuur  = .k ) = -2 i .k^ ) .b ) r (a) : We have  ar ´ b + cr = 0 Multiplying vectorially  with ar . -  1. v .q + projection of BA across AD rr r r ( p∙q ) p = .( a × a)b + a ´ c = 0 r r  ^ ^ ^ ^ ^  ^ ^ ^  a ´ c = ( j .q + r r  ( p∙p ) r r r r r r (c) :  (2a .j + 2 k . 4)  2 uuuur  \ AM = 42 + 12 + 4 2  = 33 r 2. 4)}  2 1  = (8.( a × b )a + 2( a × b )b .  (a) :  ar  = 8 b r r  c = -7 b r r r  r  a and b  are parallel and  b and c  are antiparallel.2 i .  w are non­coplanar.4 b$  r  r \ c ∙d = 0 Þ ( a$ + 2b$ ) ∙ (5 a$ .  (a) : We have ar ´ b + cr = 0 Multiplying vectorially with .4 a × a .j .  3( j^.  (a) :  a = i .d )b .  q) º  (0.  (a) :  ar = i^ .  1  p Þ 6 b$ ∙a$ . m  = 2.  4.( a.( a.k^ = 0 r  ^ ^ ^  \ b = .  we  have  r r r r r  a ´ (a ´ b ) + a ´ c = 0 r r r r r r r r  Þ ( a × b )a .  .2.2 i^ .  b = 2 i + 4 j + 4 k .  Thus  a = l ( b + c )  ^  ö ^  ö ^  ^  æ^ ^ æ^ ^  ^ ^  a i + 2  j + b k = l ç i + j + j + k ÷ = l ç i + 2  j + k ÷ è 2 è 2  ø  2  ø .b )c = ( a.i+ j.2b . 0) is the  only possibility.  (a) :  a  lies  in the  plane  of  b and c . 0.  (a) :  cr = a$ + 2b$ .k ) .j - k r  ^ ^ ^  ^ ^ Thus.  5.4b$ ∙a$ + 10b$ ∙a$ - 8 3.  (d) :  AM = 1 ( AB + AC )  2 1  = {(3.  6.k ) ´ ( i . roots are real Þ  q 2  – 24q 2 ³  0 Þ  –23q 2 ³  0 Þ  q 2 £  0 Þ  q =  0  And thus p  = 0  Thus (p. .  Also  a  bisects  the  r r r  r  r angle  b and c .j . d = 5 a$ .  (d) : We have  [la mb nc ] = lmn[a b c ]  for scalars  l.2k r  10.k ) = -2 i .  p. -  2.  3( j .  r r  r  r  12. 8) = (4.b ) × {( a × a ) b .( a × a)b + a ´ c = 0 r r  ^ ^ ^ ^ ^  ^ ^ ^  a ´ c = ( j .2(b × b )a } r r r r r r r r = (2a .k ) ´ ( i .b )c r r r r ( a × c ) b r Þ d = .j .  r r  Hence the angle between  a and c  is p.  r  ^ ^ ^ c = li + j+mk r r r r  a and c  are orthogonal    Þ  a × c = 0 giving l  –  1 +  2m  =  0  r r Also  b and c  are orthogonal Þ  2l  + 4 + 4m  =  0  Solving the  equation we get l = –3.pq + 2 q 2 )[u v w] = 0  r r r  r r r  As  u .  r 11.  r r r r r r  8.[2w qv qu ] = 0  r r r r r r r r r  Þ 3 p 2 [u v w] .c )b . q Π R  As a quadratic  in p.i+ j.93  Vector Algebra  uuuur uuur uuur  1.b ) × (b - 2a) = .^j . [u v w] ¹ 0  Hence 3p 2  –  pq  + 2q 2  = 0.b ) × {( a ´ b ) ´ a + 2( a ´ b ) ´ b } r r r r r r r r r r r r r r = (2 a .b ) × {( a ´ b ) ´ ( a + 2b )} r r r r r r r r = (2 a . we  have  r r r r r  a ´ (a ´ b ) + a ´ c = 0 r r r r r r r r  Þ ( a × b )a .  b ´ c = b ´ d Þ a ´ (b ´ c ) = a ´ (b ´ d ) rr r rr r r r r  r r Þ ( a.  n.  cr = l i^ + j^ + m k^ r r r r  a and c  are orthogonal    Þ  a × c = 0 giving l  – 1 + 2m  = 0  r r Also  b and c  are orthogonal Þ  2l  + 4 + 4m  =  0  Solving the  equation we get l = –3.[ pv w qu ] .2k r  r  ^ ^ ^  ^ ^ ^  7.   =  u 2 + v 2 + w2  .94  JEE MAIN CHAPTERWISE EXPLORER  on comparison.( b × c ) a =  1  r r r b c a  3  1  r r b c  Þ  cos q =  –1/3  3  8 2 2  16.  (d): As  a + 2 b is  collinear  with  c  r r  r  \  a + 2 b =  Pc  r r r r  r r  and b + 3 c is collinear  with  a \ b + 3 c = Qa .  a1 a2 a 3  r r r  18.  (d)  :  Let  P is  origin  uur r  uur r Let  PA = a . b .x  y x 1 + x -  y 1 0 0  = x  1 1 y C 3 ®  C 3  +  C 1  = 1(1) = 1  x 1 +  x which  is  independent  of  x  and  y.6 c =  Pc .  (d)  :  Let  a = a1i + b1 j + c1 k r  r  r \ a 2 = a12 + b12 + c1 2  \ a ´ i = . l 2  = a × b are scalar quantities)  r  r  Þ  a  ||  c  r  19.  (b) : We are given that points lies in the same plane.  M.  (a)  :  Using  the  condition  of  coplanarity  of  three  vectors a a c  r r  Þ 1 0 1 = 0  Þ c = ab  L × ( M ´ N ) = 0  c c b \  C  is  G.  (a)  :  From  given  =  sin q  =  9 3 r r r r r r  r 2  r r r r r  l ( a + b ) × ( lb ´ lc ) = a × ( b + c ) ´ b r r  v × u u × w  r r r r 22.b1k + c1 j r  r \ ( a ´ i ) 2 = b12 + c1 2  r  Similarly  ( a ´ ˆ  j ) 2 = a12 + c1 2  r  ( a ´ kˆ ) 2 = a12 + b1 2  r r r  \ ( a ´ iˆ )2 + ( a ´ ˆ  j )2 + ( a ´ kˆ )2 = 2( a12 + b12 + c1 2 )  r  = 2 a 2 .3 ˆ  j + kˆ )  C  uur  where  CA = (2 .ˆ  j + kˆ )  ( ai ˆ .  2  r r r  24.  c2 c3  r r  Þ  -b × c =  1 2 \  0 l 0 0 3  4  =  0 2l - 1 1  Þ l  =  0.2v × w + 2u × w  17.2u × v .  l = 2a .  (d)  : Þ ( ar ´ b ) ´ cr = 1 3  b r r r P(0)  ® A(a )  ®  ® C a + b  2 ®  B( b ) r  c a (As  given) r r r  ( a × c ) b .d1  =  (5 –  1)i  r  + (4  r –  2)j  +  (k  –  3k) \  W.v + w r r r r r r r r r  Þ  No  value  of l  exist  on  real  axis.  (c)  : [ a .  v  =  2.  (b) :  | 2uˆ ´ 3vˆ |= 1 Þ  6 | uˆ || vˆ || sin q |= 1  Þ  sin q = 6 2uˆ ´ 3 vˆ  is  a unit vector  for exactly  one  value of q.  (a):  Given  = and  v × w =  0  r r r r  r u u Þ l 2 a × (lb + lc ) + l 2b × (lb ´ c ) = a × (c ´ b )  r  r  r  Þ l 4 [ a b c ] = .a)i - 6k  Þ  a 2  –  3a  +  2  =  0 Þ  (a  –  2)(a  –  1)  =  0 Þ  a  =  1.  .  w  =  3  r r r  2  D  <  0 Þ ( l 2 )2  + 1 =  0 Now  u .  (d)  :  Given  ( a ´ b ) ´ c = a ´ ( b ´ c )  r r r r r  r r r Þ  Þ  l1a = l 2 c ( b × c ) a = ( a × b ) c r r r r ( l1 = b × c . l = 2 and  l = 2 b  Thus a = 1 and b = 1  1  13.  N  are  coplanar  if  23.  (c) : Total force  F = F1 + F2  = 7i + 2j – 4k and displacement  r r r  d = d 2 .  2  r r r r r r 15.  of  a  and  b.M.  20.  (a)  :  Now  CA × CB = 0  A  B  ( i ˆ .  uuur uuur  14..2 Qa .[ a b c ] Þ l 4  + 1 =  0  Also  u  =  1.  è 2  ø r 21.3 ˆ  j + kˆ )  (2 i ˆ .. We know  =  1 +  4  +  9  +  0  =  14  that  the  vector  L. c ]  = a × ( b ´ c )  = b1 b2 b 3  c1 1 0 -1  = x 1 1 .(i)  Now  by  (i)  and  (ii)  we  have  r r  r r  a .a)iˆ - 2 ˆ  j  uur ˆ  ˆ  and  CB = (1 .  PB = b r r uuur  \ PC = a + b  2  uur uur r  r Now  PA + PB = a + b r uuur  æ ar + b  ö = 2ç ÷ = 2 PC .D  = F × d =  28  +  4 +  8  =  40  r  r r 25. u .[(k . a éë ( a ´ b ) .OA = 6iˆ + 2 ˆ  j .  ( b ´ c ) . ( v ´ w ) + v . (v ´ w )  r r r r r r r r r  .25 . n  = |–3|  = 3  r r r r r r r r r  29. (a ´ b ) ùû =  é a .9 1  = = 2 b c 2 ´ 5´ 3 2  \ q  =  60°  v  u r r r r r 36.u .0  = r r r r r  r r r r  2  =  éë a . (u ´ v ) .  uuu r uuu r  1  uuur  Þ  AD  =  [ AB + AC ]  2  1  =  [8i  –  2j  +  8k] 34. (u ´ w) + v . ( v ´ w )  r r r  =  u .a ´ a r r r  r  Þ  a ´ br =  c ´ a r r r  b ´rc =  c ´ a Similarly  r r r  r  r  \ a ´ b =  b ´ c =  c ´ a r r r i j k  r r r  (b)  :  Given  c  =  a ´ b = 3 -5 0  6 3 0  r  r  c  =  39 k  \  r r  Now  a = 34. c + c . ( v ´ w ) + u .  (b)  :  Using  fact:  ( ar  ´ b ) 2  =  a 2 b 2  –  ( a .w . (u ´ w) + u .c 2  49 . (u ´ v )  r r r r r r r r r  =  u . (v ´ w )  r r r r r r r r + v .(k . ( v ´ w ) .  (*)  :  AB = OB .w . a )  r r r r r r  ( a 2 + b 2 + c 2 )  2  32.  (a)  :  a + b + c =  0  Consider   (a +  b  +  c) 2 r r r r r r = a 2  +  b 2  +  c 2  +  2 (a .  (b) : Median through any vertex divide the opposite side into  two  equal  parts  uuur uuur uuur  AB + AC =  2AD  33.u ´ w + v ´ w ]  r r Q  v ´ v = 0  r r r r r r r r r u . c + c .  (c)  :  n  r r u ´ v  ˆ  =  r r  now  n  u v |  =  1 1  ˆ )  ´ ( - 2k  =  - kˆ  2 2 35. [k ´ (c ´ a )]  where  k  =  r r r r r r r r  =  a ´ b . (u ´ w) . c )a ]  r r r r r r  ar]  r  b´c r  =  (a ´ b ) . (b ´ c ) ù =  16  ë û ( )  r r  r 37.  .  we  need  angle  between  r r r  r  r b and c  so  consider  b + c = -a 2  2  2 Þ  b  +  c  +  2|b|  |c| cos q  =  a  Þ  cos q  =  )  a 2 . ( - kˆ )  Now  w . c ùû =  (a ´ b )  [(b ´ c ) a ]c . n  28. [u ´ v .  r  ˆ =  (i + 2 ˆ  j + 3 kˆ ) .  (a)  : (u + v .v . b ) 2  =  a 2 b 2  –  a 2 b 2  cos 2q  p  =  (4  ×  2) 2  –  (4  ×  2) 2  cos 2  6 p  1  2  =  64 ×  sin  =  64 ×  =  16  4  6 . a )  =  -  (12 + 2 2 + 32 )  =  -  2 =  –7  27.  P  =  –6  Putting  value  either  in  (1)  or  in  (ii)  we  get  r these  r r  a + 2b + 6 c =  0  ( r r r  26.w. b = 45  and  r r c = 39k = 39  r r r  \  a : b :  c  =  34 : 45 : 39  r r r  (b)  :  3l c  =  2m (b ´ a )  r r r  Þ  either  3l  =  2m  or  c || b ´ a but  3l =  2m  r r r r r r r  (a)  :  We  have  a + b + c = 0 Þ ( a + b + c ) 2  = 0 r r r r r r r r r  Þ | a |2 + | b |2 + | c |2  + 2( a × b + b × c + c × a ) = 0 r r r r r r  Þ 25 + 16 + 9 + 2(a × b + b × c + c × a ) = 0 r r r r r r  Þ 2( a × b + b × c + c × a ) = - 50 r r r r r r  Þ (a × b + b × c + c × a ) = - 25 r r r r r r  \ (a × b + b × c + c × a ) = 25 r r r  (a)  :  Given  a + b + c =  0.  (a)  :  Consider  [a ´ b b ´ c c ´ r r r r r  =  (a ´ b ) .2 ˆ  j - kˆ  | CD |  =  uuur  uuur  j - 2 kˆ  | DA | =  17  DA  =  -2iˆ + 3 ˆ  41  r r r  (c) :  If rpossible  say  a + b + c =  0 r  r  Þ  b + c =  -a r r  r r r  a  × b + c =  . ( v ´ w )  (³  [a  b  c]  =  [b  c  a]  =  [c  a  b])  uuur uuur uuur 30. (v ´ w ) r r r r r r r r r  =  u .w ) .3 kˆ  uuur  \  | AB |= 49  =  7  uuur  Similarly  BC  =  2iˆ . (u ´ v ) + w. a )c .95 Vector Algebra  r  r  31.  2  uuur  \ AD  =  33  r  r r  ˆ  ||  ur ´ vr ³  u .3 ˆ  j + 6 kˆ  uuur  \  | BC  |   =  49  =  7  uuur  uuur  CD  =  .  Þ a (1  +  2Q)  +  c  (–6  –  P)  =  0 Þ 1  +  2Q  =  0  and  –  P  –  6  =  0  Q  =  –1/2.[(b ´ c ) . (b ´ c ) ùû éë c . ( w ´ u ) . b + b .6iˆ .b 2 . u ´ v . c] a ù ë û r r r r r r r  r  r r r r  . b + b . a ]c . n  ˆ  =  0  =  v .  Þ (a . é[(b ´ c) . .5.  The teacher decided to give grace marks of 10 to each of the  students.  5. 2x n  is 4. x n  be n observations.  Statement  2  is  false.  If  V A  and  V B  represent  the  variances  of  the  two  populations. and another population B has 100 observations 151.  Statement  2  is  true;  Statement  2  is  8.  (c)  (d)  (2004)  a  correct  explanation  for  Statement  1.  (c)  variance  (d)  mean  (2013)  Let x 1 . . The average marks of boys and girls combined is 50.  If  the  standard  deviation  of  the  (a)  Statement  1  is  true.  If the mean deviation about the median of the numbers a..  (d)  Statement  1  is  true.  In  a  series  of  2n  observations. 2a.  (a)  Statement  1  is  true.  2x n  is  4s 2 . 1 + 100d  from  their  mean  is  255.  then  V A /V B  is  (a)  1  (b)  9/4  (c)  4/9  (d)  2/3... Then which one of the following gives possible values  of  a  and  b?  (a)  a = 3.  (b)  Statement  1  is  true.. and let  be their arithmetic  mean and s 2  be  their  variance. 2x 2 .0  (d)  25.  x n  be  n  observations  such  that  2  5  11  13  å x1  = 400  and   å xi  = 80.  (2007)  Suppose  a  population  A  has  100  observations  101.  STATISTICS All the students of a class performed poorly in Mathematics.0  (b)  10... 1 + d.  Statement  2  is  true;  Statement  2  is  observations  is 2..1  (c)  20.  3. ..  half  of  them  equal  a  and  6  remaining  half  equal  – a.  Let  x 1 .  (d)  Statement  1  is  true.2  (d)  10..  Statement  2  is  true. then  the  d  is  equal  to  (a)  20.  (a)  2  (b)  2  (c)  Statement  1  is  false.  (2005)  n 2  - 1  is  11.  ..  In a frequency distribution.0  (2009)  The mean of the numbers a.  then  its  mode  is  approximately  Statement­2  :  The  sum  of  first  n  natural  numbers  is  (a)  20.  (2006)  For two data sets.80.  (c)  Statement  1  is  false. b... respectively.  then  |a|  equals  not  a  correct  explanation  for  Statement  1..  Then a possible value of n among  (b)  (c)  6  (d)  (a)  2  2  2  the  following  is  (2010)  (a)  18  (b)  15  Statement­1  : The variance of first n even natural numbers  (c)  12  (d)  9. the mean and median are 21 and  4  22  respectively..  (b)  Statement  1  is  true.  (2009)  n  n  .5  (b)  22.  (2012)  9.. x 2 .  250..  2.... The variance of the combined data set is  10.  Statement 1 : Variance  of 2x 1 .. .  102...  .  6... the variances are given to  be 4 and 5  and the  corresponding  means  are  given  to be 2  and 4.  respectively..  Statement  2  is  true. 50a is 50. 8.  Statement 2 : Arithmetic mean  of 2x 1. then |a| equals  (a)  4  (b)  5  (c)  2  (d)  3  (2011)  If the mean deviation of number 1.  4.  b  = 6  (2008)  The average marks of boys in class is 52 and  that of girls  is 42.  x 2 .  . 5.0  n (n + 1)  and the sum of squares of first n natural numbers is  (c)  24.. .  Statement  2  is  false. 10 is 6 and the variance  is 6. 1 + 2d...  .  The  percentage  of boys in the  class is  (a)  80  (b)  60  (c)  40  (d)  20..  b  = 7  (c)  a = 5. 2x    2 .  200.  b  = 2  (d)  a = 1. 152.96  JEE MAIN CHAPTERWISE EXPLORER  CHAPTER  15  1. Which of the following statistical measures will not  change even after the grace  marks were  given?  (a)  median  (b)  mode  7.  b  = 4  (b)  a = 0..  not  a  correct  explanation  for  Statement  1...  Statement  2  is  true;  Statement  2  is  a  correct  explanation  for  Statement  1.  (2005)  2  n( n + 1)(2n + 1)  12. each of size 5.  Statement  2  is  true;  Statement  2  is  1  2  . 00.  (a)  16.66  (b)  177.  (a)  2.  The median of a set of 9 distinct observations is 20.  Consider  the  following  statements  :  (1)  Mode  can  be  computed  from  histogram  (2)  Median  is  not  independent  of  change  of  scale  (3)  Variance  is  independent  of  change  of  origin  and  scale.  (b)  3. If each  of the largest 4 observations of the set is increased by 2.  (b)  8.  (2002)  (d)  is  increased  by  2.  (2003)  Answer  Key  1. If the average marks of the complete  (a)  is  decreased  by  2  class  is  72. Then the corrected variance  is  (a)  188.  2  å x = 2830.  In  an  experiment  with  15  observations  on  x.  (b)  12.  (2)  and  (3).97  Statistics  13.  (a)  9.  (a)  .  (b)  10.  (a)  13.  (c)  7. then  16.  (c)  6.  In  a  class  of  100  students  there  are  70  boys  whose  average  the  median  of  the  new  set  marks in a subject are 75.  (c)  11.  (a)  15. å x = 170  One observation that was 20 was found to be wrong and was  replaced by the correct value 30.  the  following  results  were  available.  (b) 5.  (d)  4.33  (c)  8.5.  (a)  14.33  (d)  78.  then  what  is  the  average  of  the  girls?  (b)  is  two  times  the  original  median  (a)  73  (b)  65  (c)  remains  the  same  as  that  of  the  original  set  (c)  68  (d)  74.  (2003)  14.  Which  of  these  is/are  correct?  (a)  only  (1)  and    (2)  (b)  only  (2)  (c)  only  (1)  (d)  (1).  (2004)  15.  1 + d. .4 + 5...( x + 10)}  Hence s 1 2  = s 2 2  n 2..( n + 1) 2  n = 4 n( n + 1)(2n + 1)  × . + (2n )2 ) .x12 ) = -1.  M= 25 a + 26 a = 25.5 a 2 Þ 50 = s= = å | ri . = x  .  1 145 ...x 2 Þ 4 = å xi2  .x ) n 1  2  2  s2  = å {( xi + 10) ..  JEE MAIN CHAPTERWISE EXPLORER  (c) : 1 st  solution : Variance doesn’t change with the change  of  origin.5 + 5. +  n)  Þ 2500 = 2| a| × 4.  3.1 + 5.(n + 1) 2  n 6  = 2  × (n + 1)(2n + 1) .  (b) :  The numbers are 1..M |  MD( M) = n1x1 + n2 x2  n1 + n2  = 5 ´ 2 + 5 ´ 4  = 3  10 d1 = ( x1 ...P..  of 2x 1 .98  1. d2 = (x 2 .... n2 = 5. n1 = 5. x n .. x 2  =  4 x12  = (a) : Median is the  mean of 25 th  and  26 th  observation.. x 2 . + xn )  = 2 x n = 2 nd  solution  :  s12  = 4.M. = 4x \  Statement  2  is  false.. 1 + 2d.  25  × 25. + 2n = 2 (1 + 2 + . \ 5 å xi2  = 40.(n + 1) 2  3  = (n + 1)  [2(2n + 1) .M.1)  =  3 3  6. ...5 + 1.3( n + 1)]  3  = (n + 1) n 2  .  x 3 .1 = 10 \ s 2  =  1  {2| a| ´ (0.M.  2x n  = 2( x1 + x 2  + .  x 1 ..( x ) 2  n = 1  2 (2 + 4 2 + . 1 + 100d.90 55 11  (40 + 105) .5) }  50 55 11  =  10 2 11  2 5.. + 24.  10 10 10 2 N 2  n1s 12 + n2s 22 + n1d12 + n2 d2  n1 + n2 5..  2  The numbers are in A. Variance = s 2  (b) :  Statement 2 :  A. 2x 2 . \ 5 s2 = 1  10 å yi2  = 105  (å xi2 + å yi2 ) .  å yi = 20 ö 1  2 æ1 s1  = ç å xi2 ÷ . \ | a| =  4  2 = 2× (b) :  1 st  solution:  2  s1  = 2  s 2  = Mean ( x ) = 4 üï x = 2  ý 5ïþ y = 4  We have  å xi = 2 5 Variance = Þ å xi = 10  Similarly.9 = = =  .. x1  = 2 s 2  2 = 5.5 + . A.  (c)  :  Sum of first  n  even natural numbers  = 2 + 4 + 6 + ..4  è5 ø 5 Þ 1  å xi2 = 8.  1  2 2  2 nd  solution   :  s1  = å( xi . .x12 ) = 1 Given  A.. + n 2 ) ..y 2 Þ 5 = å yi2  .æçè x +2 y ö÷ø n (n + 1)  = n( n + 1)  2  n(n + 1)  = n + 1  n 1  (å xi ) 2 ..(n + 1) 2  n = 1  2 2 × 2 (1 + 22 + ..  æ1 ö 1  s 22  = ç å yi2 ÷ .16  è5 ø 5 Þ 1  å yi2 = 21.1  × (n .  Then mean = 51 st  term = 1 + 50d = x  (say) . .( X ) 2  N 2 na 2  - 0  ; 2  =  a 2  ;  2 = |a|  2 n 13. The  new  median  remain  unchanged. = 255 (given)  50 × 51  Þ d =  255  101  101 ´ 255 101 ´ 255  Þ d= = = 10..  (a)  :  According  to  problem  X  x 1  x 2  –  –  –  –  x n  x n  +  1  x n  +  2  –  –  –  x n  +  n  (a) : The mean of a. + d + 0 + d + 2d + .  15.. 5.(2)  using (1) we have  a 2  + (7 –  a) 2  = 25 Þ  a 2  + 49  – 14a + a 2  =  25 Þ a 2  – 7a + 12 = 0 \  a  = 3..8 = 34 Þ a 2  + b 2  – 12  × 7 + 72 +  21 = 34 \ a 2  + b 2  = 25  .D..  .x  |  n i =1  i  1  [50 d + 49 d + 48d + . + 50 d ]  101  + d + 0 +  d  +  2d  + .  x +  y 5  a 2  a 2  –  –  –  –  –  a 2  a 2  –  –  –  a 2 2  å ( X .  (a)  :  Mode  can  be  computed  by  histogram  Median will be changed if data’s are changed so (2) is correct....  +  x 70  =  75  ×  70  7200 - 5250  Þ 400 ³ 80 Þ 20 ³  80  Þ n ³ 4 Þ n ³ 16  =  65 \  Average  of 30  girls  =  n n n  30 n 9.  250  è 15 ø  15 Series B is obtained by  adding a fixed  quantity to each item  ( x1 + x2 + .99  Statistics  Mean deviation (M.D.... b. we know that  variance  is independent of change  16....  10 is  6 a + b + 8 + 5 + 10  = 6  5  Þ a +  b + 23 =  30 Þ  a + b =  7  Þ  n  =  17  but  not  given  in  choice.. +  x70  Again  1 =  75  10.  ...  Þ å ( xi  .8  n ( a .6) 2 + ( b .6) 2  + 4 + 1 + 16  Þ = 6....  102.  152.  2  =  14.) = = 1 101  å | x .  4 also b = 3..  +  x 100  =  7200  variances  is  1.1  50 ´ 51 2550  7.æ ´ 180 ö =  222 –  (12) 2  =  78  Series  B  =  151....  (c) :  Using  fact..  of  the  numbers  x 1  +  . + 50d]  1  = × 2d (1 + 2 + ...(i)  .M... 4  (a)  :  Let  x  and  y  are  number  of  boys  and  girls  in  a  class  respectively.  (a) : Using well known fact that root mean square of number 70  ³  A.  .  (b) : Total number of observations are 9 which is odd which  means median is 5 th  item now we are increasing 2 in the last  four  items  which  does not  effect  its value.  . + 50)  101  1 50 × 51 50 × 51  = × 2 d × =  d 101 2 101  But M.    mode  =  3  median  –  2  mean  =  3  × 22  –  2  ×  21  =  3(22  –  14)  =  3  ×  8  =  24....  11.  103.  52 x + 42 y  = 50  x+ y x 4 x  4  = Þ x = 4 y  Þ  = and  y 1 x + y 5  Again variance =  8.(1)  Required percentage = x  4  ´ 100 = ´ 100 = 80%.A ) 2  = 6.  (d) : åx  =  170 and åx 2  =  2830  Increase in åx  =  10 and åx¢  =  170  +  10 =  180  Increase in åx 2  =  900  –  400  =  500  then å x¢ 2  =  2830 +  500  =  3330  2  (a)  :  Series  A  =  101..  (b) :  Using  x =  100  of origin both series have  the same variance so ratio of their  \  x 1  +  .  12..8  5  2  2  Þ a  + b  – 12(a + b) + 36 + 21+72=5×6.  Variance  depends  on  change  of  scale  so  (3)  is  not  correct... 8..X )  N = å X 2  . \  n  =  18  is  correct  number.  200  1 1  = ´ 3330 ....X )  S X  =  0 \  X  = ( X -  X ) 2 d  =  value  of  X–  X a  a  –  –  –  –  a  –a  –a  –  –  –  –a  Value  of  X  a  a  –  –  –  –  a  –a  –a  –  –  –  –a  = 2 na 2  SX  0  =  =  0  N 2 n 2  Now  SD  = å ( X .(ii) ..  x + x2 + . + x 100 )  =  72 of series A.  153...   A pair of fair dice is thrown independently three times.  (2007)  1  arranged in some order will  form an A...  (a)  Statement­1 is true. four are  10.. equals  1  5  1  1  (b)  (c)  (d)  (a)  7  14  50  14  (2009)  9.  ú ë 2 û  æ 11  ù (b)  çè 12 .  3.  0. given that their  maximum is 6.  respectively. Statement­2 is true; Statement­2 is a  correct  explanation  of  Statement­1. The  from the  set {1. ±4.  from {1.P... Then P(B)  is  4 2 3  have  different  colours  is  (a)  1/3  (b)  2/7  (c)  1/21  (d)  2/23  1  1  1  2  (a)  (b)  (c)  (d)  (2008)  (2010)  2  6  3  3  Four numbers are  chosen  at random (without  replacement)  11. 4 ú (d)  çè 4 . ±5}.  (b)  Statement­1  is  true..  A die is thrown.  It  is  given  that  the  events  A  and  B  are  such  that  blue and two are green. Each question  has three alternative answers of which exactly one is correct.  (c)  Statement­1  is  true. given  that the  product  of  these digits is zero.. The  Statement­2 : If the four chosen numbers form an A. 8}. Let A be the event that the number obtained  is greater than 3. If the probability of at least one failure is greater  than or equal to  )  1  . then  probabilities of I and II scoring a hit correctly are 0. The probability that their minimum is  3.  probability of  getting a  score  of  exactly 9  twice  is  Statement­1 : The probability that the chosen numbers when  (a)  8/729  (b)  8/243  (c)  1/729  (d)  8/9. 3. 1 ú û  æ 1 3ù æ 3 11 ù 9  (b)  log 4 - log 3 10 10  1  (d)  log 4 - log 3 10 10  (2009)  8.  One  ticket  is selected at random from 50 tickets  numbered  00.  (2010)  A multiple choice examination has 5 questions. 01. 2. 2.  Statement­2  is  not  a  correct  explanation  for  Statement­1. .  then the  correct statement  among  the  following  is  (a) P(C|D) < P(C)  P(D)  (b)  P(C | D) =  P(C ) (c) P(C|D) = P(C)  (d) P(C|D) ³ P(C)  5. P ( A | B ) = and P ( B | A ) =  .  Statement­2  is  true. 49.  ( In a binomial distribution B n.  If C and D are two events  such that C Ì D and P(D) ¹  0.100  JEE MAIN CHAPTERWISE EXPLORER  CHAPTER  PROBABILITY 16  1. is  . The  second  plane  will  bomb  only  if  the  ±2. Then P(A È B) is  2  3  (a)  (b)  (c)  0  (d)  1  (2008)  5  5  (2011)  Consider 5 independent Bernoulli’s trials each with probability  of success p.  Two aeroplanes I  and II bomb a target  in succession.  The probability that a student will get 4 or more correct answers  just by guessing is  10 3 5  (b)  17 3 5  (c)  13 3 5  (d)  11 3 5  (2013)  Three  numbers  are  chosen  at  random  without  replacement  7. Then the probability that the sum of the  digits  on the  selected  ticket  is 8. The probability that the three balls  P ( A ) = . 12 úû  (2011)  û  An urn contains  nine  balls  of which  three  are  red.P. ±3. Three balls are drawn at random without  1 1 2  replacement from the urn.  Statement­2  is  true.2.  31  .  .  Statement­2  is  false.3 and  the  set of all possible  values  of  common difference  is {±1. p =  (c)  èç 2 . Let B be the event that the number obtained  is less than 5..  ..  20}.  if the probability of  4  9  at least one success is greater than or equal to  .  85  12. then n is  10  greater than  1  (a)  log 4 + log 3 10 10  4  (c)  log 4 - log 3 10 10  (2012)  3. is  (a)  1  4  (b)  2  5  (c)  3  8  (d)  1  5  4. 02.  (d)  Statement­1  is  false..  (a)  2. …. then p  lies  in  the interval  32  é 1 ù (a)  ê 0.  6.   25.  where  A  stands  for  P ( A Ç B ) =  4  4  complement  of  event  A.  (d)  (d)  (d)  (d)  3.  Then  the  variance  of  distribution  of  success  is  (a)  8/3  (b)  3/8  (c)  4/5  (d)  5/4.  17. The  probability  that  they  contradict  each  other  when  asked  to  speak  on  a  fact  is  13.  10.05  A  and  B  are  events  such  that  P (A È  B)  =  3/4.87  (d)  0.2  (b)  0.  18.7  19. B.  15.  12.  19.  (2004)  average of 5 phone calls during 10­minute time intervals.  At  a  telephone  enquiry  system  the  number  of  phone  calls  (a)  7/20  (b)  1/5  regarding relevant enquiry follow Poisson distribution with a  (c)  3/20  (d)  4/5.  (2005)  1  16. The probability that the target is hit  (c)  0.  Then  events  A  and  B  are  (a)  equally  likely  but  not  independent  (b)  equally  likely  and  mutually  exclusive  (c)  mutually  exclusive  and  independent  (d)  independent  but  not  equally  likely.  17. P ( A ) =  2/3  then  P ( A Ç B ) is  (a)  5/12  (b)  3/8  (c)  5/8  (d)  1/4.  (d)  êë 3 .20  0.06  (d)  0.  20.  the  probability  P(E È  F)  is:  Answer  Key  1.  5  6  55  e  (2006)  20.07  0.  Mr.  (2002)  For the events E =  {X is a prime number} and  F = {X < 4}.  (a)  (c)  (d)  (d)  5.  7. Getting an odd number is considered  a  success.  B.  21.  (2003)  22.  (b)  (b)  (d)  (d)  6.  (2007)  for  B  is  3/4.  13.08  0.  C  are  mutually  exclusive  events  such  that  P( A) = 3x + 1 .  22.  (2002)  A problem in mathematics is given to three students A.  (2005)  e 15.  A die is tossed 5 times.  then  P  (X  =  1)  is  (a)  1/16  (b)  1/8  (c)  1/4  (d)  1/32.15  0. The  probability  that  all  the  three  apply  for  the  same  house  is  (a)  1/9  (b)  2/9  (c)  7/9  (d)  8/9.  P( B ) = 1 .  (2004)  by the  second plane  is  (a)  0.  P (A Ç  B)  =  1/4. while this probability  (c)  0. Three persons apply  for the houses.14.  The  mean  and  variance  of  a  random  variable  X  having  a  binomial  distribution  are  4  and  2  respectively.  The probability that A speaks truth is 4/5.  Five horses  are  in a race.  3 úû  (c)  [0. The  probability that there  is  at  the  most  one  phone  call during a  10­minute  time  period  is  6  5  6  6  (a)  e  (b)  (c)  (d)  5  .101  Probability  (a)  0.  16.  Let  A  and  B  be  two  events  such  that P ( A È B ) =  .x  and  P(C ) = 1 .  Events  A.50. C  and  their  respective  probability  of  solving  the  problem  is  1/2.  A selects two  of  the  horses  at  random and bets on them. 2 û  ú 8  P(X) :  0.  (2003)  14.  24.35  (b)  0.  18.  Then  set  of  3  4  2  possible  values  of  x  are  in  the  interval  1  2  3  4  5  6  7  é1 13 ù (b)  êë3 . A selected  the  winning  horse  is  (a)  3/5  (b)  1/5  (c)  2/5  (d)  4/5.  Probability  that  the  problem  is  solved  is  (a)  3/4  (b)  1/2  (c)  2/3  (d)  1/3.  (c)  (d)  (b)  (a)  .  A  random  variable  X  has  the  probability  distribution:  X :  é1 2 ù (a)  êë3 .2 x .23  0.  Three houses are available in a locality.10  0.  6  1  and  P ( A ) =  1 .  (d)  (d)  (d)  (a)  (a) 2.  11.  8. The probability that Mr.  Then  the  probability  of  2  successes  is:  (a)  128/256  (b)  219/256  (c)  37/256  (d)  28/256.  The mean and the variance of a binomial distribution are 4 and  2  respectively.  9. 3 úû  4.  1]  é1 1 ù .  23.  14. Each applies for one house without consulting  others.  (d)  (d)  (a)  (c)  (2003)  23.  1/3  and  1/4.5)  equals  (a)  0  (b)  2/e 2  3  (c)  3/e 2  (d)  1 -  2  .  A  random  variable X  has  Poisson  distribution with  mean  2.77  first misses the target.  (2005)  21.12  0.  The  P (X  >  1.  (2004)  25.  (2002)  24. . 3.  2. 4}  We have A È B  = {1. . P (C ) Ì P(D).. the  number must be  of the  form  either  x0  which are 5 in numbers.  9.1 / 2]  32 2 5... 5.. 2. i.  (c) :  From  the  definition of  independence  of  events  P ( A / B ) = P ( A Ç B )  P ( B )  . 9} for the product of  digits to  be  zero.. Þ  n(log104 – log103) ³  1  P(C Ç D) = P (C )  Þ n³ We have. 2.  84 7 6. 02. 1. 4} and b Π {0. 9}  The  only  number common to both  = 00  Thus the number of numbers in S. 5.  4 1∙1∙2  Þ 8  P( B / A) = C3  2 1  P( A Ç B) = = =  5  P( B) C2  10 5 Taking logarithm on base 10 we have  8  C3  P(C Ç D )  3.  (d) :  P(C | D ) =  P(D ) \ n log10(3/4) £  log1010 –1 Þ  n(log103 – log104) £  –1 as C Ì D.  (d) : A  = {4. 1. 4. 49} is of the form  ab  where  a Π {0.  8}  without  replacement. 6} =  S  Where S is the sample space of the experiment of throwing  a die..  .  = 5´2 + 1 =  11  7. 2. 2.  (d) : P(correct answer) = 1/3  The  total number of ways n(S) =  20C 4  4 æ 1ö æ 2ö æ 1 ö 5  The  required  probability = 5C4 ç ÷ ç ÷ + 5 C5 ç ÷ è 3ø è 3ø è 3 ø  The desired probability =  57 20  C 4  = 57 ´ 24 1  = 20 ´ 19 ´ 18 ´ 17 85 Now statement­2 is false and statement­1 is true..P. 1.’s  with  common  difference  6 =  57  1  log10 4 - log10 3  8.P. 3.’s  with  common  difference  3  =  11  Number of A. 2.’s  with common  difference  5  =  5  2  Number of A.  (d) : Probability of  at least  one  success  35 35 35  = 1 – No success = 1 –  n C n  q n  where q  = 1 –  p = 3/4  2.  (b) :  n(S) = 9 C3  = 9 ´ 8 ´ 7  = 84  6 n(E ) =  3C 1  ∙  4C 1  ∙  2C 1  = 3  × 4 ×  2 =  24.  (d) :3  numbers  are  chosen  from  {1.  Hence P(A È B) = 1  10.æç ö÷ ³ è 4 ø  10 Let B be the event that the minimum of chosen numbers is 3. .P.  The  desired probability  =  4  1 æ3ö 1  æ3ö ³ç ÷ Þ ç ÷ £ è 4 ø  10 10 è 4 ø 24 2  =  .  3.. 01. the product of whose digits  is zero = 10 + 5 – 1 = 14  Of these the number whose sum of digits is 8 is just one.’s  with common  difference  4  =  8  Number of A...P. 1.  we want  1 . 3...’s  with  common  difference  1 = 17  Number of A. 3. for it is a sure  event.102  JEE MAIN CHAPTERWISE EXPLORER  1.P.’s  with  common  difference  2  =  14  Number of A. 6}  Also B  = {1.  P(S) = 1. Let A  be the event that the maximum of chosen  4  3 9  numbers is 6.  08 The  required probability  = 1/14.  (c) : Number of A.  4}  or of the form 0x  which are  10 in numbers because  x Π {0. P(C | D) =  P(C )  P ( D) As 0 < P(D) £  1  we have P(C|D) ³ P (C )  4.. because x Î{0.p 5  ³  Þ p 5  £ 31  32 1 1  thus p £ \ p Π[0. 2.  (a) : Probability  of  at  least  one  failure  = 1 – P (no  failure)  = 1  – p 5  Now 1 .P.  (d) : Any number in the set  S  = {00..e. . P ( B ) + P ( A Ç B )  1 3 1  = 1 .77  1  5  3  1  P(B)  =  \  P ( B )  =  4 4  Now  we  needed  P(A)  P ( B )  +  P(B)  P ( A )  19.  3..62  P(F)  =  P(1  or  2  or  3)  (events  1.0.2  .  B.(ii)  .)  =  P(1)  +  P(2)  +  P(3) =  ..(1)  .... (6.  (d)  : We  know  that  poission  distribution  is  given  by  -l r  P ( x = r ) = e  l where l = 5  r !  Now  P(x  =  r £  1)  =  P(x  =  0)  +  P(x  =  1)  = -l e -l l e  6  + = e -5 (1 + 5) =  5  .3.  (c)  :  No..  (d)  :  P ( A È B ) = 1 .  0! 1!  e e . 4).  9  33  16. we have from (1) and (2)  P(A)P(B/A) =  P(B)P(A/B)  Þ ..7.  (b) : Possibility of getting 9 are (5.P ( A È B )  = 1 .P ( B ) +  6 4 4  P ( B )  = 1 - 1  Þ P ( B )  = 4 =  1  2 6  12 3  1 = 1 ´ 3  =  P ( A ) P ( B )  now  P ( A Ç B ) = 4 3 4  so  even  are  independent  but  not  equally  likely  as  P(A) ¹  P(B).. 5).103  Probability  Then  P(B) ∙ P(A/B) = P(A Ç  B)  Interchanging the  role of A and  B in  (1)  P(A)P(B/A) =  P(B Ç  A)  As A Ç B =  B Ç A.  è 9 ø è 9 ø  243  12.  C  are  mutually  exclusive \  0 £  P(A)  +  P(B)  +  P(C) £  1  0 £  P(A).35  P(E È  F)  =  P(E)  +  P(F)  –  P(E Ç  F)  =  .  npq  =  2 2  1  npq  =  \  q  =  p  =  and  n  =  8  Þ  4 2  np  8  æ 1 ö Now  P(X  =  r)    =  8 C r ç ÷ è 2 ø  (Use p(X  =  r) =  n C r  p r q n–r 8  15.  of  horses  =  5 4 3  ´  5 4 Probability  that  ‘A’  must  win  the  race  =  1 -  P ( A ) P ( B )  \  Probability that  A  can’t  win  the  race  =  =  1 -  12  2  =  20 5  22.P ( A ) .  (d)  :  A.  P(B).3 =  0.2. (d) :  Given  np  =  4  and  npq  =  2  npq  2 1  1  = =  so  p  =  1  –  1/2  =  q =  2  np 4 2  Now  npq =  2 \  n  =  8 \  BD  is  given  by P(X  =  r)  =  8 C r  p r  q n  –  r 8  28  æ 1 ö P(X  =  r  =  2)  =  8 C 2 ç ÷ =  256  è 2 ø  18.  (d) :  P (I) = 0.  of  favourable  cases  No.l lr  ( l = mean )  14.  5.  7  \  ‘E’  denote  prime  ‘F’  denote  the  number  <  4 \  P(E)  =  P(2  or  3  or  5  or  7)  (Events  2.5)  =  P(2)  +  P(3)    +  .  3.2 = 0.. ¥  =  1  –  [P(0)  +  P(1)]  -2 2  ù é e  ´ 2 3  = 1 ..  (a) :  P(A) =  =  4  5 \  P( A  ) =  4 1 3 1 7  ´ + ´ =  5 4 4 5 20 20. 13.ê e -2  + = 1 .7)(0. P (II ) = 1 .E)  =  P(2)  +  P(3)  +  P(5)  +  P(7)    =  . (3.  P(C) £  1  Now  on  solving  (i)  and  (ii)  we  get  1  1  £  x £  3 2 . 3).  (a)  :  No.  (d)  :  P ( X = r ) = r !  \  P(X  =  r  >  1.50  P(E Ç  F)  =  P(2  or  3)  =  P(2)  +  P(3)  =  .  (b)  :  From  the  given  table  prime  numbers  are  2.  5.35  =  .  8 1  æ 1 ö = ÷ =  2 16 ´  16 32 è ø  \  p(X  =  1)  =  8 C 1 ç 21.  P (II) =  0.(2)  1 2 1  1 2 1  × = P ( B ) ×  Þ P ( B ) = × × 2 =  4 3 2  4 3 3  11. 6)  4 1  Probability of getting score 9 in a single throw =  p = 36 =  9  Required  probability =  probability of  getting  score 9  exactly  2  twice  = 3  C2 æç 1 ö÷ ´ æç 8 ö÷ = 8  ..14. P ( I ) = 1 .8  Required probability =  P ( I Ç II) =  P ( I ) P (II)  = (0.  3  are  m.0.2) = 0.  2  úû  ë e 17.  of  houses  =  3  =  No.  2.50  –  .  of  applicants  =  3.(i)  .  7  are  M.  (d)  :  Given  mean  np  =  4.E. \  Total  number  of  events  =  3 3  (because  each  candidate  can  apply  by  3  ways)  Required  probability  = 3 =  1 . (4.62  +  .   or  P(A È  B È  C)  =  1  –  P ( A ) P ( B ) P (C )  1 2 3  =  1  –  .P ( A Ç B )  3  By  using  P(B)  =  P(A È  B)  +  P(A Ç  B)  –  P(A) 2 1  P ( A Ç  B )  =  -  =  5/12  \  3 4 25.  =  3/4  2 3 4  =  2 ´ 10  32 5  æ 1 ö ÷ è 2 ø  =  . (d) :  n  =  5  p  =  q  =  1/2  P(X  =  r)  =  5 C r ç ÷ è 2 ø  x i  f i  f i  x i  f i  x i 2  5  (0)  0  5  æ 1 ö ÷ è 2 ø  (1)  5 C  1 ç (2)  5 C  1 ç (3)  5 C  3 ç (4)  5 C  4 ç (5)  5 C  5 ç 5  æ 1 ö ÷ è 2 ø  5  æ 1 ö ÷ è 2 ø  5  æ 1 ö ÷ è 2 ø  æ 1 ö ç ÷ è 2 ø  0  1 ´ 5  32 5  32  2 2  3 ´ 10  32 3 2  4 ´ 5  32 4 2  5  ×  1  32  5 2  0  10  32  10  32  5  32  1  32 240  80 å f i x i 2  =  32  32 5  x = mean  =  2  å f i  = 1 å f i x i =  å f i xi2  .  (a)  :  Given  P  (A È  B)  =  3/4  P(A Ç  B)  =  1/4  P ( A )  =  2/3 2  \  P(B)  =  P ( A Ç B ) = P ( B ) .  (a)  :  Given  P(A)  =  1/2 \  P ( A )  =  1/2  2  P(B)  =  1/3 \  P ( B )  =  3  1  3  \  P (C )  =  P(C)  =  4 4  Now problem will be solved if any one of them will solve the  problem. \  P(at  least  one  of  them  solve  the  problem)  = 1 – probability  none of them  can  solve  the  problem.104  JEE MAIN CHAPTERWISE EXPLORER  n æ 1 ö 23.çæ å fi x i  ÷ö Now  variance  =  å f i çè å fi  ÷ø 2  240 25  -  32 4 40 5  =  32  4  A  B 24. . 1 æç 5 ö÷ = p .  The  height  of the  tower is  (a)  a / 3  (b)  a  3  (c)  2 a / 3  (d)  2 a  3.  In a DPQR.  A tower stands at the centre of a circular park. then  (2013)  (a)  7 3 2  1  m  3 + 1 (c)  7 3  ( 3 + 1) m  2 (b)  7 3 2  1  m  3 - 1 (d)  7 3  ( 3 - 1) m  2 4.  and  the  angle of elevation of the top of the tower from A or B is 30°. p ÷ö for which the function.  If x.  è 2 2 ø  .  è ø  (2007)  13.  then  (a)  2x =  3y  =  6z  (b)  6x  =  3y  =  2z  (c)  6x =  4y  =  3z  (d)  x  =  y  =  z  (2013)  2.x 2  -1 æ x  ö f ( x) = 4 + cos .  ABCD is a trapezium such that AB  and CD are parallel and  BC ^ CD. 2 ÷ ë ø  é pö (b)  ê0.1 æç x ö÷ + cosec .  5  13  4 Then tan2a  =  (a)  25 16  (b)  56 33  (c)  19 12  (d)  20 (2010)  17  7. BC = p and CD = q. A and B  are  two points on the  boundary of the  park such  that  AB  (= a)  subtends  an  angle  of  60°  at  the  foot  of  the  tower.  (2007)  . if 3 sin P + 4 cos Q = 6 and 4 sin Q + 3cos P = 1. 2 ÷ . where  0 £ a .  AB is a vertical pole with  B at the ground level and  A at the  top. He moves away  from  the  pole  along  the  line  BC  to  a  point  D  such  that  CD = 7 m.2 .  If  A =  sin 2 x  +  cos 4 x.105  Trigonometry  CHAPTER  TRIGONOMETRY 17  1.  Let cos(a + b) =  (d)  3 13  £ A £  4 16 13  £ A £ 1  16 (2011)  4  5  p and let sin(a – b) =  . z  are in A.  (2013)  Then the  height of the  pole is  tan A cot A + can  be written  as  1 . From D the angle of elevation of the point A is 45°. then  the angle R is equal to  5 2  10. A  man  finds  that the  angle  of  elevation  of  the  point  A  from a certain point C on the ground is 60°.  The expression  p2 + q2  (b)  p2 cosq + q2 sin q  (d)  ( p2 + q2 )sin q p cos q + qsin q  (b)  tanA + cotA  sinA cosA + 1  (a)  (b)  (c)  (d)  A is false and  B  is true  both  A and B  are true  both  A and B  are false  A is true and B  is false  (2009)  9. A  false statement  among the  following  is  (a)  there is a regular polygon  with r/R  = 1/2  (b)  there is a regular polygon  with r/R  =  1 / 2  (c)  there is a regular  polygon with  r/R =  2 / 3  (d)  there is a regular polygon  with  r/R =  3 / 2  8. let r and R be the radii of the inscribed  and  the circumscribed  circles.1 + log (cos x )  is defined.  b £  .  2 ÷ ë ø  (c)  [0..  The largest interval lying in  çæ -p .  tan –1 y and  tan –1 z  are also  in  A.P.cot A 1 -  tan A (a)  secA cosecA + 1  (c)  secA + cosecA(d)  3  2 If cos(b  – g)  +  cos(g  – a)  +  cos(a  – b)  = -  .  and  tan –1 x.  is  è 2  ø  é p pö (a)  ê . If ÐADB = q.  then  for  all  real  x  (a)  1 £ A £ 2  (c)  (b)  3  £ A £ 1  4 6.  then the values  of  x  is  è5ø è 4 ø  2  (a)  4  (b)  5  (c)  1  (d)  3.  If  sin .  (2007)  12. y.P.  For a regular polygon.  Let  A  and  B  denote  the  statements  A  : cos a  +  cos b  +  cos g  =  0  B  :  sin a  +  sin b  +  sin g  =  0  (2010)  (d)  (2008)  4  17  (2008)  11. p]  æ p pö (d)  ç .  The value  of   is  cot æ cosec -1 + tan -1  ö è (a) p/4  (b)  3p/4  (c)  5p/6  (d) p/6  3 3 ø  (2012)  5  6  3  (a)  (b)  (c)  17  17  17  5. then AB is equal to  (a)  (c)  p2 + q2 cos q p cos q + qsin q  ( p2 + q2 )sin q 2  ( pcos q + qsin q ) 3.4 .   If  u = a 2 cos 2 q + b 2 sin 2 q + a 2 sin 2 q + b 2 cos 2 q  (c)  tana  (d)  cot(a/2)  (2002)  then the difference between the maximum and minimum values  31. the altitudes from the vertices A.  In a triangle PQR.b  4 3 is  of  cos  2 (4 + 7)  (1 +  7)  3  3  6  - 6  (c)  -  (d)  .  (b)  (b)  (a)  (d)  (a)  .  The  upper  3/4  2  (a)  4  (b)  2sin2a  tan –1 (3/5) at a point in the horizontal plane through its foot and  (c)  –4sin 2a  (d)  4sin 2a.  In a  triangle ABC.  (2003)  17.  If  sina + sinb = –21/65.P.  medians  AD and  BE  are  drawn.  A  possible  height  of  the  vertical  pole  is  p 18. a cos 2 2 2  14.  a ¹  0  then  (a)  b =  a +  c  (b)  b  =  c  –1  –1  (c)  c  =  a +  b  (d)  a  =  b  +  c  (2005)  28. then 4x 2  –  4xy cosa  +  y 2  is equal  to  th  portion  of  a  vertical  pole  subtends  an  angle  26..  If AD  =  4.  sides  are  in  H.  If  r is  the  inradius  and  R  is  (a)  40 m  (b)  60 m  2  (c)  80 m  (d)  20  m.  (2003)  the  roots  of  ax 2  +  bx  +  c  =  0.  b  and  c  16.  (2003)  river is 60º and when he retires  40 meters away from the tree  2  2  the angle of elevation becomes 30º.  sinB. b  be  such  that p < a  – b  <  3p.  Then  the  greatest angle  of  the  triangle  is  1 1  2 -1 é 1  30.  (d)  (b)  (c)  (d)  (*)  5.  then the  area  of  the DABC is  (d)  G.  (a)  (b)  (c)  -  (d)  3 4 65  65 130  130 (2004)  15.P.  (2006)  24.  23.  The  sides of  a  triangle  are  sina.  (2005)  at  a  distance  40  m  from  the  foot. (c)  A.  30.  If in a triangle ABC.  if  ÐR =  .  In a triangle with sides a.  15.  29.  13.  In  a  triangle ABC. has a solution for  ( )  ( )  ( )  ( )  ( )  1  (a)  all  real  values  (b)  | a |  <  20.  20.  25.  let  ÐC =  . b.  24. and cosa + cosb = –27/65.  is  a  p  p  p (b) a cot  2 n (a) 2 cot  2 n æQö P  and  tan ç ÷ are  19.  21.  17.  18.  (2005)  (a)  16/3  (b)  32/3  (c)  64/3  (d)  8/3.  (b)  Arithmetic­Geometric  progression  25.  cosa and  1 + sin a cos a  for  (c)  a  >  b  and  b  <  c  (d)  a <  b  and  b  >  c.  11.P.  (2006)  . 3p] satisfying the  equation 2sin 2 x  +  5sin  x  –  3  =  0  is  2 C + c cos 2  A = 3 b . B.  (2002)  p  some 0 < a <  .  Let a.  cot ë (cos a ) 2 ùû + tan éë (cos a ) 2  ùû  = x (a)  120º  (b)  90º  then  sin  x  =  (c)  60º  (d)  150º.  (d)  (c)  (d)  (c)  (*)  (b) 2. then the value  (1 -  7 )  (4 -  7)  (b)  (a)  a .  19.  (2004)  (a)  a >  b >  c  (b)  a <  b <  c  21.P.  2  2  (c)  2(a  +  b )  (d)  (a  –  b) 2 .  The trigonometric equation sin  x = 2sin  a.  A person standing on the bank of a river observes that the angle  2  of  elevation  of  the  top  of  a  tree  on  the  opposite  bank  of  the  1  1 1  (c)  | a |  ³  (d)  2  < | a | <  .  (c)  (a)  (d)  (c)  (a)  6. r  > r  > r  (which are the ex­  1  2  3  (a)  40 m  (b)  30    m  radii)  then  (c)  20 m  (d)  60  m.  (d)  (b)  (c)  (c)  (a)  3.  The sum of the radii of inscribed and circumscribed circles for  (c)  c  +  a  (d)  a  +  b  +  c  (2005)  an  n  sides  regular  polygon  of  side  a.106  JEE MAIN CHAPTERWISE EXPLORER  23.P. C on opposite  (a)  are  in  G.  28.  (2004)  (a)  1  (b)  cot 2 (a/2)  22.  8.  10.  then  2(r  +  R)  equals  (a)  a  +  b  (b)  b  +  c  27.  27.  16.  14.  (2003)  the  circumradius  of  the  triangle ABC.  2p)  of  u 2  is  given  by  is  (a)  (a  +  b) 2  (b)  2  a 2 + b 2  (a)  2  (b)  3  (c)  0  (d)  1.cos -1  y  = a .P.  26.  If tan  2  2  è 2  ø  a  p  p  (c) 4 cot  2 n (d) a cot n .  22.  31.  7.  The number of value of x in the interval [0.  (a)  (c)  (a)  (a)  (a)  4.  then the sides  (a)  4  (b)  6  (c)  1  (d)  2.  If cos -1 x .  sinC  are  in  (c)  satisfy  a  +  b  =  c  (d)  are  in A.P.  12. c.  then  sinA.  ÐDAB = p/6 and ÐABE = p/3.  then  tanx  is  () ( )  a.  (b)  are  in  H.  (2004)  (2002)  Answer  Key  1.  The  number of solutions  of tan  x  + sec x  =  2cos  x  in  [0.  9.  (2003)  (a)  H. The breadth of the river is  29.  If in a DABC.  If  0 <  x  < p and  cosx  +  sinx  =  1/2.  n  = 4. we get  p2 + q2 sin q AB BD = Þ AB = sinq sin(q + b) sin(q + b ) As  tan b =  sin(q  + b) = sinq  cos b  +  cos q  sinb  q 2 p +q 2 + cos q × p +  q 2  = pcosq + q sin q p2 +  q2  2 2  We then get  AB = ( p + q )sin q p cos q + qsin q  3.  Remark : y ¹  0  is  implicit  to  make  any  of  the  choice  correct.  (d) : Using sine rule in triangle ABD.  (a) :  sin 2 A cos 2 A + cos A(sin A . we  have  q = sin q × 5.  n 2 n 6 p 2  But cos =   will produce no value of n.  we  get  x  =  y  =  z.  2 6 3 5  + 4 12  = 36 + 20 = 56  3 5  48 - 15 33 1 .cos 3 A (sin A -  cos A)cos A sin A (sin A .  Again A =  t + (1 –  t) 2  =  t 2  –  t + 1.b)  = = 1 .  (b) :  cos(a  + b) = 4/5 giving tan(a  + b) = 3/4  Also sin(a  – b) = 5/13 given  tan(a  – b) = 5/12  r R p 1  Let  cos =  .  (d) : As  x. t ³  0..107  Trigonometry  1.cot A 1 -  tan A = = 3  £ A £ 1 .P. R =  .cos A)(sin 2 A + sin A cos A + cos 2 A)  = (sin A -  cos A)sin A cos A 1 + sin A cos A = 1 + sec Acosec A sin A cos A 4.  \  n  = 6.  acceptable.xz ÷ø  æ 2 y ö æ x + z ö Þ tan -1 ç = tan -1 ç 2 ÷ è 1 .  (c) : We have  = cos p 2 Thus  tan(a + b) + tan(a .P.  n 3 cos 1  p p p < cos < cos  3 n 4  As 1 2 < < 2 3 Þ p p p > > Þ 3 < n <  4 (impossible)  3 n 4 2 Þ cos 8.  where minimum is 3/4  p 1  p p = Þ = .  (c) : A = sin 2 x + cos 2 x  We have  cos 4 x £  cos 2 x  sin 2 x = sin 2 x  Adding sin 2 x + cos 4 x £  sin 2 x + cos 2 x \  A £  1.  2.cos A) sin A(cos A -  sin A) = sin 3 A .  (b) : cos(b  – g) + cos(g – a) + cos(a – b) = – 3/2 Þ  (cosb cosg + sinb sing) + (cosg cosa + sing sina)  + (cosa  cosb + sina  sinb) =  – 3/2 Þ  2(cosb  cosg  + cosg  cosa  + cosa  cosb)  + 2( sinb  sing  + sing  sina  + sina  sinb) + 3 = 0 Þ  {cos 2a  + cos 2b  +  cos 2g  + 2(cosa  cosb + cosb  cosg  +  cosg  cosa)}  2 + {sin  a + sin 2b + sin 2g  + 2(sina sinb + sinb sing + sing  sina)} = 0 Þ  (cosa + cosb + cosg) 2  + (sina + sinb + sing) 2  = 0  Which yields simultaneously  cosa  + cosb  + cosg = 0 and sina  + sinb  + sing  =  0 .  cos tan A cot A + 1 .  n 2 n 3 \  n = 3.  n i.xz ÷ø è 1 .  y.  z  are  in A.  (d) :3 sin P + 4cos  Q  =  6  4 sin Q + 3cos P =  1 Þ  16 + 9 + 24 (sin (P + Q)) =  37 Þ  24 (sin (P + Q)) = 12  Þ sin ( P + Q) = But if  R =  1 1  5 p p Þ sin R =  Þ R =  or  2 2 6 6 5 p p 1  then P <  and  then  3sin P <  6 6 2 and so 3sin  P + 4 cos Q  < 6.  then  2tan –1 y =  tan –1 x +  tan –1 z  æ x + z ö 2 tan -1 y  = tan -1 ç è 1 . n 2 p p Thus we get  =  n 4 1  p + 4 ( ¹ 6)  Thus..  p 3  p p = Þ = . acceptable.  acceptable.b)  p .  4 7.´  4 12 p .tan(a + b)tan(a .e. tan –1 y  and  tan –1 z  are  in A.y ø  …  (i)  Thus y 2  =  xz  …  (ii)  From  (i) and  (ii). Þ  2y  =  x  +  z  tan –1 x. 7  of B  from C  i.1 £ 1 and cos x > 0  2  or  0 £ x £ 4 and .  BD  to BC  is  3 :1  Q  tan x < 0 Þ tan x =  8 BD BD  3  = 3.  a b c 17.x 2  1 = cos a 2 4  a  30°  B  x  12.B 2 )  Þ  Þ  18.  (c) :  0  <  x  < p  60°  45°  7  C  D  Þ h- h  = 7  Þ h ( 3 .  JEE MAIN CHAPTERWISE EXPLORER  (c) : 1 st  Solution  :  Let  height of  the  pole AB  be h.sin -1 çæ ÷ö ÷ 5 è 5 ø ø  è2 A  x = cos æ sin -1 4 ö = cos æ cos -1 3 ö = 3  ç ç 5 5 ÷ø 5 ÷ø 5  Þ  x = 3..  510°.p < x <  p \ 0 £ x <  p .36 -4 ± 7  2 nd  Solution  :  tan x = =  6 3 We use the fact that the ratio of distance of B from D and that  -4 . b ..sin -1 çæ ÷ö è5ø 2 è 5 ø  BC = h  cot 60° =  h / 3  BD = h cot 45° =  h  As BD – BC = CD  æp x 4  ö = sin ç . 2  2 2  y  = a 2  \  60°  a  )  = cos -1 AB + (1 .  Then  p -1 æ x ö -1 æ 4 ö 13.cos -1  Þ  xy y 2  + 1 . 2 D . So  number  of  values  are  4  7  3 3 7 3  like  30°.  (d) :  sin ç ÷ + sin  ç ÷ = è5ø è 5 ø 2  x p 4  Þ  sin -1 çæ ÷ö = .1) = 7 3  3  Þ h= 7 3 7 3( 3 + 1) 7 3  = = ( 3 + 1)m  2 2  3 - 1  1  Given  cos x + sin x = 2  1  1 + sin 2 x =  Þ  4  2 tan x  -3  = 4  1 + tan 2  x Þ  3tan 2  x  +  8tan  x  + 3  =  0  (By  squaring  both  sides)  -8 ± 64 .sin x ¹ - 3  2  1  there  sin x =  2 .÷ = (1 .  è è h  B  14.  (b) :  cot æ cosec -1 + tan -1  ö è 3 3 ø  \  Altitudes  are  in  H. we  know  that  each  trigonometrical  function  60°  45°  assumes  same value  twice  in  0 £  x £  360°.  CD = ×7 = ( 3 + 1)  Then  BD = 2  3 -1 3 - 1  16.  B  D  C  3 2  \ 2 D . c Π A.A2 ) (1 .  (a) : OP = Tower  OAB is equilateral triangle \  OA = OB = AB = a  In D AOP.cos -1 B ( cos -1 x . the height of the  pole =  2  2 2 × Area of  D \ = AD  5 2  a 10.e.(A)(Q  C  =  90º)    and  .P.108  9.  tan 30°  =  OP  OA Þ  OP =  a  3  A  30° a O  Þ 1 1 1  .x ) ç 1 ÷ 2 ø 4  ø è è 4x 2  –  4xy  cosa  +  y 2  =  4  (1  –  cos 2a)  =  4sin 2a.× ÷ø  çè 4 3 11.  390°. Π H.P.P.  (b) : f (x) is defined if  -1 £ .  150°.  B  C  D  In  our  problem  0° £  x £  540°.  so that  = 2  BC CD 15.  (d)  :  Using  cos -1 A . 2 D Π  HP  = cot æ tan -1 + tan -1  ö a a b c è ø  4 3 3 2  ö æ + ç -1  4 3  ÷ æ -1 17 ö 6  = cot ç tan  3 2 ÷ = cot è tan  6 ø  = 17  1 .  c  = 2 R  sin C \  c  =  2R  .  (a)  :  2  æ xy ö y 2  ö æ 2  ç cos a .  Þ a . .  (a)  :  2sin  x  +  5  sinx  –3  =  0 3 - 1  A  1  Þ  sin x = .  (c)  :  Altitude  from  A to  BC  is  AD  A  1 AD ´ BC 7 3  Area  of D  =  ( 3 + 1)m  As AB  =  BD. (B)  Q Q  tan P + tan = 1 .b )  = cos 2  =  =  4 ´ 65 ´ 65 (130) ´  (130) 2 130 a .  (c) : ÐR  =  90° \ ÐP + ÐQ  =  90°  Q  P 90  Q  P \ = -  .  (a)  :  If  a  =  sin a..109  Trigonometry  tan C = r 2  s-c A  2(1  + cos a  cos b  +  sin a  sin b)  =  \  r =  (s –  c) ( tan 2[1  +  cos  (a  – b)]  =  R )  B  C  a + b + c  = -  c ;  2r  =  a  +  b  –  c  2  adding  (A)  and  (B)  we  get  2(r  +  R)  =  a  +  b.  and  r  be  the  radius  of  inscribed  circle  then  R  =  a  a  p  p cosec  and  r  =  cot  2  2  2n n \ R  +  r  =  a æ p pö ç cosec + cot  ÷ 2 è n n ø  pö æ 1 + cos  ÷ aç n  = a cot  p =  ç ÷ 2 ç sin p ÷ 2 2 n  ç ÷ n ø  è A  O  R  π  π  R  n  n  B  C  L  B  OL  =  r  OB  =  R  ..tan a tan b  =  1 .  (a) : If R be the radius of circumcircle of regular polygon of  n  sides.  (c) : sin a  + sin b  =  -  65 27 and  cos a  +  cos b  =  -  65 by  squaring  and  adding  we  get  E  p/3 B  O  C  D 1 8 3 16 3  ´ 4  =  Area  of  triangle  ADB  =  ´ 2 9 9 16 3 32 3  =  Area  of triangle ABC  =  2  ×  9 9 + b 4 sin 2 q cos 2 q + a 2 b 2 sin 4 q + cos 4  q  = a 2  +  b 2  +  2 A  OB  25.  (d)  :  2a  cos 2  Q  æ ö P  2  ç Q tan .ç ÷ ç ÷ è 40 ø è 160 ø h 2  –  200h  +  6400  =  0 (h  –  40)  (h  –  160)  =  0 h  =  40  or  h  =  160  27.  (*)  :  =    tan  30° AO  OA 8 3 uuur  =  Þ  OB  =  9  3  26.  a 30°  20.  c  =  1  +  sin a  cos a  then  cos  c  =  60°  O  .b  = 1  Þ  c  =  a  +  b.tan A tan a  22.  (c)  :  Breadth  of  river  OC  =  AC  cos  60°  30°  =  40  cos  60°  B  C  2  2 2  2 2  21.  . u 2  =  a 2  +  b 2  +  2ab  =  (a  +  b) 2  Now Maximum  u 2  –  Minimum  u 2  = 2(a 2  +  b 2 )  –  (a 2  +  b 2  +  2ab)  =  a 2  +  b 2  –  2ab  =  (a  –  b) 2  = a 2  +  b 2  +  2  +  21 23.  = 45  2 2 2  2 2  1 tan Q  / 2  tan P / 2  = Þ  1 1 + tan Q / 2  Þ  (65)2  (65) 2  1170 130 ´ 9  9 (a .  b  =  cos a.b  3  =  \ cos  2 130  r  C = tan 45° =  1  2  (21)2 + (27) 2  Þ  Þ  Þ  3  =  5  3/4 h  b  a A 1/4 h h æ h  ö +ç÷ 40 è 160 ø æ h öæ h ö 1 . tan  are roots of ax + bx + c = 0 ÷ 2 2  è ø Þ  1170  æa -bö As p  < a  – b  <  3p  then  cos  ç ÷ =  negative  è 2 ø  19.tan P × tan  2 2 2 2  Þ  - Þ  Þ  Þ  Þ  b c  = 1 -  a a A  c .  (d)  :  u 2  =  a 2 (cos 2q  +  sin 2q)  +  b 2 (sin 2q  +  cos 2q)  + 2 C  +  2c  cos 2 A/2  =3b  (from  given) 2  a(1  + cos  C)  +  c(1  +  cos  A)  =  3b a  +  c  +  a  cos  C +  c  cos  A  =  3b  (a  cos  C  +  c  cos  A  =  b  projection  formula) a  +  c  +  b  =  3b a  +  c  =  2b  24.  u 2  =  2(a 2  +  b 2 )  and  Min.  (a)  : a  =  A  + b \ b  =  A  – a  tan A .b 2  ö a 2 b 2  + ç ÷ sin 2 q ç 2  ÷ø è \  u 2  will be maximum or minimum according as q  = p/4 or q  =  0° \  Max.sin a cos a \ cos  c  =  –1/2  2 sin a cos a (a 4 ) ( 2 2 ( 2 a b + a -b 2 2  )  )  sin Þ  2 2  q cos  q  æ a 2 .   150°.p £ sin -1 x £ p ú 2 2 û ë D D D > > s .  (*)  :  sin –1 x =  2  sin –1 a Þ  Þ  - p p  £  2  sin –1 a £  2 2 p p Þ  £  sin –1 a £  4 4 æ pö æpö Þ  sin ç .110  JEE MAIN CHAPTERWISE EXPLORER  28. sin  x  =  –1  2  so  there  are  three  solution  like x  =  30°.  (b) :  tan  x  +  sec  x  =  2  cos  x  1  + sin  x  =  2  cos 2 x  sin 150° = ½  sin 30° = ½  1 +  sin  x  =  2(1  –  sin 2 x) Þ  2 sin 2 x  + sin  x  –  1  =  0 Þ  (2 sin  x  – 1)  (1  + sin  x)  =  0 1  sin 270° = – 1 Þ  sin  x =  .÷ £  a £  sin ç ÷ è 4ø è4ø Þ  - 1  2 Þ  |a| £  £  a £  1  2  1  2 No  choice  is  matched.  (a) : As  r 1  >  r 2  >  r 3 Þ  éQ sin -1 x = 2sin -1 a  ù ê ú ê and .  (a) :  Using  tan –1q  +  cot –1q  =  \  sin  x  =  sin  p  =  x 2 p  =  1  2 31.a s .  29.  270° .b s - c Þ  s-a s-b s-c < < D D D a >  b  >  c  30.   (c)  Statement­1  is  false.  (c)  5.  (a)  (a) 2.  (2013)  (b)  Statement­  1  is  true.  R  : Suman is honest  The negation of the statement “Suman is brilliant and dishonest  Statement­1 :  r  is equivalent to either  q  or  p.  if and only if Suman is  rich” can be expressed  as  Statement­2 :  r is equivalent to ~ ( p «  ~ q).  (c)  Every rational number x Î S satisfies x £  0. Statement­2 is true.  Let S  be  a  non­empty  subset  of R . then I  (c)  Statement­1  is  false.  Statement­2  : ~  (p « ~ q) is a tautology.  (2010)  not  a  correct  explanation  for  Statement­1.  Statement­1  : ~  (p « ~ q) is equivalent to p «  q. Statement­2 is true; Statement­2 is a  4.  (b)  There is no rational number x Î S  such  that x £  0.  Statement­2 : (p ® q) «  (~ q ®  ~ p) is a  tautology.  correct  explanation  for  Statement­1  (2009)  (c)  I  will  become  a  teacher  and  I  will  not  open  a  school.  Statement­2  is  true;  Statement­2  is  (d)  Statement­1 is true.  Statement­2  is  false  (c)  ~  P Ù  (Q «  ~  R)  (d)  ~  (Q «  (P Ù  ~  R))  (b)  Statemen­1  is  false.  Statement­2  is  true;  Statement­2  is  (b)  I  will  not  become  a  teacher  or  I  will  open  a  school.  Statement­2  is  false  2.  Statement­2  is  true.  (a)  Statement­1  is  true. and r be the statement  Q  : Suman is rich “x  is a rational number iff y  is a transcendental number”.  (b)  6.  Statement­2  is  true  will open a school”. q be the  P  : Suman is brilliant statement “y is a transcendental number”.  (d)  Either  I  will  not  become  a  teacher  or  I  will  not  open  a  6.  (a)  ~ Q « ~  P Ù  R  (b)  ~  (P Ù  ~  R) «  Q  (a)  Statement­1  is  true.  (a)  Statement­1  is  true.  (d)  Statement­1  is  true.  (b)  Statement­1  is  true.  Statement­2  is  true;  Statement­2  is  P  : There  is a rational number x Î S  such that x  > 0.  7.  (d)  4.  The negation of the statement “If I become a teacher. is  (a)  Neither I will become a teacher nor I will open a school.  Consider  the  following  correct  explanation  for  Statement­1  statement: (d)  Statement­1  is  true.  Consider :  (a)  There is a rational number x Î S such that x £  0.  Let p be the statement “x is an irrational number”.  (c)  3.  The statement  p ® (q ®  p) is equivalent to  (a)  p ®  (  p «  q)  (b)  p ®  (  p ®  q)  school. Statement­2 is a  not  a  correct  explanation  for  Statement­1  correct  explanation  for  Statement­1.  Statement­2  is  (d) x Î S  and x £  0 Þ x is not rational.  Statement­1 :  (p Ù  ~ q) Ù  (~ p Ù q) is  a  fallacy.  Statement­2  is  false.  5.  not  a  correct  explanation  for  Statement­1  (2008)  Which  of  the  following  statements  is  the  negation  of  the  statement P  ?  Answer  Key  1.  Statement­2  is  true  (2011)  (c)  Statement­1 is true.  Consider the  following  statements 7.  (c)  .111  Mathematical  Logic  CHAPTER  MATHEMATICAL LOGIC  18  1.  Statement­2  is  true.  (2012)  (c)  p ®  (  p Ú  q)  (d)  p ®  (  p Ù  q)  (2008)  3.   (a) : 1 st  solution : Let's prepare the truth table for the statements. .  (d) : The  statement can be written as P Ù  ~R Û  Q  p  q  ~p  ~q  p Ù ~q  ~p Ù q  (p Ù  ~q) Ù (~p Ù  q)  T  T  F  F  F  F  F  T  F  F  T  T  F  F  F  T  T  F  F  T  F  F  F  T  T  F  F  F  Thus the negation is  ~ (Q «  P Ù  ~  R)  Then  Statement­1  is  fallacy.  3.  2 nd  solution :  ~ (~  p Ú  q) Ù  ~ (~  q Ú  p) ~(p « ~q) is NOT a tautology because it’s statement value is not  º  ~ ((~ p Ú  q) Ú  (~  q Ú  p)) º  ~ ((p ®  q) Ú  (q ®  p)) º  ~ T  always  true. .  ((p ®  q) ®  (~  q ®  ~  p) Ù  ((~  q ®  ~  p) ®  (p ®  q))  = ((~  p Ú  q) ®  (q Ú  ~  p) Ù  ((q Ú  ~  p) ®  (~  p Ú  q)) º  T Ù  T º  T.  The  negation  would  be  :  There  is  no  rational  number  x Π S  such that  x  > 0  which  is  equivalent  to  all  rational  numbers  x Π S  satisfy  x £  0.  then I  will  open a  school’’  Negation of the given  statement is  ‘‘ I will become a  teacher  and  I  will not  open a  school’’  ( .  Thus Statement­1 is true because its negation is false.  (a) : The given statement  r º ~  p «  q  The Statement­1 is  r 1 º (p Ù  ~  q) Ú  (~  p Ù  q)  The Statement­2 is  r 2 º ~ (p « ~ q) = (p Ù q) Ú (~ q Ù ~ p)  we can establish that  r =  r 1  Thus Statement­1 is true but Statement­2 is false.  2. ~  (p ®  q) =  p Ù ~  q)  6. .  4.  (c) : The given statement is  ‘‘If I become a teacher. we conclude  that  ~(p «  ~q)  is  equivalent  to  (p «  q).  (c) :  Let’s simplify the  statement  p ® (q ® p) = ~  p Ú (q ® p) = ~  p Ú  (~  q Ú  p)  = – p Ú  p Ú  ~  q = p ® (p Ú  q)  7.  (c) : The given statement is  P : at least one  rational  x Π S  such that  x > 0.112  JEE MAIN CHAPTERWISE EXPLORER  1.  Then  Statement­2  is  true.  (b) : Let’s  prepare  the  truth  table  T  T  F  F  T  T  T  T  F  F  T  F  F  T  F  T  T  F  T  T  T  F  F  T  T  T  T  T  Then Statement­2  is  tautology.  p  T  T  F  q  T  F  T  ~q  F  T  F  p «  q  T  F  F  p «  ~q  F  T  T  F  F  T  T  F  ~(p «  ~q)  T  F  F  T  As the column for ~(p « ~q) and (p « q) is the same.  p  q  ~ p  ~ q  p ®  q  ~ q ®  p  (p ®  q) ®  (~  q ®  p)  5.
Copyright © 2024 DOKUMEN.SITE Inc.